You are on page 1of 427

SCHWARTZ’S

PRINCIPLES OF SURGERY
ABSITE and Board Review

Brunicardi_FM_p00i-xii.indd 1 08/07/22 11:25 AM


NOTICE
Medicine is an ever-changing science. As new research and clinical experience broaden
our knowledge, changes in treatment and drug therapy are required. The authors and the
publisher of this work have checked with sources believed to be reliable in their efforts
to provide information that is complete and generally in accord with the standards
accepted at the time of publication. However, in view of the possibility of human error
or changes in medical sciences, neither the authors nor the publisher nor any other
party who has been involved in the preparation or publication of this work warrants
that the information contained herein is in every respect accurate or complete, and
they disclaim all responsibility for any errors or omissions or for the results obtained
from use of the information contained in this work. Readers are encouraged to confirm
the information contained herein with other sources. For example and in particular,
readers are advised to check the product information sheet included in the package
of each drug they plan to administer to be certain that the information contained in
this work is accurate and that changes have not been made in the recommended dose
or in the contraindications for administration. This recommendation is of particular
importance in connection with new or infrequently used drugs.

Brunicardi_FM_p00i-xii.indd 2 08/07/22 11:25 AM


SCHWARTZ’S
PRINCIPLES OF SURGERY
ABSITE and Board Review
11th Edition
Edited by
F. Charles Brunicardi, MD, FACS
Professor of Surgery, Department of Surgery
Senior Vice President
Dean, College of Medicine
SUNY Downstate Health and Sciences University
Brooklyn, New York

Associate Editors
Dana K. Andersen, MD, FACS Lisa Dresner, MD, FACS
Senior Scientific Advisor Program Director and Vice Chair for Education
Division of Digestive Diseases and Nutrition Associate Professor of Surgery
National Institute of Diabetes and Digestive and Kidney Disease Department of Surgery
National Institutes of Health College of Medicine, SUNY Downstate Health and Sciences University
Bethesda, Maryland Brooklyn, New York

Timothy R. Billiar, MD, FACS John G. Hunter, MD, FACS


George Vance Foster Professor and Chair Professor of Surgery, School of Medicine
Department of Surgery Executive Vice President, OHSU
Distinguished Professor of Surgery Chief Executive Officer, OHSU Health System
Executive Vice President and Chief Scientific Officer for UPMC Oregon Health and Science University
Associate Senior Vice Chancellor for Clinical Academics Portland, Oregon
University of Pittsburgh Health Sciences
Associate Medical Director, UPMC International and Commercial Lillian S. Kao, MD, MS, FACS
Services Division Jack H. Mayfield, MD, Chair in Surgery
University of Pittsburgh School of Medicine Vice-Chair for Quality of Care
University of Pittsburgh Professor and Chief, Division of Acute Care Surgery
Pittsburgh, Pennsylvania Department of Surgery
McGovern Medical School at UTHealth Houston
Karen J. Brasel, MD, MPH, FACS Houston, Texas
Professor of Surgery
Division of Trauma, Critical Care and Acute Care Surgery, Jeffrey B. Matthews, MD, FACS
School of Medicine Dallas B. Phemister Professor
Assistant Dean for Graduate Medical Education Chairman, Department of Surgery
Office of the Dean, School of Medicine Pritzker School of Medicine
Residency Program Director, Surgery, School of Medicine University of Chicago
Oregon Health and Science University Chicago, Illinois
Portland, Oregon
Raphael E. Pollock, MD, PhD, FACS
Director, Ohio State University Comprehensive Cancer Center
Kathleen Wellenreiter Klotz Chair in Cancer Research
Arthur G. James Cancer Hospital and Richard J. Solove Research Institute
Professor, Division of Surgical Oncology, Department of Surgery
Ohio State University College of Medicine
Columbus, Ohio

New York Chicago San Francisco Athens London Madrid Mexico City
Milan New Delhi Singapore Sydney Toronto

Brunicardi_FM_p00i-xii.indd 3 08/07/22 11:25 AM


Copyright © 2023, 2016, 2011, 2007 by McGraw Hill LLC. All rights reserved. Except as permitted under the United States Copyright
Act of 1976, no part of this publication may be reproduced or distributed in any form or by any means, or stored in a database or retrieval
system, without the prior written permission of the publisher.

ISBN: 978-1-26-046976-9
MHID: 1-26-046976-X

The material in this eBook also appears in the print version of this title: ISBN: 978-1-26-046975-2,
MHID: 1-26-046975-1.

eBook conversion by codeMantra


Version 1.0

All trademarks are trademarks of their respective owners. Rather than put a trademark symbol after every occurrence of a trademarked
name, we use names in an editorial fashion only, and to the benefit of the trademark owner, with no intention of infringement of the
trademark. Where such designations appear in this book, they have been printed with initial caps.

McGraw Hill eBooks are available at special quantity discounts to use as premiums and sales promotions or for use in corporate training
programs. To contact a representative, please visit the Contact Us page at www.mhprofessional.com.

TERMS OF USE

This is a copyrighted work and McGraw-Hill Education and its licensors reserve all rights in and to the work. Use of this work is subject
to these terms. Except as permitted under the Copyright Act of 1976 and the right to store and retrieve one copy of the work, you may
not decompile, disassemble, reverse engineer, reproduce, modify, create derivative works based upon, transmit, distribute, disseminate,
sell, publish or sublicense the work or any part of it without McGraw-Hill Education’s prior consent. You may use the work for your
own noncommercial and personal use; any other use of the work is strictly prohibited. Your right to use the work may be terminated if
you fail to comply with these terms.

THE WORK IS PROVIDED “AS IS.” McGRAW-HILL EDUCATION AND ITS LICENSORS MAKE NO GUARANTEES OR WAR-
RANTIES AS TO THE ACCURACY, ADEQUACY OR COMPLETENESS OF OR RESULTS TO BE OBTAINED FROM USING
THE WORK, INCLUDING ANY INFORMATION THAT CAN BE ACCESSED THROUGH THE WORK VIA HYPERLINK OR
OTHERWISE, AND EXPRESSLY DISCLAIM ANY WARRANTY, EXPRESS OR IMPLIED, INCLUDING BUT NOT LIMITED
TO IMPLIED WARRANTIES OF MERCHANTABILITY OR FITNESS FOR A PARTICULAR PURPOSE. McGraw-Hill Education
and its licensors do not warrant or guarantee that the functions contained in the work will meet your requirements or that its opera-
tion will be uninterrupted or error free. Neither McGraw-Hill Education nor its licensors shall be liable to you or anyone else for any
inaccuracy, error or omission, regardless of cause, in the work or for any damages resulting therefrom. McGraw-Hill Education has no
responsibility for the content of any information accessed through the work. Under no circumstances shall McGraw-Hill Education and/
or its licensors be liable for any indirect, incidental, special, punitive, consequential or similar damages that result from the use of or
inability to use the work, even if any of them has been advised of the possibility of such damages. This limitation of liability shall apply
to any claim or cause whatsoever whether such claim or cause arises in contract, tort or otherwise.
CONTENTS

Contributors vii PART II


Preface ix Specific Consideration 127
Acknowledgments xi
16 The Skin and Subcutaneous Tissue 129

17 Breast 137

PART I 18 Head and Neck 143


Basic Considerations 1
19 Chest Wall, Lung, Mediastinum, and Pleura 145
1 Leadership 3
20 Congenital Heart Disease 157
2 Systemic Response to Injury
21 Acquired Heart Disease 171
and Metabolic Support 7
22 Thoracic Aneurysms and Aortic Dissection 181
3 Fluid and Electrolyte Management
of the Surgical Patient 13
23 Arterial Disease 189
4 Hemostasis, Surgical Bleeding,
24 Venous and Lymphatic Disease 197
and Transfusion 21
25 The Esophagus and Diaphragmatic Hernia 207
5 Shock 29
26 Stomach 223
6 Surgical Infection 37
27 The Surgical Management of Obesity 233
7 Trauma 49
28 Small Intestine 235
8 Burns 65
29 Colon, Rectum, and Anus 241
9 Wound Healing 73
30 The Appendix 259
10 Oncology 83
31 Liver 263
11 Transplantation 87
32 The Gallbladder and
12 Patient Safety 97
Extrahepatic Biliary System 279
13 Physiologic Monitoring
33 Pancreas 289
of the Surgical Patient 105
34 The Spleen 297
14 Minimally Invasive Surgery 115
35 Abdominal Wall, Omentum, Mesentery,
15 Molecular and Genomic Surgery 119
and Retroperitoneum 307

Brunicardi_FM_p00i-xii.indd 5 08/07/22 11:25 AM


vi
36 Soft Tissue Sarcomas 311 48 Ethics, Palliative Care,
and Care at the End of Life 369
CONTENTS

37 Inguinal Hernias 313


49 Global Surgery 373
38 Thyroid, Parathyroid, and Adrenal 323
50 Optimizing Perioperative Care:
39 Pediatric Surgery 329 Enhanced Recovery and Chinese Medicine 377

40 Urology 337 51 Understanding, Evaluating, and


Using Evidence for Surgical Practice 381
41 Gynecology 341
52 Ambulatory Surgery 385
42 Neurosurgery 345
53 Skills and Simulation 387
43 Orthopedic Surgery 351
54 Web-Based Education and
44 Surgery of the Hand and Wrist 355 Implications of Social Media 389

45 Plastic and Reconstructive Surgery 361


Index 391
46 Anesthesia for the Surgical Patient 363

47 Surgical Considerations in the Elderly 365

Brunicardi_FM_p00i-xii.indd 6 08/07/22 11:25 AM


CONTRIBUTORS

Dana K. Andersen, MD, FACS Daniel T. Dempsey, MD, FACS


Senior Scientific Advisor Professor of Surgery, Perelman School of Medicine
Division of Digestive Diseases and Nutrition University of Pennsylvania
National Institute of Diabetes and Digestive and Kidney Disease Philadelphia, Pennsylvania
National Institutes of Health
Bethesda, Maryland Ankit Dhiman, MD
Research Fellow, Department of Surgery
Shay Behrens, MD Pritzker School of Medicine
General Surgery Resident University of Chicago
Department of Surgery
Chicago, Illinois
Oregon Health and Science University
Portland, Oregon Lisa Dresner, MD, FACS
Andrew J. Benjamin, MD Program Director and Vice Chair for Education
Assistant Professor of Surgery Associate Professor of Surgery
Pritzker School of Medicine Department of Surgery
University of Chicago College of Medicine
Chicago, Illinois SUNY Downstate Health and Sciences University
Brooklyn, New York
Timothy R. Billiar, MD, FACS
George Vance Foster Professor and Chair Jodi-Ann Edwards, MD
Department of Surgery Resident
Distinguished Professor of Surgery Department of Surgery
Executive Vice President and Chief Scientific Officer for UPMC College of Medicine
Associate Senior Vice Chancellor for Clinical Academics SUNY Downstate Health and Sciences University
University of Pittsburgh Health Sciences Brooklyn, New York
Associate Medical Director
UPMC International and Commercial Services Division Erfan Faridmoayer, MD
University of Pittsburgh School of Medicine Resident
University of Pittsburgh Department of Surgery
Pittsburgh, Pennsylvania College of Medicine
Jillian Bonaroti, MD SUNY Downstate Health and Sciences University
Department of Surgery Brooklyn, New York
University of Pittsburgh Medical Center
Pittsburgh, Pennsylvania Sam Guyer, MD
General Surgery Resident
Karen J. Brasel, MD, MPH, FACS Department of Surgery
Professor of Surgery Oregon Health and Science University
Division of Trauma, Critical Care and Acute Care Surgery Portland, Oregon
School of Medicine
Assistant Dean for Graduate Medical Education Patrick Harbour, MD
Office of the Dean Chief Resident in Plastic Surgery
School of Medicine Department of Plastic Surgery
Residency Program Director, Surgery Georgetown University School of Medicine
School of Medicine Washington, DC
Oregon Health and Science University
Portland, Oregon Ahmed Hozain, MD
Chief Resident
F. Charles Brunicardi, MD, FACS Department of Surgery
Professor of Surgery, Department of Surgery College of Medicine
Senior Vice President
SUNY Downstate Health and Sciences University
Dean, College of Medicine
Brooklyn, New York
SUNY Downstate Health and Sciences University
Brooklyn, New York
vii

Brunicardi_FM_p00i-xii.indd 7 08/07/22 11:25 AM


viii
John G. Hunter, MD, FACS Katherine Reitz, MD, FACS
CONTRIBUTORS

Professor of Surgery Department of Surgery


School of Medicine University of Pittsburgh Medical Center
Executive Vice President, OHSU Pittsburgh, Pennsylvania
Chief Executive Officer, OHSU Health System
Oregon Health and Science University Robert E. Roses, MD, FACS
Portland, Oregon Associate Professor of Surgery
Perelman School of Medicine
Lillian S. Kao, MD, MS, FACS University of Pennsylvania
Jack H. Mayfield, MD, Chair in Surgery Philadelphia, Pennsylvania
Vice-Chair for Quality of Care
Professor and Chief, Division of Acute Care Surgery Ashley Sweet, MD
Department of Surgery General Surgery Resident
McGovern Medical School at UTHealth Houston Department of Surgery
Houston, Texas Oregon Health and Science University
Portland, Oregon
Marina Affi Koprowski, MD Alexander Schwartzman, MD, MBA, FACS
General Surgery Resident Vice Chair and Clinical Professor of Surgery
Department of Surgery Department of Surgery
Oregon Health and Science University College of Medicine
Portland, Oregon SUNY Downstate Health Sciences University
Brooklyn, New York
Jeffrey B. Matthews, MD, FACS
Dallas B. Phemister Professor David H. Song, MD, FACS
Chairman, Department of Surgery Professor and Chairman
Pritzker School of Medicine Department of Plastic Surgery
University of Chicago Georgetown University School of Medicine
Chicago, Illinois Washington, DC

Munier Nazzal, MD, MBA, FRCS, FACS, DFSVS Joseph Tobias, MD


Professor, Department of Surgery General Surgery Resident
Vice Chair, Research and Education Department of Surgery
Chief, Division of Vascular, Endovascular and Wound Surgery Oregon Health and Science University
University of Toledo College of Medicine Portland, Oregon
Toledo, Ohio
Samuel Torres Landa Fernandez, MD
General Surgery Resident
Junaid Nizamuddin, MD
Department of Surgery
Assistant Professor of Anesthesia and Critical Care
Oregon Health and Science University
Pritzker School of Medicine
Portland, Oregon
University of Chicago
Chicago, Illinois Kiran K. Turaga MD, MPH, FACS
Professor of Surgery
Michael O’Connor, MD Pritzker School of Medicine
Professor of Anesthesia and Critical Care University of Chicago
Pritzker School of Medicine Chicago, Illinois
University of Chicago
Chicago, Illinois David Warner, MD
General Surgery Resident
Raphael E. Pollock, MD, PhD, FACS Department of Surgery
Director, Ohio State University Oregon Health and Science University
Comprehensive Cancer Center Portland, Oregon
Kathleen Wellenreiter Klotz Chair in Cancer Research
Arthur G. James Cancer Hospital and Richard J. Solove Lye-Yeng Wong, MD
Research Institute General Surgery Resident
Professor, Division of Surgical Oncology, Department of Surgery Department of Surgery
Ohio State University College of Medicine Oregon Health and Science University
Columbus, Ohio Portland, Oregon

Ryan Randle, MD Sarah Wonn, MD


General Surgery Resident General Surgery Resident
Department of Surgery Department of Surgery
Oregon Health and Science University Oregon Health and Science University
Portland, Oregon Portland, Oregon

Brunicardi_FM_p00i-xii.indd 8 08/07/22 11:25 AM


PREFACE

This 11th edition of Schwartz’s Principles of Surgery: ABSITE and Contributors of the primary book have updated the questions
Board Review marks a new milestone of excellence in surgical for each chapter since the last edition in an effort to continue to
education for the betterment of craft, quality of care, and the edification provide a high level of review on the most up-to-date information
of surgical students and colleagues alike. With 870 questions and techniques currently taught and employed in the operating
spanning the 54 updated chapters of this edition, including five new theater. We have maintained the proven format of providing the
chapters, Optimizing Perioperative Care: Enhanced Recovery and answer-bearing portion of the text immediately following the
Chinese Medicine, Understanding, Evaluating and Using Evidence question and answer as an efficient method for reinforcement and
for Surgical Practice, Ambulatory Surgery, Skills and Simulation, recall. The user may read the question followed by the answer as a
and Web-Based Education and Implications of Social Media, this is form of review, or by covering the right-hand column of the page,
the comprehensive companion text for reviewing and assessing the the user can complete the questions in a more authentic test format
information compiled in the main textbook and for preparation for and uncover the answers for review/scoring.
the American Board of Surgery In-Training Examination (ABSITE)
and Board Review.

ix

Brunicardi_FM_p00i-xii.indd 9 08/07/22 11:25 AM


This page intentionally left blank

Brunicardi_FM_p00i-xii.indd 10 08/07/22 11:25 AM


ACKNOWLEDGMENTS

To Jason Malley, Peter Boyle, Catherine Saggese, and all at McGraw We wish to thank Katie Elsbury for her dedication to the
Hill, we are thankful for the continued belief in and support for organization and editing of this book.
this book. F. Charles Brunicardi, MD, FACS

xi

Brunicardi_FM_p00i-xii.indd 11 08/07/22 11:25 AM


This page intentionally left blank

Brunicardi_FM_p00i-xii.indd 12 08/07/22 11:25 AM


PART I
Basic Considerations

Brunicardi_Ch01_p001-006.indd 1 30/06/22 10:36 AM


Brunicardi_Ch01_p001-006.indd 2 30/06/22 10:36 AM
CHAPTER 1
Leadership

1. The fundamental principles of leadership are: Answer: A


A. Vision and willingness Vision and willingness are the two fundamental principles of
B. Command and control leadership. Command and control is a colloquial term for the
C. Time management and mentoring previously predominant style of leadership in surgery based
D. Coaching, pacesetting, and democratic on fear and intimidation. Time management and mentorship
are key leadership skills. Coaching, pacesetting, and demo-
cratic are leadership styles. (See Schwartz 11th ed., p. 4.)

2. A senior resident and attending are performing a laparo- Answer: C


scopic cholecystectomy. The resident initially performs Leadership is a complex concept. Surgeons should strive to
the majority of the case following instruction from the adopt leadership qualities that provide the best outcomes for
attending. However, the resident informs the attending their patients based on the following fundamental principles:
that he feels uncertain whether he has truly obtained the vision, willingness, time management, conflict resolution,
critical view of safety. With the attending’s assistance, recruitment, and culture (see Table 1-1). (See Schwartz
the case was successfully completed. After the case, the 11th ed., p. 5.)
resident then asks the attending for a debrief of the case.
Which fundamental principle of leadership is being
demonstrated by the resident? TABLE 1-1 The fundamental principles of leadership
A. Conflict resolution Description and Application in the Field
B. Vision Leadership Skill of Medicine
C. Effective communication Vision The act of establishing tangible goals of care
D. Willingness to lead for patients on both a daily basis as well as for
long-term purposes.
Effective Establishing an open, respectful, and
communication nonjudgmental forum for communication
among different members of the health care
team and with the patient.
Willingness to Taking on full responsibility for the care of
lead patients and remaining ethical, professional,
and committed despite the especially
challenging rigors of joining the field of
surgery.
Willingness to A commitment to lifelong learning of the latest
learn scientific, medical, and surgical updates to
deliver optimized patient care.
Conflict The art of resolving conflicts in a peaceful and
resolution ethical manner in team settings.

Brunicardi_Ch01_p001-006.indd 3 30/06/22 10:36 AM


4
3. Effective communication is a key component of leader- Answer: C
ship, given that miscommunication is a leading cause Communication errors are often caused by miscommunica-
CHAPTER 1

of medical errors. Which of the following statements is tion due to hierarchical differences, concerns with upward
FALSE regarding communication? influence, conflicting roles and role ambiguity, and interper-
A. To Err Is Human, a publication by the US Institute sonal conflict. (See Schwartz 11th ed., p. 8.)
of Medicine, identified medical errors as the eighth
leading cause of death in the United States, causing
100,000 deaths annually.
Leadership

B. Effective communication that ensures all team mem-


bers understand daily goals of care for an ICU patient
can significantly decrease their length of stay in the
ICU.
C. Communication errors are often simply due to negli-
gence and failure to transmit information.
D. Information transfer and communication errors
cause delays in patient care and can cause serious
adverse events.
E. Improved communication in the OR among cardiac
surgery patients is associated with decreased adverse
outcomes.

4. Which of the following statements is FALSE regarding Answer: D


modern conflict resolution techniques? Modern conflict resolution techniques are based upon objec-
A. Based upon objectivity and willingness to listen. tivity, willingness to listen, and pursuit of principle-based
B. Should seek a solution that benefits all involved and solutions. For example, an effective style of conflict resolution
that is based upon core values of the organization. is the utilization of the “abundance mentality” model, which
C. Traditional command and control based on fear and attempts to achieve a solution that benefits all involved and is
intimidation can lead to sanctions and lawsuits. based upon core values of the organization, as opposed to the
D. Conflict resolution is more successful when both utilization of the traditional fault-finding model, which iden-
sides can admit they share some fault. tifies sides as right or wrong. Application of the abundance
mentality in surgery elevates the conflict above the affected
parties and focuses on the higher unifying goal of improved
patient care. Morbidity and mortality (M&M) conferences
are managed in this style and have the purpose of practice
improvement and improving overall quality of care within
the system, as opposed to placing guilt or blame on the sur-
geon or surgical trainees for the complication being reviewed.
The traditional style of the command-and-control technique
based on fear and intimidation is no longer welcome in
any health care system and can lead to sanctions, lawsuits,
and removal of hospital privileges or position of leadership.
(See Schwartz 11th ed., p. 9.)

5. Daniel Goleman of the Harvard Business Review Answer: A


described six key leadership styles. Which of the follow- The coercive leader demands immediate compliance. This
ing statements is FALSE regarding leadership styles? style reflects the command-and-control style that has histori-
A. The coercive style of leadership is antiquated and is cally dominated surgery. Excessive coercive leadership erodes
no longer effective in surgery. team members’ sense of responsibility, motivation, sense of
B. Democratic leadership is useful for building team participation in a shared vision, and ultimately, performance.
consensus and minimizing conflict but may frustrate The phrase “Do what I tell you!” brings to mind the coercive
team members if there is no clear, unifying vision. leader. However, it is effective in times of crisis to deliver clear,
C. The pacesetter leads by example and sets high stan- concise instruction. This style should be used sparingly and
dards for his team but typically takes over the tasks is best suited for emergencies. (See Schwartz 11th ed., p. 12.)
of something falling behind instead of building
them up.
D. The authoritative leader is often the most effective
and focuses on directing the team toward a common
vision allowing team members room for innovation
and experimentation, and supporting their efforts.

Brunicardi_Ch01_p001-006.indd 4 30/06/22 10:36 AM


5
6. Which type of program couples grade/high-school stu- Answer: B
dents with physicians to provide experiential learning Studies indicate that the bottleneck in diversity occurs at the

CHAPTER 1
and development of mentoring, presentation skills, and level of the medical school application pool, which in turn is
networking? caused by educational deficiencies at the primary, secondary,
A. Community outreach and collegiate levels. As an attempted solution, the University
B. “Pipeline” of Michigan developed a “pipeline” program that pairs grade-
C. Physician teacher school and high-school students with physicians for experien-
D. Bottleneck tial learning and the development of mentoring, presentation

Leadership
skills, and networking. It is important for departments of sur-
gery to develop a diversity program for recruitment of resi-
dents and faculty. Multi-institutional blinded studies indicate
that the implementation of formal leadership and diversity
training improves diversity leadership and strategic human
resource management. (See Schwartz 11th ed., p. 12.)

7. Which of the following statements about how leaders in Answer: D


surgery effectively address disparities in social determi- The past quarter century has seen a steady increase in diver-
nants of health and recruitment of women and under- sity within the field of surgery. Women, as of 2015, repre-
represented minority residents and faculty are TRUE? sent 38% of surgical trainees and 10% of academic professors
A. Diversity training obstructs leadership and strategic currently, but have doubled their representation in the past
human resource management due to excess use of 20 years. Some fields, such as head and neck surgery and
resources. plastic surgery, have studied their own subspecialty groups
B. Diversity programs improve recruitment of women with similar findings. African Americans comprise both 6%
and underrepresented minority residents and faculty. of medical school graduates, 6% of surgical trainees, and
C. Diversity helps address disparities in social determi- 2% to 4% of professors of surgery nationwide. Hispanics
nants of health. represent 5% of graduating medical students, 9% of general
D. B and C. surgery trainees, and 4% to 5% of persons at all levels of
academic surgery. Physician diversity is crucial and may
help to address disparities in social determinants of health.
(See Schwartz 11th ed., p. 12.)

8. The development and demonstration of effective leader- Answer: D


ship skills are essential during surgical training. Which The past decade has seen a demonstrable increase in our
of the following is considered the gold standard for the knowledge of how to develop leadership skills, particularly
training and assessment of clinical and nontechnical through simulation, as well as leadership evaluation through
skills with high reliability and validity? OSCE and other tools. Multiple groups have assessed multi-
A. Command-and-control leadership disciplinary teams, typically composed of nurses, anesthesia
B. Simulation training groups, and surgeons for the leadership-associated nontech-
C. Coworker observation reporting system nical skills of communication, teamwork, and situational
D. Objective structured clinical examination (OSCE) awareness. Through increasingly validated instruments
and assessment tools, these nontechnical skills have been
found to be trainable. The OSCE has been established as the
gold standard for the training and assessment of a wide range
of clinical and nontechnical skills with high reliability and
validity. (See Schwartz 11th ed., p. 14.)

9. Mentoring should be formally included in resident train- Answer: A


ing programs. It provides wisdom, guidance, and insight In academic medicine, evidence-based studies have shown
essential for the successful development of a surgical benefits to the mentees that include enhanced research pro-
leader. Which of the following statements is FALSE about ductivity, higher likelihood of obtaining research grants, and
mentees and mentors? greater success in obtaining desired positions in practice or
A. The benefits of mentorship are unidirectional. at academic institutions. Mentoring provides benefits to the
B. Mentees have enhanced research productivity. mentors themselves, including refinement of their own per-
C. Mentees have increased probability of obtaining sonal leadership skills and a strong sense of satisfaction and
grants/funding. accomplishment. (See Schwartz 11th ed., p. 18.)
D. Mentees have increased probability of obtaining
desired academic/practice positions.

Brunicardi_Ch01_p001-006.indd 5 30/06/22 10:36 AM


This page intentionally left blank

Brunicardi_Ch01_p001-006.indd 6 30/06/22 10:36 AM


CHAPTER 2
Systemic Response to Injury
and Metabolic Support

1. C-reactive protein (CRP): Answer: C


A. Is secreted in a circadian rhythm with higher levels in Pentraxins are a group of soluble pattern recognition mole-
the morning. cules that act to activate complement, agglutination and neu-
B. Increases after eating a large meal. tralization, and opsonization. CRP belongs to the pentraxin
C. Is a soluble pattern recognition molecule of the pen- group of molecules. CRP is low in normal circumstances, but
traxin family. is synthesized in the liver in response to IL-6 with serum levels
D. Is synthesized in lung and kidney in response to IL-6. increasing 1000-fold. In this way, CRP is a marker of acute
inflammation. CRP levels do not vary with circadian rhythms
or with caloric intake. (See Schwartz 11th ed., Ch. 2, p. 34.)

2. Which of the following is TRUE regarding the inflam- Answer: C


matory response following traumatic injury? Traumatic injury rapidly induces multiple responses from
A. There is an acute proinflammatory response caused the immune system. There is (1) an acute proinflammatory
by stimulation of the adaptive immune system. response that is triggered by activation of the innate immune
B. There is an anti-inflammatory response that leads to system via DAMPs, (2) an anti-inflammatory response that
a return to homeostasis accompanied suppression of likely serves to modulate the pro-inflammatory response and
the innate immune system. return the host toward homeostasis, and (3) suppression of
C. An acute proinflammatory response secondary to cellular-mediated adaptive immunity. Recent studies indicate
activate of innate immunity, an anti-inflammatory that these three responses occur concurrently immediately
response, and suppression of adaptive immunity following injury. (See Schwartz 11th ed., Ch. 2, p. 28.)
occur currently following injury.
D. Systemic inflammation following trauma is related to
the immune response to microbes.

3. Which of the following is TRUE regarding damage- Answer: C


associated molecular pattern (DAMP) molecules? DAMPs are endogenous molecules that are produced dur-
A. DAMPs function early in the immune response to ing tissue damage or cellular stress. These molecules interact
trauma by activating adaptive immunity. with both immune and nonimmune cells and function to
B. Pathogen-associated molecular patterns (PAMPs) activate the adaptive immune response early following injury.
and DAMPs have different signaling characteristics DAMPs are similar to PAMPs in both types of receptors they
and do not bind the same types of receptors. bind and their role in signaling during the immune response.
C. DAMPs are ligands for a group of receptors broadly Both DAMPs and PAMPs signal via PRRs, which includes
term pattern recognition receptors (PRR), which receptors such as TLRs, RAGE, calcium-dependent recep-
includes toll-like receptors (TLRs). tors (CLRs), and nucleotide-binding domain, leucine-rich
D. DAMPs signal only to immune cells. repeat-containing proteins (NLRs). (See Schwartz 11th ed.,
Ch. 2, p. 28.)

Brunicardi_Ch02_p007-012.indd 7 30/06/22 10:36 AM


8
4. High-mobility group protein B1 (HMGB1): Answer: D
A. Is one of the best-characterized damage-associated The best-characterized DAMP in the context of the injury-
CHAPTER 2

molecular pattern (DAMP) molecules, detectable in associated inflammatory response is high-mobility group
the circulation within 30 minutes of trauma. protein B1 (HMGB1), which is rapidly released into the circu-
B. Is a protein secreted by immune-competent cells lation within 30 minutes following trauma. Subsequent studies
stimulated by pathogen-associated molecular patterns have proven, however, that HMGB1 is actively secreted from
(PAMPs) or inflammatory cytokines. immune-competent cells stimulated by PAMPs (eg, endotoxin)
C. Is also secreted by endothelial cells, platelets, and also or by inflammatory cytokines (eg, tumor necrosis factor and
Systemic Response to Injury and Metabolic Support

as a part of cell death. interleukin-1). Stressed nonimmune cells, such as endothelial


D. All of the above. cells, and platelet also actively secrete HMGB1. Finally, passive
release of HMGB1 can occur following cell death, whether it
is programmed or uncontrolled (necrosis). The diverse pro-
inflammatory biological responses that result from HMGB1
signaling include: (1) the release of cytokines and chemokines
from macrophage/monocytes and dendritic cells; (2) neu-
trophil activation and chemotaxis; (3) alterations in epithe-
lial barrier function, including increased permeability; and
(4) increased procoagulant activity on platelet surfaces; among
others. (See Schwartz 11th ed., Ch. 2, p. 28.)

5. The most abundant amino acid in the human body is: Answer: C
A. Carnitine Glutamine is the most abundant amino acid in the human
B. Arginine body, comprising nearly two-thirds of the free intracellular
C. Glutamine amino acid pool. (See Schwartz 11th ed., Ch. 2, p. 69.)
D. Methionine

6. What is the role of mitochondrial damage-associated Answer: B


molecular patterns (DAMPs) in the injury-mediated Cell-free mtDNA has been shown to be significantly higher
inflammatory response? in the circulation of trauma patients than in normal controls.
A. Mitochondrial DNA (mtDNA) induces production Mitochondrial proteins and/or DNA can act as DAMPs by
of HMGB1. triggering an inflammatory response to necrosis and cel-
B. mtDNA and peptides from damaged mitochondria lular stress. Specifically, the release of mtDNA and formyl
activate the macrophage inflammasome and the peptides from damaged or dysfunctional mitochondria has
stimulator of interferon gene pathway (STING). been implicated in activation of the macrophage inflamma-
C. mtDNA and peptides modulate the anti-inflamma- some, a cytosolic signaling complex that responds to cellu-
tory response that suppresses the adaptive immune lar stress. mtDNA also activates the STING. With stress or
system. tissue injury, mtDNA and peptides are released from damaged
D. mtDNA is directly toxic to the liver and lung in high mitochondria where they can contribute to a sterile inflam-
amounts. matory response. From an evolutionary perspective, given
that eukaryotic mitochondria derive from bacterial origin, it
would make sense that they retain bacterial features capable
of eliciting a strong response that is typically associated with a
pathogen trigger. (See Schwartz 11th ed., Ch. 2, p. 30.)

7. Which of the following statements is FALSE regard- Answer: C


ing the hypothalamic-pituitary-adrenal (HPA) axis and The HPA axis is one of the main mechanism by which the brain
injury-associated stress? responds to injury. CRH is released from the paraventricular
A. The HPA axis is initiated by the hypothalamus- nucleus in response to inflammatory cytokines. CRH acts on
producing corticotropin-releasing hormone (CRH) the anterior pituitary to stimulate the secretion of ACTH into
released from the paraventricular nucleus in response the systemic circulation. ACTH acts on the zona fasciculata
to inflammatory cytokines. of the adrenal glands to synthesize and secrete glucocor-
B. CRH acts on the anterior pituitary to stimulate adre- ticoids. Cortisol is the major glucocorticoid in humans and
nocorticotropin hormone (ACTH) secretion into is essential for survival during significant physiologic stress.
systemic circulation. The resulting increase in cortisol levels following trauma has
C. CRH simulates the zona fasciculata of the adrenal several important anti-inflammatory actions. Adrenal insuf-
gland to synthesize and secrete glucocorticoids. ficiency represents a clinical syndrome highlighted largely
D. Insufficient cortisol in response to critical illness can by inadequate amounts of circulating cortisol and aldoste-
lead to tachycardia, hypotension, weakness, hypogly- rone. Classically, adrenal insufficiency is described in patients
cemia, hyponatremia, and hyperkalemia. with atrophic adrenal glands caused by exogenous steroid

Brunicardi_Ch02_p007-012.indd 8 30/06/22 10:36 AM


9
administration who undergo a stressor, such as surgery. These
patients subsequently manifest signs and symptoms, such as

CHAPTER 2
tachycardia, hypotension, weakness, nausea, vomiting, and
fever. (See Schwartz 11th ed., Ch. 2, p. 35.)

8. Enteral nutritional formulas used to treat hepatic failure Answer: A


are characterized by: Hepatic-failure enteral nutrition formulas have protein content
A. Lower fluid volume. made up of ~50% branched-chain amino acids. The increase of

Systemic Response to Injury and Metabolic Support


B. Lower concentrations of potassium, phosphorous, branched-chain amino acids and reduction of aromatic amino
and magnesium. acid levels may reverse encephalopathy in patients with hepatic
C. Predigested nutrients and minimal fat content. failure; however, no clear benefit has been demonstrated
D. ~50% branched-chain amino acids (eg, leucine, in clinical trials. Lower fluid volume and concentrations of
isoleucine, valine). potassium, phosphorous, and magnesium are characteristic of
renal-failure formulas. Predigested nutrients and minimal fat
content are characteristic of elemental formulas, which are fre-
quently used in patients with malabsorption, gut impairment,
or pancreatitis. (See Schwartz 11th ed., Ch. 2, p. 71.)

9. Which of the following statements is TRUE regarding Answer: B


autophagy? Autophagy is a process whereby cells, under normal con-
A. It is an energy-dependent, controlled mechanism for ditions, dispose of damaged organelles or debris. During
removing senescent of dysfunctional cells, with no conditions of stress, such as hypoxia or low cellular energy,
resulting activation of the inflammatory response. autophagy is activated to provide additional nutrients.
B. It is a mechanism by which cells dispose of damaged Increased levels of autophagy occur in activated immune cells
organelles and debris under normal circumstances, and autophagy plays a role in the immune response. It is stim-
but can be used to provide additional nutrients dur- ulated by Th1 cytokines and toll-like receptor (TLR) signal-
ing stress. ing in macrophages, and it is inhibited by Th2 cytokines. In
C. It is NOT stimulated by cytokines, and does not play turn, autophagy also regulates cytokine signaling by seques-
a role in cytokine signaling. tering and degrading pro-IL-1B and inflammasome compo-
D. It has been shown to contribute to lymphopenia in nents. Answers A and D describe apoptosis. (See Schwartz
sepsis. 11th ed., Ch. 2, p. 40.)

10. Which of the following cytokines is produced primarily Answer: D


by CD4+ T cells and promotes T lymphocyte activity and IL-2 plays a critical role in the immune response. It is pro-
proliferation? duced primarily by CD4+ T cells, but to a lesser degree by
A. IL-10 CD8+ T cells, NK T, mast, and activated dendritic cells. IL-2
B. IL-8 promotes the cytolytic activity of CD8+ T and natural killer
C. TNF-α cells, and also promotes naïve CD4+ T cell differentiation
D. IL-2 into Th1 and Th2 phenotypes. In addition, IL-2 is essen-
tial for development and maintenance of T regulatory cells.
(See Schwartz 11th ed., Ch. 2, p. 45.)

11. Which of the following statements best describes the role Answer: B
of IL-10 in the immune response to injury or stress? IL-10 plays an important role in the immune response in
A. Proinflammatory mediator that is rapidly mobilized that it acts in an anti-inflammatory role in order to rees-
and is one of the first cytokines released following tablish immune homeostasis. It regulates the duration and
trauma, mediates coagulation, cell migration, and magnitude of the inflammatory response. IL-10 is secreted
macrophage phagocytosis. by both myeloid and lymphoid immune cells, and its syn-
B. Anti-inflammatory mediator that is secreted from a thesis is upregulated during stress and inflammation. IL-10
variety of immune cells and acts to inhibit secretion inhibits the secretion of proinflammatory cytokines IL-1 and
of proinflammatory cytokines, such as TNF and IL-1. TNF through downregulation of NF-kB signaling. Answer A
C. Proinflammatory mediator that is secreted from a describes TNF-α, Answer C describes IL-6, and Answer D
variety of immune cells, promotes B cell maturation, describes IL-8. (See Schwartz 11th ed., Ch. 2, p. 46.)
induces Th17 development and inhibits Treg differ-
entiation, promotes angiogenesis, and increases vas-
cular permeability.
D. Proinflammatory mediator that is secreted primarily
by macrophages and monocytes, and acts as a che-
moattractant for neutrophils, basophils, eosinophils,
and lymphocytes.

Brunicardi_Ch02_p007-012.indd 9 30/06/22 10:36 AM


10
12. Which of the following statements is TRUE regarding Answer: A
transcriptional events following trauma? Recent studies examining the transcriptomic response to blunt
CHAPTER 2

A. Over 80% of cellular functions and pathways in leu- trauma in circulating leukocytes from human patients dem-
kocytes demonstrate a shift in gene expression fol- onstrated a significant shift in the leukocyte transcriptomic
lowing blunt trauma. with >80% of cellular functions and pathways demonstrating
B. Changes in gene expression take 3 to 5 days to alterations. These pathways included those involved in innate
develop. and adaptive immunity, and compensatory anti-inflammatory
C. Changes in gene expression occur rapidly and are mechanisms. Transcriptomic alterations occurred rapidly
Systemic Response to Injury and Metabolic Support

transient, present for only 1 to 2 days. (4–12 hours) and persisted for days to weeks. Different
D. Patterns of gene expression varied widely when injury patterns (blunt, burn, endotoxemia) demonstrated
blunt trauma, burn injury, and endotoxemia were similar patterns of gene expression. (See Schwartz 11th ed.,
compared. Ch. 2, p. 52.)

13. Which of the following statements about monocytes and Answer: C


macrophages is FALSE? Monocytes and macrophages are mononuclear cells that are
A. Monocytes and macrophages are the primary effector characterized by their ability to phagocytize damage-associated
cells to respond to danger signals through phagocy- molecular patterns (DAMPs) and pathogen-associated molec-
tosis, release of inflammatory mediators, and recruit- ular patterns (PAMPs) and release inflammatory mediators
ment of additional immune cells. to initiate the immune response to injury and inflamma-
B. Macrophages undergo M1 or M2 activation, with M1 tion by recruiting additional immune cells to the site of
macrophages expressing high levels of proinflamma- injury. Monocytes are derived from bone marrow progeni-
tory cytokines. tors, while tissue macrophages are derived from embryonic
C. Monocytes and tissue macrophages are derived from precursors. In response to external cues (PAMPs, DAMPs,
a common bone marrow progenitor. activated lymphocytes), macrophages undergo M1 or M2
D. An imbalance of M1/M2 macrophage populations in activation. M1 macrophages are characterized by high levels
the lung contributes to acute lung injury following of expression of proinflammatory cytokines and are able
hemorrhagic shock. to recruit additional macrophages to the site of injury. M2
macrophages are thought to promote a return to immune
homeostasis through expression of arginase-1 and IL-10, and
are additionally involved in wound repair. Recent data has
shown that an imbalance between these two phenotypes in
the lung contributes to acute lung injury after hemorrhagic
shock due to a significant decrease in the M2 population
and a delayed increase in the M1 population. (See Schwartz
11th ed., Ch. 2, p. 54.)

14. Platelet-specific TLR4 activation results in binding and Answer: D


activation of what cell type, with subsequent extraceullar The role of the platelet in the immune response has been
trap formation? recently better defined. Platelets express TLRs and are able to
A. Macrophage detect pathogen-associated molecular patterns (PAMPs) and
B. CD8+ T cell damage-associated molecular patterns (DAMPs). Platelet-
C. Mast cell specific TLR activation leads to platelets-binding and
D. Neutrophil -activating neutrophils. Neutrophils then extrude their DNA
to form neutrophil extracellular traps (NETs). This facilitates
trapping of bacteria, but can also lead to endothelial cell dam-
age. (See Schwartz 11th ed., Ch. 2, p. 56.)

15. Which of the following statements correctly pairs the Answer: D


type of CD4+ Th cell with its description? CD4+ helper T cells are divided into three subsets, Th1, Th2,
A. Th1: generally anti-inflammatory, attenuates killing and Th17, each with distinct roles in the immune response
of intracellular pathogens and a unique set of effector cytokines. Th1 cells are gener-
B. Th2: generally proinflammatory, involved in cellular ally proinflammatory in nature and are involved in cellular
immunity, promotes recognition and killing of intra- immunity. These cells secrete IL-2, IL-3, IL-6, and TNF-α and
cellular pathogens promote recognition and killing of intracellular pathogens.
C. Th17: strongly anti-inflammatory, characterized by Th2 cells are generally anti-inflammatory and play a role in
IL-10 expression humoral immunity by regulating antibody production. These
D. Th1: generally proinflammatory, promote recogni- cells secrete IL-3, IL-4, IL-5, and IL-10. Th17 cells have both
tion and killing of intracellular pathogens pro- and anti-inflammatory implications and are thought to
have two distinct phenotypes. A pathogenic Th17 phenotype

Brunicardi_Ch02_p007-012.indd 10 30/06/22 10:36 AM


11
is characterized by increased IL-17 production, whereas a
regulatory phenotype demonstrates increased IL-10 expres-

CHAPTER 2
sion. (See Schwartz 11th ed., Ch. 2, p. 55.)

16. Which of the following statements characterizes metabo- Answer: D


lism after injury and infection? Metabolism in critically ill patients following injury or infec-
A. An initial hypometabolic state following by a dra- tion varies from that of unstressed fasting due to the effects
matic increase in energy expenditure at ~2 weeks. of the immunologic and neuroendocrine responses. In the

Systemic Response to Injury and Metabolic Support


B. Adipose stores within the body are the predominant initial phases, there is an overall hypometabolic state with a
energy source during critical illness. subsequent rapid increase in energy expenditure at ~2 weeks.
C. Increased energy expenditure mediated by sympa- This increase in expenditure is mediated by sympathetic ner-
thetic activation and catecholamine release. vous system activation and catecholamine release. The mag-
D. All of the above. nitude of energy expenditure over time is proportional to the
degree of insult, and thermal injuries and severe infections
have the highest energy demands. Lipid metabolism is criti-
cally important in these disease states, as it is the predomi-
nant energy source and also impacts the structural integrity
of cell membranes and the immune response. (See Schwartz
11th ed., Ch. 2, p. 62.)

17. What is an appropriate number of kcal and grams of pro- Answer: C


tein per day for a 70-kg patient during a state of severe Table 2-1 demonstrates caloric adjustments above basal
stress? energy expenditure (BEE) for various hypermetabolic condi-
A. 2100 kcal, 70 g tions. The appropriate calculation for a patient in severe stress
B. 4200 kcal, 175 g is to use a target of 30–35 kcal/kg/day with a 1.6x adjustment,
C. 3360 kcal, 140 g and 2 g/kg/day of protein. (See Schwartz 11th ed., Ch. 2,
D. 1925 kcal, 70 g Table 2-9, p. 68.)

TABLE 2-1 Caloric adjustments above basal energy expenditure (BEE) in hypermetabolic conditions
Grams of Protein/kg Nonprotein Calories:
Condition kcal/kg per day Adjustment above BEE per day Nitrogen
Normal/moderate malnutrition 25–30 1.1 1.0 150:1
Mild stress 25–30 1.2 1.2 150:1
Moderate stress 30 1.4 1.5 120:1
Severe stress 30–35 1.6 2.0 90–120:1
Burns 35–40 2.0 2.5 90–100:1

18. Which of the following is NOT a reason that enteral Answer: D


nutrition is preferred to parenteral nutrition when Enteral nutrition (EN) is preferred to parenteral nutrition
possible? (PN) when feasible because of the lower cost of EN, lower risk
A. Lower cost of enteral feeding of vascular access and infectious complications, and lower risk
B. Lower risk of vascular access and infectious of gastrointestinal tract disuse complications. These compli-
complications cations include diminished soluble IgA and cytokine produc-
C. Fewer complications associated with gut disuse tion, bacterial overgrowth, altered mucosal barrier function,
D. Improved immune response homeostasis and immune defenses. There is no evidence of improved
immune homeostasis with the use of EN. Several studies have
been performed, and a recent meta-analysis demonstrated a
significant reduction in infectiously complications and length
of ICU stay in critically ill patients receiving EN over PN. The
positive impact of EN over PN may be more pronounced in
malnourished patients. Further, there is evidence that initiat-
ing EN early (within 48 hours) at 80% of the estimated total
energy goals provides benefit. (See Schwartz 11th ed., Ch. 2,
p. 68.)

Brunicardi_Ch02_p007-012.indd 11 30/06/22 10:36 AM


This page intentionally left blank

Brunicardi_Ch02_p007-012.indd 12 30/06/22 10:36 AM


CHAPTER 3
Fluid and Electrolyte Management
of the Surgical Patient

1. Metabolic acidosis with a normal anion gap (AG) occurs Answer: C


with: Metabolic acidosis with a normal AG results from either acid
A. Diabetic acidosis. administration (HCl or NH4+) or a loss of bicarbonate from
B. Renal failure. gastrointestinal (GI) losses, such as diarrhea, fistulas (enteric,
C. Severe diarrhea. pancreatic, or biliary), ureterosigmoidostomy, or from renal
D. Starvation. loss. The bicarbonate loss is accompanied by a gain of chlo-
ride, thus the AG remains unchanged. (See Schwartz 11th ed.,
Ch. 3, p. 91.)

2. All of the following are possible causes of postoperative Answer: C


hyponatremia EXCEPT: Hyponatremia is caused by excess free water (dilution) or
A. Excess infusion of normal saline intraoperatively. decreased sodium (depletion). Thus, excessive intake of free
B. Administration of antipsychotic medication. water (oral or IV) can lead to hyponatremia. Also, medica-
C. Transient decrease in antidiuretic hormone (ADH) tions can cause water retention and subsequent hypona-
secretion. tremia, especially in older patients. Primary renal disease,
D. Excess oral water intake. diuretic use, and secretion of ADH are common causes of
sodium depletion. ADH can be released transiently post-
operatively, or less frequently, in syndrome of inappropriate
ADH secretion. Lastly, pseudohyponatremia can be seen on
laboratory testing when high serum glucose, lipid, or protein
levels compromise sodium measurements. (See Schwartz
11th ed., Ch. 3, p. 86.)

3. Which of the following is an early sign of hyperkalemia? Answer: A


A. Peaked T waves Symptoms of hyperkalemia are primarily gastrointestinal
B. Peaked P waves (GI), neuromuscular, and cardiovascular. GI symptoms
C. Peaked (shortened) QRS complex include nausea, vomiting, intestinal colic, and diarrhea; neu-
D. Peaked U waves romuscular symptoms range from weakness to ascending
paralysis to respiratory failure; while cardiovascular mani-
festations range from electrocardiogram (ECG) changes to
cardiac arrhythmias and arrest. ECG changes that may be
seen with hyperkalemia include:
Peaked T waves (early change)
Flattened P wave
Prolonged PR interval (first-degree block)
Widened QRS complex
Sine wave formation
Ventricular fibrillation
(See Schwartz 11th ed., Ch. 3, p. 89.)

13

Brunicardi_Ch03_p013-020.indd 13 30/06/22 10:36 AM


14
4. Hypocalcemia may cause which of the following? Answer: A
A. Congestive heart failure Mild hypocalcemia can present with muscle cramping or
CHAPTER 3

B. Atrial fibrillation digital/perioral paresthesias. Severe hypocalcemia leads to


C. Pancreatitis decreased cardiac contractility and heart failure. Electrocar-
D. Hypoparathyroidism diogram (ECG) changes of hypocalcemia include prolonged
QT interval, T-wave inversion, heart block, and ventricular
fibrillation. Hypoparathyroidism and severe pancreatitis are
potential causes of hypocalcemia. (See Schwartz 11th ed.,
Fluid and Electrolyte Management of the Surgical Patient

Ch. 3, p. 90.)

5. The next most appropriate test to order in a patient with Answer: A


a pH of 7.1, Pco2 of 40, sodium of 132, potassium of 4.2, Metabolic acidosis results from an increased intake of acids,
and chloride of 105 is: an increased generation of acids, or an increased loss of
A. Serum bicarbonate bicarbonate. In evaluating a patient with a low serum bicar-
B. Serum magnesium bonate level and metabolic acidosis, first measure the anion
C. Serum ethanol gap (AG), an index of unmeasured anions.
D. Serum salicylate
AG = [Na] – [Cl + HCO3]
Metabolic acidosis with an increased AG occurs from either
exogenous acid ingestion (ethylene glycol, salicylate, or meth-
anol) or endogenous acid production of β-hydroxybutyrate
and acetoacetate in ketoacidosis, lactate in lactic acidosis, or
organic acids in renal insufficiency. (See Schwartz 11th ed.,
Ch. 3, p. 92.)

6. Which of the following statement is FALSE regarding Answer: B


hypertonic saline? Hypertonic saline (7.5%) has been used as a treatment modal-
A. It is an arteriolar vasodilator and may increase ity in patients with closed head injuries. It has been shown
bleeding. to increase cerebral perfusion and decrease intracranial pres-
B. It should be avoided in closed head injury. sure, thus decreasing brain edema. However, there also have
C. It should not be used for initial resuscitation. been concerns of increased bleeding because hypertonic
D. It increases cerebral perfusion. saline is an arteriolar vasodilator. (See Schwartz 11th ed.,
Ch. 3, p. 94.)

7. Normal saline is: Answer: D


A. 135 mEq NaCl/L. Sodium chloride is mildly hypertonic, containing 154 mEq
B. 145 mEq NaCl/L. of sodium that is balanced by 154 mEq of chloride. The high
C. 148 mEq NaCl/L. chloride concentration imposes a significant chloride load
D. 154 mEq NaCl/L. upon the kidneys and may lead to a hyperchloremic meta-
bolic acidosis. It is an ideal solution, however, for correcting
volume deficits associated with hyponatremia, hypochlore-
mia, and metabolic alkalosis. (See Schwartz 11th ed., Ch. 3,
p. 93.)

8. Fluid resuscitation using albumin: Answer: C


A. Is associated with coagulopathy. Albumin is available as 5% (osmolality of 300 mOsm/L) or
B. Is available as 1% or 5% solutions. 25% (osmolality of 1500 mOsm/L). Due to increased intra-
C. Can lead to pulmonary edema. vascular oncotic pressure, fluid is drawn into the intravas-
D. Decreases factor XIII. cular space, leading to pulmonary edema when albumin is
used for resuscitation for hypovolemic shock. Hydroxyethyl
starch solutions are associated with postoperative bleeding
in cardiac and neurosurgery patients. (See Schwartz 11th ed.,
Ch. 3, p. 94.)

9. Water constitutes what percentage of total body weight? Answer: C


A. 30%–40% Water constitutes approximately 50% to 60% of total body
B. 40%–50% weight. The relationship between total body weight and total
C. 50%–60% body water (TBW) is relatively constant for an individual
D. 60%–70% and is primarily a reflection of body fat. Lean tissues, such as

Brunicardi_Ch03_p013-020.indd 14 30/06/22 10:36 AM


15
muscle and solid organs, have higher water content than fat
and bone. As a result, young, lean men have a higher propor-

CHAPTER 3
tion of body weight as water than elderly or obese individuals.
An average young adult male will have 60% of his total body
weight as TBW, while an average young adult female’s will be
50%. The lower percentage of TBW in women correlates with
a higher percentage of adipose tissue and lower percentage of
muscle mass in most. Estimates of TBW should be adjusted

Fluid and Electrolyte Management of the Surgical Patient


down approximately 10% to 20% in obese individuals and up
by 10% in malnourished individuals. The highest percentage
of TBW is found in newborns, with approximately 80% of
their total body weight composed of water. This decreases to
about 65% by 1 year and thereafter remains fairly constant.
(See Schwartz 11th ed., Ch. 3, p. 83.)

10. If a patient’s serum glucose increases by 180 mg/dL, what Answer: C


is the increase in serum osmolality, assuming all other Osmotic pressure is measured in units of osmoles (osm)
laboratory values remain constant? or milliosmoles (mOsm) that refer to the actual number of
A. Does not change osmotically active particles. For example, 1 millimole (mmol)
B. 8 of sodium chloride contributes to 2 mOsm (one from sodium
C. 10 and one from chloride). The principal determinants of osmo-
D. 12 lality are the concentrations of sodium, glucose, and urea
(blood urea nitrogen [BUN]):
Calculated serum osmolality = 2 sodium + glucose/18
+ BUN/2.8
(See Schwartz 11th ed., Ch. 3, p. 84.)

11. What is the actual potassium of a patient with pH of 7.8 Answer: D


and serum potassium of 2.2? The change in potassium associated with alkalosis can be
A. 2.2 calculated by the following formula:
B. 2.8
Potassium decreases by 0.3 mEq/L for every 0.1 increase
C. 3.2
in pH above normal
D. 3.4
(See Schwartz 11th ed., Ch. 3, p. 90.)

12. The free water deficit of a 70-kg man with serum sodium Answer: D
of 154 is: This is the formula used to estimate the amount of water
A. 0.1 L required to correct hypernatremia
B. 0.7 L serum sodium − 140
C. 1 L Water deficit L = × TBW
140
D. 3.5 L
Estimate TBW (total body water) as 50% of lean body mass in
men and 40% in women. (See Schwartz 11th ed., Ch. 3, p. 94.)

13. A patient with serum calcium of 6.8 and albumin of 1.2 Answer: D
has a corrected calcium of: When measuring total serum calcium levels, the albumin
A. 7.7. concentration must be taken into consideration.
B. 8.0. Adjust total serum calcium down by 0.8 mg/dL for every
C. 8.6. 1 g/dL decrease in albumin. (See Schwartz 11th ed., Ch. 3,
D. 9.0. p. 90.)

14. All of the following treatments for hyperkalemia reduce Answer: D


serum potassium EXCEPT: When electrocardiogram (ECG) changes are present, cal-
A. Bicarbonate. cium chloride or calcium gluconate (5–10 mL of 10% solu-
B. Kayexalate. tion) should be administered immediately to counteract the
C. Glucose infusion with insulin. myocardial effects of hyperkalemia. Calcium infusion should
D. Calcium. be used cautiously in patients receiving digitalis, because
digitalis toxicity may be precipitated. Glucose and bicar-
bonate shift potassium intracellularly. Kayexalate is a cation

Brunicardi_Ch03_p013-020.indd 15 30/06/22 10:36 AM


16
exchange resin that binds potassium, either given enterally or
as an enema. (See Schwartz 11th ed., Ch. 3, p. 95.)
CHAPTER 3

15. An alcoholic patient with serum albumin of 3.9, K of 3.1, Answer: B


Mg of 2.4, Ca of 7.8, and PO4 of 3.2 receives three boluses Magnesium depletion is a common problem in hospitalized
of IV potassium and has serum potassium of 3.3. You patients, particularly in the ICU. The kidney is primarily
should: responsible for magnesium homeostasis through regulation
A. Continue to bolus potassium until the serum level is by calcium/magnesium receptors on renal tubular cells that
Fluid and Electrolyte Management of the Surgical Patient

>3.6. sense serum magnesium levels. Hypomagnesemia results


B. Give MgSO4 intravenously. from a variety of etiologies ranging from poor intake (star-
C. Check the ionized calcium. vation, alcoholism, prolonged use of IV fluids, and total
D. Check the blood urea nitrogen (BUN) and creatinine. parenteral nutrition with inadequate supplementation of mag-
nesium), increased renal excretion (alcohol, most diuretics,
and amphotericin B), gastrointestinal (GI) losses (diarrhea),
malabsorption, acute pancreatitis, diabetic ketoacidosis, and
primary aldosteronism. Hypomagnesemia is important not
only for its direct effects on the nervous system, but also
because it can produce hypocalcemia and lead to persistent
hypokalemia. When hypokalemia or hypocalcemia coexist
with hypomagnesemia, magnesium should be aggressively
replaced to assist in restoring potassium or calcium homeo-
stasis. (See Schwartz 11th ed., Ch. 3, p. 91.)

16. Calculate the daily maintenance fluids needed for a Answer: B


60-kg woman. A 60-kg woman would receive a total of 2300 mL of fluid
A. 2060 daily: 1000 mL for the first 10 kg of body weight (10 kg ×
B. 2300 100 mL/kg/day), 500 mL for the next 10 kg (10 kg × 50 mL/
C. 2360 kg/day), and 800 mL for the last 40 kg (40 kg × 20 mL/kg/
D. 2400 day). (See Schwartz 11th ed., Ch. 3, p. 96.)

17. A patient who has spasms in the hand when a blood Answer: B
pressure cuff is blown up most likely has: Asymptomatic hypocalcemia may occur with hypoproteinemia
A. Hypercalcemia (normal ionized calcium), but symptoms can develop with
B. Hypocalcemia alkalosis (decreased ionized calcium). In general, symptoms
C. Hypermagnesemia do not occur until the ionized fraction falls below 2.5 mg/dL,
D. Hypomagnesemia and are neuromuscular and cardiac in origin, including pares-
thesias of the face and extremities, muscle cramps, carpopedal
spasm, stridor, tetany, and seizures. Patients will demonstrate
hyperreflexia and positive Chvostek sign (spasm resulting from
tapping over the facial nerve) and Trousseau sign (spasm result-
ing from pressure applied to the nerves and vessels of the upper
extremity, as when obtaining a blood pressure). Decreased
cardiac contractility and heart failure can also accompany
hypocalcemia. (See Schwartz 11th ed., Ch. 3, p. 90.)

18. The actual anion gap (AG) of a chronic alcoholic with Answer: D
Na 133, K 4, Cl–101, HCO3– 22, albumin of 2.5 mg/dL is: The normal AG is <12 mmol/L and is due primarily to the
A. 6. albumin effect, so that the estimated AG must be adjusted for
B. 10. albumin (hypoalbuminemia reduces the AG).
C. 14.
Corrected AG = actual AG + [2.5(4.5 – albumin)]
D. 15.
(See Schwartz 11th ed., Ch. 3, p. 92.)

19. The effective osmotic pressure between the plasma and Answer: D
interstitial fluid compartments is primarily controlled by: The dissolved protein in plasma does not pass through the
A. Bicarbonate. semipermeable cell membrane, and this fact is responsible
B. Chloride ion. for the effective or colloid osmotic pressure. (See Schwartz
C. Potassium ion. 11th ed., Ch. 3, p. 83.)
D. Protein.

Brunicardi_Ch03_p013-020.indd 16 30/06/22 10:36 AM


17
20. The metabolic derangement most commonly seen in Answer: A
patients with profuse vomiting is: Hypochloremic, hypokalemic metabolic alkalosis can occur

CHAPTER 3
A. Hypochloremic, hypokalemic metabolic alkalosis. from isolated loss of gastric contents in infants with pyloric
B. Hypochloremic, hypokalemic metabolic acidosis. stenosis or in adults with duodenal ulcer disease. Unlike vom-
C. Hypochloremic, hyperkalemic metabolic alkalosis. iting associated with an open pylorus, which involves a loss
D. Hypochloremic, hyperkalemic metabolic acidosis. of gastric as well as pancreatic, biliary, and intestinal secre-
tions, vomiting with an obstructed pylorus results only in the
loss of gastric fluid, which is high in chloride and hydrogen,

Fluid and Electrolyte Management of the Surgical Patient


and therefore results in a hypochloremic alkalosis. Initially
the urinary bicarbonate level is high in compensation for
the alkalosis. Hydrogen ion reabsorption also ensues, with
an accompanied potassium ion excretion. In response to
the associated volume deficit, aldosterone-mediated sodium
reabsorption increases potassium excretion. The resulting
hypokalemia leads to the excretion of hydrogen ions in the
face of alkalosis, a paradoxic aciduria. Treatment includes
replacement of the volume deficit with isotonic saline and
then potassium replacement once adequate urine output is
achieved. (See Schwartz 11th ed., Ch. 3, p. 92.)

21. Symptoms and signs of extracellular fluid volume deficit Answer: D


include all of the following EXCEPT: High pulse pressure occurs with extracellular fluid volume
A. Anorexia. excess, but the other symptoms and signs are characteristic of
B. Apathy. moderate extracellular volume deficit. (See Schwartz 11th ed.,
C. Decreased body temperature. Ch. 3, p. 86.)
D. High pulse pressure.

22. A low urinary [NH4+] with a hyperchloremic acidosis Answer: D


indicates which of the following causes? Metabolic acidosis with a normal anion gap (AG) results
A. Excessive vomiting either from exogenous acid administration (HCl or NH4+),
B. Enterocutaneous fistula from loss of bicarbonate due to gastrointestinal (GI) disor-
C. Chronic diarrhea ders such as diarrhea and fistulas or ureterosigmoidostomy,
D. Renal tubular acidosis or from renal losses. In these settings, the bicarbonate loss
is accompanied by a gain of chloride; thus, the AG remains
unchanged. To determine if the loss of bicarbonate has a
renal cause, the urinary [NH4+] can be measured. A low uri-
nary [NH4+] in the face of hyperchloremic acidosis would
indicate that the kidney is the site of loss, and evaluation for
renal tubular acidosis should be undertaken. Proximal renal
tubular acidosis results from decreased tubular reabsorption
of HCO3–, whereas distal renal tubular acidosis results from
decreased acid excretion. The carbonic anhydrase inhibitor
acetazolamide also causes bicarbonate loss from the kidneys.
(See Schwartz 11th ed., Ch. 3, p. 92.)

23. When lactic acid is produced in response to injury, the Answer: B


body minimizes pH change by: Lactic acid reacts with base bicarbonate to produce carbonic
A. Decreasing production of sodium bicarbonate in acid. The carbonic acid is broken down into water and car-
tissues. bon dioxide that is excreted by the lungs. Any diminution in
B. Excreting carbon dioxide through the lungs. pulmonary function jeopardizes this reaction. (See Schwartz
C. Excreting lactic acid through the kidneys. 11th ed., Ch. 3, p. 92.)
D. Metabolizing the lactic acid in the liver.

24. What is the best determinant of whether a patient has a Answer: A


metabolic acidosis versus alkalosis? While bicarbonate, Pco2, and patient history often can suggest
A. Arterial pH the most likely metabolic derangement, only the measurement
B. Serum bicarbonate of arterial pH confirms acidosis versus alkalosis. (See Schwartz
C. Pco2 11th ed., Ch. 3, p. 91.)
D. Serum CO2 level

Brunicardi_Ch03_p013-020.indd 17 30/06/22 10:36 AM


18
25. If a patient’s arterial Pco2 is found to be 25 mm Hg, Answer: A
the arterial pH will be approximately: A low Paco2 indicates excess elimination of carbon dioxide by
CHAPTER 3

A. 7.52. the lungs, and the body pH will increase. Within reasonable
B. 7.40. physiologic ranges a 15 mm Hg fall in Paco2 should produce
C. 7.32. a 0.12 change from the normal body pH of 7.4. (See Schwartz
D. 7.28. 11th ed., Ch. 3, p. 91.)

26. Which of the following are NOT characteristic findings Answer: A


Fluid and Electrolyte Management of the Surgical Patient

of acute renal failure? Hyperkalemia, severe acidosis, uremic encephalopathy, and


A. BUN >100 mg/dL uremic pericarditis are all indications of life-threatening
B. Hyperkalemia problems, and urgent correction is mandatory. Elevation of
C. Severe acidosis BUN is commonly seen as well, but is not itself an indication
D. Uremic pericarditis for dialysis. (See Schwartz 11th ed., Ch. 3, p. 99.)
E. Uremic encephalopathy

27. An elderly diabetic patient who has acute cholecystitis is Answer: D


found to have a serum sodium level of 122 mEq/L and A rise in the extracellular fluid concentration of a substance
a blood glucose of 600 mg/dL. After correcting the glu- that does not diffuse passively across cell membranes (eg,
cose concentration to 100 mg/dL with insulin, the serum glucose or urea) causes an increase in effective osmotic pres-
sodium concentration would: sure, a transfer of water from cells, and dilutional hyponatre-
A. Decrease significantly unless the patient also received mia. For each 100 mg/dL rise in blood glucose above normal,
3% saline. the serum sodium level falls approximately to 3 mEq/L.
B. Decrease transiently but return to approximately Alternatively, the serum sodium level would increase by
122 mEq/L without specific therapy. about 15 mEq/L if the blood glucose level falls from 600 to
C. Remain essentially unchanged. 100 mg/dL. (See Schwartz 11th ed., Ch. 3, p. 88.)
D. Increase to the normal range without specific therapy.

28. Excessive administration of normal saline for fluid resus- Answer: B


citation can lead to what metabolic derangement? Sodium chloride is mildly hypertonic, containing 154 mEq
A. Metabolic alkalosis of sodium that is balanced by 154 mEq of chloride. The high
B. Metabolic acidosis chloride concentration imposes a significant chloride load
C. Respiratory alkalosis on the kidneys and may lead to a hyperchloremic metabolic
D. Respiratory acidosis acidosis. Sodium chloride is an ideal solution, however, for
correcting volume deficits associated with hyponatremia,
hypochloremia, and metabolic alkalosis. (See Schwartz
11th ed., Ch. 3, p. 93.)

29. The first step in the management of acute hypercalcemia Answer: A


should be: Patients with acute hypercalcemia usually have either acute
A. Correction of deficit of extracellular fluid volume. hyperparathyroidism or metastatic breast carcinoma with
B. Hemodialysis. multiple bony metastases. These patients develop severe
C. Administration of furosemide. headaches, bone pain, thirst, emesis, and polyuria. Unless
D. Administration of mithramycin. treatment is instituted promptly, the symptoms may be
rapidly fatal. Immediate correction of the associated defi-
cit of extracellular fluid volume is the most important step
in treatment. When effective, this results in the lowering of
the serum calcium level by dilution. Once extracellular fluid
volume has been replaced, furosemide is effective treat-
ment. Hemodialysis may also be employed, but its effect is
less rapid. Mithramycin is very useful in controlling meta-
static bone disease, but its effect is slow, and it cannot be
depended upon when the patient has acute hypercalcemia.
(See Schwartz 11th ed., Ch. 3, p. 95.)

Brunicardi_Ch03_p013-020.indd 18 30/06/22 10:36 AM


19
30. A victim of a motor vehicle accident arrives in hemor- Answer: C
rhagic shock. His arterial blood gases are pH, 7.25; Po2, In patients suffering from hemorrhagic shock, the presence

CHAPTER 3
95 mm Hg; Pco2, 25 mm Hg; HCO3–, 15 mEq/L. The of a metabolic acidosis early in the postresuscitative period is
patient’s metabolic acidosis would be treated best with: indicative of tissue hypoxia due to persistent inadequate tis-
A. Ampule of sodium bicarbonate. sue perfusion. Attempts to correct this problem by admin-
B. Sodium bicarbonate infusion. istering an alkalizing agent will not solve the basic problem.
C. Lactated Ringer solution. However, proper volume replacement by means of a balanced
D. Hyperventilation. salt solution such as Lactated Ringer solution will restore per-

Fluid and Electrolyte Management of the Surgical Patient


fusion and correct the metabolic acidosis by ending anaerobic
metabolism. (See Schwartz 11th ed., Ch. 3, p. 92.)

31. Three days after surgery for gastric carcinoma, a 50-year- Answer: A
old alcoholic male exhibits delirium, muscle tremors, Magnesium deficiency should be suspected in any mal-
and hyperactive tendon reflexes. Magnesium deficiency nourished patient who exhibits disturbed neuromuscular or
is suspected. All of the following statements regarding cerebral activity in the postoperative period. Laboratory con-
this situation are TRUE EXCEPT: firmation often is not reliable, and the syndrome may exist in
A. A decision to administer magnesium should be based the presence of a normal serum magnesium level. Hypocalce-
on the serum magnesium level. mia often coexists, particularly in patients who have clinical
B. Adequate cellular replacement of magnesium will signs of tetany. Intravenous magnesium can be adminis-
require 1 to 3 weeks. tered safely to a well-hydrated patient for initial treatment
C. A concomitant calcium deficiency should be of a severe deficit, but concomitant electrocardiographic
suspected. monitoring is essential. The electrocardiographic changes
D. Calcium is a specific antagonist of the myocardial associated with acute hypermagnesemia resemble those of
effects of magnesium. hyperkalemia, and calcium chloride or gluconate should be
readily available to counteract any adverse myocardial effects
of excess magnesium ions. Partial or complete relief of symp-
toms may follow the initial infusion of magnesium, although
continued replacement for a period of 1 to 3 weeks is neces-
sary to replenish cellular stores. (See Schwartz 11th ed., Ch. 3,
pp. 19, 95.)

32. Refeeding syndrome can be associated with all of the fol- Answer: B
lowing EXCEPT: With refeeding, a shift in metabolism from fat to carbohy-
A. Respiratory failure. drate substrate stimulates insulin release, which results in
B. Hyperkalemia. the cellular uptake of electrolytes, particularly phosphate,
C. Confusion. magnesium, potassium, and calcium. However, severe hyper-
D. Cardiac arrhythmias. glycemia may result from blunted basal insulin secretion.
(See Schwartz 11th ed., Ch. 3, p. 98.)

Brunicardi_Ch03_p013-020.indd 19 30/06/22 10:36 AM


This page intentionally left blank

Brunicardi_Ch03_p013-020.indd 20 30/06/22 10:36 AM


CHAPTER 4
Hemostasis, Surgical Bleeding, and Transfusion

1. Which of the following is NOT one of the four major Answer: A


physiological events of hemostasis? Hemostasis is a complex process, which limits blood loss
A. Vasodilation from an injured vessel. The four physiologic events of the
B. Platelet aggregation hemostatic process include vasoconstriction, platelet aggre-
C. Clot Formation gation, clot formation, and fibrinolysis. These processes are
D. Fibrinolysis the result of two different cascades, the extrinsic pathway and
intrinsic pathway. Both cause the conversion of prothrombin
to thrombin to fibrin, which are essential for clot formation.
(Schwartz 11th ed., p. 103.)

2. Which of the following is required for platelet adherence Answer: D


to injured endothelium? Platelets do not normally adhere to each other or to the ves-
A. Thromboxane A2 sel wall but can form a plug that aids in cessation of bleeding
B. Glycoprotein (GP) IIb/IIIa when vascular disruption occurs. Injury to the intimal layer
C. Adenosine diphosphate (ADP) in the vascular wall exposes subendothelial collagen to which
D. Von Willebrand factor (vWF) platelets adhere. This process requires vWF, a protein in the
subendothelium that is lacking in patients with von Will-
ebrand’s disease. vWF binds to glycoprotein (GP) I/IX/V on
the platelet membrane. Following adhesion, platelets initiate
a release reaction that recruits other platelets from the circu-
lating blood to seal the disrupted vessel. Up to this point, this
process is known as primary hemostasis. Platelet aggregation
is reversible and is not associated with secretion. Addition-
ally, heparin does not interfere with this reaction, and thus,
hemostasis can occur in the heparinized patient. ADP and
serotonin are the principal mediators in platelet aggregation.
(Schwartz 11th ed., p. 103.)

3. Which of the following clotting factors is the first factor Answer: C


common to both intrinsic and extrinsic pathways? The intrinsic pathway begins with the activation of factor XII
A. Factor I (fibrinogen) that subsequently activates factors XI, IX, and VIII. In this
B. Factor IX (Christmas factor) pathway, each of the primary factors is “intrinsic” to the cir-
C. Factor X (Stewart-Prower factor) culating plasma, whereby no surface is required to initiate the
D. Factor XI (plasma thromboplasma antecedent) process. In the extrinsic pathway, tissue factor (TF) is released
or exposed on the surface of the endothelium, binding to
circulating factor VII, facilitating its activation to VIIa. Each
of these pathways continues on to a common sequence that
begins with the activation of factor X to Xa (in the presence of
VIIIa). Subsequently, Xa (with the help of factor Va) converts
factor II (prothrombin) to thrombin and then factor I (fibrin-
ogen) to fibrin. Clot formation occurs after fibrin monomers
are cross-linked to polymers with the assistance of factor XIII.
(Schwartz 11th ed., p. 105.)
21

Brunicardi_Ch04_p021-028.indd 21 30/06/22 10:37 AM


22
4. Which of the following factors are NOT involved in pre- Answer: C
venting clot propagation? Feedback inhibition on the coagulation cascade deactivates
CHAPTER 4

A. Thrombomodulin the enzyme complexes leading to thrombin formation.


B. Tissue plasminogen activator (tPA) release by the Thrombomodulin (TM) presented by the endothelium serves
endothelium as a “thrombin sink” by forming a complex with thrombin,
C. Nitric oxide release from the endothelium rendering it no longer available to cleave fibrinogen. This
D. APC complexes with protein S then activates protein C (APC) and reduces further thrombin
generation by inhibiting factors V and VIII. Second, tissue
Hemostasis, Surgical Bleeding, and Transfusion

plasminogen activator (tPA) is released from the endothelium


following injury, cleaving plasminogen to initiate fibrinoly-
sis. APC then consumes plasminogen activator inhibitor-1
(PAI-1), leading to increased tPA activity and fibrinolysis.
Building on the anticoagulant response to inhibit thrombin
formation, tissue factor pathway inhibitor (TFPI) is released,
blocking the TF-VIIa complex and reducing the production
of factors Xa and IXa. Antithrombin III (AT-III) then neutral-
izes all of the procoagulant serine proteases and also inhibits
the TF-VIIa complex. The most potent mechanism of throm-
bin inhibition involves the APC system. APC forms a com-
plex with its cofactor, protein S, on a phospholipid surface.
This complex then cleaves factors Va and VIIIa so that they
are no longer able to participate in the formation of TF-VIIa
or prothrombinase complexes. (Schwartz 11th ed., p. 106.)

5. Which of the following congenital factor deficiency often Answer: D


presents in the first few days of life? Congenital factor XIII (FXIII) deficiency, originally recog-
A. Factor VII nized by Duckert in 1960, is a rare autosomal recessive disease
B. Factor IX usually associated with a severe bleeding diathesis. The male-
C. Factor XI to-female ratio is 1:1. Although acquired FXIII deficiency has
D. Factor XII been described in association with hepatic failure, inflam-
matory bowel disease, and myeloid leukemia, the only sig-
nificant association with bleeding in children is the inherited
deficiency. Bleeding is typically delayed because clots form
normally but are susceptible to fibrinolysis. Umbilical stump
bleeding is characteristic, and there is a high risk of intracra-
nial bleeding. Spontaneous abortion is usual in women with
factor XIII deficiency unless they receive replacement therapy.
Replacement can be accomplished with FFP, cryoprecipitate,
or a factor XIII concentrate. Levels of 1% to 2% are usually
adequate for hemostasis. (Schwartz 11th ed., p. 107.)

6. Which of the following is NOT a cause of Answer: D


thrombocytopenia? Decrease in number of circulating platelets can be the
A. Immune thrombocytopenia purpura (ITP) result of (1) failure of production, (2) shortened survival,
B. Hemolytic uremia syndrome (HUS) and (3) sequestration. Failure of production occurs in bone
C. B12 deficiency marrow disorders, including leukemias, myelodysplastic
D. Clopidogrel administration syndromes, severe B12 deficiency, folate deficiency, chemo-
therapy, radiation treatment, alcohol intoxication, and viral
syndromes. Shortened platelet survival is seen in idiopathic
thrombocytopenia purpura, heparin-induced thrombo-
cytopenia (HIT), thrombotic thrombocytopenia purpura
(TTP), and in hemolytic uremia syndrome (HUS). Seques-
tration occurs with trapping of platelets in enlarged spleen
secondary to portal hypertension, sarcoid, lymphoma, or
Gaucher’s disease. Clopidogrel does not cause a decrease in
platelet number, but irreversibly inhibits platelet function.
(Schwartz 11th ed., p. 108.)

Brunicardi_Ch04_p021-028.indd 22 30/06/22 10:37 AM


23
7. Primary immune thrombocytopenia (ITP): Answer: B
A. Occurs more often in children with adults, but has a Primary immune thrombocytopenia is also known as idio-

CHAPTER 4
similar clinical course. pathic thrombocytopenic purpura (ITP). In children, it is
B. Includes HIT as a subtype of drug-induced ITP. usually acute at the onset, short-lived, and typically follows
C. is also known as thrombotic thrombocytopenic pur- a viral illness. In contrast, ITP in adults is gradual in onset,
pura (TTP). chronic in nature, and has no identifiable cause. Because the
D. is a disease of impaired platelet production of circulating platelets in ITP are young and functional, bleeding
unknown etiology. is less for a given platelet count than when there is failure of

Hemostasis, Surgical Bleeding, and Transfusion


platelet production. The pathophysiology of ITP is believed to
involve both impaired platelet production and T cell–medi-
ated platelet destruction. Heparin-induced thrombocytopenia
(HIT) is a form of drug-induced immune thrombocytopenia.
It is an immunologic event during which antibodies against
platelet factor 4 formed during exposure to heparin affect
platelet activation and endothelial function with resultant
thrombocytopenia and intravascular thrombosis. TTP is a dis-
order of platelet activation and production of platelet thrombi.
(Schwartz 11th ed., p. 108.)

8. Which of the following is NOT an acquired platelet Answer: C


hemostatic defect? Impaired platelet function often accompanies thrombocyto-
A. Massive blood transfusion following Trauma penia, but may also occur in the presence of a normal platelet
B. Acute renal failure count. The importance of this is obvious when one considers
C. Disseminated intravascular coagulation (DIC) that 80% of overall clot strength is related to platelet function.
D. Polycythemia vera The life span of platelets ranges from 7 to 10 days, placing them
at increased risk for impairment by medical disorders and pre-
scription and over-the-counter medications. Impairment of
ADP-stimulated aggregation occurs with massive transfusion of
blood products. Uremia may be associated with increased bleed-
ing time and impaired aggregation. Defective aggregation and
platelet dysfunction are also seen in patients with severe trauma,
thrombocythemia, polycythemia vera, and myelofibrosis.
DIC is an acquired syndrome characterized by systemic
activation of coagulation pathways that result in excessive
thrombin generation and diffuse formation of microthrombi.
(Schwartz 11th ed., p. 110.)

9. What drug irreversibly inhibits platelet function by irre- Answer: A


versible acetylation of platelet prostaglandin synthase? Drugs that interfere with platelet function include aspirin,
A. Aspirin clopidogrel, prasugrel, dipyridamole, and GP IIb/IIIa inhibi-
B. Clopidogrel tors. Aspirin, clopidogrel, and prasugrel all irreversibly inhibit
C. Dipyridamole platelet function. Clopidogrel and prasugrel do so through
D. Glycoprotein IIB/IIIA inhibitors selective irreversible inhibition of ADP-induced platelet
aggregation. Aspirin works through irreversible acetylation
of platelet prostaglandin synthase. (Schwartz 11th ed., p. 110.)

10. Which is TRUE about trauma-induced coagulopathy Answer: C


(TIC)? Traditional teaching regarding trauma-related coagulopathy
A. The acute coagulopathy of trauma is mechanistically attributed its development to acidosis, hypothermia, and dilu-
similar to disseminated intravascular coagulation tion of coagulation factors. Recent data, however, have shown
(DIC). that over one-third of severely injured patients have laboratory-
B. Coagulopathy can develop in trauma patients fol- based evidence of coagulopathy at the time of admission, a
lowing acidosis, hypothermia, and dilution of coag- phenotype called trauma- TIC. TIC is independent of tra-
ulation factors though coagulation is normal on ditional (iatrogenic) causes of posttraumatic coagulopathy,
admission. such as hemodilution, is precipitated by tissue injury and/or
C. TIC is caused by shock and tissue injury. hemorrhagic shock, and is associated with significantly higher
D. Acute coagulopathy of trauma is mainly a dilutional risk of mortality, especially in the first 24 hours after injury.
coagulopathy. Furthermore, TIC is a separate and distinct process from dis-
seminated intravascular coagulopathy with its own specific
components of hemostatic failure. (Schwartz 11th ed., p. 111.)

Brunicardi_Ch04_p021-028.indd 23 30/06/22 10:37 AM


24
11. Warfarin use is often associated with an increased mor- Answer: B
bidity and mortality in acutely injured and emergency Although warfarin use is often associated with a signifi-
CHAPTER 4

surgery patients, with rapid reversal, these complications cant increase in morbidity and mortality in acutely injured
can be reduced: Which is NOT TRUE about rapid rever- and emergency surgery patients, with rapid reversal, these
sal of warfarin effect? complications can be reduced. There are several reversal
A. Vitamin K should be given to sustain the effects options that include vitamin K administration, plasma,
of plasma and prothrombin complex concentrate cryoprecipitate, recombinant factor VIIa, and factor con-
(PCC). centrates. The 2012 CHEST guidelines for the Manage-
Hemostasis, Surgical Bleeding, and Transfusion

B. PCC is superior to plasma. ment of Anticoagulant Therapy, Antithrombotic Therapy,


C. PCC more rapidly corrects INR but is associated with and Prevention of Thrombosis recommends patients with
excess thromboembolic events. major life-threatening bleeding due to warfarin receive
D. Four-factor PCC’s have more reliable correction of reversal with vitamin K and a rapid reversal agent such as
INR compared to three-factor PCCs. plasma or PCC. Vitamin K is given to sustain the effects of
the plasma or PCC due to their short half-lives. In major
bleeds, vitamin K 10 mg given as a slow IV infusion is uti-
lized for more rapid onset compared to the oral form. Stud-
ies have shown that PCC is superior to plasma for speed
of reversal and has decreased risk of fluid overload, but it
is equivalent in adverse and thromboembolic events and
costlier. PCC is available in two forms: three-factor PCC
(factors II, IX, and X) and four-factor PCC (factors II, VII,
IX, and X). Four-factor PCCs have been shown to have a
more reliable correction of INR compared to three-factor
PCCs. (Schwartz 11th ed., p. 112.)

12. What is the best laboratory test for determine degree of Answer: D
anticoagulation with direct oral anticoagulants such as Direct oral anticoagulants (DOACs) include direct throm-
dabigatran and rivaroxaban? bin inhibitors and factor Xa inhibitors and have no readily
A. Prothrombin time/International normalized ratio available method of detection of the degree of anticoagula-
(PT/INR) tion. More concerning is the difficulty in the reversal of these
B. Partial thromboplastin time (PTT) new anticoagulants. Recently, idarucizumab, a humanized
C. Bleeding time monoclonal antibody fragment that binds dabigatran, has
D. None of the above been approved for use for reversal of the thrombin inhibitor,
dabigatran, and dabigatran-related coagulopathy. (Schwartz
11th ed., p. 113.)

13. A 45-year-old man is now postoperative day 6 after colon Answer: A


resection for perforated diverticulitis, and is noted to HIT is a drug-induced immune thrombocytopenia. Anti-
have new bruising at venipuncture sites. Platelet count bodies against platelet factor 4 (PL4) cause destruction of
is measured at 45,000 platelets per milliliter. You sus- platelets. Platelet count falls on 5 to 7 days after starting
pect this is due to Heparin-induced thrombocytopenia heparin therapy, but sooner, 1 to 2 days after reexposure.
(HIT). Which of the following is TRUE? HIT generally occurs after treatment with full-dose unfrac-
A. HIT is due to an antibody against platelet factor 4 tionated heparins, but can occur with prophylactic doses or
(PF4). with low molecular weight (fractionated) heparins. The clini-
B. This can only occur with full dose unfractionated cal diagnosis of HIT is confirmed by a positive anti-platelet
heparin. factor 4. In addition to thrombocytopenia, HIT is associated
C. HIT is avoided by using fractionated heparins only. with arterial and venous thrombosis. Heparins are discon-
D. Anticoagulation with oral warfarin should be initi- tinued, and a direct thrombin inhibitor, such as lepirudin,
ated immediately. argatroban, or danaparoid is prescribed. Warfarin therapy
should be started only after anticoagulation it can initially
induce a hypercoagulable state (Schwartz 11th ed., p. 109.)

14. Which findings are not consistent with thrombotic Answer: D


thrombocytopenic purpura (TTP)? In TTP inhibition of a metalloproteinase enzyme, ADAM
A. Splenomegaly S13 allows for unrestrained growth of microthrombi. Von
B. Fever Willebrand factor (VWF) is secreted as large molecules.
C. Schistocytes on peripheral blood smear Normally, ADAM S13 cleaves large vWF molecules which
D. Platelet activation limits thrombi growth and prevents microvascular throm-
bosis. In TTP, the microvascular thrombosis leads to tissue

Brunicardi_Ch04_p021-028.indd 24 30/06/22 10:37 AM


25
ischemia and organ damage. TTP is clinically characterized
by thrombocytopenia as well as microangiopathic hemolytic

CHAPTER 4
anemia, fever, neurological symptoms, and renal insuffi-
ciency. Schistocytes, fragmented red blood cells, are seen on
peripheral blood smear. Plasma exchange with replacement
of fresh frozen plasma (FFP) is the treatment of acute TTP.
Additionally, Rituxamab, a monoclonal antibody against the
CD20 protein of B lymphocytes is indicated in refractory or

Hemostasis, Surgical Bleeding, and Transfusion


relapsing TTP. (Schwartz 11th ed., p. 109.)

15. Which of the following is FALSE regarding coagulation Answer: C


during cardiopulmonary bypass (CPB)? Under normal conditions, homeostasis of the coagulation
A. Contact with circuit tubing and membranes activates system is maintained by complex interactions between the
inflammatory cascades and causes anormal platelet endothelium, platelets, and coagulation factors. In patients
and clotting factor function. undergoing CPB, contact with circuit tubing and mem-
B. Coagulopathy is compounded by sheer stress. branes results in abnormal platelet and clotting factor acti-
C. Following bypass, platelet morphology and ability to vation, as well as activation of inflammatory cascades, that
aggregate are irreversibly altered. ultimately results in excessive fibrinolysis and a combination
D. Coagulopathy is compounded by hypothermia and of both quantitative and qualitative platelet defects. Platelets
hemodilution. undergo reversible alterations in morphology and their ability
to aggregate, which causes sequestration in the filter, partially
degranulated platelets, and platelet fragments. This multifac-
torial coagulopathy is compounded by the effects of shear
stress in the system, induced hypothermia, hemodilution, and
anticoagulation. (Schwartz 11th ed., p. 113.)

16. Which of the following facts about transfusion and Answer: C


crossmatching is FALSE? Platelets do not require crossmatching. In emergency situ-
A. Universal donor type O-negative red blood cells and ations, universal donor type O-negative red blood cells and
type AB plasma may be transfused to all recipients. type AB plasma may be transfused to all recipients. Due to
B. Platelets also require crossmatching. a shortage of type AB plasma, low anti-B titer type A plasma
C. The administration of Rh-positive red blood cells is has become widely adopted for emergency (uncrossmatched)
acceptable if Rh-negative red blood cells blood is not transfusion. In the United States, 85% of individuals are
available. type A or type O, making type A plasma compatible with
D. Crossmatched whole blood may be ideal therapy for the vast majority of potential recipients. Uncrossmatched
resuscitation of trauma patients. plasma is routinely transfused as part of platelet transfusions,
with major transfusion reactions reported rarely, and type
AB plasma currently carries a higher risk of TRALI com-
pared to other plasma types. Many centers have transitioned
to low-titer type A plasma for emergency transfusions, with
no increase in adverse events. O negative and type-specific
red blood cells are equally safe for emergency transfusion.
In patients known to have clinically significant cold aggluti-
nins, blood should be administered through a blood warmer.
If these antibodies are present in high titer, hypothermia is
contraindicated. Whole blood as an ideal therapy for acute
traumatic hemorrhagic shock has increased in the last several
years with multiple reports of successful use in military and
civilian trauma patients. However, there is still limited access
in most civilian centers. (Schwartz 11th ed., p. 115.)

17. Following recent abdominal surgery, your patient is Answer: D


admitted to the ICU with septic shock. Below what level A 1988 National Institutes of Health Consensus Report
of hemoglobin would a blood transfusion be indicated? challenged the dictum that a hemoglobin value of less than
A. <12 g/dL 10 g/dL or a hematocrit level <30% indicates a need for
B. <10 g/dL preoperative red blood cell transfusion. This was verified
C. <8 g/dL in a prospective randomized controlled trial in critically ill
D. <7 g/dL patients that compared a restrictive transfusion threshold to
a more liberal strategy and demonstrated that maintaining

Brunicardi_Ch04_p021-028.indd 25 30/06/22 10:37 AM


26
hemoglobin levels between 7 and 9 g/dL had no adverse
effect on mortality. In fact, patients with APACHE II scores
CHAPTER 4

of ≤20 or patients age <55 years actually had a lower mortal-


ity. SCCM/EAST and AABB guidelines recommend taking
into account patient-specific characteristics and the overall
clinical context when considering RBC transfusions in non-
acutely hemorrhaging patients. Patients with symptomatic
anemia should be transfused 1 RBC unit at a time, and iso-
Hemostasis, Surgical Bleeding, and Transfusion

lated asymptomatic anemia in and of itself is rarely an indica-


tion for RBC transfusion. (Schwartz 11th ed., p. 117.)

18. Which of the following is not part of damage control Answer: B


resuscitation (DCR)? Prior to DCR, resuscitation guidelines advocated volume
A. Permissive hypotension replacement with crystalloid, followed by packed red blood cell
B. Resuscitating with large volumes of crystalloid to and only later plasma or platelets. This conventional massive
limit RBC transfusion transfusion practice was based on a several small uncontrolled
C. Immediate release and administration of a predefined retrospective studies that used blood products containing
ration of blood products to mimic whole blood increased amounts of plasma, which are no longer available.
D. Use of hemostatic adjuncts Because of the known early coagulopathy of trauma, the cur-
rent approach to managing the exsanguinating patient involves
early implementation of DCR. Although most of the attention
to hemorrhagic shock resuscitation has centered on higher
ratios of plasma and platelets, DCR is composed of four basic
components: permissive hypotension, minimizing crystalloid-
based resuscitation, the immediate release and administration
of predefined balanced blood products (red blood cells, plasma,
and platelets) in ratios similar to those of whole blood, and the
use of hemostatic adjuncts. The Pragmatic Randomized Opti-
mal Platelet and Plasma Ratios (PROPPR) trial randomized
680 bleeding trauma patients across 12 highest-level trauma
centers to resuscitation with 1:1:1 versus 1:1:2 plasma to plate-
lets to RBCs. Although there was no significant difference in
mortality at 24 hours (13% vs 17%) or 30 days (22% vs 26%),
the 1:1:1 group had significantly decreased mortality due to
hemorrhage at 24 hours (9% vs 15%) and more patients achiev-
ing hemostasis (86% vs 78%). (Schwartz 11th ed., p. 117.)

19. In patients with significant blood loss, which of the fol- Answer: C
lowing is incorrect? In the Pragmatic Randomized Optimal Platelet and Plasma
A. Resuscitation with whole blood Ratios (PROPPR) trial patients were resuscitated with packed
B. Resuscitation with packed red blood cells, platelet, red blood cells, platelets, and plasma in various ratios. In
and FFP in a 1:1:1 ratio this trial, the 1:1:1 group had significantly decreased mor-
C. Packed red blood cells with transfusion of FFP and tality due to hemorrhage at 24 hours (9% vs 15%) and more
platelets as needed (when the measured INR > 2 and patients achieving hemostasis (86% vs 78%). Whole blood
PLT# is <75,000) transfusion appears to have similar outcome, as whole blood
D. Resuscitation with crystalloid and artificial colloid contains plasma and platelets in a similar ratio. Waiting for
to avoid transfusion reactions until hemorrhage is INR and platelet count appears to delay achieving hemostasis.
controlled. (Schwartz 11th ed., p. 119.)

20. Less than 0.5% of transfusions result in a serious Answer: A


transfusion-related complication. Which of the follow- Transfusion-related complications are primarily related
ing is the leading cause of transfusion-related deaths? to blood-induced proinflammatory responses (Table 4-9).
A. Transfusion-related acute lung injury (TRALI) Transfusion-related 149a events are estimated to occur in
B. ABO hemolytic transfusion reactions approximately 10% of all transfusions, but <0.5% are serious
C. Bacterial contamination of platelets in nature. Transfusion-related deaths, although exceedingly
D. Iatrogenic hepatitis C: infection rare, do occur and are related primarily to transfusion-related
acute lung injury (TRALI), ABO hemolytic transfusion reac-
tions, and bacterial contamination of platelets. (Schwartz
11th ed., p. 121.)

Brunicardi_Ch04_p021-028.indd 26 30/06/22 10:37 AM


27
21. TRALI does not include: Answer: A
A. Transfusion-related circulatory overload (TACO). The syndrome of TRALI is defined as noncardiogenic pul-

CHAPTER 4
B. Noncardiogenic pulmonary edema. monary edema related to transfusion. It can occur with the
C. Fever and rigors. administration of any plasma-containing blood product.
D. Bilateral pulmonary infiltrates. Symptoms are similar to circulatory overload with dyspnea
and associated hypoxemia. However, TRALI is characterized
as noncardiogenic and is often accompanied by fever, rigors,
and bilateral pulmonary infiltrates on chest X-ray. It most

Hemostasis, Surgical Bleeding, and Transfusion


commonly occurs within 1 to 2 hours after the onset of trans-
fusion but virtually always before 6 hours. Toy et al reported
a decrease in the incidence of TRALI with the reduction of
transfusion of plasma from female donors, due to a combina-
tion of reduced transfusion of strong cognate HLA class II
antibodies and HNA antibodies in patients with risk factors
for acute lung injury. TRALI now occurs <1 in 10,000 units
transfused and is usually self-limited with supportive therapy.
Treatment of TRALI entails discontinuation of any transfu-
sion, notification of the transfusion service, and pulmonary
support, which may vary from supplemental oxygen to
mechanical ventilation.
Respiratory compromise may also be associated with
TACO, which is an avoidable complication. It can occur with
rapid infusion of blood, plasma expanders, and crystalloids,
particularly in older patients with underlying heart disease.
(Schwartz 11th ed., p. 121.)

22. Diseases that are not generally transmitted by blood Answer: A


transfusion include: Malaria, Chagas’ disease, brucellosis, and, very rarely, syphilis
A. Malaria, Chagas’ disease. are among the diseases that have been transmitted by transfu-
B. CMV, hepatitis C, and HIV. sion. Transmission of hepatitis C and HIV-1 has been dra-
C. Zika and West Nile Virus. matically minimized by the introduction of better antibody
D. All of the above. and nucleic acid screening for these pathogens. The residual
risk among allogeneic donations is now estimated to be less
than 1 per 1,000,000 donations. Recent concerns about the
rare transmission of these and other pathogens, such as West
Nile virus, are being addressed by current trials of “pathogen
inactivation systems” that reduce infectious levels of all
viruses and bacteria known to be transmittable by transfusion.
Recently, there is heightened concern of transmission of Zika
virus by blood product transfusion. Studies in endemic areas
have shown rates of Zika infection detected in donor blood as
high as 2.8%. Although no such cases have been reported in
the United States, transmission of Zika virus via platelet prod-
ucts have been reported in Brazil. (Schwartz 11th ed., p. 123.)

23. What are the uses of thromboelastography (TEG)? Answer: D


A. Predicting the need for lifesaving interventions after Recent trauma data has shown TEG to be useful in predict-
resuscitation for trauma ing early transfusion of red blood cells, plasma, platelets, and
B. Predicting 24-hour and 30-day mortality following cryoprecipitate. TEG can also predict the need for lifesaving
trauma interventions shortly after arrival, 24-hour and 30-day mor-
C. Predicting early transfusion of RBC, plasma, platelet, tality, and can be used to guide administration of Tranexamic
and cryoprecipitate Acid (TXA) to injured patients with hyperfibrinolysis. Lastly,
D. All of the above some centers have demonstrated that the graphic display
options allow for more rapid return of results and may be less
expensive than standard coagulation panels. Given the strong
association of viscoelastic tests with clinical outcomes, some
centers now use TEG rather than conventional coagulation
tests to evaluate injured patients in the emergency depart-
ment. (Schwartz 11th ed., p. 124.)

Brunicardi_Ch04_p021-028.indd 27 30/06/22 10:37 AM


28
24. Which of the following is TRUE in a thromboelastogra- Answer: D
phy (TEG) tracing? Several parameters are generated from the TEG tracing. The
CHAPTER 4

A. The r-value represents the clotting factor activity and r-value (reaction time) represents the time between the start
initial fibrin formation and is increased with factor of the assay and initial clot formation. This reflects clotting
deficiency or severe hemodilution. factor activity and initial fibrin formation and is increased
B. K time is prolonged with hypofibrinogenemia and with factor deficiency or severe hemodilution. The k-time
significant factor deficiency. (clot kinetics) is the time needed to reach specified clot
C. Decreased (alpha) or ∝ angle is treated with cryopre- strength and represents the interactions of clotting factors and
Hemostasis, Surgical Bleeding, and Transfusion

cipitate transfusion or fibrinogen administration. platelets. As such, the k-time is prolonged with hypofibrino-
D. All of the above. genemia and significant factor deficiency. Prolonged r-value
and k-time are commonly addressed with plasma transfu-
sions. The alpha or angle (∝) is the slope of the tracing and
reflects clot acceleration. The angle reflects the interactions
of clotting factors and platelets. The slope is decreased with
hypofibrinogenemia and platelet dysfunction. Decreased
angles are treated with cryoprecipitate transfusion or fibrin-
ogen administration. The maximal amplitude (mA) is the
greatest height of the tracing and represents clot strength. Its
height is reduced with dysfunction or deficiencies in plate-
lets or fibrinogen. Decreased mA is addressed with platelet
transfusion and, in cases where the angle is also decreased,
with cryoprecipitate (or fibrinogen) as well. The G-value is a
parametric measure derived from the mA value and reflects
overall clot strength or firmness. An increased G-value is
associated with hypercoagulability, whereas a decrease is seen
with hypocoagulable states. Finally, the LY30 is the amount
of lysis occurring in the clot, and the value is the percentage
of amplitude reduction at 30 minutes after mA is achieved.
The LY30 represents clot stability and presence of increased
fibrinolysis. (Schwartz 11th ed., p. 124.)

Brunicardi_Ch04_p021-028.indd 28 30/06/22 10:37 AM


CHAPTER 5
Shock

1. Shock caused by a large tension pneumothorax is catego- Answer: D


rized as: In 1934, Blalock proposed four categories of shock: hypovo-
A. Traumatic shock. lemic, vasogenic, neurogenic, and cardiogenic. Hypovolemic
B. Vasodilatory shock. shock, the most common type, results from loss of circu-
C. Cardiogenic shock. lating blood volume. This may result from loss of whole
D. Obstructive shock. blood (hemorrhagic shock), plasma, interstitial fluid (bowel
obstruction), or a combination. Vasogenic shock results from
decreased resistance within capacitance vessels, usually seen
in sepsis. Neurogenic shock is a form of vasogenic shock in
which spinal cord injury or spinal anesthesia causes vasodila-
tion due to acute loss of sympathetic vascular tone. Cardio-
genic shock results from failure of the heart as a pump, as in
arrhythmias, or acute myocardial infarction (MI).
In recent clinical practice, further classification has described
six types of shock: hypovolemic, septic (vasodilatory), neuro-
genic, cardiogenic, obstructive, and traumatic shock. Obstruc-
tive shock is a form of cardiogenic shock that results from
mechanical impediment to circulation leading to depressed
cardiac output rather than primary cardiac failure. This
includes etiologies such as pulmonary embolism or tension
pneumothorax. In traumatic shock, soft tissue and bony injury
lead to the activation of inflammatory cells and the release of
circulating factors, such as cytokines and intracellular mol-
ecules that modulate the immune response. Recent investiga-
tions have revealed that the inflammatory mediators released
in response to tissue injury (damage-associated molecular
patterns [DAMPs]) are recognized by many of the same cel-
lular receptors (pattern recognition receptors [PRRs]) and
activate similar signaling pathways as do bacterial products
elaborated in sepsis (pathogen-associated molecular patterns
[PAMPs]), such as lipopolysaccharide. These effects of tissue
injury are combined with the effects of hemorrhage, creating
a more complex and amplified deviation from homeostasis.
(See Schwartz 11th ed., p. 150.)

29

Brunicardi_Ch05_p029-036.indd 29 30/06/22 10:38 AM


30
2. What is TRUE about baroreceptors? Answer: B
A. Volume receptors can be activated in hemorrhage Baroreceptors also are an important afferent pathway in initi-
CHAPTER 5

with reduction in left atrial pressure. ation of adaptive responses to shock. Volume receptors, sensi-
B. Receptors in the aortic arch and carotid bodies inhibit tive to changes in both chamber pressure and wall stretch, are
the autonomic nervous system when stretched. present within the atria of the heart. They become activated
C. When baroreceptors are stretched, they induced with low-volume hemorrhage or mild reductions in right
increased autonomic nervous system output and pro- atrial pressure. Receptors in the aortic arch and carotid bod-
duce constriction of peripheral vessels. ies respond to alterations in pressure or stretch of the arterial
Shock

D. None of the above. wall, responding to larger reductions in intravascular volume


or pressure. These receptors normally inhibit induction of
the autonomic nervous system. When activated, these baro-
receptors diminish their output, thus disinhibiting the effect
of the autonomic nervous system. The autonomic nervous
system then increases its output, principally via sympathetic
activation at the vasomotor centers of the brain stem, pro-
ducing centrally mediated constriction of peripheral vessels.
(See Schwartz 11th ed., p. 134.)

3. Chemoreceptors in the aorta and carotid bodies do NOT Answer: C


sense which of the following? Chemoreceptors in the aorta and carotid bodies are sensitive to
A. Changes in O2 tension changes in O2 tension, H+ ion concentration, and carbon diox-
B. H+ ion concentration ide (CO2) levels. Stimulation of the chemoreceptors results in
C. HCO3– concentration vasodilation of the coronary arteries, slowing of the heart rate,
D. Carbon dioxide levels and vasoconstriction of the splanchnic and skeletal circulation.
In addition, a variety of protein and nonprotein mediators
are produced at the site of injury as part of the inflammatory
response, and they act as afferent impulses to induce a host
response. These mediators include histamine, cytokines, eico-
sanoids, and endothelins. (See Schwartz 11th ed., p. 134.)

4. Neurogenic shock is characterized by the presence of: Answer: C


A. Cool, moist skin. Neurogenic shock is caused by loss of arteriolar and venular
B. Increased cardiac output. tone in response to paralysis (such as occurs with high spi-
C. Decreased peripheral vascular resistance. nal anesthesia), acute gastric dilatation, or sudden pain or
D. Decreased blood volume. unpleasant sights; as such, it is characterized by a decrease in
peripheral vascular resistance. Affected patients usually pres-
ent with warm, dry skin, a pulse rate that is slower than nor-
mal, and hypotension. A normovolemic state usually exists,
and urine output is generally well maintained. Although
blood volume measurements indicate a normal intravascular
volume, because of the greatly increased reservoir capacity of
the arterioles and venules, there is a decrease in cardiac output
secondary to decreased venous return to the right side of the
heart. (See Schwartz 11th ed., p. 151.)

5. When a patient with hemorrhagic shock is resuscitated Answer: D


using an intravenous colloid solution rather than lac- Because of higher osmotic pressure, colloid solutions draw
tated Ringer’s solution, all of the following statements are extracellular fluid into the vascular space, increasing the
TRUE EXCEPT: extracellular fluid deficit. In addition, the ionized fraction
A. Circulating levels of immunoglobulins are decreased. of serum calcium is decreased, circulating levels of immu-
B. Colloid solutions may bind to the ionized fraction of noglobulin drop; and reaction to tetanus toxoid given to the
serum calcium. patient suffering from major trauma is decreased. Endoge-
C. Endogenous production of albumin is decreased. nous production of albumin also decreases. Colloid resusci-
D. Extracellular fluid volume deficit is restored. tation is no more effective than crystalloid resuscitation, and
it is more expensive and its use in bleeding trauma patients
is associated with an increased risk of death. (See Schwartz
10th ed., p. 144.)

Brunicardi_Ch05_p029-036.indd 30 30/06/22 10:38 AM


31
6. In hemorrhage, larger arterioles vasoconstrict in response Answer: D
to the sympathetic nervous system. Which categories of The microvascular circulation plays an integral role in regu-

CHAPTER 5
shock are associated with vasodilation of larger arterioles? lating cellular perfusion and is significantly influenced in
A. Septic shock response to shock. The microvascular bed is innervated by
B. Cardiogenic shock the sympathetic nervous system and has a profound effect on
C. Neurogenic shock the larger arterioles. Following hemorrhage, larger arterioles
D. A and C vasoconstrict; however, in the setting of sepsis or neurogenic
shock, these vessels vasodilate. Additionally, a host of other

Shock
vasoactive proteins, including vasopressin, angiotensin II,
and endothelin-1, also lead to vasoconstriction to limit organ
perfusion to organs such as skin, skeletal muscle, kidneys,
and the gastrointestinal (GI) tract to preserve perfusion of
the myocardium and central nervous system (CNS). (See
Schwartz 11th ed., p. 136.)

7. Which of the following is TRUE about anti-diuretic hor- Answer: A


mone (ADH) production in injured patients? The pituitary also releases vasopressin or ADH in response to
A. ADH acts as a potent mesenteric vasoconstrictor. hypovolemia, changes in circulating blood volume sensed by
B. ADH levels fall to normal within 2–3 days of the ini- baroreceptors and left atrial stretch receptors, and increased
tial insult. plasma osmolality detected by hypothalamic osmorecep-
C. ADH decreases hepatic gluconeogenesis. tors. Epinephrine, angiotensin II, pain, and hyperglycemia
D. ADH secretion is mediated by the renin-angiotensin increase production of ADH. ADH levels remain elevated
system. for about 1 week after the initial insult, depending on the
severity and persistence of the hemodynamic abnormali-
ties. ADH acts on the distal tubule and collecting duct of
the nephron to increase water permeability, decrease water
and sodium losses, and preserve intravascular volume. Also
known as arginine vasopressin, ADH acts as a potent mes-
enteric vasoconstrictor, shunting circulating blood away
from the splanchnic organs during hypovolemia. This may
contribute to intestinal ischemia and predispose to intestinal
mucosal barrier dysfunction in shock states. Vasopressin also
increases hepatic gluconeogenesis and increases hepatic gly-
colysis. (See Schwartz 11th ed., p. 135.)

8. Which of following occurs as a result of epinephrine and Answer: A


norepinephrine? Epinephrine and norepinephrine have a profound impact
A. Hepatic glycogenolysis on cellular metabolism. Hepatic glycogenolysis, gluconeo-
B. Hypoglycemia genesis, ketogenesis, skeletal muscle protein breakdown, and
C. Insulin sensitivity adipose tissue lipolysis are increased by catecholamines. Cor-
D. Lipogenesis tisol, glucagon, and anti-diuretic hormone (ADH) also con-
tribute to the catabolism during shock. Epinephrine induces
further release of glucagon, while inhibiting the pancreatic
β-cell release of insulin. The result is a catabolic state with glu-
cose mobilization, hyperglycemia, protein breakdown, nega-
tive nitrogen balance, lipolysis, and insulin resistance during
shock and injury. The relative underuse of glucose by periph-
eral tissues preserves it for the glucose-dependent organs, such
as the heart and brain. (See Schwartz 11th ed., p. 137.)

9. A patient has a blood pressure of 70/50 mm Hg and a Answer: C


serum lactate level of 30 mg/100 mL (normal: 6 to 16). The findings given in the question are characteristic of
Their cardiac output is 1.9 L/min, and his central venous hypovolemic shock, which can be defined as inadequate tis-
pressure is 2 cm H2O. The most likely diagnosis is: sue perfusion secondary to an extracellular fluid loss. The
A. Congestive heart failure. high lactate level is a result of anaerobic metabolism due to
B. Cardiac tamponade. decreased blood flow to tissues. The hemodynamic measure-
C. Hypovolemic shock. ments indicate both low blood flow and low venous return.
D. Septic shock. The total combination is most consistent with a diagnosis of
hypovolemic shock. Pulmonary embolus, congestive heart

Brunicardi_Ch05_p029-036.indd 31 30/06/22 10:38 AM


32
failure, and cardiac tamponade are all associated with a high
central venous pressure. Septic shock, particularly in its early
CHAPTER 5

phases, is usually hyperdynamic, and affected patients have


a greater-than-normal cardiac output. Complete hemo-
dynamic monitoring is vital in hypovolemic shock so that
prompt diagnosis and rational therapy can be expeditiously
carried out. (See Schwartz 11th ed., p. 141.)
Shock

10. Which of the following cytokines is elevated after shock Answer: C


and trauma and plays an important role in the develop- IL-6 is elevated in response to hemorrhagic shock and cor-
ment of diffuse alveolar damage and acute respiratory relates with mortality in shock states. IL-6 contributes to
distress syndrome (ARDS)? organ injury after hemorrhage shock, especially alveolar
A. IL-10 damage and ARDS. IL-10 is considered an anti-inflammatory
B. TNF-α cytokine that may have immunosuppressive properties. Its
C. IL-6 production is increased after shock and trauma, and it has
D. IL-1 been associated with depressed immune function clinically,
as well as an increased susceptibility to infection. TNF-α
may be induced by bacteria or endotoxin and leads to the
development of shock and hypoperfusion, most commonly
observed in septic shock. IL-1 has similar actions to TNF-α,
and is especially notable for its production of the febrile
response to injury through the activation of prostaglandins.
(See Schwartz 11th ed., p. 138.)

11. How does activation of the complement cascade contrib- Answer: A


ute to proinflammatory activation following injury and The complement cascade can be activated by injury, shock,
shock? and severe infection, and contributes to host defense and
A. Mediates organ dysfunction through increased vascu- proinflammatory activation. In trauma patients, the degree
lar permeability, smooth muscle cell contraction, and of complement activation is proportional to the magni-
adherence of neutrophils to vascular endothelium. tude of injury and may serve as a marker for severity of
B. Binds to specific chemokine receptors and transduce injury. Activated complement factors C3a, C4a, and C5a are
chemotactic signals to leukocyte. potent mediators of increased vascular permeability, smooth
C. Depresses cytokine production and improves some muscle cell contraction, histamine and arachidonic acid
aspects of immune function. by-product release, and adherence of neutrophils to vascu-
D. Releases a number of substances that may induce lar endothelium. Activated complement acts synergistically
cell or tissue injury, such as reactive O2 species, lipid- with endotoxin to induce the release of TNF-α and IL-1.
peroxidation products, proteolytic enzymes, and The development of acute respiratory distress syndrome
vasoactive mediators. (ARDS) and multiple organ dysfunction syndrome (MODS)
in trauma patients correlates with the intensity of comple-
ment activation. Complement and neutrophil activation may
correlate with mortality in multiply injured patients. (See
Schwartz 11th ed., p. 140.)

12. A 70-kg male patient presents emergency department Answer: C


(ED) following a gunshot wound to the left lower quad- The clinical signs of shock may be evidenced by agitation,
rant wound to the abdomen. His blood pressure is 88/62 cool clammy extremities, tachycardia, weak or absent periph-
and his heart rate is 122. He is alert, but confused and eral pulses, and hypotension. Such apparent clinical shock
intermittently combative with first responders. What results from at least 25% to 30% loss of the blood volume.
percent of blood volume has he lost? However, substantial volumes of blood may be lost before
A. 5% the classic clinical manifestations of shock are evident. Thus,
B. 15% when a patient is significantly tachycardic or hypotensive,
C. 35% this represents both significant blood loss and physiologic
D. 55% decompensation. The clinical and physiologic response to
hemorrhage has been classified according to the magnitude
of volume loss. Loss of up to 15% of the circulating vol-
ume (700–750 mL for a 70-kg patient) may produce little in
terms of obvious symptoms, while loss of up to 30% of the
circulating volume (1.5 L) may result in mild tachycardia,
tachypnea, and anxiety. Hypotension, marked tachycardia

Brunicardi_Ch05_p029-036.indd 32 30/06/22 10:38 AM


33
(ie, pulse >110–120 beats per minute [bpm]), and confusion
may not be evident until >30% of the blood volume has been

CHAPTER 5
lost; loss of 40% of circulating volume (2 L) is immediately
life-threatening, and generally requires operative control of
bleeding. (See Schwartz 11th ed., p. 141.)

13. Vasodilatory shock: Answer: C


A. Is characterized by failure of vascular smooth muscle In the peripheral circulation, profound vasoconstriction

Shock
to constrict due low levels of catecholamines. is the typical physiologic response to the decreased arterial
B. Leads to suppression of the renin-angiotensin system. pressure and tissue perfusion with hemorrhage, hypovolemia,
C. Can also be caused by carbon monoxide poisoning. or acute heart failure. This is not the characteristic response
D. Is similar to early cardiogenic shock. in vasodilatory shock. Vasodilatory shock is the result of dys-
function of the endothelium and vasculature secondary to
circulating inflammatory mediators and cells or as a response
to prolonged and severe hypoperfusion. Thus, in vasodila-
tory shock, hypotension results from failure of the vascular
smooth muscle to constrict appropriately. Vasodilatory shock
is characterized by peripheral vasodilation with resultant
hypotension and resistance to treatment with vasopressors.
Despite the hypotension, plasma catecholamine levels are
elevated, and the renin-angiotensin system is activated in
vasodilatory shock. The most frequently encountered form of
vasodilatory shock is septic shock. Other causes of vasodi-
latory shock include hypoxic lactic acidosis, carbon monox-
ide poisoning, decompensated and irreversible hemorrhagic
shock, terminal cardiogenic shock, and postcardiotomy
shock. Thus, vasodilatory shock seems to represent the final
common pathway for profound and prolonged shock of any
etiology. (See Schwartz 11th ed., p. 145.)

14. A patient in septic shock remains hypotensive despite Answer: B


adequate fluid resuscitation and initiation of norepi- After first-line therapy of the septic patient with the rapid
nephrine. What is often given to patients with hypoten- administration of antibiotics, IV fluids, and intubation if nec-
sion refractory to norepinephrine? essary, vasopressors may be necessary to treat patients with
A. Dopamine septic shock. Catecholamines are the vasopressors used most
B. Arginine vasopressin often, with norepinephrine being the first-line agent followed
C. Dobutamine by epinephrine. Occasionally, patients with septic shock will
D. Milrinone develop arterial resistance to catecholamines. Arginine vaso-
pressin, a potent vasoconstrictor, is often efficacious in this
setting and is often added to norepinephrine. (See Schwartz
11th ed., p. 147.)

15. Which of the following best describes the physiologic Answer: C


response and best treatment for critically ill and septic Hyperglycemia and insulin resistance are typical in critically
patients? ill and septic patients, including patients without underly-
A. Hypoglycemia and increased insulin sensitivity best ing diabetes mellitus. A recent study reported significant
treated with liberal maintenance of blood glucose positive impact of tight glucose management on outcome in
<215 mg/dL to reduce hypoglycemic-related neuro- critically ill patients. The two treatment groups in this ran-
logic derailments domized, prospective study were assigned to receive inten-
B. Hyperglycemia and insulin resistance best treated sive insulin therapy (maintenance of blood glucose between
with liberal maintenance of blood glucose <215 mg/dL 80 and 110 mg/dL) or conventional treatment (infusion of
to minimize mortality and septicemia insulin only if the blood glucose level exceeded 215 mg/dL,
C. Hyperglycemia and insulin resistance best treated with a goal between 180 and 200 mg/dL). The mean morn-
with intensive maintenance of blood glucose between ing glucose level was significantly higher in the conventional
80 and 110 mg/dL to reduce mortality and septicemia treatment as compared to the intensive insulin therapy group
D. Hypoglycemia and insulin resistance best treated (153 vs 103 mg/dL). Mortality in the intensive insulin treat-
with intensive maintenance of blood glucose between ment group (4.6%) was significantly lower than in the conven-
80 and 110 mg/dL to reduce hypoglycemic neuro- tional treatment group (8%), representing a 42% reduction in
logic derailments, mortality, and septicemia mortality. This reduction in mortality was most notable in the

Brunicardi_Ch05_p029-036.indd 33 30/06/22 10:38 AM


34
patients requiring longer than 5 days in the ICU. Furthermore,
intensive insulin therapy reduced episodes of septicemia by
CHAPTER 5

46%, reduced duration of antibiotic therapy, and decreased


the need for prolonged ventilatory support and renal replace-
ment therapy. (See Schwartz 11th ed., p. 147.)

16. Cardiogenic shock: Answer: C


A. Is most commonly an acute exacerbation of chronic Cardiogenic shock is defined clinically as circulatory pump
Shock

congestive heart failure. failure leading to diminished forward flow and subsequent
B. Is secondary to inadequate intravascular volume. tissue hypoxia, in the setting of adequate intravascular vol-
C. Confers a mortality rate of 50%–80%. ume. Hemodynamic criteria include sustained hypotension
D. Permissive hypotension reduces afterload resulting (ie, SBP < 90 mmHg for at least 30 minutes), reduced cardiac
in improved cardiac perfusion. index (<2.2 L/min per square meter), and elevated pulmo-
nary artery wedge pressure (>15 mm Hg). Mortality rates for
cardiogenic shock are 50% to 80%. Acute, extensive MI is the
most common cause of cardiogenic shock; a smaller infarc-
tion in a patient with existing left ventricular dysfunction
also may precipitate shock. Cardiogenic shock complicates
5% to 10% of acute myocardial infarction (MIs). Conversely,
cardiogenic shock is the most common cause of death in
patients hospitalized with acute MI. Although shock may
develop early after MI, it typically is not found on admission.
Seventy-five percent of patients who have cardiogenic shock
complicating acute MIs develop signs of cardiogenic shock
within 24 hours after onset of infarction (average 7 hours).
(See Schwartz 11th ed., p. 148.)

17. Treatment for cardiogenic shock includes: Answer: C


A. Established circulatory support prior to addressing After ensuring that an adequate airway is present and venti-
associated respiratory failure. lation is sufficient, attention should be focused on support of
B. Adequate oxygenation to ensure adequate myocar- the circulation. Intubation and mechanical ventilation often
dial O2 delivery and liberal fluid administration to are required, if only to decrease work of breathing and facili-
improve peripheral perfusion. tate sedation of the patient. Rapidly excluding hypovolemia
C. Dopamine administration to increase cardiac out- and establishing the presence of cardiac dysfunction are
put and vasodilate peripheral vascular beds to total essential. Treatment of cardiac dysfunction includes mainte-
peripheral resistance. nance of adequate oxygenation to ensure adequate myocar-
D. Intra-aortic balloon pump, initiated prior to cardio- dial O2 delivery and judicious fluid administration to avoid
tonic medications. fluid overload and development of cardiogenic pulmonary
edema. When profound cardiac dysfunction exists, inotropic
support may be indicated to improve cardiac contractility
and cardiac output. Dobutamine primarily stimulates cardiac
β1-receptors to increase cardiac output, but may also vaso-
dilate peripheral vascular beds, lower total peripheral resis-
tance, and lower systemic blood pressure through effects on
β2-receptors. Ensuring adequate preload and intravascular
volume is therefore essential prior to instituting therapy with
dobutamine. Dopamine stimulates receptors (vasoconstric-
tion), β1-receptors (cardiac stimulation), and β2-receptors
(vasodilation), with its effects on β-receptors predominating
at lower doses. Dopamine may be preferable to dobutamine
in treatment of cardiac dysfunction in hypotensive patients.
Patients whose cardiac dysfunction is refractory to cardio-
tonics may require mechanical circulatory support with an
intra-aortic balloon pump. Intra-aortic balloon pumping
increases cardiac output and improves coronary blood flow
by reduction of systolic afterload and augmentation of dia-
stolic perfusion pressure. (See Schwartz 11th ed., p. 149.)

Brunicardi_Ch05_p029-036.indd 34 30/06/22 10:38 AM


35
18. Following a high-speed motor vehicle accident resulting Answer: A
in multiple right-sided rib fractures and associated ster- The diagnosis of tension pneumothorax should be made on

CHAPTER 5
nal fracture, your patient becomes increasingly hypo- clinical examination. The classic findings include respira-
tensive in route to the emergency department with no tory distress (in an awake patient), hypotension, diminished
response to 2 L of fluid administration. What signs and breath sounds over one hemithorax, hyperresonance to per-
symptoms indicate obstructive shock from a right-sided cussion, jugular venous distention, and shift of mediastinal
tension pneumothorax? structures to the unaffected side with tracheal deviation.
A. Hypotension, tachycardia, jugular venous distension, Cardiac tamponade also may be associated with dyspnea,

Shock
leftward tracheal deviation orthopnea, cough, peripheral edema, chest pain, tachycardia,
B. Hypotension, tachycardia, jugular venous distension, muffled heart tones, jugular venous distention, and elevated
muffled heart sounds central venous pressure. Beck’s triad consists of hypotension,
C. Hypotension, tachycardia, rightward tracheal devia- muffled heart tones, and neck vein distention. (See Schwartz
tion, absent right sided breath sounds 11th ed., p. 150.)
D. Hypotension, tachycardia, pulsus paradoxus

19. A 17-year-old woman dives off of a cliff into a shallow Answer: C


river. Upon arrival to the emergency department, she is Neurogenic shock refers to diminished tissue perfusion as a
alert and upset, hypotensive, bradycardic, and her cool result of loss of vasomotor tone to peripheral arterial beds.
extremities do not withdraw to stimuli. What is the most Loss of vasoconstrictor impulses results in increased vascular
likely cause of his hypotension? capacitance, decreased venous return, and decreased cardiac
A. Inadequate circulatory blood volume output. Neurogenic shock is usually secondary to spinal cord
B. Lack of venous return of blood to the heart injuries from vertebral body fractures of the cervical or high
C. Loss of vasoconstrictor impulses results in increased thoracic region that disrupt sympathetic regulation of periph-
vascular capacitance, decreased venous return, and eral vascular tone. (See Schwartz 11th ed., p. 151.)
decreased cardiac output
D. Systemic release of endotoxin secondary to exposure
to and inoculation of pathogens

20. What defines adequate resuscitation in the setting of Answer: B


shock? Recognition by care providers of the patient who is in the
A. Resolution of hypotension and tachycardia with compensated phase of shock is equally important, but more
decreased mixed venous difficult based on clinical criteria. Compensated shock exists
B. O2 debt is repaid, tissue acidosis is corrected, and when inadequate tissue perfusion persists despite normaliza-
aerobic metabolism restored tion of blood pressure and heart rate. Even with normaliza-
C. Normalization of blood pressure in the emergency tion of blood pressure, heart rate, and urine output, 80% to
department with blood products in a patient with 85% of trauma patients have inadequate tissue perfusion, as
a positive focused assessment with sonography for evidenced by increased lactate or decreased mixed venous O2
trauma examination saturation. Persistent, occult hypoperfusion is frequent in the
D. Resolution of hypotension and tachycardia without ICU, with a resultant significant increase in infection rate and
pharmacologic intervention, but rising lactate mortality in major trauma patients. Patients failing to reverse
their lactic acidosis within 12 hours of admission (acidosis
that was persistent despite normal heart rate, blood pressure,
and urine output) developed an infection three times as often
as those who normalized their lactate levels within 12 hours
of admission. In addition, mortality was fourfold higher in
patients who developed infections. Both injury severity score
and occult hypotension (lactic acidosis) longer than 12 hours
were independent predictors of infection. Thus, recognition
of subclinical hypoperfusion requires information beyond
vital signs and urinary output.
Endpoints in resuscitation can be divided into systemic
or global parameters, tissue-specific parameters, and cellular
parameters. Global endpoints include vital signs, cardiac output,
pulmonary artery wedge pressure, O2 delivery and consump-
tion, lactate, and base deficit. (See Schwartz 11th ed., p. 152.)

Brunicardi_Ch05_p029-036.indd 35 30/06/22 10:38 AM


36
21. Base deficit: Answer: A
A. Quantifies the magnitude of perfusion deficit. Base deficit is the amount of base in millimoles that is
CHAPTER 5

B. Is representative of perfusion status, but not the risk required to titrate 1 L of whole blood to a pH of 7.40 with
of death. the sample fully saturated with O2 at 37°C (98.6°F) and a par-
C. Which is persistently elevated is evidence of hospital- tial pressure of CO2 of 40 mm Hg. It usually is measured by
acquired infection following a traumatic event. arterial blood gas analysis in clinical practice as it is readily
D. Does not provide additional information in clinical and quickly available. The mortality of trauma patients can
resuscitation when serum lactates can be assessed. be stratified according to the magnitude of base deficit mea-
Shock

sured in the first 24 hours after admission. In a retrospec-


tive study of over 3000 trauma admissions, patients with a
base deficit worse than 15 mmol/L had a mortality of 70%.
Base deficit can be stratified into mild (3–5 mmol/L), moder-
ate (6–14 mmol/L), and severe (15 mmol/L) categories, with
a trend toward higher mortality with worsening base deficit
in patients with trauma. Both the magnitude of the perfusion
deficit as indicated by the base deficit and the time required
to correct it are major factors determining outcome in shock.
Indeed, when elevated base deficit persists (or lactic
acidosis) in the trauma patient, ongoing bleeding is often
the etiology. Trauma patients admitted with a base deficit
>15 mmol/L required twice the volume of fluid infusion and
six times more blood transfusion in the first 24 hours com-
pared to patients with mild acidosis. Transfusion require-
ments increased as base deficit worsened and ICU and
hospital lengths of stay increased. Mortality increased as base
deficit worsened; the frequency of organ failure increased
with greater base deficit. The probability of trauma patients
developing acute respiratory distress syndrome (ARDS)
has been reported to correlate with severity of admission
base deficit and lowest base deficit within the first 24 hours
post injury. Persistently high base deficit is associated with
abnormal O2 utilization and higher mortality. Monitoring
base deficit in the resuscitation of trauma patients assists in
assessment of O2 transport and efficacy of resuscitation. (See
Schwartz 11th ed., p. 152.)

Brunicardi_Ch05_p029-036.indd 36 30/06/22 10:38 AM


CHAPTER 6
Surgical Infection

1. Macrophage response to microbes includes: Answer: D


A. Genome-encoded pattern recognition receptors to The response in macrophages is initiated by genome-
invading microbes (pathogen-associated molecular encoded pattern recognition receptors that respond to
patterns [PAMPs] and danger-associated molecular invading microbes. With exposure to a foreign organism,
patterns [DAMPs]). these receptors recognize microbial PAMPs and endoge-
B. Upregulation of cytokine synthesis. nous DAMPs. Toll-like receptors (TLRs) are a well-defined
C. Secretion of cytokines. example of a PAMP that plays an important role in pathogen
D. All of the Above. signaling. Resident macrophages secrete a wide array of sub-
stances in response to the aforementioned processes, some
of which appear to regulate the cellular components of the
host defense response. This results in recruitment and prolif-
eration of inflammatory cells. Macrophage cytokine synthe-
sis is upregulated. Secretion of tumor necrosis factor-alpha
(TNF-α); interleukins (IL)-1β, 6, and 8; and gamma inter-
feron (IFN-γ) occurs within the tissue milieu and depends
on the magnitude of the host defense response, the systemic
circulation. Concurrently, a counterregulatory response is
initiated consisting of binding protein (TNF-BP), cytokine
receptor antagonists (eg, IL-1ra), and anti-inflammatory
cytokines (IL-4 and IL-10). (Schwartz 11th ed., p. 160.)

2. Which of the following is NOT a component of systemic Answer: C


inflammatory response syndrome (SIRS)? Infection is defined by the presence of microorganisms in host
A. Temperature tissue or the bloodstream. The classic findings of rubor, calor,
B. WBC count and dolor in areas such as the skin or subcutaneous tissue are
C. Blood pressure common at the site of infection. Most infections in normal
D. Heart rate individuals with intact host defenses are associated with these
local manifestations, plus systemic manifestations such as
elevated temperature, elevated white blood cell (WBC) count,
tachycardia, or tachypnea. The systemic manifestations noted
previously comprise what has been termed the systemic
inflammatory response syndrome (SIRS). SIRS reflects a proin-
flammatory state in response to a variety of disease processes,
including infection, pancreatitis, polytrauma, malignancy,
and burns. There are a variety of systemic manifestations of
infection, with the classic factors of fever, tachycardia, and
tachypnea broadened to include a variety of other variables
(Schwartz 11th ed., p. 160.)

37

Brunicardi_Ch06_p037-048.indd 37 30/06/22 10:38 AM


38
3. The Sequential Organ Failure Assessment (SOFA) score: Answer: B
A. Includes AST and ALT. The SOFA score looks at PaO2/FiO2 ratio, bilirubin, platelet
CHAPTER 6

B. An increase of score in ≥2 is correlated with 10% count, mean arterial pressure (MAP), Glasgow Coma Scale
in-hospital mortality risk. (GCS) score, creatinine level, and urine output. An increase
C. Does not depend on lab tests. in SOFA score of ≥2 is correlated with a 10% in-hospital mor-
D. All of the above. tality risk, which is suggestive of the life-threatening nature
of sepsis. An abbreviated version of the scoring system, the
quick SOFA (qSOFA) is recommended as a screening and
Surgical Infection

monitoring tool for patients with suspected sepsis. The


qSOFA suggests potentially life-threatening sepsis when at
least two of the following parameters are met: altered mental
status, systolic blood pressure of ≤100 mm Hg, and respira-
tory rate >22 breaths/minute. The qSOFA can readily identify
patients at risk of poor outcome from sepsis without reliance
upon laboratory or imaging data. (Schwartz 11th ed., p. 161.)

4. The best method for hair removal from an operative Answer: D


field is: Patient skin preparation should begin the night before a
A. Razor the night before. planned surgical procedure with a full body bath or shower
B. Depilatory the night before surgery. using soap or an antiseptic agent. Hair removal from an oper-
C. Razor in the operating room. ative site should be performed in the operating room with
D. Hair clippers in the operating room. clippers rather than with a razor, to avoid creating nicks in the
skin that could foster bacterial growth. (Schwartz 11th ed.,
p. 163.)

5. Source Control refers to: Answer: D


A. Drainage of purulent material. The primary precept of surgical infectious disease therapy
B. Debridement of devitalized tissue, and removal of consists of drainage of all purulent material, debridement of
foreign bodies. all infected, devitalized tissue and debris, and/or removal of
C. Fixing the underlying cause of infection. foreign bodies at the site of infection, plus remediation of the
D. All of the above. underlying cause of infection. This is termed source control. A
discrete, walled-off purulent fluid collection (ie, an abscess)
requires drainage, either surgically or via percutaneous drain
insertion. An ongoing source of contamination (eg, bowel
perforation) or the presence of an aggressive, rapidly spread-
ing infection (eg, necrotizing soft tissue infection) invariably
requires expedient, aggressive operative intervention, both to
remove contaminated material and infected tissue (eg, radical
debridement or amputation) and to remove the initial cause
of infection (eg, bowel resection). (Schwartz 11th ed., p. 163.)

6. Which of the following statements is correct? Answer: A


A. Most cases require only a single dose of prophylactic Prophylaxis consists of the administration of an antimi-
antibiotics given immediately before and during the crobial agent or agents prior to initiation of certain specific
surgery. types of surgical procedures in order to reduce the number
B. Infection can be prevented with 24 hours of prophy- of microbes that enter the tissue or body cavity. Agents are
lactic antibiotics starting from the time of incision. selected according to their activity against microbes likely to
C. Empiric antibiotics imply that the bacterial infection be present at the surgical site, based on knowledge of host
is a known/identified bacteria. microflora. For example, patients undergoing elective colorec-
D. Empiric therapy should continue for 3 days only. tal surgery should receive antimicrobial prophylaxis directed
against skin flora, gram-negative aerobes, and anaerobic
bacteria. There are a wide variety of agents that meet these
criteria with recently published guidelines. By definition, pro-
phylaxis is limited to the time prior to and during the opera-
tive procedure; in the vast majority of cases only a single dose
of antibiotic is required, and only for certain types of proce-
dures. However, patients who undergo complex, prolonged
procedures in which the duration of the operation exceeds
the serum drug half-life should receive an additional dose or

Brunicardi_Ch06_p037-048.indd 38 30/06/22 10:38 AM


39
doses of the antimicrobial agent. There is no evidence that
administration of postoperative doses of an antimicrobial

CHAPTER 6
agent provides additional benefit, and this practice should be
discouraged, as it is costly and is associated with increased
rates of microbial drug resistance. (Schwartz 11th ed., p. 164.)

7. Which of the following factors does NOT influence the Answer: D


development of surgical site infections (SSIs)? SSIs are infections of the tissues, organs, or spaces exposed by

Surgical Infection
A. Duration of procedure surgeons during performance of an invasive procedure. SSIs
B. Degree of microbial contamination of the wound are classified into incisional and organ/space infections, and
C. Malnutrition the former are further subclassified into superficial (limited
D. General anesthesia to skin and subcutaneous tissue) and deep incisional cat-
egories. The development of SSIs is related to three factors:
(a) the degree of microbial contamination of the wound during
surgery; (b) the duration of the procedure; and (c) host factors
such as diabetes, malnutrition, obesity, immune suppression,
and a number of other underlying disease states. Table 6-1
lists risk factors for development of SSIs. By definition, an
incisional SSI has occurred if a surgical wound drains puru-
lent material or if the surgeon judges it to be infected and
opens it. (Schwartz 11th ed., p. 169.)

TABLE 6-1 Risk factors for development of surgical


site infections
Patient factors
Older age
Immunosuppression
Obesity
Diabetes mellitus
Chronic inflammatory process
Malnutrition
Smoking
Renal failure
Peripheral vascular disease
Anemia
Radiation
Chronic skin disease
Carrier state (eg, chronic Staphylococcus carriage)
Recent operation
Local factors
Open compared to laparoscopic surgery
Poor skin preparation
Contamination of instruments
Inadequate antibiotic prophylaxis
Prolonged procedure
Local tissue necrosis
Blood transfusion
Hypoxia, hypothermia
Microbial factors
Prolonged hospitalization (leading to nosocomial organisms)
Toxin secretion
Resistance to clearance (eg, capsule formation)

8. During and elective cholecystectomy, a large bowel injury Answer: B


was caused during trochar placement without spillage of Surgical wounds are classified based on the presumed mag-
bowel contents into the abdomen. What class of surgical nitude of the bacterial load at the time of surgery (Table 6-2).
wound is this? Clean wounds (class I) include those in which no infection
A. Class I (Clean) is present; only skin microflora potentially contaminate
B. Class II (Clean/contaminated) the wound, and no hollow viscus that contains microbes is
C. Class III (Contaminated) entered. Class I D wounds are similar except that a prosthetic
D. Class IV (Dirty) device (eg, mesh or valve) is inserted. Clean/contaminated

Brunicardi_Ch06_p037-048.indd 39 30/06/22 10:38 AM


40
TABLE 6-2 Wound class, representative procedures,
CHAPTER 6

and expected infection rates


Expected
Wound Class Examples of Cases Infection Rates
Clean (class I) Hernia repair, breast 1%–2%
biopsy
Clean/contaminated Cholecystectomy, 2.1%–9.5%
(class II) elective GI surgery
Surgical Infection

(not colon)
Clean/contaminated Colorectal surgery 4%–14%
(class II)
Contaminated Penetrating abdominal 3.4%–13.2%
(class III) trauma, large tissue
injury, enterotomy
during bowel
obstruction
Dirty (class IV) Perforated diverticulitis, 3.1%–12.8%
necrotizing soft tissue
infections

wounds (class II) include those in which a hollow viscus such


as the respiratory, alimentary, or genitourinary tracts with
indigenous bacterial flora is opened under controlled circum-
stances without significant spillage of contents.
While elective colorectal cases have classically been
included as class II cases, a number of studies in the last decade
have documented higher surgical site infection (SSI) rates
(9%–25%). One study identified two-thirds of infections pre-
senting after discharge from hospital, highlighting the need for
careful follow-up of these patients. Infection is also more com-
mon in cases involving entry into the rectal space. In a recent
single-center quality improvement study using a multidisci-
plinary approach, one group of clinicians has demonstrated
the ability to decrease SSI from 9.8% to 4.0%. Contaminated
wounds (class III) include open accidental wounds encoun-
tered early after injury, those with extensive introduction of
bacteria into a normally sterile area of the body due to major
breaks in sterile technique (eg, open cardiac massage), gross
spillage of viscus contents such as from the intestine, or
incision through inflamed, albeit nonpurulent tissue. Dirty
wounds (class IV) include traumatic wounds in which a sig-
nificant delay in treatment has occurred and in which necrotic
tissue is present, those created in the presence of overt infec-
tion as evidenced by the presence of purulent material, and
those created to access a perforated viscus accompanied by a
high degree of contamination (Schwartz 11th ed., p. 169.)

9. Which of the following are NOT TRUE? Answer: C


A. Postoperative hyperglycemia (>200) is associated The adverse effects of hyperglycemia on white blood cell
with increased risk of wound infection. (WBC) function have been well described. A number of
B. Intraoperative hypothermia is associated with studies in patients undergoing several different types of sur-
increased risk of wound infection. gery describe increased risk of surgical site infection (SSI) in
C. High O2 levels during the surgery increases the risk of patients with hyperglycemia, and the 2017 CDC guidelines for
wound infection. prevention of surgical site infection recommend maintaining
D. Hospitalized patients have a higher rate of wound blood glucose <200 mg/dL (11.1 mmol/L) in all patients dur-
infection. ing the perioperative period.
The respective effects of body temperature and the level of
inhaled oxygen during surgery on SSI rates also have been
studied, and both hypothermia and hypoxia during surgery

Brunicardi_Ch06_p037-048.indd 40 30/06/22 10:38 AM


41
are associated with a higher rate of SSI. There is conflict-
ing evidence regarding whether supplying higher levels of

CHAPTER 6
inhaled oxygen to perioperative patients reduces the rate of
SSI. Although an initial study provided evidence that patients
who received high levels of inhaled oxygen during colorec-
tal surgery developed fewer SSIs, a later meta-analysis sug-
gested that the overall benefit is small and may not warrant
use. The 2017 CDC guidelines, however, support administra-

Surgical Infection
tion of increased FiO2 during surgery and after extubation
in patients with normal pulmonary function receiving gen-
eral anesthesia as there has been some evidence of benefit.
Further evaluation via multicenter studies is needed prior to
implementation of hyperoxia as standard therapy, but it is
clear that intraoperative hypothermia and hypoxia should be
prevented. (Schwartz 11th ed., p. 170.)

10. Which of the following is TRUE about primary micro- Answer: B


bial peritonitis? Primary microbial peritonitis occurs when microbes invade
A. In primary microbial peritonitis, bacteria come from the normally sterile confines of the peritoneal cavity via hema-
the intestinal viscera. togenous dissemination from a distant source of infection
B. In primary microbial peritonitis, infections occur in or direct inoculation. This process is more common among
patients with ascites are monomicrobial. patients who retain large amounts of peritoneal fluid due to
C. In primary bacterial peritonitis, infections are gener- ascites, and among those individuals who are being treated for
ally mixed gram-negative organisms. renal failure via peritoneal dialysis. These infections invariably
D. Primary bacterial peritonitis generally requires lapa- are monomicrobial and rarely require surgical intervention.
rotomy for source control. The diagnosis is established based on identification of risk
factors as noted previously, physical examination that reveals
diffuse tenderness and guarding without localized findings,
absence of a surgically treatable source of infection on an imag-
ing study, and the presence of more than 250 neutrophils/mL
in fluid obtained via paracentesis. Cultures typically will dem-
onstrate the presence of gram-positive organisms in patients
undergoing peritoneal dialysis. In patients without this risk
factor, the most common etiologic organisms are Escherichia
coli, Klebsiella pneumoniae, and Streptococcus pneumoniae.
Treatment consists of administration of an antibiotic to
which the organism is sensitive; often 14 to 21 days of ther-
apy are required. Removal of indwelling devices, if present,
may be required for effective therapy of recurrent infections.
(Schwartz 11th ed., p. 170.)

11. The most appropriate treatment of a 4-cm hepatic Answer: C


abscess is: Hepatic abscesses are rare, currently accounting for approxi-
A. Antibiotic therapy alone. mately 15 per 100,000 hospital admissions in the United
B. Aspiration for culture and antibiotic therapy. States. Pyogenic abscesses account for approximately 80%
C. Percutaneous drainage and antibiotic therapy. of cases, the remaining 20% being equally divided among
D. Operative exploration, open drainage of the abscess, parasitic and fungal forms. Formerly, pyogenic liver abscesses
and antibiotic therapy. mainly were caused by pylephlebitis due to neglected appen-
dicitis or diverticulitis. Today, manipulation of the biliary
tract to treat a variety of diseases has become a more com-
mon cause, although in nearly 50% of patients no cause is
identified. The most common aerobic bacteria identified in
recent series include Escherichia coli, Klebsiella pneumoniae,
and other enteric bacilli, enterococci, and Pseudomonas spp.,
while the most common anaerobic bacteria are Bacteroides
spp., anaerobic streptococci, and Fusobacterium spp. Can-
dida albicans and other related yeast cause the majority of
fungal hepatic abscesses. Small (<1 cm), multiple abscesses
should be sampled and treated with a 4- to 6-week course of

Brunicardi_Ch06_p037-048.indd 41 30/06/22 10:38 AM


42
antibiotics. Larger abscesses are generally amenable to per-
cutaneous drainage, with parameters for antibiotic therapy
CHAPTER 6

and drain removal similar to those mentioned previously.


(Schwartz 11th ed., p. 172.)

12. Which of the following is TRUE about the treatment of Answer: C


pancreatic necrosis and infection? Secondary pancreatic infections (eg, infected pancreatic
A. Enteral feeding is contraindicated until resolution. necrosis or pancreatic abscess) occur in approximately 10% to
Surgical Infection

B. Pancreatic necrosis is an indication for early opera- 15% of patients who develop severe pancreatitis with necro-
tive debridement. sis. The surgical treatment of this disorder was pioneered
C. Open necrosectomy with repeated debridements by Bradley and Allen, who noted significant improvements
should occur in the first 2 weeks after illness. in outcome for patients undergoing repeated pancreatic
D. Early enteral feeding via a nasal-jejunal tube passed debridement of infected pancreatic necrosis. Care of patients
beyond the ligament of Treitz is associated with with severe acute pancreatitis includes staging with dynamic,
decreased development of infected pancreatic contrast-enhanced helical CT scan to evaluate the extent of
necrosis. pancreatitis (unless significant renal dysfunction exists, in
which case one should forego the use of contrast material)
coupled with the use of one of several prognostic scoring sys-
tems. Patients who exhibit clinical signs of instability (eg, oli-
guria, hypoxemia, large-volume fluid resuscitation) should be
carefully monitored in the ICU and undergo follow-up con-
trast CT examination when renal function has stabilized to
evaluate for development of local pancreatic complications.
Routine use of prophylactic antibiotics to prevent infected
pancreatic necrosis is not indicated. Early enteral feeding
using nasojejunal feeding tubes placed past the ligament of
Treitz has been associated with decreased development of
infected pancreatic necrosis, possibly due to a decrease in gut
translocation of bacteria (Schwartz 11th ed., p. 172.)

13. Which of the following is NOT TRUE of necrotizing Answer: C


soft-tissue infections? Initially, the diagnosis is established solely upon a con-
A. Fifty percent are polymicrobial. stellation of clinical findings, not all of which are present
B. Elderly, immunosuppressed, and peripheral vascular in every patient. Not surprisingly, patients often develop
disease patients have increased risk. sepsis syndrome or septic shock without an obvious cause.
C. Can be treated with antibiotics alone. The extremities, perineum, trunk, and torso are most com-
D. Is a clinical diagnosis, suspected in patients with monly affected, in that order. Careful examination should be
sepsis, skin changes or creptus, and pain. undertaken for an entry site such as a small break or sinus
in the skin from which grayish, turbid semipurulent material
(“dishwater pus”) can be expressed, as well as for the presence
of skin changes (bronze hue or brawny induration), blebs, or
crepitus. The patient often develops pain at the site of infec-
tion that appears to be out of proportion to any of the physical
manifestations. Any of these findings mandates immediate
surgical intervention, which should consist of incision and
direct visualization of potentially infected tissue (including
deep soft tissue, fascia, and underlying muscle) and radical
resection of affected areas. (Schwartz 11th ed., p. 174.)

14. Postoperative urinary tract infections: Answer: B


A. Are usually treated with a 7- to 10-day course of The presence of a postoperative UTI should be considered
antibiotics. based on urinalysis demonstrating white blood cells (WBCs)
B. Initial therapy should be directed by results of urine or bacteria, a positive test for leukocyte esterase, or a combi-
culture. nation of these elements. The diagnosis is established after
C. Are established by >10 4 CFU/mL of bacteria in urine >10 4 CFU/mL of microbes are identified by culture techniques
culture in asymptomatic patients. in symptomatic patients, or >10 5 CFU/mL in asymptomatic
D. Can be reduced by irrigating indwelling Foley individuals. Treatment for 3 to 5 days with a single antibiotic
catheters daily. directed against the most common organisms (eg, Escherichia
Coli, Klebsiella pneumonia) that achieves high levels in the

Brunicardi_Ch06_p037-048.indd 42 30/06/22 10:38 AM


43
urine is appropriate. Initial therapy is directed by Gram’s stain
results and is refined as culture results become available. Post-

CHAPTER 6
operative surgical patients should have indwelling urinary
catheters removed as quickly as possible, typically within 1 to
2 days, as long as they are mobile, to avoid the development of
a urinary tract infections (UTI).

15. Indwelling intravascular catheters infections: Answer: D

Surgical Infection
A. Are not associated with duration of catheter use. Infection associated with indwelling intravascular catheters
B. Single lumen and multilumen have similar infection is a common problem among hospitalized patients. Because
rates. of the complexity of many surgical procedures, these devices
C. Are always associated with purulence at the insertion are increasingly used for physiologic monitoring, vascular
site. access, drug delivery, and hyperalimentation. Among the sev-
D. Can be asymptomatic, with rising white blood cells eral million catheters inserted each year in the United States,
(WBCs) and positive blood culture. approximately 25% will become colonized, and approxi-
mately 5% will be associated with bacteremia. Duration of
catheterization, insertion or manipulation under emergency
or nonsterile conditions, use for hyperalimentation, and the
use of multilumen catheters increase the risk of infection. Use
of a central line insertion protocol that includes full barrier
precautions and chlorhexidine skin prep has been shown to
decrease the incidence of infection. Although no random-
ized trials have been performed, peripherally inserted central
venous catheters have a catheter-related infection rate similar
to those inserted in the subclavian or jugular veins.
Many patients who develop intravascular catheter infec-
tions are asymptomatic, often exhibiting solely an elevation
in the blood white blood cell (WBC) count. Blood cultures
obtained from a peripheral site and drawn through the cath-
eter that reveals the presence of the same organism increase
the index of suspicion for the presence of a catheter infec-
tion. Obvious purulence at the exit site of the skin tunnel,
severe sepsis syndrome due to any type of organism when
other potential causes have been excluded, or bacteremia due
to gram-negative aerobes or fungi should lead to catheter
removal. (Schwartz 11th ed., p. 174.)

16. Sepsis incidence and survival in the United States are: Answer: B
A. Increasing incidence and decreased survival due to The treatment of sepsis has improved over the last decade,
increasing age of the population. with mortality rates dropping to under 30%. Factors con-
B. Increased incidence with improved survival due to tributing to this improvement relate both to recent random-
improvements in care, including Surviving Sepsis ized prospective trials demonstrating improved outcomes
Campaign. with new therapies, and to improvements in the process of
C. Decreasing incidence and improved survival due to care delivery to the sepsis patient. The “Surviving Sepsis
proper use of perioperative antibiotics. Campaign,” a multidisciplinary group that develops treat-
D. Improved survival is due to delay in starting antibiot- ment recommendations, published guidelines incorporating
ics until source of sepsis is clear. evidence-based sepsis treatment strategies most recently in
2016. (Schwartz 11th ed., p. 174.)

17. Which of the following is Not TRUE about the risk of Answer: A
human immunodeficiency virus (HIV) transmission The risk of HIV transmission from patient to surgeon is low.
from patient to surgeon? As of May 2011, there had been six cases of surgeons with HIV
A. Risk from a needlestick is 3% and 1% from mucous seroconversion from a possible occupational exposure, with
membrane exposure. no new cases reported since 1999. Of the numbers of health
B. Transmission can be minimized by observation of care workers with likely occupationally acquired HIV infec-
universal precautions. tion (n = 200), surgeons were one of the lower risk groups
C. Postexposure prophylaxis is most effective if initiated (compared to nurses at 60 cases and nonsurgeon physicians
within hours. at 19 cases). The estimated risk of transmission from a needle-
D. Is decreased where patients have low viral load. stick from a source with HIV-infected blood is estimated

Brunicardi_Ch06_p037-048.indd 43 30/06/22 10:38 AM


44
at 0.3%. Transmission of HIV (and other infections spread by
blood and body fluid) from patient to health care worker can
CHAPTER 6

be minimized by observation of universal precautions, includ-


ing: (a) routine use of barriers (gloves, gown, mask, eye pro-
tection) when anticipating contact with blood or body fluids,
(b) washing hands and other skin surfaces immediately after
contact with blood or body fluids, and (c) careful handling and
disposal of sharp instruments during and after use. Postexpo-
Surgical Infection

sure prophylaxis for HIV has significantly decreased the risk


of seroconversion for health care workers with occupational
exposure to HIV. Steps to initiate postexposure prophylaxis
should be initiated within hours for the most effective pre-
ventive therapy. Postexposure prophylaxis with a three-drug
regimen should be initiated for health care workers with sig-
nificant exposure to patients with an HIV-positive status.

18. A chronic carrier state occurs in patients with hepatitis C Answer: B


infection in what percentage of patients? Hepatitis C virus (HCV), previously known as non-A, non-B
A. 90%–99% hepatitis, is a RNA flavivirus first identified specifically in
B. 75%–80% the late 1980s. This virus is confined to humans and chim-
C. 50%–60% panzees. A chronic carrier state develops in 75% to 80% of
D. 10%–30% patients with the infection, with chronic liver disease occur-
ring in three-fourths of patients who develop chronic infec-
tion. The number of new infections per year has declined
since the 1980s due to routine testing of blood donors for
this virus. Fortunately, HCV is not transmitted efficiently
through occupational exposures to blood, with the serocon-
version rate after accidental needlestick approximately 1.8%.
(Schwartz 11th ed., p. 178.)

19. Possible exposure to anthrax should be initially treated Answer: B


with: Inhalational anthrax develops after a 1- to 6-day incubation
A. Colistin. period, with nonspecific symptoms including malaise, myal-
B. Ciprofloxacin or doxycycline. gia, and fever. Over a short period of time, these symptoms
C. Amoxcillin. worsen, with development of respiratory distress, chest pain,
D. Observation. and diaphoresis. Characteristic chest roentgenographic find-
ings include a widened mediastinum and pleural effusions. A
key aspect in establishing the diagnosis is eliciting an exposure
history. Rapid antigen tests are currently under development
for identification of this gram-positive rod. Postexposure
prophylaxis consists of administration of either ciprofloxacin
or doxycycline. If an isolate is demonstrated to be penicillin-
sensitive, the patient should be switched to amoxicillin. Inha-
lational exposure followed by the development of symptoms
is associated with a high mortality rate. Treatment options
include combination therapy with ciprofloxacin, clindamy-
cin, and rifampin; clindamycin added to blocks production
of toxin, while rifampin penetrates into the central nervous
system and intracellular locations (Schwartz 11th ed., p. 178.)

20. A patient in the ICU has been on ventilator support for Answer: B
3 weeks. He has new onset elevated white blood cells Prolonged mechanical ventilation is associated with noso-
(WBCs) count, fever, and consolidation seen on chest comial pneumonia. These patients present with more severe
x-ray. What is an appropriate next step? disease, are more likely to be infected with drug-resistant
A. Exchange endotracheal tube and change respiratory pathogens, and suffer increased mortality compared to
circuit. patients who develop community-acquired pneumonia. The
B. Obtain bronchoalveolar lavage. diagnosis of pneumonia is established by presence of a puru-
C. Start treatment with empiric penicillin G. lent sputum, elevated leukocyte count, fever, and new chest
D. Obtain chest CT. x-ray abnormalities such as consolidation. The presence of

Brunicardi_Ch06_p037-048.indd 44 30/06/22 10:38 AM


45
two of the clinical findings, plus chest X-ray findings, sig-
nificantly increases the likelihood of pneumonia. Consider-

CHAPTER 6
ation should be given to perform bronchoalveolar lavage to
obtain samples for Gram’s stain and culture. Some authors
advocate quantitative cultures as a means to identify a thresh-
old for diagnosis. Surgical patients should be weaned from
mechanical ventilation as soon as feasible, based on oxygen-
ation and inspiratory effort, as prolonged mechanical ventila-

Surgical Infection
tion increases the risk of nosocomial pneumonia. (Schwartz
11th ed., p. 174.)

21. Which of the following areas likely do NOT contain Answer: C


resident micro-organisms? The urogenital, biliary, pancreatic ductal, and distal respi-
A. Terminal ileum ratory tracts do not possess resident microflora in healthy
B. Oropharynx individuals, although microbes may be present if these bar-
C. Main pancreatic duct riers are affected by disease (eg, malignancy, inflammation,
D. Nares calculi, or foreign body), or if microorganisms are introduced
from an external source (eg, urinary catheter or pulmonary
aspiration). In contrast, significant number of microbes are
encountered in many portions of the gastrointestinal tract,
with vast numbers being found within the oropharynx and
distal colorectum, although the specific organisms differ.
(Schwartz 11th ed., p. 159.)

22. Which of the following is not TRUE about allergy to Answer: B


Antibiotics? Allergy to antimicrobial agents must be considered prior to
A. Urticaria, bronchospasm, and other systemic mani- prescribing them. First, it is important to ascertain whether
festations are signs of allergy. a patient has had any type of allergic reaction in association
B. True penicillin allergy is uncommon, occurring in with administration of a particular antibiotic. However, one
<1% of adults. should take care to ensure that the purported reaction con-
C. Cross-reactivity between penicillin and carbapenems sists of true allergic symptoms and signs, such as urticaria,
is 1%. bronchospasm, or other similar manifestations, rather than
D. Cross-reactivity between penicillin and cephalospo- indigestion or nausea. Penicillin allergy is quite common,
rins is 5%–7%. the reported incidence ranging from 0.7% to 10%. Although
avoiding the use of any β-lactam drug is appropriate in
patients who manifest significant allergic reactions to peni-
cillins, the incidence of cross-reactivity appears low for all
related agents, with 1% cross-reactivity for carbapenems, 5%
to 7% cross-reactivity for cephalosporins, and extremely small
or nonexistent cross-reactivity for monobactams. (Schwartz
11th ed., p. 168.)

23. Extended perioperative treatment with antibiotics results Answer: C


in: Misuse of antimicrobial agents is rampant in both the inpa-
A. Decreased health care costs. tient and outpatient settings, and is associated with an enor-
B. Avoids development of multidrug resistance among mous financial impact on health care costs, adverse reactions
nosocomial pathogens. due to drug toxicity and allergy, the occurrence of new infec-
C. Clostridium difficile colitis. tions such as Clostridium difficile colitis, and the development
D. Improved outcomes. of multiagent drug resistance among nosocomial pathogens.
Each of these factors has been directly correlated with overall
drug administration. It has been estimated that in the United
States in excess of $20 billion is spent on antibiotics each year.
The responsible practitioner limits prophylaxis to the period
during the operative procedure, does not convert prophylaxis
into empiric therapy except under well-defined conditions,
sets the duration of antibiotic therapy from the outset, cur-
tails antibiotic administration when clinical and microbio-
logic evidence does not support the presence of an infection,
and limits therapy to a short course in every possible instance.

Brunicardi_Ch06_p037-048.indd 45 30/06/22 10:38 AM


46
Prolonged treatment associated with drains and tubes has not
been shown to be beneficial. (Schwartz 11th ed., p. 169.)
CHAPTER 6

24. A patient with necrotizing pancreatitis undergoes com- Answer: D


puted tomography (CT)-guided aspiration, which results The primary precept of surgical infectious disease therapy
in growth of Escherichia coli on culture. The most appro- consists of drainage of all purulent material, debridement of
priate treatment is: all infected, devitalized tissue, and debris, and/or removal
A. Culture-appropriate antibiotic therapy. of foreign bodies at the site of infection, plus remediation
Surgical Infection

B. Endoscopic retrograde cholangiopancreatography of the underlying cause of infection. A discrete, walled-off


with sphincterotomy. purulent fluid collection (ie, an abscess) requires drainage
C. CT-guided placement of drain(s). via percutaneous drain insertion or an operative approach in
D. Exploratory laparotomy. which incision and drainage take place. An ongoing source
of contamination (eg, bowel perforation) or the presence of
an aggressive, rapidly spreading infection (eg, necrotizing
soft tissue infection) invariably requires expedient, aggressive
operative intervention, both to remove contaminated mate-
rial and infected tissue (eg, radical debridement or amputa-
tion) and to remove the initial cause of infection (eg, bowel
resection). (See Schwartz 11th ed., p. 163.)

25. The first step in the evaluation and treatment of a patient Answer: D
with an infected bug bite on the leg with cellulitis, bullae, The diagnosis of necrotizing infection is established solely
thin grayish fluid draining from the wound, and pain out upon a constellation of clinical findings, not all of which
of proportion to the physical findings is: are present in every patient. Not surprisingly, patients often
A. Obtain C-reactive protein. develop sepsis syndrome or septic shock without an obvious
B. Computed tomographic scan of the leg. cause. The extremities, perineum, trunk, and torso are most
C. Magnetic resonance imaging of the leg. commonly affected, in that order. Careful examination should
D. Operative exploration. be undertaken for an entry site such as a small break or sinus
in the skin from which grayish, turbid semipurulent material
(“dishwater pus”) can be expressed, as well as for the presence
of skin changes (bronze hue or brawny induration), blebs, or
crepitus. The patient often develops pain at the site of infec-
tion that appears to be out of proportion to any of the physical
manifestations. Any of these findings mandates immediate
surgical intervention, which should consist of exposure and
direct visualization of potentially infected tissue (including
deep soft tissue, fascia, and underlying muscle) and radi-
cal resection of affected areas. Radiologic studies should be
undertaken only in patients in whom the diagnosis is not
seriously considered, as they delay surgical intervention and
frequently provide confusing information. Unfortunately,
surgical extirpation of infected tissue frequently entails
amputation and/or disfiguring procedures; however, incom-
plete procedures are associated with higher rates of morbidity
and mortality. (See Schwartz 11th ed., p. 174.)

26 Which of the following is FALSE regarding intravascular Answer: B


catheter infections? Many patients who develop intravascular catheter infections
A. Select low-virulence infections can be treated with a are asymptomatic, often exhibiting solely an elevation in the
prolonged course of antibiotics. blood white blood cell (WBC) count. Blood cultures obtained
B. In high-risk patients, prophylactic antibiotics infused from a peripheral site and drawn through the catheter that
through the catheter can reduce rate of catheter reveal the presence of the same organism increase the index
infections. of suspicion for the presence of a catheter infection. Obvi-
C. Bacteremia with gram-negative bacteria or fungi ous purulence at the exit site of the skin tunnel, severe sepsis
should prompt catheter removal. syndrome due to any type of organism when other poten-
D. Many patients with intravascular catheter infections tial causes have been excluded, or bacteremia due to gram-
are asymptomatic. negative aerobes or fungi should lead to catheter removal.
Selected catheter infections due to low-virulence microbes
such as Staphylococcus epidermidis can be effectively treated

Brunicardi_Ch06_p037-048.indd 46 30/06/22 10:38 AM


47
in approximately 50% to 60% of patients with a 14- to 21-day
course of an antibiotic, which should be considered when no

CHAPTER 6
other vascular access site exists. Use of systemic antibacterial
or antifungal agents to prevent catheter infection is of no util-
ity and is contraindicated. (See Schwartz 11th ed., p. 174.)

27. Closure of an appendectomy wound in a patient with Answer: A


perforated appendicitis, who is receiving appropriate Surgical management of the wound is also a critical deter-

Surgical Infection
antibiotics, will result in a wound infection in what per- minant of the propensity to develop a surgical site infec-
centage of patients? tion (SSI). In healthy individuals, class I and II wounds
A. 3%–4% may be closed primarily, while skin closure of class III and
B. 8%–12% IV wounds is associated with high rates of incisional SSIs
C. 15%–18% (~25% to 50%). The superficial aspects of these latter types
D. 22%–25% of wounds should be packed open and allowed to heal by sec-
ondary intention, although selective use of delayed primary
closure has been associated with a reduction in incisional
SSI rates. It remains to be determined whether National
Nosocomial Infections Surveillance System (NNIS)-type
stratification schemes can be employed prospectively in
order to target specific subgroups of patients who will ben-
efit from the use of prophylactic antibiotic and/or specific
wound management techniques. One clear example based
on cogent data from clinical trials is that class III wounds
in healthy patients undergoing appendectomy for perforated
or gangrenous appendicitis can be primarily closed as long
as antibiotic therapy directed against aerobes and anaerobes
is administered. This practice leads to SSI rates of approxi-
mately 3% to 4%. (See Schwartz 11th ed., p. 170.)

Brunicardi_Ch06_p037-048.indd 47 30/06/22 10:38 AM


This page intentionally left blank

Brunicardi_Ch06_p037-048.indd 48 30/06/22 10:38 AM


CHAPTER 7
Trauma

1. A trauma patient arrives following a stab wound to the Answer: C


left chest with systolic blood pressure (SBP) 85 mm Hg, During the circulation section of the primary survey, four life-
which improves slightly with intravenous (IV) fluid threatening injuries must be identified promptly: (1) massive
resuscitation. Chest X-ray demonstrates clear lung fields. hemothorax, (2) cardiac tamponade, (3) massive hemoperi-
What is the most appropriate next step? toneum, and (4) mechanically unstable pelvic fractures with
A. Computed tomography (CT) scan of the chest bleeding. In this patient, hemothorax is unlikely given normal
B. Pelvic X-ray chest X-ray; thus, hemoperitoneum and cardiac tamponade
C. Focused abdominal sonography for trauma (FAST) should be suspected. Cardiac tamponade occurs most com-
examination monly after penetrating thoracic wounds, although occa-
D. Tube thoracostomy of the left chest sionally blunt rupture of the heart, particularly the atrial
appendage, is seen. Acutely, <100 mL of pericardial blood
may cause pericardial tamponade. The classic Beck’s triad—
dilated neck veins, muffled heart tones, and a decline in
arterial pressure—is usually not appreciated in the trauma
bay because of the noisy environment and associated hypo-
volemia. Diagnosis is best achieved by bedside ultrasound of
the pericardium, which is one of the four views of the FAST
examination. (See Schwartz 11th ed., Ch. 7, p. 188.)

2. In which of the following patients is emergency depart- Answer: D


ment (ED) thoracotomy contraindicated? The utility of resuscitative thoracotomy (RT) has been
A. Motor vehicle accident victim, cardiac tamponade debated for decades. Current indications are based on 30 years
seen on ultrasound, systolic blood pressure (SBP) of prospective data, supported by a recent multicenter pro-
decreasing to 50 mm Hg spective study. RT is associated with the highest survival rate
B. Motor vehicle accident victim, became asystolic dur- after isolated cardiac injury; 35% of patients presenting in
ing transport with 5 minutes of cardiopulmonary shock and 20% without vital signs (ie, no pulse or obtainable
resuscitation (CPR) with no signs of life BP) are salvaged after isolated penetrating injury to the heart.
C. Patient with chest stab wound, SBP decreasing to For all penetrating wounds, survival rate is 15%. Conversely,
50 mm Hg patient outcome is poor when RT is done for blunt trauma,
D. Patient with chest stab wound, became asystolic dur- with 2% survival among patients in shock and <1% survival
ing transport with 20 minutes of CPR with no signs among those with no vital signs. Thus, patients undergoing
of life CPR upon arrival to the ED should undergo RT selectively
based on injury and transport time. (See Schwartz 11th ed.,
Ch. 7, p. 189.)

49

Brunicardi_Ch07_p049-064.indd 49 04/07/22 1:26 PM


50
3. After an automobile accident, a 30-year-old woman is Answer: D
discovered to have a posterior pelvic fracture. Hypo- Severe pelvic bleeding is a major problem in the trauma
CHAPTER 7

tension and tachycardia respond marginally to volume patient. Neither external fixation nor the use of medical anti-
replacement. Once it is evident that her major problem shock trousers control free intra-abdominal hemorrhage
is free intraperitoneal bleeding and a pelvic hematoma in regardless of its source. In the unstable patient, celiotomy is
association with the fracture, appropriate management mandatory. If there is a ruptured retroperitoneal hematoma
would be: bleeding into the peritoneal cavity, control is a major prob-
A. Application of medical antishock trousers with infla- lem. Internal iliac artery ligation has been abandoned as it
Trauma

tion of the extremity and abdominal sections. is rarely effective. Angiography and arterial embolization
B. Arterial embolization of the pelvic vessels. may be effective with an arterial bleeding problem, but most
C. Celiotomy and ligation of the internal iliac arteries severe pelvic hemorrhage is venous in origin. If the hema-
bilaterally. toma is stable, it is best to leave it undisturbed. However, if
D. Celiotomy and pelvic packing. the hematoma has ruptured into the peritoneal cavity, pelvic
E. External fixation application to stabilize the pelvis. packing offers the best hope of control. (See Schwartz 11th ed.,
Ch. 7, p. 234.)

4. An 18-year-old man is admitted to the emergency Answer: E


department (ED) shortly after being involved in an auto- Ideally, a patient seriously injured in an automobile accident
mobile accident. His Glasgow Coma Score (GCS) is 7. should undergo X-rays of the cervical spine, the chest, and
His pulse is barely palpable at a rate of 140 beats per the abdomen. When he has a GCS of 7, CT scans of the head
minute, and BP is 60/0. Breathing is rapid and shallow, are certainly desirable. If the chest X-ray shows a widened
aerating both lung fields. His abdomen is moderately mediastinum, arch aortograms are indicated. However, this
distended with no audible peristalsis. There are closed patient has had no response to a rapid fluid challenge, and
fractures of the right forearm and the left lower leg. After if he is to survive, bleeding must be controlled immediately.
rapid intravenous (IV) administration of 2 L of lactated The head injury, although severe, is not responsible for his
Ringer solution in the upper extremities, his pulse is 130 hypotension and tachycardia. The most likely problem is
and BP 70/0. The next immediate step should be to: uncontrolled abdominal hemorrhage. Immediate abdominal
A. Obtain cross-table lateral X-rays of the cervical spine. exploration offers the best chance for survival. (See Schwartz
B. Obtain head and abdominal CT scans. 11th ed., Ch. 7, p. 193.)
C. Obtain supine and lateral decubitus X-rays of the
abdomen.
D. Obtain an arch aortogram.
E. Intubate and proceed to the operating room (OR) to
explore the abdomen.

5. After sustaining a gunshot wound to the right upper Answer: B


quadrant of the abdomen, the patient has no signs The liver’s large size makes it the organ most susceptible to
of peritonitis. Her vital signs are stable, and CT scan blunt trauma, and it is frequently involved in upper torso pen-
shows a grade III liver injury. What is the next step in etrating wounds. Nonoperative management of solid organ
management? injuries is pursued in hemodynamically stable patients who do
A. Exploratory laparotomy with control of hepatic not have overt peritonitis or other indications for laparotomy.
parenchymal hemorrhage Patients with more than grade II injuries should be admitted
B. Admission to surgical intensive care unit (SICU) with to the SICU with frequent hemodynamic monitoring, deter-
serial complete blood count mination of hemoglobin, and abdominal examination. The
C. Admission to SICU with repeat CT in 24 hours only absolute contraindication to nonoperative management
D. Hepatic angiography is hemodynamic instability. Factors such as high injury grade,
large hemoperitoneum, contrast extravasation, or pseudoan-
eurysms may predict complications or failure of nonoperative
management. Angioembolization and endoscopic retrograde
cholangiopancreatography (ERCP) are useful adjuncts that
can improve the success rate of nonoperative management.
The indication for angiography to control hepatic hemor-
rhage is transfusion of 4 units of RBCs in 6 hours or 6 units
of RBCs in 24 hours without hemodynamic instability. (See
Schwartz 11th ed., Ch. 7, p. 225.)

Brunicardi_Ch07_p049-064.indd 50 04/07/22 1:26 PM


51
6. Which of the following statements is correct regarding Answer: C
traumatic spleen injury? After splenectomy or splenorrhaphy, postoperative hemor-

CHAPTER 7
A. An elevation in WBC to 20,000/mm3 and platelets to rhage may be due to an improperly ligated or unrecognized
300,000/mm3 on postoperative day 7 is a common short gastric artery, or recurrent bleeding from the splenic
benign finding in postsplenectomy patients. parenchyma if splenic repair was used. An immediate post-
B. Delayed rebleeding or rupture will typically occur splenectomy increase in platelets and WBCs is normal;
within 48 hours of injury. however, beyond postoperative day 5, a WBC count above
C. Common complications after splenectomy include 15,000/mm3 and a platelet/WBC ratio of <20 are associated

Trauma
subdiaphragmatic abscess, pancreatic tail injury, and with sepsis and should prompt a thorough search for underly-
gastric perforation. ing infection. A common infectious complication after sple-
D. Postsplenectomy vaccines against encapsulated bac- nectomy is a subphrenic abscess, which should be managed
teria are optimally administered preoperatively or with percutaneous drainage. Additional sources of morbidity
immediately postoperative. include a concurrent or unrecognized iatrogenic injury to the
pancreatic tail during rapid splenectomy resulting in pancre-
atic ascites or fistula, or gastric perforation during short gas-
tric vessel ligation. Enthusiasm for splenic salvage was driven
by the rare, but often fatal, complication of overwhelming
postsplenectomy sepsis. Overwhelming postsplenectomy
sepsis is caused by encapsulated bacteria, Streptococcus pneu-
moniae, Haemophilus influenzae, and Neisseria meningitidis,
which are resistant to antimicrobial treatment. In patients
undergoing splenectomy, prophylaxis against these bacteria
is provided via vaccines administered optimally at >14 days
postinjury. (See Schwartz 11th ed., Ch. 7, p. 227.)

7. At what pressure is operative decompression of an Answer: D


extremity compartment mandatory? In comatose or obtunded patients, the diagnosis is more dif-
A. 15 mm Hg ficult to secure. A compatible history, firmness of the com-
B. 25 mm Hg partment to palpation, and diminished mobility of the joint
C. 35 mm Hg are suggestive. The presence or absence of a pulse distal to
D. 45 mm Hg the affected compartment is notoriously unreliable in the
diagnosis of a compartment syndrome. A frozen joint and
myoglobinuria are late signs and suggest a poor prognosis.
As in the abdomen, compartment pressure can be measured.
The small, hand held Stryker device is a convenient tool for
this purpose. Pressures >45 mm Hg usually require operative
intervention. Patients with pressures between 30 and 45 mm
Hg should be carefully evaluated and closely watched. (See
Schwartz 11th ed., Ch. 7, p. 237.)

8. Primary repair of the trachea should be carried out with: Answer: B


A. Wire suture. Injuries of the trachea are repaired with a running 3-0 absorb-
B. Absorbable monofilament suture. able monofilament suture. Tracheostomy is not required in
C. Nonabsorbable monofilament suture. most patients. Esophageal injuries are repaired in a similar
D. Absorbable braided suture. fashion. If an esophageal wound is large or if tissue is miss-
ing, a sternocleidomastoid muscle pedicle flap is warranted,
and a closed suction drain is a reasonable precaution. The
drain should be near but not in contact with the esophageal or
any other suture line. It can be removed in 7 to 10 days if the
suture line remains secure. Care must be taken when explor-
ing the trachea and esophagus to avoid iatrogenic injury
to the recurrent laryngeal nerve. (See Schwartz 11th ed.,
Ch. 7, p. 224.)

Brunicardi_Ch07_p049-064.indd 51 04/07/22 1:26 PM


52
9. A patient with spontaneous eye opening, who is confused Answer: C
and localizes pain has a Glasgow Coma Score (GCS) of: The GCS should be determined for all injured patients
CHAPTER 7

A. 9. (Table 7-1). It is calculated by adding the scores of the best


B. 11. motor response, best verbal response, and eye opening. Scores
C. 13. range from 3 (the lowest) to 15 (normal). Scores of 13 to 15
D. 15. indicate mild head injury, 9 to 12 moderate injury, and <9
severe injury. The GCS is useful for both triage and prognosis.
(See Schwartz 11th ed., Ch. 7, p. 192.)
Trauma

TABLE 7-1 Glasgow Coma Scalea


Adults Infants/Children
Eye opening 4 Spontaneous Spontaneous
3 To voice To voice
2 To pain To pain
1 None None
Verbal 5 Oriented Alert, normal vocalization
4 Confused Cries, but consolable
3 Inappropriate words Persistently irritable
2 Incomprehensible words Restless, agitated, moaning
1 None None
Motor response 6 Obeys commands Spontaneous, purposeful
5 Localizes pain Localizes pain
4 Withdraws Withdraws
3 Abnormal flexion Abnormal flexion
2 Abnormal extension Abnormal extension
1 None None
Score is calculated by adding the scores of the best motor response, best verbal response, and eye opening. Scores range from 3 (the lowest) to 15 (normal).
a

10. Neck injuries: Answer: A


A. Less than 15% penetrating injuries require neck explo- Zone I is inferior to the clavicles encompassing the thoracic
ration, a majority can be managed conservatively. outlet structures, zone II is between the thoracic outlet and
B. Divided into three zones, with zone I above the angle the angle of the mandible, and zone III is above the angle
of the mandible, zone II between the thoracic outlet of the mandible. Patients with symptomatic zone I and III
and angle of mandible, and zone III inferior to the injuries should ideally undergo diagnostic imaging before
clavicles. operation if they remain hemodynamically stable. Specific
C. All patients with neck injury should receive com- symptoms which indicate further imaging include dysphagia,
puted tomography angiogram (CTA) of the neck. hoarseness, hematoma, venous bleeding, minor hemoptysis,
D. Patients with dysphagia, hoarseness, hematoma, and subcutaneous emphysema. Symptomatic patients should
venous bleeding, hemoptysis, or subcutaneous undergo CTA with further evaluation or operation based
emphysema should undergo neck exploration. upon the imaging findings; <15% of penetrating cervical
trauma requires neck exploration. Asymptomatic patients are
typically observed for 6 to 12 hours. The one caveat is asymp-
tomatic patients with a transcervical gunshot wound; these
patients should undergo CTA to determine the track of the
bullet. CTA of the neck and chest determines trajectory of the
injury tract; further studies are performed based on proxim-
ity to major structures. Angiographic diagnosis, particularly
of zone III injuries, can then be managed by selective angio-
embolization. (See Schwartz 11th ed., Ch. 7, p. 198.)

Brunicardi_Ch07_p049-064.indd 52 04/07/22 1:26 PM


53
11. The most appropriate treatment for a duodenal hema- Answer: D
toma that occurs from blunt trauma is: The spectrum of injuries to the duodenum includes hema-

CHAPTER 7
A. Exploratory laparotomy and bypass of the duodenum. tomas, perforation (blunt blow-outs, lacerations from stab
B. Exploratory laparotomy and evacuation of the wounds, or blast injury from gunshot wounds), and com-
hematoma. bined pancreaticoduodenal injuries. The majority of duode-
C. Exploratory laparotomy to rule out associated nal hematomas are managed nonoperatively with nasogastric
injuries. suction and parenteral nutrition. Patients with suspected
D. Observation. associated perforation, suggested by clinical deterioration or

Trauma
imaging with retroperitoneal free air or contrast extravasa-
tion, should undergo operative exploration. A marked drop in
nasogastric tube output heralds resolution of the hematoma,
which typically occurs within 2 weeks; repeat imaging to con-
firm these clinical findings is optional. If the patient shows
no clinical or radiographic improvement within 3 weeks,
operative evaluation is warranted. (See Schwartz 11th ed.,
Ch. 7, p. 229.)

12. Cerebral perfusion pressure (CPP): Answer: D


A. Equals the systolic blood pressure (SBP) minus The goal of resuscitation and management in patients with
increased intracranial pressure (ICP). head injuries is to avoid hypotension (SBP of <100 mm Hg)
B. Should be targeted to be >100 mm Hg. and hypoxia (partial pressure of arterial oxygen of <60 or
C. Is lowered with sedation, osmotic diuresis, paralysis, arterial oxygen saturation of <90). Attention, therefore, is
ventricular drainage, and barbiturate coma. focused on maintaining cerebral perfusion rather than merely
D. Can be increased by lowering ICP and avoiding lowering ICP. Resuscitation efforts aim for a euvolemic state
hypotension. and an SBP of >100 mm Hg. CPP is equal to the mean arterial
pressure minus the ICP, with a target range of >50 mm Hg.
CPP can be increased by either lowering ICP or raising mean
arterial pressure. Sedation, osmotic diuresis, paralysis, ven-
tricular drainage, and barbiturate coma are used in sequence,
with coma induction being the last resort. (See Schwartz 11th
ed., Ch. 7, p. 218.)

13. A patient with penetrating injury to the chest should Answer: B


undergo thoracotomy if: The most common injuries from both blunt and penetrating
A. There is >500 mL of blood which drains from the thoracic trauma are hemothorax and pneumothorax. More than
chest tube when placed. 85% of patients can be definitively treated with a chest tube.
B. There is >200 mL/h of blood for 3 hours from the The indications for thoracotomy include significant initial or
chest tube. ongoing hemorrhage from the tube thoracostomy and specific
C. There is an air leak that persists for >48 hours. imaging-identified diagnoses. One caveat concerns the patient
D. There is documented lung injury on CT scan. who presents after a delay. Even when the initial chest tube out-
put is 1.5 L, if the output ceases and the lung is re-expanded, the
patient may be managed nonoperatively, if hemodynamically
stable. (See Schwartz 11th ed., Ch. 7, p. 222.)

14. A 19-year-old man fell off his skateboard, reporting Answer: C


blunt injury to his upper abdomen. Abdominal CT and Optimal management of pancreatic trauma is determined by
magnetic resonance cholangiopancreatography (MRCP) where the parenchymal damage is located and whether the
confirmed he suffered transection of the main pancre- intrapancreatic common bile duct and main pancreatic duct
atic duct at the middle of the pancreatic body. Which of remain intact. Patients with pancreatic contusions (defined
the following would be the most appropriate next step in as injuries that leave the ductal system intact) can be treated
management? nonoperatively or with closed suction drainage if undergo-
A. Nonoperative treatment ing laparotomy for other indications. Patients with proximal
B. Endoscopic retrograde cholangiopancreatography pancreatic injuries, defined as those that lie to the right of
(ERCP) with stenting of pancreatic duct the superior mesenteric vessels, are also managed with closed
C. Distal pancreatectomy with splenic preservation suction drainage. In contrast, distal pancreatic injuries are
D. Primary repair of pancreatic duct with closed suction managed based upon ductal integrity. Pancreatic duct dis-
drainage ruption can be identified through direct exploration of the
parenchymal laceration, operative pancreatography, ERCP,
or MRCP. Patients with distal ductal disruption undergo

Brunicardi_Ch07_p049-064.indd 53 04/07/22 1:26 PM


54
distal pancreatectomy, preferably with splenic preservation.
An alternative, which preserves both the spleen and distal
CHAPTER 7

transected end of the pancreas, is either a Roux-en-Y pan-


creaticojejunostomy or pancreaticogastrostomy. If the patient
is physiologically compromised, distal pancreatectomy with
splenectomy is the preferred approach. (See Schwartz 11th
ed., Ch. 7, p. 229.)
Trauma

15. The most appropriate treatment for a gunshot wound to Answer: D


the hepatic flexure of the colon that cannot be repaired Numerous large retrospective and several prospective stud-
primarily is: ies have now clearly demonstrated that primary repair is safe
A. End colostomy and mucous fistula. and effective in the majority of patients with penetrating inju-
B. Loop colostomy. ries. Colostomy is still appropriate in a few patients, but the
C. Exteriorized repair. current dilemma is how to select them. Exteriorized repair is
D. Resection of the right colon with ileocolostomy. probably no longer indicated since most patients who were
once candidates for this treatment are now successfully man-
aged by primary repair. Two methods have been advocated
that result in 75% to 90% of penetrating colonic injuries
being safely treated by primary repair. The first is to repair all
perforations not requiring resection. If resection is required
due to the local extent of the injury, and it is proximal to the
middle colic artery, the proximal portion of the right colon up
to and including the injury is resected and an ileocolostomy
performed. If resection is required distal to the middle colic
artery, an end colostomy is created and the distal colon over-
sewn and left within the abdomen. The theory behind this
approach is that an ileocolostomy heals more reliably than
colocolostomy, because in the trauma patient who has suf-
fered shock and may be hypovolemic, assessing the adequacy
of the blood supply of the colon is much less reliable than in
elective procedures. The blood supply of the terminal ileum
is never a problem. The other approach is to repair all inju-
ries regardless of the extent and location (including coloco-
lostomy), and reserve colostomy for patients with protracted
shock and extensive contamination. The theory used to sup-
port this approach is that systemic factors are more impor-
tant than local factors in determining whether a suture line
will heal. Both of these approaches are reasonable and result
in the majority of patients being treated by primary repairs.
When a colostomy is required, regardless of the theory used
to reach that conclusion, performing a loop colostomy proxi-
mal to a distal repair should be avoided because a proximal
colostomy does not protect a distal suture line. All suture lines
and anastomoses are performed with the running single-layer
technique. (See Schwartz 11th ed., Ch. 7, p. 230.)

16. Which of the following statements is true of vascular Answer: B


injuries of the extremities? Physical examination often identifies arterial injuries, and
A. In the absence of hard signs of vascular injury, if the findings are classified as either hard signs or soft signs of
difference between systolic blood pressure (SBP) in vascular injury (Table 7-2). In general, hard signs constitute
an injured limb is within 15% of the uninjured limb, indications for operative exploration, whereas soft signs are
no further evaluation is needed. indications for further testing or observation. Bony fractures
B. Occult profunda femoris injuries can result in com- or knee dislocations should be realigned before definitive vas-
partment syndrome and limb loss. cular examination. In management of vascular trauma, con-
C. All patients with significant hematoma should be troversy exists regarding the treatment of patients with soft
surgically explored. signs of injury, particularly those with injuries in proximity
D. Vascular injury repair should be performed prior to to major vessels. It is known that some of these patients will
realignment of bony fractures or dislocations. have arterial injuries that require repair. The most common

Brunicardi_Ch07_p049-064.indd 54 04/07/22 1:26 PM


55
approach has been to measure SBP using Doppler ultraso-
nography and compare the value for the injured side with that

CHAPTER 7
for the uninjured side, termed the A-A index. If the pressures
are within 10% of each other, a significant injury is unlikely
and no further evaluation is performed. If the difference is
>10%, computed tomography angiogram (CTA) or arteriog-
raphy is indicated. Others argue that there are occult injuries,
such as pseudoaneurysms or injuries of the profunda femo-

Trauma
ris or peroneal arteries, which may not be detected with this
technique. If hemorrhage occurs from these injuries, com-
partment syndrome and limb loss may occur. Although busy
trauma centers continue to debate this issue, the surgeon who
is obliged to treat the occasional injured patient may be bet-
ter served by performing CTA in selected patients with soft
signs. (See Schwartz 11th ed., Ch. 7, p. 206.)

TABLE 7-2 Signs and symptoms of peripheral arterial


injury
Hard Signs (Operation Soft Signs (Further Evaluation
Mandatory) Indicated)
Pulsatile hemorrhage Proximity to vasculature
Absent pulses Significant hematoma
Acute ischemia Associated nerve injury
A-A index of <0.9
Thrill or bruit
A-A index = systolic blood pressure on the injured side compared with that on the
uninjured side.

17. Which of the following is a cause of cardiogenic shock in Answer: C


a trauma patient? In trauma patients, the differential diagnosis of cardiogenic
A. Hemothorax shock consists of a short list: (1) tension pneumothorax,
B. Penetrating injury to the aorta (2) pericardial tamponade, (3) myocardial contusion or
C. Air embolism infarction, and (4) air embolism.
D. Iatrogenic increased afterload due to pressors Tension pneumothorax is the most frequent cause of car-
diac failure. Traumatic pericardial tamponade is most often
associated with penetrating injury to the heart. As blood
leaks out of the injured heart, it accumulates in the pericar-
dial sac. Because the pericardium is not acutely distensible,
the pressure in the pericardial sac rises to match that of the
injured chamber. Since this pressure is usually greater than
that of the right atrium, right atrial filling is impaired and
right ventricular preload is reduced. This leads to decreased
right ventricular output and increased central venous pres-
sure (CVP). Increased intrapericardial pressure also impedes
myocardial blood flow, which leads to subendocardial isch-
emia and a further reduction in cardiac output. This vicious
cycle may progress insidiously with injury of the vena cava
or atria, or precipitously with injury of either ventricle. With
acute tamponade, as little as 100 mL of blood within the peri-
cardial sac can produce life-threatening hemodynamic com-
promise. Patients usually present with a penetrating injury in
proximity to the heart, and they are hypotensive and have dis-
tended neck veins or an elevated CVP. The classic findings of
Beck’s triad (hypotension, distended neck, and muffled heart
sounds) and pulsus paradoxus are not reliable indicators of
acute tamponade. Ultrasonography (US) in the emergency
department (ED) using a subxiphoid or parasternal view is
extremely helpful if the findings are clearly positive (Fig. 7-1);

Brunicardi_Ch07_p049-064.indd 55 04/07/22 1:26 PM


56
however, equivocal findings are common. Early in the course
of tamponade, blood pressure (BP) and cardiac output will
CHAPTER 7

transiently improve with fluid administration. This may lead


the surgeon to question the diagnosis or be lulled into a false
sense of security. (See Schwartz 11th ed., Ch. 7, p. 193.)
Trauma

FIG. 7-1. Subxiphoid pericardial ultrasound reveals a large pericardial


fluid collection. LV = left ventricle; RV = right ventricle.

18. In which of the following scenarios is emergent tracheos- Answer: D


tomy indicated? Emergent tracheostomy is indicated in patients who have
A. Glasgow Coma Score (GCS) 6 and unable to protect laryngotracheal separation or in those with laryngeal frac-
airway tures because cricothyroidotomy may cause additional dam-
B. Complex maxillofacial trauma age or result in loss of the airway. The remaining scenarios
C. Chemical burn to the larynx are appropriately managed with orotracheal intubation or
D. Laryngeal fractures cricothyroidotomy if intubation is unsuccessful. Emergent
tracheostomy is most safely accomplished in the operating
room. If there is complete tracheal transection, a penetrating
towel clip should be placed on the distal segment to prevent
retraction into the mediastinum. (See Schwartz 11th ed.,
Ch. 7, p. 185.)

19. A 26-year old male patient presents to the emergency Answer: C


department (ED) after being involved in a rollover motor Tension pneumothorax is presumed in any patient manifest-
vehicle accident. He is awake and alert with GCS14. He is ing respiratory distress and hypotension in combination with
tachypneic with labored breathing and hypotensive with any of the following physical signs: tracheal deviation away
BP 80/palp. During primary survey you note distended from the affected side, lack of or decreased breath sounds
veins on the left neck and decreased breath sounds over on the affected side, and subcutaneous emphysema on the
the left chest. The best next step in this patient’s care is: affected side. Patients may have distended neck veins due to
A. Needle thoracostomy decompression with a 14-gauge impedance of venous return, but the neck veins may be flat
needle. due to concurrent systemic hypovolemia. Tension pneumo-
B. Obtain a chest X-ray and proceed to secondary thorax and simple pneumothorax have similar signs, symp-
survey. toms, and examination findings, but hypotension qualifies
C. Tube thoracostomy in the midaxillary line. the pneumothorax as a tension pneumothorax. Although
D. Orotracheal intubation. immediate needle thoracostomy decompression with a
14-gauge angiocatheter may be indicated in the field, tube
thoracostomy in the midaxillary line should be performed
immediately in the emergency department (ED) before a
chest radiograph is obtained (Fig. 7-2). In cases of tension
pneumothorax, the parenchymal tear in the lung acts as a
one-way valve, with each inhalation allowing additional air
to accumulate in the pleural space. The normally negative
intrapleural pressure becomes positive, which depresses the

Brunicardi_Ch07_p049-064.indd 56 04/07/22 1:26 PM


57
ipsilateral hemidiaphragm and shifts the mediastinal struc-
tures into the contralateral chest. Subsequently, the contra-

CHAPTER 7
lateral lung is compressed and the heart rotates about the
superior and inferior vena cava; this decreases venous return
and ultimately cardiac output, which culminates in cardiovas-
cular collapse. (See Schwartz 11th ed., Ch. 7, p. 186.)

Trauma
FIG. 7-2. A. Tube thoracostomy is performed in the midaxillary
line at the fourth or fifth intercostal space (inframammary
crease) to avoid iatrogenic injury to the liver or spleen. B. Heavy
scissors are used to cut through the intercostal muscle into the
pleural space. This is done on top of the rib to avoid injury to the
intercostal bundle located just beneath the rib. C. The incision is
digitally explored to confirm intrathoracic location and identify
pleural adhesions. D. A 28Fr chest tube is directed superiorly and
posteriorly with the aid of a large clamp.

20. Patients who are conscious, without tachypnea, and have Answer: D
a normal voice are unlikely to require air intervention. Patients who are conscious, without tachypnea, and have a
Additional findings that should prompt airway interven- normal voice are unlikely to require early airway interven-
tion include all of the following EXCEPT: tion. Exceptions are penetrating injuries to the neck with
A. Penetrating injury to the neck with expanding an expanding hematoma; evidence of chemical or thermal
hematoma. injury to the mouth, nares, or hypopharynx; extensive sub-
B. Evidence of chemical or thermal injury to mouth, cutaneous air in the neck; complex maxillofacial trauma; or
nares, or hypopharynx. airway bleeding. Although these patients may initially have
C. Extensive subcutaneous air in the neck. an adequate airway, it may become compromised if soft tissue
D. Right pneumohemothorax. swelling, hematoma formation, or edema progresses. In these
E. Airway bleeding. cases, preemptive intubation should be performed before air-
way access becomes challenging. Simple pneumohemotho-
rax is NOT an indication for airway intervention during
the primary survey.

Brunicardi_Ch07_p049-064.indd 57 04/07/22 1:26 PM


58
Patients who have an abnormal voice, abnormal breathing
sounds, tachypnea, or altered mental status require further
CHAPTER 7

airway evaluation. Blood, vomit, the tongue, teeth, foreign


objects, and soft tissue swelling can cause airway obstruction;
suctioning affords immediate relief in many patients. In the
comatose patient, the tongue may fall backward and obstruct
the hypopharynx; this can be relieved by either a chin lift or
jaw thrust. An oral airway or a nasal trumpet is also helpful
Trauma

in maintaining airway patency, although the former is usually


not tolerated by an awake patient. Establishing a definitive
airway (ie, endotracheal intubation) is indicated in patients
with apnea; inability to protect the airway due to altered
mental status; impending airway compromise due to inhala-
tion injury, hematoma, facial bleeding, soft tissue swelling,
or aspiration; and inability to maintain oxygenation. Altered
mental status is the most common indication for intubation.
Agitation or obtundation, often attributed to intoxication or
drug use, may actually be due to hypoxia. But the timing of
endotracheal intubation may be critical in the hypovolemic
patient because positive airway pressure may further com-
promise cardiac function and precipitate cardiac arrest; thus,
circulation may take priority over airway. (See Schwartz 11th
ed., Ch. 7, p. 184.)

21. Which of the following qualifies as pneumothorax as a Answer: C


tension pneumothorax? Tension pneumothorax is presumed in any patient manifest-
A. Distended neck veins ing respiratory distress and hypotension in combination with
B. Tracheal deviation any of the following physical signs: tracheal deviation away
C. Hypotension from the affected side, lack of or decreased breath sounds
D. Subcutaneous emphysema on the affected side, and subcutaneous emphysema on the
affected side. Patients may have distended neck veins due to
impedance of venous return, but the neck veins may be flat
due to concurrent systemic hypovolemia. Tension pneumo-
thorax and simple pneumothorax have similar signs, symp-
toms, and examination findings, but hypotension qualifies
the pneumothorax as a tension pneumothorax. Although
immediate needle thoracostomy decompression with a
14-gauge angiocatheter may be indicated in the field, tube
thoracostomy in the midaxillary line should be performed
immediately in the emergency department (ED) before
a chest radiograph is obtained. (See Schwartz 11th ed.,
Ch. 7, p. 186.)

22. Which of the following does NOT describe flail chest? Answer: A
A. Fracture of greater than at least three ribs on both Flail chest occurs when three or more contiguous ribs are
sides of the chest. fractured in at least two locations. Paradoxical movement
B. Three of more ribs fractured in at least two locations of this free-floating segment of chest wall is usually evident
C. Paradoxical movement of a free floating segment of in patients with spontaneous ventilation, due to the negative
chest wall intrapleural pressure of inspiration. The additional work of
D. High likelihood of associated pulmonary contusion breathing and chest wall pain caused by the flail segment is
rarely sufficient to compromise ventilation. Instead, it is the
decreased compliance and increased shunt fraction caused by
the associated pulmonary contusion that is the source of acute
respiratory failure. Pulmonary contusions often progress dur-
ing the first 12 hours. Resultant hypoventilation and hypox-
emia may require intubation and mechanical ventilation. The
patient’s initial chest radiograph often underestimates the
extent of the pulmonary parenchymal damage; close moni-
toring and frequent clinical reevaluation are warranted. (See
Schwartz 11th ed., Ch. 7, p. 186.)

Brunicardi_Ch07_p049-064.indd 58 04/07/22 1:26 PM


59
23. Which of the following describes a type I tracheobron- Answer: D
chial injury? Major air leak occurs from tracheobronchial injuries. Type I

CHAPTER 7
A. Occurring within 3 cm of the mainstem bronchus injuries are those occurring within 2 cm of the carina. These
B. Likely to cause a pneumothorax may not be associated with a pneumothorax due to the envel-
C. A distal bronchial injury opment in the mediastinal pleura. Type II injuries are more
D. Occurring within 2 cm of the carina distal injuries within the tracheobronchial tree and hence
manifest with a pneumothorax. Bronchoscopy confirms the
extent of the injury and its location, and directs management.

Trauma
(See Schwartz 11th ed., Ch. 7, p. 187.)

24. Which of the following is NOT one of the four injuries Answer: B
that should be identified during the “circulation” section During the circulation section of the primary survey, four life-
of the primary survey? threatening injuries must be identified promptly: (a) massive
A. Massive hemothorax hemothorax, (b) cardiac tamponade, (c) massive hemoperi-
B. Extremity vascular injury toneum, and (d) mechanically unstable pelvic fractures with
C. Cardiac tamponade bleeding. Critical tools used to differentiate these in the mul-
D. Massive hemoperitoneum tisystem trauma patient are the chest and pelvis radiographs,
and extended focused abdominal sonography for trauma
(eFAST). Identification of one of these injuries during the
primary surgery necessitates immediate intervention. (See
Schwartz 11th ed., Ch. 7, p. 188.)

25. For motor vehicle collisions, which of the following vari- Answer: A
ables are NOT among those strongly associated with life- Patients who have sustained blunt trauma can be separated
threatening injuries? into categories according to their risk for multiple injuries:
A. Head on impact those sustaining high-energy transfer injuries and those sus-
B. Death of another occupant in the vehicle taining low-energy transfer injuries. Injuries involving high-
C. Extrication time >20 minutes energy transfer include auto-pedestrian accidents, motor
D. Change in velocity of >20 MPH vehicle collisions in which the car’s change of velocity (ΔV)
E. Lack of restraint use exceeds 20 mph or in which the patient has been ejected,
motorcycle collisions, and falls from heights >20 ft. In fact,
for motor vehicle collisions the variables strongly associ-
ated with life-threatening injuries, and hence reflective
of the magnitude of the mechanism, are death of another
occupant in the vehicle, extrication time of >20 minutes,
ΔV >20 mph, lack of restraint use, and lateral impact. Low-
energy trauma, such as being struck with a club or falling from
a bicycle, usually does not result in widely distributed inju-
ries. However, potentially lethal injuries of internal organs can
occur because the net energy transfer to any given location
may be substantial. (See Schwartz 11th ed., Ch. 7, p. 196.)

26. When performing diagnostic peritoneal lavage (DPL) Answer: C


to rule out diaphragm injury, the appropriate laboratory Penetrating thoracoabdominal wounds may cause occult
cutoff values to use are: injury to the diaphragm. Patients with gunshot or stab
A. RBC >7500/mL, WBC >350/mL, Amylase >10 IU/L, wounds to the left lower chest should be evaluated with diag-
AP >1.5 IU/L, Bilirubin >0.01 mg/dL nostic laparoscopy or DPL to exclude diaphragmatic injury.
B. RBC >15000/mL, WBC >750/mL, Amylase >15 In general, penetrating right diaphragm injury is ignored
IU/L, AP >2.5 IU/L, Bilirubin >0.05 mg/dL unless there is a major underlying liver injury with a risk of
C. RBC >10000/mL, WBC >500/mL, Amylase >19 biliopleural fistula. Diagnostic laparoscopy may be preferred
IU/L, AP >2.0 IU/L, Bilirubin >0.01 mg/dL in patients with a positive chest radiograph (hemothorax or
D. RBC >12500/mL, WBC >1000/mL, Amylase >12 pneumothorax) or in those who would not tolerate a DPL. For
IU/L, AP >1.5 IU/L, Bilirubin >0.10 mg/dL patients undergoing DPL evaluation, laboratory value cutoffs
to rule out diaphragm injury are different from traditional
values formerly used for abdominal stab wounds (Table 7-3).
An RBC count of >10,000/μL is considered a positive find-
ing and an indication for abdominal evaluation; patients with
a DPL RBC count between 1000/μL and 10,000/μL should

Brunicardi_Ch07_p049-064.indd 59 04/07/22 1:26 PM


60
undergo laparoscopy or thoracoscopy. (See Schwartz 11th
ed., Ch. 7, p. 202.)
CHAPTER 7

TABLE 7-3 Criteria for “positive” finding on diagnostic


peritoneal lavage
Abdominal Thoracoabdominal
Trauma Stab Wounds
Red blood cell count >100,000/mL >10,000/mL
Trauma

White blood cell count >500/mL >500/mL


Amylase level >19 IU/L >19 IU/L
Alkaline phosphatase level >2 IU/L >2 IU/L
Bilirubin level >0.01 mg/dL >0.01 mg/dL

27. Focused assessment with sonography in trauma (FAST) Answer: B


examination is sensitive for detecting what volume of Blunt abdominal trauma is now evaluated initially by FAST
intraperitoneal fluid? examination, and this has supplanted diagnostic peritoneal
A. >100 cc lavage (DPL) (Fig. 7-3). FAST is not 100% sensitive, however,
B. >250 cc so diagnostic peritoneal aspiration is warranted in hemody-
C. >500 cc namically unstable patients without a defined source of blood
D. >1000 cc loss to rule out abdominal hemorrhage. FAST is used to iden-
tify free intraperitoneal fluid in Morrison’s pouch, the left
upper quadrant, and the pelvis. Although this method is sen-
sitive for detecting intraperitoneal fluid of >250 mL, it does
not reliably determine the source of hemorrhage nor grade
solid organ injuries. Patients with fluid on FAST examination,
considered a “positive FAST,” who do not have immediate
indications for laparotomy (hemodynamically stable, no evi-
dence of peritonitis) undergo CT scanning to quantify their
injuries. (See Schwartz 11th ed., Ch. 7, p. 202.)

Indications for CT:


Hemodynamically -Altered mental status Repeat FAST
No Peritonitis? No FAST + No No
stable -Confounding injury in 30 minutes
-Gross hematuria
Yes
-Pelvic fracture
No Yes -Abdominal tenderness
Candidate for
-Unexplained Hct <35%
nonoperative
management
FAST + Yes Laparotomy No
or Yes
patient with
Equivocal cirrhosis Yes Abdominal CT

DPA

FIG. 7-3. Algorithm for the initial evaluation of a patient with suspected blunt abdominal trauma. CT = computed
tomography; DPA = diagnostic peritoneal aspiration; FAST = focused abdominal sonography for trauma; Hct = hematocrit.

28. What approach is most appropriate for accessing the Answer: D


proximal left subclavian artery? A method advocated for access to the proximal left subcla-
A. Left anterolateral thoracotomy vian artery is through a fourth interspace anterolateral tho-
B. Left anterolateral thoracotomy with superior sternal racotomy, superior sternal extension, and left supraclavicular
extension and supraclavicular incision incision (“trap door” thoracotomy). Although the trap door
C. Median sternotomy procedure is appropriate after resuscitative thoracotomy, the
D. Median sternotomy with supraclavicular extension proximal left subclavian artery can be accessed more eas-
ily via a sternotomy with a supraclavicular extension. If

Brunicardi_Ch07_p049-064.indd 60 04/07/22 1:26 PM


61
the left subclavian artery is injured outside the thoracic out-
let, vascular control can be obtained via the sternotomy and

CHAPTER 7
definitive repair done through the supraclavicular incision.
Emergent median sternotomy is optimal for anterior stab
wounds to the heart. Typically, these patients have pericar-
dial tamponade and may undergo placement of a pericardial
drain before a semiurgent median sternotomy is performed.
Patients in extremis, however, should undergo anterolateral

Trauma
thoracotomy. (See Schwartz 11th ed., Ch. 7, p. 210.)

29. A left medial visceral rotation is appropriate for exposing Answer: A


injuries to all of the following EXCEPT: Supracolic injuries (aorta, celiac axis, proximal SMA, and left
A. Inferior vena cava (IVC). renal arteries) are best approached via a left medial visceral
B. Celiac axis. rotation. This is done by incising the lateral peritoneal reflec-
C. Proximal superior mesenteric artery (SMA). tion (white line of Toldt) beginning at the distal descending
D. Left renal artery. colon and extending the incision along the colonic splenic
flexure, around the posterior aspect of the spleen, and behind
the gastric fundus, ending at the esophagus. The left colon,
spleen, pancreas, and stomach are then rotated toward the
midline. The authors prefer to leave the kidney in situ when
mobilizing the viscera because this exaggerates the separation
of the renal vessels from the SMA. The operative approach
for SMA injuries is based on the level of injury. Fullen zone
I SMA injuries, located posterior to the pancreas, are best
exposed by a left medial visceral rotation. Fullen zone II SMA
injuries, extending from the pancreatic edge to the middle
colic branch, on the other hand, are approached via the lesser
sac along the inferior edge of the pancreas at the base of the
transverse mesocolon; the pancreatic body may be divided
to gain proximal vascular access. More distal SMA injuries,
Fullen zones III and IV, are approached directly within the
mesentery. A venous injury behind the pancreas, from the
junction of the superior mesenteric, splenic, and portal veins,
is accessed by dividing the neck of the pancreas. IVC injuries
are approached by a right medial visceral rotation. Proximal
control is obtained just above the iliac bifurcation with direct
pressure via a sponge stick; the injury is identified by cepha-
lad dissection along the anterior surface of the IVC. A Satin-
sky clamp can be used to control anterior caval wounds. (See
Schwartz 11th ed., Ch. 7, p. 212.)

30. Which of the following criteria should trigger angiogra- Answer: C


phy to control bleeding attributable to liver injury in a Nonoperative management of solid organ injuries is pursued
hemodynamically stable patient? in hemodynamically stable patients who do not have overt
A. Drop in Hgb of >2.0 g/dL peritonitis or other indications for laparotomy. Patients with
B. Alkaline phosphatase >250 IU/L >grade II injuries should be admitted to the surgical intensive
C. Transfusion of 4U RBC in 6 hours, or 6U over 24 hours care unit (SICU) with frequent hemodynamic monitoring,
D. Persistent right upper quadrant pain determination of hemoglobin, and abdominal examination.
The only absolute contraindication to nonoperative man-
agement is hemodynamic instability from intraperitoneal
hemorrhage. Factors such as high injury grade, large hemo-
peritoneum, contrast extravasation, or pseudoaneurysms may
predict complications or failure of nonoperative management.
Angioembolization and endoscopic retrograde cholangiopan-
creatography (ERCP) are useful adjuncts that can improve the
success rate of nonoperative management. The indication for
angiography to control hepatic hemorrhage is transfusion of
4 units of RBCs in 6 hours or 6 units of RBCs in 24 hours
attributable to the liver. (See Schwartz 11th ed., Ch. 7, p. 225.)

Brunicardi_Ch07_p049-064.indd 61 04/07/22 1:26 PM


62
31. What is the most commonly missed type of gastric Answer: D
injury? The most commonly missed gastric injury is the posterior
CHAPTER 7

A. Pre-pyloric injury wound of a through and through penetrating injury. Inju-


B. Lesser curve injury ries also can be overlooked if the wound is located within the
C. Injury at the gastroesophageal (GE) junction mesentery of the lesser curvature or high in the fundus. To
D. Posterior wound of a through and through penetrat- delineate a questionable injury, the stomach can be digitally
ing injury occluded at the pylorus while methylene blue-colored saline
is instilled via a nasogastric (NG) tube. Alternatively, air can
Trauma

be introduced via the NG tube with the abdomen filled with


saline. Little controversy exists regarding the repair of injuries
to the stomach or small bowel because of a rich blood supply.
Gastric wounds can be oversewn with a running single-layer
suture line or closed with a stapler. If a single-layer closure is
chosen, full-thickness bites should be taken to ensure hemosta-
sis from the well-vascularized gastric wall. Partial gastrectomy
may be required for destructive injuries, with resections of the
distal antrum or pylorus reconstructed using a Billroth proce-
dure. Patients with injuries that damage both Latarjet nerves
or vagi should undergo a drainage procedure. (See Schwartz
11th ed., Ch. 7, p. 228.)

32. During the initial phases of shock resuscitation, what is Answer: B


an appropriate goal hemoglobin level? The period of acute resuscitation, typically lasting for the first
A. 8.0 g/dL 12 to 24 hours after injury, combines several key principles:
B. 10 g/dL optimizing tissue perfusion, ensuring normothermia, and
C. 7 g/dL restoring coagulation status. There are a multitude of man-
D. 12 g/dL agement algorithms aimed at accomplishing these goals, the
majority of which involve goal-directed resuscitation with
initial volume loading to attain adequate preload, followed by
judicious use of inotropic agents or vasopressors. Although
the optimal hemoglobin level remains debated, during shock
resuscitation a hemoglobin level of >10 g/dL is generally
accepted to optimize hemostasis and ensure adequate oxygen
delivery. After the first 24 hours of resuscitation, a more judi-
cious transfusion trigger of a hemoglobin level of <7 g/dL in
the euvolemic patient limits the adverse inflammatory effects
of stored RBCs. (See Schwartz 11th ed., Ch. 7, p. 237.)

33. Which of the following is a physiologic change of preg- Answer: A


nancy that may influence evaluation of an injured preg- Pregnancy results in physiologic changes that may impact
nant patient? postinjury evaluation. Heart rate increases by 10 to 15 beats
A. Increased resting heart rate by 10 to 15 beats per per minute during the first trimester and remains elevated
minute until delivery. Blood pressure diminishes during the first 2 tri-
B. Increased resting blood pressure during the first mesters due to a decrease in systemic vascular resistance and
2 trimesters rises again slightly during the third trimester (mean values:
C. Decreased maternal intravascular volume first = 105/60, second = 102/55, third = 108/67). Intravascu-
D. Decreased tidal volume and minute ventilation lar volume is increased by up to 8 L, which results in a rela-
tive anemia but also a relative hypervolemia. Consequently,
a pregnant woman may lose 35% of her blood volume before
exhibiting signs of shock. Pregnant patients have an increase
in tidal volume and minute ventilation but a decreased func-
tional residual capacity; this results in a diminished PCO2 and
respiratory alkalosis. Also, pregnant patients may desaturate
more rapidly, particularly in the supine position and during
intubation. (See Schwartz 11th ed., Ch. 7, p. 240.)

Brunicardi_Ch07_p049-064.indd 62 04/07/22 1:26 PM


63
34. Which of the following is true regarding trauma in geri- Answer: B
atric patients? Mortality in patients with severe head injury more than dou-

CHAPTER 7
A. Admission Glasgow Coma Score (GCS) score after bles after the age of 55 years. Moreover, 25% of patients with
severe head injury is a good predictor of outcome. a normal GCS score of 15 had intracranial bleeding, with an
B. Rib fractures are associated with pulmonary contu- associated mortality of 50%. Just as there is no absolute age
sion in 35% of patients, and complicated by pneumo- that predicts outcome, admission GCS score is a poor predic-
nia in 10% to 30% of patients. tor of individual outcome. Therefore, the majority of trauma
C. Approximately 10% of patients older than 65 years centers advocate an initial aggressive approach with reevalu-

Trauma
will sustain a rib fracture from a fall <6 ft. ation at the 72-hour mark to determine subsequent care. Sec-
D. Chronologic age older than 65 years is associated ondly, one of the most common sequelae of blunt thoracic
with higher morbidity and mortality after trauma. trauma is rib fractures. In fact, in one study, 50% of patients
older than 65 years sustained rib fractures from a fall of <6 ft,
compared with only 1% of patients younger than 65 years.
Concurrent pulmonary contusion is noted in up to 35% of
patients, and pneumonia complicates the injuries in 10% to
30% of patients with rib fractures. (See Schwartz 11th ed.,
Ch. 7, p. 243.)

Brunicardi_Ch07_p049-064.indd 63 04/07/22 1:26 PM


This page intentionally left blank

Brunicardi_Ch07_p049-064.indd 64 04/07/22 1:26 PM


CHAPTER 8
Burns

1. A 22-year-old man is brought to the emergency room Answer: B


after a house fire. He has burns around his mouth and his Initial evaluation of the burned patient should follow the
voice is hoarse, but breathing is unlabored. What would same initial priorities of all trauma patients and involves four
be the most appropriate next step in management? crucial assessments: airway management, evaluation of other
A. Immediate endotracheal intubation injuries, estimation of burn size, and diagnosis of carboxy-
B. Examination of oral cavity and pharynx, with fiber- hemoglobin (CO) and cyanide poisoning. With direct ther-
optic laryngoscope if available mal injury to the upper airway or smoke inhalation, rapid
C. Place on supplemental oxygen and severe airway edema is potentially lethal. Anticipating
D. Placement of two large-bore IV catheters with fluid the need for intubation and establishing an early airway are
resuscitation critical. Signs of impending respiratory compromise include
a hoarse voice, wheezing, or stridor; subjective dyspnea is a
particularly concerning symptom and should trigger prompt
elective endotracheal intubation. Perioral burns and singed
nasal hairs alone do not indicate an upper airway injury,
but are signs that the oral cavity and pharynx should be fur-
ther evaluated for mucosal injury. Orotracheal intubation is
the preferred method for securing the airway. Nasotracheal
intubation may be useful for patients with associated facial
trauma when experienced providers are present, but it should
be avoided if oral intubation is safe and easy. (See Schwartz
11th ed., p. 251.)

2. What percentage burn does a patient have who has suf- Answer: C
fered partial-thickness burns to their anterior right leg Most burn resuscitation formulas estimate fluid require-
and anterior chest and abdomen as well as superficial ments based on burn size measured as a percentage of total
burns to their right arm? body surface area (TBSA) (%TBSA). The “rule of nines” is a
A. 18% crude but quick and effective method of estimating burn size
B. 27% (Fig. 8-1). In adults, the anterior and posterior trunk each
C. 36% account for 18%, each lower extremity is 18%, each upper
D. 45% extremity is 9%, and the head is 9%. In children under 3 years
of age, the head accounts for a larger relative surface area and
should be taken into account when estimating burn size. For
smaller or odd-shaped burns, the “rule of the palm” where the
palmar surface of the hand, including the digits, is 1% TBSA
is useful. Diagrams such as the Lund and Browder chart give
a more accurate accounting of the true burn size in children
and adults. The importance of an accurate burn size assess-
ment cannot be overemphasized. Superficial or first-degree
burns should not be included when calculating burn size, and
thorough cleaning of soot and debris is mandatory to avoid
confusing soiled skin with burns. Examination of referral

65

Brunicardi_Ch08_p065-072.indd 65 30/06/22 10:39 AM


66
data suggests that physicians inexperienced with burns tend
to overestimate the size of small burns and underestimate the
CHAPTER 8

size of large burns, with potentially detrimental effects on


pretransfer resuscitation. (See Schwartz 11th ed., p. 253.)

4.5% Front 4.5% Back


Burns

9% 9% 9%
18%

1%
1%

18% 18% 18%

FIG. 8-1. The “rule of nines” can be used as a quick reference for estimating a patient’s burn size by dividing the
body into regions to which total body surface area is allocated in multiples of 9.

3. A 40-year-old woman is admitted to the burn unit after Answer: B


an industrial fire at a plastics manufacturing plant with Hydrogen cyanide toxicity may also be a component of an
burns to the face and arms. Her electrocardiogram overwhelming smoke inhalation injury. Cyanide inhibits cyto-
(ECG) shows S-T elevation, and initial chemistry panel chrome oxidase, which is required for oxidative phosphory-
and arterial blood gas reveal an anion-gap metabolic aci- lation. Afflicted patients may have a persistent, severe lactic
dosis with normal arterial carboxyhemoglobin. What is acidosis, neurologic symptoms, pulmonary edema, or cardiac
the most appropriate next step? sequelae (ST elevation on electrocardiogram). Classic signs
A. Correction of acidosis by adding sodium bicarbonate of cyanide poisoning—including bitter almond breath and
to IV fluids cherry-red skin changes—are rare and should not be used as
B. Administration of 100% oxygen and hydroxocobalamin the sole diagnostic criteria. Treatment consists of sodium thio-
C. Transthoracic echocardiogram sulfate, hydroxocobalamin, and 100% oxygen. Sodium thiosul-
D. Blood culture with intravenous antibiotics fate works as a substrate for the metabolism of cyanide into a
nontoxic derivative, but it works slowly and is not effective for
acute therapy. Hydroxocobalamin—a vitamin B12 precursor—
quickly complexes with cyanide, is excreted by the kidney,
and is recommended for immediate therapy. In the major-
ity of patients, lactic acidosis will resolve with ventilation,
and sodium thiosulfate treatment becomes unnecessary. (See
Schwartz 11th ed., p. 256.)

Brunicardi_Ch08_p065-072.indd 66 30/06/22 10:39 AM


67
4. In order to assess the special concerns associated with Answer: A
electrical burns: Electrical burns make up 3% of US hospital admissions but

CHAPTER 8
A. All extremity compartments should be evaluated and have special concerns, including cardiac arrhythmia and com-
a baseline ECG should be obtained. partment syndrome with concurrent rhabdomyolysis. A base-
B. A full neurologic assessment including ophthal- line ECG is recommended in all patients with an electrical
mologic is required to evaluate acute intraocular injury, and a normal ECG in a low-voltage injury (<1000 V)
pathology. may preclude hospital admission. Because compartment syn-
C. A Foley catheter should be placed to allow for titra- drome and rhabdomyolysis are common in high-voltage elec-

Burns
tion of fluid administration. trical injuries, vigilance must be maintained for neurologic or
D. Workup should include an echocardiogram to evalu- vascular compromise, and fasciotomies should be performed
ate ventricular wall motion. even in cases of moderate clinical suspicion. For patients with
rhabdomyolysis, a Foley catheter and monitoring of elec-
trolytes may be aid in renal function monitoring, but fluid
administration and urine output is not prioritized as highly in
electrical when compared to external burns. Long-term neuro-
logic symptoms and cataract development are not uncommon
with high-voltage electrical injuries, and neurologic and oph-
thalmologic consultation should be obtained to define base-
line patient function. However, acute intraocular pathology is
unlikely to be uncovered. (See Schwartz 11th ed., p. 252.)

5. What is the zone of coagulation and what is the appropri- Answer: A


ate treatment? The zone of coagulation is the most severely burned por-
A. The most severely burned portion is typically in the tion and is typically in the center of the wound. As the name
center of the wound, which will require excision and implies, the affected tissue is coagulated and sometimes
grafting. frankly necrotic, much like a full-thickness burn, and will
B. The most severely burned portion is typically in the need excision and grafting. Peripheral to that is a zone of
center of the wound, which will likely heal without stasis, with variable degrees of vasoconstriction and resul-
surgical intervention if adequate perfusion is main- tant ischemia, much like a second-degree burn. Appropri-
tained and infection is prevented. ate resuscitation and wound care may prevent conversion to
C. It has variable degrees of vasoconstriction and resul- a deeper wound, but infection or suboptimal perfusion may
tant ischemia, which will require excision and grafting. result in an increase in burn depth. This is clinically relevant
D. It is hyperemic and extremely painful to touch, but because many superficial partial-thickness burns will heal
will not require surgical excision or grafting. with nonoperative management, and the majority of deep
partial-thickness burns benefit from excision and skin graft-
ing. The outermost area of a burn is called the zone of hyper-
emia, which will heal with minimal or no scarring and is most
like a superficial partial-thickness burn or first-degree burn.
(See Schwartz 11th ed., p. 253.)

6. Which of the following describes a partial-thickness, Answer: C


second-degree burn? Based on the original burn depth classification by Dupuytren
A. Leathery, painless, and nonblanching in 1832, burn wounds are commonly classified as superficial
B. Painful but do not blister (first-degree), partial-thickness (second-degree), full-thickness
C. Dermal involvement and are extremely painful with (third-degree), and fourth-degree burns, which affect under-
weeping and blisters lying soft tissue. Fifth-degree burns (through muscle to bone)
D. Will need excision and grafting and sixth-degree burns (charring bone) were also described
although are less common. Partial-thickness burns are clas-
sified as either superficial or deep partial-thickness burns by
depth of involved dermis. Clinically, first-degree burns are
painful but do not blister, second-degree burns have dermal
involvement and are extremely painful with weeping and
blisters, and third-degree burns are leathery, painless, and
nonblanching. Jackson described three zones of tissue injury
following burn injury. (See Schwartz 11th ed., p. 253.)

Brunicardi_Ch08_p065-072.indd 67 30/06/22 10:39 AM


68
7. What is the fluid requirement of a 50-kg man with first- Answer: A
degree burns to his left arm and leg, circumferential The most commonly used formula, the Parkland or Baxter
CHAPTER 8

second-degree burn to his right arm, and third-degree formula, consists of 3 to 4 mL/kg per % burn of Lactated
burns to his torso and right leg. What is the rate of initial Ringer’s, of which half is given during the first 8 hours after
fluid resuscitation? burn and the remaining half is given over the subsequent
A. 4.5 L over 8 hours, followed by 4.5 L over 16 hours 16 hours. The most recent American Burn Association con-
B. 4.5 L over 8 hours, followed by 6 L over 16 hours sensus formula recommends 2 mL/kg per % burn of Lactated
C. 6 L over 8 hours, followed by 6 L over 16 hours Ringers given the tendency toward excessive fluid adminis-
Burns

D. 6 L over 8 hours, followed by 9 L over 16 hours tration with the traditional formulas. The concept behind
continuous fluid requirements is simple. The burn (and/
or inhalation injury) drives an inflammatory response that
leads to capillary leak; as plasma leaks into the extravascular
space, crystalloid administration maintains the intravascular
volume. Therefore, if a patient receives a large fluid bolus in
a prehospital setting or emergency department, the fluid has
likely leaked into the interstitium, and the patient still requires
ongoing burn resuscitation according to the estimates. Con-
tinuation of fluid volumes should depend on the time since
injury, urine output, and mean arterial pressure (MAP). As
the capillary leak closes, the patient will require less volume to
maintain these two resuscitation endpoints. Children under
20 kg have the additional requirement that they do not have
sufficient glycogen stores to maintain an adequate glucose
level in response to the inflammatory response. Specific pedi-
atric formulas have been described, but the simplest approach
is to deliver a weight-based maintenance IV fluid with glu-
cose supplementation in addition to the calculated resuscita-
tion with lactated Ringer’s. (See Schwartz 11th ed., p. 254.)

8. A patient with partial- and full-thickness burns to their Answer: D


torso was intubated emergently and has become increas- Hypoventilation, increased airway pressures, and hypoten-
ingly difficult to ventilate demonstrating rising PCO2 sion may also characterize thoracic compartment syndrome.
and peak inspiratory pressure. The most important treat- Escharotomies are rarely needed within the first 8 hours fol-
ment includes: lowing injury and should not be performed unless indicated
A. Low tidal volume (6 cc/kg) for lung-protection because of the terrible aesthetic sequelae. When indicated,
ventilation. they are usually performed at the bedside, preferably with
B. High-frequency percussive ventilation (HFPV). electrocautery to minimize blood loss. Extremity incisions
C. Increase the FIO2 and decrease in positive end- are made on the lateral and medial aspects of the limbs in an
expiratory pressure (PEEP). anatomic position and may extend onto thenar and hypothe-
D. Eschar release along the anterior axillary lines with nar eminences of the hand. Digital escharotomies do not usu-
bilateral subcostal and subclavicular extensions. ally result in any meaningful salvage of functional tissue and
are not recommended. Inadequate perfusion despite proper
escharotomies may indicate the need for fasciotomy, but this
procedure should not be routinely performed as part of the
eschar release. Thoracic escharotomies should be placed along
the anterior axillary lines with bilateral subcostal and subcla-
vicular extensions. Extension of the anterior axillary incisions
down the lateral abdomen typically will allow adequate release
of abdominal eschar. (See Schwartz 11th ed., p. 259.)

9. What is the indication for the topical therapy which can Answer: A
cause neutropenia when applied to burns? Silver sulfadiazine is one of the most widely used in clinical
A. Primarily as prophylaxis against burn wound infec- practice. Silver sulfadiazine has a wide range of antimicrobial
tions with a wide range of antimicrobial activity activity, primarily as prophylaxis against burn wound infec-
B. Primarily as antimicrobial prophylaxis against eschar tions rather than treatment of existing infections. It has the
and newly grafted areas added benefits of being inexpensive, being easily applied,
C. MRSA culture-positive burn wound infections and having soothing qualities. It is not significantly absorbed
D. To improve patient comfort while reducing the need systemically and thus has minimal metabolic derangements.
for daily dressing changes Silver sulfadiazine has a reputation for causing neutropenia,

Brunicardi_Ch08_p065-072.indd 68 30/06/22 10:39 AM


69
but this association is more likely due to neutrophil margin-
ation from the inflammatory response following burn injury.

CHAPTER 8
True allergic reactions to the sulfa component of silver sul-
fadiazine are rare, and at-risk patients can have a small test
patch applied to identify a burning sensation or rash. Silver
sulfadiazine destroys skin grafts and is contraindicated on
burns or donor sites in proximity to newly grafted areas. Also,
silver sulfadiazine may retard epithelial migration in healing

Burns
partial-thickness wounds. (See Schwartz 11th ed., p. 257.)

10. Successful antibiotic penetration of a burn eschar can be Answer: A


achieved with: Mafenide acetate, either in cream or solution form, is an
A. Mafenide acetate. effective topical antimicrobial. It is effective even in the pres-
B. Neomycin. ence of eschar and can be used in both treating and prevent-
C. Silver nitrate. ing wound infections; the solution formulation is an excellent
D. Silver sulfadiazine. antimicrobial for fresh skin grafts. Use of mafenide acetate
may be limited by pain with application to partial-thickness
burns. As mafenide is a carbonic anhydrase inhibitor, a his-
torically described side effect is metabolic acidosis. However,
multiple studies have been performed using mafenide to treat
burn wounds without any significant incidence of metabolic
acidosis. (See Schwartz 11th ed., p. 257.)

11. Which of the following is TRUE regarding nutritional Answer: D


needs of burn patients? Calculating the appropriate caloric needs of the burn patient
A. The hypermetabolic response to burn wounds typi- can be challenging. A commonly used formula in nonburned
cally raises the basic metabolic rate by 120%. patients is the Harris-Benedict equation, which calculates
B. Oxandrolone, an anabolic steroid, can improve lean caloric needs using factors such as gender, age, height, and
body mass but can be associated with hyperglycemia weight. This formula uses an activity factor for specific inju-
and clinically significant rise in hepatic transaminitis. ries, and for burns, the basal energy expenditure is multi-
C. Early enteral feeding is safe when burns are <20% plied by 2. The Harris-Benedict equation may be inaccurate
total body surface area (TBSA), otherwise enteral in burns of <40% TBSA, and in these patients, the Curreri
feeding should await return of bowel function to formula may be more appropriate. This formula estimates
avoid feeding a patient with gastric ileus. caloric needs to be 25 kcal/kg/d plus 40 kcal/%TBSA/d. Indi-
D. For patients with <40% TBSA, caloric needs are esti- rect calorimetry can also be used to calculate resting energy
mated to be 25 kcal/kg/d plus 40 kcal/%TBSA/d. expenditure, but in burn patients, a “metabolic cart” has not
been documented to be more beneficial than the predic-
tive equations. Titrating caloric needs closely is important
because overfeeding patients will lead to storage of fat instead
of muscle anabolism. (See Schwartz 11th ed., p. 258.)

12. A 42-year-old man with burns of >40% TBSA was Answer: D


required a high volume of intravenous fluids over the Massive resuscitation of burned patients may lead to an
first 3 days of his ICU admission to maintain his urine abdominal compartment syndrome characterized by increased
output, which had finally stabilized. However, he has airway pressures with hypoventilation and decreased urine
become increasingly difficult to ventilate, with high peak output and hemodynamic compromise. Decompressive lapa-
pressures, and his urine output is now declining with- rotomy is the standard of care for refractory abdominal com-
out response to additional crystalloid. What treatment is partment syndrome but carries an especially poor prognosis in
now required for this patient? burn patients. Adjunctive measures such as minimizing fluid,
A. Discontinuation of crystalloid administration, favor- performing torso escharotomies, decreasing tidal volumes,
ing vasopressor initiation for hemodynamic support, and chemical paralysis should be initiated before resorting
if required to decompressive laparotomy. Patients undergoing massive
B. Transitioning from crystalloid to colloid resuscitation also develop elevated intraocular pressures
C. Torso escharotomies and may require lateral canthotomy. (See Schwartz 11th ed.,
D. Decompressive laparotomy p. 258.)

Brunicardi_Ch08_p065-072.indd 69 30/06/22 10:39 AM


70
13. Which of the following statements is FALSE concerning Answer: B
surgical treatment of burn wounds? Once the initial resuscitation is complete and the patient is
CHAPTER 8

A. Tangential excision consists of tangential slices of hemodynamically stable, attention should be turned to excis-
burn tissue until bleeding tissue is encountered. ing the burn wound. Burn excision and wound coverage
Thus, excision can be associated with potentially sig- should ideally start within the first several days, and in larger
nificant blood loss. burns, serial excisions can be performed as patient condition
B. Xenographs are a permanent alternative to split allows. Excision is performed with repeated tangential slices
thickness skin grafts when there is insufficient donor using a Watson or Goulian blade until viable, diffusely bleed-
Burns

sites. ing tissue remains. It is appropriate to leave healthy dermis,


C. Bleeding from tangential excision can be helped with which will appear white with punctate areas of bleeding. Exci-
injection of epinephrine tumescence solution, pneu- sion to fat or fascia may be necessary in deeper burns. The
matic tourniquets, epinephrine-soaked compresses, downside of tangential excision is a high blood loss, though
and fibrinogen and thrombin spray sealant. this may be ameliorated using techniques such as instilla-
D. Meshed split-thickness skin grafts allow serosan- tion of an epinephrine tumescence solution underneath the
guinous drainage to prevent graft loss and provide a burn. Pneumatic tourniquets are helpful in extremity burns,
greater area of wound coverage. and compresses soaked in a dilute epinephrine solution are
necessary adjuncts after excision. A fibrinogen and thrombin
spray sealant (Tisseel Fibrin Sealant; Baxter, Deerfield, IL) also
has beneficial effects on both hemostasis and graft adherence
to the wound bed. The use of these techniques has markedly
decreased the number of blood transfusions given during burn
surgery. For patients with clearly deep burns and concern for
excessive blood loss, fascial excision may be employed. In this
technique, electrocautery is used to excise the burned tissue
and the underlying subcutaneous tissue down to muscle fas-
cia. This technique markedly decreases blood loss but results
in a cosmetically inferior appearance due to the loss of sub-
cutaneous tissue. For excision of burns in difficult anatomic
areas, such as the face, eyelids, or hands, a pressurized water
dissector may offer more precision but is time-consuming,
has a steep learning curve, and is expensive. (See Schwartz
11th ed., p. 259.)

14. Which of the following late complications of an acute Answer: C


burn has the appropriately described treatment? HO is another long-term morbidity associated with burn
A. Hypertrophic burn scars should be excised and injury. HO is the pathologic development of lamellar bone in
revised to minimize the increasing inflammatory peripheral tissue. Its incidence has been reported to be
response. between 1% and 3% of burn patients. Symptoms include
B. The most common type of contracture is that of the decreased range of motion, pain, and swelling overlying
hand which rarely requires surgical excision. the affected joints. Oftentimes, the pathologic bone formation
C. Heterotopic ossification (HO) associated pain can can be visualized radiographically with plain X-rays. Risk fac-
be often successfully treated with oral pain control, tors include >30% total body surface area (TBSA), arm burns,
physiotherapy, and radiation therapy. arm grafts, ventilator days, and number of trips to the oper-
D. HO requires surgical excision to prevent malignant ating room. Treatment includes aggressive physiotherapy,
transformation of pathologic lamellar bone. nonsteroidal anti-inflammatory drugs (NSAIDs), bisphos-
phonates, radiation therapy, and rarely surgical excision.
A risk scoring system has been developed to predict which
burn patients are at risk of developing HO based on admis-
sion criteria; however, further validation is warranted. (See
Schwartz 11th ed., p. 261.)

Brunicardi_Ch08_p065-072.indd 70 30/06/22 10:39 AM


71
15. Which of the following descriptions of a burn disaster is Answer: C
TRUE? Although rare, burn disasters can be devastating to those

CHAPTER 8
A. Do not include exposure to radioactive materials. involved due to the sudden nature of the event, the difficulty
B. Should be coordinated by local hospital systems, of managing personnel and resources, a deficit of staff expe-
regionalization and national plans inappropriately rience in burn management, and relatively small resource
allocate resources. availability for a potentially large number of patients. The
C. Thirty percent of patients in mass casualty incidents American Burn Association has estimated that up to 30% of
suffer from burn injury. patients in mass casualty incidents suffer from burn injury.

Burns
D. Globally, they occur most commonly in the United A recent review of the literature between 1990 and 2016 iden-
States. tified 752 burn disasters worldwide, defined as an incident
with ≥50 burn injuries and/or ≥30 burn-related deaths. The
majority occurred in Asia and the Middle East and are thought
to be secondary to rapid industrialization, inadequate fire-
prevention strategies, and poor building codes. There was a
significant increase in terrorist-related incidences from 2000
to 2015. Finally, the authors demonstrated that international
adoption of the US Health and Human Services guidelines
on bed availability for burns and trauma dramatically under-
estimated the number of beds needed for burn disasters.
(See Schwartz 11th ed., p. 261.)

Brunicardi_Ch08_p065-072.indd 71 30/06/22 10:39 AM


This page intentionally left blank

Brunicardi_Ch08_p065-072.indd 72 30/06/22 10:39 AM


CHAPTER 9
Wound Healing

1. Normal wound healing is divided into phases defined Answer: A


by characteristic cellular populations and biochemical Normal wound healing is divided into phases defined by
interactions. They are: characteristic cellular populations and biochemical activi-
A. Hemostasis and inflammation, proliferation, matura- ties: (a) hemostasis and inflammation, (b) proliferation, and
tion, and remodeling. (c) maturation and remodeling. An approximate timeline of
B. Hemostasis, proliferation and inflammation, matrix these events is depicted in Fig. 9-1. (See Schwartz 11th ed.,
deposition, maturation, and remodeling. p. 273.)
C. Hemostasis and inflammation, platelet aggregation,
maturation and remodeling. Phases of healing
D. Hemostasis, inflammation, remodeling and Maturation
maturation.
Proliferation

Inflammation

0 2 4 6 8 10 12 14 16 months
Relative number of cells

Neutrophils

Macrophages

Fibroblasts

Lymphocytes
0 2 4 6 8 10 12 14 16
Relative amount of
matrix synthesis

Collagen I

Fibronectin

Collagen III

Wound-breaking
strength
FIG. 9-1. The cellular, biochemical, and 0 2 4 6 8 10 12 14 16
mechanical phases of wound healing. Days postwounding

73

Brunicardi_Ch09_p073-082.indd 73 06/07/22 9:36 AM


74
2. Which of the following is FALSE regarding polymor- Answer: E
phonuclear neutrophils (PMNs) and their role in wound PMNs are the first infiltrating cells to enter the wound site,
CHAPTER 9

healing? peaking at 24 to 48 hours. Increased vascular permeability,


A. PMNs release proteases that degrade ground sub- local prostaglandin release, and the presence of chemotactic
stance within the wound site. substances such as complement factors, interleukin-1 (IL-1),
B. Neurophils use fibrin clot generated at the wound as tumor necrosis factor-alpha (TNF-α), TGF- β, platelet factor 4,
scaffolding for migration into the wound. or bacterial products all stimulate neutrophil migration.
C. Neutrophil migration is stimulated by local prosta- The postulated primary role of neutrophils is phagocytosis
Wound Healing

glandins, complement factors, IL-1, TNFα, GF-β, of bacteria and tissue debris. PMNs are also a major source
platelet factor 4, or bacterial products. of cytokines early during inflammation, especially TNF-α,
D. PMNs are the first cells to infiltrate the wound, peak- which may have a significant influence on subsequent angio-
ing at 24 to 48 hours. genesis and collagen synthesis. PMNs also release proteases
E. Neutrophils release cytokines that later assist with such as collagenases, which participate in matrix and ground
collagen deposition and epithelial closure. substance degradation in the early phase of wound healing.
Other than their role in limiting infections, these cells do not
appear to play a role in collagen deposition or acquisition of
mechanical wound strength. On the contrary, neutrophil fac-
tors have been implicated in delaying the epithelial closure of
wounds. (See Schwartz 11th ed., p. 273.)

3. The proliferative phase of wound healing occurs how Answer: C


long after the injury? Normal wound healing follows a predictable pattern that
A. 1 day can be divided into overlapping phases defined by the cel-
B. 2 days lular populations and biochemical activities: (a) hemostasis
C. 7 days and inflammation, (b) proliferation, and (c) maturation and
D. 14 days remodeling.
The proliferative phase is the second phase of wound heal-
ing and roughly spans days 4 through 12. It is during this phase
that tissue continuity is reestablished. Fibroblasts and endo-
thelial cells are the last cell populations to infiltrate the heal-
ing wound, and the strongest chemotactic factor for fibroblasts
is platelet-derived growth factor (PDGF). Upon entering the
wound environment, recruited fibroblasts first need to prolif-
erate, and then become activated, to carry out their primary
function of matrix synthesis remodeling. This activation is
mediated mainly by the cytokines and growth factors released
from wound macrophages. (See Schwartz 11th ed., p. 273.)

4. Which of the following is TRUE regarding the fibroplas- Answer: C


tic phase of wound healing? The maturation and remodeling of the scar begins during the
A. Early during wound healing, the predominant com- fibroplastic phase, and is characterized by a reorganization of
posititon of the matrix is fibronectin and type II previously synthesized collagen. Collagen is broken down by
collagen. matrix metalloproteinases (MMPs), and the net wound col-
B. After complete replacement of the scar with type lagen content is the result of a balance between collagenolysis
III collagen, the mechanical strength will equal that and collagen synthesis. There is a net shift toward collagen
of uninjured tissue approximately 6 to 12 months synthesis and eventually the reestablishment of extracellular
postinjury. matrix composed of a relatively acellular collagen-rich scar.
C. Even though the tensile strength of a wound reaches Wound strength and mechanical integrity in the fresh
a plateau after several weeks, the tensile strength will wound are determined by both the quantity and quality of
increase over another 6 to 12 months due to fibril for- the newly deposited collagen. The deposition of matrix at the
mation and cross-linking. wound site follows a characteristic pattern: fibronectin and
D. As the scar matures, matrix metalloproteinases break collagen type III constitute the early matrix scaffolding; gly-
down type I collagen and replace it with type III cosaminoglycans and proteoglycans represent the next sig-
collagen. nificant matrix components; and collagen type I is the final
matrix. By several weeks postinjury the amount of collagen
in the wound reaches a plateau, but the tensile strength con-
tinues to increase for several more months. Fibril formation
and fibril cross-linking result in decreased collagen solubil-
ity, increased strength, and increased resistance to enzymatic

Brunicardi_Ch09_p073-082.indd 74 06/07/22 9:36 AM


75
degradation of the collagen matrix. Fibrillin, a glycoprotein
secreted by fibroblasts, is essential for the formation of elastic

CHAPTER 9
fibers found in connective tissue. Scar remodeling continues
for many (6 to 12) months postinjury, gradually resulting in a
mature, avascular, and acellular scar. The mechanical strength
of the scar never achieves that of the uninjured tissue. (See
Schwartz 11th ed., p. 275.)

Wound Healing
5. The most common causes of Ehler-Danlos syndrome
are: Ehlers-Danlos syndrome (EDS) is a group of 10 disorders
A. An acquired collagen deficit. that present as a defect in collagen formation. Over half of the
B. Genetic defects encoding α-chains of collagen type V. affected patients manifest genetic defects encoding α-chains
C. Genetic defect in platelet-derived growth factor of collagen type V, causing it to be either quantitatively or
(PDGF). structurally defective. (See Schwartz 11th ed., p. 278.)
D. Type I and type III collagen genetic defect.

6. Dermal wounds in patients with Ehlers-Danlos syn- Answer: B


drome (EDS) should be: Closing wounds in patients with EDS might represent a major
A. Closed in one layer with sutures removed early to challenge to the surgeon. Dermal wounds should be closed in
prevent deformity. two layers, approximated with the sutures under tension, and
B. Closed in two layers, under tension, with sutures the stitches should be left in place twice as long as usual. In
removed after twice as long time. addition, external fixation with adhesive tape can help rein-
C. Allowed to heal by secondary intention, as would force the scar and prevent stretching. (See Schwartz 11th ed.,
commonly fall apart. p. 278.)
D. There is no benefit to external fixation with adhesive
tapes.

7. Patients with Marfan syndrome are associated with what Answer: D


genetic defect? Patients with Marfan syndrome have tall stature, arachnodac-
A. MFN-1 gene deletion tyly, lax ligaments, myopia, scoliosis, pectus excavatum, and
B. Type I collagen gene mutation aneurysm of the ascending aorta. Patients who suffer from
C. COL7A1 gene mutation this syndrome also are prone to hernias. Surgical repair of a
D. FBN-1 gene mutation dissecting aneurysm is difficult, as the soft connective tissue
fails to hold sutures. Skin may be hyperextensible, but shows
no delay in wound healing.
The genetic defect associated with Marfan syndrome is a
mutation in the FBN-1 gene which encodes for fibrillin. Pre-
viously, it was thought that structural alteration of the micro-
fibrillar system was responsible for the phenotypic changes
seen with the disease. However, recent research indicates an
intricate relationship that FBN1 gene products play in TGF-β
signaling. (See Schwartz 11th ed., p. 278.)

8. Which of the following is FALSE regarding healing of Answer: D


full-thickness injuries of the gastrointestinal tract? The submucosa lies radially and circumferentially outside
A. Serosal healing is essential to form a water-tight bar- of these layers, is comprised of abundant collagenous and
rier to the lumen of the bowel. elastic fibers, and supports neural and vascular structures.
B. There is an early decrease in marginal strength due The submucosa is the layer that imparts the greatest tensile
to an imbalance of greater collagenolysis versus col- strength and greatest suture-holding capacity, a characteris-
lagen synthesis. tic that should be kept in mind during surgical repair of the
C. Collagen synthesis is done by fibroblast and smooth GI tract. Additionally, serosal healing is essential for quickly
muscle cells. achieving a watertight seal from the luminal side of the bowel.
D. The greatest tensile strength of the gastrointestinal The importance of the serosa is underscored by the signifi-
(GI) tract is provided by the serosa. cantly higher rates of anastomotic failure observed clinically
in segments of bowel that are extraperitoneal and lack serosa
(ie, the esophagus and rectum).
The early integrity of the anastomosis is dependent on
formation of a fibrin seal on the serosal side, which achieves
watertightness, and on the suture-holding capacity of the

Brunicardi_Ch09_p073-082.indd 75 06/07/22 9:36 AM


76
intestinal wall, particularly the submucosal layer. There is a
significant decrease in marginal strength during the first week
CHAPTER 9

due to an early and marked collagenolysis. The lysis of colla-


gen is carried out by collagenase derived from neutrophils,
macrophages, and intraluminal bacteria. Collagenase activity
occurs early in the healing process, and during the first 3 to
5 days collagen breakdown far exceeds collagen synthe-
sis. The integrity of the anastomosis represents equilibrium
Wound Healing

between collagen lysis, which occurs early, and collagen syn-


thesis, which takes a few days to initiate. Collagen synthesis in
the gastrointestinal tract is carried out by both fibroblasts and
smooth muscle cells. (See Schwartz 11th ed., p. 279.)

9. When comparing wound healing of the gastrointestinal Answer: B


(GI) tract to wound healing of skin, which of the follow- (See Table 9-1. Schwartz 11th ed., p. 280.)
ing is TRUE?
A. Wound strength is rapidly recovered in both skin and
GI tract.
B. Collagenase activity is increased throughout the GI
tract after transection and anastomoses.
C. Steroids have a greater impact on wound healing in
GI tract compared to skin.
D. d-Penicillamine has no impact on skin wound healing.

TABLE 9-1 Comparison of wound healing in the gastrointestinal tract and skin
GI Tract Skin
Wound environment pH Varies throughout GI tract in accordance with Usually constant except during sepsis or local
local exocrine secretions infection
Microorganisms Aerobic and anaerobic, especially in the colon Skin commensals rarely cause problems;
and rectum; problematic if they contaminate infection usually results from exogenous
the peritoneal cavity contamination or hematogenous spread
Shear stress Intraluminal bulk transit and peristalsis exert Skeletal movements may stress the suture
distracting forces on the anastomosis line but pain usually acts as a protective
mechanism preventing excess movement
Tissue oxygenation Dependent on intact vascular supply and Circulatory transport of oxygen as well as
neocapillary formation diffusion
Collagen synthesis Cell type Fibroblasts and smooth muscle cells Fibroblasts
Lathyrogens d-Penicillamine has no effect on collagen Significant inhibition of cross-linking with
cross-linking decreased wound strength
Steroids Contradictory evidence exists concerning Significant decrease in collagen accumulation
their negative effect on GI healing; increased
abscess in the anastomotic line may play a
significant role
Collagenase activity — Increased presence throughout GI tract after Not as significant a role in cutaneous wounds
transection and reanastomosis; during
sepsis, excess enzyme may promote
dehiscence by decreasing suture-holding
capacity of tissue
Wound strength — Rapid recovery to preoperative level. Less rapid than GI tissue
Scar formation Age Definite scarring seen in fetal wound sites Usually heals without scar formation in the
fetus

Brunicardi_Ch09_p073-082.indd 76 06/07/22 9:36 AM


77
10. Which of the following is TRUE about bone healing? Answer: A
A. Erythema and swelling commonly occur as part Following any type of injury to bone, several changes take place

CHAPTER 9
of healing, as it occurs as the bone at the frac- at the site of injury to restore structural and functional integ-
ture site is degraded and normal bone undergoes rity. Most of the phases of healing resemble those observed in
revascularization. dermal healing, but some notable individual characteristics
B. Erythema and swelling occur during formation of apply to bone injuries. The initial stage of hematoma formation
soft callus. consists of an accumulation of blood at the fracture site, which
C. Callus mineralization is complete in 1 week. also contains devitalized soft tissue, dead bone, and necrotic

Wound Healing
D. Erythema, swelling, and pain usually take 5 days to marrow. The next stage accomplishes the liquefaction and deg-
resolve. radation of nonviable products at the fracture site. The normal
bone adjacent to the injury site can then undergo revascular-
ization, with new blood vessels growing into the fracture site.
This is similar to the formation of granulation in soft tissue.
The symptoms associated with this stage are characteristic of
inflammation, with clinical evidence of swelling and erythema.
Three to four days following injury, soft tissue forms a
bridge between the fractured bone segments in the next
stage (soft callus stage). The soft tissue is deposited where
neovascularization has taken place and serves as an internal
splint, preventing damage to the newly laid blood vessels and
achieving a fibrocartilaginous union. The soft callus is formed
externally along the bone shaft and internally within the mar-
row cavity. Clinically, this phase of healing is characterized by
the cessation of pain and inflammatory signs.
The next phase consists of mineralization of the soft cal-
lus and conversion to bone (hard callus stage). This may take
up to 2 to 3 months and leads to complete bony union. (See
Schwartz 11th ed., p. 281.)

11. What type of nerve injury involves disruption of axonal Answer: B


continuity with preserved Schwann cell basal lamina? There are three types of nerve injuries: neurapraxia (focal
A. Neurapraxia demyelination), axonotmesis (interruption of axonal con-
B. Axonotemesis tinuity but preservation of Schwann cell basal lamina), and
C. Neurotmesis neurotmesis (complete transection). Following all types of
D. Axonolysis injury, the nerve ends progress through a predictable pattern
of changes involving three crucial steps: (1) survival of axo-
nal cell bodies; (2) regeneration of axons that grow across the
transected nerve to reach the distal stump; and (3) migration
and connection of the regenerating nerve ends to the appro-
priate nerve ends or organ targets.
Phagocytes remove the degenerating axons and myelin
sheath from the distal stump (Wallerian degeneration). Regen-
erating axonal sprouts extend from the proximal stump and
probe the distal stump and the surrounding tissues. Schwann
cells envelope and help in remyelinating the regenerating
axons. Functional units are formed when the regenerating
axons connect with the appropriate end targets. (See Schwartz
11th ed., p. 281.)

12. Fetal wound healing differs from adults wound healing Answer: D
in a number of ways. Which is TRUE? Although early fetal wound healing is characterized by
A. Fetal wound healing is slower. the absence of scarring and resembles tissue regeneration,
B. All fetal wounds heal without a scar. there is a phase of transition during gestational life when
C. Fetal wounds continue to be scarless and look like a more adult-like healing pattern emerges. This so-called
regeneration throughout gestation until birth. “transition wound” occurs at the beginning of the third
D. Fetal wounds in late third trimester resemble an adult trimester, and during this period, there is scarless healing;
wound healing pattern. however, there is a loss of the ability to regenerate skin append-
ages. Eventually a classic, adult-patterned healing with scar
formation occurs exclusively, although overall healing contin-
ues to be faster than in adults. (See Schwartz 11th ed., p. 282.)

Brunicardi_Ch09_p073-082.indd 77 06/07/22 9:36 AM


78
13. Steroids impair wound healing by: Answer: A
A. Decreasing angiogenesis and macrophage migration. The major effect of steroids is to inhibit the inflammatory
CHAPTER 9

B. Decreasing platelet plug integrity. phase of wound healing (angiogenesis, neutrophil and macro-
C. Increasing release of lysosomal enzymes. phage migration, and fibroblast proliferation) and the release
D. Increasing fibrinolysis. of lysosomal enzymes. The stronger the anti-inflammatory
effect of the steroid compound used, the greater the inhibi-
tory effect on wound healing. Steroids used after the first 3 to
4 days postinjury do not affect wound healing as severely as
Wound Healing

when they are used in the immediate postoperative period.


Therefore if possible, their use should be delayed or, alterna-
tively, forms with lesser anti-inflammatory effects should be
administered.
In addition to their effect on collagen synthesis, steroids
also inhibit epithelialization and contraction and contrib-
ute to increased rates of wound infection, regardless of the
time of administration. Steroid-delayed healing of cutaneous
wounds can be stimulated to epithelialize by topical appli-
cation of vitamin A. Collagen synthesis of steroid-treated
wounds also can be stimulated by vitamin A. (See Schwartz
11th ed., p. 283.)

14. Which of the following does not significantly impair Answer: D


would healing? Low-oxygen tension has a profoundly deleterious effect on
A. Low-oxygen tension all aspects of wound healing. Fibroplasia, although stimu-
B. Hypoperfusion lated initially by the hypoxic wound environment, is signifi-
C. Arterial insufficiency cantly impaired by local hypoxia. Optimal collagen synthesis
D. Mild to moderate anemia requires oxygen as a cofactor, particularly for the hydroxyl-
ation steps. Increasing subcutaneous oxygen tension levels by
increasing the fraction of inspired oxygen (Fio2) of inspired
air for brief periods during and immediately following sur-
gery results in enhanced collagen deposition and in decreased
rates of wound infection after elective surgery.
Major factors affecting local oxygen delivery include hypo-
perfusion either for systemic reasons (low volume or cardiac
failure) or due to local causes (arterial insufficiency, local
vasoconstriction, or excessive tension on tissues). The level of
vasoconstriction of the subcutaneous capillary bed is exqui-
sitely responsive to fluid status, temperature, and hyperactive
sympathetic tone as is often induced by postoperative pain.
Correction of these factors can have a remarkable influence on
wound outcome, particularly on decreasing wound infection
rates. Mild to moderate normovolemic anemia does not appear
to adversely affect wound oxygen tension and collagen synthe-
sis. However, profound anemia with 15% less hematocrit can
interfere with wound healing. (See Schwartz 11th ed., p. 283.)

15. How does diabetes mellitus impair wound healing? Answer: A


A. Local hypoxemia, reduced angiogenesis, and inflam- Uncontrolled diabetes results in reduced inflammation,
mation due to vascular disease angiogenesis, and collagen synthesis. Additionally, the large
B. Glycosylation of proteoglycans and collagen in and small vessel disease that is the hallmark of advanced dia-
wound bed due to hyperglycemia betes contributes to local hypoxemia. Defects in granulocyte
C. Decreased collagen accretion noted in patients with function, capillary ingrowth, and fibroblast proliferation all
type II diabetes mellitus have been described in diabetes. Obesity, insulin resistance,
D. Increased bacterial load due to hyperglycemia hyperglycemia, and diabetic renal failure contribute signifi-
cantly and independently to the impaired wound healing
observed in diabetics. (See Schwartz 11th ed., p. 284.)

Brunicardi_Ch09_p073-082.indd 78 06/07/22 9:36 AM


79
16. Nutrition is important in recovery from trauma or sur- Answer: B
gical injury. Which of the following is not TRUE about Malnutrition correlates clinically with enhanced rates of

CHAPTER 9
nutrition in wound healing? wound complications and increased wound failure following
A. Adequate protein and calorie intake is needed for diverse surgical procedures. This reflects impaired healing
wound healing. response as well as reduced cell-mediated immunity, phago-
B. Brief periods of starvation, or absence of nutrient cytosis, and intracellular killing of bacteria by macrophages
intake has no impact on wound healing. and neutrophils during protein-calorie malnutrition.
C. Arginine supplementation is associated with First, the degree of nutritional impairment need not be

Wound Healing
improved outcome. long-standing in humans, as opposed to the experimental
D. Brief nutritional interventions can correct collagen situation. Thus, patients with brief preoperative illnesses or
deposition deficits seen in malnutrition and postop- reduced nutrient intake in the period immediately preced-
erative starvation. ing the injury or operative intervention will demonstrate
impaired fibroplasias. Second, brief and not necessarily
intensive nutritional intervention, either via the parenteral or
enteral route, can reverse or prevent the decreased collagen
deposition noted with malnutrition or with postoperative
starvation.
The possible role of single amino acids in enhanced wound
healing has been studied for the last several decades. Arginine
appears most active in terms of enhancing wound fibropla-
sia. Arginine deficiency results in decreased wound-breaking
strength and wound-collagen accumulation in chow-fed rats.
Rats that are given 1% arginine HCl supplementation, and
therefore are not arginine-deficient, have enhanced wound-
breaking strength and collagen synthesis when compared to
chow-fed controls. Studies have been carried out in healthy
human volunteers to examine the effect of arginine supple-
mentation on collagen accumulation. Young, healthy, human
volunteers (age 25–35 years) were found to have significantly
increased wound-collagen deposition following oral supple-
mentation with either 30 g of arginine aspartate (17 g of free
arginine) or 30 g of arginine Hall (24.8 g of free arginine)
daily for 14 days. (See Schwartz 11th ed., p. 284.)

17. Vitamins most closely involved with wound healing are: Answer: D
A. Vitamin C only. The vitamins most closely involved with wound healing are
B. Vitamin C and vitamin D. vitamin C and vitamin A. Scurvy or vitamin C deficiency
C. Vitamin C and vitamin E. leads to a defect in wound healing, particularly via a failure
D. Vitamin C and vitamin A. in collagen synthesis and cross-linking. Biochemically, vita-
min C is required for the conversion of proline and lysine to
hydroxyproline and hydroxylysine, respectively. Vitamin C
deficiency has also been associated with an increased inci-
dence of wound infection, and if wound infection does occur,
it tends to be more severe. These effects are believed to be
due to an associated impairment in neutrophil function,
decreased complement activity, and decreased walling-off of
bacteria secondary to insufficient collagen deposition. The
recommended dietary allowance is 60 mg daily. This provides
a considerable safety margin for most healthy nonsmokers. In
severely injured or extensively burned patients, this require-
ment may increase to as high as 2 g daily. There is no evidence
that excess vitamin C is toxic; however, there is no evidence
that supratherapeutic doses of vitamin C are of any benefit.
Vitamin A deficiency impairs wound healing, while sup-
plemental vitamin A benefits wound healing in nondeficient
humans and animals. Vitamin A increases the inflammatory
response in wound healing, probably by increasing the lability
of lysosomal membranes. There is an increased influx of mac-
rophages, with an increase in their activation and increased

Brunicardi_Ch09_p073-082.indd 79 06/07/22 9:36 AM


80
collagen synthesis. Vitamin A directly increases collagen
production and epidermal growth factor receptors when it is
CHAPTER 9

added in vitro to cultured fibroblasts. As mentioned before,


supplemental vitamin A can reverse the inhibitory effects of
corticosteroids on wound healing. Vitamin A also can restore
wound healing that has been impaired by diabetes, tumor
formation, cyclophosphamide, and radiation. (See Schwartz
11th ed., p. 285.)
Wound Healing

18. The most dangerous deep infection is necrotizing fasci- Answer: C


itis: Which of the following is NOT TRUE about necro- The most dangerous of the deep infections is necrotizing fas-
tizing fasciitis? ciitis. It results in high mortality, particularly in the elderly.
A. Skin demonstrated hemorrhagic bullae and/or frank This is an invasive process that involves the fascia and leads
necrosis. to secondary skin necrosis. Pathophysiologically, it is a sep-
B. Fascial necrosis is wider and more extensive than the tic thrombosis of the vessels between the skin and the deep
skin involvement. layers. The skin demonstrates hemorrhagic bullae and sub-
C. Most commonly this is due to Clostridia perfringens sequent frank necrosis, with surrounding areas of inflamma-
and other related species. tion and edema. The fascial necrosis is usually wider than the
D. Removal of all necrotic material is essential for infec- skin involvement or than the surgeon estimates on clinical
tion control. grounds. The patient is toxic and has high fever, tachycar-
dia, and marked hypovolemia, which if uncorrected, pro-
gresses to cardiovascular collapse. Bacteriologically, this is a
mixed infection, and samples should be obtained for Gram
stain smears and cultures to aid in diagnosis and treatment.
As soon as bacteriologic studies have been obtained, high-
dose penicillin treatment needs to be started (20–40 million
U/d intravenously) due to concern over the presence of
C. perfringens and other related species; broad-spectrum anti-
biotics should be added and the regimen modified based on
culture results. Cardiovascular resuscitation with electrolyte
solutions, blood, and/or plasma is carried out as expeditiously
as possible prior to induction of anesthesia. The aim of surgi-
cal treatment is thorough removal of all necrosed skin and
fascia. (See Schwartz 11th ed., p. 288.)

19. Signs of malignant transformation in a chronic wound Answer: B


include: Malignant transformation of chronic ulcers can occur in
A. Persistent granulation tissue with bleeding. any long-standing wound (Marjolin’s ulcer). Any wound
B. Overturned wound edges. that does not heal for a prolonged period of time is prone
C. Nonhealing after 2 weeks of therapy. to malignant transformation. Malignant wounds are differ-
D. Distal edema. entiated clinically from nonmalignant wounds by the pres-
ence of overturned wound edges. In patients with suspected
malignant transformations, biopsy of the wound edges must
be performed to rule out malignancy. Cancers arising de novo
in chronic wounds include both squamous and basal cell car-
cinomas. (See Schwartz 11th ed., p. 289.)

20. What is the difference between hypertrophic scars (HTS) Answer: B


and keloids? HTS and keloids represent an overabundance of fibroplasia
A. Keloids are an overabundance of fibroplasia as a in the dermal healing process. HTS rise above the skin level
result of healing, hypertrophic scars are a failure of but stay within the confines of the original wound and often
collagen remodeling. regress over time. Keloids rise above the skin level as well, but
B. HTS often regress over time, whereas keloids rarely extend beyond the border of the original wound and rarely
regress. regress spontaneously (Fig. 9-2). Both HTS and keloids occur
C. HTS are more common in darker-pigmented after trauma to the skin, and may be tender, pruritic, and
ethnicities. cause a burning sensation. Keloids are 15 times more com-
D. HTS extend beyond the border of the original wound. mon in darker-pigmented ethnicities, with individuals of

Brunicardi_Ch09_p073-082.indd 80 06/07/22 9:36 AM


81
African, Spanish, and Asian ethnicities being especially sus-
ceptible. Men and women are equally affected. Genetically,

CHAPTER 9
the predilection to keloid formation appears to be autosomal
dominant with incomplete penetration and variable expres-
sion. (See Schwartz 11th ed., p. 292.)

Wound Healing
FIG. 9-2. Recurrent keloid on the neck of
a 17-year-old patient that had been revised
several times. (Reproduced with permission from
Cohen IK, Diegelmann RF, Lindblad WJ: Wound
Healing: Biochemical and Clinical Aspects.
Philadelphia, PA: WB Saunders/ Elsevier; 1992.)

Brunicardi_Ch09_p073-082.indd 81 06/07/22 9:36 AM


This page intentionally left blank

Brunicardi_Ch09_p073-082.indd 82 06/07/22 9:36 AM


CHAPTER 10
Oncology

1. Which of the following is not a hallmark of cancer? Answer: C


A. Ability to invade and metastasize There are six essential alterations in cell physiology that dic-
B. Ability to evade apoptosis tate malignant growth: self-sufficiency of growth signals,
C. Ability to evade autophagy insensitivity to growth-inhibitory signals, evasion of apop-
D. Ability to evade immune destruction tosis (programmed cell death), potential for limitless repli-
cation, angiogenesis, and invasion and metastasis. Recently,
two additional hallmarks have emerged—reprogramming of
energy metabolism and evading immune destruction. (See
Schwartz 11th ed., p. 310.)

2. Characteristics of tumorigenic transformation of cells Answer: D


include which of the following? Abnormally proliferating transformed cells outgrow normal
A. Enhanced adherence to other cells cells in the culture dish (ie, in vitro) and commonly display
B. Halt in proliferation after monolayer confluence several abnormal characteristics. These include loss of contact
C. Acquisition of chemoresistance inhibition (ie, cells continue to proliferate after a confluent
D. Immortalization monolayer is formed); an altered appearance and poor adher-
ence to other cells or to the substratum; loss of anchorage
dependence for growth; immortalization; and gain of tumori-
genicity (ie, the ability to give rise to tumors when injected
into an appropriate host). (See Schwartz 11th ed., p. 310.)

3. The cell cycle includes all of the following phases Answer: D


EXCEPT: The cell cycle is divided into four phases. During the syn-
A. S phase thetic or S phase, the cell generates a single copy of its genetic
B. G1 phase material, whereas in the mitotic or M phase, the cellular com-
C. G2 phase ponents are partitioned between two daughter cells. The G1
D. G3 phase. and G2 phases represent gap phases during which the cells
prepare themselves for completion of the S and M phases,
respectively. When cells cease proliferation, they exit the cell
cycle and enter the quiescent state referred to as G0. (See
Schwartz 11th ed., pp. 311–312.)

4. All of the following factors are suggestive of a hereditary Answer: C


cancer EXCEPT: The following factors may suggest the presence of a heredi-
A. Tumor development at a younger than typical age. tary cancer: tumor development at a much younger age than
B. Presence of bilateral disease. usual, presence of bilateral disease, presence of multiple
C. Association with paraneoplastic syndromes. primary malignancies, presentation of a cancer in the less
D. Presence of multiple primary malignancies. affected sex (eg, male breast cancer), clustering of the same
cancer type in relatives, and occurrence of cancer in asso-
ciation with other conditions such as mental retardation or
pathognomonic skin lesions. (See Schwartz 11th ed., p. 320.)

83

Brunicardi_Ch10_p083-086.indd 83 30/06/22 10:39 AM


84
5. Anticancer chemotherapy agents include all of the Answer: C
CHAPTER 10

following EXCEPT: (See Schwartz 11th ed., p. 340.)


A. Alkylating agents.
B. Antitumor antibiotics.
C. Prometabolites.
D. Plant alkaloids.

6. Approved strategies for cancer chemoprevention include Answer: A


Oncology

all of the following EXCEPT: (See Schwartz 11th ed., p. 348.)


A. Neurontin for malignant peripheral nerve sheath
tumor.
B. Tamoxifen for breast cancer.
C. Celecoxib for FAP syndrome.
D. 13-cis-retinoic acid for oral leukoplakia.

7. Which of the following statements is TRUE of apoptosis Answer: D


in cancer cells? Apoptosis is a genetically regulated program to dispose of
A. Beclin 1 is an important apoptosis regulatory gene. cells. Cancer cells must avoid apoptosis if tumors are to arise.
B. Chloroquin is an important inhibitor of apoptosis. Late in apoptosis, there are characteristic changes in nuclear
C. Anti-caspases are a family of genes involved in regu- morphology, such as chromatin condensation, nuclear frag-
lation of apoptosis. mentation, and DNA laddering. The apoptosis effectors are
D. DNA laddering is a typical characteristic observed a family of proteases called caspases. The Beclin 1 gene is an
with apoptosis. important gene for autophagy. Chloroquine is an antimalarial
drug that acts as an autophagic inhibitor. (See Schwartz 11th
ed., pp. 314–315.)

8. Components of cancer invasion include which of the Answer: B


following? A feature of malignant cells is their ability to invade the sur-
A. In situ cancers involve cells which have breached the rounding normal tissue. Tumors in which the malignant cells
basement membrane. appear to lie exclusively above the basement membrane are
B. Integrins regulate cell adhesion to the extracellular referred to as in situ cancer. Integrins are a family of glyco-
matrix. proteins that regulate cell adhesion and also relay molecular
C. Matrix metalloproteinases (MMPs) enhance the signals regarding the cellular environment that influence
structural integrity of the extracellular matrix. shape, survival, proliferation, gene transcription, and migra-
D. Vascular endothelial growth factor (VEGF) induces tion. MMPs comprise a family of metal-dependent endo-
metastases exclusively through vascular channels. peptidases. Upon activation, MMPs degrade a variety of
ECM components. VEGF has various functions, including
increasing vascular permeability, inducing endothelial cell
proliferation and tube formation, and inducing endothelial
cell synthesis of proteolytic enzymes. The proliferation of
new lymphatic vessels, lymph angiogenesis, is also thought
to be controlled by the VEGF family. (See Schwartz 11th ed.,
pp. 315–316.)

9. Which of the following statements is TRUE of metastasis? Answer: C


A. If a cell can detach from a primary tumor and invade Metastases arise due to the spread of cancer cells from the
the lymphovascular system, it has a high likelihood of primary site and the formation of new tumors in distance
successfully implanting in a distant site. locations. Overall, metastasis is an inefficient process; only a
B. Late development of metastasis as long as 20 years small subset of cancer cells is able to initiate micrometastases,
after primary tumor curative treatment is a common and an even smaller portion goes on to grow into macrome-
occurrence. tastases. Recurrences are rare after 20 years; however, cancer
C. Tumor dormancy may be due to the host losing recurrences have been reported decades after the original
immunologic control of subclinical disease. tumor episode. This phenomenon is referred to as dormancy.
D. The “seed and soil” conception of metastasis is An explanation for preferential metastasis is what is referred
explained by organ-specific patterns of blood flow to as the “seed and soil” theory, the dependence of the seed
determinants. (the cancer cell) on the soil (the secondary site). According to
this theory, once cells have reached a secondary organ, their
growth efficiency in that organ is based on the compatibility
of the cancer cell’s biology with its new microenvironment.
(See Schwartz 11th ed., pp. 316–317.)

Brunicardi_Ch10_p083-086.indd 84 30/06/22 10:39 AM


85
10. Which of the following is TRUE regarding biomarkers? Answer: C

CHAPTER 10
A. Prognostic biomarkers predict response to specific Tumor markers are substances that can be detected in higher
treatments. than normal amounts in the serum, urine, or tissues of patients
B. Predictive biomarkers predict likelihood of survival with certain types of cancer. The term “prognostic marker”
for specific tumors. generally is used to describe molecular markers that predict
C. Results of multigene assays can be linked together to disease-free survival, disease-specific survival, and overall
provide optimal survival predictability. survival, whereas the term “predictive marker” often is used in
D. Circulating free DNA is more informative as solid the context of predicting response to certain therapies. Multi-

Oncology
tumor biomarkers than circulating free proteins. gene profiles to predict prognosis are in validation phase for
many solid tumor types. (See Schwartz 11th ed., pp. 334–335.)

11. Surgical resection of distant metastatic disease for tumor Answer: D


control is enhanced by all of the following EXCEPT: The treatment of a patient with distant metastases depends
A. Pre-resection tumor shrinkage using chemotherapy. on the number and sites of metastases, the cancer type, the
B. Long interval between primary tumor treatment and rate of tumor growth, the previous treatments delivered and
subsequent development of metastasis. the responses to these treatments, and the patient’s age, physi-
C. Tumor type. cian condition, and desires. Patient selection is the key to the
D. Open versus minimally invasive surgical approaches. success of surgical therapy for distance metastases. The cancer
type is a major determinant in surgical decision-making. The
growth rate of the tumor also plays an important role. Patients
with longer disease-free intervals have a higher survival rate
after surgical metastasectomy than those with a short disease-
free interval. In curative surgery for distance metastases,
as with surgery for primary tumors, the goal is to resect the
metastases with negative margins, whether or not open or MIS
approaches are used. (See Schwartz 11th ed., p. 339.)

12. Which of the following is established as an improvement Answer: A


in cancer management? The current trend in surgery is toward more conservative
A. Use of immunotherapy as initial therapy in some resections. With earlier identification of tumors, more con-
types of lung cancer servative operations may be possible. The goal, however, is
B. Replacement of biopsy with serum biomarkers always to remove the tumor en bloc with wide negative mar-
C. Elimination of radical surgery approaches gins. With the advent of effective immune-based therapies, it
D. Use of molecular radiotherapy sensitizers is likely that patients may be given treatments that can spe-
cifically target the alterations driving tumor growth in com-
bination with drugs that can enhance the anticancer immune
response, as is now standard of care for several; tumors, includ-
ing some types of lung carcinoma. (See Schwartz 11th ed.,
pp. 349–350.)

Brunicardi_Ch10_p083-086.indd 85 30/06/22 10:39 AM


This page intentionally left blank

Brunicardi_Ch10_p083-086.indd 86 30/06/22 10:39 AM


CHAPTER 11
Transplantation

1. Which of the following definitions is FALSE? Answer: A


A. Transplantation is the process of transferring only Transplantation is the process of transferring an organ, tis-
intact organs from one person to another. sue, or cell from one place to another. An organ transplant is
B. Orthotopic transplanted organs are implanted in the a surgical procedure in which a failing organ is replaced by a
same place as the native organ. functioning one. The organ is transplanted either orthotopi-
C. Heterotopic transplanted organs in a different place, cally (implanted in the same anatomic location in the recipi-
so that the native organs do not need to be removed. ent as it was in the donor) or heterotopically (implanted in
D. Allotransplant means transplant of cells or organs another anatomic location). Orthotopic transplants require
from one person to another. the removal of the diseased organ (heart, lungs, liver, or intes-
tine); in heterotopic transplants, the diseased organ is kept in
place (kidney, pancreas).
According to the degree of immunologic similarity between
the donor and recipient, transplants are divided into three
main categories: (a) an autotransplant is the transfer of cells,
tissue, or an organ from one part of the body to another part
in the same person, so no immunosuppression is required;
this type of transplant includes skin, artery or vein, bone, car-
tilage, nerve, and islet cell transplants; (b) an allotransplant
is the transfer of cells, tissue, or an organ from one person
to another of the same species; with the exception of identi-
cal twins, the immune system of the recipient recognizes the
donated organ as a foreign body, so immunosuppression is
required in order to avoid rejection; and (c) a xenotransplant
is the transfer of cells, tissue, or an organ from one organism to
another of a different species. (See Schwartz 11th ed., p. 356.)

2. In humans: Answer: C
A. Antigen encoding genes are located on chromosome 8. The main antigens responsible for this process are part of the
B. Major histocompatibility complex (MHC) system in MHC. In humans, these antigens make up the human leuko-
humans is called the human histocompatibility com- cyte antigen (HLA) system. The antigen-encoding genes are
plex (HHC). located on chromosome 6. Two major classes of HLA anti-
C. Class I antigens (HLA-A, HLA-B, HLA-C) are expressed gens are recognized. They differ in their structure, function,
on all nucleated cells. and tissue distribution. Class I antigens (HLA-A, HLA-B,
D. Class II antigens are expressed on all nucleated cells. and HLA-C) are expressed by all nucleated cells. Class II anti-
gens (HLA-DR, HLA-DP, and HLA-DQ) are expressed by
antigen-presenting cells (APCs) such as B lymphocytes, den-
dritic cells, macrophages, and other phagocytic cells. (See
Schwartz 11th ed., p. 358.)

87

Brunicardi_Ch11_p087-096.indd 87 04/07/22 1:31 PM


88
3. Hyperacute rejection is caused by: Answer: A
CHAPTER 11

A. Preformed antibodies. Hyperacute rejection, a very rapid type of rejection, results


B. B-cell–generated antidonor antibodies. in irreversible damage and graft loss within minutes to hours
C. T-cell–mediated allorejection. after organ reperfusion. It is triggered by preformed antibod-
D. Nonimmune mechanism. ies against the donor’s HLA or ABO blood group antigens.
These antibodies activate a series of events that result in dif-
fuse intravascular coagulation, causing ischemic necrosis of
the graft. Fortunately, pretransplant blood group typing and
Transplantation

cross-matching (in which the donor’s cells are mixed with


the recipient’s serum, and then the cells are observed for any
destruction) have virtually eliminated the incidence of hyper-
acute rejection. (See Schwartz 11th ed., p. 358.)

4. Which is TRUE about mycophenolate mofetil’s (MMF) Answer: D


use in transplantation? The antimetabolite, AZA is converted to 6-mercaptopurine
A. It is safe to use in pregnancy. and inhibits both the de novo purine synthesis and salvage
B. It has fewer gastrointestinal (GI) side effects than purine synthesis. AZA decreases T-lymphocyte activity and
azathiaprine (AZA). decreases antibody production. It has been used as a first-line
C. Like AZA, it interferes with lymphocyte proliferation agent in transplant recipients for >40 years, but it became an
by interfering with purine synthesis. adjunctive agent after the introduction of cyclosporine. With
D. Its use is limited to induction immunosuppression. the development of newer agents such as MMF, the use of
AZA has decreased significantly. However, it is preferred in
recipients who are considering conceiving a child because
MMF is teratogenic and can cause birth defects. Use of AZA
remains an option for recipients who cannot tolerate the GI
side effects of MMF.
The most significant side effect of AZA, often dose-related,
is bone marrow suppression. Leukopenia is often reversible
with dose reduction or temporary cessation of the drug. Other
significant side effects include hepatotoxicity, pancreatitis,
neoplasia, anemia, and pulmonary fibrosis. Its most signifi-
cant drug interaction is with allopurinol, which blocks AZA
metabolism, increasing the risk of pancytopenia. Recom-
mendations are to not use AZA and allopurinol together, or if
doing so is unavoidable, to decrease the dose of AZA by 75%.
MMF has now been incorporated into routine maintenance
regimens after many solid organ transplants. Mycophenolate
is the prodrug of mycophenolate acid, derive from Penicillium
fungi. Mycophenolate acid is an inhibitor of inosine mono-
phosphate dehydrogenase (IMPDH) involved in the de novo
pathway of purine synthesis. MMF is available in capsules (250
and 500 mg); the starting dose is 1 g twice daily. (See Schwartz
11th ed., p. 361.)

5. Compared to cyclosporin, tacrolimus: Answer: B


A. Can be given intravenously, but cyclosporin must be Cyclosporin binds with its cytoplasmic receptor protein,
given orally. cyclophilin, which subsequently inhibits the activity of cal-
B. They both are calcineurin inhibitors. cineurin, thereby decreasing the expression of several critical
C. Levels do not need to be monitored. T-cell activation genes, the most important being for IL-2. As
D. Is more associated with gingival hyperplasia and hir- a result, T-cell activation is suppressed. Cyclosporin can be
sutism than cyclosporin. given intravenously or orally to maintain trough levels of 250
to 350 ng/mL for the first 3 months posttransplant; then it can
be tapered to 150 to 250 ng/mL.
The metabolism of cyclosporine is via the cytochrome P450
system, resulting in many significant drug interactions (see
Table 11-1). Calcineurin inhibitors are nephrotoxic and con-
strict the afferent arteriole in a dose-dependent, reversible
manner (Table 11-2). They also can cause hyperkalemia and
hypomagnesemia. Several neurologic complications, including
headaches, tremor, and seizures, also have been reported.

Brunicardi_Ch11_p087-096.indd 88 04/07/22 1:31 PM


89
TABLE 11-1 Side effects and drug interactions of the main immunosuppressive drugs

CHAPTER 11
Other Medications That Other Medications That Other Medications That
Common Side Effects Increase Blood Levels Decrease Blood Levels Potentiate Toxicity
Cyclosporine (CSA) Hypertension, nephrotoxicity, Verapamil, diltiazem, Isoniazid, carbamazepine, Nephrotoxicity: ganciclovir,
hirsutism, neurotoxicity, clarithromycin, azithromycin, phenobarbital, phenytoin, aminoglycosides, NSAIDs,
gingival hyperplasia, erythromycin, azole rifampin, St. John’s wort ACE-Is, and ARBs
hypomagnesemia, antifungals, protease
hyperkalemia inhibitors, grapefruit juice

Transplantation
Tacrolimus (FK506) Hypertension, nephrotoxicity, Verapamil, diltiazem, Isoniazid, carbamazepine, Nephrotoxicity: ganciclovir,
alopecia, hyperglycemia, clarithromycin, azithromycin, phenobarbital, phenytoin, aminoglycosides, NSAIDs,
neurotoxicity, erythromycin, azole rifampin, St. John’s wort ACE-Is, and ARBs
hypomagnesemia, antifungals, protease
hyperkalemia inhibitors, grapefruit juice
Sirolimus Thrombocytopenia and Verapamil, diltiazem, Isoniazid, carbamazepine, —
neutropenia, elevated clarithromycin, azithromycin, phenobarbital, phenytoin,
cholesterol, extremity edema, erythromycin, azole rifampin, St. John’s wort
impaired wound healing antifungals, protease
inhibitors, grapefruit juice
Mycophenolate Leukopenia, — Cholestyramine, antacids Bone marrow suppression:
mofetil thrombocytopenia, GI upset valganciclovir, ganciclovir,
TMP-SMX
Corticosteroids Hyperglycemia, osteoporosis, — — —
cataracts, myopathy, weight
gain
Azathioprine Leukopenia, anemia, — — Bone marrow suppression:
thrombocytopenia, neoplasia, allopurinol, sulfonamides
hepatitis, cholestasis
ACE-I = angiotensin-converting enzyme inhibitor; ARB = angiotensin receptor blocker; NSAID = nonsteroidal anti-inflammatory drug; TMP-SMX = trimethoprim-sulfamethoxazole

The calcineurin inhibitor tacrolimus (Prograf) is now


the backbone of most immunosuppressive regimens. Tacro-
limus acts by binding FK506-binding proteins (FKBPs),
causing roughly 10 to 100 times more potent inhibition of
IL-2 production than cyclosporine (which acts by binding
cyclophilins). It can be given intravenously, orally, or sublin-
gually to maintain trough levels of 8 to 12 ng/mL for the first
3 months posttransplant; then it can be tapered to 6 to
10 ng/mL. The metabolism of tacrolimus is via the cytochrome
P450 system, resulting in many significant drug interactions
(see Table 11-1). Tacrolimus causes a higher incidence of
new-onset diabetes posttransplant than does cyclosporine.
Other side effects include alopecia, nephrotoxicity, neuro-
toxicity, hypertension, hyperkalemia, hypomagnesemia, and
an increased incidence of certain types of infection. (See
Schwartz 11th ed., p. 362.)

TABLE 11-2 Drug interactions and side effects associated


with calcineurin inhibitors
Interactions Medications
Inhibition of Clarithromycin, erythromycin, azole antifungals,
metabolism diltiazem, verapamil, nicardipine, amiodarone,
grapefruit juice, ritonavir, azithromycin
Induction of Nevirapine, rifampin, St. John’s wort, carbamazepine,
metabolism phenobarbital, phenytoin, caspofungin
Hyperkalemia Potassium-sparing diuretics, angiotensinconverting
enzyme inhibitors (ACE-Is), angiotensin
receptor blockers (ARBs), β-blockers,
trimethoprim-sulfamethoxazole
Nephrotoxicity Nonsteroidal anti-inflammatory drugs,
aminoglycosides, amphotericin, ACE-Is, ARBs

Brunicardi_Ch11_p087-096.indd 89 04/07/22 1:31 PM


90
6. The most significant side effect of Sirolimus is: Answer: D
CHAPTER 11

A. Anemia. The first mammalian target of rapamycin (mTOR) inhibitors


B. Leukopenia. to enter clinical use was sirolimus (Rapamune). The mTOR
C. Impaired wound healing. inhibitors bind to FK506-binding protein (FKBP), and the
D. Hypertriglyceridemia. sirolimus-FKBP complex binds to mTOR. Sirolimus also
inhibits proliferation of vascular smooth muscle cells, possi-
bly easing the vasculopathy and progressive fibrosis that can
affect allografts. Sirolimus is a substrate for CYP3A4/4 and
Transplantation

has many significant drug interactions. Sirolimus has been


used in a variety of combinations for maintenance immuno-
suppression, alone or in conjunction with one of the calci-
neurin inhibitors. In such combinations, sirolimus usually is
used to help withdraw from, or completely avoid, the use of
steroids. It also has been used as an alternative to tacrolimus
or cyclosporine, in a calcineurin-sparing protocol. One of the
most significant side effects of sirolimus is hypertriglyceride-
mia, a condition that may be resistant to statins and fibrates.
(See Schwartz 11th ed., p. 361.)

7. In the prevention of graft rejection, cyclosporin: Answer: D


A. Blocks transcriptions of interleuking-1 (IL-1) and Corticosteroids have had a role in immunosuppression since
tumor necrosis factor (TNF). the beginning of the field of transplantation. Despite numer-
B. Inhibits lymphocyte nucleic acid metabolism. ous attempts to limit or discontinue their use, they remain an
C. Results in rapid decrease in the number of circulating integral component of most immunosuppressive protocols,
T lymphocytes. for both induction and maintenance. Moreover, they are often
D. Selectively inhibits T-cell activation. the first-line agents in the treatment of acute rejection. The
introduction of cyclosporine in the early 1980s dramatically
altered the field of transplantation by significantly improving
outcomes after kidney transplantation. Cyclosporine binds
with its cytoplasmic receptor protein, cyclophilin, which sub-
sequently inhibits the activity of calcineurin, thereby decreas-
ing the expression of several critical T-cell activation genes,
the most important being for IL-2. As a result, T-cell activa-
tion is suppressed. (See Schwartz 11th ed., p. 362.)

8. Late infections in transplant recipients are: Answer: D


A. Generally due to suppression of humoral immunity. Late infections primarily are due to chronic immunosup-
B. Common bacterial infections. pression, specifically the depression of cell-mediated immu-
C. Depression of cell-mediated immunity renders recip- nity that renders recipients susceptible to viruses, fungi, and
ients less susceptible to opportunistic infections. parasites.
D. Depression of cell-mediated immunity renders recip- Members of the herpesvirus group are the most common
ients more susceptible to viruses, fungi, and parasites. etiologic agents of viral infections posttransplantation, with
herpes simplex virus (HSV), cytomegalovirus (CMV), and
Epstein-Barr virus (EBV) being the most prominent. Pre-
transplant exposure to viruses may confer immunity. Recipi-
ents who are seronegative for HSV, CMV, and/or EBV have
a higher incidence of posttransplant infections, especially if
they receive donor allografts from seropositive donors. After
6 months posttransplant, the risk of invasive fungal infections
is closely associated with environmental exposures. Blastomy-
ces dermatitidis grows in moist soil in the Midwest and South-
east regions of the United States. Diagnosis is confirmed by
biopsy; the preferred treatment is IV amphotericin B. (See
Schwartz 11th ed., p. 363.)

Brunicardi_Ch11_p087-096.indd 90 04/07/22 1:31 PM


91
9. Compared to donation after rain death, donation after Answer: B

CHAPTER 11
cardiac death (DCD): Given the severe shortage of donor organs, DCD—also known
A. Most patients meet the criteria for brain death. as donation by non–heart-beating donors (NHBDs)—was
B. Is associated with increased warm ischemia time. reintroduced to the transplant community in the 1990s. The
C. In both procurement surgeons wait until cessation of category of DCD (Maastricht classification) was initially
cardiac activity. proposed at an international workshop and is now widely
D. Transfer to the operating room occurs after cessation adopted for organ procurement. Currently, most NHBDs in
of cardiac activity. the United States meet Maastricht classification III; that is, they

Transplantation
have suffered a devastating injury with no chance of a mean-
ingful recovery but do not meet the criteria for brain death.
After consent for donation is obtained from the next of kin,
the donor’s life support is removed. After the cessation of car-
diac and respiratory function, organ procurement commences.
DCD procurement protocols vary between states; religious and
cultural differences need to be taken into consideration. The
surgical team must be familiar with, and respect, the local pro-
tocol. With cardiac death (as opposed to brain death), warm
ischemic injury to organs can occur during the period between
circulatory cessation and rapid core cooling through perfu-
sion of preservation solution. However, the difference in long-
term outcomes is negligible for recipients of organs from either
type of donor. Still, a significant percentage of liver grafts pro-
cured after cardiac death, especially those with >25 minutes of
warm ischemic time, develop devastating ischemic cholangi-
opathy and fail. (See Schwartz 11th ed., p. 365.)

10. Compared to deceased donor kidneys, living donor kid- Answer: B


ney transplants: The advantages of a living donor kidney transplant include bet-
A. Must be from human leukocyte antigen (HLA) iden- ter posttransplant outcomes, avoidance of prolonged waiting
tical relatives. time and dialysis, and the ability to coordinate the donor and
B. Are associated with improved posttransplant recipient procedures in a timely fashion. Living donor kidney
outcomes. recipients enjoy better long-term outcomes, a low incidence
C. Increase in delayed graft function. of delayed graft function, and reduced risks of posttransplant
D. Have lower 1-year graft survival rate. complications. Furthermore, the elective nature of living donor
kidney transplants provides unique opportunities for recipient
desensitization treatment if the donor and recipient are ABO-
incompatible or if the HLA cross-match results are positive.
Posttransplant outcomes have continued to improve. In
2015, the 1-year graft survival rate with a living donor kidney
was nearly 98%; with a deceased donor kidney, the rate was
approximately 95%. (See Schwartz 11th ed., p. 368.)

11. The most common cause of renal failure in the United Answer: C
States is: Diabetes and hypertension are the leading causes of chronic
A. Chronic glomerulonephritis. renal disease. Concomitant cardiovascular disease (CVD)
B. Chronic pyelonephritis. is a common finding in this population. An estimated 30%
C. Diabetes mellitus. to 42% of deaths with a functioning kidney graft are due to
D. Obstructive uropathy. CVD. Therefore, assessment of the potential kidney trans-
plant candidate’s cardiovascular status is an important part of
the pretransplant evaluation. (See Schwartz 11th ed., p. 369.)

12. Kidney grafts are placed in adult recipients: Answer: C


A. Intraperitoneally, with anastomosis to inferior vena Kidney allografts usually are transplanted heterotopically.
cava and aorta. The iliac fossa is recognized as the ideal position because
B. Orthotopically, with anastomosis to native renal vessels. of its proximity to the recipient’s bladder and iliac vessels.
C. Heterotopically, in the retroperitoneal space with Retroperitoneal allograft placement also allows easy access
anastomosis to iliac vessels. for percutaneous biopsies and interventions for ureteral
D. Orthotopically, with anastomosis to the adrenal complications. In general, the donor’s renal artery and vein
artery and veins. are anastomosed to the recipient’s external iliac vessels in an

Brunicardi_Ch11_p087-096.indd 91 04/07/22 1:31 PM


92
end-to-side fashion (Fig. 11-1). In recipients with a severely
CHAPTER 11

calcified iliac artery, the internal iliac artery can be used as an


alternative, and in select cases, an endarterectomy must be
performed. (See Schwartz 11th ed., p. 370.)
Transplantation

A B

FIG. 11-1. Vascular anastomoses of kidney transplant. A. Arterial anastomosis: donor renal artery with Carrel patch to recipient external
iliac artery, end-to-side. B. Venous anastomosis: donor renal vein with caval extension conduit to recipient external iliac vein, end-to-side.

13. The most important factor in determining whether to Answer: C


perform a transplant between a specific donor and recip- ABO blood typing and HLA typing (HLA-A, -B, and -DR) are
ient is: required before a kidney transplant. The method of screen-
A. Mixed lymphocyte culture assays of the donor and ing for preformed antibodies against HLA antigens (because
recipient. of prior transplants, blood transfusions, or pregnancies) con-
B. Human leukocyte antigen (HLA) type of the donor tinues to evolve. The panel-reactive antibody (PRA) assay is a
and recipient. screening test that examines the ability of serum from a kid-
C. ABO blood types of the donor and recipient. ney transplant candidate to lyse lymphocytes from a panel of
D. Peripheral T-cell count of the recipient. HLA-typed donors. A numeric value, expressed as a percent-
age, indicates the likelihood of a positive cross-match with
a donor. A higher PRA level identifies patients at high risk for a
positive cross-match and therefore serves as a surrogate marker
to measure the difficulty of finding a suitable donor and the sub-
sequent risk of graft rejection. (See Schwartz 11th ed., p. 370.)

14. After a kidney transplant, a recipient was producing at Answer: D


least 100 cc of urine/hour. Eight hours after completing Suddenly decreased or minimal urine output requires imme-
the transplant you are informed that their urine output is diate attention. A change in volume status is the most com-
now 5 cc over the past hour. Likely etiology is: mon cause, but other culprits include blockage of the urinary
A. Hypovolemia due to postoperative bleeding. catheter, urinary leak, vascular thrombosis, hypotension,
B. Acute tubular necrosis (ATN). drug-related nephrotoxicity, ATN, and rejection (all of which
C. Graft thrombosis. must be thoroughly investigated). Diagnostic studies such as
D. All of the above. Doppler ultrasound, nuclear renograms, or biopsies should
be considered.
Postoperative bleeding is an uncommon event after a kid-
ney transplant. Recipients on anticoagulation or antiplatelet
treatments are at increased risk. Signs and symptoms (such
as an expanding hematoma over the surgical site, increased
pain over the graft, a falling hemoglobin level, hypotension,
and tachycardia) should arouse suspicion of hemorrhage.
Doppler ultrasound is useful to establish the underlying
cause. Surgical exploration seldom is required because the
accumulated hematoma tamponades the bleed. Indications
for surgical exploration include ongoing transfusion require-
ment, hemodynamic instability, and graft dysfunction from

Brunicardi_Ch11_p087-096.indd 92 04/07/22 1:31 PM


93
hematoma compression. For recipients on anticoagulation or

CHAPTER 11
antiplatelet treatments, the threshold for surgical exploration
is lower. Small unligated vessels at the donor’s renal hilum or
recipient’s retroperitoneum are likely sources of bleeding.
One of the most devastating postoperative complications
in kidney recipients is graft thrombosis. It is rare, occur-
ring in fewer than 1% of recipients. The recipient risk factors
include a history of recipient hypercoagulopathy and severe

Transplantation
peripheral vascular disease; donor-related risk factors include
the use of en bloc or pediatric donor kidneys, procurement
damage, technical factors such as intimal dissection or tor-
sion of vessels, and hyperacute rejection. Graft thrombosis
usually occurs within the first several days posttransplant.
Acute cessation of urine output in recipients with brittle post-
transplant diuresis and the sudden onset of hematuria or graft
pain should arouse suspicion of graft thrombosis. Doppler
ultrasound may help confirm the diagnosis. In cases of graft
thrombosis, an urgent thrombectomy is indicated; however, it
rarely results in graft salvage. (See Schwartz 11th ed., p. 373.)

15. Postrenal transplant graft thrombosis usually occurs: Answer: A


A. Within 2–3 days. One of the most devastating postoperative complications in
B. Within 2 weeks. kidney recipients is graft thrombosis. It is rare, occurring
C. Within the first month. in fewer than 1% of recipients. The recipient risk factors
D. Within the first 5 months. include a history of recipient hypercoagulopathy and severe
peripheral vascular disease; donor-related risk factors include
the use of en bloc or pediatric donor kidneys, procurement
damage, technical factors such as intimal dissection or tor-
sion of vessels, and hyperacute rejection. Graft thrombosis
usually occurs within the first several days posttransplant.
Acute cessation of urine output in recipients with brittle post-
transplant diuresis and the sudden onset of hematuria or graft
pain should arouse suspicion of graft thrombosis. Doppler
ultrasound may help confirm the diagnosis. In cases of graft
thrombosis, an urgent thrombectomy is indicated; however, it
rarely results in graft salvage. (See Schwartz 11th ed., p. 373.)

16. The most common causes of graft loss include: Answer: A


A. Recipient death. Currently, the most common cause of graft loss is recipi-
B. Acute rejection. ent death (usually from cardiovascular causes) with a func-
C. Surgical technique. tioning graft. The second most common cause is chronic
D. Postoperative infection. allograft nephropathy; characterized by a slow, unrelenting
deterioration of graft function, it likely has multiple causes
(both immunologic and nonimmunologic). The graft failure
rate due to complications related to surgical technique has
remained at about 1% to 2% (See Schwartz 11th ed., p. 374.)

17. After completion of the vascular anastomoses, drainage Answer: D


of the transplanted pancreas is accomplished by anasto- Over the years, different surgical techniques have been
mosis to: described for (a) the management of exocrine pancreatic
A. Right colon. secretions and (b) the type of venous drainage. For the secre-
B. Left colon. tions, the two most common techniques are drainage of
C. Duodenum. the duodenal segment to the bladder (bladder drainage) or
D. Bladder or small bowel. to the small bowel (enteric drainage) (Figs. 11-2 and 11-3).
For venous drainage, systemic venous drainage is preferred
over portal venous drainage. (See Schwartz 11th ed., p. 375,
Figures 11-12 and 11-14.)

Brunicardi_Ch11_p087-096.indd 93 04/07/22 1:31 PM


94
CHAPTER 11
Transplantation

FIG. 11-2. Whole-organ transplant with systemic vein and


bladder exocrine drainage.

FIG. 11-3. Segmental transplant with systemic vein and bladder


exocrine drainage. The donor splenic artery and splenic vein are
anastomosed end-to-side to the recipient’s external iliac artery
and vein. The splenic artery anastomosis is lateral and proximal
to the splenic vein anastomosis. A two-layer ductocystostomy
is constructed.

18. Which of the following is TRUE about islet cell Answer: B


transplants? Until recently, attempts to extend those trailblazing findings
A. Involve anastomosis of an artery and vein of a seg- of clinical islet auto transplants to clinical islet allotransplants
ment of pancreas. in patients with type 1 diabetes met with generally very poor
B. Islet allotransplants can prevent diabetes in patients success. For example, in 1995, a report of the International
who require pancreatectomy. Islet Transplant Registry indicated that of 270 recipients, only
C. Single donor islet cell transplants do not require 5% were insulin-independent at 1 year posttransplant.
immunosuppression because they are injected in the In 2000, Shapiro and colleagues reported the results of
portal vein. the Edmonton protocol, which enabled consistent diabetes
D. It has been shown to reverse diabetes in >90% of reversal and short-term (<1 year) insulin independence. The
recipients. Edmonton protocol prescribed transplanting a large number
of freshly isolated islets (>10,000 islet equivalents per kilo-
gram body weight, typically requiring the use of two to four
pancreases) with a specialized “islet-sparing,” steroid-free

Brunicardi_Ch11_p087-096.indd 94 04/07/22 1:31 PM


95
immunosuppressive protocol consisting of low-dose tacroli-

CHAPTER 11
mus, sirolimus, and IL-2 receptor antibody induction. Those
results were replicated at other experienced transplant cen-
ters, but the rates of long-term (>5 year) insulin indepen-
dence remained poor, well below those of whole-pancreas
transplants. Still, despite the low rates of long-term insulin
independence, most islet recipients were C-peptide positive
and retained hypoglycemia awareness, indicating residual

Transplantation
islet function and benefit. In fact, at 9 years posttransplant,
15% remained insulin-independent, and 73% had hypogly-
cemia awareness and corrected hemoglobin Aic levels. (See
Schwartz 11th ed., p. 378.)

19. The most common diagnosis leading to heart transplant is: Answer: C
A. COPD. The most common diagnosis leading to a heart transplant is
B. Congenital heart disease. ischemic dilated cardiomyopathy, which stems from coronary
C. Ischemic dilated cardiomyopathy. artery disease, followed by idiopathic dilated myopathy and
D. Idiopathic dilated cardiomyopathy. congenital heart disease. About 3000 patients are added to the
waiting list each year. (See Schwartz 11th ed., p. 389.)

20. The best method for monitoring the development of acute Answer: C
rejection in a patient after cardiac transplantation is: The goal of immunosuppression is to prevent rejection,
A. Dipyrimidole thallium study. which is assessed by immunosuppressive levels and, early
B. Electrocardiogram. on, by endomyocardial biopsy. Both T-cell–mediated (cellu-
C. Endomyocardial biopsy. lar) and B-cell–mediated (antibody-mediated) rejection are
D. Echocardiography (ultrasound of heart). monitored. Most of the immunosuppression used is aimed at
T-cells; however, if the recipient has many preformed anti-
bodies or develops donor-specific antibodies, other strategies
(such as plasmapheresis or rituximab) are used to reduce the
antibody load. Immunosuppressive regimens can vary by cen-
ter, but most often consist of three categories of medications: a
calcineurin inhibitor (usually tacrolimus or cyclosporine),
an antiproliferative agent (mycophenolate mofetil [MMF] or
azathioprine [AZA]), and a corticosteroid (prednisone). Other
immunosuppressive agents can be used, depending on the
needs of individual recipients. (See Schwartz 11th ed., p. 390.)

21. What is not a contraindication to liver transplantation? Answer: D


A. Chronic cardiac failure In general, contraindications to a liver transplant include
B. Stage 4 malignancy insufficient cardiopulmonary reserve, uncontrolled malig-
C. Refractory alcoholism nancy or infection, and refractory noncompliance. Older age
D. Age > 70 is only a relative contraindication: carefully selected recipi-
ents age >70 years can achieve satisfactory outcomes. (See
Schwartz 11th ed., p. 382.)

22. Which of the following patients with hepatic failure ben- Answer: D
efit from liver transplantation? The MELD was originally developed to assess risk for tran-
A. Model for End-Stage Liver Disease (MELD) > 18 sjugular intrahepatic portosystemic shunt (TIPS) placement.
B. All patients with MELD < 18 Later analysis revealed it to be an excellent model to predict
C. MELD 15–18 if they have significant morbidity from survival among patients with cirrhosis, especially those on
cirrhosis the waiting list for a liver transplant. In 2002, liver graft allo-
D. A and C only cation was restructured to be based on the MELD score.
Although the historic indication for a liver transplant is
decompensated cirrhosis, a landmark analysis comparing
waiting list mortality with posttransplant mortality estab-
lished that a minimum MELD score of 18 is necessary to have
a survival benefit posttransplant. A MELD score between 15
and 18 does not confer a survival advantage, but a transplant
may be justified if the patient has significant morbidity from
cirrhosis. (See Schwartz 11th ed., p. 381.)

Brunicardi_Ch11_p087-096.indd 95 04/07/22 1:31 PM


This page intentionally left blank

Brunicardi_Ch11_p087-096.indd 96 04/07/22 1:31 PM


CHAPTER 12
Patient Safety

1. High reliability organization theory suggests that: Answer: D


A. Eliminating problematic individuals improves orga- High reliability organization theory suggests that proper
nizational reliability. oversight of people, processes, and technology can handle
B. Strict reporting hierarchy and rules are key to success complex and hazardous activities and keep error rates accept-
at decreasing error rates. ably low. Studies of multiple high reliability organizations
C. Low error rates are not possible in large organizations. show that they share the following common characteristics:
D. Highly reliable organizations have friendly, coopera-
• People are supportive of one another.
tive, resilient staffs where creativity and open rela-
• People trust one another.
tionships are encouraged.
• People have friendly, open relationships emphasizing
credibility and attentiveness.
• The work environment is resilient and emphasizes cre-
ativity and goal achievement, providing strong feelings of
credibility and personal trust.
Developing these characteristics is an important step toward
achieving a low error rate in any organization. (See Schwartz
11th ed., p. 399.)

2. Causes of death in the United States from the highest to Answer: D


the lowest: The most commonly cited report on the incidence of deaths
A. Heart Disease – Cancer – COPD – Medical Errors due to medical error, the 1999 Institute of Medicine (IOM)
B. Cancer – Heart Disease – Medical Errors – Suicide report, describes an incidence of 44,000 to 98,000 deaths
C. Cancer – COPD – Heart Disease – Motor Vehicles annually. However, this estimate by the IOM was not based
D. Heart Disease – Cancer – Medical Errors – COPD on primary research conducted by the IOM; rather, it was
based on two older studies conducted in 1984 and 1992. Both
studies were small and limited. In 2013, after compiling more
recent evidence from multiple sources, James estimated an
incidence range of 210,000 to 400,000 deaths a year associ-
ated with medical errors among hospital patients. Any point
estimate in this range would rank the problem of dying from
“medical care gone wrong” as the third leading cause of death
in the United States. In caring for patients and considering the
risks of tests and procedures done for borderline indications,
it is important to consider the magnitude of the problem of
patients dying from the care they receive rather than from
the disease or injury that brought them to care. (See Schwartz
11th ed., p. 398.)

97

Brunicardi_Ch12_p097-104.indd 97 05/07/22 3:32 PM


98
3. The Donabedian model of measuring quality identi- Answer: C
CHAPTER 12

fies all of the following as main types of improvements The Donabedian model of measuring quality identifies three
EXCEPT: main types of improvements: changes to organizational struc-
A. Changes to structure ture, changes in organizational processes, and changes in
B. Changes to process outcomes. Structure refers to the physical and organizational
C. Changes to culture tools, equipment, and policies that improve safety. Structural
D. Changes to outcomes measures ask, “Do the right tools, equipment, and policies
exist?” Process is the application of these tools, equipment, and
Patient Safety

policies/procedures to patients (good practices and evidence-


based medicine). Process measures ask, “Are the right tools,
policies, and equipment being used?” Outcome is the result
on patients. Outcome measures ask, “How often are patients
harmed?” In this model, structure (how care is organized)
plus process (what we do) influences patient outcomes (the
results achieved). (See Schwartz 11th ed., p. 399.)

4. Surgical Care Improvement Project (SCIP) Measures Answer: A


include: SCIP has identified three broad areas within surgery where
A. Process of care performance measures and outcome potential complications have a high incidence and cost and
measures. there is a significant opportunity for prevention: surgical site
B. The training of surgeons and staff. infections (SSIs), venous thromboembolism, and adverse car-
C. How surgeons document in operative reports. diac events. The SCIP measures aim to reduce the incidence
D. Operating room turnover. of these events during the perioperative period by advocating
the use of proven process and outcome measures. These pro-
cess and outcome measures are detailed in Table 12-1. (See
Schwartz 11th ed., p. 407.)

TABLE 12-1 The Surgical Care Improvement Project


measures
Process of care performance measures
Infection
• Prophylactic antibiotic received within 1 h before surgical incision
• Prophylactic antibiotic selection for surgical patients
• Prophylactic antibiotics discontinued within 24 h after surgery end
time (48 h for cardiac patients)
• Cardiac surgery patients with controlled 6 a.m. postoperative serum
glucose
• Surgery patients with appropriate hair removal
• Colorectal surgery patients with immediate postoperative
normothermia
Venous thromboembolism
• Surgery patients with recommended venous thromboembolism
prophylaxis ordered
• Surgery patients who received appropriate venous thromboembolism
prophylaxis within 24 h before surgery to 24 h after surgery
Cardiac events
• Surgery patients on a β-blocker prior to arrival who received a β-blocker
during the perioperative period
Proposed outcome measures
Infection
• Postoperative wound infection diagnosed during index hospitalization
Venous thromboembolism
• Intra- or postoperative pulmonary embolism diagnosed during index
hospitalization and within 30 d of surgery
• Intra- or postoperative deep vein thrombosis diagnosed during index
hospitalization and within 30 d of surgery
Cardiac events
• Intra- or postoperative acute myocardial infarction diagnosed during
index hospitalization and within 30 d of surgery
Global measures
• Mortality within 30 d of surgery
• Readmission within 30 d of surgery
Data from The Joint Commission, 2012.

Brunicardi_Ch12_p097-104.indd 98 05/07/22 3:32 PM


99
5. National Surgical Quality Improvement Program: Answer: D

CHAPTER 12
A. Is essentially the same as Surgical Care Improvement The National Surgical Quality Improvement Program
Project (SCIP). (NSQIP) is a measurement program that allows hospitals to
B. Collects data on individual surgeon outcomes. sample their rates of postoperative events and compare them
C. Was created by the Institute of Medicine. to similar hospitals. Created by the Veterans Health Admin-
D. Allows hospitals to compare their rates of postopera- istration (VA) in 1991, NSQIP has been credited with mea-
tive events and compare them to similar hospitals. suring and improving morbidity and mortality outcomes at
the VA, reducing 30-day mortality rate after major surgery by

Patient Safety
31%, and 30-day postoperative morbidity by 45% in its first
decade. Beta testing at 18 non-VA sites from 2001 to 2004
demonstrated the feasibility and utility of the program in the
private sector. The program was subsequently expanded to
the private sector in 2004. (See Schwartz 11th ed., p. 407.)

6. The root cause of the majority of wrong-site surgeries Answer: A


results from: The risk of performing wrong-site surgery increases when
A. Communication errors. there are multiple surgeons involved in the same operation
B. Emergency surgery. or multiple procedures are performed on the same patient,
C. Multiple procedures. especially if the procedures are scheduled or performed on
D. Multiple surgeons. different areas of the body. Time pressure, emergency sur-
gery, abnormal patient anatomy, and morbid obesity are also
thought to be risk factors. Communication errors are the root
cause in more than 70% of the wrong-site surgeries reported
to The Joint Commission. Other risk factors include receiving
an incomplete preoperative assessment; having inadequate
procedures in place to verify the correct surgical site; or hav-
ing an organizational culture that lacks teamwork or reveres
the surgeon as someone whose judgment should never be
questioned. (See Schwartz 11th ed., p. 411.)

7. Retained surgical items: Answer: A


A. Occur in approximately 1:1500 surgeries in the A retained surgical item refers to any surgical item found to
United States. be inside a patient after he or she has left the operating room
B. Occur more frequently in elective procedures. (OR), thus requiring a second operation to remove the item.
C. Are less likely to occur when multiple surgeons take Estimates of retained foreign bodies in surgical procedures
part in an operation. range from one case per 8000 to 18,000 operations, corre-
D. Are most frequently surgical needles. sponding to one case or more each year for a typical large
hospital or approximately 1500 cases per year in the United
States. This estimate is based on an analysis of malpractice
claims and is likely to underestimate the true incidence. The
risk of having a retained surgical item increases during emer-
gency surgery, when there are unplanned changes in pro-
cedure (due to new diagnoses encountered in the OR), and
in patients with higher body mass index (Table 12-2). (See
Schwartz 11th ed., p. 410.)

TABLE 12-2 Risk factors for retained surgical sponges


• Emergency surgery
• Unplanned changes in procedure
• Patient with higher body mass index
• Multiple surgeons involved in same operation
• Multiple procedures performed on same patient
• Involvement of multiple operating room nurses/staff members
• Case duration covers multiple nursing “shifts”

Brunicardi_Ch12_p097-104.indd 99 05/07/22 3:32 PM


100
8. Regarding complications of Central lines: Answer: C
CHAPTER 12

A. Pneumothorax occurs in 10%. Pneumothorax occurrence rates from both subclavian and
B. Pneumothorax is eliminated using ultrasound when internal jugular vein approaches are 1% to 6%. Prevention
placing the line. requires proper positioning of the patient and correct inser-
C. Central line infections are associated with significant tion technique. A postprocedure chest X-ray is recommended
mortality and increased hospital costs. to confirm the presence or absence of a pneumothorax,
D. Frequent line changes are recommended to decrease regardless of whether a pneumothorax is suspected. Recent
infection risk. reports have questioned whether a chest X-ray is required
Patient Safety

when the line is placed and confirmed under ultrasound


guidance. Pneumothorax rates are higher among inexpe-
rienced providers and underweight patients but occur with
experienced operators as well.
The Centers for Disease Control and Prevention (CDC)
reports mortality rates of 12% to 25% when a central venous
line infection becomes systemic, with a cost of approximately
$25,000 per episode. The CDC does not recommend routine
central line changes, but when the clinical suspicion of infec-
tion is high, the site of venous access must be changed. (See
Schwartz 11th ed., p. 416.)

9. Laryngoscopic findings after a superior laryngeal nerve Answer: C


injury include: Superior laryngeal nerve injury is less debilitating, as the com-
A. Ipsilateral vocal cord in a paramedian position. mon symptom is loss of projection of the voice. The glottis
B. Ipsilateral vocal cord in a middling position. aperture is asymmetrical on direct laryngoscopy, and manage-
C. Asymmetry of the glottic opening. ment is limited to clinical observation. (See Schwartz 11th ed.,
D. Normal examination. p. 419.)

10. Ventilator-associated pneumonia (VAP) in ventilated Answer: C


ICU patients reaches a 70% probability at: Pneumonia is the second most common nosocomial infec-
A. 5 days. tion and is the most common infection in ventilated patients.
B. 15 days. VAP occurs in 15% to 40% of ventilated ICU patients, with
C. 30 days. a probability rate of 5% per day, up to 70% at 30 days. The
D. 45 days. 30-day mortality rate of nosocomial pneumonia can be as
high as 40% and depends on the microorganisms involved
and the timeliness of initiating appropriate antimicrobials
Protocol-driven approaches for prevention and treatment of
VAP are recognized as beneficial in managing these difficult
infectious complications. (See Schwartz 11th ed., p. 419.)

11. Included in the definition of acute respiratory distress Answer: B


syndrome (ARDS) is: The Berlin definition of ARDS developed by the American-
A. Required increased positive end-expiratory pressure European Consensus Conference of 2012 not only simplifies
(PEEP). the definition of ARDS but also eliminates the term Acute
B. PaO2:FiO2 < 200 regardless of PEEP. Lung Injury (ALI) from critical care vernacular. ARDS is
C. Cardiac failure. now classified by partial pressure of oxygen in arterial blood
D. Respiratory failure despite normal chest X-ray. (PaO2)/fraction of inspired oxygen (Fio2) ratios as mild
(300–201 mmHg), moderate (200–101 mmHg), and severe
(<100 mmHg). Elements of modification of the definition
include the following: <7 days of onset; removal of pulmo-
nary artery occlusion pressure; and clinical judgment for
characterizing hydrostatic pulmonary edema is acceptable,
unless risk factors for ARDS have been eliminated, in which
case objective analysis is necessary.
The definition of ARDS traditionally included five crite-
ria (Table 12-3). The multicenter ARDS Research Network
(ARDSnet) research trial demonstrated improved clinical
outcomes for ARDS patients ventilated at tidal volumes of
only 5 to 7 mL/kg. This strategy is no longer prescribed solely
for patients with ARDS but is also recommended for patients

Brunicardi_Ch12_p097-104.indd 100 05/07/22 3:32 PM


101
with normal pulmonary physiology who are intubated for

CHAPTER 12
reasons other than acute respiratory failure. The beneficial
effects of PEEP for ARDS were confirmed in this study as
well. (See Schwartz 11th ed., p. 420.)

TABLE 12-3 Inclusion criteria for the acute respiratory


distress syndrome
• Acute onset

Patient Safety
• Predisposing condition
• Pao2:Fio2 <200 (regardless of positive end-expiratory pressure)
• Bilateral infiltrates
• Pulmonary artery occlusion pressure <18 mm Hg
• No clinical evidence of right heart failure
Fio2 = fraction of inspired oxygen; Pao2 = partial pressure of arterial oxygen.

12. Clinical signs of pulmonary embolism include: Answer: D


A. Hypocarbia. Clinical findings include elevated central venous pressure,
B. Hypoxemia. hypoxemia, shortness of breath, hypocarbia secondary to
C. Right heart strain on electrocardiogram (ECG). tachypnea, and right heart strain on ECG. (See Schwartz
D. All of the above. 11th ed., p. 420.)

13 Tracheostomy may facilitate weaning, and improve Answer: D


patient comfort and pulmonary toilet. Tracheostomy Tracheostomy facilitates weaning from a ventilator, may
should be performed: decrease length of ICU or hospital stay, and improves pul-
A. Open tracheostomy before the fifth day of ventilator monary toilet. Tracheostomies are performed open, percuta-
support. neously, with or without bronchoscopy, and with or without
B. Percutaneous tracheostomy before the 10th day of Doppler guidance. The advantages of percutaneous tracheos-
ventilator support. tomy include efficiency and cost containment over open trache-
C. Before the 15th day of ventilator support. ostomy. A recent literature review examining early (<3–7 days)
D. There is no difference between early <3–7 days vs late vs late (>14 days) tracheostomy after endotracheal intubation
>14 days tracheostomy. demonstrates little difference in outcomes but does demon-
strate greater patient comfort in those patients with trache-
ostomy than those with an endotracheal tube. Complications
and outcomes between the two different methods remain
largely equivalent. (See Schwartz 11th ed., p. 417.)

14. Which of the following have been shown to decrease the Answer: D
time of postoperative ileus? Postoperative ileus is related to dysfunction of the neural reflex
A. Cyclooxygenase-1 inhibitors axis of the intestine. Excessive narcotic use may delay return
B. Morphine patient-controlled analgesia of bowel function. Epidural anesthesia results in better pain
C. Nasogastric drainage until full return of bowel control, and there is an earlier return of bowel function and
function a shorter length of hospital stay. The limited use of nasogas-
D. Alvimopan, a μ -opioid receptor antagonist tric tubes and the initiation of early postoperative feeding are
associated with an earlier return of bowel function. The use
of chewing gum and other oral stimulants to minimize ileus
remains controversial. Pharmacologic agents commonly used
to stimulate bowel function include metoclopramide and
erythromycin. Metoclopramide’s action is limited to the stom-
ach and duodenum, and it may help primarily with gastropa-
resis. Erythromycin is a motilin agonist that works throughout
the stomach and bowel. Several studies demonstrate significant
benefit from the administration of erythromycin in those suf-
fering from an ileus. Alvimopan, a newer agent and a μ-opioid
receptor antagonist, has shown some promise in many studies
for earlier return of gut function and subsequent reduction in
length of stay. Neostigmine has been used in refractory pan-
ileus patients (Ogilvie’s syndrome) with some degree of success.
It is recommended for patients receiving this type of therapy to
be in a monitored unit. (See Schwartz 11th ed., p. 421.)

Brunicardi_Ch12_p097-104.indd 101 05/07/22 3:32 PM


102
15. The treatment of choice for a biloma after laparoscopic Answer: C
CHAPTER 12

cholecystectomy is: Complications involving the hepatobiliary system are usually


A. Reoperation, closure of the leak, and drainage. due to technical errors. Laparoscopic cholecystectomy has
B. Percutaneous drainage. become the standard of care for cholecystectomy, but com-
C. Biliary stent. mon bile duct injury remains a nemesis of this approach.
D. Observation. Intraoperative cholangiography has not been shown to
decrease the incidence of common bile duct injuries because
the injury to the bile duct usually occurs before the cholan-
Patient Safety

giogram. Early recognition and immediate repair of an injury


are important because delayed bile duct leaks often require a
more complex repair.
Ischemic injury due to devascularization of the common
bile duct has a delayed presentation days to weeks after an
operation. Endoscopic retrograde cholangiopancreatography
(ERCP) demonstrates a stenotic, smooth common bile duct,
and liver function studies are elevated. The recommended
treatment is a Roux-en-Y hepaticojejunostomy.
A bile leak due to an unrecognized injury to the ducts may
present after cholecystectomy as a biloma. These patients
may present with abdominal pain and hyperbilirubinemia.
The diagnosis of a biliary leak can be confirmed by CT scan,
ERCP, or radionuclide scan. Once a leak is confirmed, a ret-
rograde biliary stent and external drainage are the treatment
of choice. (See Schwartz 11th ed., p. 422.)

16. All of the following are TRUE statements regarding Answer: B


wound infection EXCEPT: No prospective, randomized, double-blind, controlled stud-
A. Irrigation of the operative field and surgical wound ies exist that demonstrate antibiotics used beyond 24 hours
with antibiotic solution is not better than using saline in the perioperative period prevent infections. Prophylactic
alone. use of antibiotics should simply not be continued beyond this
B. Prophylactic use of antibiotics continued beyond 48 time. Irrigation of the operative field and the surgical wound
hours is beneficial. with saline solution has shown benefit in controlling wound
C. Irrigation with an antibiotic-based solution has not inoculum. Irrigation with an antibiotic-based solution has
been shown to be beneficial. not demonstrated significant benefit in controlling postop-
D. Antibacterial-impregnated polyvinyl placed over the erative infection.
operative wound area for the duration of the surgical Antibacterial-impregnated polyvinyl placed over the oper-
procedure is not beneficial. ative wound area for the duration of the surgical procedure
has not been shown to decrease the rate of wound infec-
tion. Although skin preparation with 70% isopropyl alcohol
has the best bactericidal effect, it is flammable and could be
hazardous when electrocautery is used. The contemporary
formulas of chlorhexidine gluconate with isopropyl alcohol
remain more advantageous. (See Schwartz 11th ed., p. 424.)

17. The most common cause of an empyema in the postop- Answer: A


erative patient is: One of the most debilitating infections is an empyema, or
A. Pneumonia. infection of the pleural space. Frequently, an overwhelm-
B. Systemic sepsis. ing pneumonia is the source of an empyema, but a retained
C. Esophageal perforation. hemothorax, systemic sepsis, esophageal perforation from
D. Retained hemothorax. any cause, and infections with a predilection for the lung
(eg, tuberculosis) are potential etiologies as well. The diag-
nosis is confirmed by chest X-ray or CT scan, followed by
aspiration of pleural fluid for bacteriologic analysis. Gram’s
stain, lactate dehydrogenase, protein, pH, and cell count
are obtained, and broad-spectrum antibiotics are initiated
while the laboratory studies are performed. Once the specific
organisms are confirmed, anti-infective agents are tailored
appropriately. Placement of a thoracostomy tube is needed to
evacuate and drain the infected pleural fluid, but depending

Brunicardi_Ch12_p097-104.indd 102 05/07/22 3:32 PM


103
on the specific nidus of infection, video-assisted thoracos-

CHAPTER 12
copy may also be helpful for irrigation and drainage of the
infection. Refractory empyemas require specialized surgical
approaches. (See Schwartz 11th ed., p. 425.)

18. The primary cause of hyperbilirubinemia in the surgical Answer: C


patient is: Hyperbilirubinemia in the surgical patient can be a complex
A. Sepsis. problem. Cholestasis makes up the majority of causes for

Patient Safety
B. Hematoma from trauma. hyperbilirubinemia, but other mechanisms of hyperbiliru-
C. Cholestasis. binemia include reabsorption of blood (eg, hematoma from
D. Increased unconjugated bilirubin due to hemolysis. trauma), decreased bile excretion (eg, sepsis), increased
unconjugated bilirubin due to hemolysis, hyperthyroidism,
and impaired excretion due to congenital abnormalities
or acquired disease. Errors in surgery that cause hyper-
bilirubinemia largely involve missed or iatrogenic injuries.
(See Schwartz 11th ed., p. 422.)

19. A patient after a laparotomy for a perforated viscus is Answer: A


noted to develop abdominal distention, increased peak The treatment of abdominal compartment syndrome (ACS) is
airway pressure, oliguria. Bladder pressure is 23 mm Hg. to open any recent abdominal incision to release the abdomi-
Appropriate next step is: nal fascia or to open the fascia directly if no abdominal
A. Reopen a laparotomy incision. incision is present. Immediate improvement in mechanical
B. Diuresis. ventilation pressures, intracranial pressures, and urine out-
C. Institute positive end-expiratory pressure (PEEP). put is usually noted. When expectant management for ACS is
D. Rectal tube. considered in the operating room (OR), the abdominal fascia
should be left open and covered under sterile conditions (eg,
a vacuum-assisted open abdominal wound closure system)
with plans made for a second-look operation and delayed
fascial closure. Patients with intra-abdominal hypertension
should be monitored closely with repeated examinations
and measurements of bladder pressure, so that any further
deterioration is detected and operative management can be
initiated. Left untreated, ACS may lead to multiple system
end-organ dysfunction or failure and has a high mortality.
(See Schwartz 11th ed., p. 423.)

20. Retained surgical items: Answer: A


A. Occur in approximately 1:1500 surgeries in the A retained surgical item refers to any surgical item found to
United States. be inside a patient after he or she has left the operating room
B. Occur more frequently in elective procedures. (OR), thus requiring a second operation to remove the item.
C. Are less likely to occur when multiple surgeons take Estimates of retained foreign bodies in surgical procedures
part in an operation. range from one case per 8000 to 18,000 operations, correspond-
D. Are most frequently surgical needles. ing to one case or more each year for a typical large hospital
or approximately 1500 cases per year in the United States.
This estimate is based on an analysis of malpractice claims
and is likely to underestimate the true incidence. The risk of
having a retained surgical item increases during emergency
surgery, when there are unplanned changes in procedure (due
to new diagnoses encountered in the OR), and in patients with
higher body mass index (Table 12-2). (See Schwartz 11th ed.,
p. 410.)

Brunicardi_Ch12_p097-104.indd 103 05/07/22 3:32 PM


This page intentionally left blank

Brunicardi_Ch12_p097-104.indd 104 05/07/22 3:32 PM


CHAPTER 13
Physiologic Monitoring of the Surgical Patient

1. Which of the following is NOT TRUE regarding VO2 Answer: D


(oxygen utilization) and DO2 (oxygen delivery)? The relationship of VO2 to DO2 over a broad range of DO2
A. The region where the two lines intersect is DO2crit and values is commonly represented as two intersecting straight
represents the transition from supply-independent lines (Fig. 13-1). In the region of higher DO2 values, the slope
uptake to supply-dependent uptake. of the line is approximately equal to zero, indicating that VO2
B. At lower DO2, VO2 is dependent on DO2. is largely independent of DO2. In contrast, in the region of
C. At higher DO2, VO2 is independent of DO2. low DO2 values, the slope of the line is nonzero and positive,
D. Below a critical threshold of oxygen delivery, indicating that VO2 is supply-dependent. The region where
increased oxygen extraction can compensate for the the two lines intersect is called the point of critical oxygen
oxygen delivery deficit. delivery (DO2crit), and represents the transition from supply-
independent to supply-dependent oxygen uptake. Below
a critical threshold of oxygen delivery, increased oxygen
extraction cannot compensate for the delivery deficit; hence,
oxygen consumption begins to decrease. The slope of the
supply-­dependent region of the plot reflects the maximal
oxygen extraction capability of the vascular bed being evalu-
ated. (See Schwartz 11th ed., p. 440.)

Supply-dependent Supply-independent
oxygen consumption oxygen consumption
Oxygen utilization, VO2

FIG. 13-1. Graphical representation of the relationship between


oxygen utilization (VO2) and oxygen delivery (DO2). Under most
normal physiologic conditions, oxygen utilization does not
DO2crit

depend on oxygen delivery, but below the critical value DO2crit


Tissue hypoxia Tissue normoxia
oxygen utilization decreases linearly as a function of oxygen
delivery, rendering tissues susceptible to ischemic injury. Oxygen delivery, DO2

105

Brunicardi_Ch13_p105-114.indd 105 04/07/22 1:36 PM


106
2. Of the following parameters, which is the least influ- Answer: B
CHAPTER 13

enced by an underdamped or overdamped intra-arterial If the system is underdamped, then the inertia of the system,
blood pressure monitoring system? which is a function of the mass of the fluid in the tubing and
A. Systolic blood pressure the mass of the diaphragm, causes overshoot of the points
B. Mean arterial blood pressure of maximum positive and negative displacement of the dia-
C. Diastolic blood pressure phragm during systole and diastole, respectively. Thus, in an
D. Pulse pressure underdamped system, systolic pressure will be overestimated
and diastolic pressure will be underestimated. In an over-
Physiologic Monitoring of the Surgical Patient

damped system, displacement of the diaphragm fails to track


the rapidly changing pressure waveform, and systolic pressure
will be underestimated and diastolic pressure will be overes-
timated. It is important to note that even in an underdamped
or overdamped system, mean pressure will be accurately
recorded, provided the system has been properly calibrated.
For these reasons, when using direct measurement of intra-
arterial pressure to monitor patients, clinicians should make
clinical decisions based primarily on the measured mean
arterial blood pressure. (See Schwartz 11th ed., p. 435.)

3. Regarding electrocardiographic monitoring in the ICU: Answer: B


A. A standard 3-lead electrocardiogram (ECG) will Continuous monitoring of the 12-lead ECG may be beneficial
detect 95% of ischemia, whereas a 12-lead ECG will in certain patient populations. In a study of 185 vascular surgi-
detect >98%. cal patients, continuous 12-lead ECG monitoring was able to
B. Lead V4 is the most sensitive for detecting periopera- detect transient myocardial ischemic episodes in 20.5% of the
tive ischemia. patients. This study demonstrated that the precordial lead V4,
C. A standard 3-lead ECG will detect ischemia at the which is not routinely monitored on a standard 3-lead ECG,
same rate as a 12-lead ECG, however it is inferior at is the most sensitive for detecting perioperative ischemia and
identifying dysrhythmias. infarction. To detect 95% of the ischemic episodes, two or
D. Lead V2 is the most sensitive for detecting periopera- more precordial leads were necessary. Furthermore, in a pro-
tive ischemia. spective observational study, 51 peripheral artery vascular sur-
gery patients underwent ambulatory continuous 12-lead ECG
monitoring in the postoperative setting. Ischemic load, defined
as the area under the curve defined by ischemic ST-segment
deviation and ischemic time, was shown to predict periop-
erative myocardial infarction with an area under the receiver
operating characteristics curve of 0.87. Notably, ischemia was
asymptomatic in 14 of the 17 identified patients, demonstrat-
ing value of this modality as a warning tool. Thus, continu-
ous 12-lead ECG monitoring may provide greater sensitivity
than 3-lead ECG for the detection of perioperative myocardial
ischemia, and may become standard for monitoring high-risk
surgical patients. (See Schwartz 11th ed., p. 435-6.)

4. Regarding preload, which of the following is TRUE? Answer: D


A. It is approximated by the systemic vascular resistance Strictly speaking, preload is determined by end-diastolic vol-
which is calculated by dividing mean arterial pres- ume (EDV). In practice, EDV is challenging to measure pre-
sure by cardiac output. cisely during the cardiac cycle, and so clinicians utilize the
B. It is approximated by the right ventricular end- end-diastolic pressure (EDP) as a reasonable surrogate. For
diastolic pressure as estimated with pulmonary artery the right ventricle, CVP approximates right ventricular EDP.
occlusion pressure. For the left ventricle, pulmonary artery occlusion pressure
C. It is approximated by the right ventricular end- (PAOP), which is measured by transiently inflating a balloon
diastolic pressure as estimated with central venous at the end of a pressure monitoring catheter positioned in a
pressure (CVP). small branch of the pulmonary artery, approximates left ven-
D. It is approximated by the left ventricular end- tricular EDP. The presence of atrioventricular valvular steno-
diastolic pressure as estimated with pulmonary artery sis may alter this relationship.
occlusion pressure. There are limits to the utilization of EDP as a surrogate for
EDV when evaluating preload. For example, EDP is deter-
mined not only by volume but also by the diastolic compli-
ance of the ventricular chamber. Ventricular compliance is
altered by various pathologic conditions and pharmacologic

Brunicardi_Ch13_p105-114.indd 106 04/07/22 1:36 PM


107
agents. Furthermore, the relationship between EDP and true

CHAPTER 13
preload is not linear, but rather is exponential (Fig. 13-2A,B).
This fact limits the utility of EDP as a surrogate marker at
extremes of EDV. (See Schwartz 11th ed., p. 436-7.)

Changing preload
Left ventricle pressure-volume loop
(end-diastolic volume)

End-systolic

El

Physiologic Monitoring of the Surgical Patient


volume

Left ventricular pressure (mm Hg)

Left ventricular pressure (mm Hg)


as
ta
End-diastolic

nc
e
volume

lin
e
Stroke volume

e
li n

li n
R R
PV PV

line

line
ED ED

VR

VR
ESP

ESP
Left ventricular volume (mm Hg) Left ventricular volume (mm Hg)

FIG. 13-2 A-B. Left ventricular pressure-volume loops constructed for various clinically relevant
scenarios. For further information refer to the text. A. Standard left ventricular pressure-volume
loop, with stroke volume, end-systolic volume, and end-diastolic volume highlighted for reference.
Note the directionality of the pressure-volume loop, which is not annotated in the figure B for
clarity. B. Demonstration of the effect of changing preload.

5. All of the following are TRUE EXCEPT: Answer: A


A. The relationship between end-diastolic pressure Strictly speaking, preload is determined by EDV. In practice,
(EDP) and preload is linear. EDV is challenging to measure precisely during the cardiac
B. EDP is determined by both volume and compliance cycle, and so clinicians utilize the EDP as a reasonable sur-
of the ventricle. rogate. For the right ventricle, central venous pressure (CVP)
C. The relationship between EDP and end-diastolic approximates right ventricular EDP. For the left ventricle,
volume (EDV) can be changed with pharmacologic pulmonary artery occlusion pressure (PAOP), which is mea-
agents. sured by transiently inflating a balloon at the end of a pres-
D. EDP is often used as a surrogate for EDV because it is sure monitoring catheter positioned in a small branch of the
easier to approximate in the clinical setting. pulmonary artery, approximates left ventricular EDP. The
presence of atrioventricular valvular stenosis may alter this
relationship.
There are limits to the utilization of EDP as a surrogate for
EDV when evaluating preload. For example, EDP is deter-
mined not only by volume but also by the diastolic compli-
ance of the ventricular chamber. Ventricular compliance is
altered by various pathologic conditions and pharmacologic
agents. Furthermore, the relationship between EDP and true
preload is not linear, but rather is exponential (Fig. 13-2A,B).
This fact limits the utility of EDP as a surrogate marker at
extremes of EDV. (See Schwartz 11th ed., p. 436-7.)

6. Which of the following does NOT reduce complications Answer: C


associated with arterial lines? Distal ischemia is an uncommon complication of intra-­arterial
A. Using a 20-guage or smaller catheter in the radial catheterization. The incidence of thrombosis is increased
artery when larger-caliber catheters are employed and when cath-
B. Ensuring good collateral flow in the distal vascu- eters are left in place for an extended period of time. The inci-
lar bed (ie, performing an Allen’s test) before line dence of thrombosis can be minimized by using a 20-gauge
placement (or smaller) catheter in the radial artery and removing the
C. Routine exchange of catheters every 5 to 7 days to catheter as soon as feasible. The risk of distal ischemic injury
prevent line-associated infections can be reduced by ensuring that adequate collateral flow is
D. Avoid flushing the arterial line when air is present in present prior to catheter insertion. At the wrist, adequate col-
the system, and only use a small volume (ie, <5 mL) lateral flow can be documented by performing a modified
for flushing the line version of the Allen test, wherein the artery to be cannulated

Brunicardi_Ch13_p105-114.indd 107 04/07/22 1:36 PM


108
is digitally compressed while using a Doppler stethoscope to
CHAPTER 13

listen for perfusion in the palmar arch vessels.


Another potential complication of intra-arterial monitor-
ing is retrograde embolization of air bubbles or thrombi into
the intracranial circulation. In order to minimize this risk
care should be taken to avoid flushing arterial lines when
air is present in the system, and only small volumes of fluid
(<5 mL) should be employed for this purpose. Catheter-
Physiologic Monitoring of the Surgical Patient

related infections can occur with any intravascular monitor-


ing device. However, catheter-related bloodstream infection
is a relatively uncommon complication of intra-arterial lines
used for monitoring, occurring in 0.4% to 0.7% of catheter-
izations. The incidence increases with longer duration of arte-
rial catheterization. (See Schwartz 11th ed., p. 435.)

7. The thermodilution technique for determining cardiac Answer: D


output: The relationship used for calculating QT is called the Stewart-
A. Calculates QT with the Fick equation. Hamilton equation:
B. Underestimates cardiac output at low values.
VK1 K 2 (TB − TI )
C. Should be performed with a cold indicator liquid to QT =
increase the signal-to-noise ratio. ∫ T (t )dt
B
D. Is influenced by respiratory cycle due to changes in
blood temperature and QT. where V is the volume of the indicator injected, TB is the
temperature of blood (ie, core body temperature), TI is the
temperature of the indicator, K1 is a constant that is the func-
tion of the specific heats of blood and the indicator, K2 is an
empirically derived constant that accounts for several fac-
tors (the dead space volume of the catheter, heat lost from
the indicator as it traverses the catheter, and the injection
rate of the indicator), and ∫TB(t)dt is the area under the time-
temperature curve. In clinical practice, the Stewart-Hamilton
equation is solved by a microprocessor.
Determination of cardiac output by the thermodilution
method is generally quite accurate, although it tends to sys-
tematically overestimate QT at low values. Changes in blood
temperature and QT during the respiratory cycle can influ-
ence the measurement. Therefore, results generally should be
recorded as the mean of two or three determinations obtained
at random points in the respiratory cycle. Using cold injec-
tate widens the difference between TB and TI and thereby
increases signal-to-noise ratio. Nevertheless, most authorities
recommend using room temperature injectate (normal saline
or 5% dextrose in water) to minimize errors resulting from
warming of the fluid as it transferred from its reservoir to a
syringe for injection. (See Schwartz 11th ed., p. 439.)

8. Software creating integrated monitoring systems (such Answer: C


as the Modified Early Warning Score [MEWS] or Roth- The Rothman Index (RI) is a proprietary data analysis tool-
man Index [RI] System) have been found to: kit encompassing a total of 26 variables including vital signs,
A. Alert teams to clinical deterioration preceding a car- nursing assessments, laboratory test values, and cardiac
diac or pulmonary arrest. rhythms and was developed to make use of the vast amount
B. Predict re-admission to the surgical intensive care of data input into the electronic medical record (EMR) on a
unit. real-time basis to help provide a global assessment of patient
C. Predict whether the patient will survive or not sur- status. In the initial derivation, Rothman and colleagues
vive a cardiac or pulmonary arrest. demonstrated concordance of the RI with the Modified Early
D. Predict Rapid Response Team (RRT) activation. Warning Score (MEWS) system, which is designed to alert
medical teams to clinical deterioration that precedes cardiac
or pulmonary arrest events. Subsequent publications evalu-
ated performance of the RI in predicting both readmission to
surgical ICUs in the postoperative setting as well as for rapid

Brunicardi_Ch13_p105-114.indd 108 04/07/22 1:36 PM


109
response team activations. Although more work is required

CHAPTER 13
to evaluate the broad applicability of the RI and similar
measures, the evidence to date is compelling. (See Schwartz
11th ed., p. 436.)

9. Which of the following steps incorrectly describes a step Answer: D


needed for placement of a pulmonary artery catheter? Cannulation of the vein is normally performed percutaneously,
A. Cannulation of the vein via percutaneous technique using the Seldinger technique. A small-bore needle is inserted

Physiologic Monitoring of the Surgical Patient


B. Inflate the balloon once respiratory excursions are through the skin and subcutaneous tissue into the vein. After
observed on the monitor documenting return of venous blood, a guidewire with a flex-
C. With the balloon inflated, constant observation of ible tip is inserted through the needle into the vein, and the
the pressure tracing in the right atrium, right ventri- needle is withdrawn. A dilator/introducer sheath is passed
cle, and pulmonary artery as the catheter is advanced over the wire, and the wire and the dilator are removed. The
D. Leaving the balloon inflated in the pulmonary artery proximal terminus of the distal port of the pulmonary artery
for constant pulmonary capillary wedge pressure catheter (PAC) is connected through low-compliance tubing
measurement to a strain-gauge transducer, and the tubing-catheter system
is flushed with fluid. While constantly observing the pressure
tracing on a monitor screen, the PAC is advanced with the bal-
loon deflated until respiratory excursions are observed. The bal-
loon is then inflated, and the catheter advanced further, while
monitoring pressures sequentially in the right atrium and right
ventricle en route to the pulmonary artery. The pressure wave-
forms for the right atrium, right ventricle, and pulmonary artery
are each characteristic (Fig. 13-3). The catheter is advanced out
the pulmonary artery until a damped tracing indicative of the
“wedged” position is obtained. The balloon is then deflated, tak-
ing care to ensure that a normal pulmonary arterial tracing is
again observed on the monitor; leaving the balloon inflated can
increase the risk of pulmonary infarction or perforation of the
pulmonary artery. Unnecessary measurements of the pulmo-
nary artery occlusion pressure are discouraged as rupture of the
pulmonary artery may occur. (See Schwartz 11th ed., pp. 437–8.)

Balloon
inflated

Right atrium Right ventricle Pulmonary artery Pulmonary artery occlusion


Pressure (mm Hg)

FIG. 13-3. Representative pressure traces at different stages of insertion of the PAC. In the
central venous circulation, the pressure remains low, with characteristic waves from atrial filling
and tricuspid valve closing. Upon entry into the right ventricle, the pressure increases sharply,
with the broadest range between systole and diastole. When in the main pulmonary artery,
the systolic pressure remains elevated to the same degree, but the diastolic pressure is now
significantly elevated due to the closure of the pulmonic valve during the cardiac cycle. Upon
further advancement with the balloon inflated, the pressure differences become smaller and the
magnitude of the mean pressure drops, reflecting an estimate of the left atrial pressure.

Brunicardi_Ch13_p105-114.indd 109 04/07/22 1:36 PM


110
10. Which of the following does NOT affect the fractional Answer: C
CHAPTER 13

saturation of hemoglobin in venous blood (SvO2)? Oxygen saturation can replace oxygen content, yielding the
A. Anemia final clinically valuable equation:
B. Fever
C. Age VO2
S v O 2 = Sa O 2 −
D. Heart failure (Q T × Hgb × 1.36)

where SVO2 is the fractional saturation of hemoglobin in mixed


Physiologic Monitoring of the Surgical Patient

venous blood, SaO2 is the fractional saturation of hemoglo-


bin in arterial blood, and Hgb is the concentration of hemo-
globin in blood. Thus, it can be seen that SVO2 is a function
of VO2 (ie, metabolic rate), QT, SaO2, and Hgb. Accordingly,
subnormal values of SVO2 can be caused by a decrease
in QT (due, eg, to heart failure or hypovolemia), a
decrease in SaO2 (due, eg, to intrinsic pulmonary disease),
a decrease in Hgb (ie, anemia), or an increase in metabolic rate
(due, eg, to seizures or fever). (See Schwartz 11th ed., p. 440.)

11. Using pulse pressure variability (PPV) to determine pre- Answer: B


load responsiveness: When intrathoracic pressure increases during the applica-
A. Is reliable for a patient in rate-controlled atrial fibril- tion of positive airway pressure in mechanically ventilated
lation, but not for a patient in atrial flutter. patients, venous return decreases, and as a consequence, left
B. Is a better predictor of preload responsiveness than ventricular stroke volume (LVSV) also decreases. Therefore,
central venous pressure (CVP). PPV during a positive pressure episode can be used to predict
C. Defines PPV as the difference between the maxi- the responsiveness of cardiac output to changes in preload.
mal pulse pressure and the minimum pulse pressure PPV is defined as the difference between the maximal pulse
observed at different points in the respiratory cycle. pressure and the minimum pulse pressure divided by the
D. Is unreliable in mechanically ventilated patients due average of these two pressures (Fig. 13-4). This approach has
to decreased venous return during inspiration. validated this by comparing PPV, CVP, PAOP, and systolic
pressure variation as predictors of preload responsiveness in a
cohort of critically ill patients. Patients were classified as being
“preload responsive” if their cardiac index increased by at least
15% after rapid infusion of a standard volume of intravenous
fluid. Receiver-operating characteristic (ROC) curves dem-
onstrated that PPV was the best predictor of preload respon-
siveness. Although atrial arrhythmias can interfere with the
usefulness of this technique, PPV remains a useful approach
for assessing preload responsiveness in most patients because
of its simplicity and reliability. (See Schwartz 11th ed., p. 444.)

Inspiration Expiration Inspiration Expiration Inspiration

PPmax PPmax – PPmin


Arterial blood pressure (mm Hg)

PPV (%) = × 100


PPmax + PPmin
2

PPmin

Time

FIG. 13-4. Calculation of pulse pressure variation as it would appear on bedside monitor. This
provides a helpful and rapid assessment of fluid responsiveness in the critically ill mechanically
ventilated patient.

Brunicardi_Ch13_p105-114.indd 110 04/07/22 1:36 PM


111
12. Which of the following does not affect SaO2 (oxygen sat- Answer: D

CHAPTER 13
uration of hemoglobin in arterial blood) in mechanically DO2 is dependent to a greater degree on the oxygen satura-
ventilated patients? tion of hemoglobin (Hgb) in arterial blood (SaO2) than on the
A. Mean airway pressure partial pressure of oxygen in arterial blood (PaO2). DO2 also
B. FiO2 (fraction of inspired oxygen) is dependent on QT and Hgb. As discussed earlier and illus-
C. SvO2 (oxygen saturation of hemoglobin in venous trated mathematically by previous equations, the dissolved
blood) oxygen in blood makes only a negligible contribution to DO2.
D. Respiratory rate SaO2 in mechanically ventilated patients depends on the mean

Physiologic Monitoring of the Surgical Patient


airway pressure, the fraction of inspired oxygen (FiO2), and
SvO2. Thus, when SaO2 is low, the clinician has only a lim-
ited number of ways to improve this parameter. The clinician
can increase mean airway pressure by increasing positive-end
expiratory pressure (PEEP) or inspiratory time. FiO2 can be
increased to a maximum of 1.0 by decreasing the amount of
room air mixed with the oxygen supplied to the ventilator.
SvO2 can be increased by increasing Hgb or QT or decreas-
ing oxygen utilization (eg, by administering a muscle relaxant
and sedation). (See Schwartz 11th ed., p. 445.)

13. Which of the following regarding Ppeak (peak airway pres- Answer: A
sure) is true? Airway pressures are routinely monitored in mechanically
A. Ppeak is measured at the end of inspiration. ventilated patients. The peak airway pressure measured at
B. To measure Ppeak, the expiratory valve must be closed. the end of inspiration (Ppeak) is a function of the tidal volume,
C. Ppeak is independent of airway resistance, chest wall the resistance of the airways, lung/chest wall compliance, and
compliance, and peak inspiratory flow. peak inspiratory flow. The airway pressure measured at the
D. Ppeak is not routinely measured in mechanically venti- end of inspiration when the inhaled volume is held in the
lated patients. lungs by briefly closing the expiratory valve is termed the pla-
teau airway pressure (Pplateau). As a static parameter, plateau
airway pressure is independent of the airway resistance and
peak airway flow and is related to the lung/chest wall com-
pliance and delivered tidal volume. Mechanical ventilators
monitor Ppeak with each breath and can be set to trigger an
alarm if the Ppeak exceeds a predetermined threshold. Pplateau
is not measured routinely with each delivered tidal volume
but rather is measured intermittently by setting the ventilator
to close the exhalation circuit briefly at the end of inspira-
tion and record the airway pressure when airflow is zero. (See
Schwartz 11th ed., p. 446.)

14. Causes of an increase in end-tidal-CO2 include: Answer: D


A. Massive pulmonary embolism. Continuous monitoring with capnography has become
B. Reduced cardiac output. routine during surgery under general anesthesia and for
C. Sustained hyperventilation. some intensive care patients. A number of situations can be
D. Reduced minute ventilation. promptly detected with continuous capnography. A sudden
reduction in Petco2 suggests either obstruction of the sam-
pling tubing with water or secretions, or a catastrophic event
such as loss of the airway, airway disconnection or obstruc-
tion, ventilator malfunction, or a marked decrease in QT. If
the airway is connected and patent and the ventilator is func-
tioning properly, then a sudden decrease in Petco2 should
prompt efforts to rule out cardiac arrest, massive pulmonary
embolism, or cardiogenic shock. Petco2 can be persistently
low during hyperventilation or with an increase in dead space
such as occurs with pulmonary embolization (even in the
absence of a change in QT). Causes of an increase in Petco2
include reduced minute ventilation or increased metabolic
rate. (See Schwartz 11th ed., p. 447.)

Brunicardi_Ch13_p105-114.indd 111 04/07/22 1:36 PM


112
15. Which of the following is NOT an indication for intra- Answer: C
CHAPTER 13

cranial pressure monitoring? Monitoring of intracranial pressure (ICP) is currently recom-


A. Glasgow Coma Scale (GCS) ≤ 8 with an abnormal mended in patients with severe TBI, defined as a GCS score
computed tomography (CT) scan ≤ 8 with an abnormal CT scan, and in patients with severe
B. Severe traumatic brain injury (TBI) in a patient with TBI and a normal CT scan if two or more of the following
age > 40 years and systolic blood pressure < 90 mm Hg are present: age > 40 years, unilateral or bilateral motor pos-
C. Intracranial hemorrhage without intraventricular turing, or systolic blood pressure < 90 mm Hg. ICP moni-
blood toring also is indicated in patients with acute subarachnoid
Physiologic Monitoring of the Surgical Patient

D. Fulminant hepatic failure with coma and cerebral hemorrhage with coma or neurologic deterioration, intracra-
edema on CT nial hemorrhage with intraventricular blood, ischemic mid-
dle cerebral artery stroke, fulminant hepatic failure with coma
and cerebral edema on CT scan, and global cerebral ischemia
or anoxia with cerebral edema on CT scan. The goal of ICP
monitoring is to ensure that cerebral perfusion pressure (CPP)
is adequate to support perfusion of the brain. CPP is equal to
the difference between MAP and ICP: CPP = MAP – ICP.
(See Schwartz 11th ed., pp. 447–8.)

16. Currently accepted uses of transcranial Doppler (TCD) Answer: B


include all of the following EXCEPT: TCD measurements of middle and anterior cerebral artery
A. Diagnosing vasospasm after subarachnoid blood flow velocity are useful for the diagnosis of cerebral
hemorrhage. vasospasm after subarachnoid hemorrhage. Qureshi and
B. Estimating cerebral perfusion pressure. associates demonstrated that an increase in the middle cere-
C. Confirming brain death after clinical examination in bral artery mean flow velocity as assessed by TCD is an inde-
patients under the influence of central nervous sys- pendent predictor of symptomatic vasospasm in a prospective
tem (CNS) depressants. study of patients with aneurysmal subarachnoid hemorrhage.
D. Confirming brain death after clinical examination in In addition, while some have proposed using TCD to esti-
patients with metabolic encephalopathy. mate intracranial pressure (ICP), studies have shown that
TCD is not a reliable method for estimating ICP and cerebral
perfusion pressure (CPP) and currently cannot be endorsed
for this purpose. TCD also is useful to confirm the clinical
examination for determining brain death in patients with
confounding factors such as the presence of CNS depressants
or metabolic encephalopathy. (See Schwartz 11th ed., p. 448.)

17. Excessive pressure on the lung parenchyma is known to Answer: C


cause all of the following EXCEPT: Ventilator-induced lung injury (VILI) is now an established
A. Diffuse alveolar damage similar to ARDS. clinical entity of great relevance to the care of critically ill
B. Impaired venous return. patients. Excessive airway pressure and tidal volume adversely
C. Improved oxygenation. affect pulmonary and possibly systemic responses to critical
D. Pneumothorax. illness. Subjecting the lung parenchyma to excessive pressure,
known as barotrauma, can result in parenchymal lung injury,
diffuse alveolar damage similar to ARDS, and pneumothorax,
and can impair venous return and therefore limit cardiac out-
put. Lung-protective ventilation strategies have been devel-
oped to prevent the development of VILI and improve patient
outcomes. (See Schwartz 11th ed., p. 446.)

18. Which of the following does NOT support a diagnosis of Answer: C


abdominal compartment syndrome (ACS)? The triad of oliguria, elevated peak airway pressures, and ele-
A. Oliguria vated intra-abdominal pressure is known as abdominal com-
B. Elevated peak airway pressures partment syndrome (ACS). This syndrome, first described in
C. Intra-abdominal pressure (IAP) consistently > 20 mm patients after repair of ruptured abdominal aortic aneurysm,
Hg, recorded by measurements 30 minutes apart is associated with interstitial edema of the abdominal organs,
D. Intra-abdominal pressure (IAP) consistently > 20 mm resulting in elevated IAP. When IAP exceeds venous or cap-
Hg, with measurements 1 to 2 hours apart illary pressures, perfusion of the kidneys and other intra-
abdominal viscera is impaired. Oliguria is a cardinal sign.
While the diagnosis of ACS is a clinical one, measuring IAP
is useful to confirm the diagnosis. Ideally, a catheter inserted

Brunicardi_Ch13_p105-114.indd 112 04/07/22 1:36 PM


113
into the peritoneal cavity could measure IAP to substanti-

CHAPTER 13
ate the diagnosis. In practice, transurethral bladder pressure
measurement reflects IAP and is most often used to confirm
the presence of ACS. After instilling 50 to 100 mL of sterile
saline into the bladder via a Foley catheter, the tubing is con-
nected to a transducing system to measure bladder pressure
in the supine position at end-expiration.
Intra-abdominal hypertension is defined as an IAP ≥ 12 mm

Physiologic Monitoring of the Surgical Patient


Hg recorded on three standard measurements conducted 4 to
6 hours apart and is separated into several grades. The diag-
nosis of ACS is the presence of an IAP ≥ 20 mm Hg recorded
by three measurements 1 to 6 hours apart, along with new
onset of organ dysfunction (Table 13-1). Less commonly,
gastric or inferior vena cava pressures can be monitored with
appropriate catheters to detect elevated intra-abdominal
pressures. (See Schwartz 11th ed., p. 447.)

TABLE 13–1 Bladder pressure measurements in the


assessment of intra-abdominal hypertension
or abdominal compartment syndrome
Recorded Pressure (mm Hg) Grade of IAH or ACS
5–7 Normal
In the absence of organ dysfunction:
12–15 Grade I IAH
16–20 Grade II IAH
21–25 Grade III IAH
>25 Grade IV IAH
In the presence of new-onset organ dysfunction:
>20 ACS
Data from Kirkpatrick AW, Roberts DJ, De Waele J, et al: Intraabdominal hypertension
and the abdominal compartment syndrome: updated consensus definitions and clinical
practice guidelines from the World Society of the Abdominal Compartment Syndrome,
Intensive Care Med. 2013;39(7):1190-1206.

Brunicardi_Ch13_p105-114.indd 113 04/07/22 1:36 PM


This page intentionally left blank

Brunicardi_Ch13_p105-114.indd 114 04/07/22 1:36 PM


CHAPTER 14
Minimally Invasive Surgery

1. The most common arrhythmia seen during laparos- Answer: D


copy is: The pressure effects of the pneumoperitoneum on cardiovas-
A. Atrial fibrillation. cular physiology also have been studied. In the hypovolemic
B. Sinus tachycardia. individual, excessive pressure on the inferior vena cava and a
C. Premature ventricular contractions. reverse Trendelenburg position with loss of lower extremity
D. Sinus bradycardia. muscle tone may cause decreased venous return and decreased
cardiac output. This is not seen in the normovolemic patient.
The most common arrhythmia created by laparoscopy is bra-
dycardia. A rapid stretch of the peritoneal membrane often
causes a vagovagal response with bradycardia and, occasion-
ally, hypotension. The appropriate management of this event
is desufflation of the abdomen, administration of vagolytic
agents (eg, atropine), and adequate volume replacement. (See
Schwartz 11th ed., p. 455.)

2. A 23-year-old man presents to the emergency depart- Answer: C


ment with a 17-hour history of abdominal pain that The pressure effects of the pneumoperitoneum on cardiovas-
started in the periumbilical region, now located in the cular physiology also have been studied. In the hypovolemic
RLQ, associated with fever, nausea, WBC 14,000, and individual, excessive pressure on the inferior vena cava and a
imaging consistent with acute appendicitis. You consent reverse Trendelenburg position with loss of lower extremity
the patient for a laparoscopic appendectomy. During ini- muscle tone may cause decreased venous return and decreased
tial insufflation of the abdomen the heart rate goes down cardiac output. This is not seen in the normovolemic patient.
to 36 and the patient becomes hypotensive. What is the The most common arrhythmia created by laparoscopy is bra-
best next step in the management of this patient? dycardia. A rapid stretch of the peritoneal membrane often
A. Give 1L of crystalloids causes a vasovagal response with bradycardia and, occasion-
B. Immediate IV atropine ally, hypotension. The appropriate management of this event
C. Desufflation of the abdomen is desufflation of the abdomen, administration of vagolytic
D. Evaluate for hemorrhage agents (eg, atropine), and adequate volume replacement. (See
Schwartz 11th ed., p. 455.)

3. A patient undergoing laparoscopic colon resection is Answer: D


noted to have decreased urine output during the last Although the effects of the pneumoperitoneum on renal
hour of the case. A bolus is given at the end of the case. blood flow are immediately reversible, the hormonally medi-
One hour later, there is still low urine output. The appro- ated changes such as elevated antidiuretic hormone levels
priate treatment is: decrease urine output for up to 1 hour after the procedure has
A. Repeat bolus. ended. Intraoperative oliguria is common during laparoscopy,
B. Intravenous (IV) furosemide. but the urine output is not a reflection of intravascular vol-
C. Check urine electrolytes. ume status; IV fluid administration during an uncomplicated
D. Observe. laparoscopic procedure should not be linked to urine output.

115

Brunicardi_Ch14_p115-118.indd 115 04/07/22 1:39 PM


116
Because insensible fluid losses through the open abdomen are
CHAPTER 14

eliminated with laparoscopy, the need for supplemental fluid


during a laparoscopic surgical procedure should only keep
up with venous pooling in the lower limbs, third-space losses
into the bowel, and blood loss, which is generally less than
occurs with an equivalent open operation. (See Schwartz 11th
ed., p. 456.)
Minimally Invasive Surgery

4. While performing a laparoscopic Nissen fundoplica- Answer: B


tion, during the transhiatal dissection the mediastinal When a pneumothorax occurs with laparoscopic Nissen fun-
pleura is compromised and a CO2 pneumothorax devel- doplication or Heller myotomy, it is preferable to place an
ops. What is the initial preferred management of the 18-French red rubber catheter with multiple side holes cut
pneumothorax? out of the distal end across the defect. At the end of the proce-
A. Needle thoracostomy over the second intercostal dure, the distal end of the tube is pulled out a 10-mm port site
space, mid-clavicular line. (as the port is removed), and the pneumothorax is evacuated
B. Enlargement of the defect and placement of an to a primitive water seal using a bowl of sterile water or saline.
18-French red rubber catheter across the defect. During laparoscopic esophagectomy, it is preferable to leave
C. Abort the procedure and emergent tube thoracos- a standard chest tube, as residual intra-abdominal fluid will
tomy with a 28-French chest tube. tend to be siphoned through the defect postoperatively if the
D. No intervention is needed. Continue with the planed tube is removed at the end of the case. (See Schwartz 11th ed.,
procedure. p. 456.)

5. During pneumoperitoneum, the increased intra-abdom- Answer: B


inal pressure directly and indirectly results in: Increased intra-abdominal pressure decreases renal blood
A. Increased renal blood flow. flow, glomerular filtration rate, and urine output. These
B. Decreased urine output. effects may be mediated by direct pressure on the kidney and
C. Decreased levels of renin. the renal vein. The secondary effect of decreased renal blood
D. Decreased sodium retention. flow is to increase plasma renin release, thereby increasing
sodium retention. (See Schwartz 11th ed., p. 456.)

6. Which type of radiofrequency electrosurgery mode has Answer: B


the highest risk for thermal injury? A short-duration, high-voltage discharge of current (coagula-
A. Bipolar coagulation tion current) provides extremely rapid tissue heating. Lower-
B. Monopolar coagulation voltage, higher-wattage current (cutting current) is better
C. Monopolar cutting for tissue desiccation and vaporization. When the surgeon
D. Monopolar blended desires tissue division with the least amount of thermal injury
and least coagulation necrosis, a cutting current is used.
With bipolar electrosurgery, the electrons flow between
two adjacent electrodes. The tissue between the two elec-
trodes is heated and desiccated. There is little opportunity
for tissue cutting when bipolar current is used alone, but the
ability to coapt the electrodes across a vessel provides the best
method of small-vessel coagulation without thermal injury to
adjacent tissues. (See Schwartz 11th ed., p. 465.)

7. Which of the following represents advantages from Answer: D


robotic surgery compared to laparoscopic surgery? The major revolution in robotic surgery was the develop-
A. Increased dexterity ment of a master-slave surgical platform that returned the
B. Tremor elimination wrist to laparoscopic surgery and improved manual dexter-
C. Scaling movement ity by developing an ergonomically comfortable work station,
D. All of the above with 3-D imaging, tremor elimination, and scaling of move-
ment (eg, large, gross hand movements can be scaled down to
allow suturing with microsurgical precision) (Fig. 14-1). (See
Schwartz 11th ed., p. 467.)

Brunicardi_Ch14_p115-118.indd 116 04/07/22 1:39 PM


117

CHAPTER 14
Minimally Invasive Surgery
FIG. 14-1. Robotic instruments and hand controls. The
surgeon is in a sitting position, and the arms and wrists
are in an ergonomic and relaxed position.

8. To date, what is the only surgery that has demonstrated Answer: C


better outcomes using robotic versus laparoscopic or The tidal wave of enthusiasm for robotic surgery came
open techniques? when most minimally invasive urologists declared robotic
A. Nissen fundoplication prostatectomy to be preferable to laparoscopic and open
B. Cholecystectomy prostatectomy. The great advantage—it would appear—of
C. Prostatectomy robotic prostatectomy is the ability to visualize and spare the
D. Roux-en-Y gastric bypass pelvic nerves responsible for erectile function. (See Schwartz
11th ed., p. 467.)

9. Which of the following are TRUE regarding safe laparo- Answer: D


scopic surgery in pregnancy? Concerns about the safety of laparoscopic cholecystectomy or
A. The patient should be positioned slightly on the left appendectomy in the pregnant patient have been thoroughly
lateral position. investigated and are readily managed. Access to the abdo-
B. Open abdominal access (Hasson) is recommended men in the pregnant patient should take into consideration
versus direct puncture laparoscopy (Veress needle). the height of the uterine fundus, which reaches the umbilicus
C. The surgery should be performed during the second at 20 weeks. In order not to damage the uterus or its blood
trimester if possible. supply, most surgeons feel that the open (Hasson) approach
D. All of the above. should be used in favor of direct puncture laparoscopy. The
patient should be positioned slightly on the left side to avoid
compression of the vena cava by the uterus. Because preg-
nancy poses a risk for thromboembolism, sequential com-
pression devices are essential for all procedures. Fetal acidosis
induced by maternal hypercarbia also has been raised as a
concern. (See Schwartz 11th ed., p. 473.)

Brunicardi_Ch14_p115-118.indd 117 04/07/22 1:39 PM


This page intentionally left blank

Brunicardi_Ch14_p115-118.indd 118 04/07/22 1:39 PM


CHAPTER 15
Molecular and Genomic Surgery

1. The process that occurs during translational control of Answer: A


eukaryotic gene expression is: See Schwartz 11th ed., Figure 15-4, p. 483.
A. Protein degradation.
B. RNA processing.
C. Posttranslational control.
D. Transcription.

2. In the transcription of prokaryotes, binding of RNA Answer: A


polymerase to the specific promoter region is achieved Initiation of transcription in prokaryotes (bacteria) begins
by: with the recognition of DNA sequences by RNA polymerase.
A. Sigma factors First, the bacterial RNA polymerase catalyzes RNA synthesis
B. Operon through loose binding to any region in the double-stranded
C. Elongation factors DNA and then through specific binding to the promoter
D. Rho factors region with the assistance of accessory proteins called σ fac-
tors (sigma factors). A promoter region is the DNA region
upstream of the transcription initiation site. RNA poly-
merase binds tightly at the promoter sites and causes the
double-stranded DNA structure to unwind. Consequently,
few nucleotides can be base-paired with the DNA template to
begin transcription. Once transcription begins, the σ factor
is released. The growing RNA chain may begin to peel off as
the chain elongates. This occurs in such a way that there are
always about 10 to 12 nucleotides of the growing RNA chains
that are base-paired with the DNA template. (See Schwartz
11th ed., Figure 15-4, p. 483.)

3. All of the following transcription mechanisms occur in Answer: C


eukaryotes EXCEPT: The unique features of eukaryotic transcription are as follows:
A. Chromatin structure changes to allow DNA to be (a) Three separate RNA polymerases are involved in eukary-
accessible to the polymerase. otes: RNA polymerase I transcribes the precursor of 5.8S, 18S,
B. Three separate RNA polymerases are involved. and 28S rRNAs; RNA polymerase II synthesizes the precur-
C. Proteins or initiation factors are not required. sors of mRNA as well as microRNA; and RNA polymerase
D. Often packaged with histone and nonhistone pro- III makes tRNAs and 5S rRNAs. (b) In eukaryotes, the initial
teins into chromatins. transcript is often the precursor to final mRNAs, tRNAs, and
rRNAs. The precursor is then modified and/or processed into
its final functional form. RNA splicing is one type of process-
ing to remove the noncoding introns (the region between
coding exons) on an mRNA. (c) In contrast to bacterial DNA,
eukaryotic DNA often is packaged with histone and nonhis-
tone proteins into chromatins. Transcription will only occur
when the chromatin structure changes in such a way that

119

Brunicardi_Ch15_p119-126.indd 119 04/07/22 5:46 PM


120
DNA is accessible to the polymerase. (d) RNA is made in
CHAPTER 15

the nucleus and transported into cytoplasm, where transla-


tion occurs. Therefore, unlike bacteria, eukaryotes undergo
uncoupled transcription and translation. (See Schwartz 11th
ed., Figure 15-4, p. 483.)

4. Which of the following statements is incorrect? Answer: D


A. A codon is a triplet of three bases that codes for a A codon, a triplet of three bases, codes for one amino acid. In
Molecular and Genomic Surgery

single amino acid. this case, random combinations of the four bases form 4 ×
B. More than one triplet codes for the same amino acid. 4 × 4, or 64 codes. Because 64 codes are more than enough
C. Codons are mRNA sequentially recognized by tRNA for 20 amino acids, most amino acids are coded by more than
adaptor proteins. one codon. The start codon is AUG, which also corresponds
D. Protein synthesis proceeds in the carboxy-to-amino- to methionine; therefore, almost all proteins begin with this
terminus direction. amino acid. The sequence of nucleotide triplets that follows
the start codon signal is termed the reading frame. The codons
on mRNA are sequentially recognized by tRNA adaptor pro-
teins. Specific enzymes termed aminoacyl-tRNA synthetases
link a specific amino acid to a specific tRNA. The translation
of mRNA to protein requires the ribosomal complex to move
stepwise along the mRNA until the initiator methionine
sequence is identified. In concert with various protein initia-
tor factors, the methionyl-tRNA is positioned on the mRNA
and protein synthesis begins. Each new amino acid is added
sequentially by the appropriate tRNA in conjunction with
proteins called elongation factors. Protein synthesis proceeds
in the amino-to-carboxy-terminus direction. (See Schwartz
11th ed., p. 484.)

5. The process of decoding information on mRNA to syn- Answer: B


thesize proteins is called: DNA directs the synthesis of RNA; RNA in turn directs the
A. Transcription. synthesis of proteins. Proteins are variable-length polypep-
B. Translation. tide polymers composed of various combinations of 20 dif-
C. Replication. ferent amino acids and are the working molecules of the cell.
D. Signaling. The process of decoding information on mRNA to synthesize
proteins is called translation (see Fig. 15-1). Translation takes
place in ribosomes composed of rRNA and ribosomal pro-
teins. (See Schwartz 11th ed., p. 483.)

Nucleus Cytoplasm
DNA mRNA Protein
turnover turnover

Nuclear envelope
Transcription RNA Protein
degradation degradation
RNA RNA Posttranslational Active
transcript mRNA mRNA Translation Protein
processing modification protein

RNA
transport

Transcriptional Posttranscriptional Translational Posttranslational


control control control control

FIG. 15-1. Four major steps in the control of eukaryotic gene expression. Transcriptional and posttranscriptional control determine the level
of messenger RNA (mRNA) that is available to make a protein, while translational and posttranslational control determine the final outcome
of functional proteins. Note that posttranscriptional and posttranslational controls consist of several steps.

Brunicardi_Ch15_p119-126.indd 120 04/07/22 5:46 PM


121
6. The human genome contains approximately: Answer: C

CHAPTER 15
A. 35,000 to 40,000 genes. The human genome has an estimated 25,000 to 30,000 genes,
B. 20,000 to 25,000 genes. and overall it is 99.9% identical in all people. Approximately
C. 25,000 to 30,000 genes. 3 million locations where single-base DNA differences exist have
D. 30,000 to 35,000 genes. been identified and termed single nucleotide polymorphisms.
Single nucleotide polymorphisms may be critical determinants
of human variation in disease susceptibility and responses to
environmental factors. (See Schwartz 11th ed., p. 485.)

Molecular and Genomic Surgery


7. If chronic kidney disease (CKD) is to a cell as an engine Answer: B
is to a car, then cyclins and cyclin-dependent kinase The cell cycle is connected with signal transduction pathways
inhibitors.(CKI) are: as well as gene expression. Although the S and M phases rarely
A. The key and ignition, respectively. are subjected to changes imposed by extracellular signals, the
B. The gas pedal and brakes, respectively. G1 and G2 phases are the primary periods when cells decide
C. The distributor and the spark plug, respectively. whether to move on to the next phase. During the G1 phase,
D. The windows and the tires, respectively. cells receive green- or red-light signals, S phase entry or G1
arrest, respectively. Growing cells proliferate only when sup-
plied with appropriate mitogenic growth factors. Cells become
committed to entry of the cell cycle only toward the end of
G1. Mitogenic signals stimulate the activity of early G1 CDKs
(eg, cyclin D/CDK4) that inhibit the activity of pRb protein
and activate the transcription factor called E2F to induce the
expression of batteries of genes essential for G1-S progression.
Meanwhile, cells also receive antiproliferative signals such as
those from tumor suppressors. These antiproliferative signals
also act in the G1 phase to stop cells’ progress into the S phase
by inducing CKI production. For example, when DNA is dam-
aged, cells will repair the damage before entering the S phase.
Therefore, G1 contains one of the most important checkpoints
for cell cycle progression. If the analogy is made that CDK is
to a cell as an engine is to a car, then cyclins and CKI are the
gas pedal and brake, respectively. Accelerated proliferation or
improper cell cycle progression with damaged DNA would be
disastrous. Genetic gain-of-function mutations in oncogenes
(that often promote expression or activity of the cyclin/CDK
complex) or loss-of-function mutations in tumor suppres-
sor (that stimulate production of CKI) are causal factors for
malignant transformation. (See Schwartz 11th ed., p. 486.)
8. The cell cycle period during which DNA is replicated is: Answer : A
A. S. See Figure 15-2: The cell cycle and its control system. M is the
B. G1. mitosis phase, when the nucleus and the cytoplasm divide;
C. M. S is the phase when DNA is duplicated; G1 is the gap between
D. G2. B/CDK1

Mitosis
M

G2

G1

FIG. 15-2. The cell cycle and its control system. M is the mitosis
phase, when the nucleus and the cytoplasm divide; S is the phase
when DNA is duplicated; G1 is the gap between M and S; G2 is the A/CDK1 S
gap between S and M. A complex of cyclin and cyclin-dependent
DNA replication D/CDK4
kinase (CDK) controls specific events of each phase. Without cyclin, D/CDK6
CDK is inactive. Different cyclin/CDK complexes are shown around
the cell cycle. A, B, D, and E stand for cyclin A, cyclin B, cyclin D, and
cyclin E, respectively. A/CDK2 E/CDK2

Brunicardi_Ch15_p119-126.indd 121 04/07/22 5:46 PM


122
M and S; G2 is the gap between S and M. A complex of cyclin
CHAPTER 15

and cyclin-dependent kinase (CDK) controls specific events


of each phase. Without cyclin, CDK is inactive. Different
cyclin/CDK complexes are shown around the cell cycle. A, B,
D, and E stand for cyclin A, cyclin B, cyclin D, and cyclin E,
respectively. (See Schwartz 11th ed., p. 486.)

9. In cellular apoptosis, the release of cytochrome c acti- Answer: D


Molecular and Genomic Surgery

vates the: See Figure 15-3. (See Schwartz 11th ed., Figure 15-8, p. 487.)
A. FAS receptor.
B. A membrane bound death receptor.
C. Tumor necrosis factor (TNF) receptor.
D. Caspase cascade.
Death signal
(e.g., TNF or Fas)

Death Plasma
receptor membrane

Mitochondrion

Death
receptor
signaling Cytochrome c
FIG. 15-3. A simplified view of the pathway release
apoptosis pathways. Extracellular death
receptor pathways include the activation
Activation of
of Fas and tumor necrosis factor (TNF) caspase cascade
receptors and consequent activation
of the caspase pathway. Intracellular
death pathway indicates the release of Apoptotic target cell
Nucleus
cytochrome c from mitochondria, which
also triggers the activation of the caspase
cascade. During apoptosis, cells undergo
DNA fragmentation and nuclear and cell
membrane breakdown and are eventually
digested by other cells. Normal target cell

10. All of the following are cell-surface receptors EXCEPT: Answer: D


A. Transmitter-gated ion channels. There are three major classes of cell-surface receptors: trans-
B. Seven-transmembrane-G-protein-coupled receptors. mitter-gated ion channels, seven-transmembrane G-protein–
C. Enzyme-linked receptors. coupled receptors (GPCRs), and enzyme-linked receptors.
D. Adhesive receptors. The superfamily of GPCRs is one of the largest families of
proteins, representing over 800 genes of the human genome.
Members of this superfamily share a characteristic seven-
transmembrane configuration. The ligands for these receptors
are diverse and include hormones, chemokines, neurotrans-
mitters, proteinases, inflammatory mediators, and even sen-
sory signals such as odorants and photons. Most GPCRs
signal through heterotrimeric G proteins, which are guanine
nucleotide regulatory complexes. Thus the receptor serves
as the receiver, the G protein serves as the transducer, and
the enzyme serves as the effector arm. Enzyme-linked recep-
tors possess an extracellular ligand-recognition domain and a
cytosolic domain that either has intrinsic enzymatic activity
or directly links with an enzyme. Structurally, these receptors
usually have only one transmembrane-spanning domain. Of
at least five forms of enzyme-linked receptors classified by
the nature of the enzyme activity to which they are coupled,
the growth factor receptors such as tyrosine kinase receptor
or serine/threonine kinase receptors mediate diverse cellular

Brunicardi_Ch15_p119-126.indd 122 04/07/22 5:46 PM


123
events including cell growth, differentiation, metabolism, and

CHAPTER 15
survival/apoptosis. Dysregulation (particularly mutations) of
these receptors is thought to underlie conditions of abnor-
mal cellular proliferation in the context of cancer. The fol-
lowing sections will further review two examples of growth
factor signaling pathways and their connection with human
diseases. (See Schwartz 11th ed., p. 488.)
11. Dysregulation of transforming growth factor-β (TGF-β) Answer: B

Molecular and Genomic Surgery


signaling is associated with all EXCEPT: Resistance to TGF-β’s anticancer action is one hallmark of
A. Cancer. human cancer cells. TGF-β receptors and SMADs are identi-
B. Inguinal hernias. fied as tumor suppressors. The TGF-β signaling circuit can
C. Marfan syndrome. be disrupted in a variety of ways and in different types of
D. Thoracic aortic aneurysm. human tumors. Some lose TGF-β responsiveness through
downregulation or mutations of their TGF-β receptors. The
cytoplasmic SMAD4 protein, which transduces signals from
ligand-activated TGF-β receptors to downstream targets, may
be eliminated through mutation of its encoding gene. The
locus encoding cell cycle inhibitor p15INK4B may be deleted.
Alternatively, the immediate downstream target of its actions,
cyclin dependent kinase 4 (CDK4), may become unresponsive
to the inhibitory actions of p15INK4B because of mutations
that block p15INK4B binding. The resulting cyclin D/CDK4
complexes constitutively inactivate tumor suppressor pRb by
hyperphosphorylation. Finally, functional pRb, the end target
of this pathway, may be lost through mutation of its gene. For
example, in pancreatic and colorectal cancers, 100% of cells
derived from these cancers carry genetic defects in the TGF-β
signaling pathway. Therefore, the antiproliferative pathway
converging onto pRb and the cell division cycle is, in one way or
another, disrupted in a majority of human cancer cells. Besides
cancer, dysregulation of TGF-β signaling also has been associ-
ated with other human diseases such as Marfan syndrome and
thoracic aortic aneurysm. (See Schwartz 11th ed., p. 489.)
12. All of the following are true regarding type 2 diabetes Answer: D
EXCEPT: See Figure 15-4. Insulin is a peptide growth factor that binds
A. More than 90% of individuals have insulin resistance. to and activates the heterotetrameric receptor complex
B. Genetic mutation in the cell-surface insulin receptors (InsR). InsR possesses protein tyrosine kinase activity and is
(InsR) cause the disease.
C. Majority of cases may result from defects in down-
Insulin Insulin
stream-signaling components in the insulin-signal- receptor Plasma
ing pathway. (InsR) membrane
D. Phosphoryl group is added by the insulin receptor
substrate (IRS).

Adaptor IRS PI3K

MAPK Lipid & glucose Cell


cascade metabolism survival
FIG. 15-4. Insulin-signaling pathway. Insulin is a
peptide growth factor that binds to and activates
the heterotetrameric receptor complex (InsR). InsR Nucleus
possesses protein tyrosine kinase activity and is able
to phosphorylate the downstream insulin receptor
substrate (IRS). Phosphorylated IRS serves as a scaffold
and controls the activation of multiple downstream Gene
pathways for gene expression, cell survival, and glucose expression
metabolism. Inactivation of the insulin pathway can
lead to type 2 diabetes.

Brunicardi_Ch15_p119-126.indd 123 04/07/22 5:46 PM


124
able to phosphorylate the downstream IRS. Phosphorylated
CHAPTER 15

IRS serves as a scaffold and controls the activation of multiple


downstream pathways for gene expression, cell survival, and
glucose metabolism. Inactivation of the insulin pathway can
lead to type 2 diabetes. (See Schwartz 11th ed., Figure 15-10,
p. 489.)

13. Which of the following statements are TRUE? Answer: A


Molecular and Genomic Surgery

A. In normal cells, oncogenes promote cell growth by There are two classes of cancer genes in which alteration has
activating cell cycle progression. been identified in human and animal cancer cells: oncogenes,
B. In cancer cells, oncogenes promote cell growth by with dominant gain-of-function mutations, and tumor sup-
activating cell cycle progression. pressor genes, with recessive loss-of-function mutations. In
C. Tumor suppressors enhance oncogene function. normal cells, oncogenes promote cell growth by activating
D. Oncogenes do not play an important role in main- cell cycle progression, whereas tumor suppressors counteract
taining controlled state of cell growth. oncogenes’ functions. Therefore, the balance between onco-
genes and tumor suppressors maintains a well-controlled
state of cell growth. (See Schwartz 11th ed., p. 490.)

14. All of the following are correct about human embryonic Answer: D
stem cells (hESCs) EXCEPT: hESCs are derived from early preimplantation embryos
A. They are derived from early preimplantation called blastocysts (5 days postfertilization) and are capable of
embryos. generating all differentiated germ layers in the body by chi-
B. They are derived from blastocysts. mera assays or 2-D/3-D differentiation in a dish—ectoderm,
C. They can generate all differential germ layers. mesoderm, and endoderm—and therefore are considered
D. They are considered multipotent. pluripotent. There are two pluripotent states associated with
hESCs, one of which is the classic culture with basic fibroblast
growth factor (bFGF) and knock out replacer (KSR), termed
as “primed” pluripotent state. More recently, “naive” hESC
culture methods have been introduced based on mouse stud-
ies, by supplementing 2i inhibitors (MEK1 and GSK3β inhib-
itors) into the medium in addition to bFGF. (See Schwartz
11th ed., p. 492.)

15. Gene expression detection method that provides infor- Answer: C


mation regarding mRNA size is: Northern blotting refers to the technique of size fractionation
A. Polymerase chain reaction. of RNA in a gel and the transferring of an RNA sample to a
B. Southern blot hybridization. solid support (membrane) in such a manner that the relative
C. Northern blot hybridization. positions of the RNA molecules are maintained. The resulting
D. Immunoblotting. membrane then is hybridized with a labeled probe comple-
mentary to the mRNA of interest. Signals generated from
detection of the membrane can be used to determine the size
and abundance of the target RNA. In principle, Northern blot
hybridization is similar to Southern blot hybridization (and
hence its name), with the exception that RNA, not DNA, is
on the membrane. Although reverse-transcriptase PCR has
been used in many applications, Northern analysis is the only
method that provides information regarding mRNA size and
has remained a standard method for detection and quanti-
tation of mRNA. The process of Northern hybridization
involves several steps, as does Southern hybridization, includ-
ing electrophoresis of RNA samples in an agarose-formalde-
hyde gel, transfer to a membrane support, and hybridization
to a radioactively labeled DNA probe. Data from hybridiza-
tion allow quantification of steady-state mRNA levels and, at
the same time, provide information related to the presence,
size, and integrity of discrete mRNA species. Thus, Northern
blot analysis, also termed RNA gel blot analysis, commonly is
used in molecular biology studies relating to gene expression.
(See Schwartz 11th ed., p. 494.)

Brunicardi_Ch15_p119-126.indd 124 04/07/22 5:46 PM


125
16. Which of the following drugs is an example of an immu- Answer: A.

CHAPTER 15
notherapy that targets an oncogene? One of the most exciting applications of immunotherapy has
A. Trastuzumab come from the identification of certain tumor targets called
B. Methotrexate antigens and the aiming of an antibody at these targets. This
C. Adriamycin was first used as a means of localizing tumors in the body
D. Gleevec for diagnosis and was more recently used to attack cancer
cells. Trastuzumab (Herceptin) is an example of such a drug.
Trastuzumab is a monoclonal antibody that neutralizes the

Molecular and Genomic Surgery


mitogenic activity of cell-surface growth factor receptor HER-
2, which is overexpressed in approximately 25% of breast
cancers. HER-2–overexpressing tumors tend to grow faster
and generally are more likely to recur than tumors that do
not overproduce HER-2. Trastuzumab is designed to attack
cancer cells that overexpress HER-2 by slowing or prevent-
ing the growth of these cells, resulting in increased survival
of HER-2–positive breast cancer patients. (See Schwartz 11th
ed., p. 491.)

17. All of the following are TRUE regarding Bifunctional Answer: D


RNAi Technology EXCEPT: Over the last 20 years, the field has worked to define onco-
A. Field has worked to define oncogene and nononco- gene and nononcogene addiction, discriminate between
gene addiction. driver and passenger genes, and appreciate the complexity of
B. It advances the understanding of discrimination complex, robust, network interactions. These insights have
between driver and passenger genes. led to a preliminary understanding of therapeutically relevant
C. The field made strides in appreciation of complexity sensitivity and resistance pathway signal patterns requiring
of network interaction. multiple target modulation. However, this knowledge has not
D. The knowledge in the field has been effectively and been effectively or reproducibly clinically translated. Clinical
reproducibly clinically translated. response is usually far greater when a combination of single-
target molecular therapy is administered. However, it must
also be realized that targeting two or more pathways may also
increase the toxicity profile, particularly if target specificity is
limited. When attempted, off-target toxicity has been dem-
onstrated with combination small-molecule therapy. In con-
trast, multitargeting bifunctional short hairpin (bi-shRNA)
DNA vectors are designed to limit off-target effect given the
high specificity for the genes they are designed to target. (See
Schwartz 11th ed., p. 505.)

18. Which of the following about CRISPR is TRUE? Answer: D


A. CRISPR-Cas9 technology can be used to edit single CRISPR stands for clustered regularly interspaced short pal-
genes through gene knockout, mutation, and addi- indromic repeats. It is a region on the genomic DNA first
tion of an epitope tag. discovered in the microbes as an adapted immune system
B. A typical CRISPR region contains a cluster of DNA against exogenous DNA. A typical CRISPR region contains
repeats interspersed with spacers. a cluster of short (21–48 bp) DNA repeats (ranging from
C. CRISPR-Cas9 mediated gene editing involves DNA 2 to 100) interspaced by nonrepetitive sequences called
break and repair. spacers. Within a CRISPR region, while each spacer has its
D. All of the above. unique sequence, the sequence of the repeats is highly con-
served. Several genes, called the CRISPR-associated (Cas)
genes, are almost always found directly flanking the CRISPR
region. Currently, CRISPR-Cas9 is most used for editing
single genes, through gene knockout, gene mutation, or the
addition of an epitope tag to a native gene, for functional char-
acterization of the gene of interest. For example, oncogenes or
tumor suppressor genes can be knocked out to identify the
causative gene for a particular cancer type; point mutations
in functional domains may illustrate the mechanism of action
of a protein; for proteins without available antibodies, epitope
tags can be inserted onto the native gene for the detection of
the native protein. (See Schwartz 11th ed., p. 506.)

Brunicardi_Ch15_p119-126.indd 125 04/07/22 5:46 PM


This page intentionally left blank

Brunicardi_Ch15_p119-126.indd 126 04/07/22 5:46 PM


PART II
Specific Consideration

Brunicardi_Ch16_p127-136.indd 127 30/06/22 11:21 AM


Brunicardi_Ch16_p127-136.indd 128 30/06/22 11:21 AM
CHAPTER 16
The Skin and Subcutaneous Tissue

1. The area most amenable to salvage by resuscitative Answer: C


and wound management techniques following thermal The management of thermal wounds is initially guided by the
injury is called the: concept of three distinct zones of injury. The focus of ther-
A. Zone of hyperemia. mal injury that has already undergone necrosis is known as
B. Zone of coagulation. the zone of coagulation. Well outside the zone of coagulation
C. Zone of stasis. is the zone of hyperemia, which exhibits signs of inflamma-
tion but will likely remain viable. In between these two zones
is a zone of stasis with questionable tissue viability, and it is
this area at which proper burn care can salvage viable tissue
and decrease the extent of injury. (See Schwartz 11th ed.,
pp. 521–522.)

2. Initial treatment of extensive cellulitis without abscess is: Answer: B


A. Vancomycin. Extensive cellulitis is managed in a similar fashion as simple
B. β-lactam. cellulitis. Initial treatment consists of intravenous antibiotics
C. Linezolid. that cover β-hemolytic streptococcus, such as cephalospo-
D. Clindamycin. rins, with the addition of methicillin-resistant Staphylococcus
aureus (MRSA) coverage if there is no improvement in symp-
toms. Vancomycin is typically the first choice for MRSA cov-
erage, but this drug is inferior to β-lactams for coverage of
methicillin-sensitive S. aureus (MSSA). Alternative antibiotics
that are typically effective against MRSA are linezolid, dapto-
mycin, tigecycline, and telavancin. Clindamycin is approved
for use against MRSA, but resistance rates are increasing, and
its use is discouraged if institutional rates of clindamycin resis-
tance are >15%. (See Schwartz 11th ed., p. 525.)

3. A 3-mm, nodular-type basal cell carcinoma (BCC) of the Answer: C


skin of the trunk should be treated with: Treatment of BCC varies according to size, location, type,
A. Mohs micrographic surgery. and high or low risk. Treatment options include surgical exci-
B. Dermatologic laser vaporization. sion and medical or destructive therapies. Surgical excision
C. Excision with a 4-mm margin of normal tissue. should include 4 mm margins for low-risk lesions. Mohs
D. Electrodesiccation. microsurgical is indicated for high-risk lesions, especially in
cosmetically sensitive areas. In high-risk lesions in which
Mohs micrographic surgery is not available, excision with
10 mm margin of normal tissue can be considered. (See
Schwartz 11th ed., p. 529.)

129

Brunicardi_Ch16_p127-136.indd 129 30/06/22 11:21 AM


130
4. The primary risk factor for the development of squa- Answer: A
mous cell carcinoma (SCC) is: The primary risk factor for the development of SCC is
A. Ultraviolet (UV) radiation exposure. UV radiation exposure; however, other risks include light
B. Tobacco use. Fitzpatrick skin type (I or II), environmental factors such as
C. Exposure to chemical agents. chemical agents, physical agents (ionizing radiation), pso-
D. Chronic, nonhealing wounds. ralen, HPV-16 and -18 infections, immunosuppression, smok-
ing, chronic wounds, burn scars, and chronic dermatoses.
Heritable risk factors include xeroderma pigmentosum,
epidermolysis bullosa, and oculocutaneous albinism. (See
CHAPTER 16

Schwartz 11th ed., p. 529.)

5. Using the ABCDE initialism for melanoma, at what Answer: C


diameter does a nevus become concerning for malignant Melanoma most commonly manifests as cutaneous disease,
transformation? and clinical characteristics of malignant transformation are
A. 2 mm often remembered by the initialism ABCDE. These lesions
B. 4 mm are typically asymmetric with irregular borders, color varia-
The Skin and Subcutaneous Tissue

C. 6 mm tions, a diameter > 6 mm, and are undergoing some sort of


D. 10 mm evolution or change. Other key clinical characteristics include
a pigmented lesion that has enlarged, ulcerated, or bled.
Amelanotic lesions appear as raised pink, purple, or flesh-
colored skin papules and are often diagnosed late. (See
Schwartz 11th ed., p. 530.)

6. What is the most common site of distant metastasis of Answer: B


cutaneous melanoma? The most common sites of metastasis of melanoma are the
A. Brain lung and liver. These are followed by the brain, gastrointesti-
B. Lung nal tract, distant skin, and subcutaneous tissue. (See Schwartz
C. Bone 11th ed., p. 533.)
D. Distant skin

7. Which aggressive subtype of melanoma is characterized Answer: D


by an early vertical growth pattern, often leading to later- The most common subtype of melanoma is superficial
stage diagnosis? spreading (Fig. 16-1). This accounts for 50% to 70% of mela-
A. Superficial spreading nomas and typically arises from a precursor melanocytic
B. Lentigo maligna nevus. Nodular subtype accounts for 15% to 30% of melano-
C. Acral lentiginous mas, and typically arises de novo, most commonly in men and
D. Nodular on the trunk (Figs. 16-2 and 16-3). This subtype is aggressive
with an early vertical growth pattern and is often diagnosed
at a later stage. Up to 5% of these lesions will lack melanin
and can be mistaken for other cutaneous lesions. Lentigo
maligna represents 10% of melanoma cases and is a less
aggressive subtype of melanoma in situ that typically arises
on sun-exposed areas of the head and neck. Acral lentiginous

FIG. 16-1. Primary cutaneous melanoma seen in the scalp


of a 61-year-old man.

Brunicardi_Ch16_p127-136.indd 130 30/06/22 11:21 AM


131
melanoma accounts for 29% to 72% of melanomas in dark-
skinned individuals, is occasionally seen in Caucasians,
and is found on palmar, plantar, and subungual surfaces.
This subtype is not thought to be due to sun exposure. (See
Schwartz 11th ed., p. 530.)

CHAPTER 16
The Skin and Subcutaneous Tissue
FIG. 16-2. Nodular melanoma seen in the leg of
a 55-year-old man.

A B

FIG. 16-3. A. AP view of advanced melanoma in a


59-year-old man. B. Lateral view. C. After resection and
C reconstruction with skin grafting.

Brunicardi_Ch16_p127-136.indd 131 30/06/22 11:21 AM


132
8. Which of the answer choices does NOT represent an Answer: B
indication for sentinel lymph node biopsy (SLNB) in the SLNB is a standard staging procedure to evaluate the
setting of malignant cutaneous melanoma? regional nodes for patients with clinically node-negative
A. Breslow depth 0.5 mm with superficial ulceration malignant melanoma. Detecting subclinical nodal metasta-
B. Clinically positive lymph nodes sis may benefit from lymphadenectomy or adjuvant therapy.
C. Breslow depth 1.5 mm without superficial ulceration This technique identifies the first draining lymph node from
D. Breslow depth 1.0 mm with lymphovascular invasion the primary lesion and has shown excellent accuracy and sig-
nificantly less morbidity compared to complete resection of
nodal basins. It is almost always performed at the time of ini-
CHAPTER 16

tial wide excision, as SLN mapping after lymphatic violation


from surgical excision could decrease the accuracy of the test.
Recently, the results of MSLT-1, an international, multicenter,
phase III trial were published. This study randomized clini-
cally node negative patients to either SLNB at the time of pri-
mary melanoma excision (and completion lymphadenectomy
if positive) or nodal basin monitoring (and delayed complete
The Skin and Subcutaneous Tissue

lymphadenectomy for recurrent lymph node disease). The


results of this study demonstrated that SLNB, with immedi-
ate lymphadenectomy if positive, improved disease-free sur-
vival by 7% and 10% in patients with intermediate thickness
(1.2–3.5 mm) and thick (>3.5 mm) lesions, respectively. The
latest NCCN guidelines suggest consideration of SLNB in
clinically stage I/II patients with Breslow depth > 1.0 mm or
>0.8 mm with high-risk features (ulceration, mitotic index
> 2/mm2, or lymphovascular invasion). A patient with clini-
cally positive nodes would not be a candidate for screening
SLNB. (See Schwartz 11th ed., p. 531.)

9. Which surgical margins are appropriate for excision of a Answer: B


primary cutaneous melanoma that has a Breslow depth The appropriate excision margin for cutaneous melanoma
of 1.5 mm? is based on primary tumor thickness. Several retrospective
A. 5 mm margins studies suggest that for melanoma in situ, 0.5 to 1 cm mar-
B. 10 mm margins gins are sufficient. 1-cm margins should be obtained in ana-
C. 30 mm margins tomically feasible areas given the possibility of an incidental
D. 1.5 mm margins finding of a small invasive component in permanent sections.
Several studies compared 1- to 3-cm margins and 2- to 5-cm
margins in melanoma <2 mm thick, and 2- to 4-cm margins
in melanoma lesions 1 to 4 mm thick and found no differ-
ence. A British trial suggested that there is a limit to how
narrow margins can be for melanomas >2 mm thick by show-
ing that 1-cm margins provide worse outcomes compared to
3-cm margins. Tumors <1 mm thick require 0.5 to 1 cm mar-
gins. Tumors 1 to 2 mm thick require 1 to 2 cm margins, and
tumors >2 mm thick require 2-cm margins. (See Schwartz
11th ed., p. 532.)

10. What is the source of regenerative keratinocytes in the Answer: D


epithelialization of partial thickness wounds of the skin? The stratum basale, or germinative layer, is a deep, single
A. Langerhans cell conversion to keratinocytes layer of asynchronous, continuously replicating cuboidal to
B. Fibroblast conversion to keratinocytes columnar epithelial cells and is the beginning of the life cycle
C. Extravasation of stem cells from the blood stream of the keratinocytes of the epidermis. This layer is bound to
D. Keratinocytes from the lining of epidermal appendages its basement membrane by complexes made of keratin fila-
ments and anchoring structures called hemidesmosomes.
They are bound to other keratinocytes by structures called
desmosomes. High mitotic activity and thus large nuclei and
basophilic staining characterize the stratum basale on light
microscopy. This layer also lines the epidermal appendages
that reside largely within the substance of the dermis and later
serves as a regenerative source of epithelium in the event of
partial thickness wounds. (See Schwartz 11th ed., p. 514.)

Brunicardi_Ch16_p127-136.indd 132 30/06/22 11:21 AM


133
11. What cell type are characterized by rod- or racket- Answer: A
shaped Birbeck granules, takes up antigens for presenta- Of the cells in the epidermis, 3% to 6% are immune cells
tion to T-cells, and makes up 3% to 6% of the cells of the known as Langerhans cells. Typically found within the stratum
epidermis? spinosum, these mobile, dendritic cells interdigitate between
A. Langerhans cell keratinocytes of the epidermis to create a dense network,
B. Melanocyte sampling any antigens that attempt to pass through the cutane-
C. Merkel cell ous tissue. Through use of their characteristic rod- or racket-
D. Mast cell shaped Birbeck granules, they take up antigens for presentation
to T-cells. These monocyte-derived cells represent a large

CHAPTER 16
part of the skin’s adaptive immunity. (See Schwartz 11th ed.,
p. 515.)

12. What type of gland is typically influenced by sex hor- Answer: C


mones and undergoes a secretion process that involves One type of sweat gland, known as the apocrine sweat gland,
decapitation of the part of the cell? is found around the axilla, anus, areola, eyelid, and external
A. Hair follicle auditory canal. The cells in this gland undergo an excretion

The Skin and Subcutaneous Tissue


B. Sebaceous gland process that involves decapitation of part of the cell. These
C. Apocrine sweat gland apocrine glands are typically activated by sex hormones
D. Eccrine sweat gland and thus activate around the time of puberty. The secretion
from apocrine glands is initially odorless, but bacteria in the
region may cause an odor to develop. Pheromone produc-
tion may have been a function of the apocrine glands, but this
may now be vestigial. While eccrine sweat glands are acti-
vated by the cholinergic system, apocrine glands are activated
by the adrenergic system. (See Schwartz 11th ed., p. 515.)

13. In the Hurley classification of hidradenitis suppurativa, Answer: B


which stage is characterized by diffuse disease with inter- The diagnosis of hidradenitis is clinical, and the presentation
connected sinus tracts and recurrent abscesses? is most commonly categorized by the Hurley classification
A. Hurley stage IV system, divided into three stages. Single or multiple nodules
B. Hurley stage III or abscesses without any sinus tracts or scarring would be
C. Hurley stage II classified as stage I disease. As abscesses recur and sinus tracts
D. Hurley stage I and scarring form, the disease is classified as Hurley stage II.
Stage III is the most advanced stage, with diffuse disease and
interconnected sinus tracts and abscesses. Hurley stage IV is
nonexistent. (See Schwartz 11th ed., p. 517.)

14. What mucocutaneous disorder, thought to be a reaction Answer: D


to various drugs, is characterized by mucocutaneous Epidermal necrolysis (EN) is a rare mucocutaneous disorder
destruction at the dermoepidermal junction affecting characterized by cutaneous destruction at the dermoepider-
<10% of the cutaneous epithelium? mal junction. EN is commonly referred to as either SJS or
A. Nikolsky syndrome TEN depending on the extent of skin involvement present.
B. Toxic epidermal necrolysis (TEN) SJS refers to cases in which <10% of total body surface area
C. Hidradenitis suppurativa is involved, while cases with >30% involvement are consid-
D. Stephens-Johnson syndrome (SJS) ered TEN, with an SJS-TEN overlap syndrome referring to
all cases in between. A positive Nikolsky sign is often present,
in which lateral pressure on the skin causes separation of the
epidermis from the dermis. (See Schwartz 11th ed., p. 518.)

15. A 55-year-old otherwise healthy man presents 1 hour Answer: C


after a human bite to his hand. After thoroughly washing Bacteria colonizing human bites are those present on the skin
out the wound, are prophylactic antibiotics necessary? If or in the mouth. These include the gram-positive aerobic
so, what antibiotic regimen is most appropriate? organisms Staphylococcus aureus, Staphylococcus epidermidis,
A. No antibiotic prophylaxis indicated and Streptococcus species, and anaerobes including Peptococ-
B. 3–7 days of cephalexin cus species, Peptostreptococcus species, Bacteroides species,
C. 3–7 days of amoxicillin/clavulanate and Eikenella corrodens (facultative anaerobe). Human bites
D. 3–7 days of vancomycin and piperacillin/tazobactam are characterized by a higher bacterial load (>105). Antibi-
otic prophylaxis after a human bite is recommended as it has

Brunicardi_Ch16_p127-136.indd 133 30/06/22 11:21 AM


134
been shown to significantly decrease the rate of infection. A
course of 3 to 7 days of amoxicillin/clavulanate is typically
used. Alternatives are doxycycline or clindamycin with cipro-
floxacin. (See Schwartz 11th ed., p. 519.)

16. Following caustic injury with a strong alkaline solution, Answer: C


what is the appropriate treatment? Treatment for acidic or alkaline chemical burns is first and
A. Neutralization with a weakly acidic solution for foremost centered around dilution of the offending agent,
30 minutes typically using distilled water or saline for 30 minutes for
CHAPTER 16

B. Irrigation with normal saline for 30 minutes acidic burns and 2 hours for alkaline injuries. Attempting to
C. Irrigation with normal saline for 120 minutes neutralize the offending agent is typically discouraged, as it
D. Immediate coverage with light compressive dressing does not offer an advantage over dilution and the neutraliza-
tion reaction could be exothermic, increasing the amount of
tissue damage. After removal of the caustic agent, the burn
is treated like other burns and is based on the depth of tissue
injury. (See Schwartz 11th ed., p. 521.)
The Skin and Subcutaneous Tissue

17. How long does it take to get irreversible tissue damage Answer: B
with external pressure greater than double capillary per- The average perfusion pressure of the microcirculation is
fusion pressure? about 30 mm Hg, and pressures greater than that cause local
A. 1 hour tissue ischemia. In animal models, pressure greater than twice
B. 2 hours the capillary perfusion pressure produces irreversible tissue
C. 3 hours necrosis in just 2 hours. Tissues with a higher metabolic
D. 4 hours demand are typically susceptible to insult from tissue hypo-
perfusion more rapidly than tissues with a lower metabolic
demand. Because of this, it is possible to have muscle necrosis
beneath cutaneous tissue that has yet to develop signs of irre-
versible damage. (See Schwartz 11th ed., p. 523.)

18. Pressure-induced ischemia to cutaneous tissue resulting Answer: B


in nonblanching erythema with intact epidermis repre- Pressure ulcers can present in several ways depending on the
sents which stage of injury? stage at presentation. They are typically grouped into four
A. Does not yet qualify as pressure-induced tissue injury stages: stage I, nonblanching erythema over intact skin; stage II,
B. Stage I partial-thickness injury with blistering or exposed dermis;
C. Stage II stage III, full-thickness injury extending down to, but not
D. Stage III including, fascia and without undermining of adjacent tis-
sue; and stage IV, full-thickness skin injury with destruction
or necrosis of muscle, bone, tendon, or joint capsule. (See
Schwartz 11th ed., p. 523.)

19. What type of necrotizing soft tissue infection (NSTI) is Answer: B


characterized by a monomicrobial source of β-hemolytic There are three types of NSTIs when classified by the offend-
Streptococcus or Staphylococcus species? ing agent. The most common is type 1, which is caused by
A. Type I a polymicrobial source including gram-positive cocci, gram-
B. Type II negative rods, and anaerobic bacteria, specifically Clostridium
C. Type III perfringens and C. septicum. Type 2 is caused by a monomi-
D. Type IV crobial source of β-hemolytic Streptococcus or Staphylococcus
species, with methicillin-resistant Staphylococcus aureus
(MRSA) contributing to the increasing number of community-
acquired NSTIs. A history of trauma is often elicited and can
be associated with toxic shock syndrome. Type III is a rare
but fulminant subset resulting from a Vibrio vulnificus infec-
tion of traumatized skin exposed to a body of salt water. (See
Schwartz 11th ed., p. 525.)

Brunicardi_Ch16_p127-136.indd 134 30/06/22 11:21 AM


135
20. What subtypes of the human papilloma virus (HPV) Answer: A
confer the highest risk for malignancy? Mucosal HPV types cause lesions in the mucosal or genital
A. Types 16 and 18 areas and behave like sexually transmitted infections. The
B. Types 6 and 11 most common mucosal types are HPV-6, -11, -16, -18, -31,
C. Types 6 and 8 and -33. These lesions present as condylomata acuminata,
D. Types 3 and 4 genital or veneral warts, papules that occur on the perineum,
external genitalia, anus, and can extend into the mucosal sur-
faces of the vagina, urethra, and rectum. These lesions are at
risk for malignant transformation, with types 6 and 11 con-

CHAPTER 16
ferring low risk, and types 16, 18, 31, and 33 conferring a high
risk. The recently developed quadrivalent HPV vaccine, tar-
geting HPV types -6, -11, -16, and -18, is now available to
both males and females age 9 to 26 and is associated with an
up to 90% reduction of infections from those HPV types. (See
Schwartz 11th ed., p. 526.)

The Skin and Subcutaneous Tissue


21. What cystic lesion is a benign, congenital occurrence Answer: C
caused by persistent epithelium across embryonic fusion Dermoid cysts are congenital variants that occur as the result
lines, most commonly between the forehead and tip of of persistent epithelium within embryonic lines of fusion.
the nose? They occur most commonly between the forehead and nose
A. Trichilemmal cyst tip, and the most frequent site is the eyebrow. They can lie
B. Epidermoid cyst in the subcutaneous tissue or intracranially, and often com-
C. Dermoid cyst municate with the skin surface via a small fistula. These cys-
D. Nevus tic structures contain epithelial tissue, hair, and a variety of
epidermal appendages. Treatment for these cystic structures
includes surgical excision with care taken to remove the cyst
lining to prevent recurrence. (See Schwartz 11th ed., p. 527.)

22. What subtype of basal cell carcinoma (BCC) is the most Answer: D
common, characterized by raised, pearly pink papules There are multiple variants of BCC, and presentation can
with telangiectasias and occasionally a depressed tumor range from red, flesh-colored, or white macule or papule
center with raised borders? to nodules and ulcerated lesions. Growth patterns of these
A. Micronodular lesions can either be well-circumscribed or diffuse and
B. Infiltrating the most common types of BCC are nodular and micronodu-
C. Superficial spreading lar, superficial spreading, and infiltrative. The most com-
D. Nodular mon subtype is the nodular variant, characterized by raised,
pearly pink papules with telangiectasias and occasionally a
depressed tumor center with raised borders giving the clas-
sic “rodent ulcer” appearance. Superficial spreading BCC is
confined to the epidermis as a flat, pink, scaling or crusting
lesion, often mistaken for eczema, actinic keratosis, fungal
infection, or psoriasis. This subtype typically appears on the
trunk or extremities and the mean age of diagnosis is 57 years.
The infiltrative form appears on the head and neck in the late
60s, often at embryonic fusion lines, with an opaque yellow-
white color that blends with surrounding skin and has no
raised edges. The morpheaform subtype represents 2% to 3%
of all BCC and is the most aggressive subtype. It usually pres-
ents as an indurated macule or papule with the appearance of
an enlarging scar. The clinical margins are often indistinct,
and the rate of positive margins after excision is high. There
is also a pigmented variant of BCC that can be difficult to
distinguish from certain melanoma subtypes. (See Schwartz
11th ed., pp. 528–529.)

Brunicardi_Ch16_p127-136.indd 135 30/06/22 11:21 AM


136
23. Which of these skin lesions can represent a precursor Answer: B
lesion to squamous cell carcinoma? Actinic keratoses are neoplasms of epidermal keratinocytes
A. Seborrheic keratosis that represent a range in a spectrum of disease from sun
B. Actinic keratosis damage to squamous cell carcinoma. They typically occur
C. Congenital nevus in fair-skinned, elderly individuals in primarily sun-exposed
D. Acrochordons areas, and ultraviolet (UV) radiation exposure is the great-
est risk factor. There are multiple variants, and they can pres-
ent as erythematous and scaly to hypertrophic, keratinized
lesions. They can become symptomatic, causing bleeding,
CHAPTER 16

pruritus, and pain. They can regress spontaneously, persist


without change, and transform into invasive squamous cell
carcinoma. It is estimated that approximately 10% of actinic
keratoses will transform into invasive squamous cell carci-
noma, and that progression takes about 2 years on average.
About 60% to 65% of squamous cell carcinomas are believed
to originate from actinic keratoses. Congenital nevi can rarely
The Skin and Subcutaneous Tissue

transform into malignant melanoma. Seborrheic keratoses


and acrochordons do not carry malignant potential. (See
Schwartz 11th ed., p. 528.)

Brunicardi_Ch16_p127-136.indd 136 30/06/22 11:21 AM


CHAPTER 17
Breast

1. Which of the following changes in the breast is not asso- Answer: C


ciated with pregnancy? With pregnancy, the breast undergoes proliferative and devel-
A. Accumulation of lymphocytes, plasma cells, and opmental maturation. As the breast enlarges in response to
eosinophils within the breast hormonal stimulation, lymphocytes, plasma cells, and eosin-
B. Enlargement of breast alveoli ophils accumulate within the connective tissues. The minor
C. Release of colostrum ducts branch and alveoli develop. Development of the alveoli
D. Accumulation of secretory products in minor duct is asymmetric, and variations in the degree of development
lumina may occur within a single lobule. With parturition, enlarge-
ment of the breasts occurs via hypertrophy of alveolar epithe-
lium and accumulation of secretory products in the lumina
of the minor ducts. Alveolar epithelium contains abundant
endoplasmic reticulum, large mitochondria, Golgi com-
plexes, and dense lysosomes. Two distinct substances are pro-
duced by the alveolar epithelium: (a) the protein component
of milk, which is synthesized in the endoplasmic reticulum
(exocrine secretion); and (b) the lipid component of milk
(apocrine secretion), which forms as free lipid droplets in the
cytoplasm. Milk released in the first few days after parturi-
tion is called colostrum and has low lipid content but contains
considerable quantities of antibodies. (See Schwartz 11th ed.,
p. 548.)

2. Which of the following statements is incorrect? Answer: D


A. Level I lymph nodes are those that are lateral to the Axillary lymph node groups are as follows: Level I includes
pectoralis minor muscle. lymph nodes located lateral to the pectoralis minor muscle;
B. Level II lymph nodes are located deep to the pectora- level II includes lymph nodes located deep to the pectoralis
lis minor muscle. minor; and level Ill includes lymph nodes located medial to
C. Level Ill lymph nodes are located medial to the pecto- the pectoralis minor. (See Schwartz 11th ed., p. 547.)
ralis minor muscle.
D. Level IV lymph nodes are the ipsilateral internal
mammary lymph nodes.

3. Concerning gynecomastia, which of the following is Answer: C


TRUE? In gynecomastia, the ductal structures of the male breast
A. During senescence gynecomastia is usually unilateral. enlarge, elongate, and branch with a concomitant increase in
B. During puberty gynecomastia is usually bilateral. epithelium. During puberty, the condition often is unilateral
C. Gynecomastia is associated with breast cancer in and typically occurs between ages 12 and 15 years. In con-
Klinefelter syndrome patients. trast, senescent gynecomastia is usually bilateral. Gynecomas-
D. Gynecomastia is classified as per a three-grade system. tia generally does not predispose the male breast to cancer.
However, the hypoandrogenic state of Klinefelter syndrome
(XXY), in which gynecomastia is usually evident, is associ-
ated with an increased risk of breast cancer. Gynecomastia is
graded based on the degree of breast enlargement, the position
137

Brunicardi_Ch17_p137-142.indd 137 30/06/22 11:21 AM


138
of the nipple with reference to the inframammary fold and the
degree of breast ptosis and skin redundancy: Grade 1: Mild
breast enlargement without skin redundancy; Grade Ila: Mod-
erate breast enlargement without skin redundancy; Grade
llb: Moderate breast enlargement with skin redundancy; and
Grade 3: Marked breast enlargement with skin redundancy
and ptosis. (See Schwartz 11th ed., p. 549.)

4. Lesions with malignant potential include all of the follow- Answer: C


CHAPTER 17

ing EXCEPT: Sclerosing adenosis is prevalent during the childbearing and


A. lntraductal papilloma. premenopausal years and has no malignant potential. Multiple
B. Atypical ductal hyperplasia (ADH). intraductal papillomas, which occur in younger women and
C. Sclerosing adenosis. are less frequently associated with nipple discharge, are suscep-
D. Atypical lobular hyperplasia (ALH). tible to malignant transformation. Individuals with a diagnosis
of ADH are at increased risk for development of breast cancer
and should be counseled appropriately regarding risk reduc-
Breast

tion strategies. ALH results in minimal distention of lobular


units with cells that are similar to those seen in lobular carci-
noma in situ (LCIS). (See Schwartz 11th ed., pp. 552–553.)

5. Risk factors for the development of breast cancer include: Answer: D


A. Early menarche Increased exposure to estrogen is associated with an increased
B. Nulliparity risk for developing breast cancer, whereas reducing exposure
C. Late menopause is thought to be protective. Correspondingly, factors that
D. Longer lactation periods increase the number of menstrual cycles, such as early men-
arche, nulliparity, and late menopause are associated with
increased risk. Moderate levels of exercise and a longer lacta-
tion period, factors that decrease the total number of men-
strual cycles, are protective. (See Schwartz 11th ed., p. 555.)

6. All of the following are TRUE concerning breast lobular Answer: D


carcinoma in situ (LCIS) EXCEPT: LCIS originates from the terminal duct lobular units and
A. Develops only in the female breast. develops only in the female breast. Cytoplasmic mucoid glob-
B. Cytoplasmic mucoid globules are a distinctive cellular ules are a distinctive cellular feature. The frequency of LCIS in
feature. the general population cannot be reliably determined because
C. Frequency of LCIS cannot be reliably determined. it usually presents as an incidental finding. The average age at
D. The average age at diagnosis is 65 to 70 years. diagnosis is 45 years, which is approximately 15 to 25 years
younger than the age at diagnosis for invasive breast cancer.
(See Schwartz 11th ed., pp. 553–554.)

7. Which of the following concerning breast cancer staging Answer: C


is correct? See Schwartz 11th ed., pp. 576–578.
A. Stage I tumors have no metastases to either lymph
nodes or distant sites.
B. Stage Ill tumors include some with distant metastases
(M1 disease).
C. Inflammatory carcinoma is considered T4 disease.
D. N4 disease includes metastases to highest contralat-
eral axillary nodes.

8. Which of the following statements about the manage- Answer: A


ment of distal carcinoma in situ (DCIS) is TRUE? Women with DCIS and evidence of extensive disease
A. DCIS treated by mastectomy has a local recurrence (>4 cm of disease or disease in more than one quadrant) usu-
rate of <2%. ally require mastectomy. For women with limited disease,
B. Extensive DCIS should be treated with tamoxifen fol- lumpectomy and radiation therapy are generally recom-
lowed by lumpectomy. mended. For no palpable DCIS, needle localization or other
C. Specimen mammography is only useful for patients image-guided techniques are used to guide the surgical resec-
with small amounts of DCIS. tion. Specimen mammography is performed to ensure that all
D. Postoperative tamoxifen is useful in DCIS patients visible evidence of cancer is excised. Adjuvant tamoxifen ther-
whose tumors are estrogen receptor negative. apy is considered for DCIS patients with estrogen-receptor

Brunicardi_Ch17_p137-142.indd 138 30/06/22 11:21 AM


139
(ER)-positive disease. The gold standard against which breast
conservation therapy for DCIS is evaluated is mastectomy.
Women treated with mastectomy have local recurrence and
mortality rates of <2%. (See Schwartz 11th ed., pp. 580–582.)

9. Patients not suitable for sentinel node biopsy include all Answer: D
of the following EXCEPT: Clinical situations where sentinel lymph node (SLN) dissec-
A. Inflammatory carcinoma of the breast tion is not recommended include patients with inflammatory
B. Prior axillary surgery breast cancers, those with palpable axillary lymphadenopathy

CHAPTER 17
C. Biopsy proven distant metastases and biopsy proven metastasis, DCIS without mastectomy, or
D. Breast lower inner quadrant carcinoma prior axillary surgery. Although limited data are available, SLN
dissection appears to be safe in pregnancy when performed
with radioisotope alone. (See Schwartz 11th ed., p. 590.)

10. Which of the following is TRUE concerning breast can- Answer: A


cer during pregnancy? Breast cancer occurs in 1 of every 3000 pregnant women, and

Breast
A. Metastases to lymph nodes occurs in approximately axillary lymph node metastases are present in up to 75% of
75% of these patients. these women. Less than 25% of the breast nodules developing
B. Approximately 50% of breast nodules developing during pregnancy and lactation will be cancerous. Mammog-
during pregnancy are malignant. raphy is rarely indicated because of its decreased sensitivity
C. Mammography is especially useful in localizing small during pregnancy and lactation; however, the fetus can be
lesions. shielded if mammography is needed. Chemotherapy admin-
D. There is risk of chemotherapy teratogenicity if used istered during the first trimester carries a risk of spontaneous
during the second, but not the third, trimester of abortion and a 12% risk of birth defects. There is no evidence
pregnancy. of teratogenicity resulting from administration of chemo-
therapeutic agents in the second and third trimesters. (See
Schwartz 11th ed., p. 600.)

11. Which of the following statements about breast anatomy Answer: C


are TRUE? Fibrous bands of connective tissue travel through the breast
A. The lateral border of the breast is the posterior axil- (Cooper suspension ligaments), insert perpendicularly into
lary line. the dermis, and promote structural support. The mature
B. The largest breast quadrant is the inner lower female breast extends from the level of the second or third
guardant. rib to the inframammary fold to the sixth or seventh rib. It
C. Vascular supply is via posterior intercostal arter- extends transversely from the border of the sternum to the
ies, the internal mammary arteries, and the axillary anterior axillary line. The upper outer quadrant of the breast
artery. contains a greater volume of tissue than the other quad-
D. Cooper ligaments are only found in the upper quad- rants. The breast receives its principal blood supply from:
rants of the breast. (a) perforating branches of the internal mammary artery;
(b) lateral branches of the posterior intercostal arteries; and
(c) branches from the axillary artery, including highest tho-
racic, lateral thoracic, and pectoral branches of the thoracoac-
romial artery. The second, third, and fourth anterior intercostal
perforators, and branches of the internal mammary artery,
arborize in the breast. (See Schwartz 11th ed., pp. 544–545.)

12. Which of the following hormonal effects on the breast is Answer: D


correct? Estrogen initiates ductal development, whereas progesterone
A. Estrogen is responsible for lobule development. is responsible for differentiation of epithelium and for lobu-
B. Progesterone initiates ductal development. lar development. Prolactin is the primary hormonal stimulus
C. Prolactin is responsible for milk letdown. for lactogenesis in late pregnancy and the postpartum period.
D. Oxytocin initiates contraction of myoepitherium. Oxytocin release is due to auditory, visual, and olfactory
stimuli associated with nursing. Oxytocin initiates contrac-
tion, which results in compression of alveoli and expulsion
of milk into the lactiferous sinuses. (See Schwartz 11th ed.,
pp. 547–548.)

Brunicardi_Ch17_p137-142.indd 139 30/06/22 11:21 AM


140
13. All of the following are mechanisms underlying develop- Answer: D
ment of gynecomastia EXCEPT: (See Schwartz 11th ed., p. 549 [see chart].)
A. Neoplasms of testis
B. Lung carcinoma
C. Cirrhosis
D. Marfan syndrome

14. Concerning infections of the breast, which of the follow- Answer: C


ing are TRUE? Staphylococcus aureus and Streptococcus species are the organ-
CHAPTER 17

A. Most common organism is Escherichia coli. isms most frequently recovered from nipple discharge from
B. Operative drainage is the most common therapeutic an infected breast. Previously almost all breast abscesses were
approach. treated by operative incision and drainage, but now the initial
C. Fungal infections of the breast are most commonly approach is antibiotics and repeated aspiration of the abscess,
initiated by nursing infants. usually ultrasound-guided aspiration. Intraoral fungi that are
D. Bilateral Mondor disease is more common than uni- introduced into the breast tissue by the suckling infant can
lateral disease. initiate infection which presents as mammary abscesses in
Breast

close proximity to the nipple-areola complex. Mondor dis-


ease is a variant of thrombophlebitis involving the superficial
veins of the anterior chest wall and breast. The presentation is
rarely bilateral. (See Schwartz 11th ed., p. 550.)

15. Correct statements about risk of developing breast can- Answer: A


cer include all of the following EXCEPT: The average lifetime risk of breast cancer for newborn US
A. Lifetime risk in US females is 20%. females is 12%. Gail et al developed the model most fre-
B. Increased risk with exposure to therapeutic radiation quently used in the United States, which incorporates age, age
in adolescence. at menarche, age at first live birth, the number of breast biopsy
C. Early menarche. specimens, any history of atypical hyperplasia, and number of
D. Older age at first live birth. first-degree relatives with breast cancer. (See Schwartz 11th
ed., p. 555.)

16. Which of the following are correct concerning BRCA Answer: B


mutations? Up to 5% of breast cancers are caused by inheritance of germ-
A. Fifteen percent of breast cancers are caused by BRCA line mutations, such as BRCA1 and BRCA2, which are inher-
mutations. ited in an autosomal dominant fashion with varying degrees
B. BRCA mutations play a role in DNA damage response of penetrance. Data accumulated from the isolation of the
pathways. BRCA1 gene suggest a role in cell cycle control and DNA
C. Male BRCA2 mutation carriers have an equal risk damage repair pathways. Female mutation carriers have been
of developing breast cancer compared to noncarrier reported to have up to 85% lifetime risk for developing breast
males. cancer. Unlike male carriers of BRCA1 mutations, men with
D. Twenty-five percent of women with BRCA mutation germline mutations in BRCA2 have an estimated breast can-
have inherited the mutation from their fathers. cer risk of 6%, which represents a 100-fold increase over the
risk in general male population. Fifty percent of the women
with a BRCA mutation have inherited the mutation from
their fathers. (See Schwartz 11th ed., pp. 558–560.)

17. Therapeutic options for early invasive breast cancer Answer: B


include all of the following EXCEPT: NSABP B-06 compared total mastectomy to lumpectomy
A. Mastectomy with axillary staging. with or without radiation therapy in the treatment of women
B. Neoadjuvant systemic therapy with radiation. with stages I and II breast cancer. After 5- and 8-year follow-
C. Lumpectomy with axillary staging and radiation. up periods, the disease-free, distance disease-free, and overall
D. Mastectomy with axillary staging and immediate survival rates for lumpectomy with or without radiation ther-
reconstruction. apy were similar to those observed after total mastectomy.
However, the incidence of ipsilateral breast cancer recurrence
was higher in the group not receiving radiation therapy. For
most patients with early stage disease, reconstruction can be
performed immediately at the time of the mastectomy. Imme-
diate reconstruction allows for skin-sparing, thus optimizing
cosmetic outcomes. (See Schwartz 11th ed., pp. 582–583.)

Brunicardi_Ch17_p137-142.indd 140 30/06/22 11:21 AM


141
18. Which of the following are true concerning mastectomy? Answer: A
A. Skin-sparing mastectomy removes all breast tissue, A skin-sparing mastectomy removes all breast tissue, the
the nipple-areola complex, and previous biopsy scars. nipple-areola complex, and scars from any prior biopsy pro-
B. Extended simple mastectomy removes all breast tis- cedures. A total (simple) mastectomy without skin sparing
sue, nipple-areola complex, and level I and II axillary removes all breast tissue, the nipple-areola complex, and skin.
lymph nodes. An extended simple mastectomy removes all breast tissue,
C. Modified radical mastectomy removes all breast tis- the nipple-areola complex, skin, and the level I axillary lymph
sue, nipple-areola complex, and level I and II axillary nodes. A modified radical (“Patey”) mastectomy removes all
lymph nodes. breast tissue, the nipple-areola complex, skin, and Ievel I, II,

CHAPTER 17
D. Radical mastectomy removes all breast tissue, nipple- and Ill axillary lymph nodes. The Halsted radical mastectomy
areola complex, and levels I, II, and III axillary lymph removes all breast tissue and skin, the nipple-areola complex,
nodes, the pectoralis major muscle, and internal the pectoralis major and pectoralis minor muscles, and the
mammary lymph nodes sampling. levels I, II, and Ill axillary lymph nodes. (See Schwartz 11th ed.,
p. 591.)

Breast

Brunicardi_Ch17_p137-142.indd 141 30/06/22 11:21 AM


This page intentionally left blank

Brunicardi_Ch17_p137-142.indd 142 30/06/22 11:21 AM


CHAPTER 18
Head and Neck

1. Which of the following statements are TRUE about leu- Answer: B


koplakia of the vocal cords? Leukoplakia of the vocal fold represents a white patch (which
A. Up to 40% risk of progression to invasive carcinoma. cannot be wiped off) on the mucosaI surface, usually on the
B. Ulceration is particularly suggestive of possible superior surface of the true vocal cord. Rather than a diagnosis
malignancy. per se, the term leukoplakia describes a finding on laryngo-
C. Initial therapy includes antihistamines. scopic examination. The significance of this finding is that it
D. Biopsy should be considered only after 6 months of may represent squamous hyperplasia, dysplasia, and/or car-
conservative therapy. cinoma. Lesions exhibiting hyperplasia have a 1% to 3% risk
of progression to malignancy. In contrast, that risk is 10% to
30% for those demonstrating dysplasia.
Furthermore, leukoplakia may be observed in association
with inflammatory and reactive pathologies, including polyps,
nodules, cysts, granulomas, and papillomas. Features of ulcer-
ation and erythroplasia are particularly suggestive of possible
malignancy. A history of smoking and alcohol abuse should
also prompt a malignancy work-up. In the absence of suspected
malignancy, conservative measures are used for 1 month. Any
lesions that progress, persist, or recur should be considered for
excisional biopsy specimen. (See Schwartz 11th ed., p. 625.)

2. Factors associated with increased incidence of head and Answer: D


neck cancers include all of the following EXCEPT: HPV is an epitheliotropic virus that has been detected to various
A. Human papillomavirus (HPV) exposure. degrees within samples of oral cavity squamous cell carcinoma.
B. Ultraviolet light exposure. Infection alone is not considered sufficient for malignant con-
C. Plummer-Vinson syndrome. version; however, results of multiple studies suggest a role for
D. Reflux esophagitis. HPV in a subset of head and neck squamous cell carcinoma.
Multiple reports reflect that up to 40% to 60% of current diag-
noses of tonsillar carcinoma demonstrate evidence of HPV
types 16 or 18. Environmental ultraviolet light exposure has
been associated with the development of lip cancer. The pro-
jection of the lower lip, as it relates to this solar exposure, has
been used to explain why the majority of squamous cell car-
cinomas arise along the vermilion border of the lower lip. In
addition, pipe smoking also has been associated with the devel-
opment of lip carcinoma. Factors such as mechanical irritation,
thermal injury, and chemical exposure have been described
as an explanation for this finding. Other entities associated
with oral malignancy include Plummer-Vinson syndrome
(achlorhydria, iron-deficiency anemia, mucosal atrophy of
mouth, pharynx, and esophagus), chronic infection with syph-
ilis, and immunocompromised status (30-fold increase with
renal transplant). (See Schwartz 11th ed., p. 630.)

143

Brunicardi_Ch18_p143-144.indd 143 30/06/22 11:21 AM


144
3. All of the following are TRUE about tracheostomy Answer: C
EXCEPT: The avoidance of prolonged orotracheal intubation decreases
A. Should be performed in patients anticipated to be the risk of laryngeal and subglottic injury and potential steno-
intubated more than 2 weeks. sis, facilitates oral and pulmonary suctioning, and decreases
B. Improves patient discomfort as compared to long- patient discomfort. When the tracheostomy is no longer
term oropharyngeal intubation. needed, the tube is removed and closure of the opening usu-
C. Usually spontaneously close within 2 months of ally occurs spontaneously over a 2-week period. Placement of
removal. a tracheostomy does not obligate a patient to loss of speech.
D. Does not obligate patient to loss of speech. When a large-cuffed tracheostomy tube is in place, expect-
CHAPTER 18

ing a patient to be capable of normal speech is impractical.


However, after a patient is downsized to an uncuffed trache-
ostomy tube, intermittent finger occlusion or Pass-Muir valve
placement will allow a patient to communicate while using
the tracheostomy to bypass the upper airway. (See Schwartz
11th ed., pp. 652–654.)
Head and Neck

4. Which of the following is TRUE concerning facial Answer: B


fractures? The most common facial fracture involves the mandible; the
A. The angle of the mandible is the most common site of most common sites of mandibular fracture include the con-
facial fracture. dyle (36%), body (35%), and angle (20%). Midface fractures
B. Zygoma fractures are typically displaced inferiorly are rarely isolated and include multiple subsites. However,
and/or medially. isolated zygoma fractures are typically displaced inferiorly
C. Midface fractures are classified as Le Fort I, II, III, or IV. and medially with disruption of the suture lines between the
D. Temporal bone fractures, especially with cerebrospi- temporal, frontal, and maxillary bones and the zygoma. There
nal fluid (CSF) leakage, usually require open repair. are three classic patterns of more extensive mid face fractures:
Le Fort I, II, and III. In patients with optic capsule involving
temporal bone fractures, typically caused by occipitomastoid
impact, sensorineural hearing loss, facial nerve paralysis, and
CSF leak are common. Regardless of the fracture pattern,
when a CSF leak is suspected, it usually resolves with conser-
vative management including bed rest, elevation of the head
of the bed, stool softeners, and avoiding sneezing or straining.
(See Schwartz 11th ed., p. 628.)

5. Concerning head and neck malignancy, which of the fol- Answer: B


lowing is FALSE? Continued smoking after completion of treatment is associ-
A. Risk of developing second primary or recurrent ated with a three- to fourfold increased risk of developing a
tumor is fourfold greater with continued smoking. second primary or recurrent tumor. Betel nut/quid chewing is
B. Lychee nut chewing predisposes to tumor endemic to some parts of Asia and India, and in these regions
development. oral cavity malignancy is one of the most common cancers.
C. Marijuana smoking predisposes to tumor In Europe and North America, there has been an increasing
development. interest in decriminalizing marijuana smoking; there is a
D. RNA virus exposure, such as human papillomavirus strong correlation between this activity and head and neck
(HPV), predisposes to tumor development. cancers. (See Schwartz, 11th ed., p. 630.)

6. The management of laryngeal cancer depends on all of Answer: D


the following EXCEPT: The primary management of laryngeal cancer depends on a
A. Stage of disease. variety of factors, including tumor extent, patient comorbidi-
B. Patient comorbidities. ties, and surgeon/center experience. Stage-specific treatment
C. Quality of life preferences. recommendations are not dependent on patient gender. (See
D. Patient gender. Schwartz 11th ed., p. 641.)

Brunicardi_Ch18_p143-144.indd 144 30/06/22 11:21 AM


CHAPTER 19
Chest Wall, Lung, Mediastinum, and Pleura

1. The tracheal blood supply includes all the following Answer: D


EXCEPT: The tracheal blood supply, which includes the inferior thyroid,
A. Inferior thyroid artery. subclavian, supreme intercostal, internal thoracic, innomi-
B. Subclavian artery. nate, and superior and middle bronchial arteries, enters the
C. Internal thoracic. airway near the junction of the membranous and cartilaginous
D. Common carotid artery. portions. Each arterial branch supplies a segment of 1.0 to
2.0 cm, thereby limiting circumferential mobilization to that
same distance. The vessels are interconnected along the lateral
surface of the trachea by an important longitudinal vascular
anastomosis that feeds transverse segmental vessels to the soft
tissues between the cartilages. The common carotid artery
does not give rise to an arterial blood supply to the trachea
(Fig. 19-1). (See Schwartz 11th ed., p. 663, Fig. 19-2.)

Inferior
thyroid a.
3
2
1
Lateral longitudinal
Branch from anastomosis
internal thoracic a.
Superior bronchial a.

Middle bronchial a.
FIG. 19-1. Arterial blood supply to the larynx and upper
trachea. a. = artery.

145

Brunicardi_Ch19_p145-156.indd 145 04/07/22 1:51 PM


146
Epiglottis

Internal
laryngeal n.
CHAPTER 19

Aryepiglottic m.

Transverse, oblique
arytenoid mm.

Lateral
Thyroepiglottic m.
cricoarytenoid m.
Chest Wall, Lung, Mediastinum, and Pleura

Posterior
Thyroarytenoid m.
cricoarytenoid m.

Thyroid cartilage
facet

Recurrent
FIG. 19-2. Anatomy of the larynx and upper trachea. laryngeal n. Cricothyroid m.
m. = muscle; n. = nerve. (cut)

2. The narrowest part of the trachea which measures Answer: B


approximately 2 cm in adults is: The trachea is composed of cartilaginous and membranous
A. Level of the carina. portions, beginning with the cricoid cartilage, the first com-
B. Subglottic space. plete cartilaginous ring of the airway. The cricoid cartilage
C. Level of cricoid cartilage. consists of an anterior arch and a posterior broad-based plate.
D. Mid trachea (~ 10th tracheal ring). Articulating with the posterior cricoid plate are the arytenoid
cartilages. The vocal cords originate from the arytenoid car-
tilages and then attach to the thyroid cartilage. The subglot-
tic space, the narrowest part of the trachea with an internal
diameter of approximately 2 cm, begins at the inferior surface
of the vocal cords and extends to the first tracheal ring. The
remainder of the distal trachea is 10.0 to 13.0 cm long, con-
sists of 18 to 22 rings, and has an internal diameter of 2.3 cm
(Fig. 19-2). (See Schwartz 11th ed., p. 663, Fig. 19-1.)

3. All of the following increase the risk for tracheal stenosis Answer: C
EXCEPT: Intubation-related risk factors include: prolonged intubation;
A. Age > 70 years. high tracheostomy through the first tracheal ring or crico-
B. Radiation. thyroid membrane; transverse rather than vertical incision
C. Male gender. on the trachea; oversized tracheostomy tube; prior tracheos-
D. Excessive corticosteroid therapy. tomy or intubation; and traumatic intubation. Stenosis is also
more common in older patients, in females, after radiation,
or after excessive corticosteroid therapy, and in the setting of
concomitant diseases such as autoimmune disorders, severe
reflux disease, or obstructive sleep apnea and the setting of
severe respiratory failure. However, even a properly placed
tracheostomy can lead to tracheal stenosis because of scar-
ring and local injury. Mild ulceration and stenosis are fre-
quently seen after tracheostomy removal. Use of the smallest
tracheostomy tube possible, rapid downsizing, and a vertical
tracheal incision minimize the risk for posttracheostomy ste-
nosis. (See Schwartz 11th ed., p. 663.)

Brunicardi_Ch19_p145-156.indd 146 04/07/22 1:51 PM


147
4. The tracheobronchial tree consists of approximately
23 airway divisions to the level of the alveoli. Mucus pro- The lung can be conveniently viewed as two linked compo-
duction in the airways is derived from which cell type? nents: The tracheobronchial tree (or conducting airways
A. Clara cells component) and the alveolar spaces (or gas exchange com-
B. Kulchitsky cells ponent). The tracheobronchial tree consists of approximately
C. Goblet cells 23 airway divisions to the level of the alveoli. It includes the
D. Type 1 pneumocytes main bronchi, lobar bronchi, segmental bronchi (to designated
bronchopulmonary segments), and terminal bronchioles
(ie, the smallest airways still lined by bronchial epithelium and

CHAPTER 19
without alveoli). The tracheobronchial tree is normally lined
by pseudostratified ciliated columnar cells and mucous (or
goblet) cells, which both derive from basal cells (Fig. 19-3).
Ciliated cells predominate. Goblet cells, which release mucus,
can significantly increase in number in acute bronchial
injury, such as exposure to cigarette smoke. The normal bron-
chial epithelium also contains bronchial submucosal glands,

Chest Wall, Lung, Mediastinum, and Pleura


which are mixed salivary-type glands containing mucous
cells, serous cells, and neuroendocrine cells called Kulchitsky
cells, which are also found within the surface epithelium. The
bronchial submucosal glands can give rise to salivary gland–
type tumors, including mucoepidermoid carcinomas and
adenoid cystic carcinomas. (See Schwartz 11th ed., p. 668.)

FIG. 19-3. Normal lung histology.


A. Pseudostratified ciliated columnar
cells and mucous cells normally line the
tracheobronchial tree. B. A Kulchitsky cell
is depicted (arrow). B

Brunicardi_Ch19_p145-156.indd 147 04/07/22 1:51 PM


148
5. Stepwise pathologic progression of normal lung cellular Answer: A
architecture to invasive adenocarcinoma follows which The incidence of adenocarcinoma in the lung has increased
of the following patterns? over the last several decades and is now the most common
A. Adenocarcinoma in-situ --> Minimally invasive ade- type of lung cancer. It occurs more frequently in females
nocarcinoma --> Lepidic predominate adenocarci- than in males and is the most frequent histologic subtype in
noma --> Invasive adenocarcinoma women, patients under the age of 45, and Asian populations.
B. Minimally invasive adenocarcinoma --> Adenocar- Adenocarcinoma can be divided into mucinous and non-
cinoma in-situ --> Lepidic predominate adenocarci- mucinous types. Based on consensus, the international working
noma --> Invasive adenocarcinoma group proposed a multidisciplinary approach, with standard-
CHAPTER 19

C. Adenocarcinoma in-situ --> Lepidic predominate ized criteria and terminology for diagnosis in cytologic and
adenocarcinoma --> Minimally invasive adenocarci- small biopsy specimens, and routine molecular testing for
noma --> Invasive adenocarcinoma known mutations, such as estimated glomerular filtration rate
D. Minimally invasive adenocarcinoma --> Lepidic pre- (EGFR) and KRAS mutations. The new classification system
dominate adenocarcinoma --> Adenocarcinoma in- delineated a stepwise pathologic progression, from Atypical
situ --> Invasive adenocarcinoma adenomatous hyperplasia (AAH) to invasive adenocarci-
noma based on the predominant histologic growth patterns;
Chest Wall, Lung, Mediastinum, and Pleura

the terms bronchioloalveolar carcinoma and mixed subtype


adenocarcinoma were eliminated in favor of more biologically
driven classification. (See Schwartz 11th ed., p. 669.)

6. The grade of neuroendocrine carcinoma (NEC) that is Answer: D


associated with hemoptysis, pneumonia, and tumor cells Grade I NEC (classic or typical carcinoid) is a low-grade
arranged in cords and clusters is: NEC; 80% arise in the epithelium of the central airways. It
A. Grade VI NEC. occurs primarily in younger patients. Because of the central
B. Grade IV NEC. location, it classically presents with hemoptysis, with or with-
C. Grade II NEC. out airway obstruction and pneumonia. Histologically, tumor
D. Grade I NEC. cells are arranged in cords and clusters with a rich vascular
stroma. This vascularity can lead to life-threatening hemor-
rhage with even simple bronchoscopic biopsy maneuvers.
Regional lymph node metastases are seen in 15% of patients,
but rarely spread systemically or cause death. (See Schwartz
11th ed., p. 672.)

7. CT imaging is used routinely in diagnosis of malignant Answer: A


lung cancer. CT imaging findings associated with malig- Spiral (helical) CT imaging is the gold standard for defini-
nancy include all the following EXCEPT: tive diagnosis of lung cancer. CT findings characteristic of
A. Size > 2 cm. malignancy include growth over time. Increasing density
B. Irregular, lobulated, or spiculated edges. on CT scan (40% to 50% of partial solid lesions are malig-
C. Corona radiata sign. nant compared to only 15% of subcentimeter solid or non-
D. Stippled or eccentric calcifications. solid nodules). Size > 3 cm. Irregular, lobulated, or spiculated
edges. The finding of the corona radiata sign (consisting of
fine linear strands extending 4 to 5 mm outward and appear-
ing spiculated on radiographs). Calcification that is stippled,
amorphous, or eccentric is usually associated with cancer.
(See Schwartz 11th ed., p. 678.)

8. The most common pattern of benign calcification in Answer: D


hamartomas is: CT findings characteristic of benign lesions include small
A. Solid. size, calcification within the nodule, and stability over time.
B. Diffuse. Four patterns of benign calcification are common: diffuse,
C. Central. solid, central, and laminated or “popcorn.” Granulomatous
D. Popcorn. infections such as tuberculosis can demonstrate the first
three patterns, whereas the popcorn pattern is most common
in hamartomas. In areas of endemic granulomatous disease,
differentiating benign versus malignant can be challenging.
Infectious granulomas arising from a variety of organisms
account for 70% to 80% of this type of benign solitary nod-
ules; hamartomas are the next most common single cause,
accounting for about 10%. (See Schwartz 11th ed., p. 685.)

Brunicardi_Ch19_p145-156.indd 148 04/07/22 1:51 PM


149
9. A patient diagnosed with lung cancer presents with Answer: D
symptoms of dyspnea with exertion, hiccups, and right Nonpulmonary thoracic symptoms are often due to tumor
shoulder pain. The most likely location of this tumor is: invasion into surrounding thoracic structures. This can lead
A. Left lower lobe central squamous cell carcinoma. to symptoms of Horner syndrome (Pancoast tumors), pericar-
B. Right upper lobe apical adenocarcinoma. dial tamponade, back pain, and recurrent laryngeal nerve palsy
C. Right lower lobe peripheral large cell carcinoma. leading to hoarseness and coughing. Phrenic nerve palsy is
D. Right middle lobe central squamous cell carcinoma. due to invasion of a tumor into the phrenic nerve. The phrenic
nerve traverses the hemithorax along the mediastinum, parallel
and posterior to the superior vena cava and anterior to the pul-

CHAPTER 19
monary hilum. Tumors at the medial lung surface or anterior
hilum can directly invade the nerve; symptoms include referred
shoulder pain, hiccups, and dyspnea with exertion because of
diaphragm paralysis. Radiographically, unilateral diaphragm
elevation on chest radiograph is present. The diagnosis can be
confirmed by fluoroscopic examination of the diaphragm with
paradoxical motion with breathing and sniffing, also known as

Chest Wall, Lung, Mediastinum, and Pleura


the “Sniff Test.” (See Schwartz 11th ed., p. 680.)

10. Which of the following is NOT a known predictive or Answer: C


prognostic tumor marker for adenocarcinoma? Lung cancer management.
A. Estimated glomerular filtration rate (EGFR) Role of histologic diagnosis and molecular testing. Establish-
B. KRAS mutation ing a clear histologic diagnosis early in the evaluation and
C. AFP (Alpha feto protein) management of lung cancer is critical to effective treatment.
D. EML4-ALK fusion gene Molecular signatures are also key determinants of treat-
ment algorithms for adenocarcinoma and will likely become
important for squamous cell carcinoma as well. Currently,
differentiation between adenocarcinoma and squamous cell
carcinoma in cytologic specimens or small biopsy specimens
is imperative in patients with advanced stage disease, as treat-
ment with pemetrexed or bevacizumab-based chemotherapy
is associated with improved progression-free survival in
patients with adenocarcinoma but not squamous cell can-
cer. Furthermore, life-threatening hemorrhage has occurred
in patients with squamous cell carcinoma who were treated
with bevacizumab. Finally, EGFR mutation predicts response
to EGFR tumor kinase inhibitors and is now recommended
as first-line therapy in advanced adenocarcinoma. Because
adequate tissue is required for histologic assessment and
molecular testing, each institution should have a clear, multi-
disciplinary approach to patient evaluation, tissue acquisition,
tissue handling/processing, and tissue analysis (Fig. 19-4).
In many cases, tumor morphology differentiates adenocar-
cinoma from the other histologic subtypes. If no clear mor-
phology can be identified, then additional testing for one
immunohistochemistry marker for adenocarcinoma and one
for squamous cell carcinoma will usually enable differentia-
tion. Immunohistochemistry for neuroendocrine markers is
reserved for lesions exhibiting neuroendocrine morphol-
ogy. Additional molecular testing should be performed on
all adenocarcinoma specimens for known predictive and
prognostic tumor markers (eg, EGFR, KRAS, and EML4-
ALK fusion gene). Ideally, use of tissue sections and cell
block material is limited to the minimum necessary at each
decision point. This emphasizes the importance of a multi-
disciplinary approach; surgeons and radiologists must work
in direct cooperation with the cytopathologist to ensure that
tissue samples are adequate for morphologic diagnosis as well
as providing sufficient cellular material to enable molecular
testing. (See Schwartz 11th ed., p. 683.)

Brunicardi_Ch19_p145-156.indd 149 04/07/22 1:51 PM


150
NE morphology, large cells, NSCLC,
STEP 1 NE IHC+
LCNEC
POSITIVE BIOPSY (FOB,
TBBx, Core, SLBx) NE morphology, small cells, no
nucleoli, NE IHC+, TTF-1 +/–, SCLC
CK+
POSITIVE CYTOLOGY
(effusion, aspirate, washings,
Keratinization, pearls
brushings)
and/or intercellular bridges
Classic Morphology:
SQCC
CHAPTER 19

Histology: Lepidic, papillary, and/or


acinar architecture(s)
Cytology: 3-D arrangements, delicate No clear ADC or
foamy/vacuolated (translucent) SQCC morphology:
cytoplasm,
Fine nuclear chromatin and often NSCLC-NOS
prominent nucleoli
Nuclei are often eccentrically situated
Chest Wall, Lung, Mediastinum, and Pleura

NSCLC, favor SQCC

Classic morphology:
ADC SQCC marker +ve
ADC marker –ve/or
ADC marker STEP 2 Mucin –ve
and/or
Mucin +ve; Apply ancillary panel of
SQCC One SQCC and one ADC marker
marker –ve +/OR Mucin
(or weak in
same cells)

ADC marker or Mucin +ve;


IHC –ve and as well as SQCC marker +ve
Mucin –ve in different cells

NSCLC, favor ADC


NSCLC NOS

NSCLC, NOS,
Molecular analysis: possible
eg, EGFR mutation† adenosquamous ca

STEP 3

If tumor tissue inadequate for molecular testing,


discuss need for further sampling — back to Step 1

FIG. 19-4. Algorithm for adenocarcinoma diagnosis in small biopsies and/or cytology. Step 1: When positive biopsies (fiberoptic
bronchoscopy [FOB], transbronchial [TBBx], core, or surgical lung biopsy [SLBx]) or cytology (effusion, aspirate, washings, and brushings)
show clear adenocarcinoma (ADC) or squamous cell carcinoma (SQCC) morphology, the diagnosis can be firmly established. If there is
neuroendocrine (NE) morphology, the tumor may be classified as small cell carcinoma (SCLC) or non-small-cell lung carcinoma (NSCLC),
probably large cell neuroendocrine carcinoma (LCNEC) according to standard criteria (+ = positive, − = negative, and ± = positive or
negative). If there is no clear ADC or SQCC morphology, the tumor is regarded as NSCLC -not otherwise specified (NOS). Step 2: NSCLC-NOS
can be further classified based on (a) immunohistochemical stains, (b) mucin (DPAS or mucicarmine) stains, or (c) molecular data. If the stains
all favor ADC-positive ADC marker(s) (ie, TTF-1 and/or mucin positive) with negative SQCC markers, then the tumor is classified as NSCLC,
favor ADC. If SQCC markers (ie, p63 and/or CK5/6) are positive with negative ADC markers, the tumor is classified as NSCLC, favor SQCC. If
the ADC and SQCC markers are both strongly positive in different populations of tumor cells, the tumor is classified as NSCLC-NOS, with a
comment it may represent adenosquamous carcinoma. If all markers are negative, the tumor is classified as NSCLC-NOS. †EGFR mutation
testing should be performed in (1) classic ADC, (2) NSCLC, favor ADC, (3) NSCLC-NOS, and (4) NSCLC-NOS, possible adenosquamous carcinoma.
In NSCLC-NOS, if EGFR mutation is positive, the tumor is more likely to be ADC than SQCC. Step 3: If clinical management requires a more
specific diagnosis than NSCLC-NOS, additional biopsies may be indicated. CD = cluster designation; CK = cytokeratin; DPAS = diastase-periodic
acid Schiff; DPAS +ve = periodic-acid Schiff with diastase; EGFR = epidermal growth factor receptor; IHC = immunohistochemistry; NB = of note;
TTF-1 = thyroid transcription factor-1; -ve = negative; +ve = positive. (Reproduced with permission from Travis WD, Brambilla E, Noguchi M, et al:
Diagnosis of lung cancer in small biopsies and cytology: implications of the 2011 International Association for the Study of Lung Cancer/American
Thoracic Society/European Respiratory Society classification, Arch Pathol Lab Med. 2013;137(5):668–684.)

Brunicardi_Ch19_p145-156.indd 150 04/07/22 1:51 PM


151
11. Evaluation of mediastinal lymphadenopathy by noninva- Answer: D
sive imaging is best performed by which of the following Mediastinal lymph node staging by PET scanning appears
modalities? to have greater accuracy than CT scanning. PET staging of
A. Positron emission tomography (PET) scan mediastinal lymph nodes has been evaluated in two meta-
B. CT scan analyses. The overall sensitivity for mediastinal lymph node
C. MRI(magnetic resonance imaging) metastasis was 79% (95% confidence interval [CI] 76%–82%),
D. PET-CT with a specificity of 91% (95% CI 89%–93%) and an accuracy
of 92% (95% CI 90%–94%). In comparing PET with CT scans
in patients who also underwent lymph node biopsies, PET

CHAPTER 19
had a sensitivity of 88% and a specificity of 91%, whereas CT
scanning had a sensitivity of 63% and a specificity of 76%.
Combining CT and PET scanning may lead to even greater
accuracy. In one study of CT, PET, and mediastinoscopy in
68 patients with potentially operable NSCLC, CT correctly
identified the nodal stage in 40 patients (59%). It understaged
the tumor in 12 patients and overstaged it in 16 patients. PET

Chest Wall, Lung, Mediastinum, and Pleura


correctly identified the nodal stage in 59 patients (87%). It
understaged the tumor in five patients and overstaged it in
four. For detecting N2 and N3 disease, the combination of
PET and CT scanning yielded a sensitivity, specificity, and
accuracy of 93%, 95%, and 94%, respectively. CT scan alone
yielded 75%, 63%, and 68%, respectively. Studies examining
combined PET-CT consistently show improved accuracy
compared to PET or CT alone; accuracy for PET-CT nodal
positivity confirmed by mediastinoscopy is approximately
75%, with a negative predictive value of approximately 90%.
(See Schwartz 11th ed., p. 687.)

12. A 57-year-old non-small-cell lung cancer patient with a Answer: A


potentially resectable tumor found on CT scan who can Assessment of functional status. Patients with potentially
walk on a flat surface indefinitely without oxygen or stop- resectable tumors require careful assessment of their func-
ping to rest, secondary to dyspnea will most likely tolerate: tional status and ability to tolerate either lobectomy or pneu-
A. Lobectomy. monectomy. The surgeon should first estimate the likelihood
B. Pneumonectomy. of pneumonectomy, lobectomy, or possibly sleeve resection,
C. Single-lung ventilation. based on the CT images. A sequential process of evaluation
D. Wedge resection. then unfolds. A patient’s history is the most important tool
for gauging risk. Specific questions regarding performance
status should be routinely asked. If the patient can walk on a
flat surface indefinitely, without oxygen and without having
to stop and rest secondary to dyspnea, he will be very likely
to tolerate lobectomy. If the patient can walk up two flights
of stairs (up two standard levels), without having to stop and
rest secondary to dyspnea, she will likely tolerate pneumo-
nectomy. Finally, nearly all patients, except those with carbon
dioxide (CO2) retention on arterial blood gas analysis, will be
able to tolerate periods of single-lung ventilation and wedge
resection. (See Schwartz 11th ed., p. 691.)

Brunicardi_Ch19_p145-156.indd 151 04/07/22 1:51 PM


152
13. A 45-year-old man with 20 pack-year history of smok- Answer: D
ing is diagnosed with squamous cell carcinoma of the The staging of solid epithelial tumors is based on the TNM
right middle lobe. Clinically the patient has symptoms of staging system. The primary tumor “T” status provides
hoarseness and coughing when drinking liquids. The T information about tumor size and relationship to surround-
staging for this tumor is: ing structures; the “N” status provides information about
A. T2a. regional lymph nodes; and the “M” status provides informa-
B. T2b. tion about the presence or absence of metastatic disease. The
C. T3. designation of lymph nodes as N1, N2, or N3 requires famil-
D. T4. iarity with the lymph node mapping system. Based on clearly
CHAPTER 19

delineated anatomic boundaries, accurate and reproducible


localization of thoracic lymph nodes is possible, facilitating
detailed nodal staging for individual patients and standard-
ization of nodal assessment between surgeons. In the case
described, the patient’s symptoms of dysphagia and hoarseness
suggests involvement of the recurrent laryngeal nerve, which
confers him a T4 designation in the TNM classification system.
Chest Wall, Lung, Mediastinum, and Pleura

(See Schwartz 11th ed., p. 689.)

14. Patients with resectable tumors require assessment of Answer: A


their pulmonary functional status to determine if they Pulmonary function studies are routinely performed when
are able to tolerate surgical resection. The two most reli- any resection greater than a wedge resection will be per-
able predictors of pulmonary functional capacity are: formed. Of all the measurements available, the two most
A. FEV1 and DLCO. valuable are forced expiratory volume in 1 second (FEV1) and
B. RV and TLC. carbon monoxide diffusion capacity (DLCO). General guide-
C. RV and DLCO. lines for the use of FEV1 in assessing the patient’s ability to tol-
D. FEV1 and RV. erate pulmonary resection are as follows: Greater than 2.0 L
can tolerate pneumonectomy, and >1.5 L can tolerate lobec-
tomy. It must be emphasized that these are guidelines only. It
is also important to note that the raw value is often imprecise
because normal values are reported as “percent predicted”
based on corrections made for age, height, and gender. Nota-
bly it is not uncommon to encounter patients with significant
reductions in FEV1 and DLCO that are inconsistent with the
patient’s functional status. In these patients,
. exercise testing
that yields maximal oxygen consumption (VO2max) has emerged
as a valuable decision-making technique to help patients
with abnormal FEV1 and DLCO. (See Schwartz 11th ed.,
p. 692.)

15. Which of the following is an indication for surgical Answer: B


drainage of a lung abscess? Surgical drainage of lung abscesses is uncommon since drain-
A. Abscess > 3 cm in diameter age usually occurs spontaneously via the tracheobronchial
B. Hemoptysis tree. Indications for intervention are listed in Table 19-1. (See
C. Failure to decrease in size after 1 week of antibiotic Schwartz 11th ed., p. 708.)
therapy
D. Persistent fever
TABLE 19-1 Indications for surgical drainage procedures
for lung abscesses
1. Failure of medical therapy
2. Abscess under tension
3. Abscess increasing in size during appropriate treatment
4. Contralateral lung contamination
5. Abscess > 4–6 cm in diameter
6. Necrotizing infection with multiple abscesses, hemoptysis, abscess
rupture, or pyopneumothorax
7. Inability to exclude a cavitating carcinoma

Brunicardi_Ch19_p145-156.indd 152 04/07/22 1:51 PM


153
16. The most likely cause of aspiration pneumonia is: Answer: C
A. A mixture of aerobes and anaerobes. Microbiology. Normal oropharyngeal secretions con-
B. Aerobes only. tain many more Streptococcus species and more anaer-
C. Anaerobes only. obes (approximately 1 × 108 organisms/mL) than aerobes
D. Gram-negative bacteria. (approximately 1 × 107organisms/mL). Pneumonia that fol-
lows from aspiration, with or without abscess development,
is typically polymicrobial. An average of two to four iso-
lates present in large numbers have been cultured from lung
abscesses sampled percutaneously. Overall, at least 50% of

CHAPTER 19
these infections are caused by purely anaerobic bacteria, 25%
are caused by mixed aerobes and anaerobes, and 25% or fewer
are caused by aerobes only. In nosocomial pneumonia, 60% to
70% of the organisms are gram-negative bacteria, including
Klebsiella pneumoniae, Haemophilus influenzae, Proteus spe-
cies, Pseudomonas aeruginosa, Escherichia coli, Enterobacter
cloacae, and Eikenella corrodens. Immunosuppressed patients

Chest Wall, Lung, Mediastinum, and Pleura


may develop abscesses because of the usual pathogens as well
as less virulent andopportunistic organisms such as Salmo-
nella species, Legionella species, Pneumocystis carinii, atypical
mycobacteria, and fungi. (See Schwartz 11th ed., p. 708.)

17. The population most at risk for developing active tuber- Answer: D
culosis is: Epidemiology. Tuberculosis is a widespread problem that
A. Elderly. affects nearly one-third of the world’s population. Between
B. Minorities. 8.3 and 9 million new cases of tuberculosis and 12 million
C. Urban residents. prevalent cases (range 10–13 million) were estimated world-
D. HIV infection. wide in 2011 according to the World Health Organization.
Only 10,521 new cases were reported to the World Health
Organization in the United States in 2011. HIV infection is
the strongest risk factor for developing active tuberculosis.
The elderly, minorities, and recent immigrants are the most
common populations to have clinical manifestations of infec-
tion, yet no age group, sex, or race is exempt from infection.
In most large urban centers, reported cases of tuberculosis are
more numerous among the homeless, prisoners, and drug-
addicted populations. Immunocompromised patients addi-
tionally contribute to an increased incidence of tuberculosis
infection, often developing unusual systemic as well as pul-
monary manifestations. (See Schwartz 11th ed., p. 710.)

18. The fungi associated with the highest mortality rate due Answer: A
to invasive mycoses in the United States is: Aspergillosis. The genus Aspergillus comprises over 150 species
A. Aspergillus. and is the most common cause of mortality due to invasive
B. Cryptococcus. mycoses in the United States. It is typically acute in onset
C. Candidia. and life-threatening and occurs in the setting of neutropenia,
D. Mucor. chronic steroid therapy, or cytotoxic chemotherapy. It can also
occur in the general intensive care unit population of critically ill
patients, including patients with underlying chronic obstructive
pulmonary disease (COPD), postoperative patients, patients
with cirrhosis or alcoholism, and postinfluenza patients, with-
out any of these factors present. The species most commonly
responsible for clinical disease include A. fumigatus, A. flavus,
A. niger, and A. terreus. Aspergillus is a saprophytic, filamen-
tous fungus with septate hyphae. Spores (2.5–3 μm in diameter)
are released and easily inhaled by susceptible patients; because
the spores are microns in size, they are able to reach the distal
bronchi and alveoli. (See Schwartz 11th ed., p. 711.)

Brunicardi_Ch19_p145-156.indd 153 04/07/22 1:51 PM


154
19. Previously, patients with severe primary pulmonary Answer: A
hypertension were listed for lung transplantation due to Currently, virtually all patients with primary pulmonary hyper-
respiratory failure. The use of the following medication tension are now treated with intravenous epoprostenol. Prosta-
has significantly altered the need for transplantation: cyclin is a member of the endogenous prostanoid family and is
A. Epoprostenol. produced from arachidonic acid by the enzymes prostacyclin
B. Midodrine. synthase and cyclooxygenase (COX). It acts to increase pul-
C. Dobutamine. monary vasodilation within the lungs. Espoprostenol must be
D. Nitrates. given by continuous infusion. (See Schwartz 11th ed., p. 719.)

Answer: A
CHAPTER 19

20. The most common benign chest wall tumor is:


A. Chondromas. Chondroma. Chondromas, seen primarily in children and
B. Osteochondromas. young adults, are one of the more common benign tumors of
C. Desmoid tumors. the chest wall. They usually occur at the costochondral junc-
D. Fibrous dysplasia. tion anteriorly and may be confused with costochondritis,
except that a painless mass is present. Radiographically, lesion
is lobulated and radiodense; it may have diffuse or focal cal-
cifications; and it may displace the bonycortex without pen-
Chest Wall, Lung, Mediastinum, and Pleura

etration. Chondromas may grow to huge sizes if left untreated.


Treatment is surgical resection with a 2-cm margin. Large
chondromas may harbor well-differentiated chondrosarcoma
and should be managed with a 4-cm margin to prevent local
recurrence. (See Schwartz 11th ed., p. 722.)

21. An “onion-peel” appearance of a rib on computed Answer: B


tomography is suggestive of: Primitive neuroectodermal tumors (PNETs) and Ewing sar-
A. Chondroma. coma. PNETs (neuroblastomas, ganglioneuroblastomas, and
B. Ewing sarcoma. ganglioneuromas) derive from primordial neural crest cells
C. Plasmacytoma. that migrate from the mantle layer of the developing spinal
D. Osteosarcoma. cord. Histologically, PNETs and Ewing sarcomas are small,
round cell tumors; both possess a translocation between the
long arms of chromosomes 11 and 22 within their genetic
makeup. They also share a consistent pattern of proto-
oncogene expression and have been found to express the
product of the MIC2 gene. Ewing sarcoma occurs in adoles-
cents and young adults who present with progressive chest
wall pain, but without the presence of a mass. Systemic symp-
toms of malaise and fever are often present. Laboratory stud-
ies reveal an elevated erythrocyte sedimentation rate and
mild white blood cell elevation.
Radiographically, the characteristic onion peel appearance
is produced by multiple layers of periosteum in the bone for-
mation. Evidence of bony destruction is also common. The
diagnosis can be made by a percutaneous needle biopsy or an
incisional biopsy. (See Schwartz 11th ed., p. 725.)

22. Desmoid tumors are associated with which of the follow- Answer: D
ing gene? Desmoid tumors. Soft tissue neoplasms arising from fascial
A. KRAS or musculoaponeurotic structures, desmoid tumors consist
B. RET of proliferations of benign appearing fibroblastic cells, abun-
C. P53 dant collagen, and few mitoses. Desmoid tumors possess
D. Adenomatous polyposis coli (APC) alterations in the APC/β-catenin pathway. Cyclin D1 dys-
regulation is thought to play a significant role in their patho-
genesis. Associations with other diseases and conditions are
well documented, especially those with similar alterations in
the APC pathway, such as familial adenomatous polyposis
(Gardner syndrome). Other conditions with increased risk of
desmoid tumor formation include increased estrogen states
(pregnancy) and trauma. Surgical incisions (abdominal and
thorax) have been the site of desmoid development, either in
or near the scar. (See Schwartz 11th ed., p. 722.)

Brunicardi_Ch19_p145-156.indd 154 04/07/22 1:51 PM


155
23. Eosinophilic granulomas are associated with: Answer: A
A. Langerhans cell histiocytosis. Eosinophilic granulomas are benign osteolytic lesions. Eosin-
B. Parasitic infections. ophilic granulomas of the ribs can occur as solitary lesions or
C. Crohn disease. as part of a more generalized disease process of the lympho-
D. Gardner syndrome. reticular system termed Langerhans cell histiocytosis (LCH).
In LCH, the involved tissue is infiltrated with large numbers
of histiocytes (similar to Langerhans cells seen in skin and
other epithelia), which are often organized as granulomas. The
cause is unknown. Of all LCH bone lesions, 79% are solitary

CHAPTER 19
eosinophilic granulomas, 7% involve multiple eosinophilic
granulomas, and 14% belong to other forms of more systemic
LCH. Isolated single eosinophilic granulomas can occur in the
ribs or skull, pelvis, mandible, humerus, and other sites. They
are diagnosed primarily in children between the ages of 5 and
15 years. Because of the associated pain and tenderness, they
may be confused with Ewing sarcoma or with an inflamma-

Chest Wall, Lung, Mediastinum, and Pleura


tory process such as osteomyelitis. Healing may occur sponta-
neously, but the typical treatment is limited surgical resection
with a 2-cm margin. (See Schwartz 11th ed., p. 722.)

24. A patient with an anterior mediastinal mass and elevated Answer: B


serum alpha fetoprotein most likely has: The use of serum markers to evaluate a mediastinal mass can
A. A teratoma. be invaluable in some patients. For example, nonseminomatous
B. A nonseminomatous germ-cell tumor. and seminomatous germ-cell tumors can frequently be diag-
C. A seminomatous germ-cell tumor. nosed and often distinguished from one another by the levels of
D. Metastatic hepatocellular carcinoma. α-fetoprotein (AFP) and human chorionic gonadotropin (hCG).
In >90% of nonseminomatous germ-cell tumors, either the
AFP or the hCG level will be elevated. Results are close to 100%
specific if the level of either AFP or hCG is >500 ng/mL. Some
centers institute chemotherapy based on this result alone, with-
out biopsy confirmation of the diagnosis. In contrast, the AFP
level in patients with mediastinal seminoma is always normal;
only 10% will have elevated hCG, which is usually <100 ng/mL.
Other serum markers, such as intact parathyroid hormone level
for ectopic parathyroid adenomas, may be useful for diagnos-
ing and also for intraoperatively confirming complete resection.
After successful resection of a parathyroid adenoma, this hor-
mone level should rapidly normalize. (See Schwartz 11th ed.,
p. 728.)

25. What is the most common anterior mediastinal tumor in Answer: C


adults? The most common tumor of the anterior mediastinum is
A. Lymphoma thymoma. Thymomas are histologically characterized by a
B. Germ-cell tumor mixture of epithelial cells and mature lymphocytes. Grossly
C. Thymoma they are well encapsulated. Between 10% and 50% of patients
D. Fibrosarcoma with thymoma will have symptoms suggestive of myasthenia
gravis or have circulating antibodies to acetylcholine recep-
tors. Most patients with thymomas are asymptomatic. Thy-
mectomy leads to improvement or resolution of symptoms
of myasthenia gravis in only about 25% of patients with thy-
momas. In contrast, in patients with myasthenia gravis and
no thymoma, thymectomy results are superior: up to 50% of
patients have a complete remission, and 90% improve. In 5%
of patients with thymomas, other paraneoplastic syndromes,
including red cell aplasia, hypogammaglobulinemia, systemic
lupus erythematosus, Cushing syndrome, or SIADH, may be
present. Large thymic tumors may present with symptoms
related to a mass effect, which may include cough, chest pain,
dyspnea, or SVC syndrome. (See Schwartz 11th ed., p. 730.)

Brunicardi_Ch19_p145-156.indd 155 04/07/22 1:51 PM


156
26. A chylothorax is likely to be present in a patient whose Answer: D
pleural fluid analysis results show a triglyceride level of: Laboratory analysis of the pleural fluid shows a high lym-
A. 80 mg/100 mL. phocyte count and high triglyceride levels. If the triglyceride
B. 100 mg/100 mL. level is >110 mg/100 mL, a chylothorax is almost certainly
C. 45 mg/100 mL. present (a 99% accuracy rate). If the triglyceride level is
D. 130 mg/100 mL. <50 mg/mL, there is only a 5% chance of chylothorax. (See
Schwartz 11th ed., p. 741.)

27. All of the following are associated with increased risk of Answer: B
CHAPTER 19

mesothelioma EXCEPT: Malignant mesothelioma is the most common type of pri-


A. Asbestos exposure. mary tumor of the pleura, with approximately 3000 cases per
B. Smoking. year in the United States. Other, less common tumors include
C. Age > 40. benign and malignant fibrous tumors of the pleura, lipomas,
D. Male gender. and cysts. The only known risk factor for mesothelioma is
exposure to asbestos, identified in >50% of cases. Exposure
is typically work-related in industries using asbestos in the
Chest Wall, Lung, Mediastinum, and Pleura

manufacturing process, such as shipbuilding and brake pad


linings. The risk extends to family members who are exposed
to the dust of the clothing or to the work environment. Asbes-
tos exposure and smoking synergistically increase the risk for
lung cancer, but smoking does not increase risk for malignant
mesotheliomas. Male predominance is 2:1, and it occurs most
commonly after the age of 40. (See Schwartz 11th ed., p. 743.)

Brunicardi_Ch19_p145-156.indd 156 04/07/22 1:51 PM


CHAPTER 20
Congenital Heart Disease

1. The most common form of atrial septal defect (ASD) is: Answer: C
A. Sinus venosus defect. ASDs can be classified into three different types (Fig. 20-1):
B. Ostium primum defect. (a) ostium secundum type defect (Fig. 20-1B,C) (deficiency
C. Ostium secundum defect. of septum primum), which are the most prevalent subtype,
D. Combined primum and secundum defect. comprising 80% of all ASDs; (b) ostium primum defects
(Fig. 20-1A), which may also be described as partial or tran-
sitional AV canal defect; and (c) sinus venosus type defects,
comprising approximately 5% to 10% of all ASDs. (See
Schwartz 11th ed., p. 752.)

A B

FIG. 20-1. A. Echocardiogram of a patient with primum


type artial septal defect (`*’ points to the atrial septal
defect). B. Echocardiogram of a large secundum type
ASD (`*’ points to the defect). C. Intraoperative picture
during repair of atrial septal defect. A large fenestrated
atrial septum is seen. Bicaval venous cannulation has
been performed and a right atriotomy provides exposure
to the atrial septum. C

157

Brunicardi_Ch20_p157-170.indd 157 06/07/22 9:41 AM


158
2. The most common age to close asymptomatic atrial sep- Answer: C
tal defects (ASDs) is: In general, ASDs are closed when patients are between 4 and
A. In the immediate newborn period. 5 years of age. Children of this size can usually be operated on
B. After the child reaches 10 kg in weight. without the use of blood transfusion and have excellent out-
C. Age 4–5 years. comes. Patients who are symptomatic may require repair ear-
D. During puberty. lier, even in infancy. Some surgeons advocate routine repair
in infants and children especially in cases where prematurity-
related lung disease may accelerate damage to the pulmonary
vascular bed, though this philosophy may not be widespread.
CHAPTER 20

In a review by Reddy and colleagues, 116 neonates weighing


less than 2500 g who underwent repair of simple and complex
cardiac defects with the use of cardiopulmonary bypass were
found to have no intracerebral hemorrhages, no long-term
neurologic sequelae, and a low operative mortality rate (10%).
These results correlated with the length of cardiopulmonary
bypass and the complexity of repair. These investigators also
Congenital Heart Disease

found an 80% actuarial survival at 1 year and, more impor-


tantly, that growth following complete repair was equivalent
to weight-matched neonates free from cardiac defects. (See
Schwartz 11th ed., p. 754.)

3. Which of the following is not acceptable treatment for Answer: A


aortic valve stenosis with a hypoplastic left ventricle? The first decision that must be made in the neonate with
A. Balloon valvotomy critical left ventricular outflow tract (LVOT) obstruction is
B. Intubation and initiation of prostaglandin whether the patient is a candidate for biventricular or univen-
C. Surgical valvotomy tricular repair. Central to this decision is assessment of the
D. Norwood procedure degree of hypoplasia of the LV and other left-sided structures.
Alsoufi and colleagues have described a rational approach to
the neonate with critical LVOT obstruction. The options vary
depending on whether the infant follows a single or a biven-
tricular pathway. The options for a single ventricle include the
Norwood operation, a hybrid strategy (initial ductal stent and
bilateral pulmonary artery bands followed by later comple-
tion of the Norwood operation) or heart transplantation. The
options for a biventricular heart include balloon valvuloplasty,
surgical valvotomy, neonatal Ross operation, or a Yasui oper-
ation. Often valvotomy is accompanied by LV rehabilitation
techniques, including endocardial fibroelastosis (EFE) resec-
tion and mitral valve interventions. Fetal aortic valvotomy,
which is now offered at specialized centers, is another prom-
ising strategy to decompress the LV in fetal life and potentially
allow growth of the left-sided structures sufficient to permit
a biventricular circulation. Regardless of whether the baby is
triaged to a single or biventricular strategy, any infant with
severe aortic stenosis (AS) requires urgent intervention. Pre-
operative stabilization, however, has dramatically altered the
clinical algorithm and outcomes for this patient population.
The preoperative strategy begins with endotracheal intuba-
tion and inotropic support. Prostaglandin infusion is initi-
ated to maintain ductal patency, and confirmatory studies are
performed prior to operative intervention. Therapy is gener-
ally indicated in the presence of a transvalvular gradient of
50 mm Hg with associated symptoms including syncope, con-
gestive heart failure (CHF), or angina, or if a gradient of 50 to
75 mm Hg exists with concomitant electrocardiography
(ECG) evidence of LV strain or ischemia. In the critically ill
neonate, a gradient across the aortic valve may not be present
because of poor LV function. However, the decision regarding
treatment options must be based on a complete understanding

Brunicardi_Ch20_p157-170.indd 158 06/07/22 9:41 AM


159
of associated defects. For example, in the presence of a hypo-
plastic LV (left ventricular end-diastolic volume < 20 mL/m2)
or a markedly abnormal mitral valve, isolated aortic valvot-
omy should not be performed because studies have demon-
strated high mortality in this population following isolated
valvotomy.
Patients who have an LV capable of providing systemic
output are candidates for intervention to relieve AS, generally
through balloon valvotomy. (See Schwartz 11th ed., p. 757.)

CHAPTER 20
4. The most common location for a coarctation of the aorta Answer: B
is: Coarctation of the aorta (COA) is defined as a luminal nar-
A. Aortic arch. rowing in the aorta that causes an obstruction to blood flow.
B. Distal to the left subclavian artery. This narrowing is most commonly located distal to the left
C. At the diaphragm. subclavian artery. The embryologic origin of COA is a sub-
D. At the level of the renal arteries. ject of some controversy. One theory holds that the obstruct-

Congenital Heart Disease


ing shelf, which is largely composed of tissue found within
the ductus, forms as the ductus involutes. The other theory
holds that a diminished aortic isthmus develops secondary to
decreased aortic flow in infants with enhanced ductal circula-
tion. (See Schwartz 11th ed., pp. 761– 762.)

5. Which of the following is a TRUE surgical emergency in Answer: C


a newborn? Total anomalous pulmonary venous connection (TAPVC)
A. Tetralogy of Fallot occurs in 1% to 2% of all cardiac malformations and is charac-
B. Truncus arteriosus terized by abnormal drainage of the pulmonary veins into the
C. Total anomalous pulmonary venous connection right heart, whether through connections into the right atrium
D. Coarctation of the aorta or into its tributaries. Accordingly, the only mechanism by
which oxygenated blood can return to the left heart is through
an ASD, which is almost uniformly present with TAPVC.
Unique to this lesion is the absence of a definitive form
of palliation. Thus, TAPVC with concomitant obstruction
(Fig. 20-2) represents one of the only true surgical emergen-
cies across the entire spectrum of congenital heart surgery.
(See Schwartz 11th ed., pp. 765– 768.)

FIG. 20-2. Infracardiac type of TAPVR. Note the stenosis (`*’) of


the descending vertical vein as it drains into the portal system.

Brunicardi_Ch20_p157-170.indd 159 06/07/22 9:41 AM


160
6. The Bidirectional Glenn procedure is used to correct: Answer: A
A. Tricuspid atresia. Recognizing the inadequacies of the initial repairs, Glenn
B. Patent ductus arteriosus. described the first successful cavopulmonary anastomo-
C. Transposition of the great arteries. sis, an end-to-side right pulmonary artery-to-superior vena
D. Total anomalous pulmonary venous connection. cava shunt in 1958, and later modified this to allow flow to
both pulmonary arteries. This end-to-side right pulmonary
artery-to-superior vena cava anastomosis was known as the
bidirectional Glenn, and it is the first stage to final Fon-
tan repair in widespread use today (Fig. 20-3). The Fontan
CHAPTER 20

repair was a major advancement in the treatment of con-


genital heart defect (CHD), as it essentially bypassed the
right heart and allowed separation of the pulmonary and
systemic circulations. It was first performed by Fontan in
1971 and consisted of a classic Glenn anastomosis, ASD clo-
sure, and direct connection of the right atrium to the proximal
end of the left pulmonary artery using an aortic homograft.
Congenital Heart Disease

The main pulmonary artery was ligated, and a homograft


valve was inserted into the orifice of the inferior vena cava.
(See Schwartz 11th ed., pp. 770– 773.)

FIG. 20-3. Angiogram showing a widely patent Glenn. The SVC


(`*’) is seen draining into the central pulmonary artery.

7. Hypoplastic left heart syndrome (HLHS) is surgically Answer: D


treated with: In 1983, Norwood and colleagues described a two-stage pal-
A. Bilateral pulmonary artery banding and stent place- liative surgical procedure for relief of HLHS that was later
ment in the patent ductus arteriosus. modified to the currently used three-stage method of pallia-
B. Norwood procedure with a Blalock-Taussig (B-T) tion. Stage 1 palliation, also known as the modified Norwood
shunt. procedure, bypasses the LV by creating a single outflow ves-
C. Norwood procedure with a right ventricle to pulmo- sel, the neoaorta, which arises from the RV.
nary artery conduit (Sano shunt). The current technique of arch reconstruction involves
D. All of the above. completion of a connection between the pulmonary root,
the native ascending aorta, and a piece of pulmonary homo-
graft used to augment the diminutive native aorta. There are
several modifications of this anastomosis, most notably the
Damus-Kaye-Stansel (DKS) anastomosis, which involves
dividing both the aorta and the pulmonary artery at the sino-
tubular junction. The proximal aorta is anastomosed to the
proximal pulmonary artery, creating a “double-barreled” out-
let from the heart. This outlet is anastomosed to the distal
aorta, which can be augmented with homograft material if
there is an associated coarctation. At the completion of arch
reconstruction, a 3.5- or 4-mm shunt is placed from the

Brunicardi_Ch20_p157-170.indd 160 06/07/22 9:41 AM


161
innominate artery to the right pulmonary artery. The inter-
atrial septum is then widely excised, thereby creating a large
interatrial communication and preventing pulmonary venous
hypertension. (See Schwartz 11th ed., pp. 773–775.)

8. The arterial switch operation for transposition of the Answer: A


great vessels is best performed: The most important consideration is the timing of surgical
A. Within 2 weeks of birth. repair because arterial switch should be performed within
B. At 1 year of age. 2 weeks after birth, before the left ventricle (LV) loses its abil-

CHAPTER 20
C. At 10 kg of weight. ity to pump against systemic afterload. In patients presenting
D. In adolescence. later than 2 weeks, the LV can be retrained with preliminary
pulmonary artery banding and aortopulmonary shunt fol-
lowed by definitive repair. Alternatively, the unprepared LV
can be supported following arterial switch with a mechanical
assist device for a few days while it recovers ability to manage
systemic pressures. Echocardiography can be used to assess

Congenital Heart Disease


left ventricular performance and guide operative planning in
these circumstances. (See Schwartz 11th ed., pp. 780–781.)

9. Which of the following is NOT one of the components of Answer: A


the tetralogy of Fallot (TOF)? The original description of tetralogy of Fallot (TOF) by
A. Atrial septal defect Ettienne Louis Fallot, as the name implies, included four
B. Ventricular septal defect (VSD) abnormalities: a large perimembranous VSD adjacent to the
C. Right ventricular hypertrophy tricuspid valve; an overriding aorta; a variable degree of right
D. Right ventricular outflow obstruction ventricular outflow tract (RVOT) obstruction, which might
include hypoplasia and dysplasia of the pulmonary valve as
well as obstruction at the subvalvar and pulmonary artery
level; and right ventricular hypertrophy. More recently, the
Van Praagh et al pointed out that TOF could be more cor-
rectly termed monology of Fallot, since the four components
are explained by the malposition of the infundibular septum.
When the infundibular septum is displaced anteriorly and
leftward, the RVOT is narrowed and its anterior displacement
results in failure of fusion of the ventricular septum between
the arms of the trabeculo-septo-marginalis. (See Schwartz
11th ed., p. 784.)

10. What is the best predictor of spontaneous closure of a Answer: B


ventricular septal defect (VSD)? VSDs may close or narrow spontaneously, and the probability
A. Size of closure is inversely related to the age at which the defect is
B. Age at diagnosis observed. Thus, infants at 1 month of age have an 80% inci-
C. Gestational age dence of spontaneous closure, whereas a child at 12 months
D. Lack of electrocardiogram changes of age has only a 25% chance of closure. This has an impor-
tant impact on operative decision-making because a small or
moderate-size VSD may be observed for a period of time in
the absence of symptoms. Large defects and those in severely
symptomatic neonates should be repaired during infancy to
relieve symptoms and because irreversible changes in pulmo-
nary vascular resistance may develop during the first year of
life. (See Schwartz 11th ed., pp. 786–787.)

Brunicardi_Ch20_p157-170.indd 161 06/07/22 9:41 AM


162
11. Flow across a ventricular septal defect (VSD) is depen- Answer: C
dent upon: The size of the VSD determines the initial pathophysiology
A. Size of defect. of the disease. Large VSDs are classified as nonrestrictive and
B. Left and right ventricular pressure and size of the are at least equal in diameter to the aortic annulus. These
defect. defects allow free flow of blood from the left ventricle (LV) to
C. Pulmonary and systemic vascular resistance and the right ventricle (RV), elevating right ventricular pressures
defect size. to the same level as systemic pressure.
D. Pulmonary and systemic vascular resistance. Consequently, the pulmonary-to-systemic flow ratio (Qp to
Qs) is inversely dependent on the ratio of pulmonary vascular
CHAPTER 20

resistance to systemic vascular resistance. Nonrestrictive VSDs


produce a large increase in pulmonary blood flow, and the
afflicted infant will present with symptoms of congestive heart
failure. However, if untreated, these defects will cause pulmo-
nary hypertension with a corresponding increase in pulmonary
vascular resistance. This will lead to a reversal of flow (a right-
to-left shunt), which is known as Eisenmenger syndrome.
Congenital Heart Disease

Small restrictive VSDs offer significant resistance to the


passage of blood across the defect, and therefore right ven-
tricular pressure is either normal or only minimally elevated
and the ratio of Qp to Qs rarely exceeds 1.5. These defects
are generally asymptomatic because there are few physiologic
consequences. However, there is a long-term risk of endo-
carditis because endocardial damage from the jet of blood
through the defect may serve as a possible nidus for coloniza-
tion. (See Schwartz 11th ed., pp. 786–787.)

12. Beyond early childhood, high pulmonary blood flow is Answer: B


most apt to produce: High pulmonary blood flow beyond infancy may pro-
A. Cyanosis on exercise. duce surprisingly little disability for a period of time, and
B. Diminished exercise tolerance. the diminished exercise tolerance may be subtle. Cyanosis,
C. Periodic episodes of hemoptysis. hemoptysis, and pneumonia are not anticipated. With the
D. Right ventricular hypertrophy. volume overloading in the right ventricle, ventricular dila-
tation is more common than ventricular hypertrophy. (See
Schwartz 11th ed., p. 751.)

13. During left thoracotomy for repair of patent ductus arte- Answer: B
riosus the blood pressure is 70/22. Immediately after The hemodynamic consequences of an unrestrictive ductal
placement of a clip across the duct the blood pressure is: shunt are left ventricular volume overload with increased left
A. 70/22. atrial and pulmonary artery pressures and right ventricular
B. 70/40. strain from the augmented afterload. These changes result
C. 90/22. in increased sympathetic discharge, tachycardia, tachypnea,
D. 90/40. and ventricular hypertrophy. The diastolic shunt results in
lower aortic diastolic pressure and increases the potential for
myocardial ischemia and underperfusion of other systemic
organs, while the increased pulmonary flow leads to increased
work of breathing and decreased gas exchange. Unrestrictive
ductal flow may lead to pulmonary hypertension within the
first year of life (See Schwartz 11th ed., pp. 759–760.)

Brunicardi_Ch20_p157-170.indd 162 06/07/22 9:41 AM


163
14. A 5-day-old man undergoes echocardiography during Answer: A
preoperative work-up of a tracheoesophageal fistula. Subvalvular aortic stenosis (AS) occurs beneath the aortic
The patient is found to have subvalvular aortic stenosis valve and may be classified as discrete or tunnel-like (diffuse).
with a left ventricular (LV) gradient of 20 mm Hg but is A thin, fibromuscular diaphragm immediately proximal to
otherwise asymptomatic and without aortic valve insuf- the aortic valve characterizes discrete subaortic stenosis. This
ficiency. What is the next best step in management of his diaphragm typically extends for 180° or more in a crescen-
congenital heart defect (CHD)? tic or circular fashion, often attaching to the mitral valve as
A. Balloon valvotomy well as the interventricular septum. The aortic valve itself is
B. Observation with follow-up usually normal in this condition, although the turbulence

CHAPTER 20
C. Aortic 1-patch repair (Doty procedure) imparted by the subvalvular stenosis may affect leaflet mor-
D. Aortoventriculoplasty phology and valve competence.
Diffuse subvalvular AS results in a long, tunnel-like obstruc-
tion that may extend to the left ventricular apex. In some
individuals, there may be difficulty in distinguishing between
hypertrophic cardiomyopathy and diffuse subaortic stenosis.
Operation for subvalvular AS is indicated with a gradient

Congenital Heart Disease


exceeding 30 mm Hg, in the presence of aortic valve insuffi-
ciency, or when symptoms indicating left ventricular outflow
tract (LVOT) obstruction are present. Given that repair of iso-
lated discrete subaortic stenosis can be done with low rates of
morbidity and mortality, some surgeons advocate repair in all
cases of discrete AS to avoid progression of the stenosis and the
development of aortic insufficiency, although more recent data
demonstrate that subaortic resection should be delayed until the
LV gradient exceeds 30 mm Hg because most children with an
initial LV gradient less than 30 mm Hg have quiescent disease.
Diffuse AS is a more complex lesion and often requires aorto-
ventriculoplasty. Results are generally excellent, with operative
mortality less than 5%. (See Schwartz 11th ed., pp. 755–758.)

15. Which structure must be identified prior to ligation and/ Answer: A


or division of a patent ductus arteriosus? Surgical closure can be achieved via either open or video-
A. Recurrent laryngeal nerve assisted approaches. The open approach employs a muscle-
B. Left superior pulmonary vein sparing posterior lateral thoracotomy in the third or fourth
C. Left bronchial artery intercostal space on the side of the aorta (generally the left).
D. Phrenic nerve The lung is then retracted anteriorly. In the neonate, the patent
ductus arteriosus (PDA) is singly ligated with a surgical clip or
permanent suture. Care must be taken to avoid the recurrent
laryngeal nerve, which courses around the PDA. The PDA can
also be ligated via a median sternotomy; however, this approach
is generally reserved for patients who have additional cardiac or
great vessel lesions requiring repair. Occasionally, a short, broad
ductus, in which the dimension of its width approaches that of
its length, will be encountered. In this case, division between
vascular clamps with oversewing of both ends is advisable. In
extreme cases, the use of cardiopulmonary bypass (CPB) to
decompress the large ductus during ligation is an option. (See
Schwartz 11th ed., pp. 759–760.)

16. Which of the following treatment paradigms most aptly Answer: D


describes the routine management of aortic coarctation Although operative repair is still the gold standard, treatment
in a 4-month-old child? of coarctation of the aorta (COA) by catheter-based inter-
A. Balloon dilation followed by stent placement for vention has become more widespread for older children and
recoarctation adults. Both balloon dilatation and primary stent implanta-
B. Endovascular stent deployment with serial balloon tion have been used successfully. The most extensive study of
dilations the results of balloon angioplasty reported on 970 procedures:
C. Observation 422 native and 548 recurrent COAs. Mean gradient reduc-
D. Surgical repair followed by catheter based interven- tion was 74% ± 24% for native and 70% ± 31% for recurrent
tions for recoarctation COA. This demonstrated that catheter-based therapy could

Brunicardi_Ch20_p157-170.indd 163 06/07/22 9:41 AM


164
produce equally effective results both in recurrent and in pri-
mary COA, a finding with far-reaching implications in the
new paradigm of multidisciplinary treatment algorithms
for congenital heart defect (CHD). In the valvuloplasty and
angioplasty of congenital anomalies (VACA) report, higher
preangioplasty gradient, earlier procedure date, older patient
age, and the presence of recurrent COA were independent
risk factors for suboptimal procedural outcome. In sum-
mary, children younger than age 6 months with native COA
CHAPTER 20

should be treated with surgical repair, while those requiring


intervention at later ages may be ideal candidates for bal-
loon dilatation or primary stent implantation. Additionally,
catheter-based therapy should be employed for those cases of
restenosis following either surgical or primary endovascular
management. (See Schwartz 11th ed., pp. 761–763.)
Congenital Heart Disease

17. Which type of aortic coarctation repair is most associ- Answer: D


ated with aneurysm formation at the repair site? The most common complications after coarctation of the
A. Resection with end-to-end anastomosis aorta (COA) repair are late restenosis (Fig. 20-4) and aneu-
B. Resection with extended end-to-end anastomosis rysm formation at the repair site. Aneurysm formation is par-
C. Endovascular stenting ticularly common after patch aortoplasty when using Dacron
D. Dacron patch aortoplasty material. In a large series of 891 patients, aneurysms occurred
in 5.4% of the total, with 89% occurring in the group who
received Dacron-patch aortoplasty and only 8% occurring
in those who received resection with primary end-to-end
anastomosis. A further complication, although uncommon,
is lower-body paralysis resulting from ischemic spinal cord
injury during the repair. This dreaded outcome complicates
0.5% of all surgical repairs, but its incidence can be lessened
with the use of some form of distal perfusion, preferably left
heart bypass with the use of femoral arterial or distal thoracic
aorta for arterial inflow and the femoral vein or left atrium
for venous return. These techniques are generally reserved
for older patients with complex coarctations that may need
prolonged aortic cross clamp times for repair, often in the
setting of large collateral vessels and/or previous surgery.
(See Schwartz 11th ed., pp. 762–763.)

FIG. 20-4. Reformatted images obtained from a CT angiogram


after recurrent coarctation repaired by an extra anatomic bypass
(`*’ points to the bypass graft).

Brunicardi_Ch20_p157-170.indd 164 06/07/22 9:41 AM


165
18. In a patient with truncus arteriosus, which of the follow- Answer: D
ing choices best describes definitive repair? Truncus arteriosus was first managed with pulmonary artery
A. Pulmonary artery banding banding as described by Armer and colleagues in 1961.
B. Division of the main pulmonary artery (PA), aortic However, this technique led to only marginal improvements
arch reconstruction with PA homograft, and creation in 1-year survival rates because ventricular failure inevitably
of an innominate artery to right pulmonary artery occurred. In 1967, however, complete repair was accomplished
shunt by McGoon and his associates based on the experimental
C. Mobilization of the coronary arteries, excision and work of Rastelli, who introduced the idea that an extracar-
transposition of the pulmonary valve into aor- diac valved conduit could be used to restore ventricular-to-

CHAPTER 20
tic position, reimplantation of the coronary arter- pulmonary artery continuity. Over the next 20 years, improved
ies, and right ventricular outflow tract (RVOT) survival rates led to uniform adoption of complete repair even
reconstruction in the youngest and smallest infants. Surgical correction entails
D. Mobilization of the pulmonary arteries, patch repair the use of cardiopulmonary bypass (CPB). Repair is com-
of the aorta and ventricular septal defect (VSD), and pleted by separation of the pulmonary arteries from the aorta,
extracardiac reconstruction of the RVOT closure of the aortic defect (occasionally with a patch) to
minimize coronary flow complications, placement of a valved

Congenital Heart Disease


cryopreserved allograft or jugular venous valved conduit
(Contegra) to reconstruct the RVOT, and VSD closure. Impor-
tant branch pulmonary arterial stenosis should be repaired at
the time of complete repair and can usually be accomplished
with longitudinal allograft patch arterioplasty. Severe truncal
valve insufficiency occasionally requires truncal valve repair
or even replacement, which can be accomplished with a cryo-
preserved allograft. (See Schwartz 11th ed., p. 764.)

19. Which type of anomalous venous connection (according Answer: C


to the Darling classification) is most commonly associ- Darling and colleagues classified total anomalous pulmonary
ated with pulmonary venous obstruction? venous connection (TAPVC) according to the site or level of
A. Supracardiac connection connection of the pulmonary veins to the systemic venous sys-
B. Cardiac connection tem: type I (45%), anomalous connection at the supracardiac
C. Infracardiac connection level; type II (25%), anomalous connection at the cardiac level;
D. Connection at multiple levels type III (25%), anomalous connection at the infracardiac level;
and type IV (5%), anomalous connection at multiple levels.
Within each category, further subdivisions can be imple-
mented, depending on whether pulmonary venous obstruction
exists. Obstruction to pulmonary venous drainage is a power-
ful predictor of adverse natural outcome and occurs most fre-
quently with the infracardiac type, especially when the pattern
of infracardiac connection prevents the ductus venosus from
bypassing the liver. (See Schwartz 11th ed., pp. 765–767.)

20. Which of the following factors confers the highest post- Answer: C
operative mortality after total anomalous pulmonary The most significant postoperative complication of TAPVC
venous connection (TAPVC) repair? repair is pulmonary venous obstruction (Fig. 20-5), which
A. Postoperative atrial arrythmias occurs 9% to 11% of the time, regardless of the surgical tech-
B. Patient age at operation nique employed. Mortality varies between 30% and 45%,
C. Pulmonary venous sclerosis and alternative catheter interventions do not offer definitive
D. Postoperative left ventricular (LV) dysfunction solutions. Recurrent pulmonary venous obstruction can be
localized at the site of the pulmonary venous anastomosis
(extrinsic), which usually can be cured with patch enlarge-
ment or balloon dilatation, or it may be secondary to endo-
cardial thickening of the pulmonary venous ostia frequently
resulting in diffuse pulmonary venous sclerosis (intrinsic),
which carries a 66% mortality rate because few good solutions
exist. More commonly, post repair left ventricular dysfunc-
tion can occur as the noncompliant LV suddenly is required
to handle an increased volume load from redirected pulmo-
nary venous return. This can manifest as an increase in pul-
monary artery pressure but is distinguishable from primary

Brunicardi_Ch20_p157-170.indd 165 06/07/22 9:41 AM


166
CHAPTER 20
Congenital Heart Disease

FIG. 20-5. Angiogram showing the discrete stenosis (`*’) of


the right-sided pulmonary veins after conventional repair for
supracardiac-type TAPVC.

pulmonary hypertension (another possible postoperative


complication following repair of TAPVC) from the elevated
left atrial pressure and LV dysfunction along with echocar-
diographic evidence of poor LV contractility. In pulmonary
hypertension, the left atrial pressure may be low, the LV may
appear “underfilled” (by echocardiography), and the RV may
appear dilated. In either case, postoperative support for a few
days with extracorporeal membrane oxygenation may be life-
saving, and TAPVC should be repaired in centers that have
this capacity. (See Schwartz 11th ed., pp. 765–767.)

21. An 8-month-old female infant is admitted for her third Answer: A


respiratory infection. She is noted by her parents to oth- The symptoms associated with vascular rings include respira-
erwise be exhibiting age-appropriate behavior, normal tory distress, barking cough, stridor, apnea, dysphagia, and
stooling, and normal feeding. A chest XRAY is performed recurrent respiratory tract infections. The diagnosis often
which demonstrates unilateral, left lung hyperinflation. requires a high index of suspicion. Minor respiratory tract
Ultimately the patient undergoes a computed tomogra- infections may precipitate serious respiratory distress. The
phy angiography (CTA) and echocardiography confirm- work-up includes chest X-rays, echocardiography, bron-
ing the presence of a pulmonary artery sling. Which of choscopy, CT scan (Fig. 20-6), magnetic resonance imaging
the following is the next best step after confirming the (MRI) (Fig. 20-7), and, rarely, cardiac catheterization. Chest
diagnosis? X-rays show the relationship of the aortic arch to the trachea.
A. Bronchoscopy Tracheal compression can be better evaluated using lateral
B. Cardiac catheterization films. Unilateral hyperinflation of the lung is sometimes
C. Intubation and mechanical ventilation seen and is often associated with a pulmonary artery sling
D. Barium swallow (Fig. 20-8). Pulmonary artery (PA) slings (Fig. 20-9) are often
associated with complete tracheal rings necessitating a bron-
choscopy when this diagnosis is made (Fig. 20-10). Patients
with dysphagia require a barium esophagogram as a part of
their work-up (Fig. 20-11). (See Schwartz 11th ed., p. 769.)

Brunicardi_Ch20_p157-170.indd 166 06/07/22 9:41 AM


167

CHAPTER 20
Congenital Heart Disease
FIG. 20-6. CT angiogram showing the four artery sign classic of FIG. 20-7. MRI showing a double aortic arch.
double aortic arch.

FIG. 20-8. Unilateral hyperinflation of the left lung associated with FIG. 20-9. CT angiogram showing a PA sling. Note the LPA
a rare vascular ring: left ascending aorta and right-sided descending wrapping around behind the trachea.
aorta.

FIG. 20-10. Rigid bronchoscopy showing complete


tracheal rings in a the patient with pulmonary artery sling.

Brunicardi_Ch20_p157-170.indd 167 06/07/22 9:41 AM


168
CHAPTER 20
Congenital Heart Disease

FIG. 20-11. Barium esophagogram showing posterior indentation


of the esophagus caused by a vascular ring (right aortic arch,
aberrant left subclavian artery, and left ligamentum).

22. In the setting of tricuspid atresia without a concurrent Answer: D


ventricular septal defect (VSD), which of the following The main pathophysiology in tricuspid atresia is that of a uni-
interventions is likely to stabilize an infant who presents ventricular heart of left ventricular morphology. That is, the
with worsening cyanosis? left ventricle (LV) must receive systemic blood via the inter-
A. Pulmonary artery banding atrial communication and then distribute it to both the pul-
B. Ligation of a patent ductus arteriosus monary circulation and the systemic circulation. Unless there
C. Closure of atrial septal defect is a ventricular septal defect (VSD) (as is found in some cases),
D. Prostaglandin E (PGE) administration pulmonary flow is dependent on the presence of a patent duc-
tus arteriosus (PDA). As the ductus begins to close shortly
after birth, infants become intensely cyanotic. Reestablish-
ing ductal patency (with PGE1) restores pulmonary blood
flow and stabilizes patients for surgical intervention. Pulmo-
nary hypertension is unusual in tricuspid atresia. However,
occasional patients have a large VSD between the LV and the
infundibular portion of the right ventricle (RV) (just below
the pulmonary valve). If there is no obstruction at the level
of this VSD or at the valve, these infants may actually pres-
ent with heart failure from excessive pulmonary blood flow.
Regardless of whether these infants are “ductal-dependent” for
pulmonary blood flow or have pulmonary blood flow pro-
vided across a VSD, they will be cyanotic since the obligatory
right-to-left shunt at the atrial level will provide complete
mixing of systemic and pulmonary venous return so that the
LV ejects a hypoxemic mixture into the aorta. (See Schwartz
11th ed., pp. 770–772.)

Brunicardi_Ch20_p157-170.indd 168 06/07/22 9:41 AM


169
23. Which of the following best describes the approach to Answer: A
surgical palliation of hypoplastic left heart syndrome? The main pathophysiology in tricuspid atresia is that of a uni-
A. Separation of the systemic and pulmonary circula- ventricular heart of left ventricular (LV) morphology. That is,
tions along with progressive off-loading of the right the LV must receive systemic blood via the interatrial commu-
ventricle (RV) nication and then distribute it to both the pulmonary circula-
B. Elimination of intracardiac shunting tion and the systemic circulation. Unless there is a ventricular
C. Relief of hypoxia through patent ductus arteriosus septal defect (VSD) (as is found in some cases), pulmonary
(PDA) closure flow is dependent on the presence of a PDA. As the ductus
D. Augmentation of left ventricular outflow obstruction begins to close shortly after birth, infants become intensely

CHAPTER 20
to increase coronary blood flow cyanotic. Reestablishing ductal patency (with PGE1) restores
pulmonary blood flow and stabilizes patients for surgical
intervention. Pulmonary hypertension is unusual in tricus-
pid atresia. However, occasional patients have a large VSD
between the LV and the infundibular portion of the RV (just
below the pulmonary valve). If there is no obstruction at the
level of this VSD or at the valve, these infants may actually pres-

Congenital Heart Disease


ent with heart failure from excessive pulmonary blood flow.
Regardless of whether these infants are “ductal-dependent”
for pulmonary blood flow or have pulmonary blood flow
provided across a VSD, they will be cyanotic since the obliga-
tory right-to-left shunt at the atrial level will provide complete
mixing of systemic and pulmonary venous return so that the
LV ejects a hypoxemic mixture into the aorta. (See Schwartz
11th ed., pp. 773–5.)

24. Wolff-Parkinson-White (WPW) is a conduction abnor- Answer: C


mality commonly associated with which form of con- A WPW syndrome (Fig. 20-12) type of accessory pathway
genital heart defect (CHD)? with associated preexcitation is present in 15% of patients.
A. Tricuspid Atresia The surgical approach in widespread use today for patients
B. Hypoplastic left heart syndrome surviving infancy was described by Danielson and col-
C. Ebstein Anomaly leagues in 1992. This procedure entails excision of redun-
D. Aortic stenosis dant right atrial tissue and patch closure of any associated
atrial septal defect (ASD), plication of the atrialized portion

FIG. 20-12. EKG of a newborn with Ebsteins anomaly and WPW syndrome. Note the pre-excitation (arrow).

Brunicardi_Ch20_p157-170.indd 169 06/07/22 9:41 AM


170
of the ventricle with obliteration of the aneurysmal cav-
ity, posterior tricuspid annuloplasty to narrow the tricuspid
annulus, reconstruction of the tricuspid valve if the ante-
rior leaflet is satisfactory, or replacement of the tricuspid
valve if necessary. If the tricuspid valve is not amenable to
reconstruction, valve replacement should be considered.
Care must be taken when performing the posterior annulo-
plasty, or during the conduct of tricuspid valve replacement,
to avoid the conduction system, because complete heart
CHAPTER 20

block can complicate this procedure. In addition, patients


who demonstrated preoperative evidence of preexcitation
should undergo electrophysiologic mapping and ablation.
(See Schwartz 11th ed., pp. 777–779.)

25. After accounting for size and location of the accompa- Answer: D
nying ventricular septal defect (VSD), which additional Patients with DORV typically present with one of the follow-
Congenital Heart Disease

factor is a critical determinant of the clinical presenta- ing three scenarios: (a) those with doubly committed or sub-
tion and management of double-outlet right ventricle aortic VSD present with congestive heart failure and a high
(DORV)? propensity for pulmonary hypertension, much like infants
A. Left ventricular hypertrophy with a large single VSD; (b) those with a subaortic VSD and
B. Posterior location of the aorta pulmonary stenosis present with cyanosis and hypoxia, much
C. Anterior location of the aorta like infants with tetralogy of Fallot; and (c) those with sub-
D. Presence of right ventricular outflow tract (RVOT) pulmonic VSD present with cyanosis, much like those with
obstruction D-TGA, because streaming directs desaturated systemic
venous blood to the aorta and oxygenated blood to the pul-
monary artery. Thus, the three critical factors influencing the
clinical presentation and subsequent management of infants
with DORV are the size and location of the VSD, the presence
or absence of important RVOT obstruction, and the presence
of other anomalies (especially associated hypoplasia of left-
sided structures sometimes seen with subpulmonic VSD).
(See Schwartz 11th ed., p. 783.)

26. Which anatomic consideration may complicate relief of Answer: D


severe right ventricular outflow tract (RVOT) obstruc- The morphology of TOF is markedly heterogeneous and
tion in a patient with tetralogy of Fallot (TOF)? includes an absent pulmonary valve, concomitant atrio-
A. Large ratio of pulmonary artery diameter to aorta ventricular (AV) septal defects, and pulmonary atresia with
B. Posterior location of the aorta major aortopulmonary collaterals. The present discussion
C. Small ventricular septal defect (VSD) will focus only on the so-called classic presentation of TOF
D. Left anterior descending artery originating from the without coexisting intracardiac defects.
right coronary Anomalous coronary artery patterns, related to either ori-
gin or distribution, have been described in TOF. However, the
most surgically important coronary anomaly occurs when the
left anterior descending artery arises as a branch of the right
coronary artery. This occurs in approximately 3% of cases of
TOF and may preclude placement of a transannular patch,
as the left anterior descending coronary artery crosses the
RVOT at varying distances from the pulmonary valve annulus.
(See Schwartz 11th ed., pp. 784–785.)

Brunicardi_Ch20_p157-170.indd 170 06/07/22 9:41 AM


CHAPTER 21
Acquired Heart Disease

1. New York Heart Association Classification (NYHA) is: Answer: A


A. A functional classification system that is strongly The NYHA functional class is a widely used classification
correlated with mortality. system in categorizing patients based on their functional
B. A functional classification system that does not cor- status (Table 21-1). The NYHA classification has become
relate with mortality. one basis by which to compare patient populations in many
C. Is an angina classification system similar to that of studies. Although less commonly used, the CCS angina clas-
the Canadian Cardiovascular Society (CCS) angina sification is also used to incorporate anginal symptoms into
classification. the functional assessment for prognostic value (Table 21-2).
D. Only calculated after cardiac catheterization. (See Schwartz 11th ed., p. 803.)

TABLE 21-1 New York Heart Association (NYHA)


functional classification
Class Description
I Physical activity not limited by symptoms: fatigue,
palpitations, or dyspnea.
II Comfortable at rest. Slight limitation of physical activity.
Fatigue, palpitations, or dyspnea with ordinary
physical activity.
III Comfortable at rest. Marked limitation of physical
activity. Fatigue, palpitations, or dyspnea with less
than ordinary physical activity.
IV Inability to carry out any physical activity. Symptoms
may be present at rest and increase with activity.

TABLE 21-2 Canadian Cardiovascular Society (CCS)


angina classification
Class Description
I Ordinary physical activity (walking, climbing stairs) does
not cause angina. Angina occurs with strenuous, rapid,
or prolonged exertion during work or recreation.
II Slight limitation of ordinary activity. Angina occurs with
climbing stairs rapidly, walking uphill in the wind,
under emotional stress, in the cold, or after meals.
Walking more than 2 blocks or climbing one flight of
stairs causes angina.
III Marked limitation of ordinary physical activity
(climbing a flight of stairs or walking 1 to 2 blocks
at a normal pace).
IV Inability to carry out any physical activity without
discomfort. Angina may be present at rest.

171

Brunicardi_Ch21_p171-180.indd 171 04/07/22 2:10 PM


172
2. Patients undergoing noncardiac surgery should be Answer: B
assessed for their risk of cardiovascular complication. Patient characteristics can be classified by the status of the
A. Perioperative risk of major adverse cardiac events patient’s cardiac disease, comorbid conditions, and functional
(MACE) is a function of preexisting cardiac disease capacity. Patients are considered to be at major perioperative
alone. clinical risk if they have one or more of the following active
B. American College of Surgeons NSQUIP Risk Calcu- cardiac conditions: acute coronary syndrome, decompen-
lator or the Revised Cardiac Risk Index can estimate sated heart failure, significant arrhythmias, or severe valvu-
patient specific risk. lar heart disease. In these patients, intensive evaluation and
C. Risk assessment requires stress testing and treatment prior to surgery (unless emergent) are warranted,
CHAPTER 21

echocardiography. prior to proceeding with the noncardiac surgery.


D. Patients with recent percutaneous coronary interven- If the patient does not have any of the previously men-
tion are at lower risk. tioned active cardiac conditions, the perioperative risk of
MACE should be estimated. Both the operation performed
and the patient’s risk factors are predictive of MACE, and
the ACC/AHA guidelines recommend the use of either the
American College of Surgeons’ NSQIP risk calculator or the
Acquired Heart Disease

Revised Cardiac Risk Index for the estimation of patient-


specific risk. Patients at low (<1%) risk or patients at elevated
risk with functional capacity ≥4 metabolic equivalents (METs)
should proceed to surgery without further testing. It is rea-
sonable to perform pharmacologic stress testing in patients
with poor or unknown functional capacity if this testing will
impact decision-making or perioperative care. Patients with
abnormal stress test results should undergo confirmatory test
such as coronary angiography, if indicated, before an elective
noncardiac surgery. The previous guidelines included inter-
mediate and low cardiovascular risk profiles, but this has
been replaced by cardiovascular risk factors in the update.
These risk factors are: history of ischemic heart disease, his-
tory of prior or compensated heart failure, history of cere-
brovascular disease, diabetes mellitus, and renal insufficiency.
Based on the number of present risk factors and the surgery-
specific risk, the guidelines recommend pathways for further
evaluation and risk management. The most recent guide-
lines from ACC/AHA were published in 2014. One impor-
tant subgroup of patients at elevated risk are those who have
recently undergone percutaneous coronary intervention. In
these patients, elective noncardiac surgery should be delayed
until the risk of stent thrombosis decreases (30 days for bare
metal stents and 180 to 365 days for drug-eluting stents),
and dual antiplatelet therapy should be continued unless
the risk of bleeding exceeds the risk of stent thrombosis.
(See Schwartz 11th ed., p. 804.)

3. In comparing transthoracic echocardiography (TTE) Answer: B


and transesophageal echocardiography (TEE): TTE requires no sedation and is generally performed with
A. Both TEE and TTE require sedation. the patient in a slight left lateral decubitus position. Standard-
B. TTE provides excellent view of mitral valve and ized views are obtained with the ultrasound probe placed in
posterior cardiac structures. the apical, parasternal, subcostal, and suprasternal positions.
C. TEE is often performed intraoperatively during car- The apical four-chamber view is a useful window for visual-
diac surgery to assess repairs and cardiac function. izing all four cardiac chambers simultaneously as well as the
D. TTE cannot see all four chambers of the heart, so tricuspid and mitral valves. Other windows can be obtained
TEE is preferred. to assess specific structures such as the individual valve anat-
omy or myocardial wall segments. Dobutamine-stress echo-
cardiography is a study similar in idea to the stress ECG that
utilizes a pharmacologic agent to assess the patient for isch-
emia or stress-induced valvular abnormalities.

Brunicardi_Ch21_p171-180.indd 172 04/07/22 2:10 PM


173
TEE, on the other hand, is performed using a special endo-
scope with an ultrasound probe mounted on its end that is
introduced orally into the esophagus under sedation. Pos-
terior structures such as the mitral valve and left atrium are
particularly well visualized. TEEs are frequently used intra-
operatively during cardiothoracic surgery to assess global
cardiac function, integrity of valve repairs and replacements,
intracavitary thrombus and/or air, and aortic atherosclerosis
or dissections that can have significant influences on opera-

CHAPTER 21
tive strategy. (See Schwartz 11th ed., p. 804.)

4. Which of the following is FALSE regarding the deleteri- Answer: B


ous effects of cardiopulmonary bypass (CPB)? The responses of the humoral and cellular immune systems
A. Classic and alternative complement pathways are partly overlap with the hemostatic pathways. The classic and
activated and generate powerful chemotactic mol- alternative complement pathways are activated by CPB gen-
ecules and anaphylatoxins. erating powerful chemotaxic molecules and anaphylatoxins.

Acquired Heart Disease


B. Deleterious effects can be ameliorated with periop- Monocytes, platelets, and neutrophils are activated releasing
erative heparin and steroid administration. acute inflammatory mediators and cytokines that persist even
C. Heparin-induced thrombocytopenia (HIT) occurs in after conclusion of CPB. These inflammatory cells also pro-
1% to 5% of patients. duce reactive oxidants that may have cytotoxic and cardiovas-
D. End-organ dysfunction occurs because of hypoper- cular effects, such as vasodilation and hypotension.
fusion, embolization, and whole body inflammatory The large quantity of unfractionated heparin used during
response. cardiac surgery predisposes patients to developing HIT with
an incidence of 1% to 5%. Platelet factor-4 (PF4) is produced
by platelets and avidly binds to heparin to form a heparin-PF4
complex that can be antigenic in some patients binding IgG.
The IgG-heparin-PF4 complex can bind to platelets, which
causes release of more PF4, perpetuating the process.

5. The bypass conduit with the highest patency rate is the: Answer: B
A. Radial artery. Bypass Conduit Selection. The most important criterion in
B. Internal thoracic artery. conduit selection is graft patency. The conduit with the high-
C. Greater saphenous vein. est patency rate (98% at 5 years and 85%–90% at 10 years)
D. Radial artery. is the internal thoracic artery which is most commonly left
attached proximally to the subclavian artery (although occa-
sionally used as a free graft) and anastomosed distally to
the target coronary artery. The use of both internal thoracic
arteries has been shown to increase event-free survival in a
number of studies. (See Schwartz 11th ed., p. 811.)

6. An absolute contraindication to a coronary artery bypass Answer: D


operation is: Patients with ischemic cardiomyopathy are a heterogeneous
A. Acute coronary artery insufficiency with persistent or group, and, as with any surgery, appropriate patient selection
progressive angina despite optimal medical therapy. is central to success. In one retrospective study of 96 patients
B. Acute subendocardial infarction with multivessel with ischemic cardiomyopathy (ejection fraction [EF] ≤ 25%),
coronary artery disease. age, and poor distal vessel quality were predictors of poor
C. Cardiogenic shock after myocardial infarction. outcomes. Mortality in patients with poor vessel quality was
D. Chronic congestive failure and ischemic cardiomy- 100%, compared with 90% when vessel quality was fair and
opathy with no signs of angina. 10% when it was good. Therefore, poor vessel quality should
be considered a contraindication to surgical revasculariza-
tion even in the presence of angina. (See Schwartz 11th ed.,
p. 810.)

Brunicardi_Ch21_p171-180.indd 173 04/07/22 2:10 PM


174
7. Which of the following is true regarding the choice of Answer: C
using mechanical or tissue valves in valve replacement? Although mechanical valves are highly durable, they require
A. Long-term systemic anticoagulation is required for permanent anticoagulation to mitigate the risk of valve
patients receiving both mechanical and tissue valves. thrombosis and thromboembolic sequelae. Due to the con-
B. Mechanical valves are recommended for all young cordant risk of hemorrhagic complications, patient charac-
men and women because of durability. teristics such as debility, lifestyle, and contraindications to
C. ACC/AHA guidelines recommend a shared process systemic anticoagulation therapy may preclude mechanical
between patient and physician to determine the best valve replacement. Moreover, young women who are plan-
choice of valve prosthesis. ning future pregnancies cannot take warfarin due to its tera-
CHAPTER 21

D. Bioprosthetic valves have similar rates of thrombosis togenic potential. Conversely, patients with other indications
compared to mechanical valves. for systemic anticoagulation, such as other risk factors for
thromboembolism (ie, atrial fibrillation), or the presence of
a mechanical prosthetic valve in place in another position,
may benefit from mechanical valve replacement. Current
ACC/AHA guidelines recommend a shared-decision-making
process between patient and physician when determining the
Acquired Heart Disease

choice of valve prosthesis, with the use of bioprosthetic valves


in all patients who have a contraindication to lifelong anti-
coagulation or who are unwilling to receive it. (See Schwartz
11th ed., p. 814.)

8. Which of the following is FALSE regarding mitral steno- Answer: D


sis (MS)? Acquired MS is most often caused by rheumatic fever, with
A. Acquired MS is most commonly caused by rheumatic approximately 60% of patients with pure MS presenting with
heart disease (60%). a clinical history of rheumatic heart disease. Rarely, other
B. MS is associated with increased valve flow velocity conditions can cause obstruction to filling of the left ventricle
and increased size of the left atrium, and elevated (LV), mimicking MS. Acquired causes of mitral valve (MV)
pulmonary artery pressures. stenosis include left atrial myxoma, prosthetic valve throm-
C. MS is associated with atrial fibrillation (AF) with left bosis, mucopolysaccharidosis, previous chest radiation, and
atrial thrombus and embolism. severe annular calcification.
D. Pulmonary edema occurs due to left ventricular The increased left atrial pressure is subsequently trans-
failure. mitted to the pulmonary venous system, causing pulmo-
nary edema as the hydrostatic pressure in the vessels exceeds
the plasma oncotic pressure. Decreased pulmonary venous
compliance exacerbates the pulmonary venous hyperten-
sion, though a concomitant decrease in microvascular per-
meability may preclude pulmonary edema in the chronic
setting. Patients may also develop pulmonary arterial hyper-
tension, owing to vasoconstriction, intimal hyperplasia, and
medial hypertrophy of the pulmonary arterioles in response
to the increased pulmonary venous pressure. The second-
ary obstruction to flow caused by reactive pulmonary arte-
rial hypertension may serve to protect against pulmonary
edema, but it also exacerbates the intractable decrease in car-
diac output that develops as stenosis worsens. Throughout
the process, the left atrium becomes dilated and hypertro-
phied due to increased work in filling the ventricle against a
fixed obstruction. AF may develop, exacerbating the patient’s
symptoms and increasing the risk of left atrial thrombus and
subsequent embolization. Left ventricular structure and func-
tion are typically preserved owing to the protective effect of
the stenotic valve. (See Schwartz 11th ed., p. 818.)

Brunicardi_Ch21_p171-180.indd 174 04/07/22 2:10 PM


175
9. The most common cause of acquired mitral stenosis Answer: A
(MS) is: Acquired MS is most often caused by rheumatic fever, with
A. Rheumatic disease. approximately 60% of patients with pure MS presenting with
B. Left atrial myxoma. a positive clinical history of rheumatic heart disease. Rarely,
C. Ball valve thrombus. other conditions can cause obstruction to filling of the left ven-
D. Previous chest radiation. tricle (LV), mimicking MS. Acquired causes of MV obstruction
include left atrial myxoma, ball valve thrombus, mucopolysac-
charidosis, previous chest radiation, and severe annular calcifi-
cation. (See Schwartz 11th ed., p. 818.)

CHAPTER 21
10. Patients undergoing mechanical mitral valve Answer: B
replacement: Although mechanical valves necessitate systemic anticoagula-
A. Have a target INR of 4 to 5 times normal. tion, careful monitoring of the international normalized ratio
B. Have increased left atrial size. (INR) reduces the risk of thromboembolic events andhemor-
C. Have atrial fibrillation. rhagic complications, and improves overall survival. Patients
D. Are at lower risk for thromboembolism. undergoing mechanical aortic valve replacement generally

Acquired Heart Disease


have a target INR of 2 to 3 times normal. Patients under-
going mechanical mitral valve replacement frequently have
increased left atrial size, concomitant atrial fibrillation, and
are at higher risk for thromboembolism than those undergo-
ing mechanical aortic valve replacement, and are thus recom-
mended to have a target INR 2.5 to 3.5 times normal. When
managed appropriately, the yearly thromboembolic and
bleeding risks in these patients are 1% to 2%, and 0.5% to 2%,
respectively. (See Schwartz 11th ed., p. 816.)

11. What valvular lesion is most commonly found in a Answer: D


patient with Marfan syndrome? The most common cause of isolated aortic insufficiency (AI)
A. Mitral stenosis in patients undergoing aortic valve replacement (AVR) is aor-
B. Mitral insufficiency tic root disease and represents over 50% of such patients in
C. Aortic stenosis some studies. Other common causes of AI include congenital
D. Aortic insufficiency abnormalities of the aortic valve such as bicuspid aortic valve,
calcific degeneration, rheumatic disease, infective endocar-
ditis, systemic hypertension, myxomatous degeneration, dis-
section of the ascending aorta, and Marfan syndrome. Less
common causes of AI include traumatic injuries to the aortic
valve, ankylosing spondylitis, syphilitic aortitis, rheumatoid
arthritis, osteogenesis imperfecta, giant cell aortitis, Ehlers-
Danlos syndrome, Reiter syndrome, discrete subaortic steno-
sis, and ventricular septal defects with prolapse of an aortic
cusp. Although most of these lesions produce chronic AI,
rarely acute severe aortic regurgitation can result, often with
devastating consequences. (See Schwartz 11th ed., p. 825.)

12. Age-related calcific aortic stenosis (AS) causes some Answer: C


degree of aortic insufficiency (AI) in approximately: There are also many primary valvular diseases that cause AI,
A. 55%. generally in association with AS. One such disorder is age-
B. 65%. related calcific AS, which causes some degree of AI in up to
C. 75%. 75% of patients. Infective endocarditis may involve the aortic
D. 85%. valve apparatus and cause AI through direct destruction of the
valve leaflets, perforation of a leaflet, or formation of vegeta-
tions that interfere with proper coaptation of the valve cusps.
Rheumatic disease causes fibrous infiltration of the valve
cusps and subsequent retraction of the valve leaflets, inhibit-
ing apposition of the cusps during diastole and producing a
central regurgitant jet. Patients with large ventricular septal
defects or membranous subaortic stenosis may develop pro-
gressive AI, owing to a Venturi effect that results in prolapse
of the aortic valve leaflets. (See Schwartz 11th ed., p. 823.)

Brunicardi_Ch21_p171-180.indd 175 04/07/22 2:10 PM


176
13. Tricuspid stenosis is: Answer: B
A. Caused by secondary dilation of the tricuspid annu- Acquired tricuspid valve (TV) disease can be classified as
lus due to pulmonary hypertension and/or right either organic or functional, and affects approximately 0.8%
heart failure. of the general population. Tricuspid stenosis is almost always
B. Commonly the result of organic disease, such as a result of organic disease, namely rheumatic heart disease
rheumatic heart disease and endocarditis. and endocarditis. In the case of rheumatic disease, tricuspid
C. Commonly caused by mitral valve disease. stenosis with or without associated insufficiency is invari-
D. Commonly associated with Marfan syndrome. ably associated with mitral valve disease. Other less common
causes of obstruction to right atrial emptying include congen-
CHAPTER 21

ital tricuspid atresia, right atrial tumors, and endomyocardial


fibrosis. (See Schwartz 11th ed., p. 829.)

14. A holosystolic murmur that is accompanied by a ventric- Answer: C


ular septal defect is associated with which of the follow- See Schwartz 11th ed., p. 813, Table 21-7.
ing etiologies?
A. Ventricular filling that follows atrial contraction
Acquired Heart Disease

B. Crescendo-decrescendo; occur as blood is ejected


into the left and right ventricular outflow tracts
C. Flow between chambers that have widely different
pressures throughout systole
D. A relative disproportion between valve orifice size
and diastolic blood flow volume

15. Which of the following statements is TRUE of left ven- Answer: B


tricular (LV) aneurysms? A transmural infarction of approximately 5% to 10% of the
A. Rupture is uncommon. myocardium may result in formation of an LV aneurysm
B. It usually occurs 4–8 weeks after myocardial infarct. as necrotic myocardium is replaced by fibrous tissue. This
C. 1 year mortality is <5%. usually occurs 4 to 8 weeks following the infarct. In the last
D. Generally, it requires >40% necrosis of the decade, prompt revascularization of the culprit artery by
myocardium. either surgical or interventional techniques generally results
in sparing of the subepicardial muscle while the subendo-
cardial muscle remains necrotic. Therefore, it is not uncom-
mon for the LV wall to show both living myocardium during
thallium testing and an akinetic zone on echocardiogram or
angiogram. It has been demonstrated that once >20% of the
myocardium is necrosed, there is irreversible progression to
ventricular dilation and failure. Once heart failure develops
after postinfarction remodeling, the 1-year mortality reaches
32% despite current therapies. The classic aneurysm is a 4 to
6 mm thick scar, which bulges outward in paradoxical motion
as the LV contracts during systole. More than 80% develop in
the anteroseptal and apical portions of the left ventricle as a
result of left anterior descending artery occlusion. The rest
are inferior in location and the result of circumflex or right
coronary occlusion. (See Schwartz 11th ed., p. 833.)

16. The most common arrhythmia worldwide is: Answer: D


A. Atrial flutter. Atrial fibrillation (AF) remains the most common arrhyth-
B. Paroxysmal supraventricular tachycardia (PSVT). mia in the world with an overall incidence of 0.4% to 1%
C. Wolff-Parkinson-White (WPW) syndrome. that increases to 8% in those older than 80 years. The most
D. Atrial fibrillation. serious complication of AF is thromboembolism with resul-
tant stroke, but serious morbidity and mortality may also
result from hemodynamic compromise due to loss of atrial
contraction, exacerbations of congestive heart failure from
atrioventricular asynchrony and tachycardia-induced cardio-
myopathy. (See Schwartz 11th ed., p. 838.)

Brunicardi_Ch21_p171-180.indd 176 04/07/22 2:10 PM


177
17. Which of the following is FALSE regarding the Cox- Answer: A
Maze IV procedure? The Cox-Maze IV uses a combination of bipolar radiofre-
A. It is indicated for ventricular tachycardia. quency (RF) ablation and cryoablation to effectively replace
B. It is indicated for treatment of Atrial Fibrillation the majority of incisions that comprise the Cox-Maze III
(AF). while significantly shortening cross-clamp time and reducing
C. It includes surgical management of left atrial append- operative complexity.
age and radiofrequency or cryotherapy ablation. The Cox-Maze IV is performed on cardiopulmonary
D. It is indicated in patients who are poor candidates for bypass through either a median sternotomy, often in combi-
catheter-based ablation, including patients with large nation with other cardiac surgery or a right minithoracotomy.

CHAPTER 21
left atria and long-standing persistent AF. In most cases, the right atrial lesion set performed on the
beating heart, whereas the left atrial lesions are performed
during cardioplegic arrest (Fig. 21-1). (See Schwartz 11th ed.,
p. 838.)

A B

Acquired Heart Disease


FIG. 21-1. The Cox-Maze IV Lesion Set. A. The left atrial lesion set is comprised of right and left
pulmonary vein isolation, connecting lesions between the left and right superior and inferior
pulmonary veins, a lesion from the left atrial appendage excision site to the pulmonary vein, and a
lesion to the mitral valve annulus. B. The right atrial lesion set consists of lines of ablation along the
superior and inferior vena cavae, the free wall of the right atrium, and down to the tricuspid valve
annulus. (Reproduced with permission from Damiano RJ Jr, Schwartz FH, Bailey MS, et al: The Cox maze IV
procedure: predictors of late recurrence. J Thorac Cardiovasc Surg. 2011;141(1):113–21.)

18. Which of the following is not TRUE in comparing Cox- Answer: A


Maze IV and pulmonary vein isolation (PVI)? PVI is an attractive therapeutic option because it can be per-
A. Both procedures require cardiopulmonary bypass. formed off of cardiopulmonary bypass (CPB) through small
B. PVI has poorer results than Cox-Maze IV procedure. or thoracoscopic incisions. The results of PVI have been vari-
C. PVI has been shown to be superior to catheter-based able and highly dependent on patient selection since outcomes
ablation procedures. are consistently worse in patients with long-standing persis-
D. PVI is performed thoracoscopically. tent atrial fibrillation (AF). In a study from Edgerton et al,
only 56% of patients were free from AF at 6 months (35%
off antiarrhythmic drugs), and with concomitant procedures,
the success rate of PVI has been even lower. Several devices
are available to close the left atrial appendage (LAA) at the
time of PVI. These include staplers and epicardial clips that
can be placed without the need for CPB.
While surgical PVI has had poorer results than a Cox-Maze
procedure, it has had superior results to catheter-based PVI.
The Atrial Fibrillation Catheter Ablation Versus Surgical
Ablation Treatment (FAST) Trial, which was a two-center,
randomized clinical trial, compared catheter-based ablation
to thoracoscopic PVI in patients with antiarrhythmic drug-
refractory AF and either left atrial dilatation and hyperten-
sion or failed prior catheter-ablation. (See Schwartz 11th ed.,
p. 839.)

Brunicardi_Ch21_p171-180.indd 177 04/07/22 2:10 PM


178
19. A patient presents with a history of fatigue and dyspnea. Answer: C
He is found to have hepatomegaly, ascites, and an ele- Classic physical exam findings include jugular venous dis-
vated jugular venous pulse. Heart sounds are normal, no tention with Kussmaul’s sign, diminished cardiac apical
murmurs are present, and the heart is of normal size. The impulses, peripheral edema, ascites, pulsatile liver, a pericar-
pulse pressure is decreased by palpation. Electrocardiog- dial knock, and, in advanced disease, signs of liver dysfunc-
raphy is normal except for low voltage. The most likely tion, such as jaundice or cachexia. The “pericardial knock” is
diagnosis is: an early diastolic sound that reflects a sudden impediment
A. Right atrial myxoma. to ventricular filling, similar to an S3 but of higher pitch.
B. Tricuspid valve disease. Several findings are characteristic on noninvasive and inva-
CHAPTER 21

C. Constrictive pericarditis. sive testing. Central venous pressure (CVP) is often elevated
D. Primary pulmonary artery hypertension. 15 to 20 mm Hg or higher. Electrocardiogram (ECG) com-
monly demonstrates nonspecific low voltage QRS complexes
and isolated repolarization abnormalities. Chest X-ray may
demonstrate calcification of the pericardium, which is highly
suggestive of constrictive pericarditis in patients with heart
failure, but this is present in only 25% of cases. Cardiac CT
Acquired Heart Disease

or MRI (cMRI) typically demonstrate increased pericardial


thickness (>4 mm) and calcification, dilation of the infe-
rior vena cava, deformed ventricular contours, and flatten-
ing or leftward shift of the ventricular septum. Pericardial
adhesions may also be seen on tagged cine MRI studies. As
discussed, it is most important to distinguish pericardial con-
striction from restrictive cardiomyopathy, which is best done
with either echocardiography or right heart catheterization.
Findings favoring constriction on echocardiography include
respiratory variation of ventricular septal motion and mitral
inflow velocity, preserved or increased mitral annulus early
diastolic filling velocity, and increased hepatic vein flow
reversal with expiration. Cardiac catheterization will show
increased atrial pressures, equalization of end-diastolic pres-
sure, and early ventricular diastolic filling with a subsequent
plateau, called the “square-root sign.” Additional findings
upon catheterization that would favor constriction include
respiratory variation in ventricular filling and increased ven-
tricular interdependence, manifest as a discordant change in
the total area of the left ventricular (LV) and right ventricular
(RV) systolic pressure curve with respiration. (See Schwartz
11th ed., p. 840.)

20. Pericarditis is usually treated with: Answer: A


A. A short course of nonsteroidal anti-inflammatory The preferred treatment depends on the underlying cause of
agents. the pericarditis. The disease usually follows a self-limited and
B. Use of steroids or IV antibiotics. benign course and can be successfully treated with a short
C. Surgical exploration and drainage. course of nonsteroidal anti-inflammatory agents (NSAIDs).
D. Observation. Some patients may require judicious use of steroids or intra-
venous (IV) antibiotics. In cases of purulent pyogenic peri-
carditis, surgical exploration and drainage are occasionally
necessary. Rarely, accumulation of fluid in the pericardium
may lead to tamponade, requiring prompt evacuation of the
pericardial space. While pericardiocentesis will typically suf-
fice, surgical drainage may be required for thick, viscous,
or clotted fluid or in patients with significant scarring from
previous surgeries. More commonly, surgical intervention is
required to manage recurrent disease. (See Schwartz 11th ed.,
p. 840.)

Brunicardi_Ch21_p171-180.indd 178 04/07/22 2:10 PM


179
21. Each of the following effects is anticipated after insertion Answer: B
of an intra-aortic balloonpump (IABP) EXCEPT: IABP is the most commonly used device for mechanical cir-
A. Preload decrease. culatory support, and it may be easily deployed in the cathe-
B. Increased total myocardial oxygen consumption. terization laboratory, in the operating room or at the bedside.
C. Improvement in cardiac index. The device is inserted percutaneously through the femoral
D. Coronary blood flow increase. artery into the thoracic aorta. It is synchronized so that the
balloon is inflated during diastole and deflated during systole,
resulting in augmentation of diastolic perfusion of the coro-
nary arteries and decreased afterload. Typically, this improves

CHAPTER 21
cardiac index and decreases both preload and myocardial
oxygen consumption. (See Schwartz 11th ed., p. 835.)

22. Destination left ventricular assist device (LVAD) and Answer: A


total artificial heart is indicated. Only in patients with Patients in need of ventricular assist devices (VADs) may
contraindications to heart transplantation they include: have preexisting chronic heart failure, refractory ventricular
A. Irreversible renal failure. arrhythmias, or acute heart failure following an MI, cardio-

Acquired Heart Disease


B. New York Heart Association Class III or Class IV. pulmonary arrest, viral illness, pregnancy, or cardiotomy.
C. Ejection fraction < 25%. Device therapy is intended to preserve end-organ perfusion
D. High oxygen requirements. and function and may be categorized as short- or long-term
support for the left heart, the right heart, or both. In general,
VADs may be used rarely for support while the heart recovers
(bridge to recovery, BTR), while the patient waits for a heart
transplant (bridge to transplant, BTT) or increasingly more
commonly to treat a chronic heart failure patient who is not a
transplant candidate (destination therapy, DT).
Current eligibility criteria for mechanical support as des-
tination therapy include (a) New York Heart Association
Classification (NYHA) class III or IV heart failure despite
guideline-directed medical therapy including cardiac resyn-
chronization therapy if indicated; (b) peak oxygen consump-
tion <12 mL/kg per min or failure to wean from continuous
IV inotropes; (c) left ventricular ejection fraction < 25%; and
(d) presence of a contraindication for heart transplantation
(ie, age > 65 years, irreversible pulmonary hypertension,
chronic renal failure, insulin-dependent diabetes with end-
organ damage, or other clinically significant comorbidities).
Once a patient has an LVAD inserted as DT, close and intensive
follow-up by a multidisciplinary heart failure team is required
in order to optimize medical therapy, reduce device-related
morbidity, and improve survival. (See Schwartz 11th ed.,
p. 837.)

23. The most common cardiac tumor is: Answer: C


A. Papillary fibroelastoma. Cardiac myxomas are the most common cardiac tumor and
B. Lymphangioma. are characterized by several distinguishing features. About
C. Myxoma. 75% of the time, they arise from the interatrial septum near
D. Metastatic tumor. the fossa ovalis in the left atrium. Most others will develop
in the right atrium, but, less commonly, they can arise from
valvular surfaces and the walls of other cardiac chambers.
Macroscopically, these tumors are pedunculated with a gelat-
inous consistency, and the surface may be smooth (65%),
villous, or friable. Size varies greatly with these tumors and
ranges from 1 to 15 cm in diameter.
Internally, myxomas are heterogeneous and often contain
hemorrhage, cysts, necrosis, or calcification. Histologically,
these tumors contain cells that arise from a multipotent mes-
enchyme and are contained within a mucopolysaccharide
stroma. (See Schwartz 11th ed., p. 842.)

Brunicardi_Ch21_p171-180.indd 179 04/07/22 2:10 PM


180
24. Which of the following is NOT true of rhabdomyomas? Answer: A
A. They often require resection. In children, rhabdomyomas are the most common primary
B. They are often multicentric in the ventricles. cardiac tumor, whereas fibromas are the most commonly
C. They are the most common primary cardiac tumor in resected cardiac tumor. Rhabdomyomas are myocardial ham-
children. artomas that are often multicentric in the ventricles. About
D. They often disappear spontaneously. 50% of cases are associated with tuberous sclerosis, and while
resection is occasionally necessary, most disappear spontane-
ously. Fibromas are congenital lesions that one-third of the
time are found in children younger than 1-year old. These
CHAPTER 21

tumors, conversely, are ordinarily solitary lesions found in


the inner interventricular septum, and they may present with
heart failure, cyanosis, arrhythmias, syncopal episodes, chest
pain, or sudden cardiac death. (See Schwartz 11th ed., p. 843.)
Acquired Heart Disease

Brunicardi_Ch21_p171-180.indd 180 04/07/22 2:10 PM


CHAPTER 22
Thoracic Aneurysms and Aortic Dissection

1. Which of the following describes aortic aneurysms that Answer: C


are true aneurysms? Aortic aneurysms can be either “true” or “false.” True aneu-
A. Fusiform rysms can take two forms: fusiform and saccular. Fusiform
B. Saccular aneurysms are more common and can be described as sym-
C. Pseudoaneurysm metrical dilatations of the aorta. Saccular aneurysms are
D. Mega-aorta localized outpouchings of the aorta. False aneurysms, also
called pseudoaneurysms, are leaks in the aortic wall that are
contained by the outer layer of the aorta and/or the peri-
aortic tissue; they are caused by disruption of the aortic
wall and lead blood to collect in pouches of fibrotic tissue.
(See Schwartz 11th ed., p. 853.)

2. Which of the following is the most common cause of Answer: D


thoracic aortic aneurysms? Nonspecific medial degeneration is the most common cause
A. Atherosclerosis of thoracic aortic disease. Histologic findings of mild medial
B. Marfan syndrome degeneration, including fragmentation of elastic fibers and
C. Takayasu’s arteritis loss of smooth muscle cells, are expected in the aging aorta.
D. Nonspecific medial degeneration However, an advanced, accelerated form of medial degenera-
tion leads to progressive weakening of the aortic wall, aneu-
rysm formation, and eventual dissection, rupture, or both.
The underlying causes of medial degenerative disease remain
unknown. (See Schwarz 11th ed., p. 855.)

3. Marfan syndrome is an autosomal dominant genetic dis- Answer :B


order characterized by connective tissue defect that leads Marfan syndrome is an autosomal dominant genetic disor-
to aneurysm formation. Which of the following state- der characterized by a specific connective tissue defect that
ments is FALSE? leads to aneurysm formation. The phenotype of patients
A. Marfan syndrome is also associated with joint hyper- with Marfan syndrome typically includes a tall stature, high
mobility and eye lens disorders. palate, joint hypermobility, eye lens disorders, mitral valve
B. The disease causes fragmentation of elastic fibers, prolapse, and aortic aneurysms. Abnormal fibrillin causes
causing the aorta wall to be less elastic. degeneration of the aortic wall matrix by increasing the activ-
C. Abnormal fibrillin and degeneration of aortic wall ity of transforming growth factor beta (TGF-β). Between 75%
matric causes abnormal elasticity. and 85% of patients with Marfan syndrome have dilatation
D. Most Marfan syndrome patients have dilation of of the ascending aorta and annuloaortic ectasia (dilatation of
ascending aorta and dilation of the aortic annulus. the aortic sinuses and annulus). Marfan syndrome also is fre-
quently associated with aortic dissection, and aortic compli-
cations are the most common cause of death among patients
with Marfan syndrome. (See Schwartz 11th ed., p. 855.)

181

Brunicardi_Ch22_p181-188.indd 181 06/07/22 9:46 AM


182
4. Vascular-type Ehlers-Danlos syndrome is characterized Answer: C
by a defect in: Ehlers-Danlos syndrome includes a spectrum of inherited
A. Elastin. disorders of collagen synthesis. The subtypes represent dif-
B. Metalloproteinase. fering defective steps of collagen production. Vascular-type
C. Type III collagen synthesis. Ehlers-Danlos syndrome is characterized by an autosomal
D. Fibrillin. dominant defect in type III collagen synthesis, which can have
life-threatening cardiovascular manifestations. Spontaneous
arterial rupture, usually involving the mesenteric vessels, is
the most common cause of death in these patients. Thoracic
CHAPTER 22

aortic aneurysms and dissections are less commonly associ-


ated with Ehlers-Danlos syndrome, but when they do occur,
they pose a particularly challenging surgical problem because
of the reduced integrity of the aortic tissue. (See Schwartz
11th ed., p. 855.)

5. The most common cause of death in patients with type Answer: C


Thoracic Aneurysms and Aortic Dissection

IV Ehlers-Danlos syndrome is: Ehlers-Danlos syndrome includes a spectrum of inherited


A. Myocardial infarction. connective tissue disorders of collagen synthesis. The sub-
B. Aortic dissection. types represent differing defective steps of collagen produc-
C. Ruptured visceral artery. tion. Vascular-type Ehlers-Danlos syndrome is characterized
D. Pulmonary emboli. by an autosomal dominant defect in type III collagen syn-
thesis, which can have life-threatening cardiovascular mani-
festations. Spontaneous arterial rupture, usually involving
the mesenteric vessels, is the most common cause of death
in these patients. Thoracic aortic aneurysms and dissections
are less commonly associated with Ehlers-Danlos syndrome,
but when they do occur, they pose a particularly challenging
surgical problem because of the reduced integrity of the aortic
tissue. An Ehlers-Danlos variant of periventricular heteroto-
pia associated with joint and skin hyperextensibility and aor-
tic dilation has been described as being caused by mutations
in the gene encoding filamin A (FLNA), an actin-binding
protein that links the smooth muscle cell contractile unit to
the cell surface. (See Schwartz 11th ed., p. 855.)

6. Bovine aortic arch: Answer: A


A. Is a normal variant where the innominate and left Bovine aortic arch—a common origin of the innominate and
common carotid arteries have a common origin. left common carotid arteries—has been considered a normal
B. Is associated with bicuspid aortic disease. anatomic variant. Studies from Yale University have identi-
C. Is associated with increased risk of aortic dissection. fied a higher prevalence of bovine aortic arch in patients with
D. Does not appear to be associated with increased risk thoracic aortic disease; an association was found between
of aneurysm. this anomaly and a generalized increase in aortic aneurys-
mal disease (without any predisposition to a particular aor-
tic region). However, bovine aortic arch was not associated
distinctly with bicuspid aortic valve or aortic dissection, but
with a higher mean aortic growth rate: 0.29 cm per year in
patients with bovine aortic arch, compared with 0.09 cm per
year in controls. Therefore, bovine aortic arch may be bet-
ter characterized as a precursor of aortic aneurysm than as a
simple normal anatomic variant. Further studies are needed
to delineate the underlying mechanism for this association.
(See Schwartz 11th ed., p. 856.)

7. Mycotic aneurysms: Answer: B


A. Are associated exclusively with fungal infections of Primary infection of the aortic wall resulting in aneurysm
the aorta. formation is rare. Although these lesions are termed mycotic
B. Can occur via bacterial invasion with endocarditis, aneurysms, the responsible pathogens usually are bacteria
endothelial trauma, or infected laminar clot. rather than fungi. Bacterial invasion of the aortic wall may
C. Is an acute complication of syphilis. result from bacterial endocarditis, endothelial trauma caused
D. Are generally fusiform type. by an aortic jet lesion, or extension from an infected laminar

Brunicardi_Ch22_p181-188.indd 182 06/07/22 9:46 AM


183
clot within a preexisting aneurysm. The most common caus-
ative organisms are Staphylococcus aureus, Staphylococcus
epidermidis, Salmonella, and Streptococcus. Unlike most other
causes of thoracic aortic aneurysms, which generally produce
fusiform aneurysms, infection often produces saccular aneu-
rysms located in areas of aortic tissue destroyed by the infec-
tious process.
Although syphilis was once the most common cause of
ascending aortic aneurysms, the advent of effective antibiotic

CHAPTER 22
therapy has made syphilitic aneurysms a rarity in developed
nations. In other parts of the world, however, syphilitic aneu-
rysms remain a major cause of morbidity and mortality. The
spirochete Treponema pallidum causes an obliterative end-
arteritis of the vasa vasorum that results in medial ischemia
and loss of the elastic and muscular elements of the aortic
wall. The ascending aorta and arch are the most commonly

Thoracic Aneurysms and Aortic Dissection


involved areas. Because syphilitic aortitis often presents
10 to 30 years after the primary infection, the incidence of
associated aneurysms may increase in the near future. (See
Schwartz 11th ed., p. 856.)

8. Aortic diameters at increased risk for rupture, dissection, Answer: A


and mortality include: An analysis by Elefteriades of data from 1600 patients with
A. Critical hinge-point, the diameter where the risk of thoracic aortic disease has helped quantify these well-
rupture increases dramatically, diameters are 6 cm recognized risks. Average expansion rates were 0.07 cm per
for ascending aorta and 7 cm for descending aorta. year in ascending aortic aneurysms and 0.19 cm per year in
B. Fusiform aneurysm is more likely to rupture than descending thoracic aortic aneurysms. As expected, aortic
saccular aneurysm. diameter was a strong predictor of rupture, dissection, and
C. Disease etiology rather than diameter is more rel- mortality. For thoracic aortic aneurysms > 6 cm in diame-
evant than diameter. ter, annual rates of catastrophic complications were 3.6% for
D. Average expansion rates are about 1 cm per year rupture, 3.7% for dissection, and 10.8% for death. Critical
for both ascending and descending thoracic aorta “hinge-point” diameters, at which the incidence of expected
aneurysms. complications significantly increased, were 6 cm for aneu-
rysms of the ascending aorta and 7 cm for aneurysms of the
descending thoracic aorta; the corresponding risks of rupture
after reaching these diameters were 31% and 43%, respec-
tively. (See Schwartz 11th ed., p. 857.)

9. The most common complication of extensive repair for Answer: C


distal aortic aneurysms is: Although spinal cord ischemia and renal failure receive the
A. Spinal cord ischemia. most attention, several other complications warrant consid-
B. Renal failure. eration. The most common complication of extensive repairs
C. Pulmonary dysfunction. is pulmonary dysfunction. With aneurysms adjacent to the
D. Left recurrent laryngeal nerve injury. left subclavian artery, the vagus and left recurrent laryngeal
nerves are often adherent to the aortic wall and thus are sus-
ceptible to injury. (See Schwartz 11th ed., p. 871.)

10. Which of the following is NOT TRUE regarding anasto- Answer: B


motic pseudoaneurysms? Anastomotic pseudoaneurysms can be caused by technical
A. These can arise from deterioration of aortic tissue problems or by deterioration of the native aortic tissue, graft
due to infection. material, or suture. Commonly, they occur in patients with
B. These have increased in incidence with an influx of Marfan syndrome. Tissue deterioration usually is related to
cardiovascular surgery. either progressive degenerative disease or infection. Improve-
C. These commonly occur in patients with Marfan ments in sutures, graft materials, and surgical techniques
syndrome. have decreased the incidence of thoracic aortic pseudoaneu-
D. These are associated with high incidence of morbid- rysms. Should thoracic aortic pseudoaneurysms occur, they
ity and rupture. typically require expeditious surgical or other intervention
because they are associated with a high incidence of morbid-
ity and rupture. (See Schwartz 11th ed., p. 857.)

Brunicardi_Ch22_p181-188.indd 183 06/07/22 9:46 AM


184
11. The most common presenting symptom in patients with Answer: A
an ascending thoracic aneurysm is: Initially, aneurysmal expansion and impingement on adja-
A. Anterior chest pain. cent structures cause mild, chronic pain. The most common
B. Posterior chest pain. symptom in patients with ascending aortic aneurysms is ante-
C. Aortic valve insufficiency. rior chest discomfort; the pain is frequently precordial in loca-
D. Sudden death. tion but may radiate to the neck and jaw, mimicking angina.
Aneurysms of the ascending aorta and transverse aortic arch
can cause symptoms related to compression of the superior
vena cava, the pulmonary artery, the airway, or the sternum.
CHAPTER 22

Rarely, these aneurysms erode into the superior vena cava or


right atrium, causing acute high-output failure. (See Schwartz
11th ed., p. 857.)

12. The primary modes to study for thoracic aorta disease Answer: C
is/are: Although catheter-based contrast aortography was previously
Thoracic Aneurysms and Aortic Dissection

A. Invasive aortography. considered the gold standard for evaluating thoracic aortic
B. Cardiac catheterization. disease, cross-sectional imaging (ie, CT and MRA) has largely
C. Cross-sectional imaging (computed tomography replaced this modality. Technologic improvements have
[CT] and magnetic resonance angiography [MRA]). enabled CT and MRA to provide excellent aortic imaging
D. Abdominal ultrasound. while causing less morbidity than catheter-based studies do,
so CT and MRA are now the primary modes for evaluating
thoracic aortic disease. Today, the use of invasive aortography
in patients with thoracic aortic disease is generally limited to
those undergoing endovascular therapies or when other types
of studies are contraindicated or have not provided satisfac-
tory results. (See Schwartz 11th ed., p. 859.)

13. Endovascular repair of thoracic artery aneurysms is gen- Answer: D


erally not recommended for: Experience with purely endovascular treatment of proxi-
A. Descending thoracic aortic aneurysm and dissection. mal aortic disease remains limited and only investigational.
B. Blunt aortic injury. Endovascular therapy has become an accepted treatment
C. Penetrating aortic ulcers. for descending thoracic aortic aneurysm. Its role in treat-
D. Repair of aneurysms of proximal aorta. ing proximal aortic disease and thoracoabdominal aortic
aneurysm remains experimental; nonetheless, endoluminal
stenting is approved by the US Food and Drug Administra-
tion for the treatment of isolated descending thoracic aortic
aneurysm, and several different devices have been approved
for the treatment of blunt aortic injury and penetrating aortic
ulcer. (See Schwartz 11th ed., pp. 868, 872.)

14. Initial assessment and management of aortic dissection: Answer: B


A. Differs depending on the location and type of the Regardless of the location of the dissection, the initial treat-
dissection. ment is the same for all patients with suspected or confirmed
B. Requires aggressive pharmacologic treatment. acute aortic dissection (see Fig. 22-1). Furthermore, because
C. Requires repair within 24 hours. of the potential for rupture before the diagnosis is confirmed,
D. Is not altered by signs of organ dysfunction. aggressive pharmacologic management is started once there
is clinical suspicion of dissection, and this treatment is con-
tinued during the diagnostic evaluation. The goals of pharma-
cologic treatment are to stabilize the dissection and prevent
rupture. (See Schwartz 11th ed., p. 878.)

Brunicardi_Ch22_p181-188.indd 184 06/07/22 9:46 AM


185

CHAPTER 22
Thoracic Aneurysms and Aortic Dissection
FIG. 22-1. Illustration of the classification schemes for aortic dissection based on which portions of the aorta are involved. Dissection
can be confined to the ascending aorta (left) or the descending aorta (middle), or it can involve the entire aorta (right). (Reproduced with
permission from Baylor College of Medicine.)

15. Mortality rates for operative repair of an aortic arch Answer: A


aneurysm have been significantly reduced intraopera- Like the operations themselves, perfusion strategies used
tively by: during proximal aortic surgery depend on the extent of the
A. Deep hypothermia to allow circulatory arrest. repair. Aneurysms that are isolated to the ascending seg-
B. Innominate and left carotid artery cannulation to ment can be replaced by using standard cardiopulmonary
permit oxygenation of the brain. bypass and distal ascending aortic clamping. This provides
C. Right heart to left subclavian artery bypass to con- constant perfusion of the brain and other vital organs during
tinue brain perfusion. the repair. Aneurysms involving the transverse aortic arch,
D. Use of an intra-aortic balloon pump to maintain dis- however, cannot be clamped during the repair, which neces-
tal circulation. sitates the temporary withdrawal of cardiopulmonary bypass
support; this is called circulatory arrest. To protect the brain
and other vital organs during the circulatory arrest period,
hypothermia must be initiated before pump flow is stopped.
However, hypothermia is not without risk, and coagulopathy
is associated with deep levels of hypothermia (<20°C), which
have been traditionally used in open arch repair. Recently,
more moderate levels of hypothermia (often between 22°C
and 24°C) have been introduced that appear to decrease risks
associated with deep hypothermia while still providing suf-
ficient brain protection. (See Schwartz 11th ed., p. 865.)

16. Endoleaks:
A. Type I and Type IV generally require early and Another significant complication of descending thoracic
aggressive intervention. aortic stent grafting is endoleak. An endoleak occurs when
B. Are uncommon. there is a persistent flow of blood (visible on radiologic imag-
C. Can during the initial procedure or over time. ing) into the aneurysm sac, and it may occur during the
D. Are categorized by leak site. initial procedure or develop over time. Although endoleaks
are a relatively common complication, they are not benign,
because they lead to continual pressurization of the sac,
which can cause expansion or even rupture. These compli-
cations are categorized (Table 22-1) according to the site
of the leak. Although all endoleaks may progress such that
they can be considered life-threatening, type I and type III

Brunicardi_Ch22_p181-188.indd 185 06/07/22 9:46 AM


186
endoleaks generally necessitate early and aggressive interven-
tion. Recently published reporting guidelines aid standard-
ized reporting. (See Schwartz 11th ed., p. 875.)

TABLE 22-1 Classification of and common treatment


strategies for endoleak
Type I
• Incomplete seal between stent graft and aorta at the proximal landing
site (Type Ia), the distal landing site (Type Ib), or branch module,
CHAPTER 22

fenestration, or plug (Type Ic)


• Early reintervention to improve seal or conversion to open surgery
Type II
• Retrograde perfusion of sac from excluded collateral arteries
• Surveillance; as-needed occlusion with percutaneous or other
interventions
Type III
• Incomplete seal between overlapping stent graft or module (Type IIIa),
Thoracic Aneurysms and Aortic Dissection

or tear in graft fabric (Type IIIb)


• Early reintervention to cover or conversion to open surgery
Type IV
• Perfusion of sac due to porosity of material
• Surveillance; as-needed reintervention to reline stent graft
Type V
• Expansion of sac with no identifiable source
• Surveillance; as-needed reintervention to reline stent graft

17. Treatment of descending aortic dissection by nonopera- Answer: C


tive, pharmacologic management: Nonoperative, pharmacologic management of acute descend-
A. Has lower morbidity and mortality rates than tradi- ing aortic dissection results in lower morbidity and mortality
tional surgical treatment. rates than traditional surgical treatment does. The most com-
B. Most common cause of death during nonopera- mon causes of death during nonoperative treatment are aor-
tive treatment are aortic rupture and end-organ tic rupture and end-organ malperfusion. Therefore, patients
malperfusion. are continually reassessed for new complications. At least two
C. A CT scan obtained on day 2 or 3, compared with the serial CT scans—usually obtained on day 2 or 3 and on day 8
initial scan, is sufficient to rule out significant aortic or 9 of treatment—are compared with the initial scan to rule
expansion. out significant aortic expansion. Once the patient’s condition
D. Inadequate blood pressure control has been found to has been stabilized, pharmacologic management is gradu-
be associated with late aneurysm formation. ally shifted from intravenous (IV) to oral medications. Oral
therapy, which usually includes a beta antagonist, is initi-
ated when systolic pressure is consistently between 100 and
110 mm Hg and the neurologic, renal, and cardiovascular
systems are stable. Many patients can be discharged after
their blood pressure is well controlled with oral agents and
after serial CT scans confirm the absence of aortic expansion.
Long-term pharmacologic therapy is important for patients
with chronic aortic dissection. β-Blockers remain the drugs
of choice. In a 20-year follow-up study, DeBakey and col-
leagues found that inadequate blood pressure control was
associated with late aneurysm formation. Aneurysms devel-
oped in only 17% of patients with “good” blood pressure con-
trol, compared with 45% of patients with “poor” control. (See
Schwartz 11th ed., p. 884.)

Brunicardi_Ch22_p181-188.indd 186 06/07/22 9:46 AM


187
18. Which of the following is the most typical presenting Answer: A
symptom in a patient with an aortic dissection? The onset of dissection often is associated with severe chest or
A. “Tearing” pain back pain, classically described as “tearing,” that migrates dis-
B. Paraplegia tally as the dissection progresses along the length of the aorta.
C. Abdominal pain The location of the pain often indicates which aortic segments
D. Cold left arm are involved. Pain in the anterior chest suggests involvement
of the ascending aorta, whereas pain in the back and abdo-
men generally indicates involvement of the descending and
thoracoabdominal aorta. (See Schwartz 11th ed., p. 879.)

CHAPTER 22
19. Delay of emergency repair of ascending aortic dissection Answer: D
should be considered in: Because of the risk of aortic rupture, acute ascending aortic
A. Patients who present with severe acute stroke or mes- dissection is usually considered an absolute indication for
enteric ischemia. emergency surgical repair. However, specific patient groups
B. Dissections that occur in the first 3 weeks after car- may benefit from nonoperative management or delayed
diac surgery. operation. Delayed repair should be considered for patients

Thoracic Aneurysms and Aortic Dissection


C. Patients who are in stable condition and may benefit who (a) present with severe acute stroke or mesenteric isch-
from transfer to specialized centers. emia, (b) are elderly and have substantial comorbidity, (c) are
D. All of the above. in stable condition and may benefit from transfer to special-
ized centers, or (d) have undergone a cardiac operation in
the remote past. Regarding the last group, it is important that
the previous operation should not be too recent; dissections
that occur during the first 3 weeks after cardiac surgery pose
a high risk of rupture and tamponade, and such dissections
warrant early operation. (See Schwartz 11th ed., p. 882.)

20. A patient with a subclavian artery malperfusion as a Answer: B


complication of aortic dissection would most likely See Schwartz 11th ed., Table 22-5, p. 879.
experience:
A. Paraplegia.
B. Cold, painful extremity.
C. Incontinence.
D. Shock.

Brunicardi_Ch22_p181-188.indd 187 06/07/22 9:46 AM


This page intentionally left blank

Brunicardi_Ch22_p181-188.indd 188 06/07/22 9:46 AM


CHAPTER 23
Arterial Disease

1. Which of the following is TRUE about computed tomog- Answer: D


raphy angiography (CTA) in evaluation of arterial CTA is a noninvasive contrast-dependent method of imaging
disease? arterial system. The contrast-filled vessels can be extracted
A. CTA is more accurate than angiography and evalua- from the slices and rendered in the three-dimensional format.
tion of lower extremity arterial occlusive disease. CTA is increasingly being used to image the carotid bifurca-
B. Blooming artifact is the term used to describe artifact tion, and as computing power increases, the speed of image
secondary to thrombus formation in the aorta and acquisition and the resolution will continue to increase. The
arteries. major limitations of multidetector CTA are use of contrast
C. As a low dose of contrast is used, it is unlikely to be and presence of artifacts caused by calcification and stents.
associated with kidney complications. CTA can overestimate the degree of in-stent stenosis, while
D. CTA has sensitivity, specificity, and accuracy that is heavy calcification can limit the diagnostic accuracy of the
equivalent to invasive angiography. method by causing a “blooming artifact.” The artifact can
be overcome with alteration in image acquisition technique.
There are no randomized trials to document the superior-
ity of multidetector CTA of compared to traditional angiog-
raphy, but there is emerging evidence to support the claim
that multidetector CTA has sensitivity, specificity, and accu-
racy that rival invasive angiography. (See Schwartz 11th ed.,
pp. 901–902.)

2. Vascular stents can be used in the vascular system for Answer: B


inadequate angioplasty with dissection or elastic recoil Vascular stents are commonly used after inadequate angio-
of an arterial stenosis. Which of the following statements plasty with dissection or elastic recall of arterial stenosis.
appropriately describes vascular stents? Appropriate indications for primary stenting of the lesion
A. All vascular stents are made of metals that contain without an initial trial of angioplasty alone are evolving and
stainless steel that will help self-expanding. managed that are dependent on the extent and the site of
B. Vascular stents should be oversized by 1 to 2 mm the lesion. Stents are manufactured from a variety of met-
relative to the largest diameter of the normal vessel als including stainless steel, tantalum, cobalt basaloid, and
adjacent to the lesion in order to prevent immediate Nitinol. Vascular stents are classified into two basic catego-
migration. ries: balloon-expandable stents and self-expanding stents.
C. Vascular stents are used in short stenotic segments These stents are always oversized by 1 to 2 mm relative to
because of their ability to expand and accommo- the largest diameter of normal vessel adjacent to the lesion in
date the area of stenosis without shortening or order to prevent immediate migration. With the involvement
lengthening. of the cisterns, there is some degree of shortening that has
D. Balloon expandable stents have a longer time to com- to be taken into account when choosing the area of deploy-
plete endothelialization. ment. Self-expanding stents can continually expand after
delivery; this allows them to accommodate adjacent vessels
of different size. Balloon-expandable stents are usually com-
posed of stainless steel, mounted on an angioplasty balloon,
and deployed by a balloon inflation. Shortening of balloon-
expandable stent during deployment depends on both the
stent geometry and the final diameter to which the balloon

189

Brunicardi_Ch23_p189-196.indd 189 08/07/22 11:41 AM


190
is expanded. The cisterns are more rigid and are associated
with a shorter time to complete endothelialization. They are
often of limited flexibility and have a higher degree of crush
resistance when compared to the self-expanding stents. (See
Schwartz 11th ed., pp. 905–906.)

3. An evaluation of the arterial segments of the lower Answer: D


extremities, pulse volume recording (PVR) has which of PVR is used specifically in patients with noncompressible
the following characteristics? vessels where segmental plethysmography can be used to
CHAPTER 23

A. The cuff is inflated to above systolic pressure to detect determine the underlying arterial occlusive disease. Pressure
pulse volume changes. measurement in such patients is not accurate because of non-
B. Brisk upstroke of the waveform in the pulse volume compressibility. Capsule placed at different levels on the leg
recording is suggestive of proximal disease. detect changes in blood volume and he produced a graft. To
C. When compared with angiography, PVR has an accu- obtain accurate PVR waveforms, the cuff is inflated to a pres-
racy of 30%. sure of 60 to 65 mm Hg, so as to detect volume changes without
D. Its best application is in patients with noncompress- causing arterial occlusion. Pulse volume tracings are suggestive
Arterial Disease

ible vessels that would interfere with accurate mea- of proximal disease of the upstroke of the pulse is not brisk,
surement of segmental pressures. the peak of the wave tracing is rounded, and there is disap-
pearance of the dicrotic notch. Although isolated segmental
limb pressures and PVR measurements are 85% accurate when
compared with angiography in detecting and localizing signifi-
cant atherosclerotic lesions, when using combination, accuracy
reaches 95%. (See Schwartz 11th ed., p. 900.)

4. Which of the following is TRUE about patients with a Answer: D


stroke? Approximately 700,000 Americans suffer an annual or recur-
A. Eighty-five percent of all strokes are hemorrhagic rent stroke each year. Eighty-five percent of all strokes are isch-
and 15% are ischemic. emic and 15% are hemorrhagic. Common causes of ischemic
B. The commonest cause of the stroke is lacunar stroke. stroke are cardiogenic emboli in 35%, carotid artery stenosis
It occurs in about 30% of the patients. in 30%, lacunar in 10%, miscellaneous in 10%, and idiopathic
C. Cerebrovascular accident is used interchangeably in 15%. The term “cerebrovascular accident” is often used
with hemorrhagic stroke. interchangeably to refer to an ischemic stroke. The severity
D. Prior history of neurological symptoms such as tran- of carotid artery stenosis is a strong predictor for stroke with
sient ischemic attack (TIA) or stroke is a predictor for more severe occlusive disease is associated with a higher inci-
recurrent ipsilateral stroke. dence of stroke. A prior history of neurologic symptoms such
as TIA or stroke is an important determinant for recurrent
ipsilateral stroke. (See Schwartz 11th ed., p. 907.)

5. The following conditions qualify patients for being high Answer: C


surgical risk for carotid endarterectomy EXCEPT: Since carotid artery stenting was approved by the US Food
A. High carotid bifurcation above C2 vertebral body. and Drug Administration (FDA) for the clinical application
B. Low common carotid artery, below clavicle. in 2004, the procedure has become a treatment alternative
C. Ipsilateral laryngeal nerve palsy. in patients who are considered high risk for endarterec-
D. Tracheostomy. tomy. Conditions that qualify patients as high surgical risk
for carotid endarterectomy include anatomical factors: High
carotid bifurcation above C2 vertebral body, low common
carotid artery below clavicle, contralateral carotid occlusion,
restenosis of ipsilateral prior carotid endarterectomy, previ-
ous neck irradiation, prior radical neck dissection, contra-
lateral laryngeal nerve palsy, and presence of tracheostomy.
In addition, physiological factors that are considered as
high surgical risk for carotid endarterectomy include: Age
≥ 80 years, left ventricular ejection fraction (<30%), New York
Heart Association class III/IV congestive heart failure, unsta-
ble angina (Canadian Cardiovascular Society class III/IV
angina pectoris), recent myocardial infarction, clinically sig-
nificant cardiac disease (congestive heart failure, abnormal
stress test, or need for coronary revascularization), severe
chronic obstructive pulmonary disease and end-stage renal
disease on dialysis. (See Schwartz 11th ed., pp. 911–912.)

Brunicardi_Ch23_p189-196.indd 190 08/07/22 11:41 AM


191
6. In regard to cerebral monitoring during carotid endar- Answer: B
terectomy, which of the following is TRUE? Of both, carotid endarterectomy is one of the earliest vascular
A. If the patient is awake, the best monitoring is stump operations ever described. This technique has been perfected
pressure. in the last two decades, surgeons continue to debate many
B. Electroencephalogram (EEG) reflecting focal aspects of this procedure. For instance, there is no universal
decreases in amplitude and slowing of the EEG waves agreement with regard to the best anesthetic of choice, the
are reflective of cerebral ischemia. best intraoperative cerebral monitoring, whether to routinely
C. A decrease to <80% of baseline velocity in the ipsi- shunt, open versus eversion endarterectomy, and patch versus
lateral middle cerebral artery is a sign of cerebral primary closure. Depending on the anesthetic given, the sur-

CHAPTER 23
ischemia. geon must decide whether intraoperative cerebral monitoring
D. Stump pressure measurement is the most sensitive is necessary or intra-arterial carotid shunting would be used.
test for potential cerebral ischemia. In general, if the patient is awake, then his or her abilities to
respond to commands during carotid clamp determine the
adequacy of cerebral perfusion to the ipsilateral hemisphere.
On the other hand, intraoperative EEG or transcranial power
Doppler (TCD) has been used to monitor for adequate cere-

Arterial Disease
bral perfusion during the clamp. For patients undergoing
surgery under general anesthesia, focal ipsilateral decreases
in amplitude and slowing of EEG waves are indicative of cere-
bral ischemia. Similarly, a decrease to <50% of baseline veloc-
ity in the ipsilateral middle cerebral artery is a sign of cerebral
ischemia. For patients with poor collateral flow exhibiting
signs of cerebral ischemia, intra-arterial carotid shunting
with removal of the clamp will restore cerebral flow for the
remaining part of the surgery. Stump pressures have been
used to determine the need for intra-arterial carotid shunt-
ing. Some surgeons prefer to shunt all patients on a routine
basis and not use intraoperative cerebral monitoring. (See
Schwartz 11th ed., pp. 912–913.)

7. Regarding carotid body tumors, which of the following is Answer: D


TRUE? The carotid body origin is from the third branchial arch and
A. The normal carotid body is located in the media or from neuro ectodermal derived in neural crest lineage. The
the transition. Between the media and intima at the normal carotid body is located in the adventitia or periadven-
bifurcation of the common carotid artery. titial tissue at the bifurcation of the common carotid artery.
B. Carotid body tumors are rare lesions of neuromuscu- The gland is innervated by the glossopharyngeal nerve. Its
lar system. blood supply is derived predominantly from the external
C. Approximately 20% to 25% of carotid body tumors carotid artery but can also come from the vertebral artery.
are malignant. Carotid body tumor is a rare lesion of the neuroendocrine
D. About 35% of the carotid body tumors are hereditary. system. Tumors involving the neural crest origin cells have
been referred to as part of a ganglioneuroma, glomus tumor,
or chemodectoma. Approximately 5% to 7% of carotid body
tumors are malignant. Although chronic hypoxemia has been
involved as a stimulus for hyperplasia of the carotid body,
approximately 35% of carotid body tumors are hereditary.
The risk of malignancy is greatest in young patients with
familial tumors. (See Schwartz 11th ed., pp. 918–919.)

8. Which of the following is TRUE in relation to abdominal Answer: B


aortic aneurysm? An AAA is defined as a pathologic focal dilatation of the
A. Sixty percent of an AAA are infrarenal in location. aorta that is >30 mm or 1.5 times the adjacent diameter of
B. There is a higher predilection for juxtarenal and the normal aorta. Male aortas tend to be larger than female
suprarenal AAAs in women compared with men. aortas, and there is generalized growth of the aortic diameter
C. Clinically significant peripheral occlusive disease is with each decade of life. Ninety percent of AAAs are infra-
common with AAA and is present in about 40% of all renal in location and have a fusiform morphology. There is
cases. a higher predilection for juxtarenal and suprarenal AAAs
D. Concomitant common iliac and/or hypogastric in women compared with men. Concomitant common iliac
artery aneurysms of care in about 50% to 55% of and/or hypogastric artery aneurysms can be found in 20%
patients. to 25% of patients. Although the etiology of most aortic

Brunicardi_Ch23_p189-196.indd 191 08/07/22 11:41 AM


192
aneurysms is atherosclerotic, clinically significant peripheral
occlusive disease is unusual and present in <10% of all cases.
(See Schwartz 11th ed., pp. 920–921.)

9. The ideal characteristics of an aneurysm for endovascu- Answer: A


lar abdominal aortic aneurysm repair include all of the Anatomic eligibility for endovascular repair is mainly based
following EXCEPT: on three areas: The proximal aortic neck, common iliac arter-
A. Neck length more than 20 mm. ies, and external iliac and common femoral arteries, which
B. Neck diameter >18 mm, and <32 mm. relate to the proximal and distal landing zones or fixation
CHAPTER 23

C. External iliac artery diameter >7 mm. site and the access vessels, respectively. The requirements
D. Aortic neck angle < 60 degrees. for proximal aortic neck are diameter of 18 to 28 mm and
the minimum length of 15 mm. Usually, multiple measure-
ments of the diameter are taken along the length of the neck
to assess its shape. All diameter measurements are made from
mid wall to mid wall of the vessel. Secondary considerations
include the mural calcification < 50% circumference, luminal
Arterial Disease

thrombus < 50% circumference, and angulation < 45 degrees.


Presence of significant amount of any one of these secondary
features in combination with a relatively short proximal neck
may compromise successful short- and long-term fixation of
the stent graft and exclusion of the aneurysm. The treatable
diameters for common iliac arteries range from 8 to 20 mm,
and there should be at least 20 mm of patent artery of uni-
form diameter to allow adequate fixation. Finally, at least one
of two common femoral and external iliac arteries must be at
least 7 mm in diameter in order to safely introduce the main
delivery sheath. Slightly smaller iliac diameters may be toler-
ated depending on the specific device and in the absence of
severe tortuosity and calcific disease. Difficult access is one of
the main causes of increased procedural time and intraopera-
tive complications. Using these criteria, approximately 60%
of all AAAs are anatomic candidates for endovascular repair.
(See Schwartz 11th ed., pp. 924–925.)

10. Which of the following is TRUE related to mesenteric Answer: B


ischemia and mesenteric circulation? Vascular occlusive disease of the mesenteric arteries is a rela-
A. Thrombosis is the commonest cause of mesenteric tively uncommon but potentially devastating condition that
ischemia in young patients. generally presents in patient over 60 years of age. It is a three
B. Nonocclusive mesenteric ischemia most frequently times more frequent in women. There are three major mecha-
occurs in critically ill patients on vasopressors. nisms for visceral ischemia involving the mesenteric arteries:
C. Duplex ultrasonography has accuracy of 50% for Acute mesenteric ischemia, chronic mesenteric ischemia,
detecting stenosis >70% in the superior mesenteric and nonocclusive mesenteric ischemia. The SMA is the most
artery (SMA). commonly involved vessel in acute mesenteric ischemia.
D. Endovascular intervention is not indicated in patients Acute thrombosis occurs in patients with underlying mes-
with asymptomatic occlusive disease of the mesen- enteric atherosclerosis, which typically involves the origin of
teric arteries. the mesenteric arteries while sparing the collateral branches.
In acute embolic mesenteric ischemia, the emboli typically
originate from the cardiac source. Nonocclusive mesenteric
ischemia is characterized by a low flow state and otherwise
normal mesenteric arteries and most frequently occurs in
critically ill patients on vasopressors. Duplex ultrasonogra-
phy is a valuable noninvasive means of assessing the patency
of mesenteric vessels. A peak systolic velocity in the SMA of
>275 cm/s demonstrated a sensitivity of 92%, specificity of
96%, and overall accuracy of 96% for detecting >70% stenosis.
Similarly, duplex ultrasound had sensitivity and specific-
ity of 87% and 82%, respectively with an accuracy of 82%
in predicting >70% stenosis in the celiac trunk. Endovascu-
lar treatment of mesenteric artery stenosis or short segment

Brunicardi_Ch23_p189-196.indd 192 08/07/22 11:41 AM


193
occlusion by balloon dilatation or stent placement represent
a less invasive therapeutic alternative to open surgical inter-
vention particularly in patients whose medical comorbidities
place them at a higher operative surgical risk. Endovascular
therapy is also suited in patients with recurrent disease or
anastomotic stenosis following previous open mesenteric
revascularization. Prophylactic mesenteric revascularization
is rarely performed in the asymptomatic patient undergo-
ing an aortic procedure for other indications. However, the

CHAPTER 23
natural history of untreated chronic mesenteric ischemia may
justify revascularization in some minimally symptomatic or
asymptomatic patients if the operative risks are acceptable,
since the first clinical presentation may be acute intestinal
ischemia in as many as 50% of patients, with a mortality
rate that ranges from 15% to 70%. Mesenteric angioplasty
and stenting is particularly suited for this patient subgroup

Arterial Disease
given its low morbidity and mortality. (See Schwartz 11th ed.,
pp. 928–933.)

11. Which of the following is TRUE about renal artery Answer: D


disease? Obstructive lesions of the renal artery can produce hyperten-
A. Renal artery occlusive disease affects 5% to 10% of all sion and it is reported in about 5% to 10% of all hyperten-
hypertensive patients in the United States. sive patients in the United States. Approximately 80% of all
B. The most common cause of renal artery occlusive renal artery occlusive lesions are caused by atherosclerosis,
disease is atherosclerosis in 80% of patients followed which typically involves a short segment of the renal artery
by fibromuscular dysplasia. ostia and represent spillover disease from a severely ath-
C. Renal artery revascularization is indicated in patients eromatous aorta. The second most common cause of renal
with recurrent congestive heart failure or flash pul- artery stenosis is fibromuscular dysplasia, which accounts for
monary edema not attributable to active coronary 20% of cases and is most frequently encountered in young,
ischemia in the presence of >70% stenosis of the renal often multiparous women. Fibromuscular dysplasia of the
artery. renal artery presents a heterogeneous group of lesions that
D. All of the above. can produce histopathological changes in the intima, media
or adventitia. The most common variety consists of medial
fibroplasia, in which thickened fibromuscular ridges alternate
with attenuated media producing the classic angiographic
“string of beads” appearance.
Indications for renal arterial revascularization include
≥70% stenosis of one or both renal arteries and at least one of
the following clinical criteria: Inability to adequately control
hypertension despite appropriate antihypertensive regime;
chronic renal insufficiency related to bilateral renal artery
occlusive disease or stenosis to a solitary functioning kidney;
dialysis-dependent renal failure in a patient with renal artery
stenosis but without another definite cause of end-stage renal
disease and recurrent congestive heart failure or flash pulmo-
nary edema not attributable to active coronary ischemia. (See
Schwartz 11th ed., pp. 935–940.)

12. Which of the following is TRUE about atherosclerotic Answer: B


aortoiliac occlusive disease? The distal abdominal aorta and the iliac arteries are common
A. Classified into three times with type I occurring sites affected by atherosclerosis. Based on the atherosclerotic
in 5% to 10% to patient extending into the lower disease pattern, aortoiliac occlusive disease can be classified
extremities below the knee. into three types. Type I aortoiliac disease, which occurs in 5%
B. Type II aortoiliac disease represents diffuse aortoiliac to 10% of patient, is confined to the distal abdominal aorta
disease above the inguinal ligament. and common iliac vessels. Due to the localized nature of this
C. Type III represents multisegment occlusive disease type of aortic obstruction and formation of collateral blood
involving the aortoiliac area and extending to the flow around the occluded segment, limb-threatening symp-
supra renal aorta. toms are rare in the absence of more distal disease. This type of
D. Type I is more frequently found in men. aortoiliac occlusive disease occurs in a relatively young group

Brunicardi_Ch23_p189-196.indd 193 08/07/22 11:41 AM


194
of patients. Patients with type I disease pattern have a lower
incidence of hypertension and diabetes with a significant fre-
quency of abnormal blood lipid levels particularly type IV
hyperlipoproteinemia. Type II aortoiliac disease represents
a more diffuse atherosclerotic progression and involves pre-
dominantly the abdominal aorta with disease extension into
the common iliac artery. This disease pattern affects approxi-
mately 25% of patients with aortoiliac occlusive disease.
Type III aortoiliac occlusive disease, which affects approxi-
CHAPTER 23

mately 65% patient with aortoiliac occlusive disease, is wide-


spread disease that is seen above and below the inguinal
ligament. Patients with multilevel disease are older, more
commonly with a male to female ratio of 6:1 and much more
likely to have diabetes, hypertension, and associated athero-
sclerotic disease involving cerebral, coronary, and basilar
arteries. (See Schwartz 11th ed., pp. 941–942.)
Arterial Disease

13. In regard to obturator bypass, which of the following is Answer: D


TRUE? An obturator bypass is used to reconstruct arterial anatomy
A. It is indicated in patients with groin hematoma and in patients with a groin sepsis resulting from prior prostatic
fever. grafting, intra-arterial drug abuse, growing neoplasm, or
B. The conduit of Dacron, polytetrafluoroethylene damage from prior groin irradiation. This bypass can origi-
(PTFE), or autologous vein is tunneled through the nate from the common iliac artery, external iliac artery, or
posteromedial portion of the obturator membrane. uninvolved limb of an aortobifemoral bypass. The conduit is
C. The obturator artery and nerve pass in the anterome- tunneled through the anteromedial portion of the obturator
dial portion of the membrane. membrane to the distal superficial femoral artery or popliteal
D. 5-year patency of 57% has been reported. artery. The membrane must be divided sharply to avoid injury
to adjacent structures. Care must be taken to identify the
obturator artery and nerve that pass posterolaterally. There
have been very good results in terms of patency and limb sal-
vage for obturator bypass. Some authors have reported 57%
5-year patency and 77% 5-year limb salvage rates, whereas
others have shown a higher rate of reinfection and low
patency requiring reintervention. (See Schwartz 11th ed.,
p. 946.)

14. In patients with acute limb ischemia, the following is Answer: B


TRUE EXCEPT: Acute lower extremity ischemia manifesting with the five Ps:
A. Immediate anticoagulation is indicated. Pain, pallor, paresthesia, paralysis, and pulselessness. The
B. Hypercoagulable workup should be performed prior sixth P, “poikilothermia” or “perishing cold,” is added. In the
to heparin initiation in all patients. absence of any significant contraindication, the patient with
C. There is no clear superiority for thrombolysis over an ischemic lower extremity should be immediately anticoag-
surgery in terms of a 30-day limb salvage on mortality. ulated. This will prevent propagation of the clot into the unaf-
D. In the United States, surgery is used three- to five fold fected vascular beds. Intravenous fluid should be started and
more frequently than thrombolysis. a Foley catheter inserted to monitor urine output. Baseline
labs should be obtained but hypercoagulable workup should
be performed only prior to initiation of heparin if there is a
sufficient suspicion. There is no clear superiority for throm-
bolysis over surgery in terms of a 30-day limb salvage on mor-
tality. Access to each treatment option is a major issue in the
decision-making process, as time is often critical. National
registry data from the United States revealed that surgery is
used in three- to five fold more frequently than thrombolysis.
(See Schwartz 11th ed., p. 954-955.)

Brunicardi_Ch23_p189-196.indd 194 08/07/22 11:41 AM


195
15. In the absence of a suitable vein for lower extremity Answer: C
bypass, prosthetic grafts can be used. Which of the fol- If a vein is unavailable, polytetrafluoroethylene (PTFE) or
lowing can be used to improve patency of prosthetic Dacron can be used for above-knee bypass or in some cases
grafts? below the knee. The addition of rings to the PTFE did not
A. Altering the geometry at the distal anastomosis by confer benefit in a single prospective, randomized clinical
increasing the size of the arteriotomy trial. For infrageniculate prosthetic bypasses, use of a vein
B. Bonding agents into the outer surface of the graft patch, cuff, or other venous anastomotic modifications can
C. Bonding anticoagulants to the conduit to improve improve patency. Methods to improve prosthetic graft per-
patency of the graft formance have consisted of altering the geometry at the dis-

CHAPTER 23
D. Applying rings to the graft to improve patency tal anastomosis to get the benefit obtained with vein cuffs
and covalently binding agents onto the luminal surface with
anticoagulant, anti-inflammatory, and antiproliferative char-
acteristics. A comparison of precuffed PTFE versus PTFE
with a vein cuff showed a 1 year and 2 years primary patency
rates were 52% and 49% in the precuffed group and 62% and
44% in the vein cuff group, respectively. Another approach

Arterial Disease
for improving outcomes using prosthetic for bypass grafts
involves binding anticoagulants to the conduit. The Gore pro-
paten graft has heparin-bonded into the luminal surface of
the PTFE graft using Carmeda bioactive surface technology.
The heparin-binding does not alter the micro structure and
handling characteristic of the PTFE. A prospective random-
ized trial suggested that heparin-bonded Dacron or PTFE
was superior to plain PTFE for above-knee popliteal bypass.
The 3-year primary patency rate for the heparin-bonded graft
was 55% compared with 42% for the PTFE. But both of these
patency rates are inferior to greater saphenous vein grafts.
(See Schwartz 11th ed., p. 967-968.)

16. The following is TRUE about fibromuscular dysplasia Answer: C


EXCEPT: FMD is a vasculopathy of uncertain etiology that is character-
A. The characteristic beaded appearance of fibromuscu- ized by segmental arterial involvement. Histologically, fibrous
lar dysplasia (FMD) is due to areas of medial thin- tissue proliferation, smooth muscle cell hyperplasia, and elas-
ning alternating with areas of stenosis. tic fiber destruction alternate with mural thinning. The char-
B. The most commonly affected are the medium-sized acteristic beaded appearance of fibromuscular dysplasia is
arteries. due to areas of medial thinning alternating with areas of ste-
C. The commonest involved arteries are the internal nosis. The most commonly affected arteries are the medium-
carotid arteries. sized arteries, including the renal, internal carotid, vertebral,
D. FMD occurs most frequently in women. subclavian, mesenteric, and iliac arteries. The internal carotid
artery is the second most common site of involvement after the
renal arteries. FMD occurs most frequently in women (90%)
and is recognized at approximately 55 years of age. Only 10%
of patients with FMD will have complications attributable to
the disease. Pathologically, FMD is heterogeneous group of
four distinct types of lesions that are subgrouped based on
the predominant site of involvement within the vessel wall. Of
the four types (medial fibroplasia, intimal fibroplasia, medial
hyperplasia, and perimedial dysplasia), medial fibroplasia is
the most common pathological type, affecting the internal
carotid artery and the renal artery and occurring in 85% of
the reported cases. (See Schwartz 11th ed., pp. 972–973.)

Brunicardi_Ch23_p189-196.indd 195 08/07/22 11:41 AM


196
17. Which of the following is TRUE about adventitial cystic Answer: B
disease of the popliteal artery? The adventitial cystic disease of the popliteal artery was first
A. It occurs in about 5% usually in the popliteal artery. described in 1954. It is a rare arterial condition occurring at
B. The disease affects men in a ratio approximately an incidence of 0.1%, usually in the popliteal artery. This dis-
of 5:1. ease affects men in a ratio of approximately 5:1. The incidence
C. Patients present in their 40s and 50s. is approximately 1 in 1200 cases of claudication or 1 and 1000
D. Computed tomography angiogram (CTA) of the peripheral arteriograms. The disease may affect other vas-
lower extremities is the best diagnostic modality. cular sites, such as the femoral, external iliac, radial, ulnar,
and brachial arteries. The diagnosis should be considered in
CHAPTER 23

young patients with a mass in a nonaxial vessel in proximity


to a related joint. The synovial-like mucin-filled cyst reside
in the subadventitial layer of the vessel wall and have a simi-
lar microscopical appearance to a ganglion cyst. Despite the
similarity and suggestion of a joint origin for these lesions,
histochemical markers have failed to link the cystic lining
to synovium. Patient presenting at a young age, with bilat-
Arterial Disease

eral lower extremity claudication and minimal risk factors


for atheroma formation should be evaluated for adventitial
cystic disease as well as for popliteal artery entrapment syn-
drome and Buerger’s disease. Peripheral pulses may be pres-
ent in the limb when extended, but then can disappear during
knee-joint flexion. Noninvasive studies may suggest arterial
stenosis with elevated velocities. Color-flow duplex scanning
followed by T2-weighted magnetic resonance imaging (MRI)
now appears to be the best diagnostic choice. Angiography
will demonstrate a smooth, well-defined, crescent-shaped
filling defect, the classic “scimitar” sign. There may be asso-
ciated calcifications in the cyst wall and no other evidence
of atherosclerotic occlusive disease. The recommended treat-
ments are excision of the cyst with the cystic wall, enucleation,
or simple aspiration when the artery is stenotic. Retention of
the cystic lining leads to continued secretion of the cystic
fluid and recurrent lesions. And 30% of patients who have
an occluded artery, resection of the affected artery, followed
by an interposition graft using autogenous saphenous vein, is
recommended. (See Schwartz 11th ed., p. 973.)

Brunicardi_Ch23_p189-196.indd 196 08/07/22 11:41 AM


CHAPTER 24
Venous and Lymphatic Disease

1. All of the following regarding venous anatomy are TRUE Answer: C


EXCEPT: Veins are thin-walled, highly distensible, and collapsible.
A. Veins are thin-walled, collapsible, and highly disten- Their structure specifically supports the primary functions of
sible to a diameter several times greater than that in veins to transport blood toward the heart and serve as a res-
the supine position. ervoir to prevent intravascular volume overload.
B. The small saphenous vein (SSV) is accompanied by Lower extremity veins are divided into superficial, deep,
the sural nerve. and perforating veins. The superficial venous system lies
C. The radial, ulnar, brachial, anterior tibial, posterior above the uppermost fascial layer of the leg and thigh and
tibial, peroneal, and popliteal veins usually exist as consists of the great saphenous vein (GSV) and SSV and
paired veins. their tributaries. The GSV originates from the dorsal pedal
D. The subclavian vein courses anterior to the anterior venous arch and courses cephalad and medially, anterior to
scalene muscle. the medial malleolus, entering the common femoral vein
approximately 4 cm inferior and lateral to the pubic tubercle.
The saphenous nerve accompanies the GSV medially from
the ankle to the level of the knee and supplies cutaneous sen-
sation to the medial leg and ankle. The SSV originates later-
ally from the dorsal pedal venous arch and courses cephalad
in the posterior calf. Most often, it penetrates the popliteal
fossa, between the medial and lateral heads of the gastrocne-
mius muscle, to join the popliteal vein. The termination of the
SSV may be quite variable, however, with a proximal exten-
sion of the SSV (the vein of Giacomini) connecting with the
deep femoral vein or GSV. The sural nerve accompanies the
SSV laterally along its course and supplies cutaneous sensa-
tion to the lateral malleolar region.
The deep veins follow the course of major arteries in the
extremities. In the lower leg, paired veins parallel the course
of the anterior tibial, posterior tibial, and peroneal arteries,
to join behind the knee forming the popliteal vein. Venous
bridges connect the paired axial tibial veins in the lower leg.
The popliteal vein continues through the adductor hiatus to
become the femoral vein. In the proximal thigh, the femoral
vein joins with the deep femoral vein to form the common
femoral vein, becoming the external iliac vein at the inguinal
ligament.
As in the lower extremity, there are deep and superficial
veins in the upper extremity. Deep digital veins form the
palmar venous arches of the hand and empty into the paired
radial and ulnar veins. These follow the named arteries in the
arm and are known as the venae comitantes. They become the
brachial veins most often near the antecubital fossa and then
combine to contribute to forming the axillary vein. Superficial

197

Brunicardi_Ch24_p197-206.indd 197 08/07/22 11:47 AM


198
veins of the upper extremity are the cephalic and basilic veins
and their tributaries. The cephalic vein originates at the lat-
eral wrist and courses over the lateral ventral surface of the
forearm. In the upper arm, the cephalic vein terminates in
the infraclavicular fossa, piercing the clavipectoral fascia to
empty into the axillary vein. The basilic vein runs medially
along the forearm and penetrates the deep fascia as it courses
past the elbow in the upper arm. It then joins with the deep
brachial veins to become the axillary vein, a landmark for
CHAPTER 24

identification of the axillary vein. The median antecubital


vein joins the cephalic and the basilic veins on the ventral
surface of the elbow.
The axillary vein becomes the subclavian vein at the lateral
border of the first rib. At the medial border of the scalenus
anterior muscle, the subclavian vein joins with the inter-
nal jugular vein to become the brachiocephalic vein, with
Venous and Lymphatic Disease

the subclavian vein coursing anterior to the scalenus ante-


rior muscle. The left and right brachiocephalic veins join to
become the superior vena cava, which empties into the right
atrium. (See Schwartz 11th ed., p. 981.)

2. The target organ(s) of chronic venous insufficiency Answer: C


is/are: Chronic venous insufficiency (CVI) may lead to characteristic
A. Perforator veins. changes in the skin and subcutaneous tissues in the affected
B. The great saphenous veins. limb. CVI results from incompetence of venous valves, venous
C. The skin. obstruction, or both. Most CVI involves venous reflux, and
D. Lymphatics. severe CVI often reflects a combination of reflux and venous
E. Venous capillaries. obstruction. It is important to remember that although CVI
originates with abnormalities of the veins, the target organ
of CVI is the skin, and the underlying physiologic and bio-
chemical mechanisms leading to the cutaneous abnormali-
ties associated with CVI are poorly understood. A typical
leg affected by CVI will be edematous, with edema increas-
ing over the course of the day. The leg may also be indurated
and pigmented with eczema and dermatitis. These changes
are associated with excessive proteinaceous capillary exudate
and deposition of a pericapillary fibrin cuff that may limit
nutritional exchange. In addition, an increase in white blood
cell trapping within the skin microcirculation in CVI patients
may lead to microvascular congestion and thrombosis. Sub-
sequently, white blood cells may migrate into the interstitium
and release necrotizing lysosomal enzymes, potentially lead-
ing to tissue destruction and eventual ulceration.
Fibrosis can eventually develop from impaired nutrition,
chronic inflammation, and fat necrosis (­lipodermatosclerosis).
Hemosiderin deposition due to the extravasation of red cells
and subsequent lysis in the skin contributes to the charac-
teristic pigmentation of chronic venous disease (Fig. 24-1).
Ulceration can develop with long-standing venous hyperten-
sion and is associated with alterations in microcirculatory
and cutaneous lymphatic anatomy and function. The most
common location of venous ulceration is approximately 3 cm
proximal to the medial malleolus (Fig. 24-2). (See Schwartz
11th ed., p. 983.)

Brunicardi_Ch24_p197-206.indd 198 08/07/22 11:47 AM


199

CHAPTER 24
Venous and Lymphatic Disease
FIG. 24-1. Characteristic hyperpigmentation
of chronic venous insufficiency.

FIG. 24-2. Venous ulceration located


proximal to the medial malleolus.

Brunicardi_Ch24_p197-206.indd 199 08/07/22 11:47 AM


200
3. Venous thromboembolism (VTE) is associated with all Answer: D
of the following EXCEPT: The incidence of VTE is approximately 100 per 100,000 peo-
A. Increased morbidity and mortality. ple per year in the general population, with 20% of the diag-
B. Pulmonary hypertension. noses made within 3 months of a surgical procedure. Of the
C. Postthrombotic syndrome. symptomatic patients, one-third will present with pulmonary
D. No change in future risk of VTE. embolism (PE) and two-thirds with deep vein thrombosis
(DVT). The estimated number of cases of VTE may well be
over 600,000 per year in the United States, making it a major
US health problem. Furthermore, death occurs in 6% of DVT
CHAPTER 24

and 12% of PE cases within 1 month of diagnosis. Not only


does VTE pose a veritable threat to life, but it also places
patients at higher risk for recurrence and post-VTE sequelae
such as pulmonary hypertension and postthrombotic
­syndrome, with 4% and up to 30% incidence, respectively.
(See Schwartz 11th ed., p. 984.)
Venous and Lymphatic Disease

4. Which of the following factors is the most important in Answer: C


the development of spontaneous deep vein thrombosis? Three conditions, first described by Rudolf Virchow in 1862,
A. Stasis contribute to venous thromboembolism (VTE) formation:
B. Endothelial Damage stasis of blood flow, endothelial damage, and hypercoagu-
C. Hypercoagulability lability. Of these risk factors, relative hypercoagulability
D. All of the above appears most important in most cases of spontaneous VTE,
or ­so-called idiopathic VTE, whereas stasis and endothe-
lial damage likely play a greater role in secondary VTE, or
so-called provoked VTE, occurring in association with
­
­transient risk factors such as immobilization, surgical proce-
dures, and trauma. (See Schwartz 11th ed., p. 984.)

5. All of the following are acquired risk factors for venous Answer: B
thromboembolism (VTE) EXCEPT: The more common acquired VTE risk factors include older
A. Nephrotic syndrome. age (>40 years), hospitalization and immobilization, hor-
B. Factor V Leiden. mone replacement and oral contraceptive therapy, pregnancy
C. Malignancy. and the recently postpartum state, prior VTE, malignancy,
D. Pregnancy. major surgery, obesity, nephrotic syndrome, trauma and spi-
E. Obesity. nal cord injury, long-haul travel (>6 hours), varicose veins,
F. Varicose veins. antiphospholipid syndrome, myeloproliferative disorders,
and polycythemia. Heritable risk factors include male sex,
factor V Leiden mutation; prothrombin 20210A gene variant;
antithrombin, protein C, and protein S deficiencies; and dys-
fibrinogenemias. In some patients, the cause of the thrombo-
philia may have both a heritable and an acquired component.
These mixed causes include homocysteinemia; factors VII,
VIII, IX, and XI elevation; hyperfibrinogenemia; and acti-
vated protein C resistance in the absence of factor V Leiden.
There may be a synergistic effect when particular multiple
inherited and acquired risk factors are present in the same
patient.
Other patient-specific factors associated with venous
thrombosis include the traditional cardiovascular risk fac-
tors of obesity, hypertension, and diabetes. VTE is more
common in whites and African Americans than Asians and
Native Americans. Certain gene variants (single nucleotide
polymorphisms) are also associated with a mildly increased
risk for VTE, and their presence may interact with other risk
­factors to increase the overall risk for venous thrombosis.
(See Schwartz 11th ed., Figure 24-3, p. 984.)

Brunicardi_Ch24_p197-206.indd 200 08/07/22 11:47 AM


201
6. May-Thurner syndrome is an anatomical factor associ- Answer: A
ated with increased deep vein thrombosis (DVT) forma- Anatomic factors may also contribute to development of DVT.
tion, and is characterized by which of the following? At the site where the right iliac artery crosses over the left iliac
A. Narrowing of the left iliac vein at the site where the vein, the left iliac vein may become chronically narrowed pre-
right iliac artery crosses over it disposing to iliofemoral venous thrombosis, so-called May-
B. Narrowing of the left renal vein as it traverses beneath Thurner syndrome. External compression of major veins by
the superior mesenteric artery masses of various types can also lead to venous thrombosis.
C. Subclavian vein narrowing due to repetitive upper (See Schwartz 11th ed., p. 985.)
extremity effort

CHAPTER 24
D. A rapidly expanding hemangioma

7. All of the following are absolute contraindications to Answer: D


catheter-directed thrombolysis EXCEPT: There are contraindications to thrombolytic therapy. Absolute
A. Prior history of ischemic or hemorrhagic stroke contraindications include prior history of ischemic or hemor-
within 3 months. rhagic stroke within 3 months, head trauma within 3 months,
B. Traumatic cardiopulmonary resuscitation within 3 neurologic surgery within 6 months, known intracranial

Venous and Lymphatic Disease


weeks. neoplasm, internal bleeding within 6 weeks, active or known
C. Known intracranial neoplasm. bleeding disorder, traumatic cardiopulmonary resuscitation
D. Age > 75 years. within 3 weeks or suspected aortic dissection. Fortunately,
serious remote bleeding is uncommon, and intracranial hem-
orrhage rarely occurs. The majority of bleeding complications
are limited to the venous access site. Symptomatic pulmo-
nary embolism occurs uncommonly and is very rarely fatal.
(See Schwartz 11th ed., pp. 990–991.)

8. Phlegmasia cerulea dolens is best described as: Answer: C


A. Asymptomatic, but extensive deep vein thrombosis Clinical symptoms may worsen as DVT propagates and
(DVT). involves the major proximal deep veins. Extensive DVT of the
B. Isolated popliteal vein thrombosis. major axial deep venous channels of the lower extremity with
C. Extensive DVT of the major axial deep venous chan- relative sparing of collateral veins causes a condition called
nels of the lower extremity potentially complicated by phlegmasia cerulea dolens (Fig. 24-3). This condition is char-
venous gangrene and/or the need for amputation. acterized by pain and pitting edema with associated cyanosis.
D. Painless lower extremity swelling. When the thrombosis extends to the collateral veins, mas-
sive fluid sequestration and more significant edema ensue,
resulting in a condition known as phlegmasia alba dolens. The
affected extremity in phlegmasia alba dolens is extremely pain-
ful and edematous and pale secondary to arterial insufficiency
from dramatically elevated below lower knee compartment
pressures. Both phlegmasia cerulean dolens and phlegmasia
alba dolens can be complicated by venous gangrene and the
need for amputation. (See Schwartz 11th ed., p. 986.)

FIG. 24-3. Phlegmasia cerulea dolens of


the left leg. Note the bluish discoloration.

Brunicardi_Ch24_p197-206.indd 201 08/07/22 11:47 AM


202
9. According to the American College of Chest Physicians, Answer: C
the recommended duration of long-term antithrombotic Table 24-1 (See Schwartz 11th ed., Table 24-4, p. 990.)
therapy after provoked deep vein thrombosis (DVT) is:
A. 2 weeks.
B. 1 month. TABLE 24-1 Summary of American College of Chest Physicians recommendations
C. 3 months. regarding duration of long-term antithrombotic therapy for deep vein
D. 6 months. thrombosis (DVT)
Clinical Subgroup Antithrombotic Treatment Duration
First episode DVT/transient risk/surgery VKA or LMWH for 3 months
CHAPTER 24

First episode DVT/unprovoked VKA or LMWH for 3 months


Consider for long-term therapy if:
• Proximal DVT
• Minimal bleeding risk
• Stable coagulation monitoring
Distal DVT/unprovoked
• Symptomatic VKA for 3 months
Venous and Lymphatic Disease

• Asymptomatic and no risk factors for progression Serial imaging in 2 weeks, if progression VKA
for 3 months
Second episode DVT/unprovoked VKA for extended therapy
DVT and cancer LMWH for extended therapy over VKA
LMWH = low molecular weight heparin; VKA = vitamin K antagonist.
Data from Kearon C, Akl EA, Comerota AJ, et al: Antithrombotic therapy for VTE disease: Antithrombotic Therapy and Prevention
of Thrombosis, 9th ed: American College of Chest Physicians Evidence-Based Clinical Practice Guidelines, Chest. 2012;
141(2 Suppl): e419S-e496S.

10. Which of the following statements related to inferior Answer: C


vena cava (IVC) filters is TRUE? When possible, anticoagulation therapy should be continued
A. Placement of an IVC filter allows for a reduced dura- in patients with vena cava filters. The duration of anticoagu-
tion of anticoagulation therapy. lation is determined by the underlying venous thromboem-
B. In patients with a proximal DVT, placement of an bolism (VTE) and not by the presence of the IVC filter itself.
IVC filter reduces rate of pulmonary embolism, and Practically speaking, however, many patients who require an
also prolongs early and late survival. IVC filter for recurrent VTE are the same ones who would
C. The rate of fatal complications related to IVC filters is benefit most from indefinite anticoagulation. In patients who
<0.12%. are not able to receive anticoagulants due to recent surgery or
D. An IVC filter is safe to leave in place after it is no trauma, the clinician should continually reassess if anticoagu-
longer needed. lation may be started safely at a later date.
E. All of above. Placement of permanent IVC filters has been evaluated as
an adjunct to routine anticoagulation in patients with proxi-
mal DVT. Routine IVC filter placement has not been shown to
prolong early or late survival in patients with proximal DVT
but did decrease the rate of PE (HR, 0.22; 95% CI, 0.05–0.90);
however, there is an increased rate of recurrent DVT in
patients with IVC filters (HR, 1.87; 95% CI, 1.10–3.20).
IVC filters are associated with acute and late complications.
Acute complications include thrombosis or bleeding at the
insertion site and misplacement of the filter. Late complications
include thrombosis of the IVC, DVT, breaking, migration, or
erosion of the filter through the IVC (Fig. 24-4). The rate of fatal
complications is <0.12%. As a result of the increasing number of
reported complications with IVC filters, the US Food and Drug
Administration (FDA) issued a warning in 2010 recommend-
ing removal of IVC filters as soon as they are no longer needed.
This was followed by an update in 2014 where the recommen-
dation was made to remove IVC filters within 29 and 54 days
after implantation based upon a mathematical model that sug-
gested an increased risk-to-benefit ratio at this time point. (See
Schwartz 11th ed., p. 991.)

Brunicardi_Ch24_p197-206.indd 202 08/07/22 11:47 AM


203

CHAPTER 24
A

Venous and Lymphatic Disease


FIG. 24-4. Preoperative computed tomography
imaging and intraoperative photo demonstrating
erosion of IVC filter through the IVC wall. B

11. All of the following are appropriate therapies for suppu- Answer: C
rative thrombophlebitis (SVT) EXCEPT: Treatment of SVT is quite variable. A Cochrane Review
A. Nonsteroidal anti-inflammatory medications. reported that low molecular weight heparin (LMWHs) and
B. Antibiotics. nonsteroidal anti-inflammatory drugs both reduce the rate of
C. Systemic steroid therapy. SVT extension or recurrence. Topical medications appear to
D. Removal of existing indwelling venous catheters. improve local symptoms. Surgical treatment, combined with
the use of graduated compression stockings, is associated with
a lower rate of venous thromboembolism (VTE) and SVT
progression. The treatment is individualized and depends on
the location of the thrombus and the severity of symptoms. In
patients with SVT not within 1 cm of the saphenofemoral junc-
tion, treatment consists of compression and administration
of an anti-inflammatory medication such as indomethacin.
In patients with suppurative SVT, antibiotics and removal of
any existing indwelling catheters are mandatory. Excision of
the vein may be necessary but is usually reserved for patients
with systemic symptoms or when excision of the involved vein
is straightforward. If the SVT extends proximally to within
1 cm of the saphenofemoral junction, extension into the com-
mon femoral vein is more likely to occur. In these patients,
anticoagulation therapy for 6 weeks and GSV ligation appear
equally effective in preventing thrombus extension into the
deep venous system. (See Schwartz 11th ed., p. 994.)

Brunicardi_Ch24_p197-206.indd 203 08/07/22 11:47 AM


204
12. All of the following findings on venous duplex ultraso- Answer: B
nography (DUS) suggest acute venous thromboembo- The examination begins at the ankle and continues proxi-
lism (VTE) EXCEPT: mally to the groin. Each vein is visualized, and the flow sig-
A. Venous distention. nal is assessed with distal and proximal compression. Lower
B. Formation of venous collaterals. extremity deep vein thrombosis (DVT) can be diagnosed by
C. Inability to compress vessel walls. any of the following DUS findings: lack of spontaneous flow
D. Loss of respiratory variation. (Fig. 24-5), inability to compress the vein (Fig. 24-6), absence
E. Lack of spontaneous flow. of color filling of the lumen by color flow DUS, loss of respi-
ratory flow variation, and venous distention. Again, lack of
CHAPTER 24

venous compression on B-mode imaging is the primary diag-


nostic variable. Several studies comparing B-mode ultrasound
to venography for the detection of femoropopliteal DVT in
patients clinically suspected to have DVT report sensitivities
of >91% and specificities of >97%. The ability of DUS to assess
isolated calf vein DVT varies greatly, with sensitivities rang-
ing from 50% to 93% and specificities approaching 100%. (See
Venous and Lymphatic Disease

Schwartz 11th ed., pp. 986–987)

FIG. 24-5. Duplex ultrasound of a femoral vein containing


thrombus demonstrating no flow within the femoral vein.

R FVP R FVP

FIG. 24-6. B-mode ultrasound of the femoral


vein in cross-section. The femoral vein does not No compression Compression
collapse with external compression (arrows).

Brunicardi_Ch24_p197-206.indd 204 08/07/22 11:47 AM


205
13. Heparin-induced thrombocytopenia (HIT) is character- Answer: B
ized by which of the following? HIT results from heparin-associated antiplatelet antibodies
A. Diagnosis based on prior exposure to heparin with (HAAbs) directed against platelet factor 4 complexed with
platelet count <120,000 and/or platelet decline of heparin. HIT occurs in 1% to 5% of patients being treated
40% following heparin exposure with heparin. In patients with repeat heparin exposure (such
B. Results from heparin-associated antiplatelet antibod- as vascular surgery patients), the incidence of HAAbs may be
ies directed against platelet factor 4 complexed with as high as 21%. HIT occurs most frequently in the second
heparin week of therapy and may lead to disastrous venous or arterial
C. Low incidence in patients with repeat exposure to thrombotic complications. Therefore, platelet counts should

CHAPTER 24
heparin be monitored periodically in patients receiving continuous
D. Minimal association with thrombotic complications heparin therapy.
HIT is diagnosed based on previous exposure to heparin,
platelet count < 100,000, and/or platelet count decline of
50% following exposure. All heparin must be stopped and
alternative anticoagulation initiated immediately to avoid
thrombotic complications, which may approach 50% over

Venous and Lymphatic Disease


the subsequent 30 days in affected individuals. (See Schwartz
11th ed., p. 988.)

14. Direct thrombin inhibiting medications include which Answer: C


of the following? Direct thrombin inhibitors (DTIs) include recombinant hiru-
A. Warfarin din, argatroban, and bivalirudin. These antithrombotic agents
B. Enoxaparin bind to thrombin, inhibiting the conversion of fibrinogen to
C. Argatroban fibrin as well as thrombin-induced platelet activation. These
D. Fondaparinux actions are independent of antithrombin. The DTIs should
be reserved for (a) patients in whom there is a high clinical
suspicion or confirmation of HIT, and (b) patients who have a
history of HIT or test positive for heparin-associated antibod-
ies. In patients with established HIT, DTIs should be adminis-
tered for at least 7 days, or until the platelet count normalizes.
Warfarin may then be introduced slowly, overlapping therapy
with a DTI for at least 5 days. (See Schwartz 11th ed., p. 988.)

15. All of the following are components of the treatment of Answer: B


lymphedema EXCEPT: Bed rest and leg elevation. Elevation is an important aspect
A. Extremity compression and elevation. of controlling lower extremity swelling and is often the first
B. Prophylactic antibiotics. recommended intervention. However, continuous elevation
C. Intermittent pneumatic compression therapy. throughout the day can interfere with quality of life more
D. Lymphatic massage. than lymphedema itself. Elevation is an adjunct to lymph-
edema therapy but is not the mainstay of treatment.
Intermittent pneumatic compression therapy. The use of
IPC with a single-chamber or multichamber pump temporar-
ily reduces edema and provides another adjunct to the use of
compression stockings. These devices have been shown to be
effective in reducing limb volume; however, use of compres-
sion stockings is necessary to maintain the volume reduction
when the patient is no longer supine because fluid transport
is not associated with the transport of macromolecules (pro-
teins) from the tissue. Typically, IPC is used for 4 to 6 hours
per day at home when the patient is supine, with pressure
ranges between 30 and 60 mm Hg demonstrated to be most
effective.
Lymphatic massage. Manual lymphatic drainage is a form
of massage developed by Vodder that is directed at reducing
edema. In combination with the use of compression stock-
ings, manual lymphatic drainage is associated with a long-
term reduction in edema and fewer infections per patient per
year.

Brunicardi_Ch24_p197-206.indd 205 08/07/22 11:47 AM


206
Antibiotic therapy. Patients with lymphedema are at
increased risk of developing cellulitis in the affected extrem-
ity due to microscopic breakdown in the skin barrier either
secondary to swelling or unrecognized and untreated tinea
pedis. Recurrent infection can damage the lymphatics, aggra-
vating the edema and increasing the risk for subsequent
infection. Staphylococcus and β-hemolytic Streptococcus are
the most common organisms causing soft tissue infection.
Aggressive antibiotic therapy and elevation with compres-
CHAPTER 24

sion are recommended at the earliest signs or symptoms of


cellulitis. The drug of choice is penicillin or a cephalosporin
active against Streptococcus for 5 days. In patients with recur-
rent cellulitis despite methods to reduced edema, treatment
with monthly intramuscular injections of benzathine penicil-
lin 1.2 MU, twice-daily erythromycin 250 mg, or penicillin V
1 g daily has proven effective at suppression. (See Schwartz
Venous and Lymphatic Disease

11th ed., pp. 1001–1003.)

16. Mesenteric vein thrombosis (MVT) is associated with all Answer: A


of the following EXCEPT: Five percent to 15% of cases of acute mesenteric ischemia
A. MVT is less common in patients with hypercoagu- occur as a result of MVT. Mortality rates in patients with
lable states, malignancy, or cirrhosis. MVT may approach 50%. The usual presenting symptom
B. 5% to 15% of cases of acute mesenteric ischemia is nonspecific abdominal pain and distention, often accom-
occur as a result of MVT. panied by nausea, vomiting, and diarrhea. Peritoneal signs,
C. Patients with MVT are treated with fluid resuscita- suggesting intestinal infarction, are present in fewer than half
tion, heparin anticoagulation, and bowel rest. of MVT patients. MVT is more common in patients with
D. Computed tomography (CT) scan and magnetic res- a hypercoagulable state, malignancy, and cirrhosis. MVT
onance imaging (MRI) are 100% sensitive and 98% occurs as a rare complication of laparoscopic surgery.
specific for MVT. Most cases of MVT are diagnosed with contrast-enhanced
CT scanning or MRI in the course of an evaluation for
abdominal pain. The sensitivity and specificity for CT and
MRI approach 100% and 98%, respectively. Ultrasound can
also be used and has reported sensitivity and specificity of
93% and 99%, respectively.
Patients with MVT are treated with fluid resuscitation,
heparin anticoagulation, and bowel rest. (See Schwartz
11th ed., p. 996.)

Brunicardi_Ch24_p197-206.indd 206 08/07/22 11:47 AM


CHAPTER 25
The Esophagus and Diaphragmatic Hernia

1. Locations of anatomic narrowing of the esophagus seen Answer: C


on an esophagram include all of the following EXCEPT: Three normal areas of esophageal narrowing are evident on
A. Lower esophageal sphincter. the barium esophagogram or during esophagoscopy. The
B. Crossing of the left mainstem bronchus and aortic uppermost narrowing is located at the entrance into the
arch. esophagus and is caused by the cricopharyngeal muscle. Its
C. Thoracic outlet. luminal diameter is 1.5 cm, and it is the narrowest point of the
D. Cricopharyngeal muscle. esophagus. The middle narrowing is due to an indentation
of the anterior and left lateral esophageal wall caused by the
crossing of the left main stem bronchus and aortic arch. The
luminal diameter at this point is 1.6 cm. The lowermost nar-
rowing is at the hiatus of the diaphragm and is caused by the
gastroesophageal sphincter mechanism. The luminal diam-
eter at this point varies somewhat, depending on the disten-
tion of the esophagus by the passage of food, but has been
measured at 1.6 to 1.9 cm. These normal constrictions tend to
hold up swallowed foreign objects, and the overlying mucosa
is subject to injury by swallowed corrosive liquids due to their
slow passage through these areas. (See Schwartz 11th ed.,
p. 1010.)

2. The cervical esophagus receives its blood supply primar- Answer: B


ily from the: The cervical portion of the esophagus receives its main blood
A. Internal carotid artery. supply from the inferior thyroid artery. The thoracic por-
B. Inferior thyroid artery. tion receives its blood supply from the bronchial arteries,
C. Superior thyroid artery. with 75% of individuals having one right-sided and two left-
D. Inferior cervical artery. sided branches. Two esophageal branches arise directly from
E. Facial artery. the aorta. The abdominal portion of the esophagus receives
its blood supply from the ascending branch of the left gas-
tric artery and from inferior phrenic arteries (Fig. 25-1).
On entering the wall of the esophagus, the arteries assume a
T-shaped division to form a longitudinal plexus, giving rise to
an intramural vascular network in the muscular and submu-
cosal layers. As a consequence, the esophagus can be mobi-
lized from the stomach to the level of the aortic arch without
fear of devascularization and ischemic necrosis. Caution
should be exercised as to the extent of esophageal mobiliza-
tion in patients who have had a previous thyroidectomy with
ligation of the inferior thyroid arteries proximal to the origin
of the esophageal branches. (See Schwartz 11th ed., p. 1013.)

207

Brunicardi_Ch25_p207-222.indd 207 04/07/22 2:39 PM


208

Esophageal branch
Inferior thyroid
artery

Superior left
Right bronchial bronchial artery
artery Inferior left
CHAPTER 25

bronchial artery

Aortic esophageal
arteries
The Esophagus and Diaphragmatic Hernia

FIG. 25-1. Arterial blood supply of the esophagus. Ascending branches of


(Reproduced with permission from Shields TW: General left gastric artery
Thoracic Surgery, 3rd ed. Philadelphia, PA: Lea & Left gastric artery
Febiger; 1989.)

3. All of the following hormones decrease lower esophageal Answer: A


sphincter (LES) tone EXCEPT: The LES has intrinsic myogenic tone, which is modulated by
A. Gastrin. neural and hormonal mechanisms. Alpha-adrenergic neu-
B. Estrogen. rotransmitters or beta blockers stimulate the LES, and alpha
C. Somatostatin. blockers and beta stimulants decrease its pressure. It is not
D. Cholecystokinin (CCK). clear to what extent cholinergic nerve activity controls LES
E. Glucagon. pressure. The vagus nerve carries both excitatory and inhibi-
tory fibers to the esophagus and sphincter. The hormones
gastrin and motilin have been shown to increase LES pres-
sure; and cholecystokinin, estrogen, glucagon, progesterone,
somatostatin, and secretin decrease LES pressure. The pep-
tides bombesin, l-enkephalin, and substance P increase LES
pressure; and calcitonin gene-related peptide, gastric inhibi-
tory peptide, neuropeptide Y, and vasoactive intestinal poly-
peptide decrease LES pressure. Some pharmacologic agents,
such as antacids, cholinergics, agonists, domperidone, meto-
clopramide, and prostaglandin F2, are known to increase LES
pressure; and anticholinergics, barbiturates, calcium channel
blockers, caffeine, diazepam, dopamine, meperidine, prosta-
glandin E1 and E2, and theophylline decrease LES pressure.
Peppermint, chocolate, coffee, ethanol, and fat are all associ-
ated with decreased LES pressure and may be responsible for
esophageal symptoms after a sumptuous meal. (See Schwartz
11th ed., pp. 1015–1016.)

4. The incidence of metaplastic Barrett esophagus (BE) Answer: B


progressing to adenocarcinoma is: If reflux of gastric juice is allowed to persist and sustained or
A. <0.1% per year. repetitive esophageal injury occurs, two sequelae can result.
B. 0.2% to 0.5% per year. First, a luminal stricture can develop from submucosal and
C. 1% to 3% per year. eventually intramural fibrosis. Second, the tubular esophagus
D. 3% to 5% per year. may become replaced with columnar epithelium. The colum-
E. >5% per year. nar epithelium is resistant to acid and is associated with the
alleviation of the complaint of heartburn. This columnar
epithelium often becomes intestinalized, identified histologi-
cally by the presence of goblet cells. This specialized intestinal

Brunicardi_Ch25_p207-222.indd 208 04/07/22 2:39 PM


209
metaplasia (IM) is currently required for the diagnosis of
BE. Endoscopically, BE can be quiescent or associated with
complications of esophagitis, stricture, Barrett ulceration,
and dysplasia. The complications associated with BE may be
due to the continuous irritation from refluxed duodenogas-
tric juice. This continued injury is pH-dependent and may
be modified by medical therapy. The incidence of metaplas-
tic Barrett epithelium becoming dysplastic and progressing
to adenocarcinoma is approximately 0.2% to 0.5% per year.

CHAPTER 25
(See Schwartz 11th ed., p. 1035.)

5. The histologic hallmark of Barrett esophagus (BE) is: Answer: B


A. Columnar epithelium. The definition of BE has evolved considerably over the past
B. Goblet cells. decade. Traditionally, BE was identified by the presence of
C. Parietal cells. columnar mucosa extending at least 3 cm into the esopha-
D. Cuboidal epithelium. gus. It is now recognized that the specialized, intestinal-type

The Esophagus and Diaphragmatic Hernia


epithelium found in the Barrett mucosa is the only tissue
predisposed to malignant degeneration. Consequently, the
diagnosis of BE is presently made given any length of endo-
scopically identifiable columnar mucosa that proves, on
biopsy, to show intestinal metaplasia (IM). Although long
segments of columnar mucosa without IM do occur, they are
uncommon and might be congenital in origin.
The hallmark of IM is the presence of intestinal goblet
cells. There is a high prevalence of biopsy-demonstrated IM
at the cardia, on the gastric side of the squamocolumnar junc-
tion, in the absence of endoscopic evidence of a columnar-
lined esophagus (CLE). Evidence is accumulating that these
patches of what appears to be Barrett in the cardia have a
similar malignant potential as in the longer segments, and
are precursors for carcinoma of the cardia. (See Schwartz
11th ed., p. 1035.)

6. Squamous cell carcinomas of the esophagus most com- Answer: B


monly occur: It is estimated that 8% of the primary malignant tumors of
A. At the gastroesophageal junction. the esophagus occur in the cervical portion. They are almost
B. In the cervical and upper thoracic esophagus. always squamous cell cancer, with a rare adenocarcinoma
C. In the lower thoracic esophagus. arising from a congenital inlet patch of columnar lining.
D. Evenly distributed throughout the esophagus. These tumors, particularly those in the postcricoid area, rep-
resent a separate pathologic entity for two reasons: (a) They
are more common in females and appear to be a unique entity
in this regard; and (b) The efferent lymphatics from the cervi-
cal esophagus drain completely differently from those of the
thoracic esophagus. The latter drain directly into the para-
tracheal and deep cervical or internal jugular lymph nodes
(LNs) with minimal flow in a longitudinal direction. Except
in advanced disease, it is unusual for intrathoracic LNs to be
involved. (See Schwartz 11th ed., p. 1685.)

7. The preoperative test most heavily correlated with the Answer: B


ability to tolerate an esophagectomy is: Patients undergoing esophageal resection should have suf-
A. DLCO. ficient cardiopulmonary reserve to tolerate the proposed
B. FEV1. procedure. The respiratory function is best assessed with the
C. Ability to climb 1 flight of stairs. forced expiratory volume in 1 second, which ideally should
D. FVC. be 2 L or more. Any patient with a forced expiratory volume
in 1 second of <1.25 L is a poor candidate for thoracotomy,
because he or she has a 40% risk of dying from respiratory
insufficiency within 4 years. In patients with poor pulmonary
reserve, the transhiatal esophagectomy should be considered,

Brunicardi_Ch25_p207-222.indd 209 04/07/22 2:39 PM


210
as the pulmonary morbidity of this operation is less than is
seen following thoracotomy. Clinical evaluation and electro-
cardiogram are not sufficient indicators of cardiac reserve.
Echocardiography and dipyridamole thallium imaging pro-
vide accurate information on wall motion, ejection fraction,
and myocardial blood flow. A defect on thallium imaging
may require further evaluation with preoperative coronary
angiography. A resting ejection fraction of <40%, particularly
if there is no increase with exercise, is an ominous sign. In
CHAPTER 25

the absence of invasive testing, observed stair-climbing is an


economical (albeit not quantitative) method of assessing car-
diopulmonary reserve. Most individuals who can climb three
flights of stairs without stopping will do well with two-field
open esophagectomy, especially if an epidural catheter is used
for postoperative pain relief. (See Schwartz 11th ed., p. 1073.)
The Esophagus and Diaphragmatic Hernia

8. Which of the following tests most accurately assesses the Answer: D


T stage of esophageal cancer? For years, clinical staging, contrast radiography, endoscopy,
A. High-resolution CT scan and computed tomography (CT) scanning formed the back-
B. Magnetic resonance imaging (MRI) bone of esophageal cancer staging. More recently, preoperative
C. Echocardiography decision-making is guided by endoscopic ultrasonography
D. Endoscopic ultrasound (EUS) and positron emission tomography (PET) scanning.
E. Esophagogastroduodenoscopy EUS provides the most reliable method of determining
depth of cancer invasion. In the absence of enlarged lymph
nodes (LNs), the degree of wall invasion dictates surgical
therapy. (See Schwartz 11th ed., p. 1073.)

9. Which of the following patients would not be considered Answer: B


a candidate for esophagectomy? If the tumor invades in to the submucosa, without visible
A. A 55-year-old man with gastroesophageal junction LN involvement, most individuals would suggest esophagec-
(GEJ) adenocarcinoma confined to the muscularis tomy with LN dissection, as positive nodes can be found in
mucosa 20% to 25% of those with cancer limited to the mucosa and
B. A 47-year-old woman with mid-esophageal cancer submucosa. If EUS demonstrates spread through the wall of
and an involved cervical lymph node (LN) the esophagus, especially if LNs are enlarged, then induc-
C. A 60-year-old man with a large GEJ carcinoma with tion chemoradiation therapy (neoadjuvant therapy) should
invasion into the pleura without a malignant effusion be strongly considered. Lastly, when the EUS demonstrates
D. A 70-year-old woman with a small GEJ cancer and ­invasion of the trachea, bronchus, aorta, or spine, then s­ urgical
three pathologic LNs nearby on EUS resection is rarely indicated. If there is invasion into the pleura
(T4a), then surgical resection can be considered in the absence
of a malignant effusion. Thus, it can be seen that the therapy
of esophageal cancer is largely driven by the f­indings of an
endoscopic ultrasonography. It is difficult to provide modern
treatment of esophageal cancer without access to this modal-
ity. (See Schwartz 11th ed., pp. 1070–1072.)

10. Patients with dysphagia secondary to esophageal cancer Answer: B


treated with radiation can expect the benefit to last: Primary treatment with radiation therapy does not produce
A. <1 month. results comparable with those obtained with surgery. Cur-
B. 2–3 months. rently, the use of radiotherapy is restricted to patients who are
C. 6–12 months. not candidates for surgery, and is usually combined with che-
D. >12 months. motherapy. Radiation alone is used for palliation of dysphagia
but the benefit is short-lived, lasting only 2 to 3 months. Fur-
thermore, the length and course of treatment are difficult to
justify in patients with a limited life expectancy. Radiation is
effective in patients who have hemorrhage from the primary
tumor. (See Schwartz 11th ed., p. 1074.)

Brunicardi_Ch25_p207-222.indd 210 04/07/22 2:39 PM


211
11. The optimal treatment of an incidentally discovered Answer: C
3 cm leiomyoma of the upper esophagus in a 45-year-old Despite their slow growth and limited potential for malignant
otherwise healthy man is: degeneration, leiomyomas should be removed unless there
A. Observation. are specific contraindications. The majority can be removed
B. Esophagectomy. by simple enucleation. If, during removal, the mucosa is
C. Enucleation. inadvertently entered, the defect can be repaired primarily.
D. Endoscopic resection. After tumor removal, the outer esophageal wall should be
reconstructed by closure of the muscle layer. The location
of the lesion and the extent of surgery required will dictate

CHAPTER 25
the approach. Lesions of the proximal and middle esopha-
gus require a right thoracotomy, whereas distal esophageal
lesions require a left thoracotomy. Videothoracoscopic and
laparoscopic approaches are now frequently used. The mor-
tality rate associated with enucleation is low, and success in
relieving the dysphagia is near 100%. Large lesions or those
involving the gastroesophageal junction (GEJ) may require

The Esophagus and Diaphragmatic Hernia


esophageal resection. (See Schwartz 11th ed., p. 1081.)

12. Following a night of heavy drinking, a 43-year-old oth- Answer: C


erwise healthy man has sudden onset of severe chest The key to optimum management is early diagnosis. The most
pain after vomiting. Esophagram confirms esophageal favorable outcome is obtained following primary c­ losure of
rupture just proximal to the GEJ. What is the preferred the perforation within 24 hours, resulting in 80% to 90% sur-
operative exposure? vival. The most common location for the injury is the left lat-
A. Right thoracotomy eral wall of the esophagus, just above the GEJ. To get adequate
B. Right thoracotomy with laparotomy exposure of the injury, a dissection similar to that described
C. Left thoracotomy for esophageal myotomy is performed. A flap of stomach is
D. Left thoracotomy with laparotomy pulled up and the soiled fat pad at the GEJ is removed. The
E. Midline laparotomy edges of the injury are trimmed and closed primarily. The
closure is reinforced with the use of a pleural patch or con-
struction of a Nissen fundoplication. (See Schwartz 11th ed.,
pp. 1084–1085.)

13. A 34-year-old man presents to the emergency Answer: B


department (ED) after an episode of hematemesis. Mallory-Weiss tears are characterized by arterial bleeding,
Esophagogastroduodenoscopy (EGD) confirms a which may be massive. Vomiting is not an obligatory factor,
­Mallory-Weiss tear with no residual bleeding. Treatment as there may be other causes of an acute increase in intra-
should consist of: abdominal pressure, such as paroxysmal coughing, seizures,
A. Esophagectomy. and retching. The diagnosis requires a high index of suspi-
B. Observation. cion, particularly in the patient who develops upper gastro-
C. Proximal gastrectomy with esophago-jejunostomy. intestinal (GI) bleeding following prolonged vomiting or
D. Injection of botulinum toxin. retching. Upper endoscopy confirms the suspicion by identi-
fying one or more longitudinal fissures in the mucosa of the
herniated stomach as the source of bleeding.
In the majority of patients, the bleeding will stop spon-
taneously with nonoperative management. In addition to
blood replacement, the stomach should be decompressed and
antiemetics administered, as a distended stomach and con-
tinued vomiting aggravate further bleeding. A Sengstaken-
Blakemore tube will not stop the bleeding, as the pressure
in the balloon is not sufficient to overcome arterial pressure.
Endoscopic injection of epinephrine may be therapeutic if
bleeding does not stop spontaneously. Only occasionally will
surgery be required to stop blood loss. The procedure consists
of laparotomy and high gastrotomy with oversewing of the
linear tear. Mortality is uncommon, and recurrence is rare.
(See Schwartz 11th ed., p. 1085.)

Brunicardi_Ch25_p207-222.indd 211 04/07/22 2:39 PM


212
14. Which of the following disorders involves simultaneous Answer: B
nonperistaltic contractions of the esophagus? The classic manometric findings in these patients are char-
A. Achalasia acterized by the frequent occurrence of simultaneous wave-
B. Diffuse esophageal spasm (DES) forms and multipeaked esophageal contractions, which may
C. Hypertensive lower esophageal sphincter be of abnormally high amplitude or long duration. Key to the
D. Nutcracker esophagus diagnosis of DES is that there remain some peristaltic wave-
forms in excess of those seen in achalasia. A criterion of ≥30%
peristaltic waveforms out of 10 wet swallows has been used
to differentiate DES from vigorous achalasia. However, this
CHAPTER 25

figure is arbitrary and often debated.


The lower esophageal sphincter (LES) in patients with DES
usually shows a normal resting pressure and relaxation on
swallowing. A hypertensive sphincter with poor relaxation
may also be present. In patients with advanced disease, the
radiographic appearance of tertiary contractions appears
helical, and has been termed corkscrew esophagus or pseudo-
The Esophagus and Diaphragmatic Hernia

diverticulosis. Patients with segmental or diffuse esophageal


spasm can compartmentalize the esophagus and develop an
epiphrenic or midesophageal diverticulum between two areas
of high pressure occurring simultaneously. (See Schwartz
11th ed., pp. 1056–1057 and Table 25-9.)

15. Damage to the recurrent laryngeal nerves could result in Answer: E


the following EXCEPT: The parasympathetic innervation of the pharynx and esopha-
A. Function of vocal cords. gus is provided mainly by the vagus nerves. The constrictor
B. Function of cricopharyngeal sphincter. muscles of the pharynx receive branches from the pharyngeal
C. Motility to cervical esophagus. plexus, which is on the posterior lateral surface of the middle
D. Increase risk of aspiration. constrictor muscle, and is formed by pharyngeal branches of
E. Innervation of stylopharyngeus muscle. the vagus nerves with a small contribution from cranial nerves
IX and XI (Fig. 25-2). The cricopharyngeal sphincter and the
cervical portion of the esophagus receive branches from both
recurrent laryngeal nerves, which originate from the vagus
nerves—the right recurrent nerve at the lower margin of the
subclavian artery and the left at the lower margin of the aortic

Recurrent
laryngeal
nerves
Right vagus nerve

Left vagus nerve

Right recurrent
laryngeal nerve
Left recurrent
laryngeal nerve

Anterior esophageal
Thoracic chain
plexus

Left or anterior
vagal trunk

FIG. 25-2. Innervation of the esophagus.


(Reproduced with permission from Shields TW. Right or posterior
General Thoracic Surgery, 3rd ed. Philadelphia, PA: vagal trunk
Lea & Febiger; 1989.)

Brunicardi_Ch25_p207-222.indd 212 04/07/22 2:39 PM


213
arch. They are slung dorsally around these vessels and ascend
in the groove between the esophagus and trachea, giving
branches to each. Damage to these nerves interferes not only
with the function of the vocal cords but also with the func-
tion of the cricopharyngeal sphincter and the motility of the
cervical esophagus, predisposing the individual to pulmonary
aspiration on swallowing. (See Schwartz 11th ed., p. 1014.)

16. Primary tumors in the thoracic esophagus tend to Answer: A

CHAPTER 25
metastasize: The lymphatics located in the submucosa of the esophagus are
A. In the submucosal lymph plexus. so dense and interconnected that they constitute a single plexus
B. Into regional nodes. (Fig. 25-3). There are more lymph vessels than blood capillar-
C. In a transverse direction. ies in the submucosa. Lymph flow in the submucosal plexus
D. Directly into the muscle layer. runs in a longitudinal direction, and, on injection of a contrast
E. In a hematogenous fashion. medium, the longitudinal spread is seen to be about six times
that of the transverse spread. In the upper two-thirds of the

The Esophagus and Diaphragmatic Hernia


esophagus, the lymphatic flow is mostly cephalad, and, in the
lower third, caudad. In the thoracic portion of the esophagus,
the submucosal lymph plexus extends over a long distance in
a longitudinal direction before penetrating the muscle layer to
enter lymph vessels in the adventitia. As a consequence of this
nonsegmental lymph drainage, a primary tumor can extend
for a considerable length superiorly or inferiorly in the sub-
mucosal plexus. Consequently, free tumor cells can follow the
submucosal lymphatic plexus in either direction for a long
distance before they pass through the muscularis and into the
regional lymph nodes (LNs). The cervical esophagus has more
direct segmental lymph drainage into the regional nodes, and,
as a result, lesions in this portion of the esophagus have less
submucosal extension and a more regionalized lymphatic
spread. (See Schwartz 11th ed., pp. 1014–1015.)

Superior Internal jugular


paraesophageal nodes nodes

Paratracheal
nodes
Pulmonary hilar
nodes Subcarinal nodes

Inferior paraesophageal
nodes
Parahiatal nodes

Splenic artery
nodes
Left gastric artery nodes
Hepatic artery
nodes

Celiac artery nodes

FIG. 25-3. Lymphatic drainage of the esophagus.

Brunicardi_Ch25_p207-222.indd 213 04/07/22 2:39 PM


214
17. The antireflux mechanism is composed of the following Answer: D
EXCEPT: If the pharyngeal swallow does not initiate a peristaltic contrac-
A. A mechanically effective lower esophageal sphincter tion, then the coincident relaxation of the LES is unguarded and
(LES). reflux of gastric juice can occur. This may be an explanation for
B. Efficient esophageal clearance. the observation of spontaneous lower esophageal relaxation,
C. An adequately functioning gastric reservoir. thought by some to be a causative factor in gastroesophageal
D. Secondary peristalsis. reflux disease (GERD). The power of the worm-drive pump
E. Intact vagal function. of the esophageal body is insufficient to force open a valve that
does not relax. In dogs, a bilateral cervical parasympathetic
CHAPTER 25

blockade abolishes the relaxation of the LES that occurs with


pharyngeal swallowing or distention of the esophagus. Conse-
quently, vagal function appears to be important in coordinat-
ing the relaxation of the LES with esophageal contraction.
The antireflux mechanism in human beings is composed
of three components: A mechanically effective LES, efficient
esophageal clearance, and an adequately functioning gastric
The Esophagus and Diaphragmatic Hernia

reservoir. A defect of any one of these three components can


lead to increased esophageal exposure to gastric juice and
the development of mucosal injury. (See Schwartz 11th ed.,
pp. 1016–1017, 1032.)

18. On upper endoscopy, a patient is found to have erosions Answer: B


in the esophagus limited to the mucosal folds that are When endoscopic esophagitis is seen, severity and the length of
>5 mm in longitudinal extent. What classification does esophagitis involved are recorded. While many different grad-
he fall under according to the Los Angeles (LA) grading ing systems have been proposed, the commonest system now
system? in use is the LA grading system. In this system, mild esophagi-
A. Grade A tis is classified LA grade A or B—one or more erosions limited
B. Grade B to the mucosal fold(s) and either less than or greater than 5 mm
C. Grade C in longitudinal extent, respectively (Fig. 25-4). More severe
D. Grade D esophagitis is classified LA grade C or D. In grade C, erosions
E. Grade E extend over the mucosal folds but over less than three-quarters
of the esophageal circumference; in grade D, confluent ero-
sions extend across more than three quarters of the esophageal
circumference. (See Schwartz 11th ed., p. 1018.)

A   B
FIG. 25-4. Complications of reflux disease as seen on endoscopy. A. Linear erosions of LA grade B esophagitis. B. Uncomplicated Barrett
mucosa. C. High-grade dysplasia in Barrett mucosa. D. Early adenocarcinoma arising in Barrett mucosa. (Continued)

Brunicardi_Ch25_p207-222.indd 214 04/07/22 2:39 PM


215

CHAPTER 25
The Esophagus and Diaphragmatic Hernia
C   D
FIG. 25-4. (Continued)

19. The most direct method of measuring gastric juice expo- Answer: C
sure in the esophagus is: The most direct method of measuring increased esophageal
A. Esophageal motility study. exposure to gastric juice is by an indwelling pH electrode,
B. High resolution manometry. or, more recently, via a radiotelemetric pH monitoring cap-
C. 24-hour ambulatory pH monitoring. sule that can be clipped to the esophageal mucosa. The lat-
D. Esophageal impedance. ter consists of an antimony pH electrode fitted inside a small,
E. Esophageal transit scintigraphy. capsule-shaped device accompanied by a battery and elec-
tronics that allow 48-hour monitoring and transmission of
the pH data via transcutaneous radio telemetry to a waist-
mounted data logger. The device can be introduced either
transorally or transnasally, and it can be clipped to the
esophageal mucosa using endoscopic fastening techniques. It
passes spontaneously within 1 to 2 weeks. Prolonged moni-
toring of esophageal pH is performed by placing the pH
probe or telemetry capsule 5 cm above the manometrically
measured upper border of the distal sphincter for 24 hours.
It measures the actual time the esophageal mucosa is exposed
to gastric juice, measures the ability of the esophagus to clear
refluxed acid, and correlates esophageal acid exposure with
the patient’s symptoms. A 24- to 48-hour period is necessary
so that measurements can be made over one or two complete
circadian cycles. This allows measuring the effect of physi-
ologic activity, such as eating or sleeping, on the reflux of gas-
tric juice into the esophagus (Fig. 25-5). (See Scwartz 11th ed.,
p. 1029–1030.)

Brunicardi_Ch25_p207-222.indd 215 04/07/22 2:39 PM


216
pH mp mp
8
6
4
2

14:00 16:00 18:00 20:00 22:00

pH sp
8
CHAPTER 25

6
4
2

22:00 00:00 02:00 04:00 06:00


FIG. 25-5. Strip chart display of a 24-hour
pH mp
esophageal pH monitoring study in a patient 8
The Esophagus and Diaphragmatic Hernia

with increased esophageal acid exposure.


mp = meal period; sp = supine period. 6
(Reproduced with permission from Zuidema 4
GD, Orringer MB. Shackelford’s Surgery of the 2
Alimentary Tract, 3rd ed. Vol 1. Philadelphia, PA:
Elsevier/ Saunders; 1991.) 06:00 08:00 10:00 12:00 14:00

20. What is the correct order in the stepwise approach in Answer: A


treatment for gastroesophageal reflux disease (GERD)? Traditionally, a stepwise approach is used for the treatment
A. PPI->endoscopy->24-hour pH study->surgery of GERD. First-line therapy entails antisecretory medication,
B. Endoscopy->PPI->24-hour pH study->surgery usually proton pump inhibitors (PPIs), in most patients. Fail-
C. PPI->24-hour pH study->endoscopy->surgery ure of medication to adequately control GERD symptoms sug-
D. 24-hour pH study->endoscopy->PPI->surgery gests either that the patient may have relatively severe disease
or a non-GERD cause for his or her symptoms. Endoscopic
examination at this stage of the patient’s evaluation is recom-
mended and will provide the opportunity to assess the degree
of mucosal injury and presence of Barrett esophagus (BE).
Treatment options for these patients entail either long-term
PPI use versus antireflux surgery. Laparoscopic antireflux
surgery in these patients achieves long-term control of symp-
toms in 85% to 90%. The measurement of esophageal acid
exposure via 24-hour pH should be undertaken when patients
are considered for surgery. The status of the lower esophageal
sphincter (LES) and esophageal body function with esopha-
geal manometry should also be performed at this stage. These
studies will serve to establish the diagnosis and assess esopha-
geal body dysfunction. (See Schwartz 11th ed., p. 1037.)

21. An antireflux surgery should ideally do all of the follow- Answer: C


ing EXCEPT: The primary goal of antireflux surgery is to safely create a
A. Create a flap valve to prevent regurgitation into the new antireflux valve at the gastroesophageal junction, while
esophagus. preserving the patient’s ability to swallow normally and to
B. Double the resting gastric pressure. belch to relieve gaseous distention. Regardless of the choice
C. Reconstruct the valve to a length <3 cm. of the procedure, this goal can be achieved if attention is
D. Create a loose wrap. paid to some basic principles when reconstructing the anti-
E. Maintain the position of the gastric fundus close to reflux mechanism. First, the operation should create a flap
the distal esophagus. valve which prevents regurgitation of gastric contents into
the esophagus. This will result in an increase in the pressure
of the distal esophageal sphincter region. Following a Nissen
fundoplication the expected increase is to a level twice the
resting gastric pressure (ie, 12 mm Hg for a gastric pressure

Brunicardi_Ch25_p207-222.indd 216 04/07/22 2:39 PM


217
of 6 mm Hg). The extent of the pressure rise is often less fol-
lowing a partial fundoplication, although with all types of
fundoplication the length of the reconstructed valve should
be at least 3 cm. This not only augments sphincter charac-
teristics in patients in whom they are reduced before surgery
but also prevents unfolding of a normal sphincter in response
to gastric distention (Fig. 25-6). Preoperative and postop-
erative esophageal manometry measurements have shown
that the resting sphincter pressure and the overall sphincter

CHAPTER 25
length can be surgically augmented over preoperative values,
and that the change in the former is a function of the degree
of gastric wrap around the esophagus (Fig. 25-7). However,
the aim of any fundoplication is to create a loose wrap and to
maintain the position of the gastric fundus close to the distal
intra-abdominal esophagus, in a flap valve arrangement. The
efficacy of this relies on the close relationship between the

The Esophagus and Diaphragmatic Hernia


fundus and the esophagus, not the “tightness” of the wrap.
(See Schwartz 11th ed., p. 1039.)

Distention

FIG. 25-6. A graphic illustration of the shortening of the lower


esophageal sphincter that occurs as the sphincter is “taken up” by
the cardia as the stomach distends.

Hill Belsey Nissen

20 N=15 N=15 N=15


∆ P mm Hg

15

10

5 Y = 4.63 + .023 (x)


P < .01

0 240 360
Degree of wrap
FIG. 25-7. The relationship between the augmentation of
sphincter pressure over preoperative pressure (ΔP) and the degree
of gastric fundic wrap in three different antireflux procedures.
(Reproduced with permission from O’Sullivan GC, DeMeester
TR, Joelsson BE, et al. Interaction of lower esophageal sphincter
pressure and length of sphincter in the abdomen as determinants of
gastroesophageal competence, Am J Surg. 1982;143(1):40–47.)

Brunicardi_Ch25_p207-222.indd 217 04/07/22 2:39 PM


218
22. What is the characterization of a rolling or type II para- Answer: C
esophageal hernia? With the advent of clinical radiology, it became evident that
A. Upward dislocation of the colon in the posterior a diaphragmatic hernia was a relatively common abnormality
mediastinum and was not always accompanied by symptoms. Three types of
B. Upward dislocation of both the cardia and gastric esophageal hiatal hernia were identified: (a) the sliding hernia,
fundus in the posterior mediastinum type I, characterized by an upward dislocation of the cardia in
C. Upward dislocation of the gastric fundus in the pos- the posterior mediastinum (Fig. 25-8A); (b) the rolling or PEH,
terior mediastinum type II, characterized by an upward dislocation of the gastric
D. Upward dislocation of the cardia in the posterior fundus alongside a normally positioned cardia (Fig. 25-8B);
CHAPTER 25

mediastinum and (c) the combined sliding-rolling or mixed hernia, type III,
characterized by an upward dislocation of both the cardia and
the gastric fundus (Fig. 25-8C). The end stage of type I and
type II hernias occurs when the whole stomach migrates up
into the chest by rotating 180° around its longitudinal axis,
with the cardia and pylorus as fixed points. In this situation,
the abnormality is usually referred to as an intrathoracic stom-
The Esophagus and Diaphragmatic Hernia

ach (Fig. 25-8D). In some taxonomies, a type IV hiatal hernia


is declared when an additional organ, usually the colon, herni-
ates as well. (See Schwartz 11th ed., pp. 1046–1047.)

A B

FIG. 25-8. A. Radiogram of a type I (sliding) hiatal hernia. B. Radiogram of a type II (rolling or paraesophageal) hernia. C. Radiogram of a
type III (combined sliding-rolling or mixed) hernia. D. Radiogram of an intrathoracic stomach. This is the end stage of a large hiatal hernia
regardless of its initial classification. Note that the stomach has rotated 180° around its longitudinal axis, with the cardia and pylorus as fixed
points. (Reproduced with permission from Nyhus LM, Condon RE. Hernia, 3rd ed. Philadelphia, PA: Lippincott Williams & Wilkins; 1989.) (Continued)

Brunicardi_Ch25_p207-222.indd 218 04/07/22 2:39 PM


219

CHAPTER 25
The Esophagus and Diaphragmatic Hernia
C

FIG. 25-8. (Continued)

23. The best treatment for a patient with a symptomatic Answer: C


Schatzki ring without complaints of reflux is: Symptoms in patients with a ring are caused more by the
A. Antireflux surgery. presence of the ring than by gastroesophageal reflux. Most
B. Excision of the ring. patients with a ring but without proven reflux respond to
C. Dilation. one dilation, while most patients with proven reflux require
D. Ablation. repeated dilations. In this regard, the majority of Schatzki
E. Proton pump inhibitor (PPI). ring patients without proven reflux have a history of ingestion
of drugs known to be damaging to the esophageal mucosa.
Bonavina and associates have suggested drug-induced injury
as the cause of stenosis in patients with a ring, but without
a history of reflux. Because rings also occur in patients with
proven reflux, it is likely that gastroesophageal reflux also
plays a part. This is supported by the fact that there is less
drug ingestion in the history of these patients. Schatzki ring
is probably an acquired lesion that can lead to stenosis from
chemical-induced injury by pill lodgment in the distal esoph-
agus, or from reflux-induced injury to the lower esophageal
mucosa.
The best form of treatment of a symptomatic Schatzki ring
in patients who do not have reflux consists of esophageal dila-
tion for relief of the obstructive symptoms. In patients with
a ring who have proven reflux and a mechanically defective
sphincter, an antireflux procedure is necessary to obtain relief
and avoid repeated dilation. (See Schwartz 11th ed., p. 1049.)

Brunicardi_Ch25_p207-222.indd 219 04/07/22 2:39 PM


220
24. A characteristic finding on Barium swallow for eosino- Answer: A
philic esophagitis (EE) is: A barium swallow should be the first test obtained in the
A. Feline esophagus. patient with dysphagia. EE has a characteristic finding often
B. Apple core esophagus. called the “ringed esophagus” or the “feline esophagus,” as the
C. Bird’s beak esophagus. esophageal rings are felt to look like the stripes on a housecat
D. Rat’s tail esophagus. (Fig. 25-9). The endoscopic appearance of EE is also char-
acteristic, and also appears as a series of rings (Fig. 25-10).
(See Schwartz 11th ed., p. 1051.)
CHAPTER 25
The Esophagus and Diaphragmatic Hernia

FIG. 25-9. The esophagus on the left shows a stacking


of rings, demonstrating eosinophilic esophagus. The
esophagus on the right is a normal barium swallow.

FIG. 25-10. The endoscopic appearance of


eosinophilic esophagitis is characteristically a
series of stacked mucosal rings.

25. Patients who undergo diverticulopexy, instead of diver- Answer: A


ticulectomy, for treatment of Zenker’s diverticulum have If the diverticulum is excessively large so that it would be
a lower risk of: redundant if suspended, or if its walls are thickened, a diver-
A. Fistula formation. ticulectomy should be performed. This is best performed
B. Hematoma. under general anesthesia by placing a Maloney dilator (48F)
C. Recurrent nerve paralysis. in the esophagus, after controlling the neck of the diverticu-
D. Difficulties in phonation. lum and after myotomy. A linear stapler is placed across the
E. Horner’s syndrome. neck of the diverticulum, and the diverticulum is excised
distal to the staple line. The security of this staple line and
effectiveness of the myotomy may be tested before hospital
discharge with a water-soluble contrast esophagogram. Post-
operative complications include fistula formation, abscess,
hematoma, recurrent nerve paralysis, difficulties in phona-
tion, and Horner syndrome. The incidence of the first two
can be reduced by performing a diverticulopexy rather than
diverticulectomy. (See Schwartz 11th ed., pp. 1053–1054.)

Brunicardi_Ch25_p207-222.indd 220 04/07/22 2:39 PM


221
26. Patients who present with achalasia should NOT be Answer: D
offered which of the following treatment options? The third issue—and one that has been long debated—is
A. Heller myotomy with 270o Belsey fundoplication the question of whether an antireflux procedure should be
B. Heller myotomy with Toupet posterior 180o added to a surgical myotomy. Excellent results have been
fundoplication reported following meticulously performed myotomy with-
C. Heller myotomy with Dor anterior 180o out an antireflux component. Retrospective studies, with
fundoplication long-term follow-up of large cohorts of patients undergoing
D. Heller myotomy with complete 360o fundoplication Heller myotomy demonstrated that, after 10 years, >50% of
patients had reflux symptoms without a fundoplication. In a

CHAPTER 25
recent randomized clinical trial, 7% of patients undergoing
Dor fundoplication following lower esophageal sphincter
(LES) myotomy had abnormal 24-hour pH probes, and 42%
of patients with a myotomy only had abnormal reflux profiles.
If an antireflux procedure is used as an adjunct to esophageal
myotomy, a complete 360° fundoplication should be avoided.
Rather, a 270° Belsey fundoplication, a Toupet posterior 180°

The Esophagus and Diaphragmatic Hernia


fundoplication, or a Dor anterior 180° fundoplication should
be used to avoid the long-term esophageal dysfunction sec-
ondary to the outflow obstruction afforded by the fundopli-
cation itself. (See Schwartz 11th ed., pp. 1055–1056.)

27. Clinical factors that are poor prognosticators in esoph- Answer: E


ageal cancer and exclude surgery for curative intent Clinical factors that indicate an advanced stage of carcinoma
include the following EXCEPT: and exclude surgery with curative intent are recurrent nerve
A. Recurrent nerve paralysis. paralysis, Horner syndrome, persistent spinal pain, paralysis
B. Horner syndrome. of the diaphragm, fistula formation, and malignant pleural
C. Persistent spinal pain. effusion. Factors that make surgical cure unlikely include a
D. Malignant pleural effusion. tumor >8 cm in length, abnormal axis of the esophagus on
E. Hematemesis. a barium radiogram, more than four enlarged lymph nodes
(LNs) on computed tomography (CT), a weight loss >20%, and
loss of appetite. Studies indicate that there are several favor-
able parameters associated with tumors <4 cm in length, there
are fewer with tumors between 4 and 8 cm, and there are no
favorable criteria for tumors >8 cm in length. Consequently,
the finding of a tumor >8 cm in length should exclude cura-
tive resection; the finding of a smaller tumor should encour-
age an aggressive approach. (See Schwartz 11th ed., p. 1073.)

28. The top three most common complications (in order of Answer: A
most common to least common) following an minimally The MIS transthoracic two-field esophagectomy is slightly
invasive surgery (MIS) three-field esophagectomy are: different. In this operation, the abdominal portions of the
A. Pneumonia, atrial fibrillation, anastomotic leak. operation are done first, including placement of the feeding
B. Anastomotic leak, pneumonia, deep vein thrombosis tube, the creation of the conduit, and the sewing of the tip of
(DVT). the conduit to the fully dissected gastroesophageal junction
C. Atrial fibrillation, thoracic duct injury, pneumonia. (GEJ). The patient is then rolled into the left lateral decubitus
D. Pneumonia, anastomotic leak, pneumothorax. position and, through right thoracoscopy, the esophagus is
E. Atrial fibrillation, anastomotic leak, pneumonia. dissected and divided 10 cm above the tumor. Once freed, the
specimen is pulled out through the mini-thoracotomy, and
an end-to-end anastomosis stapler is introduced through the
high corner of the gastric conduit and out a stab wound along
the greater curvature. The anvil of the stapler is placed in the
proximal esophagus and held with a purse-string, the stapler
is docked, the anastomosis is created, and a gastrotomy is
then closed with another firing of the gastrointestinal anas-
tomosis (GIA) stapler. The three-field esophagectomy has
the advantage of placing the anastomosis in the neck where

Brunicardi_Ch25_p207-222.indd 221 04/07/22 2:39 PM


222
leakage is unlikely to create a severe systemic consequence.
On the other hand, placement of the anastomosis in the high
chest minimizes the risks of injury to structures in the neck,
particularly the recurrent laryngeal nerve. Although the leak
of the intrathoracic anastomosis may be more likely to bear
septic consequences, the incidence of leak is diminished.
Other complications of this approach relate to pulmonary
and cardiac status. In many series, the most common com-
plication is pneumonia, the second is atrial fibrillation, and
CHAPTER 25

the third is anastomotic leak. (See Schwartz 11th ed., p. 1076.)


The Esophagus and Diaphragmatic Hernia

Brunicardi_Ch25_p207-222.indd 222 04/07/22 2:39 PM


CHAPTER 26
Stomach

1. All of the following statements about complicated pep- Answer: D


tic ulcer disease requiring hospitalization are TRUE Bleeding is the most common cause of ulcer-related death,
EXCEPT: but only rarely do patients with bleeding gastric or duode-
A. Bleeding is the most common cause of ulcer-related nal ulcer require operation today. The success of endoscopic
death. treatment and medical therapy for bleeding peptic ulcer
B. Perforation is the most common indication for disease (PUD) has resulted in the selection of a small sub-
operation. group of high-risk patients for today’s surgeon. (See Schwartz
C. Endoscopic treatment is usually effective in achiev- 11th ed., p. 1131.)
ing definitive hemostasis. Perforation is the second most common complication of
D. Obstruction rarely requires operation. peptic ulcer, but nowadays it is a much more common indi-
cation for operation than bleeding. (See Schwartz 11th ed.,
p. 1134.)
Most patients with significant obstruction from chronic
ulceration will require some sort of substantial intervention.
Endoscopic balloon dilation can often transiently improve
obstructive symptoms, but many of these patients ulti-
mately fail and come to operation. (See Schwartz 11th ed.,
p. 1136.)

2. Which of the following inhibits gastrin secretion? Answer: D


A. Histamine Luminal peptides and amino acids are the most potent
B. Acetylcholine stimulants of gastrin release, and luminal acid is the most
C. Amino acids potent inhibitor of gastrin secretion. The latter effect is pre-
D. Acid dominantly mediated in a paracrine fashion by somatostatin
released from antral D cells. Gastrin-stimulated acid secre-
tion is significantly blocked by H2 antagonists, suggesting that
the principal mediator of gastrin-stimulated acid production
is histamine from mucosal enterochromaffin-like (ECL)
cells and not direct stimulation of parietal cells by gastrin.
(See Schwartz 11th ed., p. 1109.)

3. Eradication of Helicobacter pylori infection after treat- Answer: B


ment may be confirmed by all of the following EXCEPT: A positive serologic test is presumptive evidence of active
A. Carbon-labeled urea breath test. infection if the patient has never been treated for H. pylori.
B. Serology. Histologic examination of gastric mucosal biopsy using spe-
C. Fecal antigen test. cial stains is the gold standard test for helicobacter infection.
D. Gastric mucosal biopsy. Other sensitive tests include commercially available rapid
urease tests, which assay for the presence of urease in mucosal
biopsy specimens (strong presumptive evidence of infection).
Urease is an omnipresent enzyme in H. pylori strains that
colonize the gastric mucosa. The carbon-labeled urea breath
test has become the standard test to confirm eradication of

223

Brunicardi_Ch26_p223-232.indd 223 08/07/22 11:47 AM


224
H. pylori following appropriate treatment. In this test, the
patient ingests urea-labeled with nonradioactive 13C or 14C.
The labeled urea is acted upon by the urease present in the
H. pylori and converted into ammonia and carbon dioxide.
The radiolabeled carbon dioxide is excreted from the lungs
and can be detected in the expired air. It can also be detected
in a blood sample. The fecal antigen test also is quite sensi-
tive and specific for active H. pylori infection and may also be
used to confirm cure after treatment. (See Schwartz 11th ed.,
CHAPTER 26

p. 1117.)
Because H. pylori induces a strong immunologic response,
serological testing is useful but may not be as accurate as the
urea breath test or the stool antigen test, and a positive serol-
ogy persists after eradication of H. pylori infection, so serology
is not useful to confirm successful treatment of ­Helicobacter
infection. (See Schwartz 11th ed., p. 1119.)
Stomach

4. A patient is referred 2 years following distal gastrectomy Answer: A


and Roux-en-Y reconstruction for nonhealing benign Gastric stasis following surgery on the stomach may be due to a
gastric ulcer. His chief complaint is epigastric pain problem with gastric motor function or caused by an obstruc-
relieved by vomiting undigested food. He has lost 15% of tion. The gastric motility abnormality could have been preexist-
his body weight in the past 6 months. Solid gastric emp- ing and unrecognized by the operating surgeon. Alternatively,
tying scan shows 20% gastric emptying at 4 hours, and it may be secondary to deliberate or unintentional vagotomy, or
upper endoscopy is normal except for retained food in resection of the dominant gastric pacemaker. An obstruction
the stomach. Treatment options include all the following may be mechanical (eg, anastomotic stricture, efferent limb
EXCEPT: kink from adhesions or constricting mesocolon, or a proxi-
A. Transthoracic vagotomy. mal small-bowel obstruction) or functional (eg, retrograde
B. Oral erythromycin. peristalsis in a Roux limb). Gastric stasis presents with vomit-
C. High subtotal gastrectomy. ing (often of undigested food), bloating, epigastric pain, and
D. Feeding jejunostomy. weight loss. Once mechanical obstruction has been ruled out,
medical treatment is successful in most cases of motor dysfunc-
tion following previous gastric surgery. This consists of dietary
modification and promotility agents. (If operation is required)
gastroparesis following subtotal gastric resection is best treated
with near-total (95%) or total gastric resection and Roux-en-Y
reconstruction. (See Schwartz 11th ed., pp. 1157–1158.)
Three prokinetic (metoclopramide, erythromycin, dom-
peridone) may be used to treat delayed gastric emptying. Typ-
ical doses and mechanism of action are shown in Table 26-1.
(See Schwartz 11th ed., p. 1115.)

TABLE 26-1 Drugs that accelerate gastric emptying


Agent Typical Adult Dose Mechanism of Action
Metoclopramide 10 mg PO four times a day Dopamine antagonist
Erythromycin 250 mg PO four times a day Motilin agonist
Domperidone 10 mg PO four times a day Dopamine antagonist

5. Which of the following is secreted by gastric parietal Answer: B


cells? Activated parietal cells secrete intrinsic factor in addition to
A. Pepsinogen hydrochloric acid. Presumably the stimulants are similar, but
B. Intrinsic factor acid secretion and intrinsic factor secretion may not be linked.
C. Gastrin-releasing peptide Intrinsic factor binds to luminal vitamin B12, and the com-
D. Ghrelin plex is absorbed in the terminal ileum via mucosal receptors.
E. Histamine Vitamin B12 deficiency can be life-threatening, and patients
with total gastrectomy or pernicious anemia (ie, patients with
no parietal cells) require B12 supplementation. (See Schwartz
11th ed., pp. 1108–1109.)

Brunicardi_Ch26_p223-232.indd 224 08/07/22 11:47 AM


225
6. The most accurate diagnostic test for Zollinger-Ellison Answer: D
syndrome (ZES) is: All patients with gastrinoma have an elevated gastrin level,
A. Fasting serum gastrin. and hypergastrinemia in the presence of elevated basal acid
B. Computed tomography scan. output (BAO) strongly suggests gastrinoma. Patients with
C. Endoscopy. gastrinoma usually have a BAO > 15 mEq/h or >5 mEq/h
D. Secretin stimulation test. if they have had a previous procedure for peptic ulcer. Acid
secretory medications should be held for several days before
gastrin measurement, because acid suppression may falsely
elevate gastrin levels. Causes of hypergastrinemia can be

CHAPTER 26
divided into those associated with hyperacidity and those
associated with hypoacidity (see Fig. 26-1). The diagnosis of
ZES is confirmed by the secretin stimulation test. An intra-
venous (IV) bolus of secretin (2 U/kg) is given, and gastrin
levels are checked before and after injection. An increase in
serum gastrin of 200 pg/mL or greater suggests the presence
of gastrinoma. (See Schwartz 11th ed., p. 1137.)

Stomach
Elevated serum gastrin
(off PPI + H2RA)

Measure BAO and gastric pH

BAO > 10 mEq/h BAO low


pH < 2 pH > 2

Renal insuff Previous GI surgery Pernicious Atrophic


Secretin stimulation test • Vagotomy? anemia gastritis
• Massive SB resection

Significant elevation in serum Confirm with EGD/Bx


gastrin in response to IV secretin? test/treat H. pylori
give B12

Yes No

Zollinger-Ellison G-cell +/or ECL Loop GJ with antral Antral S/P


syndrome cell hyperplasia alkalinization stasis gastrectomy B2

Octreotide scan Takedown GJ Consider TV Retained antrum on


Confirm with EGD + Bx
R/O MEN-1 Or + antrectomy “duodenal stump”
TV and A

Consider resection Resect retained antrum


Or
Convert B2 to B1

FIG. 26-1. Algorithm for diagnosis and management of hypergastrinemia. BAO = basal acid output; B1 = Billroth 1; B2 = Billroth 2; Bx = biopsy;
ECL = enterochromaffin-like; EGD = esophagogastroduodenoscopy; GJ = gastrojejunostomy; H2RA = histamine 2 receptor antagonist;
insuff = insufficiency; MEN1 = multiple endocrine neoplasia type I; PPI = proton pump inhibitor; R/O = rule out; SB = small bowel; S/P = status
post; TV = truncal vagotomy; TV and A = truncal vagotomy and antrectomy.

Brunicardi_Ch26_p223-232.indd 225 08/07/22 11:47 AM


226
7. In the patient with a secretin stimulation test suggesting Answer: E
gastrinoma, which of the following is the preoperative About 80% of primary tumors are found in the gastrinoma
imaging study of choice? triangle (Fig. 26-2), and many tumors are small (<1 cm),
A. Computed tomography (CT) making preoperative localization difficult. Transabdominal
B. Magnetic resonance imaging (MRI) ultrasound is quite specific, but not very sensitive. CT will
C. Endoscopic ultrasound (EUS) detect most lesions >2 cm in size, and MRI is comparable.
D. Angiographic localization EUS is more sensitive than noninvasive imaging tests, but
E. Somatostatin receptor scintigraphy it still misses many smaller lesions or lesions in inaccessible
locations (eg, the pancreatic tail). Somatostatin receptor scin-
CHAPTER 26

tigraphy (the octreotide scan) or Gallium-68 dotatate posi-


tron emission tomograph/computed tomography (PET/CT)
are sensitive and specific when the pretest probability of gas-
trinoma is high and may identify sites of regional or distant
metastatic disease. (See Schwartz 11th ed., p. 1137.)
Stomach

FIG. 26-2. Gastrinoma triangle.

8. A patient on chronic steroids and nonsteroidal anti- Answer: C


inflammatory drugs (NSAIDs) for rheumatoid arthritis Bleeding is the most common cause of ulcer-related death, but
requires operation for bleeding duodenal ulcer. Patients only rarely do patients with bleeding gastric or duodenal ulcer
like this: require operation today. The success of endoscopic treatment
A. Have a 90-day mortality risk of 5%. and medical therapy for bleeding peptic ulcer disease (PUD)
B. Far outnumber patients requiring operation for per- has resulted in the selection of a small subgroup of high-risk
forated duodenal ulcer. patients for today’s surgeon. It is likely that patients currently
C. Should be considered for lifelong acid suppression if coming to operation for bleeding PUD are at higher risk for a
vagotomy is omitted. poor outcome than ever before. The mortality rate for surgery
D. Are usually treated with distal gastrectomy. for bleeding peptic ulcer is around 20%. (See Schwartz 11th
ed., pp. 1131–1133.) Today, most patients undergoing emer-
gent operation have simple patch of a perforated ulcer or over-
sewing of a bleeding ulcer. (See Schwartz 11th ed., p. 1127.)
Long-term maintenance proton pump inhibitor (PPI) ther-
apy should be considered in all patients admitted to hospital
with ulcer complications, all high-risk patients on NSAIDs or
aspirin (the elderly or debilitated), and all patients requiring
anticoagulation or antiplatelet agents or those with a h­ istory of
recurrent ulcer or bleeding. (See Schwartz 11th ed., p. 1125.)

Brunicardi_Ch26_p223-232.indd 226 08/07/22 11:48 AM


227
9. A 50-year-old homeless man is brought to the emergency Answer: B
room (ER) complaining of 2 days of severe abdominal Surgery is almost always indicated for ulcer perforation,
pain. He states that he “has had ulcer trouble for 30 years.” although occasionally nonsurgical treatment can be used
Temp = 102°F, BP = 80/50, P = 130. He has abdominal dis- in the stable patient without peritonitis in whom radiologic
tention and generalized peritonitis on examination. Serum studies document a sealed perforation. Simple patch closure,
lactate is elevated. He is given intravenous (IV) antibiotics currently the most commonly performed operation for perfo-
and 2 L of crystalloid then started on norepinephrine drip. rated peptic ulcer, is the procedure of choice in patients with
Nasogastric (NG) drains 300 mL of bilious nonbloody hemodynamic instability and/or exudative peritonitis signi-
fluid. Computed tomography (CT) scan immediately after fying a perforation >24 hours old. (See Schwartz 11th ed.,

CHAPTER 26
injection of 100 mL oral contrast via NG shows extravasa- p. 1134.)
tion from the duodenal bulb, copious-free peritoneal fluid,
and air. BP is now 110/70, serum lactate is normal, and
urine output in the past hour is 20 mL. You strongly suspect
perforated duodenal ulcer and you would recommend:
A. Endoscopy and biopsy.
B. Immediate closure of perforation with Graham patch.

Stomach
C. Nonoperative management.
D. Immediate closure of perforation with definitive
ulcer operation.
E. An additional 8 hours of ICU resuscitation, followed
by laparoscopic patch and highly selective vagotomy.

10. Which of the following options is the least preferable Answer: C


reconstruction for patients undergoing antrectomy for Following antrectomy, gastrointestinal (GI) continuity may be
peptic ulcer disease? reestablished with a Billroth I gastroduodenostomy (Fig. 26-3)
A. Bilroth I. or a Billroth II loop gastrojejunostomy (Fig. 26-4). Since antrec-
B. Bilroth II. tomy routinely leaves a 60% to 70% gastric remnant, routine
C. Roux-en-Y
gastrojejunostomy.
A
D. All are equally preferable.

FIG. 26-3. A and B. Billroth I


gastroduodenostomy. (­Reproduced with
permission from Zinner MJ. Atlas of G
­ astric
Surgery. New York, NY: Elsevier/Churchill
­Livingstone; 1992.)

Brunicardi_Ch26_p223-232.indd 227 08/07/22 11:48 AM


228
reconstruction as a Roux-en-Y gastrojejunostomy should be
avoided (Fig. 26-5). Although the Roux-en-Y operation is an
excellent procedure for keeping duodenal contents out of the
stomach and esophagus, in the presence of a large gastric rem-
nant, this reconstruction will predispose to marginal ulceration
and/or gastric stasis. (See Schwartz 11th ed., p. 1129.)
CHAPTER 26
Stomach

A B C

FIG. 26-4. A through C. Billroth II antecolic gastrojejunostomy. (Reproduced with permission from
Zinner MJ, Schwartz SI, Ellis H. Maingot’s Abdominal Operations, 10th ed. Vol. I. Stamford, CT: Appleton
& Lange; 1997.)

<50% gastric
remnant

50 to 60 cm

FIG. 26-5. Roux-en-Y gastrojejunostomy.

Brunicardi_Ch26_p223-232.indd 228 08/07/22 11:48 AM


229
11. A 78-year-old man with good functional status Answer: E
­undergoes evaluation for a newly diagnosed focal antral Multimodality approaches to gastric cancer have become
gastric cancer including cross-sectional imaging, endo- well entrenched with randomized studies from the United
scopic ultrasound, and diagnostic laparoscopy. Studies States and Europe supporting adjuvant chemoradiotherapy
indicate a T2N0 tumor. The least appropriate treatment and perioperative chemotherapy approaches. Evidence from
approach is: Asia supports adjuvant chemotherapy after gastrectomy with
A. Subtotal D2 gastrectomy alone. extended regional lymphadenectomy (ie, D2 lymphadenec-
B. Subtotal D2 gastrectomy followed by chemotherapy. tomy). Although T2N0 cancers were included in some of
C. Initial systemic therapy followed by subtotal D2 the randomized trials, such cases were probably underrep-

CHAPTER 26
gastrectomy. resented and the absolute benefit of multimodality therapy
D. Subtotal gastrectomy with removal and assess- in this context may not be as great as in the overall cohort
ment of >16 regional lymph nodes and adjuvant of patients with locally advanced disease. Selective omission
chemoradiotherapy. of adjuvant therapy in patients with more favorable surgical
E. Endoscopic resection. pathology may be appropriate. T2 tumors are not well treated
with endoscopic resection given a prohibitive risk of recur-
rence. (See Schwartz 11th ed., pp. 1139–1149.)

Stomach
12. Which of the following is not TRUE of hereditary diffuse Answer: B
gastric cancer (HDGC)? Mutations in the E-cadherin gene (CDH1) are associated with
A. It is associated with a mutation in the E-cadherin HDGC. The lifetime risk of gastric cancer is approximately
gene. 70% in men and 56% in women. Mutation-carrying women
B. Lifetime risk of gastric cancer in affected women is are at increased risk for breast cancer, typically lobular carci-
greater than in men. nomas. Because of the poor prognosis associated with clini-
C. It is associated with an increased risk of lobular breast cal gastric cancer in this setting and limitations of existing
cancer. surveillance protocols, prophylactic gastrectomy is a consid-
D. Presentation with clinically significant gastric cancer eration in individuals with CDH1 mutations. (See Schwartz
is associated with a very poor prognosis. 11th ed., p. 1143.)

13. The most common premalignant condition for early Answer: D


­gastric cancer is: Some patients with atrophic gastritis develop intestinal
A. Gastric adenoma. metaplasia in the gastric mucosa that may progress to dys-
B. Chronic ulcer. plasia and then to gastric cancer. Numerous cofactors have
C. Hamartomatous polyp. been implicated, including diet, altered gastric microbiome,
D. Atrophic gastritis. genetics, and hypergastrinemia. Patients with atrophic gas-
tritis are at risk for gastric cancer and should undergo peri-
odic endoscopic surveillance. Metaplastic atrophic gastritis
and dysplastic atrophic gastritis in particular are markers of
increased risk for gastric cancer.
By far the most common precancerous lesion is atrophic
gastritis. There is a growing appreciation of the important
influence of the chronic inflammatory milieu on the genome
of mucosal cells. Chronic inflammation leads to both genetic
and epigenetic changes in mucosal cells, which in the stom-
ach leads to the development of gastritis-associated cancer.
(See Schwartz 11th ed., pp. 1138–1139; 1141–1142.)

14. A patient presents with gastric cancer diagnosed on Answer: E


endoscopy performed for weight loss and abdomi- Diagnostic laparoscopy with peritoneal lavage has emerged as
nal fullness. Cross-sectional imaging shows a diffusely an important component of the staging of gastric cancers and
thickened stomach with regional adenopathy. There is reveals microscopic or macroscopic disseminated disease in up
no evidence of distant metastatic disease. Initial manage- to 36% of cases in published series. Diffuse gastric cancers are
ment should include: often diagnosed at an advanced stage. In the context of a clini-
A. Diagnostic laparoscopy with peritoneal lavage. cal picture consistent with linitis plastica, distant metastasis
B. Consideration of enteral access for nutrition support. is usually present. Although gross disease may be technically
C. Systemic chemotherapy. resectable with total gastrectomy, initiation of systemic therapy
D. Total gastrectomy. is usually a more pressing priority. Enteral access and supple-
E. A, B, and C. mental nutrition may improve candidacy for systemic therapy
in selected cases. (See Schwartz 11th ed., pp. 1139–1149.)

Brunicardi_Ch26_p223-232.indd 229 08/07/22 11:48 AM


230
15. A fifty-year-old man undergoes resection of a 5 cm gas- Answer: E
trointestinal stromal tumor (GIST) involving the fundus Risk of GIST recurrence after resection reflects the location
of the stomach. Pathology indicates 12 mitoses/50 HPF. of the tumor (gastric tumors are less likely to recur than non-
Molecular testing shows a KIT mutation. Subsequent gastric tumors), size, and mitotic rate. Mitotic rate >5/50 HPF
management should include: is the strongest predictor of recurrence. Patients at higher risk
A. Clinical follow-up alone. of recurrence should be treated with adjuvant Imatinib bar-
B. Clinical follow-up with periodic surveillance imag- ring the presence of a PDGFRA D842V mutation, which is
ing alone. associated with Imatinib resistance. Duration of treatment for
C. Adjuvant Imatinib for 1 year. 3 years proved superior to 1 year in the Scandinavian Sar-
CHAPTER 26

D. Adjuvant Sunitinib for at least 3 years. coma Group XV111 trial and there is increasing momentum
E. Adjuvant Imatinib for at least 3 years. for indefinite treatment after resection of high-risk disease.
(See Schwartz 11th ed., pp. 1149–1151.)

16. Which of the following options is the best initial man- Answer: A
agement of a low-grade mucosa-associated lymphoid tis- Low-grade MALT lymphoma, essentially a monoclonal pro-
sue (MALT) lymphoma of the gastric antrum? liferation of B cells, presumably arises from a background of
Stomach

A. Helicobacter pylori eradication chronic gastritis associated with H. pylori. These relatively
B. Chemotherapy +/– radiation therapy innocuous tumors then undergo degeneration to high-grade
C. Wedge resection lymphoma, which is the usual variety seen by the surgeon.
D. Antrectomy Remarkably, when the H. pylori is eradicated and the gastritis
improves, the low-grade MALT lymphoma often disappears.
Thus, low-grade MALT lymphoma is not a surgical lesion.
Careful follow-up is necessary particularly in those lesions
with a t (11:18) translocation, thought to be a risk factor for a
more aggressive MALT lesion. (See Schwartz 11th ed., p. 1149.)

17. Type III gastric carcinoid tumors: Answer: C


A. Often do not require resection. Type III gastric neuroendocrine tumors are sporadic. They
B. Are associated with hypergastrinemia. are most often solitary (usually >2 cm) and occur more com-
C. Are sporadic lesions. monly in men. They are not associated with hypergastrin-
D. Have better outcomes than type I and II tumors. emia. Most patients have regional nodal or distant metastases
at the time of diagnosis, and some present with symptoms of
carcinoid syndrome. (See Schwartz 11th ed., p. 1151.)

18. In the patient with a normal liver and endoscopic diag- Answer: C
nosis of watermelon stomach (gastric antral vascular The parallel red stripes atop the mucosal folds of the distal
ectasia [GAVE]), chronic blood loss requiring multiple stomach give this rare entity its name. Histologically, gastric
transfusions is best treated by: antral vascular ectasia (GAVE) is characterized by dilated
A. Proton pump inhibitors and Carafate. mucosal blood vessels that often contain thrombi, in the
B. Beta blockers. lamina propria. Mucosal fibromuscular hyperplasia and hya-
C. Distal gastrectomy. linization often are present (Fig. 26-6). The histologic appear-
D. Total gastrectomy. ance can resemble portal hypertensive gastropathy, but the
latter usually affects the proximal stomach, whereas water-
melon stomach predominantly affects the distal stomach.

FIG. 26-6. Gastric antral vascular ectasia (watermelon


stomach). (Reproduced with permission from Godlman H,
Hayek J, Federman M. Gastrointestinal Mucosal Biopsy.
New York, NY: Churchill Livingstone; 1996.)

Brunicardi_Ch26_p223-232.indd 230 08/07/22 11:48 AM


231
Beta blockers and nitrates, useful in the treatment of portal
hypertensive gastropathy, are ineffective in patients with gas-
tric antral vascular ectasia. Patients with GAVE are usually
elderly women with chronic gastrointestinal (GI) blood loss
requiring transfusion. Most have an associated autoimmune
connective tissue disorder, and at least 25% have chronic
liver disease. Nonsurgical treatment options include estrogen
and progesterone, and endoscopic treatment with the neo-
dymium yttrium-aluminum garnet (Nd:YAG) laser or argon

CHAPTER 26
plasma coagulator. Antrectomy may be required to control
blood loss, and this operation is quite effective but carries
increased morbidity in this elderly patient group. Patients
with portal hypertension and antral vascular ectasia should
be considered for transjugular intrahepatic portosystemic
shunt (TIPSS). (See Schwartz 11th ed., p. 1154.)

Stomach
19. Treatment for severe early dumping after gastrectomy Answer: C
that is persistent despite an antidumping diet and fiber is: A variety of hormonal aberrations have been observed in
A. Expectant management. early dumping, including increased serum levels of vasoactive
B. Oral glucose for symptoms. intestinal polypeptide (VIP), cholecystokinin (CCK), neuro-
C. Octreotide. tensin, peripheral hormone peptide YY, renin-angiotensin-
D. Surgical conversion to a Roux-en-Y drainage. aldosterone, and decreased atrial natriuretic peptide. Late
dumping is associated with hypoglycemia and hyperinsu-
linemia. Medical therapy for the dumping syndrome consists
of dietary modification and somatostatin analogue (octreo-
tide). (See Schwartz 11th ed., p. 1157.)

20. Ménétrier disease is characterized by: Answer: A


A. Hypertrophic gastric folds and hypoproteinemia. There are two clinical syndromes characterized by epithe-
B. A tortuous submucosal congenital arteriovenous lial hyperplasia and giant gastric folds: Zollinger-Ellison
malformation. syndrome (ZES) and Ménétrier disease. The latter is char-
C. Gastric antral vascular ectasia. acteristically associated with protein-losing gastropathy and
D. Epithelial hyperplasia and hypergastrinemia. hypochlorhydria. There are large rugal folds in the proximal
stomach, and the antrum is usually spared. Mucosal biopsy
shows diffuse hyperplasia of the surface mucus-secreting cells
and usually decreased parietal cells (Fig. 26-7). It has recently
been suggested that Ménétrier disease is caused by local over-
expression of transforming growth factor-A in the gastric
mucosa, which stimulates the epidermal growth factor recep-
tor, a receptor tyrosine kinase, on gastric surface epithelial
cells (SECs). This results in the selective expansion of surface
mucous cells in the gastric body and fundus. A few patients
with this unusual disease have been successfully treated with
the epidermal growth factor receptor blocking monoclonal
antibody cetuximab. (See Schwartz 11th ed., pp. 1153–1154.)

FIG. 26-7. Mucosal biopsy in Ménétrier’s disease.


(Reproduced with permission from Ming S-C, Goldman
H. Pathology of the Gastrointestinal Tract, 2nd ed.
Baltimore, MD: Williams & Wilkins; 1998.)

Brunicardi_Ch26_p223-232.indd 231 08/07/22 11:48 AM


This page intentionally left blank

Brunicardi_Ch26_p223-232.indd 232 08/07/22 11:48 AM


CHAPTER 27
The Surgical Management of Obesity

1. What body mass index [BMI or wt (kg)/height (m2)] Answer: C


definition of obesity serves as the standard indication for A 1991 NIH Consensus Conference recommended that
bariatric surgery when medical therapy has failed and bariatric surgery was indicated for a BMI of 35.0 to 39.9
comorbid conditions exist? when medical therapy has failed and comorbid conditions
A. Overweight (BMI 25.0–29.9) exist. When no comorbid conditions exist, a BMI of ≥40 is
B. Class I obesity (BMI 30.0–34.9) required. This standard continues to be used by insurers,
C. Class II obesity (BMI 35.0-39.9) although recent studies have shown benefit in lower BMI
D. Class III obesity (BMI ≥ 40) groups. (See Schwartz 11th ed., pp. 1169–1170 and Table 27-2
p. 1175.)

2. Which of the following bariatric procedures is primarily Answer: A


intended to induce weight loss through malabsorption of Bariatric procedures are classified as restrictive, malabsorp-
ingested nutrients? tive, or a combination of restrictive and malabsorptive in
A. Jejunoileal bypass the mechanism of weight loss that they induce. Restrictive
B. Sleeve gastrectomy operations include adjustable gastric band and gastric sleeve,
C. Roux-en-Y gastric bypass malabsorptive operations include jejunoileal bypass and its
D. Duodenal switch derivatives, and combined restrictive and malabsorptive pro-
cedures include the Roux-en-Y gastric bypass. (See Schwartz
11th ed., p. 1168.)

3. Complications of adjustable gastric banding which Answer: A


have diminished its popularity as a bariatric procedure The adjustable gastric band procedure, usually performed
include all of the following EXCEPT: laparoscopically, has the lowest cost and mortality risk of all
A. Mortality risk. the bariatric procedures, but is the least effective for weight
B. Slippage of the band. loss. In addition, slippage and erosion of the band and com-
C. Failure to lose weight. plications related to the maintenance and use of the port for
D. Port and tubing complications. adjusting the size of the band contribute to its loss of popular-
ity. (See Schwartz 11th ed., p. 1188.)

4. Early postoperative complications after a Roux-en-Y Answer: D


gastric bypass (RYGB) procedure include all of the fol- Early postoperative complications after RYGB include
lowing EXCEPT: hematemesis due to bleeding from the gastrojejunal staple
A. Hematemesis. or suture line, gastric remnant dilation due to down-stream
B. Dilation of the distal gastric remnant. obstruction, and pulmonary complications such as atelec-
C. Pulmonary embolus. tasis and pulmonary embolus. Diabetes, if present, usually
D. Hyperglycemia. improves promptly after RYGB and hyperglycemia is unlikely
to be problematic. (See Schwartz 11th ed., p. 1185.)

233

Brunicardi_Ch27_p233-234.indd 233 30/06/22 11:24 AM


234
5. Small bowel obstruction after Roux-en-Y gastric bypass Answer: A
(RYGB) should be treated as an urgent surgical emer- Small bowel obstruction after RYGB is frequently due to an
gency because: incarcerated internal hernia at the location of the closure, or lack
A. It is frequently due to an incarcerated internal hernia thereof, of the mesenteric defect. This can progress rapidly to
which can progress to bowel necrosis and perforation. strangulation and necrosis of the bowel with subsequent perfo-
B. Abdominal distension risks disruption of suture lines. ration. Adverse outcomes with this complication have resulted
C. Signs and symptoms of peritonitis, such as pain, fever, in the uniform recommendation that small bowel obstruc-
and leukocytosis, are usually masked in the obese. tion in this setting should be regarded as a surgical emergency.
D. Nasogastric intubation will not decompress the distal Abdominal distention and difficulties with nasogastric intuba-
CHAPTER 27

gastric remnant. tion are not relevant concerns. (See Schwartz 11th ed., p. 1184.)

6. A late complication of Roux-en-Y gastric bypass is Answer: D


the development of post-gastric bypass hypoglycemia PGBH occurs in some patients after Roux-en-Y gastric bypass
(PGBH) due to inappropriately elevated insulin levels. due to exaggerated or excessive insulin levels. It is thought to
Recommended treatments of this condition include all be a consequence of greatly increased GLP-1 levels seen in
of the following EXCEPT: some patients. A very low carbohydrate diet may provide relief
The Surgical Management of Obesity

A. Very low carbohydrate diet. in some patients, and diazoxide and somatostatin have been
B. Anti-secretory agents such as diazoxide and used in those who fail diet therapy. Recently GLP-1 recep-
somatostatin. tor antagonists have been reported to be successful. Although
C. GLP-1 receptor antagonists. pancreatic resection was initially reported in patients with
D. Pancreatic resection. severe hypoglycemia, it is no longer recommended. Partial
pancreatectomy results in a recurrence of the problem, and
total pancreatectomy exchanges one severe condition for
another. (See Schwartz 11th ed., pp. 1204–1205.)

7. Complications which are associated with the gastric Answer: D


sleeve procedure include: The gastric sleeve procedure creates a high pressure tube.
A. Leak from the gastric staple line. This places the gastric staple line at risk for leak, and addi-
B. Gastroesophageal reflux. tional buttress sutures are frequently used to reinforce the
C. Stenosis of the gastric sleeve. staple line. The high pressure also increases the probability
D. All of the above. of gastroesophageal reflux disease (GERD) so patients with a
history of GERD are less favorable candidates for the gastric
sleeve procedure. The gastric tube itself is usually constructed
over a 40-French bougie. If a smaller bougie is used, the risk
of leak and reflux are increased and stenosis of the gastric
sleeve or tube may occur. (See Schwartz 11th ed., p. 1186.)

8. In addition to the effects of weight loss, the resolution of Answer: D


type 2 diabetes mellitus after the gastric sleeve procedure Gastrectomy removes much of the ghrelin-producing portion
and Roux-en-Y gastric bypass (RYGB) is thought to be of the stomach and this mechanism is thought to contribute
contributed to by: to weight loss after both gastric sleeve and RYGB procedures.
A. Reduced ghrelin production. GLP-1, the enteric hormone which augments insulin release,
B. Increased secretion of GLP-1. is dramatically increased after RYGB, and is increased after
C. Appetite suppression. the gastric sleeve procedure as well. A profound suppression
D. All of the above. of appetite and food craving has been found to follow these
procedures, presumably due to the altered hormonal status of
peptides which affect the satiety centers of the central ­nervous
system. (See Schwartz 11th ed., p. 1175.)

9. Adolescent patients with morbid obesity are increasingly Answer: D


being referred for consideration of bariatric procedures Loss of intrinsic factor produced in the gastric fundus,
due to failure of medical management and the risks impaired iron absorption, and a deficiency of the fat-soluble
associated with a lifetime of obesity. What nutritional vitamins present lifelong risks after RYGB and other mal-
deficiencies require lifelong treatment after Roux-en- absorptive bariatric procedures. Vitamin replacement and
Y gastric bypass (RYGB), the most common procedure nutritional monitoring are therefore mandatory in bariatric
performed in this age group? patients. (See Schwartz 11th ed., pp. 1191–1192.)
A. Pernicious anemia due to vitamin B12 deficiency
B. Iron deficiency anemia
C. Deficiencies of vitamins A, E, D, and K
D. All of the above

Brunicardi_Ch27_p233-234.indd 234 30/06/22 11:24 AM


CHAPTER 28
Small Intestine

1. How long is the small intestine? Answer: C


A. 4 to 6 ft The small intestine’s length varies depending on whether
B. 10 to 12 ft radiologic, surgical, or autopsy measurements are made,
C. 4 to 6 m but it is thought to measure 4 to 6 m in living subjects.
D. 10 to 12 m (See Schwartz 11th ed., p. 1219.)

2. Which of the following features is characteristic of the Answer: B


ileum, as opposed to the jejunum? The entire small intestine contains valvulae conniventes,
A. The presence of valvulae conniventes also known as plicae circularis. The jejunum has larger vasa
B. The presence of Peyer patches recta, a larger diameter, and a less fatty mesentery. The ileum
C. Larger vasa recta contains prominent lymphoid follicles called Peyer patches.
D. Less fatty mesentery (See Schwartz 11th ed., p. 1220.)

3. A pocket- or sock-like outpouching on the anti-­ Answer: C


mesenteric side of the distal ileum, called a Meckel diver- The embryonic gut communicates with the yolk sac by mean
ticulum, is caused by: of the vitelline duct. Failure of this structure to obliterate
A. Excessive traction on the intestine during childbirth. by the end of gestation can result in a Meckel diverticulum.
B. Increased intraluminal pressure. (See Schwartz 11th ed., pp. 1246–1247.)
C. A persistent vitelline duct.
D. A mutation of the c-Mec gene.

4. The most common presentations of symptomatic Meckel Answer: C


diverticuli include all of the following EXCEPT: Bleeding, intestinal obstruction, and diverticulitis are the
A. Bleeding. most common presenting symptoms of Meckel diverticuli.
B. Intestinal obstruction. Most are discovered incidentally at surgical procedures.
C. Inguinal pain. Inguinal pain is not a typical symptom. (See Schwartz
­
D. Diverticulitis. 11th ed., p. 1247.)

5. How much fluid normally enters the adult small intes- Answer: D
tine each day? About 8 to 9 L of fluid enters the small intestine daily, of
A. 2 L which over 80% is absorbed. This includes 2 L from oral
B. 4 L intake, 1.5 L of saliva, 2.5 L of gastric juice, 1.5 L of bilio-
C. 6 L pancreatic s­ ecretions, and 1 L of fluid secreted by the small
D. 8 L intestine. (See Schwartz 11th ed., p. 1222.)

235

Brunicardi_Ch28_p235-240.indd 235 30/06/22 11:24 AM


236
6. How are the digestion products of carbohydrates, such Answer: B
as glucose, galactose, and fructose, absorbed through the The three terminal products of carbohydrate digestion are
intestine? transported through the enterocyte brush border membrane
A. By passive diffusion across enterocyte plasma via facilitative transporter proteins such as the sodium-
membranes glucose co-transporter 1 (SGLT1), glucose transporter 2
B. By facilitated diffusion via specific transporters such (GLUT2) and glucose transporter 5 (GLUT5). There is
as SGLT1, GLUT2, and GLUT5 evidence of overexpression of these transporters, particu-
C. By endocytosis of enterocytes on the villus larly SGLT1, in diabetes and obesity, and new therapeutic
D. By facilitated diffusion through tight junctions approaches for these conditions are designed to inhibit these
CHAPTER 28

between enterocytes transporters. (See Schwartz 11th ed., p. 1223.)

7. What does the “enterohepatic circulation” refer to? Answer: C


A. The superior mesenteric—portal venous circuit Bile acids act as detergents which increase the solubility
B. The secretion of cholesterol in the bile and its reab- of lipid micelles which are taken up by the brush border
sorption in the distal ileum ­membrane of the jejunum, where >90% of fat is absorbed.
C. The secretion of bile acids by the liver and their reab- The bile acids themselves remain in the intestinal lumen
Small Intestine

sorption in the distal ileum and are reabsorbed in the distal ileum where they enter
D. The secretion of cholecystokinin by the jejunum and the portal venous c­ irculation and are resecreted in the bile.
its stimulation of bile flow (See Schwartz 11th ed., p. 1225.)

8. Components of the gut-associated lymphoid tissue Answer: B


(GALT) include all of the following EXCEPT: The GALT includes Peyer patches, which are microscopic
A. Plasma cells. aggregates of B-cell follicles and intervening T-cell areas
B. Stellate cells. found in the lamina propria of the small intestine, primar-
C. Dendritic cells. ily in the distal ileum. IgA-producing plasma cells and den-
D. CD4+ and CD8+ T cells. dritic cells populate these regions and serve to protect against
microbes and foreign pathogens. Stellate cells, which produce
collagen when activated, are not contained within the GALT.
(See Schwartz 11th ed., p. 1226.)

9. The radiologic evaluation of a suspected small bowel Answer: C


obstruction may require use of a contrast agent to con- Water-soluble contrast agents, such as gastrograffin, are less
firm the location and possible etiology of the obstruc- likely to provoke peritonitis if a perforation has occurred and
tion. Water-soluble contrast agents, such as gastrograffin, may be more sensitive in the detection of mucosal abnormali-
are preferable to barium for all of the following reasons ties. Gastrograffin has also been shown to be therapeutic for
EXCEPT: the treatment of partial small bowel obstruction, but gastro-
A. Gastrograffin is safer than barium if a perforation has graffin studies take longer and are more labor-intensive to
occurred. perform. (See Schwartz 11th ed., pp. 1229–1231.)
B. Gastrograffin may facilitate the resolution of a partial
small bowel obstruction.
C. Gastrograffin small bowel studies are easier to
perform.
D. Gastrograffin may detect subtle mucosal abnormali-
ties more sensitively than barium.

10. A closed loop obstruction is particularly dangerous Answer: A


because: A closed loop obstruction, in which an intestinal segment is
A. Intraluminal pressure rises high enough to cause obstructed both proximally and distally, as in a volvulus, is
ischemia and necrosis. particularly dangerous because intraluminal pressure rises
B. The obstruction is painless. quickly and can cause venous congestion and arterial obstruc-
C. Bacterial overgrowth results in sepsis. tion which leads to necrosis of the intestinal wall and perfo-
D. The obstructive segment is not apparent on imaging ration. It classically presents with “pain out of proportion to
studies. the physical exam,” and is usually apparent on CT scan which
frequently shows a U-shaped or C- shaped dilated bowel loop
associated with a radial distribution of mesenteric vessels
converging toward a torsion point. (See Schwartz 11th ed.,
p. 1229.)

Brunicardi_Ch28_p235-240.indd 236 30/06/22 11:24 AM


237
11. Therapy of a small bowel obstruction usually consists of Answer: D
prompt surgical correction. In patients with no evidence Partial small bowel obstruction and early postoperative
of closed loop obstruction, and in whom there is no fever obstruction can mimic ileus and may respond to nonop-
or leukocytosis or tachycardia, a period of careful obser- erative therapy. Crohn disease usually responds to medical
vation with nasogastric decompression may be success- therapy, although recurrent obstruction is an indication for
ful in all of the following conditions EXCEPT: surgical correction. Obstruction due to an internal hernia
A. Partial small bowel obstruction. requires prompt surgical intervention to avoid strangulation
B. Obstruction in the early postoperative period. and necrosis. (See Schwartz 11th ed., p. 1231.)
C. Obstruction due to Crohn disease.

CHAPTER 28
D. Obstruction due to an internal hernia.

12. Laparoscopic surgery for bowel obstruction may be pref- Answer: D


erable to open surgery for all of the following reasons The use of laparoscopic surgery for small bowel obstruction
EXCEPT: has been found to be highly successful when the obstruction
A. Laparoscopic surgery is associated with fewer wound is caused by a single adhesive band. The avoidance of an open
infections and a shorter length of hospital stay. procedure also reduces the risk of subsequent adhesion for-

Small Intestine
B. Laparoscopic surgery is preferable in cases where a mation. The morbidity risks of laparoscopic surgery are less
single adhesive band is causing the obstruction. than with open surgery and the risk of iatrogenic injury to the
C. Laparoscopic surgery is less likely to produce fur- bowel is similar. (See Schwartz 11th ed., p. 1232.)
ther adhesion formation which may cause recurrent
obstruction.
D. The risk of iatrogenic bowel injury is higher with
laparoscopic surgery.

13. Crohn’s disease affects the small intestine in 80% of Answer: D


cases, and the colon alone in 20%. Isolated perineal and Extraintestinal manifestations of Crohn’s disease are diffuse
anorectal disease occurs in 5% to 10% of patients. In addi- and can involve dermatologic, rheumatologic, ocular, hepa-
tion, extraintestinal manifestations of Crohn’s disease tobiliary, urologic, and pulmonary conditions. (See Schwartz
occur in up to 25% of patients and may include which of 11th ed., p. 1237.)
the following?
A. Erythema nodosum
B. Interstitial lung disease
C. Nephrolithiasis
D. All of the above

14. The primary genetic defect associated with Crohn’s dis- Answer: A
ease is a mutation of the NOD2 gene on chromosome 16. The protein product of the NOD2 gene mediates the innate
This gene encodes for a protein product which: immune response to microbial pathogens. A variety of defects
A. Mediates the innate immune response to microbial in immune regulatory mechanisms such as over responsive-
pathogens. ness of mucosal T cells to enteric flora-derived antigens can
B. Activates stellate cells to produce collagen. lead to defective immune tolerance and sustained inflamma-
C. Regulates the rate of crypt-to-villus enterocyte tion. (See Schwartz 11th ed., p. 1236.)
migration.
D. Mediates the production of enterocyte alkaline
phosphatase.

15. In the resection of a stenotic area of intestine in a patient Answer: A


with Crohn’s disease, the best approach is: There are no differences in the recurrence rates for resection
A. A resection margin of 2 cm from gross disease. with a 2 cm margin or a 12 cm margin from gross disease.
B. A resection margin of 12 cm from gross disease. The additional bowel lost may contribute to eventual short
C. A resection margin of 2 cm from microscopic disease gut syndrome in a patient who requires multiple resections,
on frozen section. so minimizing bowel loss is a priority. There is no benefit to
D. A resection margin of 12 cm from microscopic achieving frozen section negative margins in the resection
­disease on frozen section. of Crohn’s strictures; positive margin resections have the same
recurrence rate as negative margin resections. The effort
to obtain a frozen section negative margin carries the risk
of removing more intestine than is necessary. (See Schwartz
11th ed., p. 1239.)

Brunicardi_Ch28_p235-240.indd 237 30/06/22 11:24 AM


238
16. The failure of an enterocutaneous fistula to heal on a Answer: D
regimen of total parenteral nutrition and anti-secretory Factors which prevent healing of an entero-cutaneous fistula
therapy may be due to which of the following? include foreign body, epithelialization of the fistula tract,
A. A foreign body in the fistula tract down-stream obstruction, radiation enteritis, associated
B. Epithelialization of the fistula tract infection (abscess or sepsis), malignancy, and a short (<2 cm)
C. Down-stream obstruction of the fistulized segment fistula tract. (See Schwartz 11th ed., pp. 1240–1241.)
of intestine
D. All of the above
CHAPTER 28

17. Which of the following primary malignancies of the Answer: A


small intestine is most common? Adenocarcinomas of the duodenum are the most common
A. Adenocarcinoma of the duodenum primary small bowel malignancy and account for 35% to 50%
B. Carcinoid tumor of the ileum of the total. Lymphoma and GIST tumors of the small bowel
C. Lymphoma of the jejunum are the least common and each accounts for 10% to 15% of
D. Gastrointestinal stromal tumor (GIST) of the the total. (See Schwartz 11th ed., pp. 1241–1242.)
duodenum
Small Intestine

18. Adenocarcinoma of the duodenum is associated with Answer: B


what hereditary oncologic syndrome? Duodenal carcinoma is a late manifestation of the FAP
A. Hereditary nonpolyposis colorectal cancer (HNPCC) syndrome. After resolution of the colonic disease by total
B. Familial adenomatous polyposis (FAP) ­colectomy, patients with FAP must be followed with periodic
C. Peutz-Jeghers syndrome upper gastrointestinal (GI) endoscopy to maintain surveil-
D. Von Hipple-Lindau (VHL) syndrome lance for duodenal tumors. Duodenal cancer is the lead-
ing cause of death among patients with FAP. (See Schwartz
11th ed., p. 1242.)

19. Which of the following statements is TRUE regarding Answer: C


gastrointestinal stromal tumors (GISTs) involving the GISTs are a form of sarcoma which occur most commonly
small intestine? (70%) in the stomach, but comprise 10% to 15% of small
A. Most occur in the small intestine. bowel malignancies. They more frequently present with gas-
B. GISTs are usually metastatic when first diagnosed. trointestinal hemorrhage than other small bowel malignan-
C. GISTs typically present with gastrointestinal cies. They are usually refractory to conventional cytotoxic
hemorrhage. chemotherapy but are not usually metastatic on initial diag-
D. GISTs are usually responsive to cytotoxic nosis. A radical lymphadenectomy is not usually required; a
chemotherapy. segmental resection of the involved portion of the small intes-
tine is usually sufficient surgical treatment. Imatinib, but not
cytotoxic chemotherapy, has been shown to be therapeutic
for unresectable or metastatic disease. (See Schwartz 11th ed.,
pp. 1242–1245.)

20. Methods to prevent radiation enteritis of the small bowel Answer: D


during pelvic irradiation for gynecologic or rectal malig- In addition to limiting radiation exposure to <5000 cGy,
nancy include which of the following? avoiding radiation to the small intestine after pelvic surgery
A. Tilt table positioning in Trendelenberg position dur- can involve steep Trendelenberg positioning during radiation
ing radiation therapy treatments therapy sessions, closure of the pelvic peritoneum at the level
B. Closure (reapproximation) of the pelvic peritoneum of the sacral promontory to prevent small bowel filling the
after primary resection pelvis, and creating of an absorbable mesh sling to prevent
C. Placement of an absorbable mesh sling to suspend the small intestine from filling the pelvic cavity. (See Schwartz
small intestine out of the pelvis during postoperative 11th ed., p. 1246.)
radiation therapy
D. All of the above

21. Common causes of occult gastrointestinal (GI) bleeding Answer: D


include which of the following? Hematemesis, melena, or hematochezia may occur with
A. Bleeding from a Meckel diverticulum occult GI bleeding, as can unexplained iron deficiency ane-
B. Angiodysplasias of the small bowel mia. Push enteroscopy (advancing a long endoscope into the
C. Nonsteroidal anti-inflammatory drug (NSAID)- distal small bowel) or capsule endoscopy may reveal causes
induced ulcers such as a bleeding Meckel diverticulum, angiodysplasias,
D. All of the above as well as Crohn disease or NSAID-induced ulcerations.
(See Schwartz 11th ed., pp. 1250–1251.)

Brunicardi_Ch28_p235-240.indd 238 30/06/22 11:24 AM


239
22. Short bowel syndrome has been arbitrarily defined in Answer: B
adults as having a small intestine of less than what length? A functional definition in which insufficient absorptive
A. 300 cm capacity results in diarrhea, dehydration, and malnutrition
B. 200 cm is more appropriate, but a standard definition of short bowel
C. 100 cm syndrome of 200 cm has been used widely. (See Schwartz
D. 50 cm 11th ed., p. 1254.)

23. Common causes of short bowel syndrome include all of Answer: D

CHAPTER 28
the following EXCEPT: In adults, the common etiologies of short bowel syndrome
A. Mesenteric ischemia. include mesenteric ischemia, malignancy, and Crohn dis-
B. Malignancy. ease. In pediatric patients, common causes include intestinal
C. Crohn disease. atresias, volvulus, and necrotizing enterocolitis. Radiation
D. Radiation enteritis. enteritis usually involves isolated segments of small bowel of
<50% of total small intestinal length. (See Schwartz 11th ed.,
p. 1254.)

Small Intestine
24. After an emergency operation for bowel infarction in Answer: D
which more than half of the small intestine was removed Reducing gastric secretion with proton pump inhibitors or
and a jejunostomy created, high-volume ostomy losses histamine-2 receptor antagonists, reducing gastro-entero-
cause recurrent dehydration. Initial management of this pancreatic secretions with octreotide, and inhibiting motility
condition includes all of the following EXCEPT: with agents such as loperamide or diphenoxylate are useful
A. Proton pump inhibitors or histamine-2 receptor approaches to prevent dehydration as the short gut adapts to
antagonists. its new length. Small intestinal transplantation would be con-
B. Octreotide. sidered only after other therapies including total parenteral
C. Loperamide. nutrition (TPN) fail. (See Schwartz 11th ed., p. 1255.)
D. Small bowel transplantation.

Brunicardi_Ch28_p235-240.indd 239 30/06/22 11:24 AM


This page intentionally left blank

Brunicardi_Ch28_p235-240.indd 240 30/06/22 11:24 AM


CHAPTER 29
Colon, Rectum, and Anus

1. A patient presents for evaluation of blood with wiping Answer: C


and is found to have an ulcerated lesion at the anal verge, Anorectal lymphatic drainage. Lymphatic drainage of
which is biopsied and results as anal squamous cell car- the rectum parallels the vascular supply. Lymphatic chan-
cinoma. How should this patient be first assessed for nels in the upper and middle rectum drain superiorly into
lymph node involvement? the inferior mesenteric lymph nodes. Lymphatic channels
A. Computed tomography (CT) abdomen/pelvis to in the lower rectum drain both superiorly into the inferior
assess for inferior mesenteric or internal iliac nodes mesenteric lymph nodes and laterally into the internal iliac
B. Whole body positron emission tomography-CT lymph nodes. The anal canal has a more complex pattern
(PET-CT) of lymphatic drainage. Proximal to the dentate line, lymph
C. Clinical examination of inguinal nodes drains into both the inferior mesenteric lymph nodes and the
internal iliac lymph nodes. Distal to the dentate line, lymph
primarily drains into the inguinal lymph nodes, but can also
drain into the inferior mesenteric lymph nodes and internal
iliac lymph nodes. (See Schwartz 11th ed., p. 1262.)

2. Which of the following statements is TRUE regarding Answer: C


normal fecal continence and defecation? Defecation. Defecation is a complex, coordinated mecha-
A. Defecation requires a decrease in intra-abdominal nism involving colonic mass movement, increased intra-­
pressure. abdominal and rectal pressure, and relaxation of the pelvic
B. Defecation involves contraction of the puborectalis floor. Distention of the rectum causes a reflex relaxation of
muscle. the internal anal sphincter (the rectoanal inhibitory reflex)
C. The internal and external anal sphincter muscles are that allows the contents to make contact with the anal canal.
active at rest. This “sampling reflex” allows the sensory epithelium to dis-
D. Hemorrhoidal cushions have no role in fecal tinguish solid stool from liquid stool and gas. If defecation
continence. does not occur, the rectum relaxes and the urge to defecate
passes (accommodation response). Defecation proceeds by
coordinating increasing intra-abdominal pressure via a
­Valsalva maneuver with rectal contraction, relaxation of the
puborectalis muscle, and opening of the anal canal.
Continence. The maintenance of fecal continence is at
least as complex as the mechanism of defecation. Continence
requires adequate rectal wall compliance to accommodate the
fecal bolus, appropriate neurogenic control of the pelvic floor
and sphincter mechanism, and functional internal and exter-
nal sphincter muscles. At rest, the puborectalis muscle creates
a “sling” around the distal rectum, forming a relatively acute
angle that distributes intra-abdominal forces onto the pelvic
floor. With defecation, this angle straightens, allowing down-
ward force to be applied along the axis of the rectum and anal
canal. The internal and external sphincters are tonically active
at rest. The internal sphincter is responsible for most of the rest-
ing, involuntary sphincter tone (resting pressure). The external
sphincter is responsible for most of the voluntary sphincter
241

Brunicardi_Ch29_p241-258.indd 241 30/06/22 11:24 AM


242
tone (squeeze pressure). Branches of the pudendal nerve inner-
vate both the internal and external sphincter. The hemor-
rhoidal cushions may contribute to continence by mechanically
blocking the anal canal. Finally, liquid stools exacerbate abnor-
malities with these anatomic and physiologic mechanisms, so
a formed stool contributes to maintaining continence. Thus,
impaired continence may result from poor rectal compliance,
injury to the internal and/or external sphincter or puborectalis,
or neuropathy. (See Schwartz 11th ed., p. 1264.)
Chapter 29

3. A 67-year-old man is diagnosed with a rectal cancer after Answer: A


undergoing colonoscopy for bloody stools and weight Endorectal and endoanal ultrasound. Endorectal ultrasound
loss. Which of the following statements is FALSE regard- is primarily used to evaluate the depth of invasion of neoplas-
ing the use of endorectal ultrasound for rectal lesions? tic lesions in the rectum. The normal rectal wall appears as a
A. The rectal wall appears as a four-layer structure on five-layer structure (Fig. 29-1). Ultrasound can reliably differ-
ultrasound. entiate most benign polyps from invasive tumors based on the
Colon, Rectum, and Anus

B. Ultrasound can reliably distinguish between most integrity of the submucosal layer. Ultrasound can also differ-
benign and malignant rectal polyps. entiate superficial T1-T2 from deeper T3-T4 tumors. Overall,
C. Ultrasound is useful for determining depth of rectal the accuracy of ultrasound in detecting depth of mural inva-
cancer invasion. sion ranges between 81% and 94%. This modality also can
D. Identification of involved perirectal lymph nodes via detect enlarged perirectal lymph nodes, which may suggest
ultrasound is possible in most cases.

Mucosal surface
Mucosa/Muscularis mucosa
Submucosa
Muscularis propria
Serosa/Perirectal fat

FIG. 29-1. A. Schematic of the layers of


the rectal wall observed on endorectal
ultrasonography. B. Normal endorectal
ultrasonography. (A. Reproduced with
permission from Charles O. Finne III, MD,
Minneapolis, MN.) B

Brunicardi_Ch29_p241-258.indd 242 30/06/22 11:24 AM


243
nodal metastases; accuracy of detection of p
­ athologically pos-
itive lymph nodes is 58% to 83%. ­Ultrasound may also prove
useful for early detection of local recurrence after ­surgery.
(See Schwartz 11th ed., pp. 1265–1266.)

4. A 73-year-old woman is seen in preoperative consulta- Answer: F


tion for a cecal mass. You are counseling her on various Minimally invasive techniques of resection. With advances
approaches to a right hemicolectomy. Which of the fol- in minimally invasive technology, many procedures that
lowing are benefits of minimally invasive colon resec- previously have required laparotomy can now be performed

Chapter 29
tion, when compared to open surgery? laparoscopically, with hand-assisted laparoscopy (HAL), or
A. Improved cosmetic results robotically. Potential advantages of minimally invasive sur-
B. Decreased postoperative pain gery include improved cosmetic result, decreased postop-
C. Earlier return of bowel function erative pain, and earlier return of bowel function. Moreover,
D. Less expensive some experimental data suggest that minimally invasive oper-
E. All of the above ations have less immunosuppressive impact on the patient
F. A-C and thus might improve postoperative outcome and even

Colon, Rectum, and Anus


long-term survival. To date, most studies have demonstrated
equivalence between laparoscopic, HAL, and open resection
in terms of extent of resection. Return of bowel function and
length of hospital stay are highly variable. Long-term out-
come has yet to be determined; however, short-term quality
of life appears to be improved by laparoscopy. Laparoscopic
total mesorectal excision for rectal cancer, however, may not
be appropriate. The most recent advances in minimally inva-
sive surgery involve use of robotics and telemanipulation in
which the surgeon operates from a console remote from the
patient. These procedures have been rapidly gaining in popu-
larity, especially for pelvic and rectal resections. Early studies
suggest equivalence between robotic resections and laparo-
scopic/HAL resections.
In addition, some proponents have suggested that robotic
procedures may be easier to learn (a shorter “learning curve”)
and that robotic surgery may be ergonomically better for
the operating surgeon. Nevertheless, long-term superior-
ity, or even equivalence, has yet to be demonstrated, and
these advanced technologies are likely to be associated with
­significant cost. (See Schwartz 11th ed., p. 1270.)

5. Which of the following resections is incorrectly matched Answer: B


with the vessels to be ligated? Right colectomy. A right colectomy is used to remove lesions
A. Right colectomy: ileocolic, right colic, and right or disease in the right colon and is oncologically the most
branches of middle colic vessels appropriate operation for curative intent resection of proxi-
B. Total abdominal colectomy: ileocolic, right colic, mal colon carcinoma. The ileocolic vessels, right colic vessels,
middle colic, left colic, sigmoidal, and superior rectal and right branches of the middle colic vessels are ligated and
vessels divided. Approximately 10 cm of terminal ileum are usually
C. Extended left colectomy: middle colic, left colic, and included in the resection. A primary ileal-transverse colon
first branch of sigmoid vessels anastomosis is almost always possible.
D. Transverse colectomy: middle colic vessels Extended right colectomy. An extended right colectomy
E. Extended right colectomy: ileocolic, right colic, and may be used for curative intent resection of lesions located at
middle colic vessels the hepatic flexure or proximal transverse colon. A standard
F. Left colectomy: left branches of middle colic, left right colectomy is extended to include ligation of the middle
colic, and first branch of sigmoid vessels colic vessels at their base. The right colon and proximal trans-
verse colon are resected, and a primary anastomosis is created
between the distal ileum and distal transverse colon. Such an
anastomosis relies on the marginal artery of Drummond. If
the blood supply to the distal transverse colon is question-
able, the resection is extended distally beyond the splenic
flexure to well-perfused descending colon where the ileocolic
­anastomosis can be performed safely.

Brunicardi_Ch29_p241-258.indd 243 30/06/22 11:24 AM


244
Transverse colectomy. Lesions in the mid and distal trans-
verse colon may be resected by ligating the middle colic
vessels and resecting the transverse colon, followed by a
colocolonic anastomosis. However, an extended right colec-
tomy with an anastomosis between the terminal ileum and
descending colon may be a safer anastomosis with an equiva-
lent functional result.
Left colectomy. For lesions or disease states confined to the
distal transverse colon, splenic flexure, or descending colon,
Chapter 29

a left colectomy is performed. The left branches of the middle


colic vessels, the left colic vessels, and the first branches of
the sigmoid vessels are ligated. A colocolonic anastomosis can
usually be performed.
Extended left colectomy. An extended left colectomy is an
option for removing lesions in the distal transverse colon. In
this operation, the left colectomy is extended proximally to
Colon, Rectum, and Anus

include the right branches of the middle colic vessels.


Sigmoid colectomy. Lesions in the sigmoid colon require
ligation and division of the sigmoid branches of the infe-
rior mesenteric artery. In general, the entire sigmoid colon
should be resected to the level of the peritoneal reflection and
an anastomosis created between the descending colon and
upper rectum. Full mobilization of the splenic flexure is often
required to create a tension-free anastomosis.
Total and subtotal colectomy. Total or subtotal colectomy
is occasionally required for patients with fulminant colitis,
attenuated familial adenomatous polyposis (FAP), or synchro-
nous colon carcinomas. In this procedure, the ileocolic vessels,
right colic vessels, middle colic vessels, and left colic vessels are
ligated and divided. The superior rectal vessels are preserved. If
it is desired to preserve the sigmoid, the distal sigmoid vessels
are left intact, and an anastomosis is created between the ileum
and distal sigmoid colon (subtotal colectomy with ­ileosigmoid
anastomosis). If the sigmoid is to be resected, the sigmoidal
vessels are ligated and divided, and the ileum is anastomosed
to the upper rectum (total abdominal colectomy with ileorec-
tal anastomosis). If an anastomosis is contraindicated, an end
ileostomy is created, and the remaining sigmoid or rectum
is managed either as a mucus fistula or a Hartmann pouch.
(See Schwartz 11th ed., pp. 1270–1272.)

6. You are the chief resident performing an intestinal anas- Answer: A


tomosis with the help of your intern, who requests that Anastomoses. Anastomoses may be created between two seg-
you share some useful facts about anastomoses. Which ments of bowel in a multitude of ways. The geometry of the
of the following would be wrong to teach your intern? anastomosis may be end-to-end, end-to-side, side-to-end, or
A. Hand-sutured anastomoses leak less frequently than side-to-side. The anastomotic technique may be hand-sewn or
stapled anastomoses. stapled (Fig. 29-2). The submucosal layer of the intestine pro-
B. The colon should be mobilized as much as needed to vides the strength of the bowel wall and must be incorporated
ensure a tension-free anastomosis to the rectum. in the anastomosis to assure healing. The choice of anastomo-
C. The submucosal layer provides strength to an sis depends on the operative anatomy and surgeon preference.
anastomosis. Although many surgeons advocate one method over another,
D. Performing an anastomosis in the setting of feculent none has been proven to be superior. Accurate approximation
peritonitis would increase the chances of anastomotic of two well-vascularized, healthy limbs of bowel without tension
leak. in a normotensive, well-nourished patient almost always results
in a good outcome. Anastomoses at highest risk of leak or stric-
ture are those that are in the distal rectal or anal canal, involve
irradiated or diseased intestine including perforation with peri-
toneal soilage, are inadvertently fashioned above a partial distal
obstruction, or are performed in malnourished, immunosup-
pressed, or ill patient. (See Schwartz 11th ed., p. 1274.)

Brunicardi_Ch29_p241-258.indd 244 30/06/22 11:24 AM


245

Chapter 29
A

Colon, Rectum, and Anus


B

FIG. 29-2. A. Sutured end-to-end colocolic anastomosis.


B. Sutured end-to-side ileocolic anastomosis. C. Stapled
side-to-side, functional end-to-end ileocolic anastomosis.
(Reproduced with permission from Bell RH, Rikkers LF,
Mulholland M. Digestive Tract Surgery: A Text and Atlas.
Philadelphia, PA: Lippincott Williams & Wilkins; 1996.) C

7. Your elderly female patient is scheduled for repair of her Answer: C


rectal prolapse today via a transabdominal approach. Regional anesthesia. Epidural, spinal, and caudal anesthet-
When you visit the preoperative area to see if she has any ics can be used for anorectal procedures and transanal resec-
last minute questions, she asks you why she is getting an tions. In patients with severe medical comorbidity, regional
epidural. What can you tell her? anesthesia may occasionally be used for laparotomy and col-
A. Epidural analgesia does not provide as good of pain ectomy. Postoperative epidural anesthesia provides excellent
relief as intravenous (IV) medications, but it is safer pain relief and improves pulmonary function especially after
because of her age. an open operation.
B. An epidural is being used so that general anesthesia General anesthesia. General anesthesia is required for the
can be avoided due to her age and comorbidities. vast majority of intra-abdominal procedures. Patients should
C. An epidural will improve her respiratory mechanics undergo a thorough preoperative cardiovascular evaluation.
postoperatively. In patients with significant comorbid disease, an anesthesia
D. She was incorrectly scheduled for an epidural and consultation may be appropriate. (See Schwartz 11th ed.,
it will be canceled, since these are only used as local pp. 1278–1279.)
anesthesia for anorectal procedures.

Brunicardi_Ch29_p241-258.indd 245 30/06/22 11:25 AM


246
8. The medical student on the colorectal surgery service Answer: D
astutely notes that patients who have undergone colecto- Bowel preparation. The rationale for bowel preparation is
mies tend to have higher incisional infection rates than that decreasing the bacterial load in the colon and rectum will
patients on other services. You decide to review methods decrease the incidence of postoperative infection. Mechanical
for reducing surgical site infections with her. Which of bowel preparation uses cathartics to rid the colon of solid stool
the following is not TRUE? the night before surgery. The most commonly used regimens
A. Mechanical bowel preparation is thought to help include polyethylene glycol (PEG) solutions or magnesium
clear the colon of bacteria. citrate. PEG solutions require patients to drink a large volume
B. Preoperative oral antibiotics can help reduce the of fluid and may cause bloating and nausea. Magnesium citrate
Chapter 29

rate of incisional infection, even when a mechanical solutions are generally better tolerated but are more likely to
bowel prep is not used. cause fluid and electrolyte abnormalities. Both are equally
C. Routine postoperative intravenous (IV) antibiotics efficacious in bowel cleansing. Preparatory formulations
have no role in an uncomplicated colectomy. have been recently introduced in tablet form in an attempt to
D. Prophylactic IV antibiotics should only be given improve tolerance. However, these methods of bowel cleansing
before incision regardless of the length of the opera- require ingestion of ≥40 tablets with water over several hours.
tion, as additional doses during an operation can lead To date, these formulations have not been proven to be supe-
Colon, Rectum, and Anus

to antibiotic resistance. rior to the more traditional products. A ­ ntibiotic prophylaxis


also is recommended. The addition of oral antibiotics to the
preoperative mechanical bowel preparation has been thought
to decrease postoperative infection by further decreasing the
bacterial load of the colon. A recent analysis of the Surgical
Care Improvement Project-1 (SCIP-1) suggests that oral anti-
biotics reduce postoperative wound infection, especially if a
mechanical bowel preparation is not used.
Long-standing, convincing data support the efficacy of par-
enteral antibiotic prophylaxis at the time of surgery. Broad-
spectrum parenteral antibiotic(s) with activity against aerobic
and anaerobic enteric pathogens should be administered just
prior to the skin incision and redosed as needed depending
on the length of the operation. There is no proven benefit to
using antibiotics postoperatively after an uncomplicated col-
ectomy. (See Schwartz 11th ed., p. 1279.)

9. Which of the following extraintestinal manifestations Answer: B


of inflammatory bowel disease cannot be improved by Extraintestinal manifestations. The liver is a common site
treatment of colonic disease? of extracolonic disease in inflammatory bowel disease. Fatty
A. Fatty infiltration of the liver infiltration of the liver is present in 40% to 50% of patients,
B. Primary sclerosing cholangitis and cirrhosis is found in 2% to 5%. Fatty infiltration may be
C. Arthritis reversed by medical or surgical treatment of colonic disease,
D. Erythema nodosum but cirrhosis is irreversible. Primary sclerosing cholangitis
is a progressive disease characterized by intra and extrahe-
patic bile duct strictures. Forty percent to 60% of patients
with primary sclerosing cholangitis have ulcerative colitis.
Colectomy will not reverse this disease, and the only effective
therapy is liver transplantation. Pericholangitis is also associ-
ated with inflammatory bowel disease and may be diagnosed
with a liver biopsy. Bile duct carcinoma is a rare complication
of long-standing inflammatory bowel disease. Patients who
develop bile duct carcinoma in the presence of inflammatory
bowel disease are, on average, 20 years younger than other
patients with bile duct carcinoma.
Arthritis also is a common extracolonic manifestation of
inflammatory bowel disease, and the incidence is 20 times
greater than in the general population. Arthritis usually
improves with treatment of the colonic disease. Sacroiliitis
and ankylosing spondylitis are associated with inflammatory
bowel disease, although the relationship is poorly under-
stood. Medical and surgical treatment of the colonic disease
does not impact symptoms.

Brunicardi_Ch29_p241-258.indd 246 30/06/22 11:25 AM


247
Erythema nodosum is seen in 5% to 15% of patients with
inflammatory bowel disease and usually coincides with
­clinical disease activity. (See Schwartz 11th ed., p. 1281.)

10. Which of the following is an incorrect statement about Answer: C


the management of diverticulitis? Uncomplicated diverticulitis. Uncomplicated diverticulitis is
A. Uncomplicated diverticulitis is typically treated in characterized by left lower quadrant pain and tenderness. CT
the outpatient setting. findings include pericolic soft tissue stranding, colonic wall
B. Hospitalized patients who fail to improve should be thickening, and/or phlegmon. Most patients with uncompli-

Chapter 29
assessed with a computed tomography (CT) scan for cated diverticulitis will respond to outpatient therapy with
abscess formation. broad-spectrum oral antibiotics and a low-residue diet. Anti-
C. Worsening abdominal pain in a patient hospitalized biotics should be continued for 7 to 10 days. About 10% to
with diverticulitis should be addressed with broaden- 20% of patients with more severe pain, tenderness, fever, and
ing of antibiotics. leukocytosis are treated in the hospital with parenteral anti-
D. Most patients with uncomplicated diverticulitis will biotics and bowel rest. Most patients improve within 48 to
never have another episode. 72 hours. Failure to improve may suggest abscess formation.

Colon, Rectum, and Anus


CT can be extremely useful in this setting, and many pericolic
abscesses can be drained percutaneously. Deterioration in a
patient’s clinical condition and the development of peritonitis
are indications for laparotomy.
Most patients with uncomplicated diverticulitis will recover
without surgery, and 50% to 70% will have no further epi-
sodes. It has long been believed that the risk of complications
increases with recurrent disease. For this reason, elective sig-
moid colectomy has often been recommended after the second
episode of diverticulitis, especially if the patient has required
hospitalization. Resection has often been recommended after
the first episode in very young patients and is often recom-
mended after the first episode of complicated diverticulitis.
These general guidelines have been questioned in recent years,
and more recent studies suggest that the risk of complications
and/or need for emergent resection does not increase with
recurrent disease. Moreover, the rate of complications is rare
after elective surgery and recurrences do not increase the rate
of complications. As such, the rate of resection in all patients,
including young patients and those with complicated disease,
has decreased. (See Schwartz 11th ed., pp. 1286–1287.)

11. Which of the following is not a component of the Amster- Answer: B


dam II criteria for the diagnosis of Lynch syndrome, or Lynch syndrome (HNPCC). Lynch syndrome is more com-
hereditary nonpolyposis colorectal cancer (HNPCC)? mon than familial adenomatous polyposis (FAP), but it is
A. Three relatives affected by an HNPCC-associated still extremely rare (1%–3% of all colon cancers). The genetic
malignancy defects associated with Lynch syndrome arise from errors in
B. One cancer occurred before the age of 40 mismatch repair, the phenotypic result being microsatellite
C. One relative must be a first-degree relative of the instable (MSI). Lynch syndrome is inherited in an autoso-
other two mal dominant pattern and is characterized by the develop-
D. Two affected generations ment of colorectal carcinoma at an early age (average age,
40–45 years). Approximately 70% of affected individuals will
develop colorectal cancer. Cancers appear in the proximal
colon more often than in sporadic colorectal cancer and have
a better prognosis regardless of stage. The risk of synchronous
or metachronous colorectal carcinoma is 40%. Lynch syn-
drome may also be associated with extracolonic malignancies,
including endometrial carcinoma, which is most common in
women, and ovarian, pancreas, stomach, small bowel, biliary,
and urinary tract carcinomas. The diagnosis is made based
on family history. The Amsterdam I criteria for clinical diag-
nosis of Lynch syndrome are three affected relatives with his-
tologically verified adenocarcinoma of the large bowel (one

Brunicardi_Ch29_p241-258.indd 247 30/06/22 11:25 AM


248
must be a first-degree relative of one of the others) in two
successive generations of a family with one patient diagnosed
before age 50 years. The presence of other related carcino-
mas should raise the suspicion of this syndrome. Revised cri-
teria Amsterdam II requires three or more relatives with an
HNPCC-related malignancy in which at least one is a first-
degree relative of the others, two generations are affected, at
least one cancer occurred before age 50, familial adenoma-
tous polyposis (FAP) has been excluded, and pathology of the
Chapter 29

tumors has been reviewed and confirmed. In a patient with


an established diagnosis of colorectal cancer, tumor testing
for presence of mismatch repair gene products (immunohis-
tochemistry) and/or MSI can sometimes serve as screening
for this syndrome. (See Schwartz 11th ed., pp. 1292–1293.)

12. A pathologist is reviewing slides of a resected colon can- Answer: C


Colon, Rectum, and Anus

cer specimen. The cancer was noted to invade through Staging. Colorectal cancer staging is based on tumor depth
the bowel wall into the subserosa. There was one involved and the presence or absence of nodal or distant metastases.
lymph node. Assuming there was no distant metastatic Older staging systems, such as the Dukes’ Classification and
disease noted on preoperative imaging, what is the cor- its Astler-Coller modification, have been replaced by the TNM
rect tumor-node-metastasis (TNM) classification and staging system described by the American Joint Committee
stage? on Cancer (AJCC). The AJCC TNM classification has recently
A. T3N1M0, stage II been updated to reflect survival outcomes based upon the
B. T4N1M0, stage II Surveillance Epidemiology and End Results (SEER) registry.
C. T3N1M0, stage III Stage I disease includes adenocarcinomas that are invasive
D. T4N1M0, stage III through the muscularis mucosa but are confined to the sub-
mucosa (T1) or the muscularis propria (T2) in the absence
of nodal metastases. Stage II disease consists of tumors that
invade through the bowel wall into the subserosa or non-
peritonealized pericolic or perirectal tissues (T3) or into
other organs or tissues or through the visceral peritoneum
(T4) without nodal metastases. Stage III disease includes any
T stage with nodal metastases, and stage IV ­disease denotes
distant metastases. (See Schwartz 11th ed., p. 1296.)

13. How can the role of adjuvant chemotherapy following Answer: B


resection of colon cancer be most accurately described? Stages I and II: Localized colon carcinoma (T1-3, N0, M0).
A. Adjuvant chemotherapy should be used after all The majority of patients with stages I and II colon cancer will
colon cancer resections to reduce the risk of recur- be cured with surgical resection. Few patients with completely
rence and improve overall survival. resected stage I disease will develop either local or distant
B. Adjuvant chemotherapy has the best evidence for recurrence, and adjuvant chemotherapy does not improve
routine use in those with positive lymph nodes. survival in these patients. However, up to 46% of patients
C. Microsatellite instable (MSI)-high patients do not with completely resected stage II disease will ultimately die
respond to adjuvant chemotherapy and therefore from colon cancer. For this reason, adjuvant chemotherapy
should not be offered it. has been suggested for selected patients with stage II disease
D. As long as lymph nodes are negative, patients (young patients, tumors with “high-risk” histologic findings).
with stage II disease do not benefit from adjuvant It remains controversial as to whether chemotherapy improves
chemotherapy. survival rates in these patients. In some cases, molecular pro-
filing may predict prognosis, although it is important to note
that these tools have not been shown to predict response to
therapy. At present, molecular profiling for selecting patients
to receive chemotherapy remains unproven.
Stage III: Lymph node metastasis (Tany, N1, M0). Patients
with lymph node involvement are at significant risk for both
local and distant recurrence, and adjuvant chemotherapy has
been recommended routinely in these patients. 5-­Fluorouracil–
based regimens (with leucovorin) and oxaliplatin (FOLFOX)
reduce recurrences and improve survival in this patient pop-
ulation. It is important to note, however, that a subgroup of

Brunicardi_Ch29_p241-258.indd 248 30/06/22 11:25 AM


249
patients with stage III disease will do well without chemo-
therapy. MSI status in particular predicts good prognosis.
Subset analysis from the CRYSTAL trial has shown that
patients with MSI-high stage III disease do not benefit from
5-fluorouracil–based chemotherapy. Molecular profiling,
therefore, may be helpful in determining which stage III
patients can safely avoid systemic chemotherapy. (See Schwartz
11th ed., pp. 1298–1299.)

Chapter 29
14. Which of the following statements about total mesorec- Answer: C
tal excision (TME) for rectal cancer is TRUE? Total mesorectal excision (TME) is a technique that uses sharp
A. TME improves local recurrence but not long-term dissection along anatomic planes to ensure complete resection
survival. of the rectal mesentery during low and extended low anterior
B. TME is not necessary for rectosigmoid tumors. resections. For upper rectal or rectosigmoid resections, a partial
C. TME is associated with less blood loss than a blunt mesorectal excision of at least 5 cm distal to the tumor appears
dissection. adequate. TME both decreases local recurrence rates and

Colon, Rectum, and Anus


D. TME carries a higher risk of injury to the pelvic improves long-term survival rates. Moreover, this technique is
nerves than a blunt dissection. associated with less blood loss and less risk to the pelvic nerves
and presacral plexus than is blunt dissection. The principles of
TME should be applied to all radical resections for rectal can-
cer. (See Schwartz 11th ed., p. 1300.)

15. You encounter an unfamiliar-appearing mass when per- Answer: C


forming a colonoscopy. Your attending, who is standing Leiomyoma and leiomyosarcoma. Leiomyomas are benign
next to you, surmises that this is a leiomyoma of the colon tumors of the smooth muscle of the bowel wall and occur most
and asks how you plan to manage it. How do you respond? commonly in the upper gastrointestinal tract. Most patients
A. If the lesion is small and asymptomatic, it should be are asymptomatic, and lesions are often diagnosed incidentally
left alone. when a mass is seen on endoscopy or felt on digital rectal exami-
B. The lesion should be excised only if it is >5 cm due to nation. However, large lesions can cause bleeding or obstruction.
the risk of malignancy at this size. Because it is difficult to differentiate a benign leiomyoma from
C. All leiomyomas should be resected because malig- a malignant leiomyosarcoma, these lesions should be resected.
nancy (leiomyosarcoma) cannot be ruled out. Recurrence is common after local resection, but most small
D. Oncologic (radical) resections are always indicated leiomyomas can be adequately treated with limited resection.
for leiomyoma, regardless of size, due to the high Lesions > 5 cm should be treated with radical resection because
likelihood of malignancy. the risk of malignancy is high. (See Schwartz 11th ed., p. 1303.)

16. Gastrointestinal stromal tumors (GISTs) are, by defini- Answer: B


tion, malignant. Gastrointestinal stromal tumor (GIST). Gastrointestinal stro-
A. True mal tumors (GIST) are most common in the proximal GI tract
B. False but do occasionally occur in the colorectum (5%–10%) and may
be mistaken for leiomyomas. GISTs are mesenchymal tumors
that arise from the interstitial cells of Cajal. The vast majority
(>95%) of GISTs express CD117 (KIT), and as such, are sensitive
to tyrosine kinase inhibitors (TKIs), such as imatinib mesylate
and sunitinib malate. Risk stratification is based on tumor size
and mitotic activity, and 30% to 50% are malignant. Although
small GISTs may be asymptomatic and discovered incidentally,
larger lesions can cause bleeding, obstruction, or abdominal
pain. Treatment of choice is surgical resection (either local exci-
sion or radical resection) with microscopically negative margins,
if possible; however, local recurrence is common. For larger mar-
ginally resectable tumors, TKIs (imatinib) can be used to shrink
the tumor. These agents can also be considered for adjuvant ther-
apy after resection and are useful for treating metastatic disease.
(See Schwartz 11th ed., p. 1303.)

Brunicardi_Ch29_p241-258.indd 249 30/06/22 11:25 AM


250
17. Which of the following statements accurately describes Answer: A
the role of surgical therapy in anal melanoma? Melanoma. Anorectal melanoma is rare, comprising <1% of all
A. Wide local excision should be attempted first for anorectal malignancies and 1% to 2% of melanomas. Diagnosis
localized disease. is often delayed, and symptoms are attributed to hemorrhoidal
B. Abdominoperineal resection (APR) is the initial disease. Despite many advances in the treatment of cutaneous
treatment of choice due to unacceptably high rates of melanoma, prognosis for patients with anorectal disease remains
recurrence with wide local excision. poor. Overall 5-year survival is <10%, and many patients pres-
C. Surgery has no role in anal melanoma. ent with systemic metastasis and/or deeply invasive tumors at
the time of diagnosis. A few patients with anorectal melanoma,
Chapter 29

however, present with isolated local or locoregional disease that


is potentially resectable for cure, and both radical resection
(APR) and wide local excision have been advocated. Recur-
rence is common and usually occurs systemically regardless of
the initial surgical procedure. Local resection with free margins
does not increase the risk of local or regional recurrence, and
APR offers no survival advantage over local excision. Because
Colon, Rectum, and Anus

of the morbidity associated with APR, wide local excision is


recommended for initial treatment of localized anal melanoma.
In some patients, wide local excision may not be technically
feasible, and APR may be required if the tumor involves a sig-
nificant portion of the anal sphincter or is circumferential. The
addition of adjuvant chemotherapy, biochemotherapy, vaccines,
or radiotherapy may be of benefit in some patients, but efficacy
remains unproven. (See Schwartz 11th ed., p. 1305.)

18. Which operation for rectal prolapse is associated with Answer: D


the lowest recurrence rate? Because rectal prolapse occurs most commonly in elderly
A. Perineal rectosigmoidectomy (Altemeier procedure) women, the choice of operation depends in part on the
B. Reefing the rectal mucosa (Delorme procedure) patient’s overall medical condition. Abdominal rectopexy
C. Anal encirclement (with or without sigmoid resection) offers the most durable
D. Abdominal rectopexy repair, with recurrence occurring in <10% of patients. Peri-
neal rectosigmoidectomy avoids an abdominal operation and
may be preferable in high-risk patients but is associated with
a higher recurrence rate. Reefing the rectal mucosa is effec-
tive for patients with limited prolapse. Anal encirclement
procedures generally have been abandoned (See Schwartz
11th ed., p. 1306.)

19. Which of the following scenarios describes appropriate Answer: C


use of endoscopy in the setting of volvulus? Unless there are obvious signs of gangrene or peritonitis, the
A. A 75-year-old man presenting with sigmoid volvulus initial management of sigmoid volvulus is resuscitation fol-
and peritonitis, since he needs emergent detorsion lowed by endoscopic detorsion. Detorsion is usually most eas-
B. An 80-year-old woman presenting with cecal ily accomplished by using a rigid proctoscope, but a flexible
­volvulus, due to the low rates of recurrence of cecal sigmoidoscope or colonoscope may also be effective. A rectal
volvulus following colonoscopic detorsion tube may be inserted to maintain decompression. Although
C. A 90-year-old woman with sigmoid volvulus and these techniques are successful in reducing sigmoid volvu-
moderate abdominal tenderness, since she is hemo- lus in the majority of patients, the risk of recurrence is high
dynamically stable (up to 40%). For this reason, an elective sigmoid colectomy
should be performed after the patient has been stabilized and
undergone an adequate bowel preparation.
Clinical evidence of gangrene or perforation mandates
immediate surgical exploration without an attempt at endo-
scopic decompression. Similarly, the presence of necrotic
mucosa, ulceration, or dark blood noted on endoscopy
examination suggests strangulation and is an indication for
operation. If dead bowel is present at laparotomy, a sigmoid
colectomy with end colostomy (Hartmann procedure) may
be the safest operation to perform.
Cecal volvulus. Cecal volvulus results from nonfixation
of the right colon. In the majority of cases, rotation occurs

Brunicardi_Ch29_p241-258.indd 250 30/06/22 11:25 AM


251
around the ileocolic blood vessels and vascular impairment
occurs early, although 10% to 30% of the cecum folds upon
itself (cecal bascule). Plain X-rays of the abdomen show a
characteristic kidney-shaped, air-filled structure in the left
upper quadrant (opposite the site of obstruction), and a
Gastrografin enema confirms obstruction at the level of the
volvulus.
Unlike sigmoid volvulus, cecal volvulus can almost never
be detorsed endoscopically. Moreover, because vascular com-

Chapter 29
promise occurs early in the course of cecal volvulus, surgical
exploration is necessary when the diagnosis is made. Right
hemicolectomy with a primary ileocolic anastomosis can
usually be performed safely and prevents recurrence. Simple
detorsion or detorsion and cecopexy are associated with a
high rate of recurrence. (See Schwartz 11th ed., p. 1307.)

Colon, Rectum, and Anus


20. A patient on the trauma wards undergoes abdominal Answer: A
computed tomography (CT) for lack of return of bowel Colonic pseudo-obstruction (Ogilvie syndrome) is a func-
function following operative fixation of pelvic fractures tional disorder in which the colon becomes massively dilated
10 days ago. He has been using high dose opioids for in the absence of mechanical obstruction. Pseudo-­obstruction
pain. Imaging is suggestive of Ogilvie syndrome. He is most commonly occurs in hospitalized patients and is asso-
hemodynamically stable and without evidence of perito- ciated with the use of narcotics, bed rest, and comorbid dis-
nitis. What is the next best step in management? ease. Pseudo-obstruction is thought to result from autonomic
A. Conduct a discussion with the patient and the hospi- dysfunction and severe adynamic ileus. The diagnosis is made
tal’s pain service to identify alternative medications based on the presence of massive dilatation of the colon (usu-
to reduce his opioid use. ally predominantly the right and transverse colon) in the
B. Transfer to the ICU to initiate neostigmine. absence of a mechanical obstruction. Initial treatment consists
C. Consult gastroenterology for colonoscopic of cessation of narcotics, anticholinergics, or other medica-
decompression. tions that may contribute to ileus. Strict bowel rest and intra-
D. Proceed to the operating room (OR) for diagnostic venous hydration are crucial. Most patients will respond to
laparoscopy. these measures. In patients who fail to improve, colonoscopic
decompression often is effective. However, this procedure is
technically challenging, and great care must be taken to avoid
causing perforation. Up to 40% of patients recur. Intravenous
neostigmine (an acetylcholinesterase inhibitor), administered
as a single 2 mg dose, also is extremely effective in decompress-
ing the dilated colon and is associated with a low rate of recur-
rence (20%). However, neostigmine may produce transient but
profound bradycardia and may be inappropriate in patients
with cardiopulmonary disease. Because the colonic dilatation
is typically greatest in the proximal colon, placement of a rec-
tal tube is rarely effective. It is crucial to exclude mechanical
obstruction (usually with a Gastrografin enema) prior to med-
ical or endoscopic treatment. (See Schwartz 11th ed., p. 1308.)

21. In which way do small and large bowel ischemia differ? Answer: C
A. Small bowel ischemia is more common overall. Intestinal ischemia occurs most commonly in the colon.
B. Large bowel ischemia is typically due to major arte- Unlike small bowel ischemia, colonic ischemia rarely is asso-
rial or venous occlusion, whereas small bowel isch- ciated with major arterial or venous occlusion. Instead, most
emia is more frequently seen in low flow states. colonic ischemia appears to result from low flow and/or small
C. Large bowel ischemia is managed nonoperatively vessel occlusion. Risk factors include vascular disease, dia-
more frequently than small bowel ischemia. betes mellitus, vasculitis, hypotension, and tobacco use. In
addition, ligation of the inferior mesenteric artery during
aortic surgery predisposes to colonic ischemia. Occasionally,
thrombosis or embolism may cause ischemia. Although the
splenic flexure is the most common site of ischemic colitis,
any segment of the colon may be affected. The rectum is rela-
tively spared because of its rich collateral circulation.

Brunicardi_Ch29_p241-258.indd 251 30/06/22 11:25 AM


252
Treatment of ischemic colitis depends on clinical severity.
Unlike ischemia of the small bowel, the majority of patients
with ischemic colitis can be treated medically. Bowel rest and
broad-spectrum antibiotics are the mainstay of therapy, and
80% of patients will recover with this regimen. Hemodynamic
parameters should be optimized, especially if hypotension and
low flow appear to be the inciting cause. Long-term sequelae
include stricture (10%–15%) and chronic segmental ischemia
(15%–20%). Colonoscopy should be performed after recov-
Chapter 29

ery to evaluate strictures and to rule out other diagnoses


such as inflammatory bowel disease or malignancy. Failure to
improve after 2 to 3 days of medical management, progres-
sion of symptoms, and deterioration in clinical condition are
indications for surgical exploration. In this setting, all necrotic
bowel should be resected. Primary anastomosis should be
avoided. Occasionally, repeated exploration (a s­econd-look
Colon, Rectum, and Anus

operation) may be necessary. (See Schwartz 11th ed., p. 1309.)

22. Which of the following is the correct pairing of each Answer: C


internal hemorrhoid classification and its definition? Internal hemorrhoids are located proximal to the dentate line
A. First-degree: no prolapse, visualized endoscopically and covered by insensate anorectal mucosa. Internal hemor-
only rhoids may prolapse or bleed, but they rarely become painful
B. Second-degree: prolapse into the anal canal only unless they develop thrombosis and necrosis (usually related
C. Third-degree: prolapse through the anus that requires to severe prolapse, incarceration, and/or strangulation).
manual reduction Internal hemorrhoids are graded according to the extent of
D. Fourth-degree: a term reserved for prolapsed hemor- prolapse. First-degree hemorrhoids bulge into the anal canal
rhoids that are strangulated and may prolapse beyond the dentate line on straining.
­Second-degree hemorrhoids prolapse through the anus but
reduce spontaneously. Third-degree hemorrhoids prolapse
through the anal canal and require manual reduction. Fourth-
degree hemorrhoids prolapse but cannot be reduced and are at
risk for strangulation. (See Schwartz 11th ed., p. 1310.)

23. An emergency room physician calls you in the middle Answer: D


of the night. One of your patients, on whom you per- Other complications of rubber band ligation include urinary
formed rubber band ligation of internal hemorrhoids in retention, infection, and bleeding. Urinary retention occurs in
the office earlier that day, is being seen in the emergency approximately 1% of patients and is more likely if the ligation
room (ER) for severe pelvic pain and inability to urinate. has inadvertently included a portion of the internal sphincter.
How do you respond? Necrotizing infection is an uncommon, but life-threatening
A. “This is a common scenario after rubber band liga- complication. Severe pain, fever, and urinary retention are
tion. The patient should get a prescription for opioid early signs of infection and should prompt immediate eval-
pain medication and can follow up in my office next uation of the patient usually with an examination under
week to ensure the pain is resolved.” anesthesia. Treatment includes debridement of necrotic tis-
B. “This pain can be seen with urinary retention. The sue, drainage of associated abscesses, and broad-spectrum
patient can undergo straight catheterization in the antibiotics. Bleeding may occur approximately 7 to 10 days
ER and is appropriate to discharge after demonstrat- after rubber band ligation, at the time when the ligated ped-
ing the ability to void independently.” icle necroses and sloughs. Bleeding is usually self-limited,
C. “The patient should be admitted for broad-spectrum but persistent hemorrhage may require examination under
antibiotics, and I will see the patient after my sched- anesthesia and suture ligation of the pedicle. (See Schwartz
uled cases tomorrow.” 11th ed., p. 1312.)
D. “I am concerned about a rare but serious com-
plication. I will be there to examine the patient
immediately.”

Brunicardi_Ch29_p241-258.indd 252 30/06/22 11:25 AM


253
24. After drainage of a perianal abscess, for which of the Answer: C
following patients would a course of antibiotics be Anorectal abscesses should be treated by drainage as soon as
prescribed? the diagnosis is established. If the diagnosis is in question,
A. Antibiotics are never prescribed as long as source an examination and drainage under anesthesia are often the
control has been obtained. most expeditious ways both to confirm the diagnosis and
B. A 25-year-old man with approximately 2 cm of cir- to treat the problem. Delayed or inadequate treatment may
cumferential erythema surrounding the drained occasionally cause extensive and life-threatening suppura-
abscess tion with massive tissue necrosis and septicemia. Antibiotics
C. A 40-year-old diabetic patient with a fingerstick glu- are only indicated if there is extensive overlying cellulitis or if

Chapter 29
cose value of 310 mg/dL in the emergency room (ER) the patient is immunocompromised, has diabetes mellitus, or
D. A 30-year-old woman with a well-drained abscess has valvular heart disease. Antibiotics alone are ineffective at
and no cellulitis, though this is her third episode of treating perianal or perirectal infection. (See Schwartz 11th
perianal abscess in the last decade ed., p. 1314.)

25. Which of the following is TRUE regarding rectovaginal Answer: B


fistula? A rectovaginal fistula is a connection between the vagina and

Colon, Rectum, and Anus


A. Obstetric injury is a rare cause of rectovaginal fistula. the rectum or anal canal proximal to the dentate line. Rec-
B. Rectovaginal fistulas in the setting of prior pelvic tovaginal fistulas are classified as low (rectal opening close
radiation can represent cancer recurrence. to the dentate line and vaginal opening in the fourchette),
C. Rectovaginal fistulas in Crohn disease often heal middle (vaginal opening between the fourchette and cervix),
spontaneously. or high (vaginal opening near the cervix). Low rectovaginal
D. Rectovaginal fistulas following obstetric injury typi- fistulas are commonly caused by obstetric injuries or trauma
cally require operative repair. from a foreign body. Mid-rectovaginal fistulas may result
from more severe obstetric injury, but they also occur after
surgical resection of a mid-rectal neoplasm, radiation injury,
or extension of an undrained abscess. High rectovaginal fis-
tulas result from operative or radiation injury. Complicated
diverticulitis may cause a colovaginal fistula. Crohn disease
can cause rectovaginal fistulas at all levels, as well as colovagi-
nal and enterovaginal fistulas.
Diagnosis. Patients describe symptoms varying from the
sensation of passing flatus from the vagina to the passage of
solid stool from the vagina. Most patients experience some
degree of fecal incontinence. Contamination may result in
vaginitis. Large fistulas may be obvious on anoscopic and/or
vaginal speculum examination, but smaller fistulas may be
difficult to locate. Occasionally, a barium enema or vagino-
gram may identify these fistulas. Endorectal ultrasound may
also be useful. With the patient in the prone position, instal-
lation of methylene blue into the rectum while a tampon is in
the vagina may confirm the presence of a small fistula.
Treatment. The treatment of rectovaginal fistula depends
on the size, location, etiology, and condition of surround-
ing tissues. Because up to 50% of fistulas caused by obstetric
injury heal spontaneously, it is prudent to wait 3 to 6 months
before embarking on surgical repair in these patients. If the
fistula was caused by a cryptoglandular abscess, drainage of
the abscess may allow spontaneous closure.
Low and mid-rectovaginal fistulas are usually best treated
with an endorectal advancement flap. The principle of this
procedure is based on the advancement of healthy mucosa,
submucosa, and circular muscle over the rectal opening
(the high-pressure side of the fistula) to promote healing
(­Fig. 29-3). If a sphincter injury is present, an overlapping
sphincteroplasty should be performed concurrently. Fecal
diversion is rarely required. High rectovaginal, colovagi-
nal, and enterovaginal fistulas are usually best treated via a
transabdominal approach. The diseased tissue, which caused
the fistula (upper rectum, sigmoid colon, or small bowel), is

Brunicardi_Ch29_p241-258.indd 253 30/06/22 11:25 AM


254

Probe in
rectovaginal
fistula

Flap of mucosa
and internal
sphincter muscle
Chapter 29

Rectal mucosa and Attenuated


internal sphincter rectovaginal
muscle incised septum
Colon, Rectum, and Anus

Excess flap of
mucosa excised

Internal sphincter Internal sphincter


muscle mobilized muscle approximated

FIG. 29-3. Endorectal advancement flap for rectovaginal fistula.

resected and the hole in the vagina closed. Healthy tissue,


such as omentum or muscle, frequently is interposed between
the bowel anastomosis and the vagina to prevent recurrence.
Rectovaginal fistulas caused by Crohn disease, radia-
tion injury, or malignancy almost never heal spontaneously.
In Crohn disease, treatment is based on adequate drainage
of perianal sepsis and nutritional support. An endorectal
advancement flap may be performed if the rectum is spared
from active Crohn disease. Fistulas resulting from radiation
damage are not amenable to local repair with an advance-
ment flap because of damage to the surrounding rectal and
vaginal tissues. Such mid and high rectovaginal fistulas are
occasionally repaired successfully with a transabdominal
approach in which healthy tissue (omentum, muscle, or non-
radiated bowel) is interposed between the damaged rectum

Brunicardi_Ch29_p241-258.indd 254 30/06/22 11:25 AM


255
and vagina. Fistulas caused by malignancy should be treated
with resection of the tumor. Because differentiating radiation
damage from malignancy can be extremely difficult, all fistu-
las resulting from radiation should be biopsied to rule out the
presence of cancer. (See Schwartz 11th ed., pp. 1317–1318.)

26. A patient is seen in consultation in the emergency Answer: A


room (ER) after presenting with moderate generalized Foreign body entrapment in the rectum is not uncommon.
abdominal pain. He reports insertion of a rectal foreign Depending on the level of entrapment, a foreign body may

Chapter 29
body 3 days ago, which remains in place. He is hemody- cause damage to the rectum, rectosigmoid, or descending
namically stable and without peritonitis on examination. colon. Generalized abdominal pain suggests intraperitoneal
What is the next best step in management? perforation. Evaluation of the patient includes inspection of
A. Upright abdominal X-ray to evaluate for free air the perineum and a careful abdominal examination to detect
B. CT abdomen/pelvis to locate foreign body any evidence of perforation. Plain films of the abdomen are
C. Removal of foreign body in the ER mandatory to detect free intra-abdominal air.
D. Proceed to operating room (OR) for laparotomy Foreign bodies lodged low in the rectum may often be

Colon, Rectum, and Anus


removed under conscious sedation with or without a local
anesthetic block. Objects impacted higher in the rectum
may require regional or general anesthesia for removal. Only
rarely will a laparotomy be required to remove the object,
either through manual manipulation of the object to expel
from the anus, or via colotomy. After removal of the foreign
body, it is crucial to evaluate the rectum and sigmoid colon
for injury. Proctoscopy and/or flexible sigmoidoscopy should
be performed. A hematoma without evidence of perforation
requires no surgical treatment. Perforation of the rectum or
sigmoid colon should be managed as described in the preced-
ing sections. (See Schwartz 11th ed., p. 1323.)

27. A patient on the hematology service status post bone Answer: D


marrow transplant develops abdominal cramping, nau- Neutropenic enterocolitis (typhlitis) is a life-threatening
sea/vomiting, and diarrhea with specks of bright blood problem with a mortality rate of >50%. This syndrome is
in it. The general surgery team is consulted and reviews characterized by abdominal pain and distention, fever, diar-
the available computed tomography (CT) scan, which rhea (often bloody), nausea, and vomiting in a patient with
shows pneumatosis and stranding of the right colon fewer than 1000 neutrophils/μL blood from any cause (bone
without evidence of frank perforation. Upon evaluation, marrow transplantation, solid-organ transplantation, or
­
the patient is noted to have some distension and mod- ­chemotherapy). Its etiology is poorly understood. Histologic
erate diffuse abdominal tenderness without peritonitis. features can be seen on biopsy or surgical resection and include
Vitals are within normal limits. What recommendation a paucity of inflammatory and leukemic infiltrates but with
should the general surgery consulting team provide? mucosal and submucosal edema, villous sloughing, ­stromal
A. The presence of pneumatosis implies a contained hemorrhage, and patchy-to-complete epithelial necrosis.
perforation, so the patient should be taken urgently CT scan of the abdomen often shows a dilated cecum with
to the operating room (OR) for exploration and pericolic stranding. However, a normal-appearing CT scan
resection. does not exclude the diagnosis. Some patients will respond
B. Mesenteric ischemia is highest on the differential, and to bowel rest, broad-spectrum antibiotics, parenteral nutri-
the gastroenterologist on call should be contacted to tion, and granulocyte infusion or colony-stimulating factors.
perform a colonoscopy for definitive diagnosis. Evidence of perforation, generalized peritonitis, and dete-
C. This is not an uncommon development after bone rioration in clinical condition are indications for operation.
marrow transplant. Physical examination is unreli- (See Schwartz 11th ed., pp. 323–324.)
able, so daily CT scans should be obtained to evaluate
for perforation.
D. The patient should be prescribed bowel rest and
broad-spectrum antibiotics with serial abdominal
examinations to evaluate for progression.

Brunicardi_Ch29_p241-258.indd 255 30/06/22 11:25 AM


256
28. A 74-year-old man with biopsy-proven rectal adenocar- Answer: C
cinoma is undergoing a low anterior resection. Which The wall of the colon and rectum are made of five separate
layers must be stapled through when resecting the distal layers: mucosa, submucosa, circular muscle layer, longitu-
portion of resection specimen? dinal muscle layer, and serosa. The mid and lower rectum
A. Mucosa, submucosa, circular muscle layer, longitudi- lack serosa so this layer would not be stapled through if the
nal muscle layer, and serosa ­surgeon were stapling through the mid or lower rectum.
B. Mucosa, submucosa, longitudinal muscle layer, (See Schwartz 11th ed., p. 1260.)
­circular muscle layer, and serosa
C. Mucosa, submucosa, longitudinal muscle layer,
Chapter 29

­circular muscle layer

29. A healthy 48-year-old physician with no family history Answer: B


of caner and who strictly adheres to a high protein, high FOBT has been a nonspecific test for peroxidase contained
fiber diet, exercises five times per week for 50 minutes, in hemoglobin; consequently, occult bleeding from any
and takes vitamin C supplements daily performs a fecal gastrointestinal source will produce a positive result. Simi-
occult blood test (FOBT) on herself and tests positive. larly, many foods (red meat, some fruits and vegetables, and
Colon, Rectum, and Anus

Should she have any further colon screening? ­vitamin C) will produce a false-positive result. Any positive
A. No, vitamin C can produce a false positive result. FOBT mandates further investigation, usually by colonos-
B. Yes, all positive FOBT requires further investigation copy. (See Schwartz 11th ed., p. 1293.)
with a colonoscopy.
C. Yes, all positive FOBT requires further investigation
with FOBT in 1 year.
D. No, she has no risk factors for colon cancer and
should follow the United States Preventive Ser-
vices Task Force (USPSTF) screening guidelines for
colorectal cancer.

30. A 76-year-old man undergoes an emergent sigmoid- Answer: A


ectomy for a perforated colon mass. The surgeon per- In an emergency operation, like this one, where the stoma site
forms a Hartmann procedure and brings up a colostomy. has not been marked, an attempt should be made to place a
In an emergency setting, where is the most appropriate stoma within the rectus muscle and away from both the costal
­location to seat a colostomy? margin and iliac crest. In emergencies, placement high on the
A. Above the beltline, within the rectus abdominus abdominal wall is preferred to a low-lying site. (See Schwartz
muscle, away from the costal margin 11th ed., pp. 1275–1276.)
B. Below the beltline, within the rectus abdominus
­muscle, near the iliac crest
C. Above the beltline, within the rectus abdominus
muscle, near the costal margin
D. Below the beltline, within the rectus abdominus
­muscle, away from the iliac crest

31. A 50-year-old woman who underwent a total colectomy Answer: B


with ileal pouch-anal reconstruction 5 years ago presents This patient is likely presenting with pouchitis. Pouchitis is
to the emergency room with diarrhea, fever, 2 weeks of an inflammatory condition that affects both ileoanal pouches
malaise, and severe abdominal pain. What is the most and continent ileostomy reservoirs. The incidence of pou-
appropriate differential diagnosis? chitis ranges from 30% to 55%. Symptoms include increased
A. Parasitic infection, ulcerative colitis of the remaining diarrhea, hematochezia, abdominal pain, fever, and malaise.
rectal cuff, undiagnosed Crohn disease Diagnosis is made endoscopically with biopsies. Differential
B. Bacterial or viral infection, undiagnosed Crohn dis- diagnosis includes infection and undiagnosed Crohn disease.
ease, and pouchitis (See Schwartz 11th ed., p. 1278.)
C. Rectal cancer of remaining rectal cuff, bacterial or
viral infection, and undiagnosed Crohn disease
D. Parasitic infection, bacterial or viral infection, and
pouchitis

Brunicardi_Ch29_p241-258.indd 256 30/06/22 11:25 AM


257
32. An 18-year-old woman is undergoing workup to deter- Answer: A
mine if she has ulcerative colitis, Crohn disease, or Ulcerative colitis is a mucosal process in which the colonic
indeterminate colitis. What diagnostic findings would mucosa and submucosa are infiltrated with inflammatory
indicate that she has ulcerative collitis? cells. The mucosa may be atrophic, and crypt abscesses are
A. Atrophic mucosa, crypt abscesses, inflammatory common. Endoscopically, the mucosa is frequently friable
pseudopolyps, scarred and shortened colon, continu- and may possess multiple inflammatory pseudopolyps. In
ous involvement of rectum and colon long-standing ulcerative colitis, the colon may be foreshort-
B. Mucosal ulcerations, noncaseating granulomas, ened and the mucosa replaced by scar. A key feature of ulcer-
fibrosis, strictures, and fistulas in the colon with deep ative colitis is the continuous involvement of the rectum and

Chapter 29
serpiginous ulcers colon; rectal sparing or skip lesions suggest a diagnosis of
C. Atrophic mucosa, noncaseating granulomas, stric- Crohn disease. Crohn disease is a transmural inflammatory
tures, “cobblestone” appearance on endoscopy process that can affect any part of the gastrointestinal tract
D. Mucosal ulcerations, crypt abscesses, inflammatory from mouth to anus. Mucosal ulcerations, an inflammatory
pseudopolyps, continuous involvement of colon and cell infiltrate, and noncaseating granulomas are characteristic
rectum pathologic findings. Chronic inflammation may ultimately
result in fibrosis, strictures, and fistulas in either the colon or

Colon, Rectum, and Anus


small intestine. The endoscopic appearance of Crohn colitis is
characterized by deep serpiginous ulcers and a “cobblestone”
appearance. (See Schwartz 11th ed., p. 1280.)

Brunicardi_Ch29_p241-258.indd 257 30/06/22 11:25 AM


This page intentionally left blank

Brunicardi_Ch29_p241-258.indd 258 30/06/22 11:25 AM


CHAPTER 30
The Appendix

1. A 25-year-old man presents with migratory right lower Answer: E


quadrant (RLQ) pain, leukocytosis, and a computed Emergent surgery is often performed in patients with appen-
tomography (CT) scan consistent with acute, uncompli- dicitis, but studies have evaluated the performance of urgent
cated appendicitis. He is physiologically normal and it is surgery (waiting < 12 hours) in a semi-elective setting after
2 AM. You are planning an appendectomy, what differ- administering antibiotics upon admission. The studies did
ence might be expected in his outcome if his operation is not reveal any significant difference in outcomes, except for a
delayed until the next morning? slightly longer hospital stay in those undergoing urgent sur-
A. Increased risk of an intra-abdominal abscess gery. Currently, delaying surgery <12 hours is acceptable in
B. Increased risk of surgical-site infection patients with short duration of symptoms (<48 hours) and in
C. Decreased operative time nonperforated, non-gangrenous appendicitis. (See Schwartz
D. Increased risk of perforation 11th ed., p. 1335.)
E. No difference in perforation rates, surgical-site
­infection, abscess, conversion rate, or operative time

2. A 55-year-old man has computed tomography (CT) Answer: C


evidence of complicated appendicitis with a contained Perforated appendicitis can be managed either operatively or
abscess in the right lower quadrant (RLQ). He is mildly nonoperatively. Immediate surgery is necessary in patients
tachycardic, afebrile, and normotensive with local RLQ that appear septic, but this is usually associated with higher
tenderness but no peritonitis. What is the optimal complications, including abscesses and enterocutaneous
approach to this patient? ­fistulae due to dense adhesions and inflammation. The man-
A. Immediate laparotomy agement of long-duration, complicated appendicitis is often
B. Laparoscopic exploration and abscess drainage staged. Patients are resuscitated and treated with IV antibiot-
C. Percutaneous drainage, intravenous (IV) fluids, ics. Patients with long-standing perforation are better treated
bowel rest, and broad spectrum antibiotics with adequate percutaneous image-guided drainage. This
D. IV fluids, bowel rest, and broad spectrum antibiotics strategy is successful in 79% of patients who achieve complete
resolution, which occurs more often in lower-grade abscesses,
trans-gluteal drainage, and with CT- (vs. ultrasound-) guided
drainage. Operative intervention is performed in patients
who fail conservative management and in patients with free
intra-peritoneal perforation. (See Schwartz 11th ed., p. 1335.)

3. An 8-year-old boy presents to the emergency depart- Answer: B


ment complaining of generalized abdominal pain over Previously considered a vestigial organ, the appendix is now
the past 24 hours. Laboratory tests reveal a leukocytosis linked to the development and preservation of gut-associated
of 13,000 and he is tender in the right lower quadrant lymphoid tissue (GALT) and to the maintenance of intestinal
(RLQ) on physical examination. He is taken to the oper- flora. It has been suggested that appendectomy is associated
ating room for laparoscopic appendectomy. Removal of with increased C. difficile infections and increased subsequent
the appendix has been associated with a protective effect cancer (colon, esophageal) as a result of microbial alteration,
to which of the following? although this is currently unproven. The protective effect of
A. Crohn colitis an early appendectomy against development of ulcerative
B. Ulcerative colitis colitis has been proposed to be mechanistically linked to the
C. Clostridium difficile release of dimeric forms of IgA from plasma B cells and the
D. Carcinoid Th2 response mediated by IL-13–producing natural killer T
cells. (See Schwartz 11th ed., p. 1331.)
259

Brunicardi_Ch30_p259-262.indd 259 30/06/22 11:25 AM


260
4. While reviewing pathology of a recent laparoscopic Answer: B
appendectomy, you note that in addition to acute appen- Appendiceal carcinoid tumors are submucosal rubbery
dicitis, the patient had a 1.5-cm carcinoid tumor located masses that are detected incidentally on the appendix.
at the base of the appendix with mesenteric invasion. ­Carcinoid tumors of the appendix are relatively indolent but
The patient is otherwise healthy and recovering well can develop nodal or hepatic metastases. Infrequently, these
from surgery. What would you recommend? can be associated with carcinoid syndrome if there are hepatic
A. No additional therapy is necessary. metastases (2.9%). Upon incidental findings of a suspected
B. Right hemicolectomy. carcinoid, the surgeon must evaluate the nodal basin along
C. Radical appendectomy. the ileocolic pedicle and also examine the liver for any signs
CHAPTER 30

D. Adjuvant chemotherapy. of metastases. For lesions that are <1 cm (95% of all lesions),
a ­negative margin appendectomy is adequate. For tumors
≥2 cm, a right hemicolectomy is recommended. For lesions
1 to 2 cm in size, there is no consensus on a completion col-
ectomy. A right colectomy is often performed for mesenteric
invasion, enlarged nodes, or positive or unclear margins.
Measurement of serum chromogranin A is recommended.
The Appendix

A radical appendectomy is not a described operation and


adjuvant chemotherapy could be considered but only after
­definitive surgical care. (See Schwartz 11th ed., p. 1338.)

5. A previously healthy 20-year-old man was brought to Answer: D


the emergency room with a 10-hour history of fever, Upon performing a laparoscopy or laparotomy for suspected
anorexia, vomiting, and pain in the right lower abdo- appendicitis, if one finds no evidence of appendicitis, a thor-
men. On examination, he has tenderness localized to ough exploration of the peritoneum must be performed to
McBurney’s point. His laboratory data show leukocyto- rule out contributing pathology. A normal appendix is often
sis of 13,000/mm3. On surgical exploration, you note a removed to reduce future diagnostic dilemma. Management
normal-appearing appendix and cecum. The terminal of incidentally found common conditions is summarized in
ileum appeared inflamed and edematous, concerning for Table 30-1. (See Schwartz 11th ed., p. 1337 and Table 30-3.)
Crohn disease without stigmata of obstruction. What is
the most appropriate further course of action?
A. Abort the planned appendectomy.
B. Resect the inflamed terminal ileum.
C. Bypass the inflamed ileum by performing
ileo-colostomy.
D. Perform an appendectomy if the base is uninflamed.

TABLE 30-1 Management of Intraoperative Findings Mimicking Appendicitis


Ovarian torsion Conservative management with detorsion and oophoropexy
Crohn terminal ileitis Appendectomy if base uninflamed
Meckel diverticulitis Segmental small bowel resection and primary anastomosis
Appendiceal mass Laparoscopic appendectomy/ ileocecectomy without capsular disruption or spillage and retrieval in a bag

6. During a laparoscopic appendectomy on a 59-year-old Answer: C


woman with clinical presentation of acute appendicitis, Low-grade appendiceal mucinous neoplasms (LAMN) are
you find an enlarged cystic mass replacing the appendix nonmalignant neoplasms of the appendix that present in a
with a normal appendiceal stump. The mass is nonad- third of cases with appendicitis. It is important to carefully
herent to the rest of the structures, and the rest of the assess for the presence of ascites, peritoneal disease, and scal-
abdominal cavity appears normal. What is the most loping of the liver surface on imaging upon initial evaluation.
appropriate next step? A reliable diagnosis cannot be established using imaging
A. Perform a right hemicolectomy after obtaining con- alone, and it is recommended that surgical excision without
sent from the family capsular disruption is undertaken. The importance of care-
B. Close the abdomen and arrange a return visit for ful handling of a LAMN and the avoidance of rupture cannot
reevaluation be overemphasized because the intraperitoneal spread of epi-
C. Laparoscopically excise the abnormal appendix with- thelial cells with subsequent development of pseudomyxoma
out capsular disruption by dividing the appendix at peritonei is a dreaded consequence. In cases where a homo-
the normal stump and retrieve the specimen in a bag geneous cyst without nodularity or signs of dissemination is
D. Limited ileocecectomy encountered, laparoscopic excision is acceptable, provided

Brunicardi_Ch30_p259-262.indd 260 30/06/22 11:25 AM


261
that a stapler is fired across the base of the cecum to avoid a
positive margin. The specimen should be placed in a plastic
bag and carefully removed without disruption. (See Schwartz
11th ed., p. 1339.)

7. A 30-year-old woman presented with pain in the right Answer: D


lower abdomen, fever, nausea, and leukocytosis. With In the case of a nonviable appendix base, a staple line through
a diagnosis of appendicitis, you took her to operating the cecum that avoids the ileocecal valve might be sufficient,
room (OR) for laparoscopic appendectomy. During the unless significant inflammation is present. The appendix

CHAPTER 30
surgery, you noticed the appendix base is perforated and is retrieved through the midline port in a s­pecimen bag,
appeared nonviable (Fig. 30-1). What is the most appro- especially if an appendiceal lesion is suspected. If a peri-­
priate management? appendiceal phlegmon is encountered or if the operation is
A. Place a drain and close the abdomen being performed for perforated appendicitis, careful sweeping
B. Place an endoloop around the site of perforation and of the bowel with a blunt dissector can release the appendix. It
divide the appendix is important to carefully separate adjacent bowel, which can
C. Resect the appendix at the site of perforation and be friable in such settings. (See Schwartz 11th ed., p. 1336.)

The Appendix
suture the base
D. Perform appendectomy using a stapling device, with
staple line through the healthy cecum

FIG. 30-1. Gangrenous appendicitis with


perforation is seen at laparoscopy.

8. A 26-year-old man presents with a 1-day history of right Answer: B


lower abdominal pain and fever. He undergoes a laparo- In patients with perforated appendicitis undergoing operative
scopic converted to open appendectomy due to difficulty intervention, preoperative antibiotics are necessary to cover
in dissecting the appendiceal stump. Intraoperatively, he gram-negative bacteria and anaerobes. Monotherapy with
is noted to have a perforated appendix base with mini- piperacillin/tazobactam or combination of cephalosporin
mal peritoneal contamination. The appendectomy is with metronidazole are reasonable choices. The duration of
completed by stapling the cecum at a healthy area. What postoperative antibiotics is generally <4 days once complete
is the most appropriate postoperative antibiotic therapy? source control has been achieved (STOP-IT trial). Patients
A. Single dose cefazolin. with incomplete drainage, persistent catheters, complica-
B. Cefazolin and metronidazole for 3 to 4 days. tions from surgery, and uncertain resolution of inflammation
C. Piperacillin/Tazobactam for 7 to 8 days. might need a longer duration of antibiotics. (See Schwartz
D. Postoperative antibiotics are not needed. 11th ed., p. 1336.)

9. A 24-year-old woman presents with pain in her abdo- Answer: C


men, fever and tenderness in right lower quadrant for the The preferred approach to manage patients with uncompli-
last 8 hours. A computed tomography (CT) scan reveals cated appendicitis is an appendectomy. Several recent ran-
uncomplicated appendicitis. She is concerned about an domized trials and cohort studies have examined the role
appendectomy and is afraid of surgery. She asks for alter- of nonoperative management of adult patients with appen-
nate treatment options. What are appropriate alternate dicitis. A majority of the patients in the nonoperative arm
options for this otherwise healthy patient? received IV antibiotics for a short course followed by a course
A. Counsel that surgery is the only option. of a fluoroquinolone and metronidazole, or oral amoxicil-
B. Prescribe oral fluoroquinolones with analgesics and lin/clavulanic acid. Currently, conservative management can
ask her to follow up after a week. be offered to informed patients using techniques of shared
C. Admit and start intravenous (IV) antibiotics. decision-making, but it is not the standard modality of man-
D. Reassure her that surgery is not emergent, and she agement of appendicitis, except in patients with significant
can reconsider it after a few weeks. phobia of surgery (Fig. 30-2). (See Schwartz 11th ed., p. 1335.)

Brunicardi_Ch30_p259-262.indd 261 30/06/22 11:25 AM


262
CHAPTER 30
The Appendix

FIG. 30-2. Coronal view of CT shows


inflammatory changes involving the appendix
without evidence of abscess, consistent with
uncomplicated acute appendicitis.

10. A 32-year-old otherwise healthy male patient presents Answer: C


with suspicious symptoms of appendicitis but the physi- Imaging is often utilized to confirm a diagnosis of appendi-
cal examination is inconclusive. Which statement about citis because a negative operation rate is acceptable in <10%
imaging in this clinical situation is correct? of male patients and <20% of female patients. A contrast-
A. An ultrasound is preferred to computed tomography enhanced CT scan has a sensitivity of 0.96 (95% confidence
(CT) scan due to its higher sensitivity and reduced interval [CI] 0.95–0.97) and specificity of 0.96 (95% CI 0.93–
risk of radiation to patients. 0.97) in diagnosing acute appendicitis. Features on a CT scan
B. A CT finding of enlarged appendiceal lumen and that suggest appendicitis include enlarged lumen and double
double wall thickness (>6 mm) suggests inflamma- wall thickness (>6 mm), wall thickening (>2 mm), periap-
tory enteritis instead of appendicitis. pendiceal fat stranding, appendiceal wall thickening, and/or
C. Low dose CT scans (2 mSv) have equivalent clinical an appendicolith. While there remains a concern of ionizing
accuracy as high dose CT scans in the diagnosis of radiation exposure with a CT scan, typical low-dose CT scans
appendicitis. result in exposure of 2 to 4 mSv, which is not significantly
D. Graded compression ultrasound suggestive of eas- higher than background radiation (3.1 mSv). Recent trials
ily compressible appendix < 5 mm in diameter is have also suggested that although low-dose CT scans of 2 mSv
pathognomonic of appendicitis. do not generate high-resolution images, using these lower
resolution images does not affect clinical outcomes. Ultraso-
nography has a sensitivity of 0.85 (95% CI 0.79–0.90) and a
specificity of 0.90 (95% CI 0.83–0.95). Graded compression
ultrasonography is used to identify the anteroposterior diam-
eter of the appendix. An easily compressible appendix <5 mm
in diameter generally rules out appendicitis. (See Schwartz
11th ed., p. 1333.)

Brunicardi_Ch30_p259-262.indd 262 30/06/22 11:25 AM


CHAPTER 31
Liver

1. With regard to hepatic anatomy, the falciform ligament Answer: C


divides the _______ from the _______: The falciform ligament divides the left lateral section from
A. caudate lobe, quadrate lobe the left medial section. The plane between the gallbladder
B. right lobe, left lobe fossa and the inferior vena cava (IVC)—referred to as Cantlie
C. left medial section, left lateral section line—divides the right and left lobes. The falciform ligament,
D. left medial section, right lobe along with the round, triangular, and coronary ligaments
may be divided in a bloodless plane during liver resection
(Figs. 31-1 through 31-3). (See Schwartz 11th ed., Figs. 31-1
through 31-3, p. 1347.)

Diaphragm

Right triangular
ligament

Left triangular
ligament

Falciform
ligament

Round
ligament

FIG. 31-1. Hepatic ligaments


suspending the liver to the diaphragm
and anterior abdominal wall.

Liver in situ

Gastrohepatic
ligament

Foramen of
Winslow Open hepato-
duodenal ligament

FIG. 31-2. In situ liver hilar anatomy


with hepatoduodenal and gastrohepatic
ligaments. Foramen of Winslow is depicted.
263

Brunicardi_Ch31_p263-278.indd 263 04/07/22 2:42 PM


264
Right lobe Left lobe

II

IVa III

VIII
VII
CHAPTER 31

IVb
V
VI
Liver

Right lobe Left lobe

V
IVb
VI

III

FIG. 31-3. Couinaud’s liver


VII I
segments (I through VIII) numbered
II
in a clockwise manner. The left lobe
includes segments II to IV, the right
lobe includes segments V to VIII, and
the caudate lobe is segment I. IVC =
inferior vena cava. IVC Caudate lobe

2. The most common variant of normal hepatic artery Answer: D


anatomy is: Understanding the anatomic variants of the hepatic arte-
A. Replaced left hepatic artery from the left gastric rial supply is important to avoid complications during liver
artery. ­surgery. The standard arterial anatomy is as follows: the com-
B. Completely replaced common hepatic artery from mon hepatic artery arises from the celiac trunk, and then
the superior mesenteric artery. divides into the gastroduodenal and proper hepatic artery. In a
C. Replaced right and left hepatic arteries. standard configuration, the proper hepatic artery gives rise to
D. Replaced right hepatic artery from the superior mes- the right gastric artery, but this is variable. The proper hepatic
enteric artery. artery then divides into the right and left hepatic artery. How-
ever, this standard arterial configuration only occurs in 76%
of patients. The most common variants include: replaced
or accessory right hepatic artery from the superior mesen-
teric artery (10%–15%), replaced left hepatic artery from the
left gastric artery (3%–10%), replaced right and left hepatic
arteries (1%–2%), and the completely replaced common
hepatic artery from the superior mesenteric artery (1%–2%)
(Figs. 31-4 and 31-5). (See Schwartz 11th ed., Figs. 31-4 and
31-5, p. 1349.)

Brunicardi_Ch31_p263-278.indd 264 04/07/22 2:42 PM


265

LHA

RHA Left gastric


Hepatic artery artery
proper Celiac trunk
Right gastric

CHAPTER 31
artery Splenic artery
Common hepatic artery
Gastroduodenal artery

Liver
FIG. 31-4. Arterial anatomy of the upper abdomen and liver, including the celiac trunk and
hepatic artery branches. a. = artery; LHA = left hepatic artery; RHA = right hepatic artery.

Replaced right hepatic Replaced left hepatic artery


artery from SMA (10%–15%) from left gastric artery (3%–10%)

Replaced right and replaced left Completely replaced common


hepatic arteries (1%–2%) hepatic artery from SMA (1%–2%)

FIG. 31-5. Common hepatic artery anatomic variants. SMA = superior mesenteric artery.

3. Which of the following correctly pairs the segments of Answer: C


the liver and their associated systemic venous drainage? There are three hepatic veins (right, middle, and left) that
A. Segments I, II, III: right hepatic vein serve as the outflow for the hepatic circulation and drain
B. Segment IV: right hepatic vein into the suprahepatic inferior vena cava (IVC). The right
C. Segment I: IVC hepatic vein drains segments V–VIII; the middle hepatic
D. Segments V, VI, VII, VIII: left hepatic vein vein drains segment IV, as well as segments V and VIII; and
the left hepatic vein drains segments II and III. The caudate
lobe (segment I) drains directly into the IVC (Fig. 31-6). (See
Schwartz 11th ed., Fig. 31-8, p. 1350.)

Brunicardi_Ch31_p263-278.indd 265 04/07/22 2:42 PM


266
IVC

IVC and 3 HVs

Right lobe Lateral


segment
structures
Posterior
segment Middle Left HV
CHAPTER 31

structures hepatic
v.
Right HV

Left lobe

Middle HV
Liver

Medial
segment
structures

Falciform
ligament

Hepatic a.
Anterior
segment Portal v.
structures

Gallbladder

FIG. 31-6. Confluence of the three hepatic veins (HVs) and the inferior vena cava (IVC). Note that the middle and left HVs drain
into a common trunk before entering the IVC. a. = artery; v. = vein. (Adapted with permission from Cameron JL: Atlas of Surgery.
Vol. I, Gallbladder and Biliary Tract, the Liver, Portasystemic Shunts, the Pancreas. Toronto: BC Decker; 1990.)

4. There is considerable variability in hepatic duct conflu- Answer: A


ence anatomy, in 60%–70% of cases what is the normal In general, the hepatic ducts follow the arterial branching
anatomy? pattern inside the liver. The right anterior hepatic duct usu-
A. Hepatic ducts follow arterial branching inside the ally enters the liver above the hilar plate, whereas the right
liver. posterior duct dives behind the right portal vein and can be
B. The right anterior hepatic duct enters the liver above found on the surface of the caudate process before entering
the hilar plate and the posterior duct enters segment the liver. The left hepatic duct typically has a longer extra-
V, lateral to the portal vein. hepatic course before giving off segmental branches behind
C. The left hepatic duct has a short extrahepatic course, the left portal vein at the base of the umbilical fissure.
coursing with the left portal vein. There is a nonstandard hepatic duct confluence with acces-
D. Hepatic ducts follow hepatic vein branching inside sory or aberrant ducts (Fig. 31-7). (See Schwartz 11th ed.,
the liver. Fig. 31-9, p. 1351.)

Brunicardi_Ch31_p263-278.indd 266 04/07/22 2:42 PM


267
ra ra
A: Normal bifurcation 57% lh lh
rp rp
B: Trifurcation of 3 ducts 12%

A57% B12%

ra ra
C: R anterior (C1, 16%) or R posterior lh
(C2, 4%) duct draining into CHD rp lh

CHAPTER 31
rp

C20% 16% 4%
C1 C2

ra ra

D: R posterior (D1, 5%) or R anterior rp

Liver
duct (D2, 1%) draining into the left rp lh
lh
hepatic duct
D6%
5% 1%
D1 D2
III
IV
ra IV III
E: Absence of hepatic duct ra
rp
confluence 3%
II
II
rp
E3% I
I
2% 1%
E1 E2

ra
F: Drainage of R posterior duct into rp
cystic duct 2% lh

F2%

FIG. 31-7. Main variations of hepatic duct confluence. As described by Couinaud in 1957, the bifurcation of
the hepatic ducts has a variable pattern in approximately 40% of cases. CHD = common hepatic duct; lh = left
hepatic; R = right; ra = right anterior; rp = right posterior. (Reproduced with permission from Blumgart LH, Fong Y:
Surgery of the Liver and Biliary Tract, 3rd ed, Vol. I. London: Elsevier; 2000.)

5. In a fasting state, how does the liver generate glucose? Answer: B


A. Early and persistent breakdown of glycogen storage The liver maintains glucose concentrations in a normal
B. Early glycogen breakdown through glycogenolysis range over both short and long periods by performing sev-
and later gluconeogenesis from lactate, amino acids, eral important roles in carbohydrate metabolism. In the fast-
and glycerol ing state, the liver ensures a sufficient supply of glucose to
C. Immediate gluconeogenesis from noncarbohy- the central nervous system. The liver can produce glucose
drate precursors including lactate, amino acids, and by breaking down glycogen through glycogenolysis and by
glycerol de novo synthesis of glucose through gluconeogenesis from
D. Protein catabolism through amino acid deamination noncarbohydrate precursors such as lactate, amino acids, and
glycerol. In the postprandial state, excess circulating glucose
is removed by glycogen synthesis or glycolysis and lipogen-
esis. (See Schwartz 11th ed., p. 1351.)

Brunicardi_Ch31_p263-278.indd 267 04/07/22 2:42 PM


268
6. What is the cause of steatorrhea in a patient with Crohn’s Answer: C
disease? Bile salts are sodium and potassium salts of bile acids derived
A. Excess excretion of bilirubin and cholesterol in stool from cholesterol by hepatocytes. After synthesis, the primary
B. Concurrent obstructive primary sclerosing bile acids cholic and chenodeoxycholic acid are conjugated
cholangitis to either taurine or glycine and then secreted into the ­biliary
C. Terminal ileum resections resulting in bile acid and system. Approximately 90%–95% of these primary bile salts
salts malabsorption and acids are absorbed by active transport at the terminal
D. Intestinal bacterial depletion resulting in inadequate ileum, a region commonly affected by Crohn disease, while
bilirubin conjugation the remainder enter the colon and are converted to second-
CHAPTER 31

ary bile acids (deoxycholic and lithocolic acids) and their


associated salts by resident bacteria. Bile acids and salts
­
­reabsorbed in the terminal ileum are reabsorbed through the
portal circulation, while those lost in the stool are replaced by
hepatic ­synthesis. The continuous process of secretion of bile
salts in the bile, their passage through the intestine, and their
subsequent return to the liver is termed the ­enterohepatic
Liver

­circulation. (See Schwartz 11th ed., p. 1352.)

7. In your patient taking 5 mg of warfarin daily for a pros- Answer: A


thetic mitral valve, how might metronidazole treatment Many factors can affect drug metabolism in the liver. When
for Clostridium difficile affect their international normal- the rate of metabolism of a drug is increased (ie, enzyme
ized ratio (INR)? induction), the duration of the drug action will decrease.
A. Increase the INR due to cytochrome P-450 isoform However, when the metabolism of a drug is decreased (ie,
inhibition. enzyme inhibition), then the drug will circulate for a longer
B. Increase the INR due to induction of P-450 isoform. period of time, potentially increasing the effect of the medica-
C. No change, metronidazole does not affect hepatic tion. Warfarin is a vitamin K antagonist and therefore reduces
metabolism of P-450 isoforms. hepatic synthesis of coagulation factors. As metronidazole
D. Reduction of INR due to alterations in intestinal inhibits the metabolism of warfarin, the anti-coagulation
microflora. effects are increased and the INR may rise. (See Schwartz
11th ed., p. 1352.)

8. Which of the following compounds is not synthesized Answer: B


predominantly by the liver? The liver is the largest gland in the body, and responsible for
A. Albumin synthesis of the majority of plasma proteins. The liver pro-
B. Factor VIII duces approximately 10 g of albumin per day, and albumin
C. Factor VII measurement can therefore be used as a surrogate for liver
D. Factor II synthetic function. This must be interpreted with caution, as
albumin levels can be influenced by a host of factors unrelated
to hepatic function, and albumin’s long half-life (15–20 days)
makes it a poor marker for acute hepatic dysfunction. Most
clotting factors are synthesized predominantly in the liver,
except for factor VIII. Due to this fact the prothrombin time
(PT) and international normalized ratio (INR) may also be
used as markers of hepatic synthetic function. However,
these too should be interpreted with caution as other condi-
tions, including vitamin K deficiency and warfarin use, may
­prolong a patient’s PT/INR. (See Schwartz 11th ed., p. 1353.)

9. What technique is the most sensitive for the identifica- Answer: A


tion of liver lesions? Imaging modalities have rapidly improved over the recent
A. Intraoperative ultrasound decades, and CT and MRI constitute the mainstays of diag-
B. Computed tomography (CT) with triple-phase nostic imaging for patients with hepatic pathology. While
contrast routine B-mode and Doppler ultrasound represent excellent
C. Magnetic resonance imaging (MRI) with gadoxetate- initial screening tests due to the fact that they are both widely
based contrast accessible and inexpensive, most patients with pathology
D. Abdominal ultrasound identified on ultrasound will require further imaging. When
characterizing liver lesions by CT scan, contrast enhancement
is necessary for a complete evaluation. Leveraging the dual
blood supply of the liver with the hemodynamics of hepatic

Brunicardi_Ch31_p263-278.indd 268 04/07/22 2:42 PM


269
tumors, radiologists are able to selectively enhance the liver
parenchyma or tumor during the portal venous or arterial
phases of the scan, respectively. Limitations of ultrasound
include incomplete imaging of the liver, most often at the
dome or beneath ribs on the surface, and incomplete visual-
ization of lesion boundaries. Moreover, obesity and overlying
bowel gas also can interfere with image quality. Thus, ultraso-
nographically detected masses usually require further evalu-
ation by other imaging modalities due to the lower sensitivity

CHAPTER 31
and specificity of ultrasound compared with CT and MRI.
MRI offers advantages over CT in allowing higher soft tissue
contrast and superior depiction of fluid-containing structures
while eliminating the need for ionizing radiation. Develop-
ment of specialized liver-specific contrast agents, including
ferumoxide and gadoxetate, has led to the development of
MRI that allows characterization of both hepatic structure

Liver
and function. While positron emission tomography/com-
puted tomography (PET/CT) is used frequently in the stag-
ing and follow-up of patients with metastatic liver lesions, its
role in the diagnosis and management of primary liver lesions
is uncertain. The reported sensitivity of fluorodeoxyglucose
(FDG)-PET/CT for the detection of hepatocellular carci-
noma is only 50%–65%. While this has been improved with
development of dual-tracer PET/CT, the clinical benefits of
this modality have yet to be clearly defined. Despite advances
in other imaging modalities, intraoperative liver ultrasound
remains the gold standard. This technique has the ability to
identify 20%–30% more lesions than preoperative imaging,
and it is estimated that approximately 50% of planned sur-
gical resections are influenced by information gleaned from
intraoperative ultrasound. (See Schwartz 11th ed., p. 1357.)

10. In a patient without prior liver disease, what is the most Answer: D
common cause of hepatic encephalopathy occurring Differences in etiology, management, and patient outcomes
within 26 weeks of severe liver injury? have been described for various regions of the globe. In the
A. Alcohol in developing nations. East and developing portions of the world, the most com-
B. Causes are unknown. mon causes of acute liver failure (ALF) are viral infections,
C. Hepatitis B, A, and E in the United States. primarily hepatitis B, A, and E. In these areas, there are a rela-
D. Drug-/toxin-induced (including acetaminophen) in tively small number of drug-induced cases. In contrast, 65%
the United States. of cases of ALF in the West are thought to be due to drugs
and toxins, with acetaminophen (paracetamol) being the
most common etiologic agent in the United States, ­Australia,
United Kingdom, and most of Europe. In France and Spain,
where acetaminophen sales are restricted, the rate of acet-
aminophen-induced ALF is quite low. Acetaminophen-
induced ALF is also uncommon in South America. The US
Acute Liver Failure Study Group identified several other
causes of ALF, including autoimmune hepatitis, hypoperfu-
sion of the liver (in cardiomyopathy or cardiogenic shock),
pregnancy-related conditions, and Wilson disease. Even with
exhaustive efforts to identify a cause, approximately 20% of all
cases of ALF remain indeterminate in origin. (See Schwartz
11th ed., p. 1361.)

Brunicardi_Ch31_p263-278.indd 269 04/07/22 2:42 PM


270
11. A patient presents with painless jaundice, and is found to Answer: D
have cirrhosis. They have no history of alcohol abuse, but Chronic hepatitis C infection is the most common cause of
do note a history of diabetes mellitus and pseudogout. chronic liver disease in the United States. Other etiologies
They also mention that multiple members of their fam- include alcohol abuse, nonalcoholic steatohepaitis, and auto-
ily have suffered from cirrhosis. What is the most likely immune diseases (primary biliary cirrhosis, primary scle-
etiology for their cirrhosis? rosing cholangitis, and autoimmune hepatitis). Hereditary
A. Alcohol abuse hemochromatosis is the most common metabolic cause of
B. Wilson disease cirrhosis, and should be suspected if a patient presents with
C. α1-antitrypsin deficiency skin hyperpigmentation, diabetes mellitus, pseudogout, car-
CHAPTER 31

D. Hemochromatosis diomyopathy, or a family history of cirrhosis. Elevated plasma


ferritin and increased iron saturation levels suggest the pres-
ence of iron overload, but these findings also can be seen
in other diseases of the liver. Confirmatory testing can be
achieved by means of genetic testing, liver biopsy, or by assess-
ing the response to phlebotomy. Other uncommon metabolic
disorders leading to cirrhosis include Wilson ­disease and α1-
Liver

antitrypsin deficiency. (See Schwartz 11th ed., p. 1364.)

12. Which of the following clinical observations are associ- Answer: D


ated with the correct pathophysiologic explanations of The clinical history associated with cirrhosis can include
cirrhosis? fatigue, anorexia, weight loss, jaundice, abdominal pain,
A. Spider angiomata, palmar erythema, caput medusae peripheral edema, ascites, gastrointestinal (GI) bleeding, and
due to portosystemic collateral formation in the set- hepatic encephalopathy. On physical examination, a number
ting of portal hypertension of findings have been described in patients with cirrhosis.
B. Gynecomastia, loss of chest and axillary hair, and tes- ­Spider angiomata and palmar erythema are believed to be
ticular atrophy from chronic spironolactone use caused by alterations in sex hormone metabolism. Finger club-
C. Abdominal wall hernias secondary to destruction of bing may be a consequence of hypoalbuminemia, while the
type IV collagen pathogenesis of white nail beds and Dupuytren contractures
D. Caput medusae and the presence of the Cruveilhier- are less well understood. Males may develop features of femi-
Baumgarten murmur from collaterals between the nization such as gynecomastia, loss of chest and axillary hair,
portal system and the remnant of the umbilical vein and testicular atrophy. Splenomegaly is common, whereas the
cirrhotic liver itself may be enlarged, normal sized, or small.
Ascites and pleural effusion can be seen with fluid accumula-
tion. Abdominal hernias are common with ascites and should
be electively repaired only in patients with well-compensated
cirrhosis; otherwise, the hernia should be repaired at the
time of or after hepatic transplantation. Portal hyperten-
sion can manifest as caput medusae and/or the presence of
the Cruveilhier-Baumgarten murmur, a venous hum that can
be ­auscultated in the epigastrium resulting from collaterals
between the portal system and the remnant of the umbilical
vein. (See Schwartz 11th ed., p. 1364.)

13. Clinically significant portal hypertension is evident Answer: C


when the _______ exceeds ______ mm Hg. Portal hypertension occurs when the pressure in the portal
A. wedged hepatic venous pressure; 10 system is increased due to factors that may be divided into
B. free hepatic venous pressure; 20 three categories. Presinusoidal causes of portal hyperten-
C. hepatic venous pressure gradient; 10 sion include sinistral/extrahepatic (splenic vein thrombosis,
D. hepatic venous pressure gradient; 20 splenomegaly, splenic AV fistula) and intrahepatic (schisto-
somiasis, congenital hepatic fibrosis, idiopathic portal fibro-
sis, myeloproliferative disorder, sarcoid, graft-versus-host
disease) etiologies. Sinusoidal portal hypertension is a conse-
quence of cirrhosis of any etiology. Postsinusoidal hyperten-
sion can also be divided into intrahepatic (vascular occlusive
disease) and posthepatic (Budd-Chiari, congestive heart
failure [CHF], inferior vena cava [IVC] webs) etiologies. In
evaluating patients with suspected portal hypertension, an
enlarged portal vein on routine abdominal ultrasonography
may suggest portal hypertension but this is not diagnostic.

Brunicardi_Ch31_p263-278.indd 270 04/07/22 2:42 PM


271
Doppler ultrasound allows identification of vascular occlusion
and the direction of portal venous flow. Computed tomogra-
phy (CT) and magnetic resonance angiography (MRA) are
useful for evaluating portal venous patency and anatomy.
The most accurate method for measuring portal hyperten-
sion is hepatic venography. This procedure introduces a bal-
loon catheter directly into the hepatic vein where free hepatic
venous pressure (FHVP) is measured. The hepatic vein is then
occluded by inflation of the balloon allowing measurement of

CHAPTER 31
the wedged hepatic venous pressure (WHVP). The hepatic
venous pressure gradient (HVPG) may then be calculated by
subtracting the FHVP from the WHVP (HVPG = WHVP −
FHVP). Clinically significant portal hypertension is defined
as HVPG > 10 mm Hg. (See Schwartz 11th ed., p. 1365.)

14. What is the leading cause of morbidity and mortality Answer: A

Liver
among patients with cirrhosis? The most significant manifestation and the leading cause
A. Portal hypertension associated variceal bleeding of morbidity and mortality related to portal hypertension is
B. Encephalopathy variceal bleeding. Approximately 30% of patients with com-
C. Development of hepatocellular carcinoma pensated cirrhosis and 60% of patients with decompensated
D. Hepatorenal syndrome cirrhosis have esophageal varices. One-third of all patients
with varices will experience variceal bleeding. Each episode
of bleeding is associated with a 20% to 30% risk of mortality.
If left untreated, 70% of patients who survive the initial bleed
will experience recurrent variceal hemorrhage within 2 years
of the index hemorrhage. (See Schwartz 11th ed., p. 1367.)

15. How should acute variceal hemorrhage be managed? Answer: C


A. Balanced blood product resuscitation supplemented Patients with acute variceal hemorrhage should be admitted
with recombinant factor VIIa to an ICU for resuscitation and management. Blood resusci-
B. Intravenous vasopressin and crystalloid infusion tation should be performed carefully to reach a hemoglobin
C. Emergent endoscopic evaluation and variceal level of approximately 8 g/dL. Overzealous replacement of
banding blood products and administration of saline can lead to both
D. Trans jugular intrahepatic portosystemic shunting rebleeding and increased mortality. Administration of fresh
(TIPS) followed by variceal banding frozen plasma and platelets can be considered in patients with
severe coagulopathy. Use of recombinant factor VIIa has not
been shown to be more beneficial than standard therapy and
therefore is not recommended at this time.
Vasoactive medications decrease blood flow to the gastro-
esophageal varices and can be initiated as soon as the diagno-
sis of variceal bleeding is made. Although vasopressin is the
most potent available vasoconstrictor, its use is limited by its
systemic vasoconstrictive effects that can produce hyperten-
sion, myocardial ischemia, arrhythmias, ischemic abdominal
pain, and limb gangrene. Octreotide, a somatostatin analog,
has the advantage that it can be administered for 5 days or
longer, and it is currently the preferred pharmacologic agent
for initial management of acute variceal bleeding. In addi-
tion to pharmacologic therapy, endoscopy with variceal band
ligation should be carried out as soon as possible. This com-
bination of pharmacologic and endoscopic therapy has been
shown both to improve the initial control of bleeding and to
increase the 5-day hemostasis rate. TIPS can be performed
in 95% of patients by an experienced interventional radiolo-
gist, controls variceal bleeding in >90% of cases refractory to
medical treatment, and should not affect subsequent hepatic
transplantation. (See Schwartz 11th ed., p 1367.)

Brunicardi_Ch31_p263-278.indd 271 04/07/22 2:42 PM


272
16. A 52-year-old man with no travel history, chronic pul- Answer: D
monary obstructive disease, and recurrent diverticulitis Pyogenic liver abscesses are the most common liver abscesses
presents with increasing right upper quadrant abdomi- seen in the United States. Patients commonly present with
nal pain, fevers, and jaundice. Computed tomography right upper quadrant pain and fever. Jaundice occurs in up to
(CT) imaging notes a single, 5 cm hypodense lesion with one-third of affected patients. They may be single or multiple
peripheral enhancement in the right lobe of the liver. and are more frequently found in the right lobe of the liver.
What is the best treatment for this patient? The abscess cavities are variable in size and, when multiple,
A. Metronidazole and percutaneous drainage may coalesce to give a honeycomb appearance. Approximately
B. Metronidazole 40% of abscesses are monomicrobial, an additional 40% are
CHAPTER 31

C. Mebendazole polymicrobial, and 20% are culture-negative. The most com-


D. Intravenous piperacillin/tazobactam and percutane- mon infecting agents are gram-negative bacteria. Escherichia
ous drainage coli is found in two-thirds of cases, and other common organ-
isms include Streptococcus faecalis, Klebsiella, and Proteus
vulgaris. Anaerobic organisms such as Bacteroides fragilis also
are seen frequently. In patients with endocarditis and infected
indwelling catheters, Staphylococcus and Streptococcus species
Liver

are more commonly found. Ultrasound examination of the


liver reveals pyogenic abscesses as round or oval hypoechoic
lesions with well-defined borders and a variable number of
internal echoes. CT scan is highly sensitive in the localiza-
tion of pyogenic liver abscesses, which appear hypodense
with peripheral enhancement and may contain air-fluid levels
indicating a gas-producing infectious organism.
The current cornerstones of treatment include correction
of the underlying cause and intravenous (IV) antibiotic ther-
apy. Empiric antibiotic therapy should cover gram-negative
and anaerobic organisms; percutaneous needle aspiration and
culture of the aspirate may be useful in guiding subsequent
antibiotic therapy. IV antibiotic therapy should be contin-
ued for at least 8 weeks and can be expected to be effective in
80% to 90% of patients. Placement of a percutaneous drain-
age catheter is beneficial only for a minority of patients, as
most pyogenic abscesses are quite viscous and catheter drain-
age is often ineffective (Fig. 31-8). (See Schwartz 11th ed.,
Figure 31-16, p. 1369.)

FIG. 31-8. Computed tomographic scan of pyogenic liver


abscesses. Multiple hepatic abscesses are seen in a patient after an
episode of diverticulitis. Note the loculated large central abscess as
well as the left lateral segment abscess.

Brunicardi_Ch31_p263-278.indd 272 04/07/22 2:42 PM


273
17. Upon return from New Zealand, a 33-year-old woman Answer: C
presents with dull right upper quadrant pain and fevers Hydatid disease is most common in sheep-raising areas,
diagnosed by the presence of echinococcal antigens. where dogs have access to infected offal. These include South
What does the computed tomography (CT) imaging Australia, New Zealand, Africa, Greece, Spain, and the Mid-
demonstrate. dle East. Hydatid cysts commonly involve the right lobe of
A. Multiple, well circumscribed lesions with homog- the liver, usually the anterior-inferior or posterior-inferior
enous arterial enhancement and central scar segments. Occasionally, the affected patient presents with
B. Well-defined low-density round lesions that have symptoms such as dull right upper quadrant pain or abdomi-
enhancement of the wall, somewhat ragged in nal distention.

CHAPTER 31
appearance with a peripheral zone of edema in the The diagnosis of hydatid disease is based on the findings of
left lobe an enzyme-linked immunosorbent assay (ELISA) for echino-
C. Anteroinferior hypodense lesion with a distinct wall coccal antigens, and results are positive in approximately 85%
and ring-like calcifications in the right lobe of infected patients. Ultrasonography and CT scanning of the
D. Asymmetrical nodular peripheral enhancement that abdomen are both quite sensitive for detecting hydatid cysts.
is isodense with large vessels and exhibit progressive The appearance of the cysts on images depends on the stage
centripetal enhancement fill-in over time in the right of cyst development. Typically, hydatid cysts are well-defined

Liver
lobe hypodense lesions with a distinct wall. Ring-like calcifica-
tions of the pericysts are present in 20% to 30% of cases. As
healing occurs, the entire cyst calcifies densely, and a lesion
with this appearance is usually dead or inactive. Daughter
cysts generally occur in a peripheral location within the main
cyst and are typically slightly hypodense compared with the
mother cyst. Magnetic resonance imaging (MRI) of the abdo-
men may be useful to evaluate the pericyst, cyst matrix, and
daughter cyst characteristics. (See Schwartz 11th ed., p. 1370.)

18. What is the most common complication following tran- Answer: A


sjugular intrahepatic portosystemic shunt (TIPS)? TIPS is a percutaneous procedure used for treatment of
A. Encephalopathy patients who have gastroesophageal varices in the setting of
B. Hepatic ischemia portal hypertension. It has largely replaced surgical portosys-
C. Infection temic shunts due to the fact that it is both safe and effective
D. Life-threatening hemorrhage while also providing a minimally invasive alternative to major
abdominal surgery. TIPS functions by creating an intrahe-
patic shunt between the portal and systemic circulation which
causes a reduction in the portal pressure and ultimately in the
blood flow through varices. It is accomplished by endovascu-
lar access through the jugular vein to a hepatic vein radical
and subsequent creation of a needle tract that connects it to a
branch of the portal vein. After dilation of the tract, a metallic
stent is deployed to hold the new portosystemic connection
open. Because this shunt reduces first pass metabolism of the
liver, the most common complication of TIPS is encephalopa-
thy which occurs in 25%–30% of patients. Other complica-
tions such as hepatic ischemia, infection, renal failure and
hemorrhage may occur, but are rare. (See Schwartz 11th ed.,
p. 1367.)

Brunicardi_Ch31_p263-278.indd 273 04/07/22 2:42 PM


274
19. A patient presents to your clinic for follow up of his Answer: D
Hepatitis C cirrhosis with mild ascites, an albumin of The MELD is a linear regression model based on three objec-
2.9 g/dL, creatinine of 2.1 mg/dL, sodium of 121 mEq/L, tive laboratory values (INR, bilirubin level, and creatinine
international normalized ratio (INR) of 1.6, and bili- level). It was originally developed as a tool to predict mortal-
rubin of 1.9 mg/dL and complete resolution of prior ity after transjugular intrahepatic portosystemic shunt (TIPS)
hepatic encephalopathy. but has been validated and used as the sole method of liver
A. Based upon his clinical presentation and labora- transplant allocation in the United States since 2002. The
tory findings, his Model for End-Stage Liver disease MELD formula accounts for objective valuation creatinine,
(MELD) score is 26. INR, bilirubin, creatinine with serum sodium subsequently
CHAPTER 31

B. Summation of his creatinine, INR, and bilirubin indi- added to improve sensitivity and specificity.
cates he has a Child-Pugh Score in Class B. The Child-Turcotte-Pugh (CTP) score was originally
C. Based upon clinical and laboratory values, he has a developed to evaluate the risk of portocaval shunt procedures
Child-Pugh Score in Class A. performed for portal hypertension and subsequently has
D. Formulation of his laboratory values indicate his been shown to be useful in predicting surgical risks of other
MELD is 29. intra-abdominal operations on cirrhotic. Numerous studies
have demonstrated overall surgical mortality rates of 10%
Liver

for patients with class A cirrhosis, 30% for those with class
B cirrhosis, and 75% to 80% for those with class C cirrhosis.
Ranging from 5 to 15, the CTP is calculated based upon a
summation of points derived from the presence of ascites and
encephalopathy, serum bilirubin and albumin levels, as well
as INR. Multiple retrospective studies have demonstrated
that perioperative mortality and morbidity rates correlate
well with the CTP score, and for over 30 years, this measure
had been used as the principal predictor of operative risk
(Table 31-1). (See Schwartz 11th ed., Table 31-4, p. 1365.)

TABLE 31-1 Child-Turcotte-Pugh (CTP) score


Variable 1 Point 2 Points 3 Points
Bilirubin level < 2 mg/dL 2–3 mg/dL > 3 mg/dL
Albumin level > 3.5 g/dL 2.8–3.5 g/dL < 2.8 g/dL
International normalized ratio < 1.7 1.7–2.2 > 2.2
Encephalopathy None Controlled Uncontrolled
Ascites None Controlled Uncontrolled
Child-Turcotte-Pugh class
Class A = 5–6 points
Class B = 7–9 points
Class C = 10–15 points

20. The most common benign hepatic lesion is the: Answer : B


A. Hemangioma. While hemangiomas are the most common solid benign
B. Simple cyst. masses found in the liver, the simple hepatic cyst is still the
C. Adenoma. most common overall. Simple cysts have a prevalence of
D. Bile duct hamartoma. approximately 2.8%–3.6%, and are more common in women
by a ratio of 4:1. Cysts are generally found incidentally dur-
ing abdominal imaging, and small, asymptomatic cysts may
be managed conservatively. Large cysts may begin to cause
abdominal pain, epigastric fullness and early satiety. These
patients may be treated with percutaneous cyst aspiration
and sclerotherapy which is effective in approximately 90%
of patients. For those that fail percutaneous treatment, or
where percutaneous treatment is not available, surgical cyst
fenestration may be considered. If surgical fenestration is per-
formed, the cyst wall should be sent for pathologic analysis to
exclude carcinoma. (See Schwartz 11th ed., p. 1373.)

Brunicardi_Ch31_p263-278.indd 274 04/07/22 2:42 PM


275
21. Which of the following liver lesions carry a significant Answer: C
risk of spontaneous rupture? Hemangiomas are congenital vascular lesions that may range
A. Hemangioma in size from <1 cm to ≥25 cm. They are predominantly found
B. Hepatic cyst in women, and are generally asymptomatic. Large lesions
C. Adenoma may result in discomfort from compression of nearby organs.
D. Bile duct hamartoma Though hemangiomas are at risk for bleeding if they are biop-
sied, spontaneous rupture is rare. Adenomas, on the other
hand, carry a significant risk for spontaneous rupture with
intraperitoneal bleeding. For this reason, along with their

CHAPTER 31
potential for malignant degeneration, it is generally recom-
mended that hepatic adenomas be resected once discovered.
(See Schwartz 11th ed., p. 1375.)

22. A patient presents with results from a contrast-enhanced Answer: A


computed tomography (CT) scan that describe a well- On contrast-enhanced imaging, an focal nodular hyperpla-
circumscribed lesion that demonstrates homogenous sia (FNH) can be recognized as a well-circumscribed mass

Liver
enhancement during arterial phase, isodensity on the that demonstrates enhancement on the arterial phase and
venous phase, and a central scar. In general, what would isointensity on the venous phase. FNH also demonstrates a
be the recommended treatment? characteristic central scar. FNH are solid benign lesions are
A. Reassurance and observation similar to adenomas in that they are more common in women
B. Percutaneous radio frequency ablation of childbearing age. Unlike adenomas, however, they are not
C. Resection prone to malignant degeneration or spontaneous rupture. For
D. Transarterial chemoembolization this reason, asymptomatic FNHs may be managed conserva-
tively unless adenoma or hepatocellular carcinoma (HCC)
cannot be definitively excluded. Gadolinium-enhanced mag-
netic resonance imaging (MRI) may allow better visualization
of the fibrous septa extending from the FNH’s central scar.
While FNH and adenomas ma appear similar on computed
tomography (CT) or standard MRI, new MRI contrast agents
like gadobenate dimeglumine (MultiHance) allow supe-
rior discrimination between these two lesions (Fig. 31-9).
(See Schwartz 11th, Fig. 31-18, p. 1375.)

FIG. 31-9. Computed


tomographic scans showing
classic appearance of
benign liver lesions. Focal
nodular hyperplasia (FNH)
is hypervascular on arterial
phase, isodense to liver on
venous phase, and has a
central scar (upper panels).
Adenoma is hypovascular
(lower left panel). Hemangioma
shows asymmetrical
peripheral enhancement
(lower right panel).

Brunicardi_Ch31_p263-278.indd 275 04/07/22 2:42 PM


276
23. What is the annual conversion rate to hepatocellular Answer: C
­carcinoma (HCC) for patients with cirrhosis? HCC is the fifth most common malignancy worldwide, and
A. Less than 1% its risk factors include viral hepatitis, alcoholic cirrhosis,
B. 1%–2% hemochromatosis, and NASH. Cirrhosis is present in 70% to
C. 2%–6% 90% of patients who develop HCC, and the annual conver-
D. 6%–10% sion rate from cirrhosis is 2% to 6%. (See Schwartz 11th ed.,
p. 1376.)

24. Which of the following is TRUE in regard to patients Answer: C


CHAPTER 31

with cholangiocarcinoma? Cholangiocarcinoma, or bile duct cancer, is the second most


A. Surgical resection is the treatment of choice for common primary malignancy of the liver. Cholangiocarci-
patients with primary sclerosing cholangitis and noma is an adenocarcinoma of the bile ducts; it forms in the
cholangiocarcinoma limited to the common bile biliary epithelial cells and can be subclassified into peripheral
duct. (intrahepatic) bile duct cancer and central (extrahepatic) bile
B. Cholangiocarcinoma is very radiosensitive, there- duct cancer. Extrahepatic bile duct cancer can be located dis-
fore external-beam radiation and intravenous tally or proximally. When proximal, it is referred to as a hilar
Liver

5-­fluorouracil constitute destination therapy with cholangiocarcinoma (Klatskin’s tumor).


60% 5-year survival. In the absence of associated primary sclerosing cholan-
C. 5-year survival rate following the Mayo Clinic proto- gitis (PSC), surgical resection is the treatment of choice for
col and transplantation is 70%. hilar cholangiocarcinoma. However, approximately 10% of
D. Patients with primary sclerosing cholangitis are not patients with cholangiocarcinoma have PSC. Furthermore,
candidates for the Mayo Clinic protocol. cholangiocarcinoma in the setting of PSC is frequently mul-
ticentric and often is associated with underlying liver disease,
with eventual cirrhosis and portal hypertension. As a result,
experience has shown that resection of cholangiocarcinoma
in patients with PSC yields dismal results.
The pretransplant Mayo protocol consists of external-
beam radiation therapy plus a protracted course of intrave-
nous 5-fluorouracil followed by iridium-192 brachytherapy.
Patients then undergo an abdominal exploration with stag-
ing. If findings are negative, patients are given capecitabine
for 2 of every 3 weeks until transplantation. Even after restag-
ing with CT/MRI and endoscopic ultrasonography, approxi-
mately 15% to 20% of patients will have positive findings for
tumor on abdominal exploration. The 5-year survival rate for
those undergoing transplantation for cholangiocarcinoma at
the Mayo Clinic is approximately 70% and compares favor-
ably with the rate for resection. Current eligibility criteria for
this Mayo Clinic protocol include unresectable hilar cholan-
giocarcinoma or hilar cholangiocarcinoma with PSC. The
tumor must have a radial dimension of ≤3 cm with no intra-
hepatic or extrahepatic metastases, and the patient must not
have undergone prior radiation therapy or transperitoneal
biopsy. Many centers have adopted similar protocols with
comparable results. (See Schwartz 11th ed., p. 1377.)

25. A patient undergoes routine cholecystectomy and is Answer: A


incidentally found to have gallbladder carcinoma with- Gallbladder cancer is a rare and aggressive form of biliary
out invasion into the muscular layer. Further treatment malignancy. In approximately one-third of cases it is diag-
should include: nosed incidentally following routine cholecystectomy. Treat-
A. No further treatment. ment for these patients is guided by T stage of the tumor. In
B. External beam radiation with systemic chemotherapy. those patients with T1a tumors, as in this question, no further
C. Reoperation with a segment IVb/V resection and treatment is necessary. If the tumor invades into the muscular
hilar lymphadenectomy. layer (T1b), reoperation with central liver resection and hilar
D. Reoperation with formal lobectomy and bile duct lymphadenectomy is recommended. The role for more radi-
resection. cal resections is unclear. (See Schwartz 11th ed., p. 1378).

Brunicardi_Ch31_p263-278.indd 276 04/07/22 2:42 PM


277
26. Which of the following is considered a primary determi- Answer: C
nant of suitability for resection when evaluating a patient The liver is a common site for metastatic disease in patients
with hepatic colorectal metastases? with colorectal disease, and approximately 50% to 60% of
A. Number of metastatic tumors patients diagnosed with colorectal cancer will develop liver
B. Size of metastatic tumors metastases within their lifetime. With the advent of more
C. Predicted volume of hepatic remnant aggressive strategies for the management of metastatic colorec-
D. Prior therapy tal cancer, including improved chemotherapeutic regimens
and expanded use of metastasectomy, the 5-year survival for
patients with isolated metastases to the liver may exceed 30%.

CHAPTER 31
Given these encouraging results, the paradigm for surgical
evaluation and treatment of these patients has shifted to pri-
marily consider the health of the background liver and volume
of the hepatic remnant, and not tumor characteristics like size
and number. (See Schwartz 11th ed., p. 1378.)

27. Based on the standard Milan criteria, which of the fol- Answer: B

Liver
lowing patients with hepatocellular carcinoma (HCC) Orthotopic liver transplantation (OTL) was first attempted in
would be eligible for transplantation? the 1980s and 1990s, with initial series reporting 5-year sur-
A. One 4.5 cm lesion in segment VI with invasion of the vival rates of 20% to 50%. This led to the introduction of the
right portal vein Milan criteria which limited eligibility to patients with one
B. Three lesions confined to the right lobe, with the tumor <5 cm or up to three tumors <3 cm and no evidence of
largest being 2.5 cm gross intravascular or extrahepatic spread. Adoption of these
C. A single, 5.5 cm lesion in segment II guidelines resulted in significant improvement in 5-year sur-
D. Three lesions spread throughout the liver, with the vival for patients with HCC treated with OTL. (See Schwartz
largest being 3.5 cm 11th ed., p. 1380.)

28. The only FDA-approved systemic chemotherapeutic Answer: D


agent for hepatocellular carcinoma (HCC) is: Though systemic chemotherapy has not proven very effective
A. Epirubicin. in the treatment of HCC, the multikinase inhibitor sorafenib
B. Cisplatin. has been approved for use specifically in these patients. Based
C. 5-fluorouracil. on results of the SHARP trial, there sorafenib demonstrated
D. Sorafenib. a 3-month survival benefit versus placebo. Though these
results are modest, it remains a treatment option for patients
with advanced, unresectable HCC. (See Schwartz 11th ed.,
p. 1381.)

Brunicardi_Ch31_p263-278.indd 277 04/07/22 2:42 PM


This page intentionally left blank

Brunicardi_Ch31_p263-278.indd 278 04/07/22 2:42 PM


CHAPTER 32
The Gallbladder and Extrahepatic Biliary System

1. The arterial supply of the common bile duct is derived Answer: D


from: The majority of the blood flow to the human common bile
A. The left hepatic artery. duct originates from the right hepatic artery and gastro duo-
B. The right hepatic artery. denal arteries, with major trunks running along the medial
C. The gastroduodenal artery. and lateral aspects of the common duct (often referred to as
D. The right hepatic and gastroduodenal arteries. the 3 o’clock and 9 o’clock positions). (See Schwartz 11th ed.,
E. The left hepatic and gastroduodenal arteries. p. 1396.)

2. Relaxation of the sphincter of Oddi in response to a meal Answer: B


is largely under the control of which hormone? The sphincter of Oddi is a complex structure that is function-
A. Gastrin ally independent from the duodenal musculature and creates
B. Cholecystokinin (CCK) a high-pressure zone between the bile duct and the duode-
C. Motilin num. The sphincter of Oddi is about 4 to 6 mm in length
D. Secertin and has a basal resting pressure of about 13 mm Hg above
E. Ghrelin the duodenal pressure. On manometry, the sphincter shows
phasic contractions with a frequency of about four per min-
ute and an amplitude of 12 to 140 mm Hg. The spontaneous
motility of the sphincter of Oddi is regulated by the intersti-
tial cells of Cajal through intrinsic and extrinsic inputs from
hormones and neurons acting on the smooth muscle cells.
Relaxation occurs with a rise in CCK, leading to diminished
amplitude of phasic contractions and reduced basal pressure,
allowing increased flow of bile into the duodenum. During
fasting, the sphincter of Oddi activity is coordinated with the
periodic partial gallbladder emptying and an increase in bile
flow that occurs during phase II of the migrating myoelectric
motor complexes. (See Schwartz 11th ed., pp. 1397–1398.)

3. Acute cholecystitis is considered: Answer: C


A. A primary infectious process with secondary Obstruction of the cystic duct by a gallstone is the initiating
inflammation. event that leads to gallbladder distention, inflammation, and
B. A sterile primary inflammatory process. edema of the gallbladder wall. Why inflammation develops
C. A primary inflammatory process with occasional only occasionally with cystic duct obstruction is unknown. It
bacterial contamination. is probably related to the duration of obstruction of the cystic
D. A primary autoimmune process. duct. Initially, acute cholecystitis is an inflammatory process,
probably mediated by the mucosal toxin lysolecithin, a prod-
uct of lecithin, as well as bile salts and platelet-activating factor.
Increase in prostaglandin synthesis amplifies the inflammatory
response. Secondary bacterial contamination is documented
in 15% to 30% of patients undergoing cholecystectomy for
acute uncomplicated cholecystitis. In acute cholecystitis, the
gallbladder wall becomes grossly thickened and reddish with
subserosal hemorrhages. Pericholecystic fluid often is present.
279

Brunicardi_Ch32_p279-288.indd 279 04/07/22 5:43 PM


280
The mucosa may show hyperemia and patchy necrosis. In
severe cases, about 5% to 10%, the inflammatory process pro-
gresses and leads to ischemia and necrosis of the gallblad-
der wall. More frequently, the gallstone is dislodged and the
inflammation resolves. (See Schwartz 11th ed., p. 1405.)

4. Appropriate management of a patient with cirrhosis sec- Answer: C


ondary to sclerosing cholangitis includes: Sclerosing cholangitis (primary or secondary) is an uncom-
A. Systemic immunosuppression with corticosteroids mon disease characterized by inflammatory strictures involv-
CHAPTER 32

and calcineurin inhibitors. ing the intrahepatic and extrahepatic biliary tree. It is a
B. Anti-TNF monoclonal antibodies (infliximab). progressive disease which can lead to biliary cirrhosis. Medi-
C. Consideration for transplantation. cal therapy has long been attempted with immunosuppres-
D. Ursodeoxycholic acid. sants, antibiotics, steroids, and ursodeoxycholic acid, and has
E. Excision of the extrahepatic biliary tree (Kasai been disappointing. Surgical management with resection of
procedure). the extrahepatic biliary tree and hepaticojejunostomy has
produced reasonable results in patients with extrahepatic
The Gallbladder and Extrahepatic Biliary System

and bifurcation strictures, but without cirrhosis or significant


hepatic fibrosis. In patients with sclerosing cholangitis and
advanced liver disease, liver transplantation is the only option.
It offers excellent results, with overall 5-year survival as high
as 85%. Primary sclerosing cholangitis recurs in 10% to 20%
of patients and may require retransplantation. (See Schwartz
11th ed., p. 1417.)

5. Over a 10-year period, what percentage of patients with Answer: D


asymptomatic gallstones will remain symptom-free? Gallstones in patients without biliary symptoms are com-
A. 10% monly diagnosed incidentally on ultrasonography, computed
B. 25% tomography (CT) scans, or abdominal radiography or at
C. 50% laparotomy. Several studies have examined the likelihood of
D. 66% developing biliary colic or developing significant complica-
E. 90% tions of gallstone disease. Approximately 3% of asymptomatic
individuals become symptomatic per year (ie, develop bili-
ary colic). Once symptomatic, patients tend to have recurring
bouts of biliary colic. Complicated gallstone disease develops
in 3% to 5% of symptomatic patients per year. Over a 20-year
period, about two-thirds of asymptomatic patients with gall-
stones remain symptom-free. (See Swartz 11th ed., p. 1401.)

6. The most common type of gallbladder cancer is: Answer: B


A. Oat cell. Between 80% and 90% of the gallbladder tumors are adeno-
B. Adenocarcinoma. carcinomas. Squamous cell, adenosquamous, oat cell, and
C. Adenosquamous. other anaplastic lesions occur rarely. The histologic subtypes
D. Anaplastic. of gallbladder adenocarcinomas include papillary, nodular,
E. Squamous cell. and tubular. Less than 10% are of the papillary type, but these
are associated with an overall better outcome, as they are
most commonly diagnosed while localized to the gallbladder.
(See Schwartz 11th ed., p. 1421.)

7. The gallbladder lymphatics drain into which of the Answer: C


­following liver segments? Lymphatic flow from the gallbladder drains first to the c­ ystic
A. III and IV duct node (Calot’s), then the pericholedochal and hilar nodes,
B. V and VI and finally the peripancreatic, duodenal, periportal, celiac,
C. IV and V and superior mesenteric artery nodes. The gallbladder veins
D. III and V drain directly into the adjacent liver, usually segments IV and
E. I and IV V, where tumor invasion is common. (See Schwartz 11th ed.,
p. 1421.)

Brunicardi_Ch32_p279-288.indd 280 04/07/22 5:43 PM


281
8. Adequate treatment for a gallbladder lesion involving the Answer: D
lamina propria of the gallbladder includes: Tumors limited to the muscular layer of the gallbladder (T1)
A. Cholecystectomy followed by adjuvant chemotherapy. are usually identified incidentally, after cholecystectomy for
B. Neoadjuvant chemo-XRT followed by surgical gallstone disease. There is near universal agreement that
resection. simple cholecystectomy is an adequate treatment for T1
C. Segemental liver resection and lymphadenectomy lesions and results in a near 100% overall 5-year survival rate.
alone. (See Schwartz 11th ed., p. 1422.)
D. Cholecystectomy alone.
E. Extended right hepatectomy.

CHAPTER 32
9. The best initial imaging test for evaluating suspected Answer: C
cholangiocarcinoma includes: The initial tests are usually ultrasound or computed tomog-
A. Percutaneous cholangiography. raphy (CT) scan. A perihilar tumor causes dilatation of the
B. Endoscopic retrograde cholangiopancreatography. intrahepatic biliary tree, but normal or collapsed gallbladder
C. Ultrasound. and extrahepatic bile ducts distal to the tumor. Distal bile
D. Magnetic resonance cholangiopancreatography duct cancer leads to dilatation of the extra- and intrahepatic

The Gallbladder and Extrahepatic Biliary System


(MRCP). bile ducts as well as the gallbladder. Ultrasound can establish
E. Hepatobiliary iminodiacetic acid (HIDA) scan. the level of obstruction and rule out the presence of bile duct
stones as the cause of the obstructive jaundice. It is usually dif-
ficult to visualize the tumor itself on ultrasound or on a stan-
dard CT scan. Either ultrasound or spiral CT can be used to
determine portal vein patency. The biliary anatomy is defined
by cholangiography. Papillary thyroid cancer (PTC) defines
the proximal extent of the tumor, which is the most important
factor in determining resectability. Endoscopic retrograde
cholangiopancreatography (ERCP) is used, particularly in
the evaluation of distal bile duct tumors. For the evaluation
of vascular involvement, celiac angiography may be necessary.
With the newer types of magnetic resonance imaging (MRI), a
single noninvasive test has the potential of evaluating the bili-
ary anatomy, lymph nodes, and vascular involvement, as well
as the tumor growth itself. (See Schwartz 11th ed., p. 1423.)

10. Patients with a history of choledochal cysts are at Answer: E


increased risk of developing biliary cancer: Patients with choledochal cysts have an increased risk of
A. In the gallbladder alone. developing cancer anywhere in the biliary tree, but the inci-
B. Predominantly intra-hepatic portions of the biliary dence is highest in the gallbladder. Sclerosing cholangitis,
tree. anomalous pancreaticobiliary duct junction, and exposure
C. In the distal common bile duct. to carcinogens (azotoluene, nitrosamines) also are associ-
D. At the site of the previous cyst. ated with cancer of the gallbladder. (See Schwartz 11th ed.,
E. Throughout the biliary tree. p. 1417.)

11. What is the best initial management for an intraopera- Answer: A


tively identified minor lateral injury to the common bile Lateral injury to the common bile duct or the common
duct? hepatic duct, recognized at the time of surgery, is best man-
A. Placement of a T-tube through the site of injury in aged with a T-tube placement. If the injury is a small incision
the duct in the duct, the T-tube may be placed through it as if it were
B. Primary oversew of the injury intra-operatively a formal ­choledochotomy. In more extensive lateral injuries,
C. Resection of the injured portion of the duct with end- the T-tube should be placed through a separate choledochot-
to-end anastomosis omy and the injury closed over the T-tube end to minimize
D. Intraoperative placement of endoscopic biliary stent the risk of s­ubsequent stricture formation. (See Schwartz
E. Resection of the bile duct and a roux-en-Y hepatico- 11th ed., pp. 1419–1420.)
jejunostomy reconstruction

Brunicardi_Ch32_p279-288.indd 281 04/07/22 5:43 PM


282
12. After identification of a postoperative biliary stricture, Answer: C
what is the best initial management? Patients with bile duct stricture from an injury or as a sequela
A. Operative resection of the involved biliary segment of previous repair usually present with either progressive
and reconstruction with an end-to-end roux-en-Y elevation of liver function tests or cholangitis. The initial
hepatico-jejunostomy management usually includes transhepatic biliary drainage
B. Endoscopic retrograde cholangiopancreatography catheter placement for decompression as well as for defining
(ERCP) with sphincterotomy and stenting of the the anatomy and the location and the extent of the damage.
pancreatic duct These catheters will also serve as useful technical aids dur-
C. Transhepatic catheter placement for biliary ing subsequent biliary enteric anastomosis. An anastomosis
CHAPTER 32

decompression is performed between the duct proximal to the injury and a


D. Hepatobiliary iminodiacetic acid (HIDA) scan Roux loop of jejunum. Balloon dilatation of a stricture usually
E. Operative placement of a T-tube at site of biliary requires multiple attempts and rarely provides adequate long-
stricture term relief. Self-expanding metal or plastic stents, placed
either percutaneously or endoscopically across the stricture,
can provide temporary drainage and, in the high-risk patient,
permanent drainage of the biliary tree. (See Schwartz 11th ed.,
The Gallbladder and Extrahepatic Biliary System

pp. 1420–1421.)

13. In the early postoperative period, what is the most com- Answer: D
mon presentation of a patient with a biliary injury? In the early postoperative period, patients present either
A. Fever with progressive elevation of liver function tests due to an
B. Abdominal pain occluded or a stenosed bile duct, or with a bile leak from an
C. Steatorrhea injured duct. (See Schwartz 11th ed., pp. 1419–1420.)
D. Elevated transaminases
E. Nausea

14. The gallbladder wall layers include: Answer: A


A. Mucosa, muscularis, adventitia, serosa. The gallbladder differs histologically from the rest of the gas-
B. Mucosa, submucosa, muscularis, adventitia, serosa. trointestinal (GI) tract in that it lacks a muscularis mucosa
C. Mucosa, submucosa, muscularis mucosa, muscularis, and submucosa. The muscular layer has circular, longitu-
adventitia, serosa. dinal, and oblique fibers, but without well-defined layers.
D. Mucosa, muscularis mucosa, muscularis, adventitia, The adventitia contains connective tissue, nerves, vessels,
serosa. lymphatics, and adipocytes. The gallbladder is covered by
serosa except where the gallbladder is embedded in the liver.
(See Schwartz 11th ed., p. 1393.)

15. The most commonly encountered arterial anomaly seen Answer: D


related to the gallbladder is: Anomalies of the hepatic artery and the cystic artery are quite
A. Replaced cystic artery. common, occurring in as many as 50% of cases. While the
B. Duplicated cystic artery. right hepatic artery usually originates from the proper hepatic
C. Accessory right hepatic artery. branch of the celiac trunk, up to 20% of patients will have a
D. Replaced right hepatic artery. replaced right hepatic artery coming off the superior mesen-
teric artery. In about 5% of cases, there are two right hepatic
arteries, one from the proper hepatic artery and the other
from the superior mesenteric artery (accessory right hepatic
artery). While the right hepatic artery typically runs posterior
to the bile ducts, variations may allow it to course anterior to
the common duct, making it vulnerable during surgical pro-
cedures, particularly if it runs parallel to the cystic duct or in
the mesentery of the gallbladder. The cystic artery arises from
the right hepatic artery in about 90% of cases, but it may arise
from the left hepatic, common hepatic, gastroduodenal, or
superior mesenteric arteries. (See Schwartz 11th ed., p. 1396.)

Brunicardi_Ch32_p279-288.indd 282 04/07/22 5:43 PM


283
16. ___% of bile acids are reabsorbed in the body with the Answer: A
majority of absorption occurring within the ___. About 80% of the secreted conjugated bile acids are reab-
A. 95; ileum and colon sorbed in the terminal ileum. The remainder is dehydroxyl-
B. 95; duodenum and ileum ated (deconjugated) by gut bacteria, forming the secondary
C. 80; ileum and colon bile acids deoxycholate and lithocholate. These are absorbed
D. 80; duodenum and ileum in the colon and can then be transported back to the liver.
Eventually, about 95% of the bile acid pool is reabsorbed,
the so-called enterohepatic circulation. Only a small amount
(5%) is excreted in the stool, allowing the relatively small

CHAPTER 32
quantity of bile acids produced to have maximal effect. (See
Schwartz 11th ed., Chapter 32, pp. 1396–1397.)

17. What constitutes as normal filling times in a fasting sub- Answer: D


ject when utilizing a hepatobiliary iminodiacetic acid 99m
Technetium-labeled derivatives of iminodiacetic acid are
(HIDA) scan? injected intravenously, taken up by the Kupffer cells in the
A. Liver uptake within 15 minutes, and gallbladder, bile liver, and excreted in the bile. Uptake by the liver is usually

The Gallbladder and Extrahepatic Biliary System


duct, and duodenum uptake within 60 minutes detected within 10 minutes, and the gallbladder, bile ducts,
B. Liver uptake within 15 minutes, and gallbladder, bile and duodenum are typically visualized within 60 minutes in
duct, and duodenum uptake within 45 minutes fasting subjects. (See Schwartz 11th ed., p. 1399.)
C. Liver uptake within 10 minutes, and gallbladder, bile
duct, and duodenum uptake within 45 minutes
D. Liver uptake within 10 minutes, and gallbladder, bile
duct, and duodenum uptake within 60 minutes

18. A 36-year-old woman presents with several days of right Answer: C


upper quadrant (RUQ) abdominal pain; which of the The primary use of biliary scintigraphy is in the diagnosis of
following findings would best support a diagnosis acute acute cholecystitis, which appears as a nonvisualized gallblad-
cholecystitis? der, with prompt filling of the common bile duct and duode-
A. Cholelithiasis on ultrasound with a 2 mm gallbladder num. The lack of gallbladder filling is due to inflammatory
wall closure of the cystic duct preventing bile backflow into the
B. Delayed gallbladder filling on hepatobiliary gallbladder (Fig. 32-1). Evidence of cystic duct obstruction on
­iminodiacetic acid (HIDA) biliary scintigraphy is highly diagnostic for acute cholecysti-
C. Lack of gallbladder filling on HIDA tis. The sensitivity and specificity for the diagnosis are about
D. Decreased ejection fraction (EF) on HIDA 95% each. (See Schwartz 11th ed., p. 1399.)

A B

FIG. 32-1. HIDA scanning. A. Normal HIDA scan showing filling of the extrahepatic
biliary tree and gallbladder (white arrow). B. HIDA scan in a patient with acute
cholecystitis showing no filling of the gallbladder.

Brunicardi_Ch32_p279-288.indd 283 04/07/22 5:43 PM


284
19. A 26-year-old who is 25 weeks pregnant presents to the Answer: C
emergency department (ED) with several days of right Pregnant women with symptomatic gallstones who cannot
upper quadrant (RUQ) associated with meals and ultra- be managed expectantly with diet modifications can safely
sound (US) concerning for acute cholecystitis, what is undergo laparoscopic cholecystectomy. The operation should
the best plan to proceed? be performed during the second trimester if possible. (See
A. Observation with antibiotics, elective cholecystec- Schwartz 11th ed., 1405.)
tomy after delivery
B. Antibiotics and supportive care
C. Antibiotics and laparoscopic cholecystectomy this
CHAPTER 32

admission
D. Antibiotics and laparoscopic cholecystectomy after
28 weeks of pregnancy

20. A 76-year-old man who is medically unfit for surgery Answer: D


presents to the emergency department (ED) with right When patients are medically unfit for surgery due to the
upper quadrant (RUQ) tenderness and ultrasound (US) severity of their illness or medical comorbidities, they can
The Gallbladder and Extrahepatic Biliary System

findings concerning for acute cholecystitis, what is the be treated with antibiotics and biliary decompression with
best course of action? cholecystostomy tube placement, which is usually effective in
A. Antibiotics, endoscopic retrograde cholangiopancrea- stabilizing the patient. For those who do recover after chole-
tography (ERCP), lap chole when stable this admission cystostomy, the tube can be removed once the track is mature
B. Antibiotics, lap chole when stable this admission (approximately 4 weeks) and cholangiography through it
C. Antibiotics, cholecystostomy tube, lap chole this shows a patent cystic duct. Elective laparoscopic cholecystec-
admission tomy can be scheduled within approximately 6 to 8 weeks,
D. Antibiotics, cholecystostomy tube, lap chole in 6 to assuming their medical fitness recovers. Failure to improve
8 weeks if able after cholecystostomy may be due to gangrene of the gallblad-
der or perforation, in which case, damage control surgery
may be unavoidable. (See Schwartz 11th ed., p. 1406.)

21. Charcot triad is defined as which of the following? Answer: A


A. Fever, pain, jaundice The most common presentation is fever, epigastric or right
B. Fever, pain, shock upper quadrant pain, and jaundice. These classic symptoms,
C. Fever, altered mental status, jaundice known as Charcot triad, are present in about two-thirds of
D. Pain, altered mental stasis, jaundice patients. The illness can progress rapidly with the develop-
ment of shock and altered mental status, known as Reynolds
pentad (eg, fever, jaundice, right upper quadrant pain, septic
shock, and mental status changes). However, the presentation
may be atypical, with little if any fever, jaundice, or pain. (See
Schwartz 11th ed., p. 1408.)

22. What is the treatment for a gallstone ileus? Answer: B


A. Endoscopic retrieval at the ileocecal valve Management of gallstone ileus focuses on relieving the intesti-
B. Surgical enterolithotomy to retrieve the stone and nal obstruction and removing the stone. In cases of very prox-
primary closure or bowel resection imal obstructions in the stomach or duodenum, endoscopic
C. Surgical enterolithotomy to retrieve the stone if retrieval can be effective. For more distal stones, surgical
beyond the ileocecal valve enterolithotomy can be accomplished either ­laparoscopically
D. Supportive management or open. This procedure entails the removal of the stone
through an enterotomy that is then either repaired or resected
depending on its size (Fig. 32-2). Stones that have successfully
traversed the ileocecal valve are likely to pass without further
intervention. The role of pursuing cholecystectomy and/or
choledochoenteric fistula closure at the time of enterolithot-
omy or addressing it at a later time remains a topic of debate,
but it should be considered to reduce the risk of recurrence.
(See Schwartz 11th ed., p. 1409.)

Brunicardi_Ch32_p279-288.indd 284 04/07/22 5:43 PM


285

CHAPTER 32
The Gallbladder and Extrahepatic Biliary System
FIG. 32-2. Gallstone Ileus. Intraoperative photo showing a
longitudinal enterotomy and extraction of an impacted stone from
the distal small bowel.

23. Which of the following choledochal cysts require Answer: E


transplantation? Choledochal cysts are classified into five types depending
A. I on the location and structure of the cysts. The subcategories
B. II of choledochal cysts are defined in Fig. 32-3. Type I cysts
C. III (fusiform common bile duct [CBD] dilations) are the most
D. IV common form, accounting for approximately 50% of cases,
E. V and have the highest risk of malignancy (>60%). For types I
and II (saccular diverticula of the common bile duct), exci-
sion of the cystic dilations in the extrahepatic biliary tree,
including cholecystectomy, with either simple cyst excision or
duct resection with Roux-en-Y hepaticojejunostomy is ideal.
Type III cysts (intraduodenal) create a treatment challenge
as full resection would require pancreaticoduodenectomy.
Given that type III cysts are associated with the lowest malig-
nancy risk of any choledochal cyst (~2%), sphincterotomy
and surveillance are generally recommended over formal
excision. In Type IV (multiple cysts), excision of all cystic tis-
sues and reconstruction is again recommended. For type IVa,
which is characterized by multiple cysts with intrahepatic
involvement, additional segmental resection of the liver may
be required if intrahepatic stones, strictures, or abscesses are
present. Type V choledochal cysts (Caroli disease) are very
rare and account for <1% of patients with choledochal cysts.
These cysts are multiple and can affect the entire liver. In
advanced stages, this may result in cirrhosis and liver f­ailure
­necessitating liver transplantation. (See Schwartz 11th ed.,
­p. 1417.)

Brunicardi_Ch32_p279-288.indd 285 04/07/22 5:43 PM


286
Type I Type II Type III
CHAPTER 32
The Gallbladder and Extrahepatic Biliary System

Type IVa Type IVb Type V

FIG. 32-3. Classification of choledochal cysts. Type I, fusiform or cystic dilations of the extrahepatic biliary
tree, is the most common type, making up >50% of the choledochal cysts. Type II, saccular diverticulum of
an extrahepatic bile duct. Rare, <5% of choledochal cysts. Type III, bile duct dilatation within the duodenal
wall (choledochoceles), makes up about 5% of choledochal cysts. Types IVa and IVb, multiple cysts, make
up 5% to 10% of choledochal cysts. Type IVa affects both extrahepatic and intrahepatic bile ducts, whereas
type IVb cysts affect the extrahepatic bile ducts only. Type V (Caroli disease), intrahepatic biliary cysts, is very
rare and makes up 1% of choledochal cysts.

24. Which of the following features is an indication for Answer: B


cholecystectomy? Polypoid lesions of the gallbladder, which are present in
A. Gallbladder polyp >5 mm as many as 5% of adults, are also associated with increased
B. Gallbladder polyp >10 mm risk of cancer. This is particularly true for polyps measuring
C. Three gallbladder polyps of various sizes >10 mm, which carry a 25% risk of malignancy. Solitary or
D. An 8 mm polyp stable in size over 1 year sessile polyps, or those showing rapid growth on serial imag-
ing, particularly if in the presence of gallstones or age > 50,
are also concerning for malignancy. When such findings are
identified, the patient should have their gallbladder removed,
even if they are asymptomatic. Polyps that are not removed
should be monitored on serial imaging. (See Schwartz 11th ed.,
p. 1421.)

25. A 55-year-old man presents with painless jaundice and is Answer: B


found to have a Bismuth-Corlette type II cholangiocarci- Patients suspected of having resectable disease should first
noma, which of the following is the next best step? undergo diagnostic laparoscopy. Those who are found to have
A. Whipple previously unidentified metastatic disease should undergo
B. Primary resection followed by an hepaticojejunostomy cholecystectomy and surgical bypass for biliary decompres-
C. Local tumor excision with lymphadenectomy, chole- sion. For curative resection, the location and local exten-
cystectomy, common bile duct (CBD) excision, and sion of the tumor dictates the extent of the surgery required.
bilateral Roux-en-Y hepaticojejunostomies Distal bile duct tumors are often resectable but may require
D. Diagnostic laparoscopy pancreaticoduodenectomy (Whipple procedure). For patients

Brunicardi_Ch32_p279-288.indd 286 04/07/22 5:43 PM


287
with distal bile duct cancer found to be unresectable on sur-
gical exploration, Roux-en-Y hepaticojejunostomy, chole-
cystectomy, and gastrojejunostomy to prevent gastric outlet
obstruction should be performed. Perihilar tumors involving
the bifurcation or proximal common hepatic duct (Bismuth-
Corlette types I or II) with no signs of vascular involvement
are candidates for local tumor excision with portal lymphad-
enectomy, cholecystectomy, common bile duct excision, and
bilateral Roux-en-Y hepaticojejunostomies. If the tumor

CHAPTER 32
involves the right or left hepatic duct (Bismuth-Corlette types
IIIa or IIIb), right or left hepatic lobectomy, respectively, should
also be performed. Frequently, resection of the adjacent cau-
date lobe is required because of direct extension into caudate
biliary radicals or parenchyma. Type IV Klatskin tumors, those
with more extensive involvement of both hepatic ducts and
intrahepatic spread, are often considered unresectable or only

The Gallbladder and Extrahepatic Biliary System


treatable with liver transplantation. (See Schwartz 11th ed.,­
p. 1423.)

26. Which of the following scenarios can result in a 5-year Answer: C


disease survival rate as high as 68%? Most patients with perihilar cholangiocarcinoma present
A. Resectable perihilar cholangiocarcinoma with advanced, unresectable disease. Median survival in
B. Resectable distal cholangiocarcinoma this population is between 5 and 8 months. The most com-
C. Cholangiocarcinoma treated with liver mon causes of death are hepatic failure and cholangitis. The
transplantation overall 5-year survival rate for patients with resectable peri-
D. None of the above hilar cholangiocarcinoma is between 10% and 30%, but for
patients with negative margins, it may be as high as 40%. The
operative mortality for perihilar cholangiocarcinoma is 6% to
8%. Patients with distal cholangiocarcinoma are more likely
to have resectable disease and improved prognosis com-
pared to perihilar cholangiocarcinoma. The overall 5-year
survival rate for resectable distal disease is 30% to 50%, and
the median survival is 32 to 38 months. Patients who receive
liver transplantation for cholangiocarcinoma can experi-
ence 5-year disease-free survival rates as high as 68%. (See
Schwartz 11th ed., p. 1423.)

Brunicardi_Ch32_p279-288.indd 287 04/07/22 5:43 PM


This page intentionally left blank

Brunicardi_Ch32_p279-288.indd 288 04/07/22 5:43 PM


CHAPTER 33
Pancreas

1. From which vessel does the most common anatomic Answer: C


variant of the right (replaced) hepatic artery arise from? In 15% to 20% of patients, the right hepatic artery will arise
A. Left gastric artery from the superior mesenteric artery and travel upward
B. Aorta toward the liver along the posterior aspect of the head of the
C. Superior mesenteric artery pancreas (referred to as a replaced right hepatic artery). It is
D. Gastroduodenal artery important to look for this variation on preoperative computed
tomographic (CT) scans and in the operating room so the
replaced hepatic artery is recognized and injury is avoided.
(See Schwartz 11th ed., p. 1433.)

2. According to Ranson’s criteria a 67-year-old female Answer: A


patient suspected of acute pancreatitis presenting to See Table 33-1. (See Schwartz 11th ed., p. 1445, Table 33-7.)
the operating room (OR) with sudden onset of severe
abdominal pain, a serum aspartate aminotransferase
(AST) > 250 U/dL, a WBC > 16,000/mm3, and a blood TABLE 33-1 Ranson’s prognostic signs of pancreatitis
glucose > 200 mg/dL would receive a disease classifica- Criteria for acute pancreatitis not due to gallstones
tion of: At admission During the initial 48 h
A. Severe.
Age > 55 y Hematocrit fall > 10 points
B. Mild, uncomplicated.
WBC > 16,000/mm 3
BUN elevation > 5 mg/dL
Blood glucose > 200 mg/dL Serum calcium < 8 mg/dL
Serum LDH > 350 IU/L Arterial PO2 < 60 mmHg
Serum AST > 250 U/dL Base deficit > 4 mEq/L
Estimated fluid sequestration > 6 L
Criteria for acute gallstone pancreatitis
At admission During the initial 48 h
Age > 70 y Hematocrit fall > 10 points
WBC > 18,000/mm 3
BUN elevation > 2 mg/dL
Blood glucose > 220 mg/dL Serum calcium < 8 mg/dL
Serum LDH > 400 IU/L Base deficit > 5 mEq/L
Serum AST > 250 U/dL Estimated fluid sequestration > 4 L
Note: Fewer than three positive criteria predict mild, uncomplicated disease, whereas
more than six positive criteria predict severe disease with a mortality risk of 50%.
Abbreviations: AST = aspartate transaminase; BUN = blood urea nitrogen; LDH = lactate
dehydrogenase; PO2 = partial pressure of oxygen; WBC = white blood cell count.
Data from Ranson JHC. Etiological and prognostic factors in human acute pancreatitis:
a review. Am J Gastroenterol. 1982;77:633 and from Ranson JH, Rifkind KM, Roses DF,
et al. Prognostic signs and the role of operative management in acute pancreatitis.
Surg Gynecol Obstet. 1974;139:69.

289

Brunicardi_Ch33_p289-296.indd 289 08/07/22 11:50 AM


290
3. In the setting of acute pancreatitis, following aggressive Answer: B
fluid resuscitation, what is the most ideal nutritional In contrast to analgesia and fluid therapy, there is a sound evi-
approach? dence base for nutritional support in acute pancreatitis. It is
A. Patient should undergo strict bowel rest until resolu- no longer acceptable to “rest the pancreas” by avoiding enteral
tion of inflammation. nutrition, now the mainstay of nutritional support. Paren-
B. Enteral feeding can be started as early as 24 hours fol- teral nutrition is now known to be more expensive, riskier,
lowing stabilization. and not more effective than enteral nutrition and should only
C. Parenteral nutrition is preferred, to be started early be offered if the patient’s calculated nutritional requirements
after stabilization. cannot be achieved by the enteral route. Early initiation of
CHAPTER 33

D. Tube feeding is inferior to parenteral nutrition dur- enteral nutrition (within the first 24 hours of admission) is
ing inflammation. not superior to delaying an oral diet until 72 hours. If this is
not tolerated over 48 to 72 hours, then nasogastric tube feed-
ings can be started and increased in step-wise fashion over 2
to 3 days. (See Schwartz 11th ed., p. 1446.)

4. What is the most common cause of chronic pancreatitis? Answer: C


Pancreas

A. Hyperparathyroidism There are a variety of chronic pancreatitis etiologies, the most


B. Alcohol abuse common of which are idiopathic, genetic, heavy drinking,
C. Idiopathic obstructive, hyperlipidemia, hyperparathyroidism, etc. Idio-
D. Genetic pathic and genetic predispositions together comprise of more
than half of the root causes of chronic pancreatitis. A variety
of genes have been identified to play a role in chronic pan-
creatitis, with the cystic fibrosis transmembrane conductance
regulator (CFTR) gene being the most prevalent influencer
(Fig. 33-1). (See Schwartz 11th ed., p. 1452, Figure 33-17.)

Very heavy drinking (15%)

Alcohol + genetic (3%)

CFTR
Idiopathic Genetic (14%) CFTR +
(42%) (24%) SPINK1
(3%)

SPINK1
(4%)
Hyperlipidemia, PRSS1
Gallstone / Autoimmune, (3%)
severe AP Obstructive Other (4%)
(3%) (9%)

FIG. 33-1. Etiologies of chronic pancreatitis. (Reproduced with


permission from Whitcomb DC: Going MAD: development of a “matrix
academic division” to facilitate translating research to personalized
medicine, Acad Med. 2011;86(11):1353–1359.)

5. In patients undergoing endoscopic retrograde cholan- Answer: C


giopancreatography (ERCP) for diagnosis and staging For the diagnosis and staging of chronic pancreatitis, ERCP
of chronic pancreatitis, the population most at risk of is considered to be the gold standard. It also serves as a
developing procedure-induced pancreatitis are those vehicle that enables other diagnostic and therapeutic maneu-
with: vers, such as biopsy or brushing for cytology, or the use of
A. Calculus disease. stents to relieve obstruction or drain a pseudocyst. Unfortu-
B. Intraductal lesions. nately, ERCP also carries a risk of procedure-induced pan-
C. Sphincter of Oddi dysfunction. creatitis that occurs in approximately 5% of patients. Patients
D. High percentage of parenchymal calcification. at increased risk include those with sphincter of Oddi

Brunicardi_Ch33_p289-296.indd 290 08/07/22 11:50 AM


291
dysfunction and those with a previous history of post-ERCP
pancreatitis. Post-ERCP pancreatitis occurs after uncompli-
cated procedures, as well as after those that require prolonged
manipulation. Severe pancreatitis and deaths have occurred
after ERCP. It should be reserved for patients in whom the
diagnosis is unclear despite the use of other imaging meth-
ods, or in whom a diagnostic or therapeutic maneuver is spe-
cifically indicated. (See Schwartz 11th ed., pp. 1459–1560.)

CHAPTER 33
6. Pain from chronic pancreatitis can be caused by: Answer: D
A. Ductal hypertension. Pain from chronic pancreatitis has been ascribed to three
B. Parenchymal disease. possible etiologies. Ductal hypertension, due to strictures or
C. Obstructive pancreatopathy. stones, may predispose to pain that is initiated or exacerbated
D. All of the above. by eating. Chronic pain without exacerbation may be related
to parenchymal disease or retroperitoneal inflammation with
persistent neural involvement. Acute exacerbations of pain in

Pancreas
the setting of chronic pain may be due to acute increases in duct
pressure or recurrent episodes of acute inflammation in the set-
ting of chronic parenchymal disease. Nealon and Matin have
described these various pain syndromes as being predictive of
the response to various surgical procedures. Pain that is found
in association with ductal hypertension is most readily relieved
by pancreatic duct decompression, through endoscopic stent-
ing or surgical decompression. (See Schwartz 11th ed., p. 1460.)

7. What is the most ideal modality for evaluation of chronic Answer: D


pancreatitis? EUS has heavily impacted the evaluation and management of
A. Computed tomography (CT) abdomen pelvis patients with chronic pancreatitis. Although it is more oper-
B. Magnetic resonance cholengiopancreatography ator-dependent than transabdominal ultrasonography, EUS
(MRCP) provides not only imaging capability but also adds the capac-
C. Endoscopic retrograde cholangiopancreatography ity to obtain cytologic and chemical samples of tissue and fluid
(ERCP) aspirated with linear array monitoring (Fig. 33-2). EUS images
D. Endoscopic ultrasonography (EUS) obtained through a high-frequency (7.5- to 12.5-mHz) trans-
ducer are able to evaluate subtle changes in 2- to 3-mm struc-
tures within the pancreas and can detect indolent neoplasms in
the setting of chronic inflammation. Small intraductal lesions,
intraductal mucus, cystic lesions, and subtle ductular abnormal-
ities are recognizable by EUS (Table 33-2). This allows ERCP to
be reserved for these patients who require therapeutic maneu-
vers, or for the evaluation of more complex problems. EUS is
comparable to ERCP in the detection of advanced changes in
chronic pancreatitis and may be more sensitive than ERCP in
the detection of mild disease. (See Schwartz 11th ed., p. 1457.)

FIG. 33-2. Endoscopic ultrasound of


chronic pancreatitis. The endoscopic
ultrasound appearance of the
parenchyma is heterogeneous, and
dilated ducts are seen, indicating early
obstructive pancreatopathy. (Reproduced
with permission from Mark Topazian,
Division of Digestive Diseases, Department
of Medicine, Mayo Clinic.)

Brunicardi_Ch33_p289-296.indd 291 08/07/22 11:50 AM


292
TABLE 33-2 Endoscopic ultrasound features of chronic pancreatitis
Endoscopic Ultrasound Feature Implication
Ductal changes
Duct size > 3 mm Ductal dilation
Tortuous pancreatic duct Ductal irregularity
Intraductal echogenic foci Stones or calcification
Echogenic duct wall Ductal fibrosis
Side-branch ectasia Periductal fibrosis
CHAPTER 33

Parenchymal changes
Inhomogeneous echo pattern Edema
Reduced echogenic foci (1–3 mm) Edema
Enhanced echogenic foci Calcifications
Prominent interlobular septae Fibrosis
Lobular outer gland margin Fibrosis, glandular atrophy
Pancreas

Large, echo-poor cavities (>5 mm) Pseudocyst


Reproduced with permission from Catalano MF, Lahoti S, Geenen JE, et al.: P­ rospective evaluation of
endoscopic ultrasonography, endoscopic retrograde pancreatography, and secretin test in the diagnosis of
chronic pancreatitis, Gastrointest Endosc. 1998;48(1):11-17.

8. Which component of the pancreas is associated with the Answer: A


long-term inflammatory process in chronic pancreatitis, The common element of these variations on the theme of
subsequently addressed in surgical resection? LR-LPJ remains the excavation or “coring out” of the central
A. The head portion of the pancreatic head. It remains uncertain, how-
B. The body ever, whether and to what degree the dichotomy needs to be
C. The neck extended into the body and tail. The logical conclusion of all
D. The tail of these efforts is that the head of the pancreas is the nidus of
the chronic inflammatory process in chronic pancreatitis and
that removal of the central portion of the head of the gland is
the key to the successful resolution of pain in the long term.
(See Schwartz 11th ed., p. 1478.)

9. Which of the following is the imaging modality of choice Answer: C


for initial diagnosis of exocrine tumors? As with pancreatic exocrine tumors, the initial diagnostic
A. Computed tomography (CT) with oral and intrave- imaging test of choice for pancreatic endocrine tumors is a
nous (IV) contrast multidetector CT scan with four phases of contrast and fine
B. Positron emission tomography-CT (PET-CT) cuts through the pancreas and liver. Neuroendocrine tumors
C. CT with four-phase contrast of the pancreas often enhance with contrast. EUS can be
D. Magnetic resonance imaging (MRI) superior to CT in localizing these tumors, which can produce
dramatic symptoms despite their small (<1 cm) size. In con-
trast to pancreatic exocrine tumors, many of the endocrine
tumors have somatostatin receptors (SSTRs) that allow them
to be detected by a radiolabeled octreotide scan. A radioactive
somatostatin analogue is injected intravenously, followed by
whole-body radionuclide scanning (Fig. 33-3). The success of
this modality in localizing tumors and detecting metastases
has decreased the use of older techniques such as angiogra-
phy and selective venous sampling. (See Schwartz 11th ed.,
p. 1480.)

Brunicardi_Ch33_p289-296.indd 292 08/07/22 11:50 AM


293

CHAPTER 33
Pancreas
FIG. 33-3. Radioactive octreotide scan
demonstrating pancreatic endocrine
tumor in the body of the pancreas
(arrow).

10. Which of the following is the most common presenting Answer: A


symptom in patients with a somatostatinoma? Because somatostatin inhibits pancreatic and biliary secre-
A. Cholelithiasis tions, patients with a somatostatinoma present with gall-
B. Constipation stones due to bile stasis, diabetes due to inhibition of insulin
C. Hypoglycemia secretion, and steatorrhea due to inhibition of pancreatic
D. Hypocalcemia exocrine secretion and bile secretion. Most somatostatino-
mas originate in the proximal pancreas or the pancreato-
duodenal groove, with the ampulla and periampullary area
as the most common site (60%). The most common pre-
sentations are abdominal pain (25%), jaundice (25%), and
cholelithiasis (19%). This rare type of pancreatic endocrine
tumor is diagnosed by confirming elevated serum soma-
tostatin levels, which are usually above 10 ng/mL. Although
most reported cases of somatostatinoma involve metastatic
disease, an attempt at complete excision of the tumor and
cholecystectomy is warranted in fit patients. (See Schwartz
11th ed., p. 1483.)

Brunicardi_Ch33_p289-296.indd 293 08/07/22 11:50 AM


294
11. Treatment of a 1 cm gastrinoma in the wall of the duode- Answer: B
num is best accomplished by: Fifty percent of gastrinomas metastasize to lymph nodes or
A. Enucleation. the liver, and are therefore considered malignant. Patients who
B. Full-thickness resection. meet criteria for operability should undergo exploration for
C. Duodenectomy. possible removal of the tumor. Although the tumors are sub-
D. Whipple procedure. mucosal, a full-thickness excision of the duodenal wall is per-
formed if a duodenal gastrinoma is found. All lymph nodes in
Passaro triangle are excised for pathologic analysis. If the gas-
trinoma is found in the pancreas and does not involve the main
CHAPTER 33

pancreatic duct, it is enucleated. Pancreatic resection is justified


for solitary gastrinomas with no metastases. A highly selective
vagotomy can be performed if unresectable disease is identified
or if the gastrinoma cannot be localized. This may reduce the
amount of expensive proton pump inhibitors required. In cases
in which hepatic metastases are identified, resection is justified
if the primary gastrinoma is controlled and the metastases can
Pancreas

be safely and completely removed. Debulking or incomplete


removal of multiple hepatic metastases is probably not help-
ful, especially in the setting of MEN1. The application of new
modalities such as radiofrequency ablation seems reasonable,
but data to support this approach are limited. Postoperatively,
patients are followed with fasting serum gastrin levels, secretin
stimulation tests, octreotide scans, and CT scans. In patients
found to have inoperable disease, chemotherapy with strepto-
zocin, doxorubicin, and 5-fluorouracil (5-FU) is used. Other
approaches such as somatostatin analogues, interferon, and
chemoembolization also have been used in gastrinoma with
some success. (See Schwartz 11th ed., p. 1482.)
12. Which of the pancreatic endocrine tumors are associated Answer: A
with diabetes and dermatitis? Diabetes in association with dermatitis should raise the sus-
A. Glucagonoma picion of a glucagonoma. The diabetes usually is mild. The
B. Somatostatinoma classic necrolytic migratory erythema manifests as cyclic
C. Insulinoma migrations of lesions with spreading margins and healing
D. VIPoma centers typically on the lower abdomen, perineum, perioral
area, and feet. Patients also complain of an enlarged, sensi-
tive tongue. The diagnosis is confirmed by measuring serum
glucagon levels, which are usually >500 pg/mL. Glucagon is
a catabolic hormone, and most patients present with malnu-
trition. The rash associated with glucagonoma is thought to
be caused by low levels of amino acids. Preoperative treat-
ment usually includes control of the diabetes, parenteral
­nutrition, and octreotide. Like VIPomas, glucagonomas are
more often in the body and tail of the pancreas and tend to
be large tumors with metastases. Again, debulking operations
are recommended in good operative candidates to relieve
­symptoms. (See Schwartz 11th ed., p. 1483.)

13. Which of the following have been proven to signifi- Answer: D


cantly reduce the rate of pancreatic leak following Considerable attention has been focused on the prevention
pancreaticoduodenectomy? of pancreatic leak after pancreas resection. Modifications of
A. Stent the anastomotic technique (end-to-side or end-to-end, duct-
B. Glue to-mucosa, or invaginated), the use of jejunum or the stom-
C. Octreotide ach for drainage, the use of pancreatic duct stents, the use of
D. None of the above octreotide, and various sealants have all been evaluated.
Long-acting synthetic analogues of somatostatin have been
evaluated as a pharmacologic therapy to reduce pancreatic
secretion and the rate of pancreatic fistula after pancreatic
resection. Some European studies supported benefit partic-
ularly in selected higher risk patients, while previous North

Brunicardi_Ch33_p289-296.indd 294 08/07/22 11:50 AM


295
American trials concluded there was no benefit. A recent
single-center, randomized trial with Pasireotide (a newer
­
analog) suggested potential benefit.
Use of a pancreatic duct stent across the anastomosis has
been suggested as a means of preventing a pancreatic leak and
as an aid in technical precision. Both internal stenting as well
as external stenting have been practiced. A recent Cochrane
analysis of eight randomized, controlled trials failed to iden-
tify any convincing evidence of benefit with internal or exter-

CHAPTER 33
nal pancreatic duct stents. Some previous studies indicated
that stents might be harmful. A recent multicenter random-
ized trial comparing external to internal pancreatic duct
stents during pancreaticoduodenectomy showed a lower rate
of pancreatic fistula with internal stents, so this controversy is
likely to continue. (See Schwartz 11th ed., p. 1497.)

Pancreas
14. A pancreatic cystic neoplasms that is <3 cm has atypical Answer: D
cells present and has a solid component requires: See Fig. 33-4. (See Schwartz 11th ed., p. 1501, Figure 33-78.)
A. A repeat computed tomography (CT) scan in 3 to 6
months.
B. A repeat CT scan in 1 year.
C. Continued observation.
D. Resection.
Are any of the following high-risk stigmata of malignancy present?
(i) obstructive jaundice in a patient with cystic lesion of the head of the pancreas,
(ii) enhancing solid component within cyst, (iii) main pancreatic duct ≥10 mm in size

Yes
No

Are any of the following worrisome features present?


Clinical: Pancreatitisa
Imaging: (i) cyst ≥3 cm, (ii) thickened/enhancing cyst walls, (iii) main duct size 5–9 mm, (iii) nonenhancing
mural nodule, (iv) abrupt change in caliber of pancreatic duct with distal pancreatic atrophy.
Consider
surgery,
if clinically
appropriate If yes, perform endoscopic ultrasound No

Yes Are any of the following features present? No


What is the size of largest cyst?
(i) Definite mural nodule(s)b
(ii) Main duct features suspicious for involvementc
(iii) Cytology: suspicious or positive for malignancy Inconclusive

<1 cm 1–2 cm 2–3 cm >3 cm

CT/MRI CT/MRI EUS in 3–6 months, then Close surveillance alternating


in 2–3 yearsd yearly × 2 years, lengthen interval alternating MRI MRI with EUS every 3–6 months.
then lengthen with EUS as appropriate.d Strongly consider surgery in young,
interval Consider surgery in young, fit patients
if no changed fit patients with need for
prolonged surveillance

a. Pancreatitis may be an indication for surgery for relief of symptoms.


b. Differential diagnosis includes mucin. Mucin can move with change in patient position, may be dislodged on
cyst lavage and does not have Doppler flow. Features of true tumor nodule include lack of mobility, presence of
Doppler flow and FNA of nodule showing tumor tissue.
c. Presence of any one of thickened walls, intraductal mucin or mural nodules is suggestive of main duct
involvement. In their absence main duct involvement is inconclusive.
d. Studies from Japan suggest that on follow-up of subjects with suspected BD-IPMN there is increased incidence
of pancreatic ductal adenocarcinoma unrelated to malignant transformation of the BD-IPMN(s) being followed.
However, it is unclear if imaging surveillance can detect early ductal adenocarcinoma, and, if so, at what interval
surveillance imaging should be performed.

FIG. 33-4. Algorithm for management of pancreatic cystic neoplasms. CEA = carcinoembryonic antigen;
CT = computed tomography; ERCP = endoscopic retrograde cholangiopancreatography; EUS = endoscopic
ultrasound; FNA = fine-needle aspiration; Hx = history; IPMN = intraductal papillary mucinous neoplasm of the
pancreas; MCN = mucinous cystic neoplasm; MRCP = magnetic resonance cholangiopancreatography. (Reproduced
with permission from Tanaka M, Adsay V, Chari S, et al. International consensus guidelines 2012 for the management of
IPMN and MCN of the pancreas, Pancreatology. 2012;12(3):183–197.)

Brunicardi_Ch33_p289-296.indd 295 08/07/22 11:50 AM


296
15. The endoscopic retrograde cholangiopancreatography Answer: A
(ERCP) finding that is virtually diagnostic of intraductal IPMNs usually occur within the head of the pancreas and
papillary mucinous neoplasms (IPMNs) is: arise within the pancreatic ducts. The ductal epithelium
A. A fish-eye lesion. forms a papillary projection into the duct, and mucin pro-
B. Calcification. duction causes intraluminal cystic dilation of the pancreatic
C. Beaded or chain-of-lakes appearance of the duct. ducts (Fig. 33-5). Imaging studies demonstrate diffuse dila-
D. Cysts that resemble serous cystadenomas. tion of the pancreatic duct, and the pancreatic parenchyma
is often atrophic due to chronic duct obstruction. However,
classic features of chronic pancreatitis, such as calcifica-
CHAPTER 33

tion and a beaded appearance of the duct, are not present.


At ERCP, mucin can be seen extruding from the ampulla of
Vater, a so-called fish-eye lesion, that is virtually diagnostic of
IPMN. (See Schwartz 11th ed., p. 1502.)
Pancreas

FIG. 33-5. Intraductal papillary mucinous neoplasm histology.


Papillary projections of ductal epithelium resemble villous
morphology and contain mucin-filled vesicles. (Reproduced with
permission from Asiyanbola B, Andersen DK. IPMN. Editorial Update.
accesssurgery.com McGraw Hill; 2008.)

Brunicardi_Ch33_p289-296.indd 296 08/07/22 11:50 AM


CHAPTER 34
The Spleen

1. A patient with which of the following diseases would Answer: B


most benefit from splenectomy? Chronic lymphocytic leukemia (CCL) is a subtype of non-
A. Acute myeloid leukemia with neutropenia Hodgkin lymphoma (NHL) with the main characteristic
B. Chronic lymphocytic leukemia with thrombocyto- being a progressive accumulation of old and nonfunctional
penia lymphocytes. Symptoms of CLL are nonspecific and include
C. Hairy cell leukemia with abnormal lymphocytes weakness, fatigue, fever without illness, night sweats, and fre-
D. Hodgkin lymphoma with lymphocytopenia quent bacterial and viral infections. The most frequent find-
ing is lymphadenopathy. When the spleen is enlarged, it may
be massive or barely palpable below the costal margin. Sple-
nectomy is indicated to improve cytopenias and was shown
to be 75% effective in a combined group of patients who had
either CLL or nonmalignant Hodgkin disease (HD). Splenec-
tomy may thus facilitate chemotherapy in patients whose cell
counts are prohibitively low before spleen removal. Palliative
splenectomy also is indicated for symptomatic splenomegaly.
The remaining disease processes are not likely to improve
with splenectomy. (See Schwartz 11th ed., p. 1528.)

2. You are seeing a 13-year-old girl in clinic for a painless Answer: C


lump in the left upper abdominal quadrant. Her father Hereditary spherocytosis (HS) is the most common type of
reports she had jaundice as a newborn and on occasion, hemolytic anemia for which splenectomy is indicated and
she becomes pale. The father also says multiple relatives the third most common type of congenital hemolytic anemia
have had their spleen removed for hemolytic anemia. overall. Patients with typical HS forms may have mild jaun-
You order a complete blood count. Which of the follow- dice. Splenomegaly usually is palpable on physical exami-
ing findings would identify the most likely etiology for nation. Laboratory examination reveals varying degrees of
this presentation? anemia: patients with mild forms of the disease may not have
A. Anemia and thrombocytopenia anemia; patients with moderate to severe forms may have
B. Thrombocytopenia and leukocytosis hemoglobin levels as low as 4 to 6 g/dL. The mean corpus-
C. Elevated mean corpuscular hemoglobin and elevated cular volume is typically low to normal or slightly decreased.
red cell distribution width For screening, a combined elevated mean corpuscular hemo-
D. Lymphocytosis and monocytosis globin concentration and an elevated erythrocyte distribution
width are an excellent predictor. Other laboratory indicators
of HS include those providing evidence of rapid red blood cell
destruction, including elevated reticulocyte count, elevated
lactate dehydrogenase level, and increased level of unconju-
gated bilirubin. Spherocytes are readily apparent on periph-
eral blood film. (See Schwartz 11th ed., p. 1524.)

297

Brunicardi_Ch34_p297-306.indd 297 30/06/22 11:26 AM


298
3. A patient is referred to you for surgical management of Answer: D
medically refractory idiopathic thrombocytopenic pur- IV immunoglobulin, given at 1.0 g/kg per day for 2 to 3 days,
pura, currently being treated with daily prednisone. On is indicated for internal bleeding when platelet counts remain
your assessment, they would benefit from a splenectomy, <5000/mm3, when extensive purpura exists, or to preop-
but their platelet count is low enough that you are con- eratively boost platelets. IV immunoglobulin is thought to
cerned for intraoperative bleeding. Which preoperative impair clearance of immunoglobulin G–coated platelets by
intervention would be most beneficial? competing for binding to tissue macrophage receptors. An
A. Platelet transfusion immediate response is common, but a sustained remission is
B. Whole blood transfusion not. (See Schwartz 11th ed., p. 1526.)
CHAPTER 34

C. Fresh frozen plasma


D. IV immunoglobulin

4. A patient has been referred to you on suspicion of hemo- Answer: A


lytic anemia by their primary care physician, who thinks A positive result on direct Coombs test confirms the antibody
they may benefit from a splenectomy. Which test would autoimmune hemolytic anemia (AIHA) diagnosis by distin-
confirm a diagnosis of autoimmune hemolytic anemia? guishing autoimmune from other forms of hemolytic anemia.
The Spleen

A. Positive direct Coombs test (See Schwartz 11th ed., pp. 1524–1525.)
B. Negative indirect Coombs test
C. Increased red cell distribution width
D. Increased lactate dehydrogenase

5. Which malignancy most commonly spreads to the Answer: A


spleen? Lung carcinoma is the tumor that most commonly spreads
A. Lung to the spleen, although colorectal, ovarian, and melanoma
B. Colorectal may also metastasize to the spleen. (See Schwartz 11th ed.,
C. Ovarian p. 1529.)
D. Melanoma

6. A 45-year-old woman with a diagnosis of rheumatoid Answer: B


arthritis also develops splenomegaly and neutropenia. The triad of rheumatoid arthritis, splenomegaly, and neutro-
What is the mechanism by which neutropenia develops? penia is called Felty syndrome. It exists in approximately 3%
A. Sequestration of leukocytes in the spleen of all patients with rheumatoid arthritis, two-thirds of whom
B. Autoimmune reaction to neutrophils are women. Immune complexes coat the surface of white
C. Marrow underproduction of the granulocyte lineage blood cells, which leads to their sequestration and clearance
D. Increased spleen sinusoid permeability in the spleen with subsequent neutropenia. This neutrope-
nia (<2000 neutrophils/mm3) increases the risk for recurrent
infections and often drives the decision for splenectomy. (See
Schwartz 11th ed., p. 1532.)

7. You are evaluating a 55-year-old man in clinic for an Answer: A


elective operation. His surgical history is notable for a International guidelines recommend annual influenza vac-
traumatic, open splenectomy when he was 25 following cine for asplenic patients. The influenza vaccination provides
a motor vehicle collision. He says he hasn’t had any vac- protection from influenza syndrome and secondary bacte-
cines in the last 20 years, which of the following is the rial infection, and is associated with a 54% reduced risk of
most important for him to get with regard to his history death compared with unimmunized asplenic persons. (See
of splenectomy? Schwartz 11th ed., pp. 1532–1533.)
A. Influenza
B. TDap
C. Herpes Zoster
D. Measles, mumps, rubella

Brunicardi_Ch34_p297-306.indd 298 30/06/22 11:26 AM


299
8. You are performing an open splenectomy for symptom- Answer: A
atic splenomegaly. The spleen is exposed and you have In patients with significant splenomegaly, once lesser sac
excellent visualization of the associated blood supply. access has been achieved through either the gastrosplenic
What vessel do you ligate first? or gastrohepatic attachments, ligating the splenic artery in
A. Splenic artery continuity along the superior border of the pancreas may
B. Splenic vein be preferable. This maneuver may serve several purposes:
C. Short gastric arteries allowing safer manipulation of the spleen and dissection of
D. Short gastric veins the splenic hilum, facilitating some shrinkage of the spleen,
and providing an autotransfusion of erythrocytes and plate-

CHAPTER 34
lets. Further medial mobilization of the spleen is achieved by
incising its lateral peritoneal attachments, most notably the
splenophrenic ligament. Then follows individual ligation and
sequential division of the short gastric vessels, steps that if
carefully executed reduce the risk of these vessels’ retracting
and bleeding. Splenic hilar dissection then takes place. When-
ever possible, care should be taken to dissect and individually

The Spleen
ligate the splenic artery and vein (in that order) before divid-
ing them. (See Schwartz 11th ed., pp. 1533–1534.)

9. You are counselling a patient on the risks of an elective Answer: B


laparoscopic splenectomy. Which of the following can- A Taiwanese population-based study found that individuals
cers will the patient be at increased risk for following the who had splenectomy have higher risks of developing certain
operation? types of cancer (adjusted hazard ratios were 2.64 and 1.29 for
A. Ovarian nontraumatic and traumatic reasons, respectively). Splenec-
B. Non-Hodgkin lymphoma tomy patients were found to have significantly higher risks in
C. Sarcoma esophagus, stomach, liver, other head and neck, non-Hodgkin
D. Pancreatic lymphoma, and leukemia cancers. Although the exact mecha-
nism for the possible association between splenectomy and
cancer remains unclear, a plausible explanation is that the
spleen is thought to be involved in immunological defenses
and provides active response through humoral and cell-medi-
ated pathways and that splenectomy may impair immune sur-
veillance in the host. (See Schwartz 11th ed., p. 1539.)

10. A 31-year-old man is admitted to the hospital 6 months Answer: D


following a posttraumatic splenectomy with evidence of More recently, the bacterial patterns of post-splenectomy
sepsis. He has completed all recommended postopera- sepsis have been changing. After the introduction of vaccina-
tive follow-ups. Which organism is the most likely cause tions and new oral antibiotics, postsplenectomy patients can
of the infection? suffer from diverse strains of bacterial infection, which are
A. Staphylococcus aureus not strictly correlated with the splenic function. In a recent
B. Streptococcus pneumonia cohort series, gram-negative bacteria are prevalent, repre-
C. Neisseria meningitidis senting 45% to 50% of infections in asplenic patients. In vac-
D. Escherichia coli cinated patients, the rate of sepsis by pneumococcus is very
low. In fact, encapsulated bacteria, such as S. pneumoniae, N.
meningitidis, and H. influenzae, were rarely encountered in
those series in whom vaccination was routinely adopted. (See
Schwartz 11th ed., pp. 1538–1539.)

11. Which disorder is most likely to benefit from Answer: C


splenectomy? Favorable responses to splenectomy have been reported in
A. Glucose-6-phosphate dehydrogenase deficiency patients with warm-antibody autoimmune hemolytic anemia
B. Pyruvate kinase deficiency (AIHA), with a recent series showing complete remission
C. Warm-agglutinin disease of refractory AIHA following laparoscopic splenectomy at
D. Child with immune thrombocytopenic purpura 35-month follow-up in patients over 60 years old. Transient
responses are more common, however, and many patients
eventually experience hemolysis again despite splenectomy.
The decision regarding splenectomy in the case of AIHA
should be individualized based on careful consideration of
the clinical history and frank discussion with the patient. It is

Brunicardi_Ch34_p297-306.indd 299 30/06/22 11:26 AM


300
considered as a third-line therapy after failure of steroids or
anti-CD20 antibody administration. Conventional wisdom
is that splenectomy is not indicated in glucose-6-phosphate
dehydrogenase deficiency, and certainly the overwhelming
majority of patients with G6PD deficiency will neither require
nor benefit from splenectomy. However, one report described
a small case series of six symptomatic G6PD deficiency
patients who had severe hemolytic anemia and required
transfusion, all of whom were identified to share a common
CHAPTER 34

mutation at exon 10. All underwent splenectomy. A complete


response occurred in four patients (transfusion requirement
eliminated), and a partial response occurred in one patient
(transfusion requirement reduced); no follow-up data were
provided for the remaining patient. This study indicates that
for a carefully select group of patients with severe hemolytic
anemia attributable to G6PD deficiency, splenectomy may be
The Spleen

of benefit, although more data is needed before strong recom-


mendation can be made. The majority of pediatric immune
thrombocytopenia (ITP) patients recover spontaneously in a
few months. (See Schwartz 11th ed., pp. 1524–1527.)

12. Following a splenectomy for chronic hemolytic anemia, Answer: D


what hemoglobin level is indicative of success? For chronic hemolytic anemias, a rise in hemoglobin levels to
A. >7 g/dL >10 g/dL without the need for transfusion signifies a success-
B. >8 g/dL ful response to splenectomy. By this criterion, splenectomy has
C. >9 g/dL been reported to be successful for the vast majority of patients
D. >10 g/dL with chronic hemolytic anemia. For hemolytic anemia due to
spherocytosis, the success rate is usually higher, ranging from
90% to 100%. (See Schwartz 11th ed., pp. 1524–1525.)

13. Which of the following is NOT a location where acces- Answer: B


sory spleens can be found? The most common anomaly of splenic embryology is the
A. Gastrocolic ligament accessory spleen. Present in up to 20% of the population, one
B. Gerota fascia or more accessory spleen(s) may occur in up to 30% of patients
C. Large bowel mesentery with hematologic disease. Over 80% of accessory spleens are
D. Broad ligament found in the region of the splenic hilum and vascular pedicle.
Other locations for accessory spleens in descending order of
frequency are: the gastrocolic ligament, the tail of the pan-
creas, the greater omentum, the greater curve of the stomach,
the splenocolic ligament, the small and large bowel mesen-
tery, the left broad ligament in women, and the left spermatic
cord in men. (See Schwartz 11th ed., p. 1519.)

14. Which of the following splenic ligaments is NOT an Answer: A


avascular plane? Of particular clinical relevance, the spleen is suspended
A. Gastrosplenic in position by several ligaments and peritoneal folds to the
B. Splenocolic colon (splenocolic ligament); the stomach (gastrosplenic
C. Phrenosplenic ligament); the diaphragm (phrenosplenic ligament); and the
D. Splenorenal kidney, adrenal gland, and tail of the pancreas (splenorenal
ligament) (Fig. 34-1). Whereas the gastrosplenic ligament
contains the short gastric vessels, the remaining ligaments are
usually avascular, with rare exceptions, such as in a patient
with portal hypertension. The relationship of the pancreas to
the spleen also has important clinical implications. In cadav-
eric anatomic series, the tail of the pancreas has been dem-
onstrated to lie within 1 cm of the splenic hilum 75% of the
time and to actually abut the spleen in 30% of patients. (See
Schwartz 11th ed., p. 1519.)

Brunicardi_Ch34_p297-306.indd 300 30/06/22 11:26 AM


301
Gastrosplenic
ligament

Phrenicocolic
ligament

Lesser sac

CHAPTER 34
Sustentaculum
lienis

The Spleen
Splenocolic
ligament

FIG. 34-1. Suspensory ligaments of the spleen. Greater omentum

15. All of the following are functions of the spleen EXCEPT: Answer: C
A. Clearance of damaged or aged red blood cells from The spleen has both fast and slow circulation of blood. It is
the blood. during slow circulation that blood travels through the reticu-
B. Extramedullary site for hematopoesis and recycling lar spaces and splenic cords where it is exposed to contact
iron. with splenic macrophages, which remove senescent blood
C. Initiation of adaptive immune response from filtra- cells. Through this process, the spleen is also able to remove
tion of lymph. erythrocyte inclusions such as Heinz bodies without lysing
D. Clearance of encapsulated bacteria from the blood the cells. Through the reticuloendothelial system, the spleen
stream. clears encapsulated bacteria such as pneumococcus and
Haemophilus influenzae, which are poorly opsonized from
the hepatic reticuloendothelial system. In addition to these
functions, the spleen serves as an extramedullary site for
hematopoiesis and plays a functional role in the recycling of
iron. While the white pulp of the spleen is important in the
initiation of the adaptive immune response, material is deliv-
ered to the spleen through the blood and not the lymph. (See
Schwartz 11th ed., pp. 1520–1521.)

16. Which of the following proteins is not altered in heredi- Answer: A


tary spherocytosis (HS)? The underlying abnormality in HS is an inherited dysfunc-
A. Pyruvate kinase tion or deficiency in one of the erythrocyte membrane pro-
B. Spectrin teins (spectrin, ankyrin, band 3 protein, or protein 4.2), which
C. Ankyrin results in destabilization of the membrane lipid bilayer. This
D. Band 3 protein destabilization allows a release of lipids from the membrane,
causing a reduction in membrane surface area and a lack of
deformability, leading to sequestration and destruction of the
spherocytic erythrocytes in the spleen.
Although less common than glucose-6-phosphate dehy-
drogenase (G6PD) deficiency overall, pyruvate kinase defi-
ciency is the most common red blood cell enzyme deficiency
to cause congenital chronic hemolytic anemia. (See Schwartz
11th ed., p. 1524.)

Brunicardi_Ch34_p297-306.indd 301 30/06/22 11:26 AM


302
17. The disproportionately high rate of overwhelming post- Answer: A
splenectomy infection in thalassemia patients is thought The increase in infectious complications associated with
to be due to an immune deficiency. Which of the follow- splenectomy in thalassemia patients is thought to be due to a
ing strategies has been shown to reduce mortality? coexisting immune deficiency that is caused by iron overload.
A. Partial splenectomy Iron overload is associated with both thalassemia as well as
B. Prophylactic antibiotic therapy the transfusions that accompany treatment for thalassemia.
C. Delaying splenectomy until after 2 years of age Some investigators have tried partial splenectomy with some
D. Transfusion to maintain a hemoglobin of >9 mg/dL success in reducing mortality associated with splenectomy in
these patients. In addition, splenectomy should be delayed
CHAPTER 34

until the patient is older than 4 years of age unless absolutely


necessary. While transfusion to maintain a hemoglobin of
>9 mg/dL is part of the treatment for thalassemia it does not
reduce infectious complications associated with splenectomy
in these patients. There is little evidence supporting efficacy
of prophylactic antibiotics in asplenic patients in preventing
infectious complications associated with splenectomy. (See
The Spleen

Schwartz 11th ed., p. 1525.)

18. A 30-year-old woman presents to her primary care pro- Answer: A


vider with complaints of bleeding gums while brushing Idiopathic thrombocytopenic purpura is an autoimmune
her teeth as well as menorrhagia and several episodes of disorder characterized by a low platelet count and mucocuta-
epistaxis within the past month. She has been previously neous and petechial bleeding. The usual first line of therapy
healthy with no prior medical problems or surgeries. for idiopathic thrombocytopenic purpura is oral prednisone
Examination reveals petechiae and ecchymosis over the with most responses occurring within the first 3 weeks after
lower extremities. Laboratory results show white blood initiating therapy. IV immunoglobulin is given for internal
cell count 7000/mm3, hemoglobin 14 g/dL, hematocrit bleeding with platelet counts < 5000/mm3, when extensive
42%, and platelet count 28,000/mm3 with numerous purpura exists, or to increase platelets preoperatively and is
megakaryocytes on peripheral smear. First-line therapy thought to work by impairing clearance of immunoglobulin
for this condition would be: G-coated platelets by competing for binding to tissue macro-
A. Oral prednisone phage receptors. Both rituximab and thrombopoietin receptor
B. IV immunoglobulin antagonists are second-line treatment options. Splenectomy
C. Rituximab is an option for refractory immune thrombocytopenia (ITP)
D. Splenectomy and can provide a permanent response in about 75% to 85%
of patients. (See Schwartz 11th ed., pp. 1526–1527.)

19. The most common physical finding in a patient with Answer: A


hairy cell leukemia (HCL) is: HCL is an uncommon blood disorder, representing only 2%
A. Massive splenomegaly. of all adult leukemias. HCL is characterized by splenomegaly,
B. Shortness of breath. pancytopenia, and large numbers of abnormal lymphocytes
C. Abdominal pain. in the bone marrow. These lymphocytes contain irregular
D. Joint pain. hair-like cytoplasmic projections identifiable on the periph-
eral smear. Most patients seek medical attention because of
symptoms related to anemia, neutropenia, thrombocytope-
nia, or splenomegaly. The most common physical finding is
splenomegaly, which occurs in 80% of patients with HCL and
is often palpable 5 cm below the costal margin. Many patients
with HCL have few symptoms and require no specific ther-
apy. Treatment is indicated for those with moderate to severe
symptoms related to cytopenias, such as repeated infections
or bleeding episodes, or to splenomegaly, such as pain or early
satiety. Splenectomy does not correct the underlying disor-
der, but does return cell counts to normal in 40% to 70% of
patients and alleviates pain and early satiety. Newer chemo-
therapeutic agents (the purine analogues 2ʹ-deoxycoformycin
[2ʹ-DCF] and 2-chlorodeoxyadenosine [2-CdA]) are able to
induce durable complete remission in most patients. (See
Schwartz 11th ed., p. 1527.)

Brunicardi_Ch34_p297-306.indd 302 30/06/22 11:26 AM


303
20. Which of the following is an indication for splenectomy Answer: C
in a patient with chronic myelogenous leukemia (CML)? CML is a disorder of the primitive pluripotent stem cell in the
A. Failure of chemotherapy to decrease splenomegaly bone marrow, resulting in a significant increase in erythroid,
B. Sequestration requiring transfusion megakaryotic, and pluripotent progenitors in the peripheral
C. Symptomatic relief of early satiety blood smear. The genetic hallmark is a transposition between
D. Presence of bcr gene mutation the bcr gene on chromosome 9 and the abl gene on chromo-
some 22. CML accounts for 7% to 15% of all leukemias, with
an incidence of 1.5 in 100,000 in the United States. CML is
frequently asymptomatic in the chronic phase, but symptom-

CHAPTER 34
atic patients often present with the gradual onset of fatigue,
anorexia, sweating, and left upper quadrant pain and early
satiety secondary to splenomegaly. Enlargement of the spleen
is found in roughly one half of patients with CML. Splenec-
tomy is indicated to ease pain and early satiety. (See Schwartz
11th ed., p. 1528.)

The Spleen
21. Which of the following is an indication for splenectomy Answer: C
in polycythemia vera? Polycythemia vera (PV) is a clonal, chronic, progressive
A. Failure of aspirin to prevent thrombotic complications myeloproliferative disorder characterized by an increase in
B. Frequent need for phlebotomy red blood cell mass, frequently accompanied by leukocyto-
C. Symptoms related to splenomegaly sis, thrombocytosis, and splenomegaly. Patients affected by
D. Prevention of progression to myeloid metaplasia PV typically enjoy prolonged survival compared to others
affected by hematologic malignancies, but remain at risk for
transformation to myelofibrosis or acute myeloid leukemia
(AML). The disease is rare, with an annual incidence of 5
to 17 cases per million population. Although the diagnosis
may be discovered by routine screening laboratory tests in
asymptomatic individuals, affected patients may present with
any number of nonspecific complaints, including headache,
dizziness, weakness, pruritus, visual disturbances, excessive
sweating, joint symptoms, and weight loss. Physical findings
include ruddy cyanosis, conjunctival plethora, hepatomegaly,
splenomegaly, and hypertension. The diagnosis is established
by an elevated red blood cell mass (>25% of mean predicted
value), thrombocytosis, leukocytosis, normal arterial oxygen
saturation in the presence of increased red blood cell mass,
splenomegaly, low serum erythropoietin (EPO) stores, and
bone marrow hypercellularity. Treatment should be tailored to
the risk status of the patient and ranges from phlebotomy and
aspirin to chemotherapeutic agents. As in essential thrombo-
cythemia (ET), splenectomy is not helpful in the early stages
of disease and is best reserved for late-stage patients in whom
myeloid metaplasia has developed and splenomegaly-related
symptoms are severe. (See Schwartz 11th ed., p. 1529.)

22. Which of the following is the most common etiology of Answer: C


splenic cyst worldwide? Splenic cysts are rare lesions. The most common etiology for
A. Bacterial infection splenic cysts worldwide is parasitic infestation, particularly
B. Trauma echinococcal. Symptomatic parasitic cysts are best treated
C. Parasitic infection with splenectomy, though selected cases may be amenable to
D. Congenital anomaly percutaneous aspiration, instillation of protoscolicidal agent,
and reaspiration. Nonparasitic cysts most commonly result
from trauma and are called pseudocysts; however, dermoid,
epidermoid, and epithelial cysts have been reported as well.
The treatment of nonparasitic cysts depends on whether or
not they produce symptoms. Asymptomatic nonparasitic
cysts may be observed with close ultrasound follow-up to
exclude significant expansion. Patients should be advised of
the risk of cyst rupture with even minor abdominal trauma

Brunicardi_Ch34_p297-306.indd 303 30/06/22 11:26 AM


304
if they elect nonoperative management for large cysts. Small
symptomatic nonparasitic cysts may be excised with splenic
preservation, and large symptomatic nonparasitic cysts may
be unroofed. Both of these operations may be performed lap-
aroscopically. (See Schwartz 11th ed., p. 1530.)

23. Which of the following is an indication for surgical treat- Answer: A


ment of a splenic aneurysm? Although rare, splenic artery aneurysm (SAA) is the most
A. Pregnancy common visceral artery aneurysm. Women are four times
CHAPTER 34

B. Size >1.5 cm more likely to be affected than men. The aneurysm usually
C. History of thrombocytopenia arises in the middle to distal portion of the splenic artery. The
D. History of neutropenia risk of rupture is between 3% and 9%; however, once rupture
occurs, mortality is substantial (35%–50%). According to a
recent series, mortality is significantly higher in patients with
underlying portal hypertension (>50%) than in those with-
out it (17%). SAA is particularly worrisome when discovered
The Spleen

during pregnancy, as rupture imparts a high risk of mortal-


ity to both mother (70%) and fetus (95%). Most patients are
asymptomatic and seek medical attention based on an inci-
dental radiographic finding. About 20% of patients with SAA
have symptoms of left upper quadrant pain. Indications for
treatment include presence of symptoms, pregnancy, inten-
tion to become pregnant, and pseudoaneurysms associated
with inflammatory processes. For asymptomatic patients,
size > 2 cm constitutes an indication for surgery. Aneurysm
resection or ligation alone is acceptable for amenable lesions
in the mid-splenic artery, but distal lesions in close proxim-
ity to the splenic hilum should be treated with concomitant
splenectomy. An excellent prognosis follows elective treat-
ment. Splenic artery embolization has been used to treat SAA,
but painful splenic infarction and abscess may follow. (See
Schwartz 11th ed., p. 1531.)

24. A 45-year-old man presents to the emergency depart- Answer: D


ment with emesis of bright red blood. Labs include HGB While portal hypertension is most commonly a result of cir-
10 g/dL, HCT 30%, platelets 300,000/mm3, INR 1.0, AST rhosis it can result from other causes such as splenic vein
30 U/L, ALT 45 U/L, and albumin 4.0 g/dL. After appro- thrombosis. Patients with splenic vein thrombosis can pres-
priate resuscitation, he undergoes esophagogastroduo- ent with bleeding from gastric varices in the setting of normal
denoscopy (EGD) which is notable for gastric varices. liver function and test results. These patients also often have a
What is the appropriate treatment for his condition? history of pancreatic disease. Portal hypertension secondary
A. Transjugular intrahepatic portosystemic shunt to splenic vein thrombosis is potentially curable with splenec-
B. Variceal band ligation tomy. (See Schwartz 11th ed., p. 1531.)
C. Splenorenal shunt
D. Splenectomy

25. Which of the following is NOT part of the triad seen with Answer: D
Felty syndrome? The triad of RA, splenomegaly, and neutropenia is called
A. Rheumatoid arthritis (RA) Felty syndrome. It exists in approximately 3% of all patients
B. Splenomegaly with RA, two-thirds of which are women. Immune complexes
C. Neutropenia coat the surface of white blood cells, leading to their seques-
D. Thrombocytopenia tration and clearance in the spleen with subsequent neutro-
penia. This neutropenia (<2000/mm3) increases the risk for
recurrent infections and often drives the decision for splenec-
tomy. The size of the spleen is variable, from nonpalpable in
5% to 10% of patients, to massive enlargement in others. The
spleen in Felty syndrome is four times heavier than normal.
Corticosteroids, hematopoietic growth factors, methotrex-
ate, and splenectomy have all been used to treat the neu-
tropenia of Felty syndrome. Responses to splenectomy have

Brunicardi_Ch34_p297-306.indd 304 30/06/22 11:26 AM


305
been excellent, with over 80% of patients showing a durable
increase in white blood cell count. More than one-half of
patients who had infections prior to surgery did not have any
infections after splenectomy. Besides symptomatic neutrope-
nia, other indications for splenectomy include transfusion-
dependent anemia and profound thrombocytopenia. (See
Schwartz 11th ed., p. 1532.)

26. Which of the following is the most effective preven- Answer: B

CHAPTER 34
tion strategy against overwhelming postsplenectomy Asplenic patients have an increased susceptibility to infection
infection? for the remainder of their lives and although the overall life-
A. Vaccination 2 weeks after splenectomy time risk of overwhelming postsplenectomy infection (OPSI)
B. Vaccination 2 weeks before splenectomy is low the consequences can be devastating. Patients under-
C. Daily antibiotic prophylaxis going splenectomy for hematologic or malignant indications
D. Carrying a reserve supply of antibiotics for self- have a greater risk of OPSI than patients undergoing sple-
administration nectomy for trauma or iatrogenic injury and OPSI is more

The Spleen
common in children than adults. Providers need to have a
high index of suspicion when evaluating asplenic patients
for possible infection. Patient education and vaccinations
against encapsulated pathogens is the mainstay of preventa-
tive therapy. Patients should be vaccinated 2 weeks prior to
elective splenectomy in order to optimize antigen recognition
and processing. If splenectomy is performed emergently, vac-
cinations are given postoperatively with an attempt to delay
administration for 2 weeks to avoid the transient immuno-
suppression associated with surgery. There is little evidence
supporting efficacy of prophylactic antibiotics in asplenic
patients and vaccination remains the most effective preven-
tion strategy. (See Schwartz 11th ed., p. 1532.)

27. All of the following are TRUE regarding laparoscopic Answer: B


splenectomy EXCEPT: Laparoscopic splenectomy has become the favored procedure
A. It is associated with shorter hospital stays. versus open splenectomy for elective splenectomy over the
B. It is associated with increased intraoperative blood past two decades and is now considered the gold standard for
loss. elective splenectomy in patients with normal-sized spleens.
C. It is associated with decreased morbidity. With experienced surgeons, laparoscopic splenectomy is
D. Patients are positioned in the right lateral decubitus associated with decreased intraoperative blood loss, shorter
position or the 45o right lateral decubitus position. hospital length of stay, and lower morbidity rates as compared
to open splenectomy. Laparoscopic splenectomy is often per-
formed with the patient in the right lateral decubitus position;
patients are sometimes placed in a 45o right lateral decubitus
position to facilitate easier access for concomitant procedures
such as laparoscopic cholecystectomy. (See Schwartz 11th ed.,
p. 1534.)

28. What is the most common complication following open Answer: B


splenectomy? Complications following splenectomy can be divided into
A. Pancreatitis pulmonary, hemorrhagic, infectious, pancreatic, and throm-
B. Left lower lobe atelectasis boembolic. Pulmonary complications include left lower lobe
C. Pleural effusion atelectasis, pleural effusion, and pneumonia with left lower
D. Wound infection lobe atelectasis being the most common complication overall.
Hemorrhagic complications include intraoperative hemor-
rhage, postoperative hemorrhage, and subphrenic hematoma.
Infectious complications include subphrenic abscess and
wound infection. Placement of a drain in the left upper quad-
rant can be associated with postoperative subphrenic abscess
and is therefore not routinely recommended. Pancreatic
complications include pancreatitis, pseudocyst formation,
and pancreatic fistula and often result from intraoperative

Brunicardi_Ch34_p297-306.indd 305 30/06/22 11:26 AM


306
trauma to the pancreas during dissection of the splenic hilum.
Thromboembolic complications include deep vein throm-
bosis and portal vein thrombosis. (See Schwartz 11th ed.,
p. 1539.)

29. Which of the following patients is at highest risk for Answer: C


overwhelming postsplenectomy infection (OPSI)? While the all lifetime risk of OPSI is low (ranging from <1%
A. A 30-year-old who underwent splenectomy for idio- to 5%) the consequences are serious. The reason for splenec-
pathic thrombocytopenic purpura tomy is the single most influential determinant of OPSI risk.
CHAPTER 34

B. A 25-year-old who underwent splenectomy for iatro- There is evidence that those who undergo splenectomy for
genic bleeding after a total colectomy hematologic disease are far more susceptible to OPSI than
C. A 3-year-old who underwent splenectomy for hered- patients who undergo splenectomy for trauma or iatrogenic
itary spherocytosis reasons. When taking age into consideration, children who
D. A 4-year-old who underwent splenectomy due to are 5 years of age or younger and adults who are 50 years of
bleeding after a motor vehicle crash age or older seem to be at an elevated risk. The interval since
splenectomy also seems to be a factor with the greatest risk
The Spleen

occurring in the first 2 years after splenectomy; however,


it is important to remember that cases of OPSI can occur
decades later and asplenic patients remain at lifelong risk.
(See Schwartz 11th ed., pp. 1538–1539.)

30. Which of the following asplenic patients should receive Answer: D


prophylactic antibiotic therapy to protect against over- Antibiotic therapy in asplenic patients falls into three cate-
whelming postsplenectomy infection (OPSI)? gories: deliberate therapy for established or presumed infec-
A. A 35-year-old man undergoing a tooth extraction. tions, prophylaxis in anticipation of invasive procedures,
B. A 4-year-old child who recently underwent and general prophylaxis. There is little evidence supporting
splenectomy. efficacy of prophylactic antibiotics in anticipation of invasive
C. A 15-year-old boy who underwent splenectomy at procedures or efficacy of general prophylaxis and guidelines
age 13. are not uniform. Common recommendations include daily
D. There is little evidence supporting efficacy of prophy- antibiotics until 5 years of age or at least 5 years after sple-
lactic antibiotics. nectomy with some advocating continuing antibiotics until
young adulthood; however, there is little evidence support-
ing efficacy. It is unlikely that randomized controlled trials on
this issue will be performed due to the low incidence of OPSI
as well as its serious consequences. (See Schwartz 11th ed.,
p. 1542.)

Brunicardi_Ch34_p297-306.indd 306 30/06/22 11:26 AM


CHAPTER 35
Abdominal Wall, Omentum, Mesentery,
and Retroperitoneum

1. A cutaneous malignancy of the anterior abdominal wall Answer: B


2 inches above the umbilicus will drain to which of the The lymphatic drainage of the anterior abdominal wall is
following lymphatic basins? principally to the axillary nodal basin and the inguinal nodal
A. Umbilical basin. The area of demarcation is roughly the arcuate line
B. Axillary (semilunar line of Douglas) at the level of the anterior iliac
C. Retroperitoneal spine. Malignancies above the arcuate line drain superiorly and
D. Inguinal those below the line drain inferiorly. (See Schwartz 11th ed.,
p. 1550.)

2. Materials used for mesh repair of an abdominal wall Answer: A


hernia include synthetic meshes and biologic meshes. Biologic mesh is derived from decellularized, collagen-
Which of the following statements is TRUE regarding rich porcine, bovine, or human tissue. The biologic mesh is
biologic mesh materials? designed to be absorbed over time after allowing for ingrowth
A. Biologic mesh is designed to be absorbed over time. of host tissues. They are more expensive than synthetic
B. Biologic mesh is manufactured from commercially mesh but may be advantageous in the setting of an infected
prepared algae. wound. Their benefit in preventing hernia recurrence is as yet
C. Biologic mesh is similar in cost to synthetic mesh unproven. (See Schwartz 11th ed., p. 1555.)
materials.
D. Biologic mesh is superior to synthetic mesh in pre-
venting hernia recurrence.

3. The appropriate treatment of rectus abdominis diastasis Answer: A


is: Rectus abdominis diastasis (or diastasis recti) is a separation
A. Observation. of the two rectus abdominis muscular pillars. This results in
B. Resection and primary repair. a bulge of the abdominal wall that is sometimes mistaken for
C. Mesh overlay. a ventral hernia despite the fact that the midline aponeurosis
D. Lateral component separation. is intact and no hernia defect is present. Computed tomog-
raphy (CT) scanning can provide an accurate measure of the
distance between the rectus pillars and will differentiate rec-
tus diastasis from a true ventral hernia. Surgical correction
has been described for cosmetic reasons but is unnecessary
and risks the formation of a true postoperative hernia. (See
Schwartz 11th ed., p. 1556.)

4. Postoperative incisional hernia of a midline incision is Answer: B


best prevented by which of the following suture tech- Classically, the midline fascia is closed with interrupted
niques for closure of an abdominal incision? 1 cm synthetic sutures placed 1 cm apart. Recent studies indi-
A. Widely spaced 1.5 to 2.0 cm interrupted mattress cate a reduced risk of incisional hernia with narrowly spaced
sutures sutures of 5 to 8 mm in length placed 5 mm apart. The value
B. Narrowly spaced 5 to 8 mm interrupted sutures of mesh reinforcement of midline closure is still under evalu-
C. A running suture of absorbable suture material ation. (See Schwartz 11th ed., pp. 1550–1551.)
D. A running suture of polypropylene suture material

307

Brunicardi_Ch35_p307-310.indd 307 30/06/22 11:26 AM


308
5. The use of mesh in the repair of an abdominal incisional Answer: D
hernia has become common for both open and laparo- The sublay technique has been found to be associated with a
scopic repairs. Which position of the mesh relative to the lower incidence of recurrent hernia and wound related com-
components of the abdominal wall is associated with the plications. (See Schwartz 11th ed., p. 1555.)
lowest hernia recurrence rate?
A. Overlay technique, where the mesh is fixed to the
anterior fascia above (superficial to) the repair
B. Underlay technique, where the mesh is placed in the
abdominal cavity under the repair
CHAPTER 35

C. Interlay technique, where the mesh is placed across


fascial defects below the anterior fascia
D. Sublay technique, where the mesh is placed under the
anterior fascia

6. A 40-year-old woman who underwent total abdominal Answer: D


colectomy for familial adenomatous polyposis (FAP) Desmoid tumors of the abdominal wall are fibrous neoplasms
Abdominal Wall, Omentum, Mesentery, and Retroperitoneum

5 years previously presents with a gradually expand- that occur sporadically or in the setting of FAP. The condi-
ing painless 4 cm mass of the anterior abdominal wall. tion can result in mortality due to aggressive local growth, so
A biopsy is returned as “desmoid tumor with no sign of radical excision with confirmation of tumor-free margins of
malignancy.” The correct management is: resection is required.
A. Observation. Medical treatment with an antineoplastic agent such as
B. A course of doxorubicin, dacarbazine, or carboplatin. doxorubicin, dacarbazine, or carboplatin can produce remis-
C. Enucleation. sion but the prognosis of advanced desmoids is poor. (See
D. Wide local excision. Schwartz 11th ed., p. 1557.)

7. The greater omentum, referred to as the “policeman of Answer: D


the abdomen,” includes all of the following EXCEPT: The greater omentum extends from the greater curve of the
A. The gastroepiploic vessels. stomach to the transverse colon and is supplied by the gastro-
B. The gastrosplenic ligament. epiploic vessels. It includes the gastrocolic and gastrosplenic
C. The gastrocolic ligament. ligaments. The lesser omentum extends from the lesser
D. The gastrohepatic ligament. curve of the stomach to the underside of the liver forming
the gastrohepatic ligament and the opening of the foramen of
Winslow. (See Schwartz 11th ed., p. 1558.)

8. Most tumors of the omentum are metastatic in nature. Answer: A


Which of the following primary tumor sites are most Primary tumors of the omentum are rare, but the omentum
commonly associated with omental metastases? is a common site of metastasis for ovarian carcinoma. Other
A. Ovarian tumors tumors are sometimes metastatic to the omentum including
B. Endometrial tumors renal cell carcinomas, endometrial cancer, gastrointestinal
C. Melanoma tumors, and melanoma. (See Schwartz 11th ed., p. 1559.)
D. Renal tumors

9. The mesentery of the small bowel is usually fixed in the Answer: A


left upper quadrant at the ligament of Treitz and the A narrow fixation point for the small bowel mesentery is usu-
right lower quadrant at the ileo-cecal valve. Defects in ally the cause of intestinal malrotation. Although this condi-
the rotation and fixation of the mesentery can result in a tion usually presents in infancy and early childhood, it may
narrow small bowel mesentery that allows all of the fol- go undetected until adulthood when unexplained attacks of
lowing complications to occur EXCEPT: abdominal pain occur. It can result in internal hernia forma-
A. Irritable bowel syndrome. tion and intermittent bowel obstruction but is not usually a
B. Intestinal malrotation with volvulus. cause of the symptoms that lead to a diagnosis of irritable
C. Internal hernia formation. bowel syndrome. (See Schwartz 11th ed., p. 1559.)
D. Recurrent attacks of acute abdominal pain.

Brunicardi_Ch35_p307-310.indd 308 30/06/22 11:26 AM


309
10. The retroperitoneum contains all of the following organs Answer: D
EXCEPT: The retroperitoneum contains the ascending and descend-
A. The aorta. ing colon, the duodenum and the pancreas, the inferior vena
B. The pancreas. cava, the aorta, the kidneys and the adrenal glands. It does not
C. The kidneys. contain the pelvic organs. (See Schwartz 11th ed., p. 1561.)
D. The ovaries.

11. Surgical infections of the retroperitoneum typically arise Answer: C


from all of the following conditions EXCEPT: Retroperitoneal infections typically arise from infections of

CHAPTER 35
A. Perforated retrocecal appendicitis. retroperitoneal organs such as the duodenum, the pancreas,
B. Infected acute pancreatitis. the retroperitoneal region behind the cecum, or the descend-
C. Crohn’s disease of the small bowel. ing colon. They do not typically complicate Crohn’s disease of
D. Diverticulitis. the small bowel or other intraperitoneal organs. (See Schwartz
11th ed., pp. 1561–1562.)

12. Retroperitoneal infections are notable due to their: Answer: A

Abdominal Wall, Omentum, Mesentery, and Retroperitoneum


A. Lack of abdominal signs and symptoms. Retroperitoneal infections and abscesses are difficult to diag-
B. Failure to be detected on computed tomography (CT) nose on history and physical but are usually apparent on CT
scan. scan. They carry a high risk of mortality and may require
C. Benign course. operative debridement if percutaneous drainage is infeasible
D. Association with tooth decay. or insufficient. Oropharyngeal infections are not typically
associated with retroperitoneal abscess except in patients with
severe immunodeficiency. (See Schwatrz 11th ed., p. 1562.)

13. Which of the following statements about sclerosing mes- Answer: A


enteritis is FALSE? The etiology of sclerosing mesenteritis is unknown but its car-
A. It is always associated with acute abdominal pain. dinal feature is increased tissue density within the mesentery.
B. It can appear as a mass on computed tomography This can be associated with a discreet nonneoplastic mass
(CT) scan. or it can be more diffuse involving large swaths of thickened
C. It can improve or resolve without surgical therapy. mesentery without well-defined borders. Most cases present
D. It can be mistaken for primary or metastatic tumor. with chronic abdominal pain but many cases are discovered
incidentally on CT scans performed for unrelated reasons.
The process is self-limited and may demonstrate regres-
sion on follow-up imaging studies. (See Schwartz 11th ed.,
pp. 1559–1560.)

14. The primary treatment of retroperitoneal fibrosis is: Answer: A


A. Corticosteroids. Once malignancy, drug-induced disease, and infectious
B. Cyclosporine. etiologies are ruled out, corticosteroids are the mainstay of
C. Radiation therapy. medical therapy. Surgical intervention is reserved for ureter-
D. Surgical resection. olysis or ureteral stenting, or endovascular interventions for
ileocaval obstruction. (See Schwartz 11th ed., p. 1563.)

Brunicardi_Ch35_p307-310.indd 309 30/06/22 11:26 AM


This page intentionally left blank

Brunicardi_Ch35_p307-310.indd 310 30/06/22 11:26 AM


CHAPTER 36
Soft Tissue Sarcomas

1. All of the following are TRUE about soft tissue sarcoma Answer: A
EXCEPT: Most primary soft tissue sarcomas originate in an extremity
A. Most common site is trunk and retroperitoneum. (50%–60%); the next most common sites the trunk (19%),
B. There are more than 11,000 new diagnoses of soft tis- retroperitoneum (15%), and head and neck (9%). The overall
sue sarcoma annually in the United States. 5-year survival rate for patients with all stages of soft tissue
C. Most soft tissue sarcoma-specific deaths are due to sarcoma is 50% to 60%. Of the patients who die of sarcoma,
uncontrolled pulmonary metastases. most succumb to lung metastasis; 80% of these occur within 2
D. The overall 5-year survival rate for all stages of soft to 3 years after initial diagnosis. In the United States in 2012,
tissue sarcoma approximates 50%–60%. approximately 11,280 new cases of soft tissue sarcoma were
diagnosed, and 3900 deaths were attributable to this disease.
The incidence rates are declining for most cancer sites, but
they are increasing among both men and women for mela-
noma of the skin, cancers of the liver, sarcoma, and thyroid.
(See Schwartz 11th ed., p. 1567.)

2. Which of the following is not associated with the devel- Answer: D


opment of sarcoma? External radiation therapy is a rare but well-established risk
A. Radiation exposure factor for soft tissue sarcoma that may be associated with
B. Herbicide exposure radiation-induced mutations of the p53 gene. Exposure to
C. Chronic lymphedema herbicides such as phenoxyacetic acid and to wood pre-
D. History of trauma servatives containing chlorophenols has been linked to an
increased risk of soft tissue sarcoma. In 1948, Stewart and
Treves first described the association between chronic lymph-
edema after axillary dissection and subsequent lymphan-
giosarcoma. Although patients with sarcoma often report a
history of trauma, no causal relationship has been established.
More often, a minor injury calls attention to a pre-existing
tumor. (See Schwartz 11th ed., p. 1568.)

3. All of the following are known molecular pathogenic


events in sarcoma EXCEPT: In general, sarcomas resulting from identifiable molecular
A. Chromosomal translocations. events tend to occur in younger patients with histology sug-
B. Oncogene amplification. gesting a clear line of differentiation. The identifiable molec-
C. Complex genomic rearrangements. ular events include point mutations, translocations causing
D. Proteomic suppression. overexpression of an autocrine grow factor, and oncogenic
fusion transcription factor producing a cellular environment
prone to malignant transformation. In contrast, sarcomas
without identifiable genetic changes or expression profile sig-
natures tend to occur in older patients and exhibit pleomor-
phic cytology and p53 dysfunction. (See Schwartz 11th ed.,
pp. 1569–1570.)

311

Brunicardi_Ch36_p311-312.indd 311 30/06/22 11:26 AM


312
4. Factors that are part of the American Joint Committee Answer: D
on Cancer (AJCC) sarcoma staging system include all of The AJCC staging system for soft tissue sarcomas is based on
the following EXCEPT: histologic grade, tumor size and depth, and the presence of
A. Tumor location. metastases. Histologic grade is the most important prognos-
B. Tumor size. tic factor for patients with soft tissue sarcoma. For accurate
C. Number of mitoses per high powered field. determination of grade, an adequate tissue sample must be
D. Ki-67 positivity. appropriately fixed, stained, and reviewed by an experienced
sarcoma pathologist. The features that define grade are cel-
lularity, differentiation (good, moderate, or poor/anaplastic),
CHAPTER 36

pleomorphism, necrosis, and number of mitoses per high-


power field. (See Schwartz 11th ed., p. 1573.)

5. Current therapy for American Joint Committee on Cancer Answer: B


(AJCC) Stage III soft tissue sarcoma of the extremity Primary tumors with no evidence of distant metastasis are
includes all of the following EXCEPT: managed with surgery alone or, when wide pathologic mar-
A. Chemotherapy. gins cannot be achieved because of anatomic constraints and/
Soft Tissue Sarcomas

B. Immunotherapy. or the grade is high, surgery plus radiation therapy. Radia-


C. Radiation therapy. tion therapy is part of the standard treatment for high-grade
D. Surgical excision. extremity and trunk wall soft tissue sarcomas either in the
pre- or postoperative setting. Because the evidence regarding
adjuvant systemic therapy for stage III soft tissue sarcoma is
inconclusive, considerable variation still exists in treatment
recommendations even though patients with large, stage II or
stage III soft tissue sarcomas are at high risk for recurrence
and metastasis. Chemotherapy may be considered to down-
stage large tumors to enable limb-sparing procedures, partic-
ularly for tumors known to be chemosensitive. (See Schwartz
11th ed., pp. 1574–1580.)

6. Concerning sarcoma of the breast, which of the following Answer: D


is TRUE? Angiosarcoma of the breast accounts for about 50% of all
A. Modified radical mastectomy is a standard treatment sarcomas of the breast and has increasingly been associated
for tumors > 5 cm. with radiation therapy for treatment of primary breast can-
B. Sentinel node biopsy is a standard treatment. cer. Complete excision with negative margins is the primary
C. Leiomyosarcoma is the most common histology of therapy. Simple mastectomy confers no additional benefit if
breast sarcoma. complete excision can be accomplished by segmental mas-
D. Lumpectomy is a standard treatment for T1 breast tectomy. Because of low rates of regional lymphatic spread,
sarcoma. axillary dissection is not routinely indicated. (See Schwartz
11th ed., p. 1584.)

Brunicardi_Ch36_p311-312.indd 312 30/06/22 11:26 AM


CHAPTER 37
Inguinal Hernias

1. The incidence of inguinal hernias in men has a bimodal Answer: B


distribution, which peaks before the: Approximately 75% of abdominal wall hernias occur in the
A. second year of life and after age 50. groin. The lifetime risk of inguinal hernia is 27% in men and
B. first year of life and after age 40. 3% in women. Of inguinal hernia repairs, 90% are performed
C. eighth year of life and after age 40. in men and 10% in women. The incidence of inguinal her-
D. fifth year of life and after age 50. nias in men has a bimodal distribution, with peaks before the
first year of age and after age 40. Abramson demonstrated the
age dependence of inguinal hernias in 1978. Those age 25 to
34 years had a lifetime prevalence rate of 15%, whereas those
age ≥ 75 years had a rate of 47% (Table 37-1). Approximately
70% of femoral hernia repairs are performed in women;
however, inguinal hernias are five times more common than
femoral hernias. The most common subtype of groin her-
nia in men and women is the indirect inguinal hernia. (See
Schwartz 11th ed., p. 1599.)

TABLE 37-1 Inguinal hernia prevalence by age


Age (Y) 25–34 35–44 45–54 55–64 65–74 75+
Current prevalence (%) 12 15 20 26 29 34
Lifetime prevalence (%) 15 19 28 34 40 47
Current = repaired hernias excluded; lifetime = repaired hernias included.

2. According to the Nyhus classification system, which cat- Answer: D


egorizes hernia defects by location, size, and type, type
IIIC represents: TABLE 37-2 Nyhus classification system
A. Indirect hernia internal abdominal ring normal; Type I Indirect hernia; internal abdominal ring normal; typically
typical in infants, children, small adults. in infants, children, small adults
B. Direct hernia; size is not taken into account. Type II Indirect hernia; internal ring enlarged without
C. Recurrent hernia; modifiers A–D are sometimes impingement on the floor of the inguinal canal;
added, which correspond to indirect, direct, femoral, does not extend to the scrotum
and mixed. Type IIIA Direct hernia; size is not taken into account
D. Femoral hernia. Type IIIB Indirect hernia that has enlarged enough to encroach
upon the posterior inguinal wall; indirect sliding or
scrotal hernias are usually placed in this category
because they are commonly associated with extension
to the direct space; also includes pantaloon hernias
Type IIIC Femoral hernia
Type IV Recurrent hernia; modifiers A–D are sometimes added,
which correspond to indirect, direct, femoral, and mixed,
respectively. (See Schwartz 11th ed., Table 37-2, p. 1602.)

313

Brunicardi_Ch37_p313-322.indd 313 30/06/22 11:27 AM


314
3. All of the following are TRUE regarding an incidence Answer: D
of acquired inguinal hernias EXCEPT: Several studies have documented strenuous physical activity
A. Increases with strenuous activity. as a risk factor for acquired inguinal hernia. A case-
B. Increases with family history. controlled study of over 1400 male patients with inguinal her-
C. Increases with chronic obstructive pulmonary dis- nia revealed that a positive family history was associated with
ease (COPD). an eightfold lifetime incidence of inguinal hernia. Chronic
D. Increases with obesity. obstructive pulmonary disease also significantly increases the
risk of direct inguinal hernias, thought to be due to repeated
instances of intra-abdominal pressure during coughing. Sev-
CHAPTER 37

eral studies have suggested a protective effect of obesity. In


a large, population-based prospective study of American
individuals (First National Health and Nutrition Examina-
tion Survey), the risk of inguinal hernia development in
obese men was only 50% that of normal-weight men, while
the risk in overweight men was 80% that of nonobese men.
A possible explanation is the increased difficulty in detecting
Inguinal Hernias

inguinal hernias in obese individuals. (See Schwartz 11th ed.,


Table 37-2, p. 1604.)

4. The high incidence of inguinal hernias in preterm babies Answer: A


is most often due to: Inguinal hernias may be congenital or acquired. Most adult
A. Failure of the peritoneum to close. inguinal hernias are considered acquired defects in the
B. Familial history. abdominal wall although collagen studies have demonstrated
C. Female gender. a heritable predisposition. A number of studies have attempted
D. Developmental dysplasia of the hip. to delineate the precise causes of inguinal hernia formation;
however, the best-characterized risk factor is weakness in the
abdominal wall musculature (Table 37-3). Congenital her-
nias, which make up the majority of pediatric hernias, can
be considered an impedance of normal development, rather
than an acquired weakness. During the normal course of
development, the testes descend from the intra-abdominal
space into the scrotum in the third trimester. Their descent
is preceded by the gubernaculum and a diverticulum of peri-
toneum, which protrudes through the inguinal canal and
becomes the processus vaginalis. Between 36 and 40 weeks
of gestation, the processus vaginalis closes and eliminates the
peritoneal opening at the internal inguinal ring. Failure of the
peritoneum to close results in a patent processus vaginalis
(PPV), hence the high incidence of indirect inguinal hernias

TABLE 37-3 Presumed causes of groin herniation


Coughing
Chronic obstructive pulmonary disease
Obesity
Straining
Constipation
Prostatism
Pregnancy
Birthweight <1500 g
Family history of a hernia
Valsalva’s maneuver
Ascites
Upright position
Congenital connective tissue disorders
Defective collagen synthesis
Previous right lower quadrant incision
Arterial aneurysms
Cigarette smoking
Heavy lifting
Physical exertion

Brunicardi_Ch37_p313-322.indd 314 30/06/22 11:27 AM


315
in preterm babies. Children with congenital indirect inguinal
hernias will present with a PPV; however, a patent processus
does not necessarily indicate an inguinal hernia (Fig. 37-1). In
a study of nearly 600 adults undergoing general laparoscopy,
bilateral inspection revealed that 12% had PPV. None of these
patients had clinically significant symptoms of a groin hernia.
In a group of 300 patients undergoing unilateral laparoscopic
inguinal hernia repair, 12% were found to have a contralateral
PPV, which was associated with a fourfold 5-year incidence of

CHAPTER 37
inguinal hernia.(See Schwartz 11th ed., p. 11604.)

Inguinal Hernias
FIG. 37-1. Varying degrees of closure of the processus
vaginalis (PV). A. Closed PV. B. Minimally patent PV.
C. Moderately patent PV. D. Scrotal hernia.

5. The two types of collagen found to exist in a decreased Answer: C


ratio of the skin of inguinal hernia patients are: Epidemiologic studies have identified risk factors that may
A. Types I and II. predispose to a hernia. Microscopic examination of skin of
B. Types II and III. inguinal hernia patients demonstrated significantly decreased
C. Types I and III. ratios of type I to type III collagen. Type III collagen does not
D. Types III and VI. contribute to wound tensile strength as significantly as type I
collagen. Additional analyses revealed disaggregated collagen
tracts with decreased collagen fiber density in hernia patients’
skin. Collagen disorders such as Ehlers-Danlos syndrome are
also associated with an increased incidence of hernia forma-
tion (Table 37-4). Recent studies have found an association
between concentrations of extracellular matrix elements and
hernia formation. Although a significant amount of work
remains to elucidate the biologic nature of hernias, current
evidence suggests they have a multifactorial etiology with
both environmental and hereditary influences. (See Schwartz
11th ed., p. 1605.)

Brunicardi_Ch37_p313-322.indd 315 30/06/22 11:27 AM


316
TABLE 37-4 Connective tissue disorders associated
with groin herniation
Osteogenesis imperfecta
Cutis laxa (congenital elastolysis)
Ehlers-Danlos syndrome
Hurler-Hunter syndrome
Marfan syndrome
Congenital hip dislocation in children
Polycystic kidney disease
α1-Antitrypsin deficiency
CHAPTER 37

Williams syndrome
Androgen insensitivity syndrome
Robinow syndrome
Serpentine fibula syndrome
Alport syndrome
Tel Hashomer camptodactyly syndrome
Leriche syndrome
Testicular feminization syndrome
Inguinal Hernias

Rokitansky-Mayer-Küster syndrome
Goldenhar syndrome
Morris syndrome
Gerhardt syndrome
Menkes syndrome
Kawasaki disease
Pfannenstiel syndrome
Beckwith-Wiedemann syndrome
Rubinstein-Taybi syndrome
Alopecia-photophobia syndrome

6. Use of antibiotics in open repair of inguinal hernias: Answer: C


A. Is supported by multiple randomized studies. The debate as to whether or not to administer preoperative
B. Has a larger impact when mesh is not used. prophylactic antibiotics in elective inguinal hernia repair still
C. There is no universal guideline for open elective her- remains controversial as elective hernia repair is considered
nia repair. a clean procedure and as such are exempt from SCIP surgi-
D. Is a part of Surgical Care Improvement Project (SCIP) cal prophylaxis guidelines. A Cochrane review of 17 random-
surgical prophylaxis guidelines. ized controlled trials in 2012 revealed an overall decrease
in infection rates (3.1% vs 4.5%, odds ratio [OR] 0.64, 95%
confidence interval [CI] 0.50–0.82) when prophylactic anti-
biotics are administered in patients. In subgroup analyses, the
difference was smaller in patients without mesh placement
(3.5% vs 4.9%, OR 0.71, 95% CI 0.51–1.00) than in those with
mesh placement (2.4% vs 4.2%, OR 0.56, 95% CI 0.38–0.81).
However, with inguinal hernia repair, overall wound infec-
tion rates were higher than those expected for clean opera-
tions, as a result, they were unable to definitively recommend
for or against antimicrobial prophylaxis. Although there is no
universal guideline regarding the administration of prophy-
lactic antibiotics for open elective hernia repair, the routine
indexing of cases for quality improvement databases have
resulted in the routine administration of prophylactic peri-
operative antibiotics in inguinal hernia repairs. (See Schwartz
11th ed., p. 1609.)

7. A hernia sac that extends into the scrotum may require: Answer: B
A. extensive dissection and reduction. In cases where the viability of sac contents is in question, the
B. division within the inguinal canal. sac should be incised, and hernia contents should be evalu-
C. amputation of the sac. ated for signs of ischemia. The defect should be enlarged to
D. the sac to be inverted into the preperitoneum. augment blood flow to the sac contents. Viable contents may
be reduced into the peritoneal cavity, while nonviable con-
tents should be resected, and synthetic prostheses should
be avoided in the repair. In elective cases, the sac may be

Brunicardi_Ch37_p313-322.indd 316 30/06/22 11:27 AM


317
amputated at the internal inguinal ring or inverted into the
preperitoneum. Both methods are effective; however, patients
undergoing sac excision had significantly increased postoper-
ative pain in a prospective trial. Dissection of a densely adher-
ent sac may result in injury to cord structures and should be
avoided; however, sac ligation at the internal inguinal ring is
necessary in these cases. A hernia sac that extends into the
scrotum may require division within the inguinal canal, as
extensive dissection and reduction risks injury to the pampi-

CHAPTER 37
niform plexus, resulting in testicular atrophy and orchitis.
At this point, the inguinal canal is reconstructed, either
with native tissue or with prostheses. The following sections
describe the most commonly performed types of tissue-based
and prosthetic-based reconstructions. (See Schwartz 11th ed.,
p. 1610.)

Inguinal Hernias
8. The technique indicated for femoral hernias in cases Answer: C
where prosthetic material is contraindicated is: The McVay repair addresses both inguinal and femoral ring
A. The Bassini repair. defects. This technique is indicated for femoral hernias and in
B. The Shouldice repair. cases where the use of prosthetic material is contraindicated
C. The McVay repair. (Fig. 37-2). Once the spermatic cord has been isolated, an
D. Lichtenstein tension-free repair. incision in the transversalis fascia permits entry into the pre-
peritoneal space. The upper flap is mobilized by gentle blunt
dissection of underlying tissue. Cooper’s ligament is bluntly
dissected to expose its surface. A 2- to 4-cm relaxing incision
is made in the anterior rectus sheath vertically from the pubic
tubercle. This incision is essential to reduce tension on the
repair; however, it may result in increased postoperative pain
and higher risk of ventral abdominal herniation. Using either
interrupted or continuous suture, the superior transversalis
flap is then fastened to Cooper ligament, and the repair is con-
tinued laterally along Cooper ligament to occlude the femoral
ring. Lateral to the femoral ring, a transition stitch is placed,
affixing the transversalis fascia to the inguinal ligament. The
transversalis is then sutured to the inguinal ligament laterally
to the internal ring. (See Schwartz 11th ed., p. 1610.)

FIG. 37-2. McVay Cooper ligament repair. Cooper’s ligament

Brunicardi_Ch37_p313-322.indd 317 30/06/22 11:27 AM


318
9. Desarda repair of an inguinal hernia utilizes: Answer: C
A. Internal oblique, transversus abdominis and trans- The Desarda hernia repair was recently described in 2001,
versalis fascia. and it consists of a mesh-free repair utilizing a strip of exter-
B. Cooper ligament. nal oblique aponeurosis. An oblique skin incision is made,
C. A strip of the external oblique aponeurosis. and dissection is carried down to the external oblique fascia.
D. Prosthetic mesh. The integrity of the fascia is preserved as much as possible.
The cremasteric muscle is then incised, and the spermatic
cord along with the cremasteric muscle is separated from the
inguinal floor. Excision of the sac is done in all cases except
CHAPTER 37

in small direct hernias, where it is inverted. The medial leaf


of the external oblique aponeurosis is sutured to the ingui-
nal ligament from the pubic tubercle to the abdominal ring
using 1–0 ethilon or prolene interrupted sutures. The first
two sutures are taken at the junction of the anterior rectus
sheath and esophageal obturator airway (EOA). The last
suture is taken so as to sufficiently narrow the abdominal
Inguinal Hernias

ring without constricting the spermatic cord (Fig. 37-3). Each


suture is passed first through the inguinal ligament, then the
transversalis fascia, and then the EOA. The index finger of the
left hand is used to protect the femoral vessels and retract
the cord structures laterally while taking lateral sutures. A
splitting incision is then taken in the EOA, partially separat-
ing a strip. This splitting incision is extended medially up to
the pubic symphysis and laterally 1 to 2 cm beyond the recon-
structed abdominal ring. (See Schwartz 11th ed., p. 1611.)

1
1
2
6
4 3
2
5 7

6 4
8
3 5

FIG. 37-3. The Desarda repair. A. The medial leaf of the external oblique aponeurosis is sutured to the inguinal ligament. 1 Medial leaf,
2 interrupted sutures taken to suture the medial leaf to the inguinal ligament, 3 pubic tubercle, 4 abdominal ring, 5 spermatic cord,
6 lateral leaf. B. Undetached strip of the external oblique aponeurosis forming the posterior wall. 1 Reflected medial leaf after a strip has
been separated, 2 internal oblique muscle seen through the splitting incision made in the medial leaf, 3 interrupted sutures between
the upper border of the strip and conjoined muscle and internal oblique muscle, 4 interrupted sutures between the lower border
of the strip and the inguinal ligament, 5 pubic tubercle, 6 abdominal ring, 7 spermatic cord, 8 lateral leaf.

Brunicardi_Ch37_p313-322.indd 318 30/06/22 11:27 AM


319
10. Robotic-assisted inguinal hernia repair: Answer: C
A. Does not decrease in cost as the volume increases at Retrospective data have had mixed results when compar-
each center. ing robot-assisted surgery vs laparoscopy. One recent study
B. Cost of robotic and laparoscopic repairs converge as has shown longer operative time, another analysis has shown
volume grows at each center. increased cost. It should be noted, however, that there is a
C. Complications are likely to be less in obese patients decrease in cost with robotic surgery as the volume of proce-
compared to open repair. dures increases at each center, though it is still unlikely that
D. Is less suited than laparoscopic repair for recurrent the costs will ever converge to that of laparoscopic surgery.
hernias. A retrospective, single-institution study has shown greatly

CHAPTER 37
reduced complication rates with robotic-assisted surgery in
obese patients; however, this was compared against open
inguinal hernia repair (10.8% vs 3.2%, P = 0.047), the two
groups were covariate matched for preoperative risk. Studies
have also shown excellent long-term (36-month) quality of
life indicators in robot-assisted transabdominal preperitoneal
(TAPP), though this was a single surgeon survey. Further ran-

Inguinal Hernias
domized trials will shed more light into cost issues as surgeons
gain more experience with robotic application that would lead
to shorter operative time and minimize additional instrument
use. (See Schwartz 11th ed., p. 1616.)

11. The medical issue NOT associated with hernia recur- Answer: D
rence is: Hernia recurrence: When a patient develops pain, bulging,
A. Malnutrition. or a mass at the site of an inguinal hernia repair, clinical
B. Steroid use. entities such as seroma, persistent cord lipoma, and hernia
C. Smoking. recurrence should be considered. Common medical issues
D. Alcohol use. associated with recurrence include malnutrition, immuno-
suppression, diabetes, steroid use, and smoking. Technical
causes of recurrence include improper mesh size, tissue isch-
emia, infection, and tension in the reconstruction. A focused
physical examination should be performed. As with primary
hernias, ultrasound (US), computed tomography (CT), or
magnetic resonance imaging (MRI) can elucidate ambiguous
physical findings. When a recurrent hernia is discovered and
warrants re-operation, an approach through a virgin plane
facilitates its dissection and exposure. Extensive dissection of
the scarred field and mesh may result in injury to cord struc-
tures, viscera, large blood vessels, and nerves. After an initial
anterior approach, the posterior laparoscopic approach will
usually be easier and more effective than another anterior
dissection. Conversely, failed preperitoneal repairs should
be approached using an open anterior repair. (See Schwartz
11th ed., p. 1618.)

12. Fibrin glue fixation of a synthetic prosthesis is: Answer: B


A. Vastly inferior to the suture fixation. Fibrin glue fixation is a successful alternative to tack fixation
B. Causes less pain than tacker fixation. in hernia repair with a synthetic prosthesis. Recent studies
C. More likely to cause a hernia recurrence compared to comparing fibrin glue fixation and suture fixation in open
tacker fixation. hernia repair show superior rates of chronic pain with both
D. Best for larger hernias. Lichtenstein and Plug and Patch techniques. Meta-analyses
of endoscopic hernia repair determined the incidence of
chronic postoperative pain after tacker fixation was signifi-
cantly higher than after fibrin glue fixation, with one showing
a relative risk of 4.64 (CI 1.9–11.7). Rates of other postop-
erative complications and recurrence were similar between
both fixation methods. Glue fixation is a promising technical
refinement, and several studies have shown long-term ben-
efit; however, its questionable efficacy in larger hernias and
cost remain considerations. (See Schwartz 11th ed., p. 1618.)

Brunicardi_Ch37_p313-322.indd 319 30/06/22 11:27 AM


320
13. Nonciceptive pain is: Answer: A
A. The result of ligamentous or muscular trauma and Pain after inguinal hernia repair is classified into acute or
inflammation. chronic manifestations of three mechanisms: nociceptive
B. The result of direct nerve damage or entrapment. (somatic), neuropathic, and visceral pain. Nociceptive pain is
C. The result of pain conveyed through afferent auto- the most common of the three. Because it is usually a result of
nomic pain fibers. ligamentous or muscular trauma and inflammation, nocicep-
D. Characterized as diffuse. tive pain is reproduced with abdominal muscle contraction.
Treatment consists of rest, nonsteroidal anti-inflammatory
drugs (NSAIDs), and reassurance, as it resolves spontane-
CHAPTER 37

ously in most cases. Neuropathic pain occurs as a result of


direct nerve damage or entrapment. It may present early or
late, and it manifests as a localized, sharp, burning or tear-
ing sensation. It may respond to pharmacologic therapy and
to local steroid or anesthetic injections when indicated. Vis-
ceral pain refers to pain conveyed through afferent autonomic
pain fibers. It is usually poorly localized and may occur dur-
Inguinal Hernias

ing ejaculation as a result of sympathetic plexus injury. (See


Schwartz 11th ed., p. 1619.)

14. Injury to the lateral femoral cutaneous nerve results in: Answer: C
A. Inguinodynia. Other chronic pain syndromes include local nerve entrap-
B. Osteitis pubis. ment, meralgia paresthetica, and osteitis pubis. At greatest
C. Meralgia paresthetica. risk of entrapment are the ilioinguinal and iliohypogastric
D. Nerve entrapment. nerves in anterior repairs and the genitofemoral and lateral
femoral cutaneous nerves in laparoscopic repairs. Clinical
manifestations of nerve entrapment mimic acute neuropathic
pain, and they occur with a dermatomal distribution. Injury
to the lateral femoral cutaneous nerve results in meralgia par-
esthetica, a condition characterized by persistent paresthesias
of the lateral thigh. Initial treatment of nerve entrapment
consists of rest, ice, nonsteroidal anti-inflammatory drugs
(NSAIDs), physical therapy, and possible local corticosteroid
and anesthetic injection. Osteitis pubis is characterized by
inflammation of the pubic symphysis and usually presents as
medial groin or symphyseal pain that is reproduced by thigh
adduction. Avoiding the pubic periosteum when placing
sutures and tacks reduces the risk of developing osteitis pubis.
Computed tomography (CT) scan or magnetic resonance
imaging (MRI) excludes hernia recurrence, and bone scan is
confirmatory for the diagnosis. Initial treatment is identical
to that of nerve entrapment; however, if pain remains intrac-
table, orthopedic surgery consultation should be sought for
possible bone resection and curettage. Irrespective of treat-
ment, the condition often takes 6 months to resolve. (See
Schwartz 11th ed., p. 1619.)

15. Ischemic orchitis at the time of an inguinal hernia is: Answer: A


A. Most commonly caused by an injury to the pampini- Injury to spermatic cord structures may result in ischemic
form plexus. orchitis or testicular atrophy. Ischemic orchitis is most com-
B. Most commonly caused by an injury to testicular monly caused by injury to the pampiniform plexus and not
artery. to the testicular artery. It usually manifests within 1 week of
C. Manifests immediately after the injury. inguinal hernia repair as an enlarged, indurated, and pain-
D. Occurs in >1% of primary hernia repair. ful testis, and it is almost certainly self-limited. It occurs in
<1% of primary hernia repairs; however, this figure is larger
for recurrent inguinal hernia repairs. Ultrasound (US) will
demonstrate testicular blood flow to differentiate between
ischemia and necrosis. Emergent orchiectomy is only nec-
essary in the case of necrosis. Injury to the testicular artery
itself may lead to testicular atrophy, which is manifest over

Brunicardi_Ch37_p313-322.indd 320 30/06/22 11:27 AM


321
a protracted period but does not always lead to testicular
necrosis. This is because despite compromise of the artery,
there is collateral flow from the inferior epigastric, vesi-
cal, prostatic, and scrotal arteries that supply the testes, and
in the case of insufficiency, there is atrophy. Treatment for
ischemic orchitis most frequently consists of reassurance,
nonsteroidal anti-inflammatory drugs (NSAIDs), and comfort
measures. Intraoperatively, proximal ligation of large hernia
sacs to avoid cord manipulation minimizes the risk of injury.

CHAPTER 37
(See Schwartz 11th ed., p. 1620.)

16. Regarding recurrences after a hernia repair, which one is Answer: A


TRUE? A 2012 meta-analysis from the Cochrane database demon-
A. Among tissue repair, Shouldice repair is associated strated significantly lower rates of hernia recurrence (OR 0.62,
with the least incidence of recurrence. CI 0.45–0.85) in patients undergoing Shouldice operations
B. Shouldice repair is primarily of historical significance when compared with other open tissue-based methods. In

Inguinal Hernias
and is rarely performed. experienced hands, the overall recurrence rate for the Shoul-
C. Compared with mesh repairs, Shouldice operation dice repair is about 1%. Although it is an elegant procedure,
has a lower incidence of recurrence. its meticulous nature requires significant technical expertise
D. Shouldice technique may be reliably replicated by a to achieve favorable outcomes, and it is associated with longer
novice surgeon. operative duration and longer hospital stay. One study found
the recurrence rate for Shouldice repairs decreased from
9.4% to 2.5% after surgeons performed the repair six times.
Compared with mesh repairs, the Shouldice technique
resulted in significantly higher rates of recurrence (OR 3.65,
CI 1.79–7.47); however, it is the most effective tissue-based
repair when mesh is unavailable or contraindicated. (See
Schwartz 11th ed., p. 1621.)

17. The outcome found more commonly with transabdomi- Answer: C


nal preperitoneal (TAPP) repair compared to totally Although controversy persists regarding the utility of TEP vs
extraperitoneal (TEP) repair is: TAPP, reviews to date find no significant differences in opera-
A. Length of stay. tive duration, length of stay, time to recovery, or short-term
B. Time to recovery. recurrence rate between the two approaches. In TAPP repair,
C. Risk of intra-abdominal injuries. the risk of intra-abdominal injury is higher than in TEP repair.
D. Higher short-term recurrence rates. This finding prompted the International Endohernia Society
(IEHS) to recommend that TAPP should only be attempted
by surgeons with sufficient experience. A Cochrane system-
atic review found that rates of port-site hernias and visceral
injuries were higher for the TAPP technique, whereas TEP
may be associated with a higher rate of conversion to an alter-
native approach; however, neither finding was sufficiently
compelling to recommend one technique over the other. (See
Schwartz 11th ed., p. 1622.)

Brunicardi_Ch37_p313-322.indd 321 30/06/22 11:27 AM


This page intentionally left blank

Brunicardi_Ch37_p313-322.indd 322 30/06/22 11:27 AM


CHAPTER 38
Thyroid, Parathyroid, and Adrenal

1. What condition occurs as a complication of total thy- Answer: B


roidectomy in patients, particularly in children, who do Myxedema, with the development of cretinous features, was
not receive thyroid replacement therapy? an early complication of total thyroidectomy in children.
A. Sarcoidosis It is prevented by thyroid hormone replacement therapy.
B. Myxedema (See Schwartz 11th ed., p. 1625.)
C. Dysphonia
D. Pseudohyperparathyroidism

2. What congenital anomaly arises from the formation of Answer: C


the thyroid gland? The medial thyroid anlage descends from the base of the
A. The thyroid isthmus tongue through a channel called the thyroglossal duct at
B. The cricothyroid arch week 3–4 of gestation. The duct normally closes after its
C. A thyroglossal duct cyst descent, but may remain patent and is susceptible to second-
D. An endobranchial cyst ary infection and dilatation, referred to as a thyroglossal duct
cyst. Removal is accomplished with the Sistrunk operation,
which also removes the central portion of the hyoid bone.
(See Schwartz 11th ed., pp. 1625–1626.)

3. The arterial supply of the thyroid arises from which of Answer: D


the following vessels? The superior thyroid arteries arise from the external carotid
A. The aorta arteries, and the inferior thyroid arteries arise from the
B. The external carotid arteries ­thyrocervical trunk shortly after their origin from the sub-
C. The thyrocervical trunk clavian arteries. A thyroid ima artery arises directly from the
D. All of the above aorta or innominate artery in 1% to 4% of cases. (See Schwartz
11th ed., p. 1628.)

4. In what location, relative to the inferior thyroid artery Answer: D


(ITA), is the recurrent laryngeal nerve (RLN) found? The RLN courses within the tracheoesophageal groove after
A. Medial or posterior to the ITA emerging from the vagus nerve at the level of the aortic arch.
B. Lateral or anterior to the ITA As it ascends in the neck, the RLN may branch, and may pass
C. Passing between the branches of the ITA anterior, posterior, or interdigitate with branches of the ITA.
D. All of the above The location of the RLN must be confirmed before the ITA is
divided. (See Schwartz 11th ed., pp. 1628–1629.)

5. Although injury to the recurrent laryngeal nerve results in Answer: A


hoarseness (unilateral injury) or airway obstruction (bilat- The external branch of the SLN lies on the inferior pharyngeal
eral injury), injury to the superior laryngeal nerve (SLN) constrictor muscle and descends alongside the superior thyroid
results in a more subtle injury, affecting the ability to: vessels before innervating the cricothyroid muscle. Therefore
A. Sing high notes. the superior pole vessels should not be ligated en masse, but
B. Whisper. should be individually divided low on the thyroid gland. Injury
C. Protrude the tongue. to the SLN leads to inability to tense the ipsilateral vocal cord,
D. Grimace. and impairs the ability to “hit high notes” while singing, or pro-
jecting the voice loudly. (See Schwartz 11th ed., pp. 1628–1629.)
323

Brunicardi_Ch38_p323-328.indd 323 05/07/22 1:32 PM


324
6. Thyroid hormones (T3 and T4) have regulatory roles in Answer: D
all of the following EXCEPT: Thyroid hormones are responsible for maintaining the nor-
A. The hypoxia and hypercapnia drives of the respira- mal hypoxic and hypercapnic drive in the respiratory center of
tory center in the brain. the brain, and regulate cholesterol synthesis and degradation.
B. Cholesterol synthesis and degradation. They also regulate bone and protein turnover and the speed
C. The speed of muscle contraction and relaxation. of muscle contraction and regulation, gastrointestinal motil-
D. Auditory acuity. ity, hepatic gluconeogenesis, and intestinal glucose absorp-
tion but are not regulators of auditory acuity. (See Schwartz
11th ed., pp. 1632–1633.)
Chapter 38

7. Which hormone directly mediates the release of thyroid Answer: C


hormones from the gland? Thyroglobulin is contained within the follicles of the thyroid
A. Thyroglobulin gland and its tyrosine residues are iodinated to produce tetra-
B. Thyrotropin-releasing hormone (TRH) iodothyronine or thyroxin (T4) or triiodothyronine (T3).
C. Thyroid Stimulating Hormone (TSH) TRH is secreted by the hypothalamus and stimulates the pitu-
D. Adrenalin itary gland to secrete TSH which directly mediates the release
Thyroid, Parathyroid, and Adrenal

of T4 and T3 from the gland. Adrenalin does not directly


regulate T4 or T3 release. (See Schwartz 11th ed., p. 1630.)

8. Near-total or total thyroidectomy is preferred for the Answer: D


treatment of Graves disease: Near-total or total thyroidectomy is now preferred over sub-
A. When radioactive iodine therapy is contraindicated. total thyroidectomy due to a lower recurrence rate. Surgery
B. When the goiter is large (>80 g) or airway obstruc- is preferred over medical therapy (radioactive iodine) in
tion appears immanent. childbearing women who desire to have children in the near
C. In patients with demonstrated poor compliance with future, in non-compliant patients, or when airway obstruc-
or tolerance of anti-thyroid medications. tion appears likely. (See Schwartz 11th ed., p. 1637.)
D. All of the above.

9. What is the recommended course of action when fine Answer: B


needle aspiration biopsy (FNAB) of a thyroid nodule is Most papillary and other cancers can be diagnosed by FNAB
interpreted as “suspicious for malignancy”? but the features can be subtle and lobectomy or near-total
A. Repeat FNAB thyroidectomy is recommended because 60% to 75% turn
B. Lobectomy out to be malignant. Follicular neoplasms of the thyroid are
C. Lobectomy and isthmusectomy less aggressive than papillary neoplasms, and an FNAB may
D. Total thyroidectomy be unable to differentiate between a follicular adenoma and
a follicular carcinoma. For this reason unilateral lobectomy
is recommended for this FNAB diagnosis. (See Schwartz
11th ed., p. 1644.)

10. Which diseases are associated with germline mutations Answer: D


in the RET tyrosine kinase receptor gene? Mutations in the extracellular domain of the RET tyrosine
A. Multiple endocrine neoplasia type 2A (MEN2A) kinase receptor are associated with MEN2A, familial med-
B. Multiple endocrine neoplasia type 2B (MEN2B) ullary thyroid cancer (FMTC), and Hirschsprung disease.
C. Hirschsprung Disease Mutations in the intracellular domain are associated with
D. All of the above MEN2B, FMTC, and Hirschsprung disease. (See Schwartz
11th ed., pp. 1645–1646.)

11. Children exposed to the Chernoble disaster in 1986 sub- Answer: A


sequently demonstrated an increased incidence of which Papillary thyroid cancer accounts for 80% of all thyroid malig-
thyroid cancer? nancies and is the predominant thyroid cancer in children
A. Papillary thyroid cancer (PTC) and individuals exposed to external radiation. (See Schwartz
B. Follicular thyroid cancer (FTC) 11th ed., p. 1647.)
C. Medullary thyroid cancer (MTC)
D. Anaplastic thyroid cancer (ATC)

Brunicardi_Ch38_p323-328.indd 324 05/07/22 1:32 PM


325
12. The recommended treatment for an otherwise healthy Answer: D
50-year-old man with a 2 cm follicular thyroid cancer in Total thyroidectomy is the recommended treatment particu-
the left lobe diagnosed by fine needle aspiration biopsy larly in younger patients. Prophylactic neck dissection is not
(FNAB) is: required unless nodal involvement is suspected. Recurrent
A. Left lobectomy. disease can be treated with radioiodine therapy. (See Schwartz
B. Left lobectomy and isthmusectomy. 11th ed., p. 1651.)
C. Total left lobectomy and subtotal right lobectomy.
D. Total thyroidectomy.

Chapter 38
13. An adolescent patient with a thyroid mass undergoes Answer: D
fine needle aspiration biopsy which returns as medullary MCT can be spontaneous (in 75%) or familial (in 25%) in
carcinoma of the thyroid (MCT). What other diseases multiple endocrine neoplasia syndrome type 2 (MEN2).
should be screened for before treatment is undertaken? MEN2A is associated with pheochromocytoma and hyper-
A. Hyperparathryroidism parathyroidism, whereas MEN2B is associated with pheo-
B. Pheochromocytoma chromocytoma, Marfanoid habitus, and mucocutaneous
C. Mucocutaneous ganglioneuromas ganglioneuromas. (See Schwartz 11th ed., p. 1655.)

Thyroid, Parathyroid, and Adrenal


D. All of the above

14. An asymptomatic child with a normal physical examina- Answer: C


tion is found to harbor a mutation in codon 918 of the Children with mutations at codon 634 of the RET tyrosine
RET tyrosine kinase receptor, compatible with MEN2B. kinase receptor gene (MEN2A) are advised to undergo thy-
Ultrasound of the neck is unremarkable and serum cal- roidectomy before age 5, whereas children with mutations at
citonin levels are normal. What course is indicated? codon 918 (MEN2B) should undergo thyroidectomy before
A. Repeat examination and ultrasound yearly age 1. If ultrasound of the neck is normal and calcitonin
B. Planned thyroidectomy in 3 to 5 years levels are normal, a formal neck dissection can be avoided.
C. Total thyroidectomy (See Schwartz 11th ed., pp. 1656–1657.)
D. Total thyroidectomy with bilateral neck dissection

15. Postoperative complications of thyroid surgery which Answer: D


may require immediate reoperation include: Inadvertent injury (ischemia) or removal of the parathy-
A. Hypocalcemia. roid glands can cause acute neuromuscular excitability due
B. Dyspnea. to hypocalcemia. This can be ameliorated with intravenous
C. Dystonia. calcium. Dyspnea may be a consequence of pain or anxiety
D. Wound hematoma. but rarely indicates airway compromise. Bilateral vocal cord
dysfunction with airway compromise requires reintubation.
An expanding hematoma in the neck may not cause bleed-
ing from the wound but can compress the membranous por-
tion of the trachea and cause dyspnea. The wound should be
opened at the bedside and the patient returned to the operat-
ing room for wound exploration and reclosure. (See Schwartz
11th ed., p. 1663.)

16. A patient with primary hyperparathyroidism undergoes Answer: D


neck exploration where four small, normal appearing Supernumerary parathyroid glands occur in 7% to 13% of
glands are found. What are the possible locations of an people, and may be located in the thymus (most commonly),
additional, supernumerary gland? within the parenchyma of the thyroid gland, or in the tra-
A. In the thyroid gland cheoesophageal groove, the mediastinum, or elsewhere in the
B. In the thymus neck. (See Schwartz 11th ed., pp. 1663–1664.)
C. In the tracheoesophageal groove
D. All of the above

Brunicardi_Ch38_p323-328.indd 325 05/07/22 1:32 PM


326
17. A 70-year-old woman with early dementia but other- Answer: B
wise good physical health has an elevated parathyroid Localization studies such as sestamibi scans have been shown
hormone (PTH) level and a sestamibi scan which local- to allow more limited operations, including those utilizing
izes a single focus of increased activity to the left lower “min-incisions” under local anesthesia, for patients who are
neck. An ultrasound conforms an enlarged gland in the not good risks for general anesthesia. Improved cosmesis,
same area. What treatment is likely to provide the best shorter lengths of stay, and reduced complications are ben-
outcome? efits from this approach. (See Schwartz 11th ed., p. 1674.)
A. Bilateral neck exploration under general anesthesia
B. Unilateral, “mini-incision” parathyroidectomy under
Chapter 38

local anesthesia
C. Minimally invasive videoscopic parathyroidectomy
from a left axillary approach under general anesthesia
D. Percutaneous alcohol ablation with ultrasound guid-
ance under local anesthesia

18. A 65-year-old woman with mild dementia and elevated Answer: D


Thyroid, Parathyroid, and Adrenal

calcium and parathyroid hormone levels undergoes The position of the parathyroid glands can be quite vari-
bilateral neck dissection. Two upper pole parathyroid able. In addition to the tracheoesophageal groove adjacent
glands and one lower pole parathyroid gland are found to the posterior capsule of the thyroid, the gland may be
but despite a diligent search, the fourth gland is not intra-­thyroidal, intra-thymic, within the carotid sheath, ret-
apparent. What steps are appropriate to locate the fourth roesophageal, or in the posterior mediastinum. (See Schwartz
gland? 11th ed., p. 1675.)
A. The thyrothymic ligament should be mobilized and
the upper end of the thymus gently retracted into the
field for dissection and removal if any mass is found.
B. The carotid sheath should be opened from the bifur-
cation to the base of the neck and explored.
C. Intraoperative ultrasonography can be employed to
examine the thyroid lobe on the side of the missing
parathyroid to detect an intrathyroidal location.
D. All of the above.

19. Gastrointestinal complications of primary hyperparathy- Answer: D


roidism include all of the following EXCEPT: Primary hyperparathyroidism has been associated with
A. Peptic ulcer disease. peptic ulcer disease, both with and without an accompany-
B. Acute pancreatitis. ing gastrinoma. It is sometimes a cause of acute pancreatitis
C. Cholelithiasis. although usually only when serum calcium levels are very
D. Hepatitis. high (>12.5 mg/dL). An increased incidence of cholelithiasis
is also seen, presumably due to an increase in biliary calcium
excretion, but hepatitis is not associated with the disease.
(See Schwartz 11th ed., p. 1668.)

20. A 50-year-old man with chronic renal failure develops Answer: A


hypercalcemia (>11 mg/dL) and an elevated parathor- Total parathyroidectomy is contraindicated in patients who
mone level (>800 pg/mL). Ultrasound and radionuclide are candidates for renal transplantation. Subtotal parathy-
studies of the neck reveal no localized mass. On bilateral roidectomy with retention of one-half of the smallest gland,
neck exploration, four variably sized but enlarged para- with or without upper thymectomy, is considered appropri-
thyroid glands are found. Which of the following surgi- ate, as is total parathyroidectomy with auto-transplantation of
cal options is NOT indicated? a portion of the smallest gland, depending on surgeon experi-
A. Total parathyroidectomy ence and preference. Recurrent disease after this procedure
B. Subtotal parathyroidectomy with retention of one- can be managed with removal of the auto-transplanted gland
half of the smallest gland avoiding repeat neck exploration. (See Schwartz 11th ed.,
C. Total thyroidectomy with reimplantation of 1 mm pp. 1679–1680.)
cubes of one gland into the brachioradialis muscle of
the nondominant forearm
D. Subtotal parathyroidectomy with retention of one-
half of the smallest gland combined with upper
thymectomy

Brunicardi_Ch38_p323-328.indd 326 05/07/22 1:32 PM


327
21. A 50-year-old, healthy-appearing man undergoes evalua- Answer: C
tion of persistent hypertension. Serum chemistries reveal Primary aldosteronism, or Conn’s syndrome, is seen in about
hypokalemia (<3.2 mmol/L) and imaging studies reveal 1% of hypertensive patients. It is more common in middle
a unilateral adrenal mass. What is the likely diagnosis? aged individual and is usually associated with a single ade-
A. Secondary hypercortisolism (Cushing’s disease) noma of the adrenal cortex. The hypertension is usually
B. Primary hypercortisolism (Cushing’s syndrome) refractory to medical treatment, and is classically associated
C. Hyperaldosteronism (Conn’s syndrome) with hypokalemia, but may be seen in normokalemic indi-
D. Pheochromocytoma viduals. (See Schwartz 11th ed., pp. 1685–1688.)

Chapter 38
22. A 35-year-old woman undergoes an evaluation for Answer: B
infertility. She has gained almost 100 pounds in the past Cushing’s syndrome refers to any cause of hypercortisolism
year, is hypertensive, and is borderline diabetic. She also caused by either an adrenal source or exogenous administra-
complains of easy bruising. Her serum chemistries are tion of steroids. Cushing’s disease refers only to an adreno-
normal with the exception of an elevated glucose. Imag- corticotropin hormone (ACTH)-secreting adenoma of the
ing studies reveal a unilateral adrenal mass. What is the pituitary gland. Cushing’s syndrome due to an isolated adre-
likely diagnosis? nal adenoma is far less common than hypercortisolism due

Thyroid, Parathyroid, and Adrenal


A. Secondary hypercortisolism (Cushing’s disease) to a pituitary adenoma, but adrenalectomy is curative for pri-
B. Primary hypercortisolism (Cushing’s syndrome) mary adrenal tumors or for adrenal hyperplasia that persists
C. Hyperaldosteronism (Conn’s syndrome) despite efforts to resect a pituitary tumor. (See Schwartz 11th
D. Pheochromocytoma ed., pp. 1688–1689.)

23. All of the following imaging techniques are useful to Answer: D


localize a pheochromocytoma EXCEPT: Pheochromocytomas are solid tumors which appear on
A. Computed tomography (CT) scan. CT scan as soft tissue masses. They are detected with 85%
B. Magnetic resonance imaging (MRI) scan. to 95% accuracy, but it is important to avoid intravenous
C. Metaiodobenzylguanidine (MIBG) scan. contrast enhancement when a pheochromocytoma is sus-
D. Octreotide scan. pected; intravenous contrast can provoke a hypertensive cri-
sis due to release of catecholamines. MRI is useful to identify
pheochromocytomas, both because they identify soft tissue
masses, but also because this tumor tends to enhance on
T2-weighted images. Radio-labeled MIBG is taken up avidly
by the pheochromocytoma because its structure is similar to
norepinephrine.
Therefore, the MIBG scan can localize an occult tumor.
Octreotide scans are not used for pheochromocytoma as
the tumor does not overexpress somatostatin receptors. (See
Schwartz 11th ed., p. 1694.)

24. Pheochromocytomas are associated with all of the fol- Answer: D


lowing diseases EXCEPT: Pheochromocytomas are caused by germline mutations in
A. Multiple endocrine neoplasia (MEN) type 2A. the RET proto-oncogene (the origin of both MEN 2A and
B. Multiple endocrine neoplasia (MEN) type 2B. MEN 2B) as well as the VHL gene mutation. Adrenal and
C. Von Hippel Lindau (VHL) disease. extra-adrenal pheochromocytomas may occur spontaneously
D. Familial adenomatous polyposis (FAP) disease. or as part of a neoplastic syndrome, but they are not associ-
ated with the FAP syndrome. (See Schwartz 11th ed., p. 1693.)

25. The preoperative preparation of a patient with pheochro- Answer: D


mocytoma should include all of the following EXCEPT: The preoperative preparation of a patient with a catechol-
A. An alpha-adrenergic blocker such as phentolamine. amine-secreting tumor includes alpha-adrenergic blockade
B. A beta-adrenergic blocker such as propranolol. to control hypertension, a beta-adrenergic blocker to prevent
C. Intravenous hydration to avoid volume depletion. tachycardia, and volume replacement to avoid hypotension
D. Systemic steroids to avoid adrenal insufficiency. due to alpha- and beta-blockade. Steroids are not needed
to prevent adrenal insufficiency. (See Schwartz 11th ed.,
pp. 1694–1695.)

Brunicardi_Ch38_p323-328.indd 327 05/07/22 1:32 PM


328
26. A follow-up computed tomography (CT) scan in a Answer: A
60-year-old patient with previous nephrolithiasis reveals The adrenal “incidentaloma” is an increasingly common
a 1.5 cm hypovascular round lesion with clear margins finding with the ubiquitous use of CT scanning, with an inci-
in the right adrenal gland. The patient is not hyperten- dence of 0.4% to 4.4%. A variety of benign and malignant
sive, hyperglycemic, or hypokalemic. Urinary catechol lesions can account for these findings, and a distant history
metabolites are within normal limits, and serum corti- of malignancy elsewhere should raise the possibility of met-
sol and adrenocorticotropin hormone (ACTH) levels are astatic disease. Primary malignancy of the adrenal gland is
normal. Which course is advisable? rare, and the functioning tumors are excluded by screening
A. Repeat CT scan and chemical tests annually tests for cortisol and catecholamine excess. In the absence of
Chapter 38

B. Percutaneous fine needle aspiration biopsy symptoms associated with adrenal disease, annual follow-up
C. Adrenal venous sampling for cortisol, renin, and of these lesions with imaging and chemical tests seems pru-
angiotensin dent. (See Schwartz 11th ed., pp. 1696–1697.)
D. Laparoscopic adrenalectomy

27. Advantages of laparoscopic adrenalectomy compared Answer: C


to open adrenalectomy include all of the following Laparoscopic (videoscopic) approaches to adrenalectomy
Thyroid, Parathyroid, and Adrenal

EXCEPT: have been shown to be advantageous for several outcomes


A. Decreased blood loss. including wound complications, analgesic use, and length of
B. Faster return to work. hospital stay. These advantages are in balance to adverse con-
C. Decreased operative time. siderations including length of operative time and cost. (See
D. Decreased narcotic analgesic use. Schwartz 11th ed., p. 1698.)

28. In patients who undergo bilateral adrenalectomy in treat- Answer: B


ment of Cushing disease after failed attempts at resection Nelson syndrome describes symptoms due to the progressive
of an adrenocorticotropin hormone (ACTH)-secreting enlargement of a persistent ACTH-secreting pituitary fossa
pituitary adenoma, the subsequent development of Nel- tumor. These symptoms include hyperpigmentation, visual
son syndrome is associated with which of the following? field loss, headaches, and extraocular muscle palsies. Inter-
A. Hypertension ference with the olfactory nerve, hypertension, and inconti-
B. Diminished visual fields nence is not part of the syndrome. (See Schwartz 11th ed.,
C. Loss of hearing p. 1701.)
D. Incontinence

Brunicardi_Ch38_p323-328.indd 328 05/07/22 1:32 PM


CHAPTER 39
Pediatric Surgery

1. Operative management of a newborn with the chest Answer: D


X-ray shown in Fig. 39-1 should occur: The diagnosis of congenital diaphragmatic hernia (CDH) is
A. Immediately after birth. made by chest X-ray with the vast majority of infants devel-
B. Within 24 hours after birth. oping immediate respiratory distress and pulmonary hyper-
C. Within 72 hours after birth. tension. Congenital diaphragmatic hernia care has improved
D. None of the above. considerably through effective use of improved methods of
ventilation and timely cannulation for extracorporeal mem-
brane oxygenation. In the past, correction of the hernia was
believed to be a surgical emergency and patients underwent
surgery shortly after birth. It is now accepted that the pres-
ence of persistent pulmonary hypertension that results in
right-to-left shunting across the patent foramen ovale or
the ductus arteriosus and the degree of pulmonary hypopla-
sia are the leading causes of cardiorespiratory insufficiency.
­Current management is therefore directed toward managing
the ­pulmonary hypertension, which is usually seen within 7
to 10 days, but in some infants, may take up to several weeks
to develop. (See Schwartz 11th ed., p. 1712.)

FIG. 39-1.

329

Brunicardi_Ch39_p329-336.indd 329 08/07/22 11:51 AM


330
2. A 4-year-old boy presents with a midline neck mass. On Answer: C
physical examination, the mass is nontender and moves Resection of the thyroglossal duct cyst in continuity with the
up and down with swallowing. There is no drainage or central portion of the hyoid bone and the tract connecting to
erythema and the thyroid gland is palpable in its normal the pharynx in addition to ligation at the foramen cecum (the
anatomic location. A thyroglossal duct cyst is suspected. Sistrunk operation) is curative in >90% of patients. Lesser
What is the appropriate treatment? operations result in unacceptably high recurrence rates, and
A. Observation recurrence is more frequent following infection. According
B. Incision and drainage of the cyst to a recent review, factors predictive of recurrence included
C. Resection of the cyst, central portion of the hyoid more than two infections prior to surgery, age < 2 years, and
Chapter 39

bone and tract to the pharynx inadequate initial operation. (See Schwartz 11th ed., p. 1710.)
D. Excision of the cyst

3. An infant comes to the emergency room with bilious Answer: D


emesis and irritability. Physical examination is notable The cardinal symptom of intestinal obstruction in the new-
for abdominal tenderness and erythema of the abdomi- born is bilious emesis. Prompt recognition and treatment of
nal wall. Abdominal X-ray demonstrates dilated proximal neonatal intestinal obstruction can truly be life-saving. Bilious
Pediatric Surgery

bowel with air-fluid levels. What is the most appropriate vomiting is usually the first sign of volvulus, and all infants
next step in management, after resuscitation? with bilious vomiting must be evaluated rapidly to ensure
A. Upper gastrointestinal series that they do not have intestinal malrotation with volvulus.
B. Barium enema The child with irritability and bilious emesis should raise par-
C. Gastrostomy ticular suspicions for this diagnosis. If left untreated, vascular
D. Laparotomy compromise of the midgut initially causes bloody stools, but
eventually results in circulatory collapse. Additional clues to
the presence of advanced ischemia of the intestine include
erythema and edema of the abdominal wall, which progress
to shock and death. It must be reemphasized that the index
of suspicion for this condition must be high, since abdominal
signs are minimal in the early stages. Abdominal films show
a paucity of gas throughout the intestine with a few scattered
air-fluid levels. When these findings are present, the patient
should undergo immediate fluid resuscitation to ensure ade-
quate perfusion and urine output followed by prompt explor-
atory laparotomy. (See Schwartz 11th ed., p. 1725.)

4. A 4-week-old full-term, otherwise healthy female is admit- Answer: A


ted with nonbilious projectile emesis. Ultrasound shows a Given frequent fluid and electrolyte abnormalities at time of
pyloric channel length of 16 mm and a pyloric thickness of presentation, pyloric stenosis is never a surgical emergency.
4 mm. Labs show hypokalemic, hypochloremic metabolic Fluid resuscitation with correction of electrolyte abnormali-
alkalosis. What is the next best step in management? ties and metabolic alkalosis is essential prior to induction of
A. Fluid resuscitation with 5% dextrose and 0.45% saline general anesthesia for operation. For most infants, fluid con-
with added potassium until correction of electrolyte taining 5% dextrose and 0.45% saline with added potassium
abnormalities of 2 to 4 mEq/kg over 24 hours at a rate of approximately 150
B. Fluid resuscitation with normal saline until achieving to 175 mL/kg for 24 hours will correct the underlying deficit.
a urine output of >2 mL/kg per hour for 24 hours It is important to ensure that the child has an adequate urine
C. Immediate laparoscopic pyloromyotomy output (>2 mL/kg per hour) as further evidence that rehydra-
D. Emergent open pyloromyotomy tion has occurred. (See Schwartz 11th ed., p. 1722.)

5. The most common form of esophageal atresia (EA) is: Answer: C


A. Pure EA (no fistula). The five major varieties of EA and TEF are shown in Fig. 39-2.
B. Pure tracheoesophageal fistula (TEF) (no atresia). The most commonly seen variety is EA with distal TEF
C. EA with distal TEF. (type C), which occurs in approximately 85% of the cases in
D. EA with proximal TEF. most series. The next most frequent type is pure EA (type A),
occurring in 8% to 10% of patients, followed by TEF without
EA (type E). This occurs in 8% of cases and is also referred to
as an H-type fistula, based on the anatomic similarity to that
letter (Fig. 39-3). EA with fistula between both proximal and
distal ends of the esophagus and trachea (type D) is seen in
approximately 2% of cases, and type B, EA with TEF between
proximal esophagus and trachea, is seen in approximately 1%
of all cases. (See Schwartz 11th ed., p. 1717.)

Brunicardi_Ch39_p329-336.indd 330 08/07/22 11:51 AM


331

Chapter 39
FIG. 39-2. The five varieties of esophageal atresia and tracheoesophageal fistula. A. Isolated esophageal atresia.
B. Esophageal atresia with tracheoesophageal fistula between proximal segment of esophagus and trachea.
C. Esophageal atresia with tracheoesophageal fistula between distal esophagus and trachea. D. Esophageal atresia
with fistula between both proximal and distal ends of esophagus and trachea. E. Tracheoesophageal fistula without

Pediatric Surgery
esophageal atresia (H-type fistula).

FIG. 39-3. Barium esophagram showing H-type


tracheoesophageal fistula (arrow).

6. The predicted 4-year survival rate of a child with a Wilms Answer: D


tumor that is confined to one kidney and is grossly Following nephroureterectomy for Wilms tumor, the need for
excised is: chemotherapy and/or radiation therapy is determined by the
A. 24%. histology of the tumor and the clinical stage of the patient
B. 38%. (Table 39-1). Essentially, patients who have disease confined
C. 68%. to one kidney that is completely excised surgically receive a
D. 97%. short course of chemotherapy and can expect a 97% 4-year
survival, with tumor relapse rare after that time. Patients with
more advanced disease or with unfavorable histology receive
more intensive chemotherapy and radiation. Even in stage IV,
cure rates of 80% are achieved. The survival rates are worse in
the small percentage of patients considered to have unfavor-
able histology. (See Schwartz 11th ed., p. 1747.)

Brunicardi_Ch39_p329-336.indd 331 08/07/22 11:51 AM


332
TABLE 39-1 Staging of Wilms tumor
Stage I: Tumor limited to the kidney and completely excised.
Stage II: Tumor that extends beyond the kidney but is completely excised. This includes penetration of the renal capsule, invasion of the soft tissues of
the renal sinus, or blood vessels within the nephrectomy specimen outside the renal parenchyma containing tumor. No residual tumor is apparent at or
beyond the margins of excision.a
Stage III: Residual nonhematogenous tumor confined to the abdomen. Lymph nodes in the abdomen or pelvis contain tumor. Peritoneal contamination
by the tumor, such as by spillage or biopsy of tumor before or during surgery. Tumor growth that has penetrated through the peritoneal surface.
Implants are found on the peritoneal surfaces. Tumor extends beyond the surgical margins either microscopically or grossly. Tumor is not completely
resectable because of local infiltration into vital structures. The tumor was treated with preoperative chemotherapy with or without biopsy. Tumor is
removed in greater than one piece.
Chapter 39

Stage IV: Hematogenous metastases or lymph node involvement outside the abdomino-pelvic region.
Stage V: Bilateral renal involvement.
International Neuroblastoma Staging System
Stage 1: Localized tumor with complete gross resection, with or without microscopic residual disease
Stage 2A: Localized tumor with incomplete gross excision; representative ipsilateral nonadherent lymph nodes negative for tumor
Stage 2B: Localized tumor with or without complete gross excision, with ipsilateral nonadherent lymph nodes positive for tumor. Enlarged contralateral
lymph nodes must be negative microscopically
Stage 3: Unresectable unilateral tumor crossing midline, with or without regional lymph node involvement; or localized unilateral tumor with contralateral
Pediatric Surgery

regional lymph node involvement; or midline tumor


Stage 4: Any primary tumor with dissemination to distant lymph nodes, bone, bone marrow, liver, skin, and/or other organs
Stage 4S: In infants <1 year of age; localized primary tumor with dissemination limited to skin, liver, and/or bone marrow
International Neuroblastoma Risk Group Staging System
L1 Localized tumor not involving vital structures as defined by the list of IDRFs and confined to one body compartment
L2 Locoregional tumor with the presence of one or more IDRFs
M Distant metastatic disease (except MS)
MS Metastatic disease in children <18 months confined to skin, liver, and bone marrow
Rupture or spillage confined to the flank, including biopsy of the tumor, is no longer included in stage II and is now included in stage III.
a

7. A premature infant boy has been started on enteral feeds Answer: A


shortly after birth, but develops feeding intolerance The radiograph demonstrates pneumatosis intestinalis and,
2 weeks postnatally. He displays abdominal tenderness, in conjunction with the clinical scenario described, exhibits
distention, and bloody stools. An abdominal radiograph Bell stage II necrotizing enterocolitis (NEC). In all infants sus-
is obtained and is shown in Fig. 39-4. What should be the pected of having NEC, feeds are discontinued, a nasogastric
next step in management? tube is placed, total parenteral nutrition (TPN) is started and
A. Nasogastric decompression, parenteral nutrition, broad-spectrum parenteral antibiotics are given. The infant
broad spectrum antibiotics is resuscitated and inotropes are administered to maintain
B. Laparotomy, excision of the affected bowel with perfusion as needed. Intubation and mechanical ventilation
ostomy formation may be required to maintain oxygenation. Subsequent treat-
C. Laparotomy, reduction of the volvulus, division of ment may be influenced by the particular stage of NEC that is
adhesions, appendectomy present. Patients with Bell stage II disease merit close obser-
D. Water-soluble contrast enema vation. Serial physical examinations are performed looking

FIG. 39-4. Abdominal radiograph of


infant with necrotizing enterocolitis.
Arrows point to area of pneumatosis
intestinalis.

Brunicardi_Ch39_p329-336.indd 332 08/07/22 11:51 AM


333
for the development of diffuse peritonitis, a fixed mass, pro-
gressive abdominal wall cellulitis, or systemic sepsis. If infants
fail to improve after several days of treatment or if abdominal
radiographs show a fixed intestinal loop, consideration should
be given to exploratory laparotomy. (See Schwartz 11th ed.,
p. 1727.)

8. An infant girl is found to have persistent jaundice after Answer: B


birth. A metabolic screen is normal, ultrasound dem- This patient has biliary atresia, a rare disease characterized by

Chapter 39
onstrates an absent gallbladder, and a technetium-99m fibroproliferative obliteration of the biliary tree, which pro-
iminodiacetic acid scan shows radionuclide that is con- gresses toward hepatic fibrosis, cirrhosis, and end-stage liver
centrated in the liver, but not excreted into the intestine. failure. Surgical treatment is the first-line therapy, consisting
Which of the following statements is TRUE? of creation of a hepatoportoenterostomy (Kasai procedure).
A. This condition is usually managed nonoperatively. Numerous studies suggest that the likelihood of surgical
B. Surgery should be performed within 60 days of life. success is inversely related to the age at the time of porto-
C. Cystoenterostomy provides adequate biliary enterostomy. Infants treated prior to 60 days of life are more

Pediatric Surgery
drainage. likely to achieve successful and long-term biliary drainage
D. Most of these patients will not require transplantation. than older infants. Although the outlook is less favorable for
patients after the 12th week, it is reasonable to proceed with
surgery even beyond this time point, as the alternative is cer-
tain liver failure. Approximately one-third of patients remain
symptom-free after portoenterostomy; the remainder require
liver transplantation due to progressive liver failure. Indepen-
dent risk factors that predict failure of the procedure include
bridging liver fibrosis at the time of surgery and postoperative
cholangitic episodes. (See Schwartz 11th ed., p. 1737.)

9. The leading cause of death among children older than 1 Answer: C


year of age is: Injury is the leading cause of death among children older than
A. Malignancy. 1 year. In fact, trauma accounts for almost half of all pediatric
B. Infection. deaths, more than cancer, congenital anomalies, pneumonia,
C. Injury. heart disease, homicide, and meningitis combined. Motor
D. Congenital anomalies. vehicle collisions are the leading cause of death in people age
1 to 19 years, followed by homicide or suicide (predominantly
with firearms) and drowning. Unintentional injuries account
for 65% of all injury-related deaths in children younger than
19 years. Each year, approximately 20,000 children and teen-
agers die as a result of injury in the United States. For every
child who dies from an injury, it is calculated that 40 others
are hospitalized and 1120 are treated in emergency depart-
ments. An estimated 50,000 children acquire permanent
disabilities each year, most of which are the result of head
injuries. Thus, the problem of pediatric trauma continues to
be one of the major threats to the health and well-being of
children. (See Schwartz 11th ed., p. 1751.)

10. A “double bubble” on an abdominal radiograph in an Answer: A


infant is characteristic of: Whenever the diagnosis of duodenal obstruction is enter-
A. Duodenal atresia. tained, malrotation and midgut volvulus must be excluded.
B. Jejunal atresia. Other causes of duodenal obstruction include duodenal atre-
C. Meconium ileus. sia, duodenal web, stenosis, annular pancreas, or duodenal
D. Pyloric stenosis. duplication cyst. The classic finding on abdominal radiogra-
phy is the “double bubble” sign, which represents the dilated
stomach and duodenum (Fig. 39-5). In association with the
appropriate clinical picture, this finding is sufficient to con-
firm the diagnosis of duodenal obstruction. (See Schwartz
11th ed., p. 1724.)

Brunicardi_Ch39_p329-336.indd 333 08/07/22 11:51 AM


334
Chapter 39

FIG. 39-5. Abdominal X-ray


showing “double bubble” sign in
Pediatric Surgery

a newborn infant with duodenal


atresia. The two “bubbles” are
numbered.

11. How is the diagnosis of Hirschsprung disease definitively Answer: C


made? The definitive diagnosis of Hirschsprung disease is made by
A. Observation of failure to pass meconium within rectal biopsy. Samples of mucosa and submucosa are obtained
48 hours after birth at 1 cm, 2 cm, and 3 cm from the dentate line. This can be per-
B. Barium enema showing a rectosigmoid ratio < 1 formed at the bedside in the neonatal period without anesthe-
C. Rectal biopsies with no ganglion cells in the myen- sia, as samples are taken in bowel that does not have somatic
teric plexus innervation and is thus not painful to the child. In older chil-
D. Barium enema showing a rectosigmoid ratio > 1 dren, the procedure should be performed using IV sedation.
The histopathology of Hirschsprung disease is the absence of
ganglion cells in the myenteric plexuses, increased acetylcho-
linesterase staining, and the presence of hypertrophied nerve
bundles. It is important to obtain a barium enema in children
in whom the diagnosis of Hirschsprung disease is suspected.
This test may demonstrate the location of the transition zone
between the dilated ganglionic colon and the distal con-
stricted aganglionic rectal segment. Our practice is to obtain
this test before instituting rectal irrigations if possible so that
the difference in size between the proximal and distal bowel is
preserved. Although the barium enema can only suggest, but
not reliably establish, the diagnosis of Hirschsprung disease,
it is very useful in excluding other causes of distal intestinal
obstruction. (See Schwartz 11th ed., p. 1734.)

12. Which of the following statements regarding pediatric Answer: B


inguinal hernia repair (herniorrhaphy) is FALSE? When the diagnosis of inguinal hernia is made in an oth-
A. Herniorrhaphy should be performed following erwise normal child, operative repair should be planned.
reduction of an incarcerated hernia either during the Spontaneous resolution does not occur, and therefore a non-
index admission or promptly on an elective basis. operative approach cannot ever be justified. (See Schwartz
B. An asymptomatic inguinal hernia may be monitored 11th ed., p. 1743.)
for spontaneous closure.
C. Inguinal hernias in children result from a congeni-
tally patent processus vaginalis.
D. Spinal anesthesia in preterm infants undergoing her-
niorrhaphy decreases the risk of postoperative apnea
when compared to general anesthesia.

Brunicardi_Ch39_p329-336.indd 334 08/07/22 11:51 AM


335
13. During attempted reduction of bowel in a newborn with Answer: A
gastroschisis who has a 3 cm abdominal wall defect to Care must be taken to prevent markedly increased abdominal
the right of the umbilical cord, the ventilator begins to pressure during the reduction, which will lead to compres-
alarm with high peak inspiratory pressures (>25 cm sion of the inferior vena cava, respiratory embarrassment,
H2O). What is the next best step in management? and abdominal compartment syndrome. To avoid this
A. Place the remaining bowel into a plastic silo and per- complication, it is helpful to monitor the bladder or airway
form delayed reduction. pressures during reduction. In infants whose intestine has
B. Proceed with reduction but avoid primary closure. become thickened and edematous, it may be impossible to
C. Decompress the stomach with an orogastric tube and reduce the bowel into the peritoneal cavity in the immediate

Chapter 39
the rectum with irrigations. postnatal period. Under such circumstances, a plastic spring-
D. Proceed with reduction and primary closure. loaded silo can be placed onto the bowel and secured beneath
the fascia or a sutured silastic silo constructed. The silo cov-
ers the bowel and allows for graduated reduction on a daily
basis as the edema in the bowel wall decreases. (See Schwartz
11th ed., p. 1741.)

Pediatric Surgery
14. A 3-year-old patient presents with recurrent lower gas- Answer: C
trointestinal bleeding. What imaging modality is used to A Meckel diverticulum is a remnant of a portion of the embry-
diagnose a bleeding Meckel’s diverticulum? onic omphalomesenteric (vitelline) duct. It is located on the
A. Doppler ultrasound antimesenteric border of the ileum, usually within 2 ft of the
B. Computed tomography ileocecal valve. Ectopic gastric mucosa within a Meckel diver-
C. Technetium pertechnetate scan ticulum may produce ileal ulcerations that bleed and lead
D. Capsule endoscopy to the passage of maroon-colored stools. Pancreatic mucosa
may also be present. Diagnosis may be made by technetium
pertechnetate scans when the patient presents with bleeding.
(See Schwartz 11th ed., p. 1733.)

15. What type of choledochal cyst accounts for 80% to 90% Answer: D
of all cases and has a fusiform dilatation of the common Type I choledochal cyst is characterized by fusiform dila-
bile duct? tation of the bile duct. This is the most common type and
A. Type IV is found in 80% to 90% of cases. Type II choledochal cysts
B. Type III appear as an isolated diverticulum protruding from the wall
C. Type II of the common bile duct. The cyst may be joined to the com-
D. Type I mon bile duct by a narrow stalk. Type III choledochal cysts
arise from the intraduodenal portion of the common bile
duct and are also known as choledochoceles. Type IVA cysts
consist of multiple dilatations of the intrahepatic and extra-
hepatic bile ducts. Type IVB choledochal cysts are multiple
dilatations involving only the extrahepatic bile ducts. Type V
(Caroli disease) consists of multiple dilatations limited to the
intrahepatic bile ducts. (See Schwartz 11th ed., p. 1739.)

16. A 9-month-old boy presents with two episodes of vomit- Answer: A


ing as well as episodes of colicky pain. His abdominal Patients with intussusception should be assessed for the pres-
examination is notable for upper abdominal tenderness ence of peritonitis and for the severity of systemic illness. In the
and his stools are guaiac positive. Ultrasound dem- absence of peritonitis, the child should undergo radiographic
onstrates a target sign. He is taken for an air-contrast reduction. The air enema is diagnostic and may also be cura-
enema. What is the likelihood that he will need operative tive, and it is the preferred method of diagnosis and treat-
intervention? ment of intussusception. Air is introduced with a manometer,
A. 25% and the pressure that is administered is carefully monitored.
B. 50% Under most instances, this should not exceed 120 mm Hg.
C. 75% Successful reduction is marked by free reflux of air into mul-
D. 90% tiple loops of small bowel and symptomatic improvement as
the infant suddenly becomes pain-free. Unless both of these
signs are observed, it cannot be assumed that the intussus-
ception is reduced. If reduction is unsuccessful and the infant
remains stable, the infant should be brought back to the radi-
ology suite for a repeat attempt at reduction after a few hours.
This strategy has improved the success rate of nonoperative

Brunicardi_Ch39_p329-336.indd 335 08/07/22 11:51 AM


336
reduction in many centers. In addition, hydrostatic reduction
with barium may be useful if pneumatic reduction is unsuc-
cessful. The overall success rate of radiographic reduction
varies based on the experience of the center and is typically
between 60% and 90%. (See Schwartz 11th ed., p. 1731.)

17. A male infant is born with an imperforate anus. Prior to Answer: D


surgery, a VACTERL workup is performed. Which study Approximately 60% of patients with imperforate anus have
is not part of the typical workup? an associated malformation. The most common is a urinary
Chapter 39

A. Echocardiogram tract defect, which occurs in approximately 50% of patients.


B. Renal ultrasound Skeletal defects are also seen, and the sacrum is most com-
C. Spinal radiographs monly involved. Spinal cord anomalies especially tethered
D. Brain MRI cored are common, particularly in children with high lesions.
Gastrointestinal anomalies occur, most commonly esopha-
geal atresia. Cardiac anomalies may be noted, and occasion-
ally patients present with a constellation of defects as part of
Pediatric Surgery

the VACTERL syndrome. (See Schwartz 11th ed., p. 1735.)

18. A newborn female with cystic fibrosis and meconium Answer: B


ileus without perforation has been managed with If surgical intervention is required because of failure of con-
N-­acetylcysteine (Mucomyst) infused transanally every trast enemas to relieve obstruction in meconium ileus, opera-
12 hours. She remains obstructed and is taken to the tive irrigation with dilute contrast agent, N-acetylcysteine,
operating room for intervention. What is the optimal or saline through a purse-string suture may be successful.
surgical procedure? Alternatively, resection of the distended terminal ileum is
A. Enterectomy with ileostomy creation performed, and the meconium pellets are flushed from the
B. Simple enterotomy with lavage distal small bowel. At this point, an end ileostomy may be cre-
C. Ileocecectomy with primary anastomosis ated. The distal bowel may be brought up as a mucus fistula or
D. Colotomy with retrograde lavage sewn to the side of the ileum as a classic Bishop-Koop anas-
tomosis. An end-to-end anastomosis may also be considered
in the appropriate setting. (See Schwartz 11th ed., p. 1726.)

19. Standard volume of packed red blood cell transfusion in Answer: A


the newborn infant is: A useful guideline for estimating blood volume for the new-
A. 10 mL/kg. born infant is approximately 80 mL/kg of body weight. When
B. 5 mL/kg. packed red blood cells are required, the transfusion require-
C. 20 mL/kg. ment is usually administered in 10 mL/kg increments, which
D. 7 mL/kg. is roughly equivalent to a 500-mL transfusion for a 70-kg
adult. (See Schwartz 11th ed., p. 1707.)

20. Undescended testes are usually repaired by what age? Answer: C


A. 6 months Males with bilateral undescended testicles are often infertile.
B. 1 year of age When the testicle is not within the scrotum, it is subjected to a
C. 2 years of age higher temperature, resulting in decreased spermatogenesis.
D. 4 years of age Mengel and coworkers studied 515 undescended testicles by
histology and demonstrated a decreasing presence of sper-
matogonia after 2 years of age. Despite orchidopexy, the inci-
dence of infertility is approximately two times higher in men
with unilateral orchidopexy compared to men with normal
testicular descent. Consequently, it is now recommended that
the undescended testicle be surgically repositioned by 1 year
of age. (See Schwartz 11th ed., p. 1744.)

Brunicardi_Ch39_p329-336.indd 336 08/07/22 11:51 AM


CHAPTER 40
Urology

1. Which of the following is TRUE about bladder cancer? Answer: B


A. For patients with bladder, cancer invading into the For patients who have disease invading into bladder muscle
bladder muscle (T2 lesion) immediate radiotherapy (T2), immediate (within 3 months of diagnosis) cystectomy
followed by surgery offers the best chance of cure. with extended lymph node dissection offers the best chance
B. Patients with limited lymph node involvement may of survival. Patients with limited lymph node involvement
be cured by surgery alone. may be cured with surgery alone. Patients have multiple
C. Continent neobladders have yet to be successfully reconstructive options, including continent and noncon-
utilized in patients undergoing cystectomy. tinent urinary diversions. The orthotopic neobladder has
D. lntravesical chemotherapy prior to surgery is rou- emerged as a popular urinary diversion for patients without
tinely used for bladder cancers invading into the urethral involvement. This diversion type involves the detu-
bladder muscle (T2 lesion). bularization of a segment of bowel, typically distal ileum,
which is then refashioned into a pouch that is anastomosed to
the proximal urethra (neobladder) or to the skin (continent
cutaneous diversion). Patients with nonmuscle-invasive blad-
der cancer (confined to the bladder mucosa or submucosa)
can be managed with transurethral resection alone and adju-
vant intravesical (instilled into the bladder) chemotherapy/­
immunotherapy. (See Schwartz 11th ed., pp. 1770–1771.)

2. Which of the following are TRUE concerning testicular Answer: A


cancer? Testicular cancer is the most common solid malignancy in
A. Most common malignancy in men aged 15–35. men age 15 to 35 years. Chest and abdominal imaging must
B. Most commonly presents as a painful enlarging mass. be performed to evaluate for evidence of metastasis. The most
C. Initial work up includes chest, abdominal, and brain common site of spread is the retroperitoneal lymph nodes
imaging. extending from the common iliac vessels to the renal vessels.
D. Most common site of metastases is to the lungs. (See Schwartz 11th ed., pp. 1771–1772.)

3. Which of the following statements about renal cell carci- Answer: D


noma (RCC) are FALSE? Renal tumors are usually solid, but they also can be cystic.
A. Lesions are usually solid but can be cystic. Most cases of RCC are sporadic, but many hereditary forms
B. May be sporadic or hereditary. have been described. Up to 20% to 30% of patients may pres-
C. Surgical debulking can improve survival in patients ent with metastatic disease, in which case, surgical debulking
who present with metastatic disease. can improve survival, as shown in randomized controlled tri-
D. Patients are not curable (and therefore should not be als. Up to 10% of RCC invades the lumen of the renal vein or
operated on) if tumor thrombus extends proximally vena cava. The degree of venous extension directly impacts
into the vena cava. the surgical approach. Patients with thrombus below the
level of the liver can be managed with cross-clamping above
and below the thrombus and extraction from a cavotomy
at the insertion of the renal veins. (See Schwartz 11th ed.,
pp. 1772–1773.)

337

Brunicardi_Ch40_p337-340.indd 337 30/06/22 11:27 AM


338
4. Which of the following statements is FALSE concerning Answer: C
carcinoma of the prostate? The American Urological Association has advised screen-
A. Annual digital rectal examination and serum pros- ing for men 55 to 69 years of age. The most common site of
tate-specific antigen (PSA) determinations are rec- spread of prostate cancer is to the pelvic lymph nodes and
ommended beginning at age 55. bone. Radical prostatectomy is associated with early incon-
B. Lung metastasis is less common than bone metastasis. tinence and erectile dysfunction. Incontinence improves sig-
C. Radical prostatectomy is associated with a 5% inci- nificantly with time, with <1% of men, in experienced hands,
dence of permanent urinary incontinence suffering severe long-term problems with urinary control.
D. Once prostate cancer has spread, it is no longer cur- (See Schwartz 11th ed., pp. 1774–1775.)
CHAPTER 40

able but can be contained by lowering serum testos-


terone and/or by administration of androgen receptor
blockers.

5. Concerning ureteric trauma, which of the following Answer: A


statements is TRUE? A retrograde pyelogram is the most sensitive test for ureteral
A. Retrograde pyelogram is the most sensitive test to injury. Partial injuries can be primarily repaired, although
Urology

detect ureteral injury. all devitalized tissues must be debrided to avoid delayed
B. Bladder mobilization is not integral to repair of ure- tissue breakdown and urinoma formation. Ureteral stents
teric injury. should be placed in this situation to facilitate healing with-
C. Kidney mobilization is not integral to repair of ure- out stricture. Midureteral-level injuries can be treated with a
teric injury. uretero-­ureterostomy if a spatulated tension-free repair can
D. Use of ureteric stents is not useful in preventing post- be achieved. For longer defects, the bladder can be mobi-
repair strictures. lized and brought up to the psoas muscle (psoas hitch). For
­additional length, a tubularized flap of bladder (Boari flap)
can be created and anastomosed to the remaining ureter.
(See Schwartz 11th ed., pp. 1765–1766.)

6. All of the following are TRUE concerning priapism Answer: B


EXCEPT: Priapism is a persistent erection for >4 hours unrelated to
A. Priapism is defined as a persistent erection for more sexual stimulation. Risk factors include sickle cell disease or
than 4 hours unrelated to sexual stimulation. trait, malignancy, medications, cocaine abuse, certain antide-
B. Etiologic factors include sickle cell disease, malig- pressants, and total parenteral nutrition. Low-flow priapism
nancy, total parenteral nutrition, and penile shaft can be confirmed with a penile blood gas of the cavernosal
fractures. bodies demonstrating hypoxic, acidotic blood. Injection of
C. Low-flow priapism can be confirmed with a penile phenylephrine (up to 200 mg in 20 mL normal saline) into
blood gas determination. the corporal bodies may be required. (See Schwartz 11th ed.,
D. Treatment may require injection of phenylephrine. pp. 1769–1770.)

7. Prostatitis is typified by all of the following EXCEPT: Answer: B


A. Pain, dysuria, urgency. Acute prostatitis is marked by fever, suprapubic or perineal
B. Fever in both acute and chronic prostatitis. pain, and new onset lower urinary tract symptoms, includ-
C. Treatment consists of long-term antibiotics. ing dysuria, frequency, urgency, changes in stream caliber,
D. Workup may include culture of expressed prostatic or difficulty emptying the bladder. Treatment consists of
secretions. a long-term course (4–6 weeks) of antibiotics. Fever is not
observed in chronic prostatitis, and onset may occur over
many months. Patients with chronic bacterial prostatitis may
also report recurrent urinary tract infections (UTIs). Differ-
entiation between acute and chronic etiologies requires cul-
ture of expressed prostatic secretion to confirm the presence
or absence of bacteria. (See Schwartz 11th ed., p. 1761.)

8. Which of the following statements is correct about Answer: A


urolithiasis? Stone formation has been associated with obesity, metabolic
A. Stone formation is associated with diabetes. syndrome, and diabetes mellitus. Stones are most commonly
B. Uric acid stones are the most common type. composed of calcium oxalate. A noncontrast computer
C. Magnetic resonance imaging (MRI) is the most accu- tomography (CT) scan is the most sensitive (98%) and spe-
rate diagnostic test. cific (97%) examination to detect urolithiasis. Ureteroscopy
D. Lithotripsy is best for stones located in the middle or is the procedure of choice for patients with middle or distal
distal ureter. ureteral stones. (See Schwartz 11th ed., pp. 1762–1763.)

Brunicardi_Ch40_p337-340.indd 338 30/06/22 11:27 AM


339
9. Concerning benign prostatic hyperplasia (BPH), which Answer: D
of the following statements is FALSE? The prevalence of BPH in men over the age of 50 is estimated
A. It is found in 80% of patients 70 years or older. at 50% to 75% and increases with age with a prevalence of
B. Complications can include renal failure. 80% in men over the age of 70. Complications of BPH include
C. Transurethral resection is the mainstay of invasive urinary retention, incontinence, renal failure, hematuria, or
therapeutic interventions. recurrent infections. The first line of treatment is most com-
D. Beta-blockers are the most common pharmacologic monly pharmacotherapy for those men with bothersome
intervention. symptoms; alpha blockers work by relaxing the smooth mus-
cle of the prostate and bladder neck. Transurethral resection

CHAPTER 40
of the prostate (TURP) remains the mainstay of endoscopic
procedures. (See Schwartz 11th ed., pp. 1763–1764.)

10. Which of the following statements is FALSE concerning Answer: A


Fournier’s gangrene (necrotizing fasciitis)? Fournier’s gangrene is a necrotizing fasciitis of the male geni-
A. Mortality is 25%. talia and perineum that can be rapidly progressive and fatal if
B. Risk factors include diabetes, obesity, and alcoholism. not treated promptly. The mortality rate is as high as 67%. Risk

Urology
C. Often polymicrobial. factors for Fournier’s gangrene include perirectal abscesses,
D. Prompt surgical debridement mandatory. diabetes, obesity, and chronic alcoholism. The often polymi-
crobial infection spreads and can dissect along Scarpa and
Colles fascia. Prompt and aggressive surgical debridement of
nonviable tissue and broad spectrum antibiotics are necessary
to prevent further spread. (See Schwartz 11th ed., p. 1769.)

11. Concerning testicular cancer, which of the following Answer: C


statements are TRUE? Testicular cancer is the most common cancer in men age 20 to
A. Second most common cancer in men ages 20–40. 40 years and the second most common cancer in young men
B. Nonseminomatous variants are the most common age 15 to 19 years. Standard initial workup includes scrotal
subtype. ultrasound and serum tumors markers (α-fetoprotein, quan-
C. Radical inguinal orchiectomy is the “gold standard” titative human chorionic gonadotropin, and lactate dehydro-
of treatment. genase). Radical inguinal orchiectomy is the gold standard
D. Workup includes biomarker assays for mutated p53; treatment for excision of the primary tumor. (See Schwartz
overexpression of matrix metalloproteinases, and 11th ed., pp. 1771–1772.)
estrogen receptor.

12. All of the following statements are TRUE of renal cell Answer: D
carcinoma (RCC), EXCEPT: Most patients diagnosed with RCC in the modern era typi-
A. Renal CT imaging with contrast is the definitive cally present with an incidentally discovered renal mass
radiologic test. on the abdominal radiographic imaging. Renal computed
B. Risk factors include smoking, obesity, and hypertension. tomography (CT) imaging with intravenous contrast remains
C. Familial syndromes should be suspected in younger the single most important radiographic test to determine the
patients and in patients with bilateral tumors. nature of the mass. Major recognized risk factors for RCC
D. Renal masses of any size, if suspicious for malignancy, include obesity and hypertension. Familial RCC subtypes
require excision as the first therapeutic maneuver. with classical clinical manifestations are also well described.
Management options for small renal masses (<4 cm) include
active surveillance, thermoablative techniques, or surgical
excision. Percutaneous or laparoscopic techniques (cryo-
ablation, radiofrequency ablation, high intensity-focused
­ultrasound) have been used to treat small renal masses, but
they are associated with an increased risk of local recurrence.
(See Schwartz 11th ed., pp. 1772–1773.)

Brunicardi_Ch40_p337-340.indd 339 30/06/22 11:27 AM


This page intentionally left blank

Brunicardi_Ch40_p337-340.indd 340 30/06/22 11:27 AM


CHAPTER 41
Gynecology

1. Concerning uterine leiomyoma, all of the following are Answer: D


TRUE EXCEPT: Leiomyoma, also known colloquially as fibroids, are the
A. It is most common pelvic tumor. most common female pelvic tumor and occur in response
B. It has a racial predilection. to growth of the uterine smooth muscle cells (myometrium).
C. It is classified according to anatomic location. They are common in the reproductive years, and by age 50, at
D. It rarely necessitates hysterectomy. least 60% of white and up to 80% of black women are or have
been affected. Leiomyoma are described according to their
anatomic location (Fig. 41-1) as intramural, subserosal, sub-
mucosal, pedunculated, cervical, and rarely ectopic. Most are
asymptomatic; however, abnormal uterine bleeding caused by
leiomyoma is the most common indication for hysterectomy
in the United States. (See Schwartz 11th ed., p. 1795.)

Intramural

Intercavitary

Pedunculated
Subserous
Submucous

Prolapsed

FIG. 41-1. Types of uterine myomas.

2. Typical indications for cesarean delivery include all of Answer: D


the following EXCEPT: Typical indications for cesarean delivery include question-
A. Questionable fetal status. able fetal status, breech or other maIpresentations, triplet and
B. Breech presentation. higher order gestations, cephalopelvic disproportion, failure
C. Cephalopelvic disproportion. to progress, placenta previa, and active genital herpes. (See
D. Maternal coagulopathy. Schwartz 11th ed., p. 1806.)

341

Brunicardi_Ch41_p341-344.indd 341 30/06/22 11:28 AM


342
3. Pelvic floor dysfunction includes all of the following Answer: D
EXCEPT: Pelvic floor disorders can be categorized, from an urogyne-
A. Urinary incontinence. cologic perspective, into three main topics: female urinary
B. Pelvic organ prolapse. incontinence and voiding dysfunction, pelvic organ pro-
C. Fecal incontinence. lapse, and disorders of defecation. (See Schwartz 11th ed.,
D. Dyspareunia. pp. 1807–1809.)

4. All of the following are TRUE concerning stress inconti- Answer: D


nence EXCEPT: Stress incontinence is believed to be caused by lack of uretero-
CHAPTER 41

A. It can be due to lack of urethrovaginal support. vaginal support (urethral hypermobility) or intrinsic sphincter
B. It can be due to intrinsic sphincter deficiency. deficiency (ISD). ISD is a term applied to a subset of stress-
C. Goal of surgical repair is to create a partial urethral incontinent patients who have particularly severe symptoms,
obstruction. including urine leakage with minimal exertion. This condition
D. Urethral rein plantation is sometimes necessary if is often recognized clinically as the low pressure or “drainpipe”
other approaches fail. urethra. The urethral sphincter mechanism in these patients
is severely damaged, limiting cooptation of the urethra.
Gynecology

Standard surgical procedures used to correct stress inconti-


nence share a common feature: partial urethral obstruction
that achieves urethral closure under stress. (See Schwartz 11th
ed., pp. 1808–1809.)

5. Concerning vuIvar carcinoma, all of the following are Answer: B


TRUE EXCEPT: Evidence supports a HPV-dependent pathway of carcinogen-
A. Etiology may be due to an human papillomavirus esis. Vulvar carcinomas are squamous in 90% of cases. Spread
(HPV)-dependent pathway of carcinogenesis. of vulvar carcinoma is by direct local extension and via lym-
B. Approximately 50% are squamous lesions. phatic microembolization. Hematogenous spread is uncom-
C. Hematogenous dissemination is rare. mon. Staging and primary surgical treatment are typically
D. Staging and primary surgical treatment are typically performed as a single procedure and tailored to the individual
performed as a single procedure. patient. (See Schwartz 11th ed., pp. 1809–1810.)

6. Which of the following is FALSE concerning epithelial Answer: C


ovarian cancer (EOC) risk factors? Risk factors for development of EOC include events that
A. Risk factors include early menarche. appear to increase the number of lifetime ovulations (eg, early
B. Risk factors include late menopause. menarche, late menopause, nulliparity), whereas events that
C. Risk factors include previous hysterectomy. decrease the number of ovulations decrease risk (eg, preg-
D. Risk factors include nulliparity. nancy, breast feeding, oral contraceptives). Additionally, a
history of tubal ligation or hysterectomy also decreases EOC
risk. (See Schwartz 11th ed., pp. 1815–1820.)

7. Concerns human papillomavirus (HPV) vaccination, Answer: A


which of the following is TRUE? Three HPV vaccines have been approved by the US Food
A. Vaccination generates high concentrations of neu- and Drug Administration (FDA). Vaccination generates high
tralizing antibodies to HPVL1 protein. concentrations of neutralizing antibodies to HPV L1 protein.
B. There is only one FDA-approved vaccination available. Current recommendations include HPV vaccination for boys
C. Vaccination should start at age 11 for girls, and age and girls at age 11 and 12 years, respectively. Two doses are
13 for boys. given 6 to 12 months apart for patients with an intact immune
D. Two doses are given 3 weeks apart. system. (See Schwartz 11th ed., p. 1790.)

8. Concerning endometriosis, which of the following is Answer: A


TRUE? Endometriosis commonly causes elevations in serum CA-125.
A. It causes elevation of serum CA-125. Definitive diagnosis usually requires laparoscopy and visual-
B. Biopsy is routinely performed due to variability in ization of the pathognomonic endometriotic implants. These
appearance of lesions. appear as blue, brown, black, white, or yellow lesions that can
C. It is a premalignant lesion. be raised and at times puckered giving them a “gunpowder”
D. Symptomatically mild cases are best treated with appearance. Biopsy is not routinely done but should be
gonadotropin releasing hormone (GnRH). obtained if the diagnosis is in doubt. Those with mild symp-
toms can be managed with oral contraceptive pills and/or
anti-inflammatory analgesia; moderate symptoms are treated
with medroxyprogesterone acetate. Severe symptoms are

Brunicardi_Ch41_p341-344.indd 342 30/06/22 11:28 AM


343
treated with gonadotropin releasing hormone (GnRH) to
induce medical pseudomenopause. Although endometriosis
is not generally thought to be a premalignant lesion, there is
an increased risk of type I ovarian cancer in women with a his-
tory of endometriosis. (See Schwartz 11th ed., pp. 1802–1803.)

9. Which of the following statements is FALSE concerning Answer: D


pelvic inflammatory disease (PID)? Sexually transmitted organisms, especially Neisseria gonor-
A. Etiology is sexually transmitted organisms. rhoeae and Chlamydia trachomatis, are implicated in many

CHAPTER 41
B. Sequellae include increased risk of ectopic pregnancy. cases, although microorganisms that comprise the vaginal
C. Initial therapy requires intravenous antibiotics. flora (eg, anaerobes, Gardnerella vaginalis, Haemophilus influ-
D. Clinical criteria include an oral temperature >101° F. enza, enteric Gram-negative rods, and Streptococcus agalac-
tiae) have been implicated as well. Because of the psychosocial
complexity associated with a diagnosis of PID; when possible,
additional criteria should be used to enhance the specificity
of the minimum clinical criteria. These include oral temper-

Gynecology
ature >101° F (>38.3° C). Several outpatient parenteral and
oral antimicrobial regimens have been effective in achieving
clinical and microbiologic cure. Hospitalization for intrave-
nous antibiotics may be necessitated in cases where surgical
emergencies cannot be ruled out. (See Schwartz 11th ed.,
pp. 1803–1804.)

10. Options for management of stress urinary incontinence Answer: D


include all of the following EXCEPT: Stress incontinence is believed to be caused by lack of ure-
A. Creation of mid urethral sling. throvaginal support (urethral hypermobility) or intrinsic
B. Urethral bulking injections. sphincter deficiency. Despite wide acceptance of midurethral
C. Mesh reconstruction. sling procedures, a retropubic urethropexy procedure called
D. Bilateral urogenital diaphragm plication. the Burch procedure is still performed for stress incontinence.
A transurethral or periurethral injection of bulking agents
is indicated for patients with intrinsic sphincter deficiency.
Pelvic reconstructive surgery frequently uses polypropylene
mesh to augment procedures in the hope of providing long-
lasting repair. (See Schwartz 11th ed., pp. 1808–1809.)

11. Concerning vaginal cancer, which of the following state- Answer: C


ments is TRUE? Vaginal carcinoma is a rare gynecologic malignancy and
A. Squamous lesions are associated with in utero expo- accounts for about 3% of cancers affecting the female repro-
sure to DES. ductive system. Rare clear cell carcinoma of the vagina is
B. Staging is by a combination of clinical and pathologic associated with in utero exposure to diethylstilbestrol (DES).
assessments. Vaginal cancer is staged clinically by pelvic examination, chest
C. Early disease prognosis is >90% 5-year overall X-ray, cystoscopy, and proctoscopy. External beam pelvic radi-
survival. ation is the mainstay of treatment for stages II, III, and IV and
D. Surgery is indicated for stages III and IV disease. may be followed by intracavitary and/or interstitial brachy-
therapy. Prognosis for treated early stage disease is excel-
lent with >90% 5-year survival rates. (See Schwartz 11th ed.,
pp. 1810–1811.)

Brunicardi_Ch41_p341-344.indd 343 30/06/22 11:28 AM


This page intentionally left blank

Brunicardi_Ch41_p341-344.indd 344 30/06/22 11:28 AM


CHAPTER 42
Neurosurgery

1. Which intracranial pathology is associated with the cor- Answer: A


rect sign or symptom? Increased ICP can injure the brain in several ways. Focal mass
A. Medialization of the uncus and compression of the lesions cause shift and herniation. Temporal lesions push the
midbrain leading to uncal herniation uncus medially and compress the midbrain. This phenom-
B. Increased intracranial pressure (ICP) causing men- enon is known as uncal herniation. Masses higher up in the
tal status decline due to compression of the bilateral hemisphere can push the cingulate gyrus under the falx cere-
frontal lobes bri. This process is known as subfalcine herniation. Diffuse
C. Choroid plexus production of cerebral spinal fluid in increases in pressure in the cerebral hemispheres can lead
excess of 1L/day leading to subfalcine herniation to central, or transtentorial, herniation. Increased pressure
D. ICP classically presenting as hypotension, bradycar- in the posterior fossa can lead to upward central herniation
dia, and irregular respirations or downward tonsillar herniation through the foramen mag-
num. Uncal, transtentorial, and tonsillar herniation can cause
direct damage to the brain stem (Fig. 42-1). (See Schwartz
11th ed., Figure 42-4, p. 1831.)

3
2
FIG. 42-1. Schematic drawing of brain herniation patterns.
1. Subfalcine herniation. The cingulate gyrus shifts across midline
under the falx cerebri. 2. Uncal herniation. The uncus (medial
temporal lobe gyrus) shifts medially and compresses the midbrain
and cerebral peduncle. 3. Central transtentorial herniation. The
diencephalon and midbrain shift caudally through the tentorial 4
incisura. 4. Tonsillar herniation. The cerebellar tonsil shifts
caudally through the foramen magnum. (Reproduced with
permission from Wilkins RH, Rengachary SS: Neurosurgery,
2nd ed. New York, NY: McGraw Hill; 1996.)

345

Brunicardi_Ch42_p345-350.indd 345 04/07/22 2:47 PM


346
2. The lesion that can cause mass effect and rapidly kill the Answer: B
patient is: The posterior fossa (brain stem and cerebellum) requires
A. Inferior fossa lesions. special consideration because the volume of the posterior
B. Posterior fossa lesions. fossa within the cranial vault is small. Posterior fossa lesions
C. Progressive obtundation. such as tumors, hemorrhage, or stroke can cause mass effect
D. Bradycardial lesions. that can rapidly kill the patient in two ways. Occlusion of the
fourth ventricle can lead to acute obstructive hydrocephalus,
raised increased intracranial pressure (ICP), herniation, and
eventually death. This mass effect can also lead directly to
CHAPTER 42

brain stem compression (Fig. 42-2). Symptoms of brain stem


compression include hypertension, agitation, and progressive
obtundation, followed rapidly by brain death. A patient exhib-
iting any of these symptoms needs an emergent neurosurgi-
cal evaluation for possible ventriculostomy or suboccipital
craniectomy (removal of the bone covering the cerebellum).
This situation is especially critical, as expeditious decompres-
Neurosurgery

sion can lead to significant functional recovery. (See Schwartz


11th ed., p. 1833.)

FIG. 42-2. Maturing cerebellar stroke seen as a hypodense


area in the right cerebellar hemisphere (arrowhead) on head
computed tomography in a patient with rapidly progressing
obtundation 2 days after the initial onset of symptoms.
Swelling of the infarcted tissue causes posterior fossa mass
effect. The fourth ventricle is obliterated and not visible,
and the brain stem is being compressed.

3. Post trauma day 4, a 19-year-old man with significant Answer: D


facial trauma and associated temporal bone articulates Fractures of the skull base are common in head-injured
which of the following symptoms? patients, and they indicate significant impact. They are gener-
A. Double vision and facial asymmetry ally apparent on routine head CT, but they should be evaluated
B. Headache, light sensitivity, and fatigue with dedicated fine-slice coronal-section CT scan to docu-
C. Anosmia and rhinorrhea ment and delineate the extent of the fracture and involved
D. Vertigo, ipsilateral deafness, and facial paralysis structures. If asymptomatic, they require no treatment. Skull
base fractures requiring intervention include those with an
associated cranial nerve deficit or cerebrospinal fluid (CSF)
leak. A fracture of the temporal bone, for instance, can dam-
age the facial or vestibulocochlear nerve, resulting in vertigo,
ipsilateral deafness, or facial paralysis. A communication may
be formed between the subarachnoid space and the middle
ear, allowing CSF drainage into the pharynx via the Eusta-
chian tube or from the ear (otorrhea). Extravasation of blood
results in ecchymosis behind the ear, known as Battle’s sign.

Brunicardi_Ch42_p345-350.indd 346 04/07/22 2:47 PM


347
A fracture of the anterior skull base can result in anosmia
(loss of smell from damage to the olfactory nerve), CSF drain-
age from the nose (rhinorrhea), or periorbital ecchymosis,
known as raccoon eyes. (See Schwartz 11th ed., p. 1835.)

4. In regard to the halo test, a positive indicator for cerebro- Answer: B


spinal fluid (CSF) is when tinged with blood will show Copious clear drainage from the nose or ear makes the diag-
which of the following when dropped on an absorbent nosis of CSF leakage obvious. Often, however, the drainage
tissue? may be discolored with blood or small in volume if some

CHAPTER 42
A. A single ring with a darker center spot containing drains into the throat. In indeterminate cases, it is important
blood components surrounded by a light halo of CSF to consider radiographic findings on the computed tomogra-
B. A double ring with a darker center spot containing phy (CT) scan near the fracture that suggest CSF leak, such as
blood components surrounded by a light halo of CSF pneumocephalus, subarachnoid, or intraparenchymal blood
C. A single ring with a lighter center spot containing CSF at the fracture site. The “halo” test assesses for a double ring
surrounded by a darker halo of blood components when a drop of the fluid is allowed to fall on an absorbent
D. A double ring with a lighter center spot containing CSF surface, but it has been shown to have poor clinical utility. The

Neurosurgery
surrounded by a darker halo of blood components fluid can be sent for β-2 transferrin testing, a carbohydrate-
free isoform of transferrin exclusively found in the CSF;
however, these tests often take 1 to 2 weeks to result and also
can be difficult to incorporate into clinical practice. (See
Schwartz 11th ed., p. 1835.)

5. Which of the following closed brain injury is paired with Answer: C


the correct description? A concussion is defined as temporary neuronal dysfunction
A. Contusion, bruise of the brain from breakdown of following nonpenetrating head trauma. The head computed
small blood vessels and extravasation into the brain tomography (CT) is normal, and deficits resolve over min-
most commonly affecting the posterior fossa utes to hours. Definitions vary; some require transient loss of
B. Diffuse axonal injury, due to acceleration and decel- consciousness, while others include patients with any altera-
eration leading to axonal disruption and/or detach- tion of mental status. Memory difficulties, especially amne-
ment which is not visible on magnetic resonance sia of the event, are very common. Studies have shown that
imaging (MRI) the brain remains in a hypermetabolic state for up to a week
C. Concussion, temporary neuronal dysfunction lead- after injury. The brain is also much more susceptible to injury
ing to susceptibility to secondary brain injury from even minor head trauma in the first 1 to 2 weeks after
D. Traumatic brain injury, diffuse intracranial hyperten- concussion. This is known as second-impact syndrome, and
sion with parenchymal hyperemia visible on MRI patients should be informed that, even after mild head injury,
they might experience memory difficulties or persistent
headaches. Return to play guidelines after sports-related con-
cussions are controversial and are under active debate. (See
Schwartz 11th ed., p. 1836.)

6. A patient who withdraws from pain, is mumbling inap- Answer: C


propriate words and opens his eyes to pain has a Glasgow See Table 42-1. (See Schwartz 11th ed., Table 42-2, p. 1830.)
Coma Scale score of:
A. 3.
B. 6.
C. 9.
D. 12.

TABLE 42-1 The Glasgow Coma Scale scorea


Motor Response Verbal Response Eye-Opening Response
Obeys commands 6 Oriented 5 Opens spontaneously 4
Localizes to pain 5 Confused 4 Opens to speech 3
Withdraws from pain 4 Inappropriate words 3 Opens to pain 2
Flexor posturing 3 Unintelligible sounds 2 No eye opening 1
Extensor posturing 2 No sounds 1
No movement 1
Add the three scores to obtain the Glasgow Coma Scale (GCS) score, which can range from 3 to 15. Add “T” after the GCS if intubated and no verbal score is possible. For these
a

patients, the GCS can range from 3T to 10T.

Brunicardi_Ch42_p345-350.indd 347 04/07/22 2:47 PM


348
7. The most common malignant tumor of the brain is: Answer: B
A. Ependymoma. Astrocytoma is the most common primary central nervous
B. Astrocytoma. system (CNS) neoplasm. The term glioma often is used
C. Ganglioglioma. to refer to astrocytomas specifically, excluding other glial
D. Teratoma. tumors. Astrocytomas are graded from I to IV. Grades I and II
are referred to as low-grade astrocytoma or low-grade
glioma, grade III as anaplastic astrocytoma, and grade IV
as glioblastoma multiforme (GBM). Prognosis varies sig-
nificantly between grades I/II, III, and IV, but not between
CHAPTER 42

I and II. Median survival is 8 years after diagnosis with a low-


grade tumor, 2 to 3 years with an anaplastic astrocytoma, and
roughly 1 year with a GBM. GBMs account for almost two-
thirds of all astrocytomas, anaplastic astrocytomas account
for two-thirds of the rest, and low-grade astrocytomas the
remainder. Figure 42-3 demonstrates the typical appearance
of a GBM. (See Schwartz 10th ed., p. 1855.)
Neurosurgery

A B

FIG. 42-3. A. Postcontrast T1-weighted axial magnetic resonance imaging demonstrating a ring-enhancing lesion in the anteromedial
right temporal lobe with central necrosis (dark area) consistent with glioblastoma multiforme. B. T2-weighted axial magnetic resonance
imaging with extensive bright signal signifying peritumoral edema seen with glioblastoma multiformes.

8. Medical management of traumatic brain injury with an Answer: B


associated subdural hematoma requires which steps to Several medical steps may be taken to minimize secondary
minimize secondary injury including: injury and the systemic consequences of head injury. Patients
A. High-dose methylprednisolone to reduce 6-month with a documented closed head injury and evidence of intra-
mortality, especially in severe traumatic brain injury. cranial hemorrhage or a depressed skull fracture should
B. Antiseizure and peptic ulcer prophylaxis as well as receive a 1 g Keppra loading dose, followed by 1 week of ther-
blood glucose, vital sign, and temperature monitoring. apeutic maintenance Keppra, typically 500 mg twice a day.
C. Permissive hypotension to reduce intracranial hema- Antiseizure prophylaxis has been shown to decrease the inci-
toma growth. dence of early posttraumatic seizures. There is no evidence
D. Prophylactic administration of platelets for patients to support long-term use of prophylactic antiepileptic agents.
with intracranial hemorrhage and exposure to aspirin. Blood glucose levels should be closely monitored by free

Brunicardi_Ch42_p345-350.indd 348 04/07/22 2:47 PM


349
blood sugar checks and controlled with sliding scale insulin.
Fevers also should be evaluated and controlled with antipyret-
ics, as well as source-directed therapy when possible. Hyper-
glycemia and hyperthermia are toxic to injured neurons and
contribute to secondary injury. Head-injured patients have
an increased prevalence of peptic ulceration and gastrointes-
tinal (GI) bleeding. Peptic ulcers occurring in patients with
head injury or high increased intracranial pressure (ICP)
are referred to as Cushing ulcers. Ulcer prophylaxis should

CHAPTER 42
be used. Compression stockings or athrombic pumps should
be used when the patient cannot be mobilized rapidly for pro-
phylaxis of deep venous thrombosis. (See Schwartz 11th ed.,
p. 1839.)

9. The most common level of cervical radiculopathy from Answer: C


cervical disc herniation is: The cervical nerve roots exit the central canal above the

Neurosurgery
A. C4–C5. pedicle of the same-numbered vertebra and at the level of the
B. C5–C6. higher adjacent intervertebral disc. For example, the C6 nerve
C. C6–C7. root passes above the C6 pedicle at the level of the C5–C6
D. C7–T1. discs. The cervical nerve roots may be compressed acutely by
disc herniation, or chronically by hypertrophic degenerative
changes of the discs, facets, and ligaments. Table 42-2 sum-
marizes the effects of various disc herniations. Most patients
with acute disc herniations will improve without surgery.
Nonsteroidal anti-inflammatory drugs (NSAIDs) or cervical
traction may help alleviate symptoms. Patients whose symp-
toms do not resolve or who have significant weakness should
undergo decompressive surgery. The two main options for
nerve root decompression are anterior cervical discectomy
and fusion (ACDF) and posterior cervical foraminotomy
(keyhole foraminotomy). ACDF allows more direct access
to and removal of the pathology (anterior to the nerve root).
However, the procedure requires fusion because discectomy
causes a collapse of the interbody space and instability will
likely occur. Figure 42-4 demonstrates a C6–C7 ACDF with
the typical interposed graft and plating system. Keyhole
foraminotomy allows for decompression without requiring
fusion, but it is less effective for removing centrally located
canal pathology. (See Schwartz 11th ed., p. 1862.)

TABLE 42-2 Cervical disc herniations and symptoms by level


Level Frequency (%) Root Injured Reflex Weakness Numbness
C4–C5 2 C5 — Deltoid Shoulder
C5–C6 19 C6 Biceps Biceps brachii Thumb
C6–C7 69 C7 Triceps Wrist extensors (wrist drop) Second and third digits
C7–T1 10 C8 — Hand intrinsics Fourth and fifth digits
Adapted with permission from Greenberg MS. Handbook of Neurosurgery, 7th ed. New York, NY: Thieme; 2010.

Brunicardi_Ch42_p345-350.indd 349 04/07/22 2:47 PM


350
CHAPTER 42
Neurosurgery

A B

FIG. 42-4. A. Anteroposterior cervical spine X-ray showing the position of an anterior cervical plate used for stabilization after
C6–C7 discectomy. Patient presented with right triceps weakness and dysesthesias in the right fifth digit. Magnetic resonance imaging
revealed a right paracentral C6–C7 herniated disc compressing the exiting C7 nerve root. B. Lateral cervical spine X-ray of the same
patient clearly demonstrates the position of the plate and screws. The allograft bone spacer placed in the drilled-out disc space is
also apparent.

10. Urinary retention, saddle anesthesia, and progressing leg Answer: C


weakness are: Cauda equina syndrome is due to compression of the cauda
A. Most often secondary to compression of the thoracic equina and may result from massive disc herniation, epidural
spine secondary to massive disc herniation into the hematoma (EDH), epidural abscess, tumor, or subluxation
cauda equina. from trauma. Patients with cauda equina compression often
B. A surgical emergency and diagnosis should be made present with urinary retention, saddle anesthesia, or pro-
clinically without evaluation of or confirmation by gressing leg weakness. Saddle anesthesia is numbness in the
preoperative imaging. perineum, genitals, buttocks, and upper inner thighs. Patients
C. Due to compression of the lumbar nerve roots, which with suspected cauda equina syndrome should undergo
require urgent intervention to preserve sphincter immediate magnetic resonance imaging (MRI) of the lum-
function and ambulation. bar spine to evaluate for a surgical lesion. Mass lesions should
D. A diagnosis of exclusion following failure to respond be removed urgently via laminectomy to preserve sphincter
to systemic steroid treatment (steroid challenge). function and ambulation. (See Schwartz 11th ed., p. 1865.)

Brunicardi_Ch42_p345-350.indd 350 04/07/22 2:47 PM


CHAPTER 43
Orthopedic Surgery

1. Long bone fractures can be described as any of the Answer: A


­following recognized types EXCEPT: Musculoskeletal injuries resulting from trauma include
A. Convoluted. ­fractures of bones, damage to joints, and injuries to soft tis-
B. Transverse. sues. Long bone fractures can be described as transverse,
C. Oblique. oblique, spiral, segmental, or comminuted. (See Schwartz
D. Spiral. 11th ed., p. 1881.)

2. Goals of fracture reduction include all of the following Answer: B


EXCEPT: Reduction is performed with axial traction and reversal of the
A. Restore length. mechanism of injury in order to restore length, rotation, and
B. Restore marrow integrity. angulation. (See Schwartz 11th ed., p. 1881.)
C. Restore rotation.
D. Restore angulation.

3. Which of the following is TRUE concerning compart- Answer: C


ment syndromes? Compartment syndrome is an orthopedic emergency caused
A. It is due to decreased intracompartmental pressure. by significant swelling within a compartment of an injured
B. It is typified by hyperesthesia. extremity that jeopardizes blood flow to the limb. Increased
C. It can be assessed by needles placed into affected pressure within the compartment compromises perfusion to
compartment. muscles and can cause ischemia or necrosis. Patients com-
D. Pain is relieved by passive muscle stretching. plain of pain and numbness, and passive stretch of muscles
within the compartment causes severe pain. While the diag-
nosis is based on clinical examination, pressures can be mea-
sured with needles placed into the compartment, which is
necessary in unconscious patients who may not show these
typical exam findings. (See Schwartz 11th ed., p. 1883.)

4. Each of the following is associated with pelvic fracture Answer: A


EXCEPT: Pelvic fractures are indicative of high-energy trauma and are
A. Prompt operative intervention for pubic rami associated with head, chest, abdominal, and urogenital inju-
fractures. ries. Hemorrhage from pelvic trauma can be life-threatening
B. Life-threatening hemorrhage. and patients can present with hemodynamic instability requir-
C. Associated genitourinary (GU) injury. ing significant fluid resuscitation and blood transfusions. The
D. Displacement associated with two or more fractures bleeding that occurs is often due to injury of the venous plexus
in the pelvic ring. in the posterior pelvis, though it can also be due to a large ves-
sel injury such as a gluteal artery. Other associated injuries
are bladder and urethral injuries that manifest with bleed-
ing from the urethral meatus or blood in the bladder cath-
eter, and need to be assessed with a retrograde urethrogram.
The pelvis is a ring structure made up of the sacrum and the
two innominate bones that are held together by strong liga-
ments. Because it is a ring, displacement can only occur if the

351

Brunicardi_Ch43_p351-354.indd 351 30/06/22 11:28 AM


352
ring is disrupted in two places. Displaced sacral fractures and
iliac wing fractures are treated with screws or plates, while
pubic rami fractures can usually be managed nonoperatively.
(See Schwartz 11th ed., pp. 1887–1888.)

5. Chronic unremitting back pain suggests all of the follow- Answer: C


ing possibilities EXCEPT: Back pain occurs in the majority of adults but is usually self-
A. Infection. limited, resolving in 1 to 2 weeks. Chronic unremitting back
B. Malignancy (primary). pain suggests the possibility of infection, malignancy, or met-
CHAPTER 43

C. Spinal cord infarction. astatic disease. (See Schwartz 11th ed., p. 1904.)
D. Metastatic disease.

6. Concerning long bone fractures, which of the following Answer: B


is TRUE? An open fracture occurs when the bone breaks through the
A. These are classified into four categories (Gustillo- skin. These injuries are classified into three types accord-
Anderson classification). ing to the Gustillo-Anderson classification. These injuries
Orthopedic Surgery

B. Treatment of these fractures consists of immediate require immediate administration of antibiotics and irri-
antibiotics, irrigation, debridement. gation and debridement of the wound. When the wound is
C. External fixation needed for clean and contaminated contaminated, an external fixator is initially used. Early cov-
wound. erage of the wound is important to avoid infection. Usually
D. Distal tibial wounds can be covered by local muscle a large wound in the proximal or middle third of the tibia
flaps. can be covered using local muscle flaps, while the distal third
of the tibia will require a free flap. (See Schwartz 11th ed.,
pp. 1881–1883.)

7. Which of the following is FALSE concerning humeral Answer: C


shaft fractures? The majority of humeral shaft fractures can heal with non-
A. Radial nerve injuries are primarily neuropraxias. surgical management if they are within an acceptable degree
B. Management is primarily nonsurgical. of angulation. The radial nerve spirals around the humeral
C. Gentle motion exercises are begun at 8 weeks. shaft and is at risk for injury. Most radial nerve injuries are
D. Radial nerve injury healing is monitored by neurapraxias, or stretching of the nerve, and function typi-
­electromyography (EMG). cally returns within 3 to 4 months. Gentle motion exercises
are begun within 1 to 2 weeks. The patient should have
an EMG to monitor recovery of the nerve. (See Schwartz
11th ed., pp. 1884–1885.)

8. Concerning knee dislocations, which of the following Answer: D


statements is FALSE? When the knee dislocates, the anterior cruciate ligament
A. Anterior and posterior cruciate ligaments are torn. and posterior cruciate ligament are torn. The neurovascular
B. Common peroneal nerve is prone to injury. examination is extremely important, focusing on the com-
C. Immediate reduction is critical. mon peroneal nerve and the vascular status of the extremity,
D. Normal arterial pulses preclude need for further vas- followed by immediate reduction of the knee and subsequent
cular assessment. neurovascular examination. If the pulses are normal, the
ankle brachial index (ABI) should be measured. If the ABI
is >0.5, the patient should be monitored with serial examina-
tion. If the ABI is less than 0.9, then a CT scan or an arte-
rial duplex ultrasound should be performed. (See Schwartz
11th ed., p. 1890.)

9. Concerning shoulder dislocations, which of the follow- Answer: C


ing statements is FALSE? Prolonged immobilization is not recommended because this
A. Association with impingement syndromes. will often lead to substantial stiffness in the shoulder and does
B. Therapy with prolonged immobilization. not appreciably decrease the redislocation rate. Arthroscopic
C. Open stabilization procedures are the “gold stan- stabilization procedures have been the gold standard treat-
dard” of treatment for majority of injuries associated ment for the majority of injuries related to shoulder disloca-
with shoulder dislocation. tions. Posterior dislocations of the shoulder are rare and could
D. Humeral head fractures common. be missed. This dislocation can occur due to electric shock or
seizures. Radiographs are generally obtained to evaluate for
concomitant body injuries, such as fractures of the humeral

Brunicardi_Ch43_p351-354.indd 352 30/06/22 11:28 AM


353
head. After minor trauma or repetitive injury, patients may
experience pain and discomfort, which can be due to irrita-
tion of the tissues in the subacromial space. In many cases,
such shoulder impingement syndromes are caused by sim-
ple bursitis or tendonitis and the long head of the biceps or
supraspinatus tendon. Occasionally, impingement syndromes
can progress to tears of the supraspinatus tendon, which can
be confirmed by magnetic resonance imaging (MRI) or ultra-
sound. (See Schwartz 11th ed., pp. 1895–1897.)

CHAPTER 43
10. Management of arthritis includes which of the following? Answer: A
A. Injection Measures to treat arthritis include weight loss, rest, physi-
B. Osteoplasty cal therapy, nonsteroidal antiinflammatory drugs (NSAIDs),
C. Arthroclasty bracing, and assistive devices such as cane or walker. Joint
D. Arthrotomy injections are commonly performed into the knee and
shoulder. Common injections include corticosteroids and

Orthopedic Surgery
hyaluronic acid gels. Corticosteroid injections can decrease
inflammation within the joint. These injections are usually
administered in combination with a local anesthetic, such as
lidocaine, in order to provide more immediate relief for both
diagnostic and therapeutic purposes. (See Schwartz 11th ed.,
pp. 1905–1907.)

Brunicardi_Ch43_p351-354.indd 353 30/06/22 11:28 AM


This page intentionally left blank

Brunicardi_Ch43_p351-354.indd 354 30/06/22 11:28 AM


CHAPTER 44
Surgery of the Hand and Wrist

1. The median, radial, and ulnar nerves serve the forearm, Answer: A
wrist, and hand. Which of the following statements about Three principal nerves serve the forearm, wrist, and hand: the
these nerves are TRUE? median, radial, and ulnar nerves. The most critical of these from
A. Median nerve receives fibers from C5–T1 and travels a sensory standpoint is the median nerve. The median nerve
through the carpal tunnel. begins as a terminal branch of the medial and lateral cords of
B. The radial nerve receives fibers from C5–T1 and the brachial plexus. It receives fibers from C5–T1. The ulnar
innervates the intrinsic muscles of the hand. nerve is a terminal branch of the medial cord of the brachial
C. The ulnar nerve receives fibers from C5–T1 and plexus. It receives innervation from C8 and T1 roots. The radial
innervates the extrinsic muscles of the hand. nerve is the larger of two terminal branches of the posterior
D. The ulnar nerve with the median nerve travel through cord of the brachial plexus. It receives fibers from C5–T1 nerve
the carpal tunnel. roots. It innervates all of the extensor muscles of the forearm
and wrist. See also Fig. 44-1. (See Schwartz 11th ed., p. 1930.)

Ulnar
FCR A N
TCL
Radial A Median N

3 4
APL 2 5
EPB 5
S 2 3 4

FPL H U
C
EC

EC
RL
/E
CR
B
EP Q
L ED
EIP/EDC

FIG. 44-1. Cross-section of the wrist at the midcarpal level. The relative geography
of the neurologic and tendinous structures can be seen. The transverse carpal
ligament (TCL) is the roof of the carpal tunnel, passing volar to the median nerve
and long flexor tendons. The TCL is also the floor of the ulnar tunnel, or Guyon’s
canal, passing dorsal to the ulnar artery and nerve. The wrist and digital extensor
tendons are also seen, distal to their compartments on the distal radius and
ulna. Bones: C = capitate; H = hamate; P = pisiform; S = scaphoid. Tendons (flexor
digitorum superficialis is volar to flexor digitorum profundus within the carpal
tunnel): 2 = index finger; 3 = middle finger; 4 = ring finger; 5 = small finger. A =
artery; APL = abductor pollicis longus; ECRB = extensor carpi radialis brevis; ECRL =
extensor carpi radialis longus; ECU = extensor carpi ulnaris; EDC = extensor
digitorum communis; EDQ = extensor digiti quinti; EIP = extensor indices proprius;
EPB = extensor pollicis brevis; EPL = extensor pollicis longus; FCR = flexor carpi
radialis; FPL = flexor pollicis longus; N = nerve.
355

Brunicardi_Ch44_p355-360.indd 355 30/06/22 11:28 AM


356
2. For vascular injuries to the hand requiring tourniquet, Answer: B
the maximum time the tourniquet should be applied to Initial treatment for an actively bleeding wound should be
prevent tissue necrosis is: direct local pressure for no less than 10 continuous minutes.
A. 1 hour. If this is unsuccessful, an upper extremity tourniquet inflated
B. 2 hours. to 100 mmHg above the systolic pressure should be used. One
C. 3 hours. should keep this tourniquet time to <2 hours to avoid tissue
D. 4 hours. necrosis. Once bleeding is controlled well enough to evaluate
the wound, it may be cautiously explored to evaluate for bleed-
ing points. One must be very cautious if attempting to ligate
Chapter 44

these to ensure that adjacent structures such as nerves are not


included in the ligature. (See Schwartz 11th ed., p. 1936.)

3. Anesthetic agents with epinephrine should not be used in: Answer: A


A. The fingertip. A commonly held axiom is that epinephrine is unaccept-
B. The hand. able to be used in the hand. Several recent large series have
C. The wrist. dispelled this myth. Epinephrine should not be used in the
Surgery of the Hand and Wrist

D. The forearm. fingertip and not in concentrations higher than 1:100,000


(ie, what is present in commercially available local anesthetic
with epinephrine). Beyond that, its use is acceptable and may
be useful in an emergency room (ER) where tourniquet con-
trol may not be available. Also, because most ER procedures
are done under pure local anesthesia, many patients will not
tolerate the discomfort of the tourniquet beyond 30 minutes.
Epinephrine will provide hemostasis and also prolong the
effect of the local anesthetic. (See Schwartz 11th ed., p. 1936.)

4. Proper handling of an amputated digit or limb includes Answer: C


which of the following? In preparation for replantation, the amputated part and prox-
A. Place dry in a waterproof bag. imal stump should be appropriately treated. The amputated
B. Immerse in an antiseptic solution. part should be wrapped in moistened gauze and placed in a
C. Prep and wrapped in moistened gauze. sealed plastic bag. This bag should then be placed in an ice
D. Place on dry ice. water bath. Do not use dry ice, and do not allow the part to
contact ice directly; frostbite can occur in the amputated part,
which will decrease its chance of survival after replantation.
Bleeding should be controlled in the proximal stump by as
minimal a means necessary, and the stump should be dressed
with a nonadherent gauze and bulky dressing. (See Schwartz
11th ed., p. 1938.)

5. Contraindications to reimplantation include: Answer: D


A. Multiple digit amputations. After replantation was first reported, replantation was
B. Amputations in children. attempted for nearly all amputations. Over the ensuing
C. Amputation of the thumb. decades, more stringent guidelines have been established
D. Injury to a single digit distal to the proximal inter- regarding what should be replanted. Indications for replan-
phalangeal (PIP) joint. tation include amputations of the thumb, multiple digit
amputations, and amputations in children. Relative contra-
indications to replantation include crush injuries, injuries
to a single digit distal to the PIP joint, and patients who are
unable to tolerate a long surgical procedure. As with all guide-
lines, one should evaluate the particular needs of the injured
patient. (See Schwartz 11th ed., p. 1938.)

Brunicardi_Ch44_p355-360.indd 356 30/06/22 11:28 AM


357
6. Which of the following is incorrect about high-pressure Answer: B
injection injuries? High-pressure devices are commonly used for cleaning and
A. They occur most commonly in the nondominant applications of liquids such as lubricants and paint. Most
hand at the base of the digit. commonly, the inexperienced worker accidentally discharges
B. They should be treated with elevation and antibiotics, the device into his nondominant hand at the base of the
with high rates of recovery. digit. Severity of injury depends on the amount and type of
C. They require exploration within 6 hours of injury. liquid injected; hydrophobic compounds cause greater dam-
D. Half of these injuries are associated with digit loss. age. These injuries are typically quite innocuous to inspec-
tion. They are, however, digit-threatening emergencies. The

Chapter 44
patient should be informed of the severity of the injury, and
exploration is ideally performed within 6 hours of injury. Up
to 50% of such injuries result in loss of the digit, but early rec-
ognition and treatment are associated with increased chance
of digit survival. Early frank discussion with the patient and
initiation of appropriate treatment produce the best results
and medicolegal protection. (See Schwartz 11th ed., p. 1939.)

Surgery of the Hand and Wrist


7. Which of the following statements about carpal tunnel Answer: A
syndrome is FALSE? The most common location of upper extremity nerve com-
A. Carpal tunnel syndrome involved the compression of pression is the median nerve at the carpal tunnel, called car-
radial nerve under the flexor retinaculum. pal tunnel syndrome (CTS). The carpal tunnel is bordered by
B. There is an association with exposure to force and the scaphoid bone radially, the lunate and capitate bones dor-
repetition activities, and force and posture. sally, and the hook of the hamate bone ulnarly (see Fig. 44-1).
C. Syndrome is associated with thenar wasting and loss The transverse carpal ligament, also called the flexor reti-
of thumb strength. naculum, is its superficial border. The flexor pollicis longus
D. Treatment consists of splinting of the wrist at 20° to (FPL), four flexor digitorum superficialis (FDS), and four
be worn during sleep, steroid injection, and carpal flexor digitorum profundus (FDP) tendons pass through the
tunnel release. carpal tunnel along with the median nerve. Of these 10 struc-
tures, the median nerve is relatively superficial and radial to
the other nine. There is strong evidence of a positive asso-
ciation between exposure to a combination of risk factors
(eg, force and repetition, force and posture) and CTS. There
is disagreement among hand surgeons regarding whether
occurrence of CTS in a patient who does repetitive activities
at work represents a work-related injury.
Physical examination should begin with inspection. Look
for evidence of wasting of the thenar muscles. Tinel sign
should be tested over the median nerve from the volar wrist
flexion crease to the proximal palm, although this test has
significant interexaminer variability. Applying pressure over
the carpal tunnel while flexing the wrist has been shown in
one series to have the highest sensitivity when compared to
Phalen and Tinel signs. Strength of the thumb in opposition
should also be tested.
Early treatment of CTS consists of conservative manage-
ment. The patient is given a splint to keep the wrist at 20°
extension worn at nighttime. Many patients can have years
of symptom relief with this management. As a treatment and
diagnostic modality, corticosteroid injection of the carpal
tunnel is often employed. Mixing local anesthetic into the
solution provides the benefit of early symptom relief (corti-
costeroids often take 3–7 days to provide noticeable benefit),
and report of postinjection anesthesia in the median nerve
distribution confirms the injection went into the correct loca-
tion. Multiple authors have shown a strong correlation to relief
of symptoms with corticosteroid injection and good response
to carpal tunnel release. (See Schwartz 11th ed., p. 1944.)

Brunicardi_Ch44_p355-360.indd 357 30/06/22 11:28 AM


358
8. A patient shown to have wasting at the interdigital web Answer: A
spaces, experiences numbness of the ring finger and The ulnar nerve also innervates the dorsal surface of the
exhibits Wartenberg sign on physical examination most small finger and ulnar side of the ring finger, so numbness in
likely is suffering from: these areas can be explained by cubital tunnel syndrome. The
A. Cubital tunnel syndrome. patient may also report weakness in grip due to effects on the
B. Carpal tunnel syndrome. flexor digitorum profundus (FDP) tendons to the ring and
C. Compartment syndrome. small fingers and the intrinsic hand muscles. Patients with
D. Rheumatoid arthritis. advanced disease may complain of inability to fully extend
the ring and small finger interphalangeal (IP) joints.
Chapter 44

Physical examination for cubital tunnel syndrome begins


with inspection. Look for wasting in the hypothenar emi-
nence and the interdigital web spaces. When the hand rests
flat on the table, the small finger may rest in abduction with
respect to the other fingers; this is called Wartenberg sign.
Tinel sign is often present at the cubital tunnel. Elbow flexion
test will often be positive. Grip strength and finger abduction
Surgery of the Hand and Wrist

strength should be compared to the unaffected side. Froment


sign can be tested by placing a sheet of paper between the
thumb and index finger and instructing the patient to hold on
to the paper while the examiner pulls it away without flexing
the finger or thumb (this tests the strength of the adductor
pollicis and first dorsal interosseous muscles). If the patient
must flex the index finger and/ or thumb (FDP-index and
flexor pollicis longus [FPL], both median nerve supplied) to
maintain traction on the paper, this is a positive response.
(See Schwartz 11th ed., p. 1944.)

9. Necrotizing infections: Answer: A


A. Often present with pain out of proportion to findings. Bacteria spread along the fascial layer, resulting in the death
B. Often have discharge present. of soft tissues, which is in part due to the extensive blood ves-
C. Debridement should begin following confirmation sel thrombosis that occurs. An inciting event is not always
by way of radiograph findings. identified. Immunocompromised patients and those who
D. Oral antibiotics should begin immediately. abuse drugs or alcohol are at greater risk, with intravenous
drug users having the highest increased risk. The infection
can by mono- or polymicrobial, with group A β-hemolytic
Streptococcus being the most common pathogen, followed by
α-hemolytic Streptococcus, S. aureus, and anaerobes. Prompt
clinical diagnosis and treatment are the most important fac-
tors for salvaging limbs and saving life. Patients will pres-
ent with pain out of proportion with findings. Appearance
of skin may range from normal to erythematous or maroon
with edema, induration, and blistering. Crepitus may occur if
a gas-forming organism is involved. “Dirty dishwater fluid”
may be encountered as a scant grayish fluid, but often there
is little to no discharge. There may be no appreciable leuko-
cytosis. The infection can progress rapidly and can lead to
septic shock and disseminated intravascular coagulation.
Radiographs may reveal gas formation, but they must not
delay emergent debridement once the diagnosis is suspected.
Intravenous antibiotics should be started immediately to
cover gram-positive, gram-negative, and anaerobic bacteria.
Patients will require multiple debridements, and the spread
of infection is normally wider than expected based on initial
assessment. Necrotizing myositis, or myonecrosis, is usually
caused by Clostridium perfringens due to heavily contami-
nated wounds. Unlike necrotizing fasciitis, muscle is univer-
sally involved and found to be necrotic. Treatment includes
emergent debridement of all necrotic tissue along with empir-
ical intravenous antibiotics. (See Schwartz 11th ed., p. 1949.)

Brunicardi_Ch44_p355-360.indd 358 30/06/22 11:28 AM


359
10. The majority of acute cases of infections flexor tenosyno- Answer: D
vitis (FTS) are due to: FTS is a severe pathophysiologic state causing disruption of
A. Systemic lupus erythematosus. normal flexor tendon function in the hand. A variety of eti-
B. Chronic inflammation as a result of diabetes. ologies are responsible for this process. Most acute cases of
C. Rheumatoid arthritis (RA). FTS are due to purulent infection. FTS also can occur second-
D. Purulent infection after penetrating trauma. ary to chronic inflammation as a result of diabetes, RA, crys-
talline deposition, overuse syndromes, amyloidosis, psoriatic
arthritis, systemic lupus erythematosus, and sarcoidosis. (See
Schwartz 11th ed., p. 1950.)

Chapter 44
11. All of the following are TRUE about felon EXCEPT: Answer: D
A. Abscess of the fingertip that is caused by penetrating A felon is a subcutaneous abscess of the fingertip and is most
trauma. commonly caused by penetrating trauma. S. aureus is the
B. Staphylococcus aureus is the most common infectious most common pathogen. The fingertip contains multiple
agent. septa connecting the distal phalanx to the skin. These septa
C. Incision and drainage should be through a longitudi- are poorly compliant, and presence of an abscess will increase

Surgery of the Hand and Wrist


nal incision. pressure and lead to severe pain and tissue death. Patients will
D. The infection extends to the nail bed, and may require experience erythema, swelling, and tenderness of the volar
removal of the nail. digital pad. Oral antibiotics may resolve the infection if diag-
nosed very early, but incision and drainage is indicated when
fluctuance is identified. A digital block should be performed,
followed by a longitudinal incision over the point of maximal
fluctuance (Fig. 44-2). (See Schwartz 11th ed., p. 1951.)

FIG. 44-2. Felon. A. Lateral view of the digit


showing fluctuance between the skin of the pad
and the underlying distal phalanx bone. B. The
authors prefer to drain felons with a longitudinal
incision (dashed line) directly over the area of
maximal fluctuance. A B

12. All hand infections EXCEPT the following require surgi- Answer: C
cal management. All hand infections other than cellulitis will require surgical
A. Paronychia. management. Clinical examination, particularly noting the
B. Felon. area of greatest tenderness and/or inflammation, is the single
C. Cellulitis. most useful diagnostic tool to localize any purulence requir-
D. Osteomyelitis. ing drainage. Specific recommendations for differentiating
among the possible locations of hand infection are included
in the diagnostic algorithm shown in Fig. 44-3. (See Schwartz
11th ed., pp. 1947–1948.)

Brunicardi_Ch44_p355-360.indd 359 30/06/22 11:28 AM


360

Hand inflammation
Plain X-rays

Nondiagnostic Fracture Foreign body

Localized fluctuance
Ulnar to
No Yes SF MC Hypothenar
Cellulitis Site of fluctuance abscess
Loss of
admit, IV Abx Palm
Chapter 44

palmar
serial exam concavity
Midpalm
No improvement Entire finger Partial finger Distal abscess
in 48 hours
Radial to
MRI if no Kanavel’s Dorsal Centered Between
IF MC Thenar
fluctuance signs present on joint digits
abscess
Pain with
Subcutaneous axial loading
Surgery of the Hand and Wrist

Pyogenic FTS
abscess of joint
Web space
abscess
Pyogenic vs.
crystalline
arthritis

Consider
arthrocentesis

FIG. 44-3. Diagnostic algorithm. Diagnostic workup for a patient with hand inflammation to evaluate for
infection. See text for details about particular infectious diagnoses. Abx = antibiotics; FTS = flexor tenosynovitis;
IF MC = index finger metacarpal; MRI = magnetic resonance imaging; SF MC = small finger metacarpal.

13. Treatment of ganglion cyst consists all of the following Answer: A


EXCEPT: Treatment consists of observation if asymptomatic. If
A. Splinting and pressure. symptoms exist or the patient desires removal for cosmetic
B. Aspiration. appearance, aspiration of the cyst may be performed with a
C. Surgical excision. successful cure rate ranging from 15% to 89%. The benefit of
D. Observation. injected steroids is inconclusive. Aspiration of a volar wrist
ganglion cyst can be dangerous due to the potential of injur-
ing neurovascular structures. Open excision and arthroscopic
excision of the cyst stalk are surgical options for cysts that are
not amendable to aspiration. A recent meta-analysis reported
recurrence rates after either needle aspiration, open excision,
and arthroscopic excision as 59%, 21%, and 6%, respectively.
(See Schwartz 11th ed., p. 1953.)

14. The most common primary malignant tumor of the Answer: C


hand is: Squamous cell carcinoma (SCC) is the most common primary
A. Melanoma. malignant tumor of the hand, accounting for 75% to 90% of
B. Basal cell carcinoma. all malignancies of the hand. Eleven percent of all cutaneous
C. Squamous cell carcinoma (SCC). SCC occur in the hand. It is the most common malignancy
D. Epithelioid sarcoma. of the nail bed. Risk factors include sun exposure, radiation
exposure, chronic ulcers, immunosuppression, xeroderma
pigmentosa, and actinic keratosis. (See Schwartz 11th ed.,
p. 1955.)

Brunicardi_Ch44_p355-360.indd 360 30/06/22 11:28 AM


CHAPTER 45
Plastic and Reconstructive Surgery

1. All of the following are TRUE about split-thickness skin Answer: D


grafts EXCEPT: Many of the characteristics of a split-thickness graft are deter-
A. Degree of contraction dependent on amount of mined by the amount of dermis present. Less dermis trans-
­dermis in graft. lates into less primary contraction (the degree to which a graft
B. High reliability of take. shrinks in surface area after harvesting and before g­ rafting),
C. Healing with abnormal pigmentation more common more secondary contraction (the degree to which a graft
in thin than thick grafts. shrinks during healing), and better chance of graft survival.
D. Meshing grafts improves their ultimate cosmetic Thin split grafts have low primary contraction, high second-
appearance. ary contraction, and high reliability of graft take, often even
in imperfect recipient beds. Thin grafts, however, tend to heal
with abnormal pigmentation and poor durability compared
with thick split grafts and full-thickness grafts. Split grafts
may be meshed to expand the surface area that can be cov-
ered. (See Schwartz 11th ed., pp. 1975–1976.)

2. Which of the following definitions is incorrect? Answer: B


A. Flap composition: description of the tissue compo- The composition of a flap describes its tissue components.
nents within the flap The contiguity of a flap describes its position related to its
B. Flap contiguity: the position of a flap relative to its source. Distant flaps are transferred from a different anatomic
recipient bed region to the defect. They may remain attached to the source
C. Pedicle: bridge of tissue that remains between a flap anatomic region (pedicle flaps) or may be transferred as free
and its source; blood vessels that nourish a flap flaps by microsurgery. These are completely detached from
D. Free flap: flaps that are completely detached from the the body, and their blood supply is reinstated by microvas-
body prior to their preimplantation with microvascu- cular anastomoses to recipient vessels close to the defect. The
lar anastomoses term pedicle was originally used to describe a bridge of tissue
that remained between a flap and its source, similar to how
a peninsula remains attached to its mainland. However, as
knowledge of flap blood supply and (micro)vascular anatomy
has improved over the years, the term pedicle has increasingly
become reserved for describing the blood vessels that nourish
the flap. (See Schwartz 11th ed., pp. 1977–1980.)

3. Phases of wound healing include all of the following Answer: C


EXCEPT: There are different processes that characterize healing in sev-
A. Hemostasis. eral types of tissue, such as skin, muscle, or bone, and there
B. Proliferation. is a strong underlying mechanism that is best understood in
C. Contraction. terms of a simple skin injury. The process of wound heal-
D. Remodeling. ing is comprised of four integrated processes that overlap:
(a) bleeding and hemostasis, (b) inflammation, (c) prolifera-
tion, and (d) tissue modeling or resolution. (See Schwartz
11th ed., p. 1971.)

361

Brunicardi_Ch45_p361-362.indd 361 30/06/22 11:29 AM


362
4. Which of the following is TRUE about vascular Answer: B
anomalies? Hemangiomas are the most common vascular tumor in chil-
A. Hemangiomas affect 5% of premature infants. dren, presenting in up to 20% of premature infants. Oral
B. Oral propranolol is the treatment of choice for high- propranolol therapy has emerged as the first-line treatment
risk infantile hemangiomas. for complicated or high-risk infantile hemangiomas. While
C. Spontaneously involving hemangiomas was result in hemangioma involution may result in no visible sequelae, up
visible sequellae in 10% of patients. to 50% of patients are left with a residual fibrofatty mass with
D. Sturge-Weber syndrome present with capillary mal- antrophic, hypopigmented, and/or telangiectatic overlying
formations involving only superficial cutaneous skin. Sturge-Weber syndrome may be accompanied by vascu-
CHAPTER 45

tissues. lar malformations of the underlying leptomeninges or globe.


(See Schwartz 11th ed., pp. 1996–1997.)
Plastic and Reconstructive Surgery

Brunicardi_Ch45_p361-362.indd 362 30/06/22 11:29 AM


CHAPTER 46
Anesthesia for the Surgical Patient

1. Malignant hyperthermia (MH) can develop after receiv- Answer: B


ing general anesthesia. Which of the following represents MH is a hereditary, life-threatening, hypermetabolic acute
a triggering agent? disorder, developing during or after receiving general anes-
A. Nitrous Oxide thesia. The clinical incidence of MH is about 1:12,000 in
B. Isoflurane children and 1:40,000 in adults. A genetic predisposition and
C. Cisatracurium one or more triggering agents are necessary to evoke MH. Trig-
D. Rocuronium gering agents include all volatile anesthetics (eg, halothane,
enflurane, isoflurane, sevoflurane, and desflurane) and the
depolarizing muscle relaxant succinylcholine. Volatile anes-
thetics and/or succinylcholine can cause a rise in the myoplas-
mic calcium concentration in susceptible patients, resulting
in persistent muscle contraction. (See Schwartz 11th ed.,
p. 2040.)

2. ASA Physical Status III of the American Society of Anes- Answer: D


thesiologists physical status classification system denotes American Society of Anesthesiologists physical status classi-
which of the following classes of patients? fication system:
A. A normal healthy patient
P1 A normal healthy patient
B. A moribund patient who is not expected to survive
P2 A patient with mild systemic disease
without operation
P3 A patient with severe systemic disease
C. A patient with severe systemic disease that is a con-
P4 A patient with severe systemic disease that is a constant
stant threat to life
threat to life
D. A patient with severe systemic disease
P5 A moribund patient who is not expected to survive with-
out the operation
P6 A declared braindead patient whose organs are being
removed for donor purposes
(See Schwartz 11th ed., p. 2034.)

3. A sudden rise in end-tidal CO2 concentration from 11 to Answer: C


37 mm Hg during cardiopulmonary resuscitation most End tidal CO2 (etCO2) concentration monitoring during car-
likely represents which of the following? diopulmonary resuscitation (CPR) is becoming increasingly
A. Adequate chest compressions common. And etCO2 value of ≥10 represents adequate chest
B. Adequate respirations compressions. A sudden rise in etCO2 during CPR most likely
C. Return of spontaneous circulation represents return of spontaneous circulation. (See Schwartz
D. Measurement artifact 11th ed., p. 2033.)

363

Brunicardi_Ch46_p363-364.indd 363 30/06/22 11:29 AM


364
4. A patient presents for elective laparoscopic cholecys- Answer: D
tectomy. The patient reports a history of cardiac disease Patients who are able to perform 4 METs of exercise or more
including a non-ST elevation MI 4 years prior that was can typically proceed to surgery without additional evalua-
treated with a drug-eluting stent. The patient reports tion. The patient presents with a history of stable coronary
being able to climb two flights of stairs. Which of the fol- disease and good exercise tolerance; no further workup
lowing BEST represents workup needed prior to induc- is needed prior to proceeding. (See Schwartz 11th ed.,
tion of anesthesia? pp. 2034–2035.)
A. Electrocardiogram (EKG).
B. Transthoracic echocardiography.
Chapter 46

C. Stress nuclear perfusion imaging.


D. No further workup is needed.

5. A 34-year-old woman presents for elective hernia repair. Answer: A


According to American Society of Anesthesiologists NPO guidelines for preoperative fasting safe intervals after
(ASA) guidelines, after what interval from consumption intake of food and drink when anesthesia can be induced.
of black coffee can anesthesia be induced in this patient? The suggested interval after intake of clear liquids is 2 hours,
Anesthesia for the Surgical Patient

A. 2 hours after breast milk is 4 hours, after a light meal (small amount
B. 4 hours of easily digestible food such as toast or crackers) or infant
C. 6 hours formula is 6 hours, and after more food intake is 8 hours. In
D. 8 hours this patient, anesthesia can be induced 2 hours after intake of
clear liquids. (See Schwartz 11th ed., p. 2036.)

6. In patients who cannot be mask ventilated or intubated Answer: B


via direct laryngoscopy after induction of anesthesia, The American Society of Anesthesiologists (ASA) difficult
which of the following represents the best next step? airway algorithm covers strategies to deal with difficult intu-
A. Videolaryngosocpy bation and difficult mask ventilation scenarios. For patients
B. Laryngeal mask airway insertion in whom anesthesia has been induced, if initial intubation
C. Fiberoptic intubation attempts are unsuccessful and face mask ventilation is inad-
D. Surgical airway equate, supraglottic airway insertion should be considered.
(See Schwartz 11th ed., pp. 2037–2039.)

7. According to ACC/AHA guidelines, how long after bare Answer: A


metal coronary stent placement should elective surgery The ACC/AHA guidelines recommend delaying elective sur-
be delayed? gery for 30 days after bare metal stent placement and for 1 year
A. 30 days after drug-eluting stent placement. Dual antiplatelet therapy
B. 3 months should be continued for urgent or emergent procedures
C. 6 months that take place before the minimum recommended waiting
D. 1 year period. For semi-elective surgeries in patients with drug-
eluting stents, where the risk of delaying surgery is greater
than the risk of in-stent thrombosis, ACC/AHA guidelines
recommend surgery be delayed for 180 days. Observational
data suggest that the risk of in-stent thrombosis is increased
for 180 days after stent implantation, regardless of whether a
bare metal stent or drug eluting stent is used. (See Schwartz
11th ed., p. 2035.)

8. Local anesthetic systemic toxicity can be treated by Answer: D


administration of which of the following? In addition to treating symptomatology, local anesthetic sys-
A. Sugammadex temic toxicity can be treated with intravenous administration
B. Neostigmine of lipid emulsion. (See Schwartz 11th ed., p. 2031.)
C. Flumazenil
D. Lipid emulsion

Brunicardi_Ch46_p363-364.indd 364 30/06/22 11:29 AM


CHAPTER 47
Surgical Considerations in the Elderly

1. Which of the following conditions are considered com- Answer: D


ponents of “geriatric syndromes”? Geriatric syndromes are clinical syndromes that do not fit
A. Frailty into discreet categories but which can negatively impact qual-
B. Urinary incontinence ity of life, result in disability, and contribute to morbidity and
C. Malnutrition mortality in elderly surgical patients. They include frailty,
D. All of the above falls, dementia, malnutrition, and urinary incontinence, and
each should be evaluated carefully preoperatively in surgical
patients. (See Schwartz 11th ed., p. 2045.)

2. Criteria to define frailty include all of the following Answer: D


EXCEPT: Unintentional weight loss of >5% from that of the previous
A. Weight loss. year is considered a criterion of frailty. A standard algorithm
B. Physical activity quantified as kcal/week. is used to calculate physical activity and is corrected for gen-
C. Grip strength. der. Men who expend <383 kcal/wk or women who expend
D. Orientation to time and location. <270 kcal/wk may be frail. Grip strength is stratified by body
mass index and by gender and is also used as a criterion.
Orientation to time and place may or may not indicate dementia
and is not a specific criterion of frailty. (See Schwartz 11th ed.,
p. 2046.)

3. Risks for postoperative delirium include which of the Answer: D


following? In addition to management of fluid and electrolyte imbalance
A. Age > 70 years and optimization of preoperative hematocrit and intraopera-
B. History of alcohol abuse tive blood loss, other risk factors include undertreatment of
C. Intraoperative blood loss pain and a history of antecedent alcohol abuse. (See Schwartz
D. All of the above 11th ed., p. 2047.)

4. The preoperative assessment and counseling should Answer: D


include which of the following? Preoperative assessment and counseling establishes the
A. Confirm patient treatment goals including advance patient’s choices and the identity of a proxy or surrogate deci-
directives sion maker if the patient is unable to assess postoperative
B. Confirm the patient’s health care proxy or surrogate complications. A detailed discussion of the risks and likely
decision maker resumption of full function will inform decisions regarding
C. Discuss the specific risks of the procedure and whether to proceed with the recommended procedure or an
alternative treatments based on patient’s expressed alternative treatment. (See Schwartz 11th ed., p. 2049.)
preferences
D. All of the above

365

Brunicardi_Ch47_p365-368.indd 365 30/06/22 11:29 AM


366
5. Risks for postoperative delirium include which of the Answer: D
following? The American College of Surgeons and the American Geri-
A. Untreated or inadequately controlled pain atrics Society compiled a list of risk factors for postoperative
B. Renal insufficiency delirium that include untreated or undertreated pain; sleep
C. Dehydration deprivation; renal insufficiency; anemia; dehydration; immo-
D. All of the above bilization; and the use of psychotropic medications such as
benzodiazepines, anticholinergics, and antihistamines. (See
Schwartz 11th ed., p. 2050.)
CHAPTER 47

6. Best practice guidelines for the assessment of nutritional Answer: D


status include which of the following? A BMI < 18.5 kg/m2 increases the risk of complications due
A. Document height, weight, and body mass index to nutritional impairment and may indicate a period of pre-
(BMI) operative nutritional support. A serum albumin of <3.0 g/dL
B. Measure baseline serum albumin and prealbumin should also prompt a referral for a detailed nutritional assess-
levels ment. Unintentional weight loss of >5% compared to the
C. Document unintentional weight loss during the previous year and weight loss of >10%–15% in the preceding
Surgical Considerations in the Elderly

preceding year 6 months may also be an indication for preoperative nutri-


D. All of the above tional restoration. (See Schwartz 11th ed., p. 2051.)

7. Factors which adversely affect the cardiac function Answer: D


of older adults postoperatively include which of the Aging has been shown to cause a 1% decrease in cardiac out-
following? put per year, even in the absence of arrhythmias and cardiac
A. Depletion of intravascular volume ischemia. The inability to increase cardiac output during
B. Age-related impairment in the response to stress is dependent on ventricular dilatation and volume sta-
catecholamines tus. Incomplete emptying of the ventricle at end systole and
C. Increased myocardial relaxation time reduced ejection fraction leads to impaired coronary perfu-
D. All of the above sion and ischemia. (See Schwartz 11th ed., p. 2051.)

8. Medications which should be discontinued or avoided Answer: D


in the elderly postoperative patient include which of the Medications with psychotropic effects such as meperidine,
following? diphenhydramine and anticholinergics may contribute to
A. Meperidine (Demerol) postoperative delirium and should be avoided. (See Schwartz
B. Diphenhydramine (Benadryl) 11th ed., p. 2052.)
C. Antihistamines
D. All of the above

9. Which forms of oral intake may be allowed up until 2 Answer: A


hours before general anesthesia? The American Society of Anesthesiologists issued new guide-
A. Clear liquids lines in 2011 that reduced the fasting period prior to gen-
B. Light food and full liquids eral anesthesia. Clear liquids were judged permissible until
C. Fried and fatty foods 2 hours before anesthesia, and light foods and full liquids
D. Cooked meat were permissible until 6 hours before anesthesia. Fried and
fatty foods and meats continue to be restricted for 8 hours
before anesthesia, but other factors which can affect gastric
emptying such as diabetes may require longer periods of pre-
operative fasting. (See Schwartz 11 ed., p. 2053.)

10. Prophylaxis against deep venous thrombosis has been Answer: B


shown to reduce the incidence of postoperative venous Low molecular weight heparin (LMWH) together with lower
thromboembolism. For older adults undergoing nonor- extremity compression devices are recommended. Full-dose
thopedic procedures which of the following regimens is heparin and intracaval filter placement are accompanied by
recommended? unnecessary risks and costs. (See Schwartz 11th ed., p. 2053.)
A. Full-dose heparization
B. Low molecular weight heparin and lower extremity
compression devices
C. Low molecular weight heparin only
D. Intracaval filter placement

Brunicardi_Ch47_p365-368.indd 366 30/06/22 11:29 AM


367
11. Preoperative prehabilitation programs have been Answer: D
shown to reduce complications in elderly compromised The Michigan Surgical Home and Optimization Program has
patients. What are the components of such a prehabilita- included preoperative home-based walking; incentive spi-
tion program? rometry; and education about nutrition, stress management,
A. An at home-based walking program with daily and care planning, The program has been found to reduce
follow up postoperative complications, length of hospital stay and costs.
B. Incentive spirometry training beginning one week (See Schwartz 11 ed., p. 2053.)
prior to surgery
C. Education on stress management

CHAPTER 47
D. All of the above

12. What is the role of surgical palliative care? Answer: C


A. To avoid high-risk operations The role of surgical palliative care is to offer the treatment
B. To minimize intervention in patients at high risk of which will most likely improve symptoms and reduce anxi-
immanent mortality ety. Quality of life issues are paramount, even if for limited
C. To improve symptoms, reduce anxiety, and improve periods of time. The purpose of surgical palliative care is to

Surgical Considerations in the Elderly


quality of life provide the best care for the patient who is compromised by
D. To support patient’s family members and caregivers age, disease, or diminished function. (See Schwartz 11th ed.,
p. 2053.)

13. In frail nursing home residents, what percent of patients Answer: B


are alive 1 year after colon cancer resection? Among frail older adults, functional decline after surgery is
A. 70% substantial and sustained. In one study of frail nursing home
B. 50% patients only 53% were alive 1 year after colon resection for
C. 30% malignancy. The same was true for patients who underwent
D. 10% lower extremity bypass. Hip replacement was associated with
a 1 year mortality rate of more than one-third of patients. (See
Schwartz 11 ed., p. 2054.)

14. Cancer surgery in elderly patients is: Answer: B


A. Always less successful. The frequency of referrals for surgical treatment of equiva-
B. Complicated by the fact that clinical trials usually do lent stage cancer is decreased in the elderly for virtually all
not include elderly subjects. tumors. Despite this, survival after surgery is nearly equiva-
C. Recommended equally as in younger patients. lent for same stage malignancy as in younger patients. The
D. Does not change overall life expectancy. data derived from clinical trials of adjuvant and neoadjuvant
therapy are less helpful for decision making in elderly patients
because elderly subjects are usually not included in clinical
studies. (See Schwartz 11 ed., p. 2055.)

15. Elderly patients with acute peritonitis may not present Answer: A
with typical symptoms of acute abdominal pain, fever, In elderly patients with acute appendicitis or acute cholecys-
or leukocytosis due to an impaired immune response. A titis, one-third lack symptoms of abdominal pain, one-third
high index of suspicion is needed as the initial clinical are afebrile, and one-third have a normal white blood cell
diagnosis in elderly patients with acute appendicitis is count. Therefore an “unimpressive” abdominal examination
correct in what percent of cases? is irrelevant in the evaluation of the elderly patient whose tol-
A. Less than 50% erance for food has suddenly changed. (See Schwartz 11th ed.,
B. 70%–80% p. 2055.)
C. 90%–95%
D. 100%

16. In elderly patients undergoing heart valve replacement, Answer: A


bioprosthetic valves are preferred over synthetic valves Prolonged anticoagulation is more hazardous in the elderly
because: where the risk of falls is increased. Even a fall from the
A. There is less need for anticoagulation which is standing position can result in a fatal intracranial bleed in
hazardous in the elderly. an anticoagulated patient. (See Schwartz 11th ed., p. 2056.)
B. The operative time is shorter which reduces the risk
of pulmonary complications.
C. Synthetic valves have a higher incidence of manufac-
turing defects.
D. The extent of hemolysis is less with bioprosthetic valves.

Brunicardi_Ch47_p365-368.indd 367 30/06/22 11:29 AM


368
17. Transcatheter aortic valve implantation/replacement Answer: C
(TAVI/TAVR) is indicated in which of the following TAVI/TAVR was initially considered as an alternative to
patients? operative replacement of aortic stenosis only in high risk
A. Only those with high surgical risk patients. Recent trials have shown that the risks and outcomes
B. Those with minimal aortic stenosis are comparable in older low- and medium-risk patients as in
C. Elderly patients with aortic valve disease younger patients. (See Schwartz 11th ed., p. 2056.)
D. Patients with congenital bicuspid aortic valve disease

18. What are the advantages of endovascular repair (EVAR) Answer: D


CHAPTER 47

of an abdominal aortic aneurysm (AAA) in elderly Randomized controlled trials of EVAR versus open AAA
patients? repair have shown equal or improved survival, fewer compli-
A. A reduction in blood loss and operative time cations, and shorter length of stay. The procedure can be per-
B. A reduction in the need for postoperative intensive formed under epidural anesthesia in patients who are at high
care risk for general inhalational anesthesia and has comparable or
C. The avoidance of abdominal and pulmonary compli- improved outcomes. (See Schwartz 11th ed., p. 2056.)
cations associated with open AAA repair
Surgical Considerations in the Elderly

D. All of the above

Brunicardi_Ch47_p365-368.indd 368 30/06/22 11:29 AM


CHAPTER 48
Ethics, Palliative Care, and Care at the End of Life

1. Biomedical ethics is a system of analysis and deliberation Answer: D


which is intended to direct physicians and surgeons to The patient and the doctor decide together what treatment is
moral “goodness” in patient care. It includes consider- in the best interest of the patient and share the benefits and
ation of all of the following EXCEPT: the burdens of this joint decision-making. The physician’s
A. Autonomy—the patient’s right to decide for himself/ role is to clarify the indications, risks, and benefits of the pos-
herself what care will be provided. sible treatment courses; the patient’s role is to decide what
B. Beneficence—the concept that proposed treatments course to take. Equipoise suggests that neither the patient nor
will benefit the patient. the doctor has a preference, which is not the case. The fourth
C. Nonmaleficence—the avoidance of treatments which component of biomedical ethics is justice, meaning fairness
may harm the patient. where both the benefits and burdens are distributed equally.
D. Equipoise—the lack of a preference for one treatment (See Schwartz 11th ed., p. 2061.)
over another.

2. What is implied by the “principalist approach” to medi- Answer: B


cal ethics? In an apparent conflict or conundrum, the relevant principles
A. The principal caregiver determines which course will of autonomy, beneficence, nonmaleficence, and justice are
be followed after discussions with the patient. weighed together with the values at stake to determine jointly
B. Identify the principles, such as autonomy and benefi- the best course of action. (See Schwartz 11th ed., p. 2061.)
cence, which appear to be in conflict and evaluate the
conflict taking into consideration of values such as
self-determination and quality of life.
C. The guiding principle of “first do no harm” requires
that risks to the patient be minimized.
D. The fundamental principle is that the discussion of
biomedical ethics needs to involve all members of the
care team as well as the patient.

3. Informed consent contains all of the following elements Answer: C


EXCEPT: Documentation that the patient or the surrogate decision
A. The physician must document that the patient or the maker has the capacity to make a decision is a primary ele-
surrogate decision maker has the capacity to make a ment. To the extent known, the risks of any treatment options
medical decision. as well as the likelihood of functional recovery need to be
B. The surgeon describes the risks and benefits of treat- explained in detail. In elective situations, patients are allowed
ment options sufficiently for the patient to make an time to consult with others before making a decision, but
informed decision. family members are not required to witness the authorization
C. A member of the patient’s family must witness the to proceed with care. (See Schwartz 11th ed., p. 2063.)
consent authorization.
D. The patient authorizes a treatment plan without
undue influence.

369

Brunicardi_Ch48_p369-372.indd 369 30/06/22 11:29 AM


370
4. Living wills are documents which are meant to guide Answer: B
decision-making when: Living wills are intended to guide decisions by physicians,
A. The patient’s family cannot be contacted. family members, and/or surrogate decision makers when the
B. The patient is rendered incompetent or unresponsive patient himself/herself is unable to render an opinion and the
by an illness judged to be terminal. condition or disease is judged to be terminal or “hopeless.”
C. Multiple attempts at resuscitation have failed. (See Schwartz 11th ed., p. 2064.)
D. The patient’s family disagrees with the course of
treatment.
CHAPTER 48

5. “Informed consent” implies all of the following, EXCEPT: Answer: D


A. The patient has been provided with the pertinent The “reasonable person” standard for informed consent has
details of his/her diagnosis, prognosis, and the precedent in a 1972 court case which rejected the notion
options for and risks of treatment. that “simple consent” for treatment was sufficient. The court
B. The information has been provided according to decided that the facts of diagnosis, treatment options, and
what a reasonable person would be expected to risks that a “reasonable person” would want to know should
understand. be included in the discussion to obtain consent for treatment.
Ethics, Palliative Care, and Care at the End of Life

C. The discussion of the options, risks, and possible haz- (See Schwartz 11th ed., pp. 2062–2063.)
ards has been documented.
D. There are witnesses to the discussion who also under-
stand the discussion.

6. The durable power of attorney for health care is an Answer: C


alternative to “living wills” where a patient designates Studies indicate that individual identified as the durable
a specific individual to make decisions if and when the power of attorney for health care makes decisions which are
patient is unable to. All of the following statements about consistent with the patient’s own determination only about
the durable power of attorney for health care are true two-thirds of the time. (See Schwartz 11th ed., p. 2064.)
EXCEPT:
A. The durable power of attorney has the authority to
make decisions regarding the provision or the with-
drawal of care.
B. The durable power of attorney is expected to provide
decisions which are consistent with the patient’s pref-
erences and judgment.
C. Decisions made by durable power of attorneys
are highly accurate in predicting the patient’s own
decision.
D. Respecting the decision of the durable power of attor-
ney is a form of respecting the self-determination of
the incapacitated patient.

7. Surgical palliative care can be defined as “the treatment Answer: A


of suffering and the promotion of quality of life for seri- Surgical interventions are not prohibited if the procedure
ously or terminally ill patients under the care of sur- provides relief of pain or distressing symptoms or otherwise
geons.” All of the following statements about surgical improves the quality of life. Adequate treatment of pain is
palliative care are true EXCEPT: a fundamental pillar of surgical palliative care, and is also
A. Surgical intervention is prohibited in patients who accompanied by psychosocial support for family member and
are receiving palliative care. for members of the care team upon the death of a patient. (See
B. The lessening of pain is a priority for surgical pallia- Schwartz 11th ed., pp. 2066–2067.)
tive care.
C. Psychosocial support for family members is part of
surgical palliative care.
D. Bereavement support for the patient care staff is
included in surgical palliative care.

Brunicardi_Ch48_p369-372.indd 370 30/06/22 11:29 AM


371
8. The distinction between surgical innovation and surgi- Answer: C
cal research can be unclear and requires ethical consid- Modification to an existing device or the development of a
erations. All of the following statements about surgical new device to be used in surgery requires approval of the
innovation are true EXCEPT: Office of Product Evaluation and Quality of the Food and
A. A novel or revised part of an operation that a surgeon Drug Administration. IRB approval alone is not sufficient
makes in order to best treat an individual patient is approval alone to authorize its use. Surgical innovations are
considered surgical innovation. part of managing operative challenges but when they are used
B. An innovation introduced into an operation requires on more than three patients they become surgical research
approval of Institutional Review Board (IRB) if it is and require IRB approval. Informed consent of patients is

CHAPTER 48
used on more than three patients. required. (See Schwartz 11th ed., p. 2073.)
C. An innovative modification of an existing surgical
instrument or device can be used in patients with IRB
approval.
D. When employing an innovative modification to an
operative procedure, explicit informed consent of the
patient is required.

Ethics, Palliative Care, and Care at the End of Life

Brunicardi_Ch48_p369-372.indd 371 30/06/22 11:29 AM


This page intentionally left blank

Brunicardi_Ch48_p369-372.indd 372 30/06/22 11:29 AM


CHAPTER 49
Global Surgery

1. A 55-year-old East African woman presents with a pal- Answer: A


pable mass in her right breast that has been present for Previously thought to be a disease almost exclusive to high-
several years. You are concerned for cancer and know income countries, nearly two-thirds of the 7.6 million cancer
her case fatality rate is significantly higher than a similar deaths worldwide occur in low- and middle-income coun-
patient in North America. Why? tries (LMICs). Mortality from cancer correlates inversely with
A. Patients present with later stages of cancer. a country’s economy for certain treatable cancers, including
B. Patients often decline screening. breast, testicular, and cervical cancer—LMICs have higher
C. Cultural prohibitions preclude surgery as an accept- case fatality rates. Breast cancer case fatality rates illustrate
able treatment option. the great disparity in outcomes between regions. Case fatal-
D. Improper treatment of surgically correctable disease. ity rates in East Africa reach an unacceptable 59% compared
to 19% in the United States. In LMICs, patients have very
limited access to screening. They present for care with much
later stages of cancer. The number and quality of training
programs in surgical oncology are also inversely related to a
country’s income, leaving LMICs with few adequately trained
providers. Collaborative training programs between high-
income countries (HIC) and LMIC centers, as well as tele-
teaching and mobile consultation, may address this shortage
in a relatively low-cost, high-impact way. (See Schwartz 11th
Ed., pp. 2082–2083.)

2. When considering surgical care in low- and middle- Answer: B


income countries (LMICs), “task sharing” is one approach Primary care physicians, nurses, midwives, or advanced
that can provide patients with expanded access to surgical care practitioners (ACPs) provide much of the basic surgi-
care. What does this term refer to? cal and anesthetic care in LMICs. Where regulations allow,
A. Transporting patients from lower to higher resource “task sharing,” or training ACPs to deliver surgery and anes-
settings thesia services previously allowed only under the purview of
B. Training advanced care practitioners to deliver sur- fully trained specialists, can provide expanded access to care.
gery and anesthesia services Non-MD practitioners, known as assistant medical officers
C. Establishing a system that allows select surgical dis- (AMOs) or tecnicos de cirurgia in Mozambique, often have
ease to be treated medically extensive operative experience, including obstetrical care,
D. Sharing training materials and tele-teaching between and are the primary surgical providers in some regions. Task
high-income countries (HIC) and LMIC centers sharing with ACPs also occurs in the United States and other
countries where they fill a need otherwise unmet by specialists
even in major tertiary care centers. However, concerns about
the quality of care, lack of adequate supervision, and the effect
on prestige and professional development for specialists and
ACPs, continue to be topics for debate. (See Schwartz 11th Ed.,
p. 2081.)

373

Brunicardi_Ch49_p373-376.indd 373 30/06/22 11:30 AM


374
3. You are planning to work abroad but a colleague men- Answer: D
tions that “surgery is too expensive” in low- and middle- The World Bank arbitrarily defined U.S. $100 per DALY averted
income countries (LMICs) and you should focus on pro- per day in low-income countries as highly cost-effective.
viding mosquito netting instead. You disagree. Which of Compared to other public health initiatives, developing basic
the following answers could help you support your case? and emergency surgical care at the district level hospital is as
A. Developing surgical care capabilities can be below cost-effective as, or more so, than typical public health programs
the U.S. $100 per disability-adjusted life year (DALY) such as retroviral treatments for HIV/AIDS or immunization
averted per day threshold for cost-effective care. for measles (Fig. 49-1). Male circumcision is one example of
B. Surgical treatment is often primary prevention for a well-documented preventative, minor surgical procedure,
CHAPTER 49

additional disease. capable of reducing the transmission of HIV. The DCP3 adopted
C. Essential surgical procedures rank among the most a working definition of essential surgical conditions as those
cost-effective of all health interventions. that (a) are primarily or extensively treated by surgery; (b) have
D. All of the above. a large health burden; and (c) can be successfully treated by a
surgical procedure that is cost-effective and feasible to promote
globally. Using this definition, the DCP3 identified 44 essen-
tial procedures, most of which can be performed in first-level
Global Surgery

hospitals (Table 49-1). The first-level (district) hospital is the


appropriate platform to provide essential surgical service. These
procedures rank among the most cost-effective of all interven-
tions and include those that treat injuries, obstetric complica-
tions (including fistulas), abdominal emergencies, cataracts,
and congenital anomalies. (See Schwartz 11th Ed., p. 2086.)

Orthopedic surgery trip


Trauma center
Cesarean delivery
Hydrocephalus repair
Trachoma surgery
Cleft lip and palate repair
Hernia repair
Surgical hospital
Cataract surgery
Obstetric hospital

Measles vaccination
Antiviral therapy for HIV
Y

Y
AL

AL

AL

AL
/D

/D

/D
FIG. 49-1. Cost-effectiveness of surgical interventions,
/
$1

00

0
$1

00
$1

compared to two key medical interventions. Note:


$1

DALY = disability-adjusted life year. (Illustration reproduced Range - Cost/DALY (2012 US$)
with permission from Intermountain Healthcare.)

TABLE 49-1 Essential surgical procedures that can be performed in first level (district)
hospitals (DCP3)
Obstetric Complications
Severe postpartum hemorrhage, obstructed labor, prolonged labor, eclampsia, prolapsed cord, fetal distress,
tubal pregnancy, postabortion endometritis/myometritis, postabortion sepsis, intrauterine fetal death
Trauma and Violence
Major limb fracture/injury, joint dislocation, major soft tissue injury, pneumo/hemothorax, ruptured spleen
Acute Surgical Emergencies
Strangulated hernia, intestinal obstruction, intestinal perforation, appendicitis, liver abscess, major wound
infection, osteomyelitis/septic arthritis
Nonacute Surgical Conditions
Congenital hernia, hernia, breast cancer, chronic osteomyelitis, hydrocele, urethral stricture, prostatic
hypertrophy, cataract, eye injury

Brunicardi_Ch49_p373-376.indd 374 30/06/22 11:30 AM


375
4. Global surgery interventions can be prioritized by target- Answer: D
ing diseases that impose the largest burden on a society In 2015, the Lancet Commission on Global Surgery (LCGS)
and have a highly successful surgical outcome. The four reported that maternal mortality was closely related to den-
broad, high-priority surgical areas that contribute most sity of surgical, anesthetic, and obstetrical providers (SAOPs).
to the public health mission in low- and middle-income They showed that maternal mortality throughout the world
countries (LIMCs) include all of the following EXCEPT: appeared to decrease—by 13.1% on average—for every
A. Acute surgical emergencies. 10 unit increase in SAOPs per 100,000 persons, a strong argu-
B. Nonacute surgical conditions. ment for addressing maldistribution of providers related to
C. Obstetrical emergencies. surgical disease. High-priority surgical procedures to improve

CHAPTER 49
D. Organ transplantation. maternal health include cesarean section, hysterectomy for
E. Trauma care. postpartum bleeding and uterine rupture, management of
ectopic pregnancy, and dilatation and curettage. In terms
of nonobstetrical acute surgical emergencies, about 90%
could be addressed by developing the capability to care for
the 10 most common acute surgical conditions in any local
region. Even common nonacute conditions can have sig-

Global Surgery
nificant impact on quality of life and socioeconomic status,
such as cleft lip and cleft palate, hernias, and cataracts. Nearly
2 million lives could be saved each year if LMICs could design
and implement simple trauma care initiatives that reduced
the case fatality rates among seriously injured patients to
equal those in high-income countries (HICs) (Fig. 49-2).
Transplantation, though an area of great interest to people
in poor countries, still eludes most of the developing world,
where cultural and legal prohibitions preclude cadaveric and
nonrelated organ donation. (See Schwartz 11th Ed., p. 2095.)

Kumasi,
80 Ghana Monterrey,
Mexico
Seattle,
60
Fatalities (percent)

Washington
United States
(ISS ≥ 9)

40

20
FIG. 49-2. Case fatality rates for severely injured people,
1998. Note: ISS = injury severity score. Income classifications
are based on status at the time of the study. Ghana is 0
now a lower-middle income country, and Mexico is an Low Middle High
upper-middle-income country. (Illustration reproduced with
Country income
permission from Intermountain Healthcare.)

5. You are developing a novel device to facilitate virtual lap- Answer: D


aroscopic skills training for surgeons in LMICs. Which The pressing need for surgical care at all levels and the short-
of the following will help innovations such as yours be age of fully trained surgeons, anesthesiologists, and support
effective in resource-limited settings? personnel as well as equipment and supplies means that
A. Collaborate with a single, local manufacturer opportunities abound for innovation. Innovation that radi-
B. Utilize qualitatively superior technology to the status cally changes the way we do things and that changes a para-
quo digm of a service or system is called “disruptive”; it abruptly
C. Establish a profit-driven business model changes an older and more expensive system in favor of a less
D. Employ widely accessible technologies expensive, more widely available technology or process. The
ability for disruptive innovations to transform products and
services into affordable realities requires three main factors:
a sophisticated technology that simplifies, a low-cost busi-
ness model, and an economically coherent value network
(Fig. 49-3). Regulations and standards that vary between
countries and locales can facilitate or impede disruptive
change. While disruptions often are not qualitatively superior

Brunicardi_Ch49_p373-376.indd 375 30/06/22 11:30 AM


376
to the status quo, they make the process both less expensive
and more accessible, and through multiple iterations, ulti-
mately improve quality as they cycle through the transforma-
tive process. (See Schwartz 11th Ed., p. 2106.)
CHAPTER 49

Sophisticated
technology that
simplifies

Regulations and
standards that
Global Surgery

facilitate
change
Low-cost, Economically
innovative coherent value
business models network

FIG. 49-3. Elements of disruptive innovation. (Illustration


reproduced with permission from Intermountain Healthcare.)

6. Patients in rural areas and in countries with poor infra- Answer: A


structure often lack access to safe, timely, and affordable Disparities in care and outcomes are multidimensional, and
surgical care. Support for Global Surgery has grown sig- no simple solution exists to improve access to appropriate and
nificantly in recent years due in part to all of the follow- affordable surgical care. Yet, five major forces are reshaping pri-
ing EXCEPT: orities and strategies leading the charge for the globalization of
A. Trained surgeons migrating to areas of need. surgical care:
B. Improved control of acute infectious diseases previ- 1. The epidemiologic transition of diseases from primarily
ously the cause of significant morbidity. infectious to more chronic conditions
C. Technology allowing improved access to health care 2. The mobile nature of the world’s populations, allowing
information and training. people to move freely between more isolated areas of the
D. Recognition of the cost-effectiveness of surgery as world, leading to a more integrated global community
a public health intervention. 3. Ubiquitous information access exponentially enabling
widespread participation in understanding and designing
innovative opportunities for high-quality surgical care
4. A revolution for equity and human rights where the
world’s poor are demanding benefits to surgical care simi-
lar to those found in high-income countries (HICs)
5. Recognition of the cost-effectiveness of surgical care and
its potential to build economies, demonstrating the value
of including surgery in global health strategies
Trained practitioners commonly migrate to economically and
culturally favorable locales, leaving low-resource countries
underserved. Investments in training greater numbers of doc-
tors in these countries, including surgical specialists, have been
only partially successful in meeting demand in poor countries.
Until economic conditions improve or opportunities for profes-
sional development increase, and incentives enticing migration
of health care workers to HICs abate, it is unlikely that the most
skilled practitioners will remain in resource-poor areas beyond
their immediate obligations. (See Schwartz 11th Ed., p. 2078.)

Brunicardi_Ch49_p373-376.indd 376 30/06/22 11:30 AM


CHAPTER 50
Optimizing Perioperative Care:
Enhanced Recovery and Chinese Medicine

1. The use of preoperative evaluation is associated with Answer: B


which of the following? The use of a preoperative evaluation results in identifying
A. Increased preoperative testing patients at elevated respiratory risk, a 55% decrease in preop-
B. Decreased case cancellations erative testing, an 88% reduction in case cancellations, reduc-
C. Decreased time from clinic to surgery tion in day of surgery delays, reduced total length of stay, a
D. Decreased patient satisfaction positive impact on hospital finances with cost reduction, and
lower in-hospital mortality. (See Schwartz 11th ed., p. 2113.)

2. Preoperative immunonutrition supplements provide Answer: C


greater quantities of which of the following as compared Immunonutrition supplements have the addition of arginine
to preoperative standard oral supplements? to improve immunity and tissue repair and omega-3 fatty acids
A. Trace minerals to mediate the inflammatory response. However, the dos-
B. Arachidonic acid ages, duration, and frequency of immunonutrition needed to
C. Arginine improve surgical outcomes are unknown. Both types of preop-
D. Amino acids erative nutritional supplementation have been demonstrated
to be beneficial, but there is no statistically significant differ-
ence in infectious complications, complications, or hospital
length of stay between the two types. (See Schwartz 11th ed.,
p. 2115.)

3. A 65-year-old woman is undergoing an elective low- Answer: D


anterior resection for rectal cancer. Which of the follow- Low glycemic index diets are not part of enhanced recovery
ing would not be recommended to counter the effects of pathways. Preoperative and postoperative fasting states
insulin resistance perioperatively? trigger insulin resistance resulting in a catabolic state with
A. Preoperative carbohydrate supplementation gluconeogenesis and protein breakdown. Preoperative car-
B. Use of a laparoscopic versus open approach bohydrate supplementation and early postoperative feeding
C. Appropriate pain management are both recommended. Hyperglycemia that results from
D. Low glycemic index diet postoperatively postoperative fasting can be corrected with exogenous insu-
lin, and achievement of normoglycemia in the perioperative
period normalizes the main components of metabolism. Both
pain and magnitude of surgery are associated with increased
insulin resistance; thus, adequate pain control and use of a
minimally invasive approach are preferred. When epidural
analgesia is added to preoperative carbohydrate supplementa-
tion, there is even greater improvement in insulin resistance.
(See Schwartz 11th ed., pp. 2116–2117.)

377

Brunicardi_Ch50_p377-380.indd 377 30/06/22 11:30 AM


378
4. Which of the following fluids used for periopera- Answer: A
tive resuscitation has been associated with increased Normal saline can result in hyperchloremia and has been
complications? associated with postoperative morbidity and mortality when
A. Normal saline compared to balanced crystalloids in studies of cardiac and
B. Balanced crystalloids other surgical patients. A 2013 Cochrane review did not show
C. Albumin any difference in outcomes when colloids were compared to
D. Synthetic colloids crystalloids for fluid resuscitation. Thus, balanced crystal-
loids tend to be utilized. (See Schwartz 11th ed., p. 2119.)
CHAPTER 50

5. Perioperative fluid management using goal-directed Answer: C


therapy seeks to achieve which of the following? Goal-directed therapy aims to maintain zero fluid balance
A. Minimal crystalloid use coupled with minimal weight gain or loss. Normovolemia
B. Fixed fluid administration is desired to maintain perfusion without volume overload.
C. Zero fluid balance Goal-directed therapy does not mean that there is minimal
D. Normal cardiac output fluid or fixed fluid administration. For some procedures,
fluid requirements may be more than anticipated and for oth-
Optimizing Perioperative Care: Enhanced Recovery and Chinese Medicine

ers, fluid requirements may be less than anticipated. Goal-


directed therapy aims to avoid the complications associated
with both hypo- and hypervolemia. (See Schwartz 11th ed.,
pp. 2118–2119.)

6. Which of the following is associated with increased post- Answer: A


operative opioid requirements? While opioid administration reduces pain immediately after
A. Intraoperative intravenous opioids administration, they worsen pain scores after they wear off.
B. Intraoperative ketamine infusion In fact, they can increase postoperative opioid requirements.
C. Neuraxial opioid analgesia Alternatively, there are non-opioid adjuncts that can be used
D. Peripheral nerve blocks in multimodal pain regimens to reduce opioid requirements
such as non-steroidal anti-inflammatory drugs (NSAIDs),
gabapentinoids, acetaminophen, local anesthetics, ketamine,
and lidocaine. Neuraxial opioid analgesia can reduce postop-
erative pain scores, and regional blocks can be opioid sparing.
(See Schwartz 11th ed., pp. 2119–2121.)

7. A 59-year-old woman who is planned for a total joint Answer: B


replacement tells her anesthesiologist that she has a his- Strategies for avoiding postoperative nausea and vomiting
tory of severe postoperative nausea and vomiting. Which include the avoidance of general anesthesia, the use of totally
of the following would be associated with a decreased intravenous anesthesia, avoidance of nitrous oxide and vola-
risk of nausea and vomiting? tile agents, minimizing intraoperative and postoperative
A. General anesthesia opioids, and adequate hydration. (See Schwartz 11th ed.,
B. Total intravenous anesthesia p. 2121.)
C. Nitrous oxide and volatile agents
D. Intravenous fentanyl infusion

8. Which of the following strategies should be utilized to Answer: D


reduce the incidence of postoperative ileus after a lapa- Multimodal pain strategies and neuraxial blocks reduce opi-
roscopic colon resection? oid use and therefore minimize nausea and improve early
A. Placement of a nasogastric tube enteral nutrition. Routine use of nasogastric tubes is not rec-
B. Administration of a mu-opioid receptor antagonist ommended to prevent ileus; nasogastric tubes actually delay
C. Maintenance of hypovolemia return of gastrointestinal activity. Administration of a periop-
D. Use of multimodal pain strategies erative mu-opioid receptor antagonist (alvimopan) has been
associated with decreased ileus and length of stay in open
not laparoscopic surgeries. Maintenance of normovolemia is
desired as both fluid overload and dehydration are associated
with negative effects on bowel function. Early resumption
of enteral feeding is recommended. (See Schwartz 11th ed.,
pp. 2121–2122.)

Brunicardi_Ch50_p377-380.indd 378 30/06/22 11:30 AM


379
9. A surgical quality officer is tasked with setting up an Answer: B
enhanced recovery after surgery (ERAS) program. Which The first step in setting up an ERAS program is to identify the
of the following should be her first step? current state and the goal state of care. After that, stakehold-
A. Engagement of stakeholders ers should be identified and engaged; the stakeholders should
B. Identification of current and goal states work together to develop the protocol and identify facilitators
C. Development of standardized order sets and barriers to implementation of the protocol. Ultimately,
D. Determination of facilitators and barriers development of standardized order sets and education of
frontline providers will be necessary to ensure appropriate
implementation. (See Schwartz 11th ed., p. 2124.)

CHAPTER 50
10. Traditional Chinese medicine techniques such as acu- Answer: C
puncture can be used as an adjunct to Western medicine Acupuncture can be used to both prevent and treat post-
in achieving the goals of enhanced recovery. Which of operative nausea and vomiting, and it is associated with
the following is associated with the use of acupuncture? improved gastrointestinal function and decreased ileus. It can
A. Increased intravenous opioid use also be used for perioperative pain management, and there is
B. Decreased anesthetic requirements evidence that it reduces opioid use. There is no evidence to

Optimizing Perioperative Care: Enhanced Recovery and Chinese Medicine


C. Decreased nausea and vomiting suggest that acupuncture has an effect on anesthetic require-
D. Decreased fluid administration ments, fluid administration, or other complications. (See
Schwartz 11th ed., pp. 2124–2128.)

Brunicardi_Ch50_p377-380.indd 379 30/06/22 11:30 AM


This page intentionally left blank

Brunicardi_Ch50_p377-380.indd 380 30/06/22 11:30 AM


CHAPTER 51
Understanding, Evaluating, and
Using Evidence for Surgical Practice

1. Of the study designs listed below, which is generally Answer: A


considered to be the strongest level of evidence? Randomized controlled trials (RCTs) are often considered
A. Systematic review the “gold standard” of evidence; however, RCTs are relatively
B. Cross-sectional study rare in the surgical literature. In the absence of RCTs, a meta-
C. Case-control study analysis will likely provide the highest level of evidence for
D. Animal studies a given subject. Meta-analyses use a standardized method
E. Case series to search for, appraise, and pool published data in order to
increase the overall statistical power compared to individual
studies. New statistical conclusions can then be made using
quantitative methods based on a larger sample size. Similar
to meta-analyses, systematic reviews use standardized meth-
ods to search for and appraise studies; however, they do not
utilize standardized quantitative methods to summarize the
results. Therefore, they are generally considered lower quality
evidence compared to a meta-analysis.
Although the hierarchy of evidence remains largely intact
(Fig. 51-1), many newer frameworks of evidence evaluation
allow for movement of studies up or down based upon study
design and potential unaddressed biases. (See Schwartz
11th ed., pp. 2139–2140.)

RCT

Cohort study

Case control study

Case series

Case reports

Animal research

In-vitro research

Expert experience/opinion
FIG. 51-1. Evidence-based hierarchy.

381

Brunicardi_Ch51_p381-384.indd 381 04/07/22 2:50 PM


382
2. A type II error occurs when: Answer: B
A. The null hypothesis is rejected but is actually true. A type II error is the failure to reject the null hypothesis when
B. The null hypothesis is not rejected when it is actually the null hypothesis is false. This error may also be referred to
false. as a false negative. Alternatively, a type I error occurs when
C. The null hypothesis is not rejected when it is actually the null hypothesis is rejected but is actually true. The type II
false. error is related to the power of the study and may also be
D. The incorrect statistical test was performed. referred to as the significance level, often a value of 0.05. This
error may also be referred to as a false positive. The type of
statistical test is unrelated to type I or II errors. (See Schwartz
CHAPTER 51

11th ed., p. 2147.)

3. Which of the following is a properly constructed patient/ Answer: B


population, intervention, comparison and outcome Effective and efficient use of search engines is enhanced by
(PICO) question? framing the clinical question in a format designed to improve
A. Does [surgery] provide a [good recovery] in [adults] the relevancy of search results. PICO is one such format and
with [acute appendicitis]? stands for:
Understanding, Evaluating, and Using Evidence for Surgical Practice

B. In [patients > 18 years old with uncomplicated


• Patient or population is the specific group of individuals
acute appendicitis] do [antibiotics] compared to
for which the questions is being asked.
[laparoscopic appendectomy] result in fewer [major
• Intervention is the treatment or technique of interest for
complications]?
the defined patient or population. Intervention might be
C. In [patients] do [antibiotics] work better than [surgery]
a procedure, such as “laparoscopic appendectomy” or
for [appendicitis]?
be defined as an exposure of interest, such as “smoking.”
D. In [adult patients] are [antibiotics] superior for treat-
• Comparison is the alternative treatment or technique
ing [acute appendicitis]?
to which you are comparing the intervention. Terms
might include, for example, “open appendectomy” or
“observation.”
• Outcome of interest is the final step of the PICO format.
Examples include “mortality,” “operative time,” and “wound
infection.”
Option B is written in the correct order with a well-defined
population, intervention, comparison, and outcome. The
remaining answers are either out of order, nonspecific, or
missing a component of the PICO question. (See Schwartz
11th ed., p. 2139.)

4. A trauma surgeon at your hospital examines data from Answer: C


motor vehicle collisions and determines that there is an In a case-control study, cohorts are determined by the pres-
association between blood alcohol content and fatalities. ence or absence of a particular outcome of interest, in this
This kind of study is an example of a: case motor vehicle collision fatality. Case-control studies are
A. Randomized controlled trial. considered to be a lower level of evidence for risk factors,
B. Case series. given that they are more susceptible to multiple types of bias
C. Case-control study. than cohort studies. However, well-designed and properly
D. Cross-sectional study. analyzed case-control studies can provide solid evidence, for
instance on risk factors for specific conditions. This is in con-
trast to a cross-sectional study where cohorts are determined
by presence or absence of an exposure. A case series involves
reporting on a group of patients that share specified clini-
cal features, but generally does not include a control group.
(See Schwartz 11th ed., p. 2139.)

Brunicardi_Ch51_p381-384.indd 382 04/07/22 2:50 PM


383
5. Which of the following would increase the external Answer: C
validity of a study? External validity or generalizability refers to how results of a
A. Surgical interventions in the study are performed by study translate to “real-world” situations with greater hetero-
a single surgeon. geneity within the potential target population. Involvement
B. The study has strict inclusion criteria. of multiple institutions across multiple cities would help
C. Involvement of multiple institutions across a number increase the chances that a study could be generalized to the
of cities. population of interest as compared to a study performed at
D. Increasing the sample size. a single institution. In general, external validity is related to
E. Performing the study at a single center. the representativeness of the sample population, environment,

CHAPTER 51
and procedures to the target population at large. Alternatively,
Internal validity describes the degree to which a study’s causal
conclusion is warranted. Factors such as randomization, blind-
ing, completeness of follow-up, equivalence among groups,
and accuracy of analysis affect the internal validity of a study.
(See Schwartz 11th ed., p. 2146.)

Understanding, Evaluating, and Using Evidence for Surgical Practice

Brunicardi_Ch51_p381-384.indd 383 04/07/22 2:50 PM


This page intentionally left blank

Brunicardi_Ch51_p381-384.indd 384 04/07/22 2:50 PM


CHAPTER 52
Ambulatory Surgery

1. Based on the Centers of Medicare and Medicaid Services Answer: B


(CMS), ambulatory surgery centers (ASCs) are: ASCs are independent health care facilities that offer patients
A. Any Center that provides same day medical service the convenience of having surgery performed safely without
with minor procedures. admission to a hospital. According to the Centers for Medi-
B. Any distinct entity that operates exclusively for the care and Medicaid Services (CMS), effective May 18, 2009,
purpose of providing surgical services to patients not ASCs are any distinct entity that operates exclusively for the
requiring hospitalization and which services will not purpose providing surgical services to patients not requiring
exceed 24 hours following admission. hospitalization and which the expected duration of services
C. Any office outside a hospital that provides invasive would not exceed 24 hours following an admission. ASCs
procedures to patients outside hospital setting. should not be confused with office-based surgery practices or
D. Does not need referral from a primary care physician. with other outpatient centers that provide diagnostic services
or primary health care, such as urgent care centers, commu-
nity health centers, mobile diagnostic units, or rural health
clinics. ASCs are distinguished from other health care facili-
ties by (a) there use of a referral system for accepting patients
and (b) their maintenance of a dedicated operating room.
(See Schwartz 11th ed., p. 2153.)

2. The top five procedures performed at ambulatory sur- Answer: A


gery centers (ASCs) are: The top five in procedures they have formed at ASCs are
A. Cataract surgery, esophagogastroduodenoscopy with cataract surgery with intraocular lens, esophagogastroduode-
biopsy, colonoscopy with biopsy, and spine epidural noscopy with biopsy, colonoscopy and biopsy, colonoscopy
injection foraminal. with lesion removal, and spine epidural injection foraminal.
B. Laparoscopic cholecystectomy, inguinal hernia In contrast, the top five procedures performed at the hospi-
repair, epidural injection, diagnostic colonoscopy. tal outpatient departments by volume are: Subcutaneous tis-
C. Laparoscopic cholecystectomy, inguinal hernia repair, sue debridement, esophagogastroduodenoscopy with biopsy,
abdominal wall hernia repair, debridement of skin aspiration/injection of joint, cataract surgery with intra ocu-
ulcers, skin lesion removal. lar lens implant, and colonoscopy and biopsy. (See Schwartz
D. Injection paravertebral facet joint, insertion of tem- 11th ed., p. 2155.)
porary bladder catheter, diagnostic colonoscopy,
abdominal wall hernia repair, inguinal hernia repair.

3. Cost of performing procedures at ambulatory surgery Answer: D


centers (ASCs) are lower than those performed in hospi- In many cases an outpatient procedure performed in an ASC
tals due to which of the following factors? is between 1/2 and 1/3 of their costs as the same procedure
A. Avoiding emergency procedures performed in a hospital. In large part, ASCs affect cost savings
B. Fewer diagnostic tests and fewer medications by eliminating overnight hospitalizations and emergency pro-
C. Less staffing and less expensive instruments cedures. ASCs perform fewer extensive diagnostic tests and
D. All of the above dispense fewer medications. These facilities are not staffed
around the clock and are not encumbered by the need for
expensive and highly specialized equipment as are hospitals.
(See Schwartz 11th ed., p. 2157.)

385

Brunicardi_Ch52_p385-386.indd 385 30/06/22 11:30 AM


386
4. Requirements to open an ambulatory surgery center Answer: D
(ASC) in any state include which of the following? Health care facilities in the United States are highly regu-
A. Accreditation by regulating state bodies lated by federal and state entities. ASCs are included in this
B. Certificate of need oversight. Independent observers evaluate the safety and
C. Accreditation by JCAHO quality of care provided in ASC through three processes:
D. None of the above Medicare certification, state licensure, and voluntary accredi-
tation. To obtain Medicare certification, ASCs must meet the
Medicare certification requirements, known as the conditions
for coverage of these conditions include specifying standards
CHAPTER 52

for administration of anesthesia, quality evaluation, operat-


ing and recovery rooms, medical staff, nursing services, and
other aspects of care. An ASC must have an inspection con-
ducted by a state official or a representative of an organiza-
tion authorized by the government. Each state determines
the specific requirements ASCs must meet for licensure. An
ASC does not have to be certified by Medicare in order to
Ambulatory Surgery

be accredited by JCAHO, but if they will be reimbursed by


the Medicare they should meet their requirements.
In 37 states, any party looking to open an ASC must demon-
strate the need for ASC to exist. State licensure requirements
generally exist for both health care facilities and health care
professionals. (See Schwartz 11th ed., p. 2158.)

5. Unplanned admission to a hospital after a procedure in Answer: B


ambulatory surgery centers (ASCs) occur in 0.5% to 2% Safe use of ASC is based on identification of patients who are
of cases with highest odds in the following EXCEPT: unlikely to require admission to a hospital after their proce-
A. Medicaid patients. dure. Thus far, ASCs do very well in this aspect. Unplanned
B. Patients with private insurance. admissions after ambulatory surgery occurring approxi-
C. Lower median household income. mately 0.5% to 2.0% of cases. In the future, ASCs will be chal-
D. Greater preoperative comorbidity burden. lenged to reduce this unplanned admissions rate even further.
Patients with Medicaid insurance, lower median household
income, and greater preoperative comorbidity burden have
the highest odds of unplanned acute care use. These patients
may benefit from interventions that enhance and streamline
possible to follow-up. Additionally, the potential costs asso-
ciated with postoperative acute care following procedures
performed in ASCs are not insignificant. Patient-specific pre-
dictors of unplanned hospital admissions include age 65 years
or older, anticipated operating time longer than 120 minutes,
cardiac comorbidities, peripheral vascular disease, cerebral
vascular disease, malignancy, positive for human immuno-
deficiency virus (HIV) and regional or general anesthesia
use. The strongest predictor for unplanned inpatient hospital
admission was the individual patient’s own history of previ-
ous hospitalizations, particularly among older adults. African
American and Hispanic individuals also have had a mark-
edly elevated risk of inpatient hospital admission, possible
related to cultural or socioeconomic issues for access to care.
(See Schwartz 11th ed., pp. 2159–2160.)

Brunicardi_Ch52_p385-386.indd 386 30/06/22 11:30 AM


CHAPTER 53
Skills and Simulation

1. Virtual reality (VR) simulation has been used for training Answer: C
in laparoscopy, flexible endoscopy, sinuscopic surgery, VR simulators allow for automated performance measure-
and endovascular interventions. Which of the following ment such as time, instrument motion, and electrosurgery
is an advantage of VR simulator–based training as com- use measurements as well as a tally of the occurrence of
pared to physical laparoscopic video trainers? predefined errors. These measurements are not only auto-
A. Significantly improved operative performance mated but also free of observer bias. Both virtual reality and
B. Ability to perform proficiency-based training physical laparoscopic video trainers (“box” trainers) allow
C. Automated performance measurement for proficiency-based training. Good training results can be
D. Lower upfront costs for development obtained with proficiency-based training within a larger cur-
riculum, regardless of training platform. Although there may
be some advantage with the virtual reality trainer, no signifi-
cant difference has been identified. (See Schwartz 11th ed.,
pp. 2169–2171.)

2. Which of the following is a nontechnical skill that can be Answer: A


taught using simulation training? Nontechnical skills refer to the cognitive knowledge and
A. Situational awareness teamwork-related abilities that must be integrated with psy-
B. Instrument selection chomotor skills and abilities. Examples of nontechnical skills
C. Economy of movement include situational awareness, communication, and team-
D. Respect for tissue work. Examples of technical skills include respect for tissue,
time and motion, instrument handling, and knowledge of
instruments; these skills, for example, can be evaluated using
a global ratings scale such as in the objective structured assess-
ment of technical skills (OSATS). (See Schwartz 11th ed.,
pp. 2166–2167, 2175–2177.)

3. Which of the following strategies for simulation-based Answer: A


error training allows the instructor to engage in a broad The error-enabled approach is where the learner can make
assessment of learning needs? any of a variety of errors during the course of a task. This
A. Error-enabled approach allows for a broad assessment of learning needs. The forced-
B. Forced-error approach error approach is where the learner experiences a specific,
C. Error-centric skills assessments usually unexpected error and demonstrates error manage-
D. Graduated autonomy approach ment as well as options for correction. This approach allows
instructors to engage in specific error management skills and
metrics. Error-centric assessments allow for evaluation and
categorization of errors; these types of assessments allow for
focused error training and feedback. While gradual increases
in patient autonomy and autonomous decision-making are
an important component of resident training, this is not
a specific simulation-based strategy. (See Schwartz 11th ed.,
pp. 2177–2178.)

387

Brunicardi_Ch53_p387-388.indd 387 30/06/22 11:31 AM


388
4. Proficiency-based simulation training on central venous Answer: C
catheter insertion has been successful in improving Proficiency-based simulation training on central venous cathe-
which of the following outcomes? ter insertion is one of the best examples of evidence for improved
A. Iatrogenic pneumothoraces patient outcomes with simulation. Multiple studies have
B. Misplaced central venous catheters demonstrated reductions in the incidence of catheter-related
C. Catheter-related bloodstream infections bloodstream infections. (See Schwartz 11th ed., pp. 2178–2179.)
D. Failure of catheter insertion

5. A surgical instructor wishes to evaluate the effects of a Answer: C


CHAPTER 53

simulation-based training program to reduce bile duct While ideally, simulation training should lead to improved
injuries during laparoscopic cholecystectomy. Which of patient outcomes, the incidence of bile duct injury with lapa-
the following would be the most feasible, relevant learner roscopic cholecystectomy is uncommon. Large, unrealistic
metric in assessing the success of this training? sample sizes would be necessary to demonstrate a clinically
A. Perceptions about the training relevant difference. In evaluating the effectiveness of educa-
B. Knowledge about bile duct injuries tional intervention efforts, the Kirkpatrick four-level scale is
C. Skills transfer to the clinical environment often used. The four levels are: Reaction, learning, behavioral
Skills and Simulation

D. Institutional incidence of bile duct injuries change, and institutional impact. Since institutional impact
cannot be assessed in this example, the next highest level
would be behavioral change—did the knowledge, skills, and
attitudes acquired during simulation transfer to the clinical
environment? (See Schwartz 11th ed., p. 2178.)

Brunicardi_Ch53_p387-388.indd 388 30/06/22 11:31 AM


CHAPTER 54
Web-Based Education and
Implications of Social Media

1. Which of the following terms describes computer- Answer: B


mediated platforms that are used for creating and shar- Social media is a term describing websites and web-based
ing information, ideas, and other content? applications that enable users to share ideas, information, and
A. Websites content through virtual networking. Commonly used social
B. Social media media platforms include Facebook, Twitter, and YouTube.
C. Simulators (See Schwartz 11th ed., pp. 2187–2189.)
D. Applications

2. Which of the following is not a barrier to the more wide- Answer: D


spread adoption of web-based educational strategies? Web-based education and coaching allow for asynchronous
A. Lack of data for superior effectiveness learning whereby students and instructors do not have to
B. Up-front costs for development be on the same schedule. Additionally, web-based educa-
C. Surgeon uptake of the technology tion allows for access to educational materials regardless of
D. Ability to have asynchronous learning time of day or night. However, barriers include lack of high-
quality studies demonstrating that web-based education is
superior to traditional educational interventions. Addition-
ally, the high up-front costs for development and need for
technical expertise are also barriers. Lastly, lack of surgeons’
uptake of the technology is a barrier. (See Schwartz 11th ed.,
pp. 2187–2188.)

3. Which of the following is not a social media strategy Answer: C


for disseminating information about peer-reviewed Conference live-tweeting describes the posting of social
publications? media posts on Twitter about an event while it is ongoing.
A. Visual abstracts Usually, peer review and publication lag behind confer-
B. Online journal clubs ence proceedings. Visual abstracts are graphical summaries
C. Conference live-tweeting of the main results of journal articles. They may be created
D. Journal postings by the author or the journal, and they typically accompany
social media posts referencing the peer-reviewed publication.
Online journal clubs are similar to in-person journal clubs
that discuss peer-reviewed publications. Social media-based
journal clubs, however, allow asynchronous discussions
across multiple time zones and geographic locations. Journals
often post links to the abstract and/or peer-reviewed publica-
tion on social media in order to draw attention to the article.
These posts can be accompanied by a visual abstract. (See
Schwartz 11th ed., pp. 2190–2192.)

389

Brunicardi_Ch54_p389-390.indd 389 30/06/22 11:31 AM


390
4. Which of the following increases the quality and accu- Answer: B
racy of web-based educational materials for patients? One of the potential pitfalls associated with web-based edu-
A. Crowdsourcing cation is inaccuracy in information in web-based educa-
B. Peer review tional materials. High-quality online resources should have
C. Physician author peer-reviewed content or verifiable mechanisms for quality
D. Number of viewers control of information. Crowdsourcing describes the process
whereby information is gathered by enlisting the assistance of
a large number of participants. Crowdsourcing often involves
the general public or a loosely defined group of individuals.
CHAPTER 54

Physician authorship is not necessarily associated with


increased accuracy of the online content. Number of view-
ers of the web-based content does not guarantee quality or
accuracy. (See Schwartz 11th ed., pp. 2190–2193.)

5. Which of the following uses of social media does not Answer: C


require patient consent? Another potential pitfall of web-based education is ethical
Web-Based Education and Implications of Social Media

A. Case report of a patient with an unusual complication considerations surrounding patient confidentiality and pri-
B. Email between physician and established patient vacy. Appropriate safeguards must be taken to guard against
C. Online journal club about a published randomized violation of patient confidentiality and privacy. Although
trial patients must consent to participate in a randomized trial,
D. Posting of a photo that does not have the patient’s discussion of an already published and peer-reviewed ran-
face domized trial does not require additional patient consent.
Case reports, whether published in a journal or posted on
social media, require patient consent. Emails or other elec-
tronic communications should only be used by physicians in
an established patient–physician relationship and with patient
consent. Photos should not be posted that have any details that
would result in the patient being identifiable. (See Schwartz
11th ed., pp. 2190–2193.)

Brunicardi_Ch54_p389-390.indd 390 30/06/22 11:31 AM


INDEX

Note: Page numbers followed by italic f or t denote figures or tables, respectively.


A Acrochordons, 136 African Americans, 5
A-A index, 55 ACTH. See Adrenocorticotropin AG. See Anion gap
AAA. See Abdominal aortic hormone AI. See Aortic insufficiency
aneurysms Actinic keratoses, 136 AIHA. See Autoimmune hemolytic
AAH. See Atypical adenomatous Activated protein C, 22 anemia
hyperplasia Acupuncture, 379 Air enema, 335
ABCDE initialism, for melanoma, 130 Acute cholecystitis, 279–280, 283f, Air leak, 59
Abdomen, blunt trauma to, 60, 60f 283–284 Airway pressure, 111
Abdominal aortic aneurysms, Acute hypercalcemia, 18 Airways
191–192, 368 Acute limb ischemia, 194 interventions for, 57–58
Abdominal compartment syndrome, Acute liver failure, 269 mucus production in, 147
69, 103, 112–113 Acute lung injury AJCC. See American Joint
Abdominal hemorrhage, 50 description of, 100 Committee on Cancer
Abdominal hernias, 270 transfusion-related, 25–27 Albumin, 268
Abdominal rectopexy, 250 Acute myeloid leukemia, 303 anion gap affected by, 16
Abdominal wall Acute myocardial infarction, 33 fluid resuscitation uses of, 14
anterior, 307 Acute pancreatitis, 42, 290 serum calcium and, 15
desmoid tumors of, 308 Acute prostatitis, 338 Aldosterone, 17
malignancy of, 307 Acute renal failure, 18 ALF. See Acute liver failure
Abdominal wall hernia, 307 Acute respiratory distress syndrome, ALH. See Atypical lobular hyperplasia
ABI. See Ankle brachial index 32, 36, 100–101, 101t ALI. See Acute lung injury
Abscess ADAM S13, 24 Alkaline chemical burns, 134
anorectal, 253 Adenocarcinoma Alkalosis
breast, 140 in Barrett mucosa, 214f–215f acidosis versus, 17
cellulitis without, 129 duodenal, 238 metabolic, 17
hepatic, 41–42 gallbladder, 280 potassium change associated
lung, surgical drainage of, 152, 152t lung, 148–149, 150f with, 15
perianal, 253 metastatic Barrett esophagus and, Allen test, 107
pyogenic, 41, 272, 272f 208–209 Allergy, antibiotics, 45
retroperitoneal, 309 rectal, 256 Allografts, kidney, 91–92, 92f
subphrenic, 51 Adenomas, liver, 275, 275f Allotransplant, 87
Abstracts, visual, 389 Adenomatous polyposis coli, 154 Alpha angle, 28
Abundance mentality model, of Adenosine diphosphate, 21 Alvimopan, 101, 378
conflict resolution, 4 ADH. See Antidiuretic hormone; Ambulatory surgery, 385–386
Accessory spleen, 300 Atypical ductal hyperplasia Ambulatory surgery centers, 385–386
Accommodation response, 241 Adjustable gastric banding, 233 Amelanotic lesions, 130
ACDF. See Anterior cervical ADP. See Adenosine diphosphate American Joint Committee
discectomy and fusion Adrenal glands, 8 on Cancer
Acetaminophen, 269 Adrenal incidentaloma, 328 description of, 248
Achalasia, 212, 221 Adrenal insufficiency, 8–9 sarcoma staging, 312
Acidic chemical burns, 134 Adrenal mass, 327 American Society of
Acidosis Adrenalectomy, laparoscopic, 328 Anesthesiologists physical
alkalosis versus, 17 Adrenocorticotropin hormone, 8 status classification
metabolic. See Metabolic Adventitial cystic disease, of popliteal system, 363
acidosis artery, 196 Amino acids
Acral lentiginous melanoma, Aerobic bacteria, 41 branched-chain, 9
130–131 AF. See Atrial fibrillation gastrin secretion inhibition by, 223

Brunicardi_Index_p391-420.indd 391 30/06/22 11:32 AM


392
Amino acids (Cont.): Ankylosing spondylitis, 246 Aortic aneurysms
questions regarding, 8 Anomalous coronary artery abdominal, 191–192, 368
in wound healing, 79 patterns, 170 ascending, 183–184
INDEX

Aminoacyl-tRNA synthetases, 120 Anorectal abscess, 253 distal, 183


AML. See Acute myeloid leukemia Anorectal lymphatic drainage, 241 false, 181
AMOs. See Assistant medical officers Anorectal melanoma, 250 thoracic, 181
Amoxicillin/clavulanate, for human Anosmia, 347 true, 181
bites, 134 Anterior abdominal wall, 307 Aortic arch
Amsterdam I criteria, 247–248 Anterior cerebral artery, 112 aneurysms of, 185
Amsterdam II criteria, 247–248 Anterior cervical discectomy and bovine, 182
Anal canal, 241 fusion, 349, 350f Aortic arch receptor, 30
Anastomoses Anterior mediastinal mass, 155 Aortic dissection, 184, 185f, 186–187
arterial, 92f Anterior mediastinal tumors, 155 Aortic insufficiency, 175
Damus-Kaye-Stansel, 160 Anthrax Aortic stenosis
end-to-end, 244, 245f inhalational, 44 age-related, 175
end-to-side, 244, 245f treatment of, 44 congenital, 163
intestinal, 244–245 Antibiotic prophylaxis Aortic valve
questions regarding, 244–245 for colectomy, 246 replacement of, 175
side-to-side, 244, 245f for human bites, 133–135 stenosis of, 158–159
venous, 92f, 93 for inguinal hernias, 316 Aortoiliac occlusive disease, 193–194
wound healing, 75 for overwhelming postsplenectomy APC. See Activated protein C;
Anastomotic pseudoaneurysms, 183 infection, 306 Adenomatous polyposis coli
Anemia for surgical infections, 38–39, 45, 102 APCs. See Antigen-presenting cells
autoimmune hemolytic, 298–300 Antibiotics Apocrine sweat glands, 133
wound healing and, 78 allergy to, 45 Apoptosis
Anesthesia diverticulitis treated with, 247 of cancer cells, 84, 122
American Society of lymphedema treated with, 206 pathways of, 122f
Anesthesiologists physical methicillin-resistant Staphylococcus Appendectomy, laparoscopic, 259–261
status classification system, 363 aureus treated with, 129 Appendiceal carcinoid tumors, 260
epidural, 101 perioperative treatment with, 45 Appendicitis, 259–262, 260t, 262f, 367
fasting before, 364, 366 postappendectomy, 261 Appendix, 259–262
general, 245, 363, 366 primary microbial peritonitis ARDS. See Acute respiratory distress
malignant hyperthermia after, 363 treated with, 41 syndrome
regional, 245 pyogenic liver abscess treated Arginine, 79
saddle, 350 with, 272 Arginine vasopressin, 33
Aneurysms uncomplicated diverticulitis Arrhythmia
aortic. See Aortic aneurysms treated with, 247 in laparoscopy, 115
aortic arch, 185 Anticoagulation, in elderly, 367 questions regarding, 176
ascending thoracic, 184 Antidiuretic hormone Arterial anastomosis, 92f
endovascular repair of, 184, 192 as mesenteric vasoconstrictor, 31 Arterial disease
left ventricular, 176 sodium depletion caused by computed tomography
mycotic, 182–183 secretion of, 13 angiography of, 189
splenic, 304 Antigen-presenting cells, 87 vascular stents for, 189–190
splenic artery, 304 Antigens, 125 Arterial lines, 107–108
Angina pectoris, 171, 171t Antireflux, 214, 216–217, 217f Arterial pH, 17–18
Angiography Antithrombin, 205 Arterial switch operation, 161
computed tomography. See Antithrombin III, 22 Arthritis
Computed tomography Antithrombotic therapy, for deep inflammatory bowel disease
angiography vein thrombosis, 202 and, 246
hepatic hemorrhage controlled Antrectomy, 227f–228f, 227–228, 231 management of, 353
using, 61 Anus ASA physical status classification
indications for, 50 imperforate, 336 system. See American Society
Anion gap questions regarding, 241–257 of Anesthesiologists physical
equation for calculating, 14 Aorta status classification system
metabolic acidosis with, 13–14, 17 chemoreceptors in, 30 Asbestos exposure, 156
normal, 13–14, 16 coarctation of the, 159, 163–164 Ascending aortic aneurysms, 183–184
Ankle brachial index, 352 diameter of, 183 Ascending aortic dissection, 187

Brunicardi_Index_p391-420.indd 392 30/06/22 11:32 AM


393
Ascending thoracic aneurysms, 184 Battle’s sign, 346 Biosynthetic heart valves, 367
ASCs. See Ambulatory surgery centers Baxter formula, 68 Bipolar electrosurgery, 116
ASD. See Atrial septal defects B-cell–mediated rejection, 95 Birbeck granules, 133

INDEX
Aspergillosis, 153 BE. See Barrett esophagus bi-shRNA. See Bifunctional short
Aspergillus sp., 153 Beck’s triad, 49, 55 hairpin
Aspiration pneumonia, 153 Beclin I gene, 84 Bites, human, 133–134
Aspirin, 23 BEE. See Basal energy expenditure Bladder cancer, 337
Asplenia, 298 Belsey fundoplication, 221 Bladder pressure, 112t
Assistant medical officers, 373 Benign calcifications, Blastocysts, 124
Astrocytoma, 348 in hamartomas, 148 Blastomyces dermatitidis, 90
Atherosclerotic aortoiliac occlusive Benign prostatic hyperplasia, 339 Blood loss, 26, 32–33
disease, 193–194 Betel nut chewing, 144 Blood pressure
AT-III. See Antithrombin III Bevacizumab, 149 intra-arterial monitoring
Atresia bFGF. See Basic fibroblast growth system for, 106
biliary, 333 factor pregnancy-related changes, 62
esophageal, 330, 331f Bicarbonate Blood transfusion
Atrial fibrillation, 174, 176 chemoreceptor sensitivity to complications of, 26
Atrial septal defects, 157f, 157–158 levels of, 30 description of, 25
Atrophic gastritis, 229 metabolic acidosis with anion gap diseases not transmitted by, 27
Atypical adenomatous hyperplasia, 148 caused by loss of, 13, 17 indications for, 25–26
Atypical ductal hyperplasia, 137 Bidirectional Glenn procedure, Blood urea nitrogen, 18
Atypical lobular hyperplasia, 137 160, 160f “Blooming artifact,” 189
AUG, 120 Bifunctional RNAi technology, 125 Blotting, 124
Authoritative leader, 4 Bifunctional short hairpin, 125 Blunt trauma
Autoimmune hemolytic anemia, Bile acids to abdomen, 60, 60f
298–300 description of, 236, 268 duodenal hematoma from, 53
Automobile accidents, 50, 59 reabsorption of, 283 to liver, 50
Autophagy, 9 Bile ducts to pancreas, 53
Autotransplant, 87 cancer/carcinoma of, 246, 276 transcriptomic response to, 10
Axillary lymph nodes, 137 common, 279 BMI. See Body mass index
Axillary vein, 198 stricture of, 282 Body mass index, 233
Axonotmesis, 77 Bile leak, 102 Bone healing, 77
AZA. See Azathioprine Bile salts, 268 Bone marrow suppression, 88
Azathioprine, 88, 89t Biliary atresia, 333 Bottleneck, 5
Biliary cancer, 281 Bougie, 234
B Biliary injury, 282 Bovine aortic arch, 182
Back pain, 352 Biliary scintigraphy, 283 Bowel preparation, 246
Balloon valvotomy, 158–159 Biliary stricture, 282 BPH. See Benign prostatic hyperplasia
Balloon-expandable stents, 189 Bilious emesis, 330 Brachiocephalic veins, 198
Bare coronary stent placement, 364 Billroth I gastroduodenostomy, Bradycardia, 115
Bariatric surgery 227, 227f Brain
gastric stasis after, 224 Billroth II gastroduodenostomy, herniation of, 345, 345f
questions regarding, 233–234 227, 228f malignant tumors of, 348
Barium esophagogram Biloma, after laparoscopic traumatic injury of. See Traumatic
of H-type tracheoesophageal cholecystectomy, 102 brain injury
fistula, 331f Biologic mesh, for abdominal wall Brain stem compression, 346
of vascular rings, 168f hernia repair, 307 Branched-chain amino acids, 9
Barium swallow, for eosinophilic Biomarkers, cancer, 85 BRCA1, 140
esophagitis, 220 Biomedical ethics, 369 BRCA2, 140
Baroreceptors, 30 Biopsy Breast
Barotrauma, 112 fine needle aspiration, of thyroid abscess of, 140
Barrett esophagus, 208–209 nodules, 324 anatomy of, 139
Barrett mucosa, 214f–215f primitive neuroectodermal tumor blood supply to, 139
Basal cell carcinoma, 129, 135 diagnosis, 154 Cooper suspension
Basal energy expenditure, 11, 11t sentinel lymph node ligaments of, 139
Base deficit, 36 for breast cancer, 139 hormonal effects on, 139
Basic fibroblast growth factor, 124 for melanoma, 132 infection of, 140

Brunicardi_Index_p391-420.indd 393 30/06/22 11:32 AM


394
Breast (Cont.): Burn disasters, 71 tumorigenic transformation of
pregnancy-related changes in, 137 Burn wounds cells in, 83
sarcoma of, 312 excision of, 70 vaginal, 343
INDEX

Breast cancer silver sulfadiazine for, neutropenia Cancer cells


BRCA1/BRCA2, 140 caused by, 68–69 apoptosis of, 84
distal carcinoma in situ, surgical treatment of, 70 metastasis by, 84
137–138 transforming growth factor-ß
estrogen exposure and, 137 C resistance by, 123
lobular carcinoma in situ, 137 CA-125, 342 tumorigenic transformation of, 83
lymph node metastasis of, 139 Calcifications, in hamartomas, 148 Cancer genes, 124
mastectomy for, 140–141 Calcineurin inhibitors, 88, 89t Candida albicans, 41
in pregnancy, 139 Calcium Capnography, 111
radiation therapy for, 140 hyperkalemia treated with, 15 Caput medusae, 270
risk factors for, 137, 140 serum levels of, 15 Carbohydrate
sentinel lymph node Calcium chloride, 15 digestion of, 236
biopsy for, 139 Calcium gluconate, 15 preoperative supplementation
staging of, 137 Callus formation, 77 of, 377
treatment of, 140–141 Canadian Cardiovascular Society Carbon monoxide diffusion
Breast lesions, 137 angina classification, capacity, 152
Bronchioles, 147 171, 171t Carbonic acid, 17
Bronchoscopy, 59, 167f Cancer. See also Carcinoma; Carcinoid tumors, 230, 260
Brush border membrane, 236 Tumor(s) Carcinoma. See also Cancer; Tumor(s)
BUN. See Blood urea nitrogen biomarkers of, 85 basal cell, 129, 135
Burch procedure, 343 bladder, 337 bile duct, 246
Burn(s) breast. See Breast cancer hepatocellular, 276–277
caloric needs in, 69 chemoprevention of, 84 neuroendocrine, 148
caustic, 134 chemotherapy agents for, 84 renal cell, 337, 339
chemical, 134 in chronic wounds, 80 squamous cell. See Squamous cell
complications of, 70 colorectal, 123, 247–249 carcinoma
cyanide toxicity from, 66 conservative resections in, 85 vulvar, 342
depth classification of, 67 dormancy of, 84 Cardiac myxomas, 179
electrical, 67 epithelial ovarian, 342 Cardiac output, 108, 366
eschar, 69 esophageal, 210, 221 Cardiac tamponade, 35, 49
escharotomies for, 68 gallbladder, 276, 280 Cardiac transplantation, 95
first-degree, 66, 68 gastric, 229 Cardiac tumors, 179–180
fluid resuscitation for, 68 hallmarks of, 83 Cardiogenic shock, 29, 34,
full-thickness, 67–68 head and neck, 143 55–56, 56f
heterotopic ossification hereditary, 83 Cardiopulmonary bypass
associated with, 70 hereditary nonpolyposis colorectal coagulation during, 25
initial evaluation of, 65 cancer, 247–248 Cox-Maze IV procedure,
Lactated Ringer solution for, 68 immune-based therapies for, 85 177, 177f
Lund and Browder chart for, 66 invasion, 84 deleterious effects of, 173
massive resuscitation of, 69 laryngeal, 144 Cardiopulmonary resuscitation, 363
nutritional requirements, 69 lip, 143 Cardiovascular disease, renal
partial-thickness, 67–68 in low- and middle-income failure and, 91
resuscitation of, 69 countries, 373 Caroli disease, 285
“rule of nines” for, 65, 66f lung. See Lung cancer Carotid bodies, 30
second-degree, 67 management of, 85 Carotid body tumors, 191
smoke inhalation injuries, 66 metastasis of, 84–85 Carotid endarterectomy, 190–191
superficial, 66 ovarian, 342 Carpal tunnel syndrome, 357
third-degree, 67 pancreatic, 123 Cas9, 125
thoracic compartment syndrome papillary thyroid, 281, 324 Case fatality rates, 375f
caused by, 68 prostate, 338 Case reports, 390
total body surface area rectal. See Rectal cancer Case series, 382
calculations, 65–66, 66f in situ, 84 Case-control study, 382
zone of coagulation for, 67, 129 splenectomy and, 299 Caspases, 84
zone of hyperemia, 67, 129 testicular, 337, 339 Catecholamines, for septic shock, 33

Brunicardi_Index_p391-420.indd 394 30/06/22 11:32 AM


395
Catheter(s) Chemotherapy Closed brain injury, 347
indwelling intravascular, 43, 46–47 adjuvant, for colorectal cancer, Closed loop obstruction, 236
pulmonary artery, 109, 109f 248–249 Clostridium difficile, 259

INDEX
thrombosis and, 107 agents used in, 84 Clostridium perfringens, 80
Catheter-based contrast hairy cell leukemia treated Clot
aortography, 184 with, 302 formation of, 21
Catheter-directed thrombolysis, 201 hepatocellular carcinoma treated propagation of, 22
Catheter-related bloodstream with, 277 Clotting factors, 268
infection, 108 Chenodeoxycholic acid, 268 congenital deficiency of, 22
Cauda equina syndrome, 350 Chernobyl disaster, 324 in intrinsic and extrinsic
Cause of death Chest injury, penetrating, 53 pathways, 21
medical errors as, 97 Chest wall tumors, 154 CML. See Chronic myelogenous
in pediatric patients, 333 Children. See Pediatric patients leukemia
Caustic injury, 134 Child-Turcotte-Pugh score, 274, 274t CMS. See Centers of Medicare and
CCK. See Cholecystokinin Chin lift, 58 Medicaid Services
CD117, 249 Chinese medicine, traditional, 379 CMV. See Cytomegalovirus
CD4+ T cells Chlamydia trachomatis, 343 COA. See Coarctation of the aorta
cytokines produced by, 9 Chloroquine, 84 Coaching, 3
helper T cells, 10 Cholangiocarcinoma, 276, 281, Coagulation
CD8+ T cells, 9 286–287 in cardiopulmonary bypass, 25
CDC. See Centers for Disease Cholecystectomy extrinsic pathway of, 21
Control and Prevention indications for, 286 intrinsic pathway of, 21
CDH. See Congenital diaphragmatic laparoscopic Coagulation cascade, 22
hernia acute cholecystitis treated with, 284 Coagulopathy, trauma-induced, 23
Cecal volvulus, 250–251 anesthesia for, 364 Coarctation of the aorta, 159, 163–164
Cell(s) biloma after, 102 Codon, 120
apoptosis of, 84, 122 safety concerns for, 117 Coercive style, of leadership, 4
physiologic alterations of, Cholecystitis, acute, 279–280, 283f, Colectomy
in cancer, 83 283–284 bowel preparation for, 246
tumorigenic transformation of, 83 Cholecystokinin, 279 left, 244
Cell cycle, 83, 121f, 121–122 Choledochal cysts, 281, 285, 286f, 335 right, 243
Cell-free mitochondrial DNA, 8 Cholestasis, 103 sigmoid, 244, 250
Cell-surface receptors, 122–123 Cholic acid, 268 subtotal, 244
Cellulitis, 129 Chondromas, 154 total, 244
Centers for Disease Control and Chondrosarcoma, 154 transverse, 244
Prevention, 100 Chronic allograft nephropathy, 93 Collagen
Centers of Medicare and Medicaid Chronic bacterial prostatitis, 338 in inguinal hernias, 315, 316t
Services, 385 Chronic liver disease, 270 synthesis of, steroid effects on, 78
Central line infections, 100 Chronic lymphedema, 311 in wound healing, 74, 76t
Central transtentorial herniation, 345f Chronic lymphocytic leukemia, 297 Collagen disorders, 315
Central venous pressure, 55, Chronic myelogenous leukemia, 303 Collagenase, 74, 76t
106–107, 178 Chronic pancreatitis, 290f–291f, Colloid osmotic pressure, 16
Cephalic vein, 198 290–292, 292t Colloid resuscitation, 30
Cerebellar stroke, 346f Chronic venous insufficiency, 198, 199f Colocolic anastomosis, 245f
Cerebral perfusion pressure, 53, 112 Chvostek sign, 16 Colon
Cerebrospinal fluid leak/leakage, 144, Chylothorax, 156 hepatic flexure of, 54
346–347 Cingulate gyrus, 345f laparoscopic resection of, 378
Cerebrovascular accident, 190 Cirrhosis, 270–271, 276, 280 layers of, 256
Cervical disc herniation, 349, 349t CKI. See Cyclin-dependent kinase leiomyoma of, 249
Cervical esophagus, 207, 213 inhibitors leiomyosarcoma of, 249
Cervical radiculopathy, 349 Claudication, 196 minimally invasive
Cesarean delivery, 341 Clean wound, 39, 40t resection of, 243
Chagas’ disease, 27 Clean/contaminated wound, penetrating injuries to, 54
Charcot triad, 284 39–40, 40t questions regarding, 241–257
Chemical burns, 134 CLL. See Chronic lymphocytic resection of, 243–244
Chemoprevention, of cancer, 84 leukemia Colonic pseudo-obstruction, 251
Chemoreceptors, 30 Clopidogrel, 22–23 Colonoscopy, 252

Brunicardi_Index_p391-420.indd 395 30/06/22 11:32 AM


396
Colorectal cancer of pulmonary artery slings, 167f Cox-Maze IV procedure, 177, 177f
adjuvant chemotherapy for, of total anomalous pulmonary CPB. See Cardiopulmonary bypass
248–249 venous connection repair, 166f CPP. See Cerebral perfusion pressure
INDEX

description of, 123 Concussion, 347 CPR. See Cardiopulmonary


in elderly, 367 Conference live-tweeting, 389 resuscitation
hepatic metastasis of, 277 Conflict resolution, 3t, 4 C-reactive protein, 7
hereditary nonpolyposis, 247–248 Congenital diaphragmatic hernia, 329 CRH. See Corticotropin-releasing
lymph node metastasis of, 248–249 Congenital factor XIII deficiency, 22 hormone
resection of, 348, 367 Congenital heart disease Cricoid cartilage, 146, 146f
staging of, 248 aortic stenosis, 163 Cricopharyngeal muscle, 207
TNM staging of, 248 aortic valve stenosis, 158–159 Cricopharyngeal sphincter, 212
Colostomy, 54, 256 arterial switch operation for, 161 Cricothyroidotomy, 56
Colostrum, 137 atrial septal defects, 157f, 157–158 CRISPR, 125
Columnar epithelium, 208–209 bidirectional Glenn procedure for, CRISPR-associated genes, 125
Command-and-control 160, 160f Critically ill patients
leadership, 3–5 double aortic arch, 167f hyperglycemia in, 33
Common bile duct double-outlet right ventricle, 170 insulin resistance in, 33
arterial supply to, 279 Ebstein’s anomaly, 169f metabolism in, 11
lateral injury to, 281 hypoplastic left heart syndrome, treatment of, 33–34
Common carotid artery, 145, 145f 160–161, 169 Crohn’s disease
Communication management of, 163 description of, 237
errors in, 4 patent ductus arteriosus, 162–163 rectovaginal fistulas caused by, 254
in leadership, 3–4 pulmonary artery slings, 166, steatorrhea in, 268
Community-acquired 167f–168f Crossmatching, 25
pneumonia, 44 tetralogy of Fallot, 161, 170 Cross-sectional study, 382
Compartment syndrome total anomalous pulmonary Crowdsourcing, 390
abdominal, 69, 103, 112–113 venous connection, 159, 159f, CRP. See C-reactive protein
definition of, 351 165–166, 166f Cruveilhier-Baumgarten murmur, 270
description of, 51 transposition of the great vessels, 161 CT. See Computed tomography
Compensated shock, 35 tricuspid atresia, 168 CTA. See Computed tomography
Complement cascade, 32 truncus arteriosus, 165 angiogram
Complicated diverticulitis, 247 ventricular septal defect, 161–162 CTS. See Carpal tunnel syndrome
Computed tomography Wolff-Parkinson-White syndrome Cubital tunnel syndrome, 358
adenomas on, 275f associated with, 169f, 169–170 Cushing ulcers, 349
adrenal incidentaloma on, 328 Congenital inguinal hernias, Cushing’s disease, 327
appendicitis on, 262, 262f 314–315, 315f Cushing’s syndrome, 327
cerebellar stroke on, 346f Congenital nevi, 136 Cutaneous melanoma, 130, 132
cerebrospinal fluid leakage Conn’s syndrome, 327 CVD. See Cardiovascular disease
findings, 346 Consent, informed, 369–370 CVI. See Chronic venous insufficiency
hernia recurrence evaluations, 319 Constructive pericarditis, 178 CVP. See Central venous pressure
inferior vena cava filters on, 203f Contaminated wound, 40, 40t Cyanide poisoning, 66
liver lesions on, 268, 275f Continence, 241–242 Cyanide toxicity, 66
lung cancer diagnosis using, 148 Contusions Cyclin D1, 154
magnetic resonance imaging pancreatic, 53 Cyclin-dependent kinase
versus, 269 pulmonary, 58, 63 inhibitors, 121
mediastinal lymph node staging Cooper’s ligament Cyclins, 121
with, 151 of breast, 139 Cyclophilin, 88
mesenteric vein thrombosis on, 206 of inguinal region, 317 Cyclosporin, 88, 89t, 90
pheochromocytomas on, 327 Corona radiata sign, 148 Cyst(s)
pyogenic liver abscess on, 272f Coronary artery bypass, 173 choledochal, 281, 285,
urolithiasis on, 338 Corticosteroids 286f, 335
Computed tomography angiogram arthritis treated with, 353 dermoid, 135
of arterial disease, 189 immunosuppression uses of, 89t, 90 ganglion, 360
of coarctation of the aorta, 164f side effects of, 89t splenic, 303–304
of double aortic arch, 167f wound healing affected by, 78 thyroglossal duct, 323, 330
of extremities, 55 Corticotropin-releasing hormone, 8 Cystic neoplasms, pancreatic, 295f
of neck, 52 Cortisol, 8 Cytochrome c, 122

Brunicardi_Index_p391-420.indd 396 30/06/22 11:32 AM


397
Cytokines Diffuse alveolar damage, 32 DTIs. See Direct thrombin inhibitors
autophagy regulation of, 9 Diffuse axonal injury, 347 Dumping syndrome, 231
CD4+ T cell production of, 9 Diffuse esophageal spasm, 212 Duodenum

INDEX
proinflammatory, 9 Digestion, carbohydrate, 236 adenocarcinoma of, 238
shock-related elevations of, 32 Digit amputation, 356 gastrinoma of, 294
Th1, 9 Direct Coombs test, 298 hematoma of, 53
Cytomegalovirus, 90 Direct oral anticoagulants, 24 obstruction of, 333, 334f
Direct thrombin inhibitors, 24, 205 ulcers of, 226–227
D Dirty wound, 40, 40t Duplex ultrasonography, for venous
Dabigatran, 24 Disability-adjusted life year, 374 thromboembolism, 204, 204f
Dacron patch aortoplasty, for Disasters, 71 Durable power of attorney for health
coarctation of the aorta, 164 Dislocation care, 370
DALY. See Disability-adjusted life year knee, 352 DVT. See Deep vein thrombosis
Damage control resuscitation, 26 shoulder, 352 Dysphagia, 210
Damage-associated molecular Disseminated intravascular
patterns, 7–8, 10, 29, 37 coagulation, 23 E
DAMPs. See Damage-associated Distal aortic aneurysms, 183 EA. See Esophageal atresia
molecular patterns Distal carcinoma in situ, 137–138 Ebstein’s anomaly, 169f
Damus-Kaye-Stansel anastomosis, 160 Distal ischemia, 107 EBV. See Epstein-Barr virus
DCD. See Donation after cardiac death Distal pancreatectomy, 54 E-cadherin gene, 229
DCIS. See Distal carcinoma in situ Distant flaps, 361 Eccrine sweat glands, 133
DCP3, 374, 374t Diversity, in surgery, 5 Echinococcal antigens, 273
DCR. See Damage control resuscitation Diverticulectomy, 220 Echocardiography
Death Diverticulitis, 247 dobutamine-stress, 172
cardiac, organ donation after, 91 Diverticulopexy, 220 transesophageal, 172–173
causes of, 97 Diverticulum transthoracic, 172–173
medical errors as cause of, 97 Meckel, 235, 335 Edmonton protocol, 94
Decompressive laparotomy, 69 Zenker’s, 220 EDP. See End-diastolic pressure
Deep vein thrombosis, 200–202, DKS anastomosis. See Damus-Kaye- EDS. See Ehler-Danlos syndrome
202t, 366 Stansel anastomosis EDV. See End-diastolic volume
Defecation, 241 DLCO. See Carbon monoxide EE. See Eosinophilic esophagitis
Delirium, postoperative, 365–366 diffusion capacity eFAST. See Extended focused
Democratic leadership style, 3–4 DNA abdominal sonography
Dermatitis, 294 eukaryotic, 119 for trauma
Dermoid cysts, 135 replication of, 121–122 EGD. See
DES. See Diffuse esophageal spasm in RNA synthesis, 120 Esophagogastroduodenoscopy
Desarda repair, of inguinal hernias, DO2. See Oxygen delivery EGFR. See Estimated glomerular
318, 318f DOACs. See Direct oral anticoagulants filtration rate
Desmoid tumors Dobutamine, 34 Ehler-Danlos syndrome, 75,
of abdominal wall, 308 Dobutamine-stress 182, 315
description of, 154 echocardiography, 172 Elderly
Desufflation, 115 Donation after cardiac death, 91 abdominal aortic
Diabetes mellitus Donation by non–heart-beating aneurysms in, 368
dermatitis and, 294 donors, 91 anticoagulation in, 367
pancreatic endocrine tumors Donebedian model of measuring appendicitis in, 367
associated with, 294 quality, 98 cardiac function in, 366
questions regarding, 78, 123–124 Dopamine, 34 colon cancer resection in, 367
Roux-en-Y gastric bypass Dor fundoplication, 221 deep venous thrombosis
effects on, 234 Dormancy, 84 prophylaxis in, 366
wound healing affected by, 78 DORV. See Double-outlet right falls in, 367
Diagnostic peritoneal lavage, ventricle frailty in, 365
59–60, 60t Double aortic arch, 167f heart valve replacement in, 367
Diaphragm, penetrating trauma to, 59 “Double bubble” sign, 333, 334f medications in, 366
Diaphragmatic hernia, 218, 329 Double-outlet right ventricle, 170 necrotizing fasciitis in, 80
DIC. See Disseminated intravascular DPL. See Diagnostic peritoneal nutritional status in, 366
coagulation lavage preoperative assessment and
Difficult airway algorithm, 364 Drug metabolism, 268 counseling in, 365

Brunicardi_Index_p391-420.indd 397 30/06/22 11:32 AM


398
Elderly (Cont.): Endovascular repair Esophagus
preoperative prehabilitation of abdominal aortic aneurysms, 368 antireflux, 214, 216–217, 217f
programs in, 367 of thoracic artery aneurysms, blood supply to, 207, 208f
INDEX

surgery in, 365–368 184, 192 cancer of, 210, 221


transcatheter aortic valve End-tidal CO2, 111, 363 cervical, 207, 213
implantation/replacement End-to-end anastomoses, 244, 245f columnar-lined, 209
in, 368 End-to-side anastomoses, 244, 245f contractions of, 212
trauma in, 63 Enhanced recovery after surgery, 379 gastric juice exposure in, 215, 216f
Electrical burns, 67 Enteral nutrition hernias of, 218, 218f–219f
Electrocardiography/ formulas, 9 innervation of, 212f
electrocardiogram hepatic failure treated with, 9 leiomyoma of, 211
electrical burn evaluations, 67 pancreatic necrosis and, 42 lymphatic drainage of, 213f
hyperkalemia signs, 13 parenteral nutrition versus, 11 Mallory-Weiss tears of, 211
hypocalcemia signs, 14 Enterocutaneous fistula, 238 narrowing of, 207
ICU monitoring uses of, 106 Enterohepatic circulation, 236, 283 nonperistalic contractions of, 212
Elongation factors, 120 Enterolithotomy, laparoscopic, 284 pH monitoring of, 216f
Emergency department Enzyme-linked receptors, 122 resection of, 209
thoracotomy, 49 EOA. See Esophageal obturator rupture of, 211
Emergent orchiectomy, 320 airway Schatzki ring of, 219
Emergent tracheostomy, 56 Eosinophilic esophagitis, 220, 220f squamous cell carcinoma of, 209
Emesis, 330 Eosinophilic granulomas, 155 submucosa of, 213
Empyema, 102–103 Epidermal necrolysis, 133 thoracic, 213
EN. See Enteral nutrition; Epidermal Epidermis, 133 tumors of, 209, 213
necrolysis Epidural analgesia, 245 Essential thrombocythemia, 303
Encephalopathy Epidural anesthesia, 101 Estimated glomerular filtration rate, 148
hepatic, 269 Epinephrine Estrogen
transjugular intrahepatic anesthetic agents with, 356 breast cancer and, 137
portosystemic shunt as questions regarding, 31 ductal development in breasts, 139
cause of, 273 Epithelial ovarian cancer, 342 ET. See Essential thrombocythemia
End colostomy, 54 Epitope tags, 125 Ethics, 369–371, 390
Endarterectomy, 92 Epoprostenol, for pulmonary Eukaryotes
End-diastolic pressure, 106–107 hypertension, 154 DNA, 119
End-diastolic volume, 106–107 Epstein-Barr virus, 90 gene expression, 119, 120f
Endemic granulomatous disease, 148 ERAS. See Enhanced recovery after transcription in, 119
Endoleaks, 185–186, 186t surgery EUS. See Endoscopic ultrasound
Endometriosis, 342–343 ERCP. See Endoscopic retrograde Evidence levels, 381, 381f
Endorectal advancement flap, for cholangiopancreatography Evidence-based hierarchy, 381f
rectovaginal fistula, 253, 254f Error-centric assessments, 387 Ewing sarcomas, 154–155
Endorectal ultrasound, 242–243 Error-enabled approach, 387 Exocrine pancreatic secretions, 93
Endoscopic retrograde Erythema nodosum, 247 Experiential learning, 5
cholangiography, 281 Erythrocyte membrane proteins, 301 Extended focused abdominal
Endoscopic retrograde Erythromycin, 101 sonography for trauma, 59
cholangiopancreatography ESC. See Endoscopic retrograde Extended left colectomy, 244
for chronic pancreatitis, 290–291 cholangiography Extended right colectomy, 243
description of, 50, 53, 61, 102 Eschar, 69 Extended simple mastectomy, 141
for intraductal papillary mucinous Escharotomies, 68 External sphincter, of rectum, 241–242
neoplasms, 296 Escherichia coli, 272, 299 External validity, 383
Endoscopic ultrasound Esophageal atresia, 330, 331f Extracellular fluid volume deficit, 17–18
of chronic pancreatitis, 291, 291f Esophageal dilation, 219 Extremities
of esophageal cancer, 210 Esophageal myotomy, 211 soft tissue sarcomas of, 312
Endoscopy Esophageal obturator airway, 318 vascular injuries of, 54–55, 55t
for hernia repair, 319 Esophagectomy Extrinsic pathway, 21
for ulcerative colitis, 257 description of, 209–210
for volvulus, 250–251 minimally invasive surgery, F
Endothelium 221–222 Facial fractures, 144
platelet adherence to, 21 Esophagitis, 214, 214f, 220, 220f Factor IX, 21
thrombomodulin from, 22 Esophagogastroduodenoscopy, 211 Factor V, 22

Brunicardi_Index_p391-420.indd 398 30/06/22 11:32 AM


399
Factor VIII, 21–22 Fluid resuscitation Gallbladder
Factor X, 21 albumin for, 14 acute cholecystitis of, 279–280
Factor Xa inhibitors, 24 for burns, 68 arterial anomalies of, 282

INDEX
Factor XI, 21 normal saline for, 18, 378 cancer of, 276, 280
Factor XII, 21 Fluid volume deficit, 17–18 lesions of, 281
Factor XIII deficiency, Fluorodeoxyglucose-PET/CT, 269 lymphatics of, 280
congenital, 22 FMD. See Fibromuscular dysplasia polyps of, 286
Falciform ligament, 263, 263f FNAB. See Fine needle aspiration wall layers of, 282
Falls, 367 biopsy Gallstone(s)
False aneurysms, 181 FNH. See Focal nodular hyperplasia description of, 280
Familial adenomatous polyposis, FOBT. See Fecal occult blood test ileus, 284, 285f
238, 308 Focal nodular hyperplasia, 275, 275f pancreatitis, 289
FAP. See Familial adenomatous Focused assessment with sonography GALT. See Gut-associated lymphoid
polyposis in trauma, 60 tissue
FAST. See Focused assessment with Fontan repair, 160 Ganglion cyst, 360
sonography in trauma Forced expiratory volume in Gangrenous appendicitis, 261f
Fasting, 364, 366 1 second, 152 Gardner syndrome, 154
FBN-1 gene, 75 Forced-error approach, 387 Gastrectomy
Fecal continence, 241–242 Foreign bodies dumping after, 231
Fecal occult blood test, 256 rectal, 255 partial, 62
Felon, 359, 359f retained, 99 Gastric antral vascular ectasia,
Felty syndrome, 298, 304–305 Fournier’s gangrene, 339 230–231, 231f
Femoral hernia, 313, 317 Fourth-degree hemorrhoids, 252 Gastric banding, adjustable, 233
Ferritin, 270 Fractional inspiration of Gastric bypass
Fetal acidosis, 117 oxygen, 41 hypoglycemia after, 234
Fetal aortic valvotomy, 158 Fractional saturation of Roux-en-Y. See Roux-en-Y gastric
Fetal wound healing, 77 hemoglobin, 110 bypass
α-Fetoprotein, 155 Fractures Gastric cancer, 229
FEV1. See Forced expiratory volume facial, 144 Gastric carcinoid tumors, 230
in 1 second humeral shaft, 352 Gastric emptying, 224t
FFP. See Fresh frozen plasma long bone, 351–352 Gastric injuries, 62
Fibrillin, 75, 181 pelvic, 351–352 Gastric juice, esophageal exposure to,
Fibrin glue fixation, for inguinal reduction of, 351 215, 216f
hernia repair, 319 rib, 63 Gastric sleeve procedure, 234
Fibrinogen, 21 sacral, 352 Gastric stasis, after bariatric
Fibroblastic phase, of wound healing, skull base, 346–347 surgery, 224
73f, 74 Frailty, 365 Gastrin, 208, 223
Fibroids, uterine, 341 Free flaps, 361 Gastrinoma, 225–226, 294
Fibromas, 180 Free water deficit, 15 Gastroduodenal arteries, 279
Fibromuscular dysplasia, 195 Fresh frozen plasma, for thrombotic Gastroesophageal junction, 211, 216
Fibroplasia, 78 thrombocytopenic purpura, 25 Gastroesophageal reflux disease, 214,
Fine needle aspiration biopsy, of FTS. See Flexor tenosynovitis 216, 219, 234
thyroid nodules, 324 Full-thickness burns, 67–68 Gastrografin, 236
First-degree burns, 66, 68 Full-thickness wounds, 75–76 Gastrointestinal anastomosis
First-degree hemorrhoids, 252 Fundoplication stapler, 221
Fistula Belsey, 221 Gastrointestinal bleeding, 238
enterocutaneous, 237 Dor, 221 Gastrointestinal stromal tumors,
pancreatic, 294 goal of, 217 230, 238, 249
rectovaginal, 253–255, 254f Nissen, 116, 216 Gastrointestinal tract, wound healing
tracheoesophageal, 330, 331f Fungi, 153 in, 75–76, 76t
FK506-binding proteins, 89 Furosemide, 18 Gastroschisis, 335
FKBPs. See FK506-binding Gastrosplenic ligament, 300, 301f
proteins G GBM. See Glioblastoma multiforme
Flail chest, 58 G0 phase, of cell cycle, 83 GCS. See Glasgow Coma Scale
Flaps, 361 G1 phase, of cell cycle, 83, 121 GEJ. See Gastroesophageal junction
Flexor tenosynovitis, 359 G2 phase, of cell cycle, 83, 121 Gene expression, 10, 124
Fluid maintenance, 16 Gain-of-function mutations, 121 General anesthesia, 245, 363, 366

Brunicardi_Index_p391-420.indd 399 30/06/22 11:32 AM


400
Generalizability, 383 Gunshot wounds, 50, 54 pulmonary vein isolation for, 177
Genomic surgery, 119–125 Gustillo-Anderson classification, 352 transesophageal echocardiography
GERD. See Gastroesophageal Gut-associated lymphoid tissue, for, 172–173
INDEX

reflux disease 236, 259 transthoracic echocardiography


Geriatric patients G-value, 28 for, 172–173
necrotizing fasciitis in, 80 Gynecology, 341–343 tricuspid stenosis, 176
trauma in, 63 Gynecomastia, 137–138, 140 Heart murmurs, 175
Geriatric syndromes, 365 Heart transplantation, 95
GIA stapler. See Gastrointestinal H Heart valve replacement, 174–175,
anastomosis stapler Hair removal, 38 268, 367
GISTs. See Gastrointestinal stromal Hairy cell leukemia, 302 Helicobacter pylori, 223–224, 230
tumors HAL. See Hand-assisted laparoscopy Heller myotomy, 116, 221
Glasgow Coma Scale, 52, Halo test, 347 Helper T cells, 10
52t, 63, 347, 347t Halsted radical mastectomy, 141 Hemangiomas, 274–275, 362
Glenn procedure, bidirectional, Hamartomas, 148 Hematoma
160, 160f Hand in bone healing, 77
Glioblastoma multiforme, 348, 348f felon of, 359, 359f duodenal, 53
Global surgery, 373–376 flexor tenosynovitis of, 359 Hemidesmosomes, 132
GLP-1, 234 high-pressure injection Hemodialysis, 18
Glucagon, 294 injuries to, 357 Hemoglobin
Glucagonoma, 294 infections of, 359, 360f oxygen saturation of, 111
Gluconeogenesis, 267 malignant tumor of, 360 during shock resuscitation, 62
Glucose, hepatic production of, 267 necrotizing myositis of, 358 Hemolytic anemias, 298–300
Glucose transporter 2, 236 replantations, 356 Hemorrhage
Glucose transporter 5, 236 squamous cell carcinoma of, 360 abdominal, 50
Glucose-6-phosphate dehydrogenase surgery of, 355–360 hepatic, angiography for, 61
deficiency, 300 Hand-assisted laparoscopy, 243 intraperitoneal, 61
GLUT2. See Glucose transporter 2 Harris-Benedict equation, 69 pelvic, 50
GLUT5. See Glucose transporter 5 Hasson approach, 117 posttransplant, 92
Glutamine, 8 HCC. See Hepatocellular carcinoma variceal, 271
Glycogenolysis, 31, 267 hCG. See Human chorionic Hemorrhagic shock, 19, 30
Glycoprotein I/IX/V, 21 gonadotropin Hemorrhoids, 252
GnRH. See Gonadotropin releasing HCL. See Hairy cell leukemia Hemosiderin, 198
hormone HD. See Hodgkin disease Hemostasis
Goal-directed therapy, 378 HDGC. See Hereditary diffuse physiological events of, 21
Goblet cells, 147, 147f, 209 gastric cancer primary, 21
Goleman, Daniel, 4 Head and neck region, 143–144 Hemothorax, 53
Gonadotropin releasing hormone, 343 Head injuries, 53 Heparin, low molecular weight,
Gore Propaten graft, 195 Healing 203, 366
GP. See Glycoprotein I/IX/V bone, 77 Heparin-induced thrombocytopenia,
GPCRs. See G-protein–coupled wound. See Wound healing 23–24, 173, 205
receptors Health care facilities, 386 Hepatic abscess, 41–42
G6PD. See Glucose-6-phosphate Heart disease Hepatic artery
dehydrogenase deficiency aortic stenosis, 175 anatomy of, 264, 265f
G-protein–coupled receptors, 122 arrhythmias, 176 anomalies of, 282
Graft loss, 93 bypass conduits for, 173 replaced right, 289
Graft rejection, 90 congenital. See Congenital heart right, 282, 289
Graft thrombosis, 93 disease Hepatic duct confluence, 266, 267f
Granuloma Cox-Maze IV procedure for, Hepatic encephalopathy, 269
eosinophilic, 155 177, 177f Hepatic failure, 9
infectious, 148 intra-aortic balloon pump for, 179 Hepatic glycogenolysis, 31
Graves disease, 324 left ventricular assist devices for, 179 Hepatic veins, 265, 266f
Great saphenous vein, 197 in Marfan syndrome, 175 Hepatitis C
Greater omentum, 308 mitral stenosis, 174–175 blood transfusion transmission
Groin hernia, 313, 314t New York Heart Association of, 27
Growth factor receptors, 122 Classification of, 171, 171f chronic carrier state, 44
GSV. See Great saphenous vein pericarditis, 178 chronic liver disease caused by, 270

Brunicardi_Index_p391-420.indd 400 30/06/22 11:32 AM


401
Hepatobiliary iminodiacetic acid HPA axis. See Hypothalamic- Hypokalemia, 16
scan, 283, 283f pituitary-adrenal axis Hypomagnesemia, 16, 19
Hepatocellular carcinoma, 276–277 HPV. See Human papillomavirus Hyponatremia, postoperative, 13

INDEX
Herbicide exposure, 311 HS. See Hereditary spherocytosis Hypoparathyroidism, 14
Herceptin. See Trastuzumab HSV. See Herpes simplex virus Hypoplastic left heart syndrome,
Hereditary cancer, 83 HTS. See Hypertrophic scars 160–161, 169
Hereditary diffuse gastric cancer, 229 Human bites, 133–134 Hypoplastic left ventricle, 159
Hereditary hemochromatosis, 270 Human chorionic gonadotropin, 155 Hypotension, in shock, 35
Hereditary nonpolyposis colorectal Human embryonic stem cells, 124 Hypothalamic-pituitary-adrenal axis,
cancer, 247–248 Human genome, 121 8–9
Hereditary spherocytosis, 297, 301 Human immunodeficiency virus Hypothermia, 40–41, 185
Hernia blood transfusion transmission of, 27 Hypovolemic shock, 29, 31–32
abdominal, 270 surgeon-to-patient transmission Hypoxia, intraoperative, 40–41
abdominal wall, 307 of, 43–44
congenital diaphragmatic, 329 transmission of, 27, 43–44 I
diaphragmatic, 218 tuberculosis risks, 153 IABP. See Intra-aortic balloon pump
esophageal, 218, 218f–219f Human leukocyte antigen system, 87, IAP. See Intra-abdominal pressure
femoral, 313, 317 91–92 ICP. See Intracranial pressure, increased
fibrin glue fixation for, 319 Human papillomavirus Idarucizumab, 24
groin, 313, 314t head and neck cancers, 143 Idiopathic thrombocytopenic
hiatal, 218, 218f–219f malignancy risks, 135 purpura, 23, 298, 302
incisional, 307–308 vaccine for, 135, 342 IL-2. See Interleukin-2
inguinal. See Inguinal hernias Humeral shaft fractures, 352 IL-6. See Interleukin-6
Nyhus classification of, 313, 313t Hurley classification, of hidradenitis IL-10. See Interleukin-10
paraesophageal, 218, 218f–219f suppurativa, 133 IL-17. See Interleukin-17
recurrence of, 319, 321 Hydatid disease, 273 Ileocolic anastomosis, 245f
Shouldice repair for, 321 Hydroxocobalamin, for cyanide Ileocolostomy, 54
transabdominal preperitoneal poisoning, 66 Ileum, 235
repair of, 321 Hydroxyethyl starch solutions, 14 Ileus
Herpes simplex virus, 90 Hyperacute rejection in gallstone, 284, 285f
hESCs. See Human embryonic transplantation, 88 postoperative, 101, 378
stem cells Hyperbilirubinemia, 103 IM. See Intestinal metaplasia
Heterotopic ossification, 70 Hypercalcemia, acute, 18 Imatinib, 230, 249
Hiatal hernia, 218, 218f–219f Hyperchloremic metabolic Iminodiacetic acid, 283
HIDA scan. See Hepatobiliary acidosis, 18 Immune response
iminodiacetic acid scan Hypercoagulability, 30 to injury, 9
Hidradenitis suppurativa, 133 Hypergastrinemia, 225, 225f platelet’s role in, 10
High reliability organization Hyperglycemia Immune thrombocytopenia,
theory, 97 in critically ill patients, 33 23, 300, 302
High-mobility group protein B1, 8 postoperative, 40 Immune-based therapies,
High-pressure injection injuries, 357 postoperative fasting as cause of, 377 for cancer, 85
Hirschsprung disease, 334 Hyperkalemia Immunonutrition supplements, 377
Hispanics, 5 signs and symptoms of, 13 Immunosuppression
HIT. See Heparin-induced treatment of, 15–16 cardiac transplantation, 95
thrombocytopenia Hyperreflexia, 16 drugs for, 88–90, 89t
HLA system. See Human leukocyte Hypertonic saline, 14 indications for, 87
antigen system Hypertriglyceridemia, 90 lung abscess secondary to, 153
HLHS. See Hypoplastic left heart Hypertrophic scars, 80–81 tuberculosis secondary to, 153
syndrome Hypoalbuminemia, 16 Immunotherapy, oncogene
HMGB1. See High-mobility group Hypocalcemia, 14 targeting by, 125
protein B1 asymptomatic, 16 IMPDH. See Inosine monophosphate
HNPCC. See Hereditary nonpolyposis hypomagnesemia and, 16, 19 dehydrogenase
colorectal cancer Hypochloremic, hypokalemic Imperforate anus, 336
HO. See Heterotopic ossification metabolic alkalosis, 17 Impingement syndromes, 352–353
Hodgkin disease, 297 Hypocoagulability, 30 In situ cancers, 84
Holosystolic murmur, 175 Hypoglycemia, post-gastric Incisional hernia, 307–308
Horner syndrome, 149 bypass, 234 Incontinence, stress, 342–343

Brunicardi_Index_p391-420.indd 401 30/06/22 11:32 AM


402
Indwelling intravascular catheter in scrotum, 316–317 Intra-aortic balloon pump, 34, 179
infection, 43 Shouldice repair for, 321 Intra-arterial blood pressure
Infection transabdominal preperitoneal monitoring system, 106
INDEX

breast, 140 repair of, 321 Intracranial pressure


central line, 100 Inhalational anthrax, 44 increased, 345, 349
definition of, 37 Injury monitoring of, 112
hand, 359, 360f complement cascade activation Intraductal papillary mucinous
indwelling intravascular catheter, 43 after, 32 neoplasms, 296
after injury, 11 gastric, 62 Intraductal papillomas, 137
intravascular catheter, 43, 46–47 head, 53 Intraperitoneal hemorrhage, 61
necrotizing, 46, 358 immune response to, 9 Intravascular catheter infections, 43,
necrotizing fasciitis, 80 infection after, 11 46–47
overwhelming postsplenectomy, lactic acid production in Intravascular space, 14
305–306 response to, 17 Intravascular volume, pregnancy-
pancreatic, 42 life-threatening, 49, 59 related changes in, 62
postsplenectomy, 302, 305–306 metabolic support after, 7–11 Intrinsic factor, 224, 234
posttransplantation, 90 metabolism after, 11 Intrinsic pathway, 21
retroperitoneal, 309 in pregnancy, 62 Intubation, 146
surgical site, 39t, 39–41, 47, 246 superior laryngeal nerve, 100 Intussusception, 335–336
systemic manifestations of, 37 systemic response to, 7–11 IOM. See Institute of Medicine
in transplant recipients, 90 Injury-mediated inflammatory IPC. See Intermittent pneumatic
wound, 102 response, 8 compression therapy
Infectious granulomas, 148 Inosine monophosphate IPMNs. See Intraductal papillary
Inferior thyroid artery, 207, 323 dehydrogenase, 88 mucinous neoplasms
Inferior vena cava INR. See International normalized IRB. See Institutional Review Board
filters, 202, 203f ratio IRS. See Insulin receptor substrate
suprahepatic, 265 Insensible fluid losses, 116 Ischemia
Inflammatory bowel disease, Institute of Medicine, 97 acute limb, 194
246–247 Institutional Review Board, 371 distal, 107
Inflammatory response Insulin, 123–124 intestinal, 251–252, 330
injury-mediated, 8 Insulin receptor substrate, 123–124 large bowel, 251–252
in traumatic injury, 7–8 Insulin resistance local tissue, 134
Informed consent, 369–370 in critically ill patients, 33 mesenteric, 192–193
Inguinal hernias perioperative, 377 pressure-induced, 134
acquired, 314 Insulin-signaling pathway, 123f small bowel, 251–252
age-based incidence of, 313 Integrins, 84 Ischemic cardiomyopathy, 173
antibiotic prophylaxis for, 316 Interatrial septum, 161 Ischemic colitis, 251–252
collagen types in, 315, 316t Interleukin-2, 9, 89 Ischemic load, 106
congenital, 314, 315f Interleukin-6, 7, 32 Ischemic orchitis, 320–321
Desarda repair of, 318, 318f Interleukin-10, 9, 32 Islet cell transplantation, 94–95
extracellular matrix elements Interleukin-17, 11 ITA. See Inferior thyroid artery
and, 315 Intermittent pneumatic compression ITP. See Idiopathic
fibrin glue fixation for, 319 therapy, 205 thrombocytopenic purpura;
incidence of, 313, 313t Internal hemorrhoids, 252 Immune thrombocytopenia
ischemic orchitis and, 320–321 Internal sphincter, of rectum, 241 IVC. See Inferior vena cava
McVay Cooper ligament repair of, Internal thoracic artery, 173
317, 317f International normalized ratio, J
nociceptive pain associated with, 320 175, 268 Jaw thrust, 58
Nyhus classification of, 313, 313t Interstitial cells of Cajal, 279 JCAHO, 386
open repair of, 316 Interstitial fluid compartments, 16 Jejunoileal bypass, 233
in pediatric patients, 334 Intestinal ischemia, 251–252, 330 Jejunum, 235
in preterm infants, 314 Intestinal malrotation, 308
prevalence of, 313t Intestinal metaplasia, 209–210 K
recurrence of, 319, 321 Intestinal obstruction, 330 Kayexalate, 15–16
repair of, 316–317, 317f, 321, 334 Intra-abdominal hypertension, 113 Keloids, 80–81, 81f
risk factors for, 314–315 Intra-abdominal pressure, Keratin, 132
robotic-assisted repair of, 319 112–113, 116 Keratinocytes, 132

Brunicardi_Index_p391-420.indd 402 30/06/22 11:32 AM


403
Keyhole foraminotomy, 349 small bowel obstruction Liver
Kidney(s) treatment, 237 acute failure of, 269
magnesium homeostasis by, 16 splenectomy, 299, 305 adenomas of, 275, 275f

INDEX
transplantation of, 91–93, 92f Large bowel ischemia, 251–252 anatomy of, 263, 263f–264f
Kidney grafts, 91–92, 92f Larynx benign lesions of, 274–275, 275f
Kidney transplant anatomy of, 146, 146f Child-Turcotte-Pugh
in adult recipients, 91–92 cancer of, 144 score of, 274, 274t
complications of, 92–93 fractures of, 56 cholangiocarcinoma of, 276
graft thrombosis in, 93 Lateral femoral cutaneous nerve chronic disease of, 270
living donor, 91 injury, 320 cirrhosis of, 270–271, 276, 280
Kirkpatrick four-level LCGS. See Lancet Commission on colorectal metastases to, 277
scale, 388 Global Surgery drug metabolism in, 268
Klatskin’s tumor, 276, 287 LCH. See Langerhans cell ducts of, 266, 267f
Knee dislocation, 352 histiocytosis failure of, 95
KRAS mutations, 148 LCIS. See Lobular carcinoma in situ falciform ligament of, 263, 263f
K-time, 28 Le Fort fractures, 144 focal nodular hyperplasia of,
Kulchitsky cells, 147, 147f Leadership 275, 275f
Kupffer cells, 283 communication in, 3–4 glucose production by, 267
conflict resolution in, 3t, 4 hemangiomas of, 274–275
L principles of, 3, 3t hemorrhage of, angiography for, 61
Lactated Ringer solution styles of, 3–4 hepatic artery of, 264, 265f
for burns, 68 Leadership skills, 5 hepatocellular carcinoma, 276–277
intravenous colloid solution Learning inflammatory bowel disease
versus, for hemorrhagic experiential, 5 effects on, 246
shock, 30 willingness to learn, 3t lesions of, 268–269, 274–275
for metabolic acidosis, 19 Left atrial pressure, 174 lobes of, 263, 264f
Lactic acid, 17 Left colectomy, 244 Model for End-Stage Liver Disease,
LAMN. See Low-grade appendiceal Left subclavian artery, proximal, 95, 274
mucinous neoplasms 60–61 portal hypertension, 270–271
Lancet Commission on Global Left ventricle pyogenic abscesses of, 41, 272, 272f
Surgery, 375 aneurysms of, 176 transplantation of, 95, 276–277,
Langerhans cell histiocytosis, 155 hypoplastic, 159 280, 287
Langerhans cells, 133 tricuspid atresia and, 168 variceal hemorrhage of, 271
Laparoscopic cholecystectomy Left ventricular assist devices, 179 veins of, 265, 266f
acute cholecystitis treated Left ventricular outflow tract Liver trauma
with, 284 obstruction, 158 bleeding caused by, 61
anesthesia for, 364 Left ventricular pressure-volume description of, 50
biloma after, 102 loops, 107 Living donor kidney transplant, 91
safety concerns for, 117 Left ventricular stroke volume, 110 Living donors, 91
Laparoscopy Leiomyoma Living wills, 370
abdominal compartment of colon, 249 LMICs. See Low- and middle-income
syndrome after, 103 of esophagus, 211 countries
adrenalectomy, 328 of uterus, 341 LMWH. See Low molecular weight
antireflux surgery, 216 Leiomyosarcoma, 249 heparin
appendectomy, 259–261 Lentigo maligna melanoma, 130 Lobular carcinoma in situ, 137
arrhythmia during, 115 LES. See Lower esophageal sphincter Local anesthetic systemic toxicity, 364
cholecystectomy. See Laparoscopic Lesions Local tissue ischemia, 134
cholecystectomy gallbladder, 281 Long bone fractures, 351–352
enterolithotomy, 284 liver, 274–275, 275f Loop colostomy, 54
gangrenous appendicitis on, 261f Leukoplakia, vocal cord, 143 Low- and middle-income countries,
hand-assisted, 243 Life-threatening injuries, 49, 59 373–375
ileus after, 378 Ligament of Treitz, 42 Low glycemic index diets, 377
Nissen fundoplication, 116 Limb amputation, 356 Low molecular weight heparin,
in pregnancy, 117 Linitis plastica, 229 203, 366
robotic surgery versus, 116–117, Lip cancer, 143 Lower esophageal sphincter, 208,
117f, 319 Lipid metabolism, 11 212, 214, 217f, 221
skills training in, 375–376 Lipodermatosclerosis, 198 Lower extremity veins, 197

Brunicardi_Index_p391-420.indd 403 30/06/22 11:32 AM


404
Low-grade appendiceal mucinous Magnetic resonance angiography, 184 Medullary carcinoma of the
neoplasms, 260–261 Magnetic resonance thyroid, 325
Low-oxygen tension, 78 cholangiopancreatography, 53 Melanoma
INDEX

Luminal peptides, 223 Magnetic resonance imaging ABCDE initialism for, 130
Lumpectomy, 138, 140 computed tomography versus, 269 acral lentiginous, 130–131
Lund and Browder chart, 66 of double aortic arch, 167f advanced, 131f
Lung glioblastoma multiforme on, 348f anorectal, 250
abscess of, 152, 152t hernia recurrence evaluations, 319 cutaneous, 130, 132
adenocarcinoma of, 148–149, 150f mesenteric vein thrombosis on, 206 description of, 130
histology of, 147f pheochromocytomas on, 327 excision of, 132
Lung cancer Major adverse cardiac events, 172 lentigo maligna, 130
algorithm for, 150f Major histocompatibility complex, 87 metastasis of, 130
computed tomography of, 148 Malaria, 27 nodular, 130, 131f
management of, 149 Malignant hyperthermia, 363 sentinel lymph node biopsy for, 132
non-small-cell, 151 Malignant mesothelioma, 156 superficial spreading, 130, 130f
pathologic progression of, 148 Malignant wounds, 80 MELD. See Model for End-Stage
predictive markers for, 149 Mallory-Weiss tears, 211 Liver Disease
prognostic markers for, 149 Malnutrition/malnourished patients MEN2A, 325
pulmonary function magnesium deficiency in, 19 MEN2B, 325
studies in, 152 total body water in, 15 Ménétrier disease, 231, 231f
signs and symptoms of, 149 wound healing, 79 Mentees, 5
staging of, 152 Maloney dilator, 220 Mentoring, 5
TNM staging of, 152 MALT lymphoma. See Mucosa- Mentors, 5
LVADs. See Left ventricular associated lymphoid tissue Meralgia paresthetica, 320
assist devices lymphoma 6-Mercaptopurine, 88
LVOT. See Left ventricular outflow Mammalian target of rapamycin Mesenteric circulation, 192–193
tract obstruction inhibitors, 90 Mesenteric ischemia, 192–193
LVSV. See Left ventricular Mammography, 138 Mesenteric vein thrombosis, 206
stroke volume Marfan syndrome, 75, 175, 181, 183 Mesothelioma, 156
LY30, 28 Marginal artery of Drummond, 243 Meta-analyses, 381
Lymph nodes Marijuana smoking, 144 Metabolic acidosis
axillary, 137 Marjolin’s ulcer, 80 diagnosis of, 17
breast cancer metastasis to, 139 Mass casualty incidents, 71 in hemorrhagic shock patients, 19
colorectal cancer metastasis to, Massive resuscitation, 69 hyperchloremic, 18
248–249 Mastectomy, 140–141 laboratory tests for, 14
mediastinal, 151 Maternal hypercarbia, 117 Lactated Ringer solution for, 19
metastases, 139, 148 Matrix metalloproteinases, 74, 84 metabolic alkalosis versus, 17
Lymphatic drainage Mayo protocol, 276 with normal anion gap, 13, 17
of anorectal region, 241 May-Thurner syndrome, 201 Metabolic alkalosis
of gallbladder, 280 MCT. See Medullary carcinoma of diagnosis of, 17
Lymphatic massage, 205 the thyroid hypochloremic, hypokalemic, 17
Lymphedema, 205–206, 311 McVay Cooper ligament repair, metabolic acidosis versus, 17
Lynch syndrome, 247–248 317, 317f Metabolic support, 7–11
Mechanical bowel preparation, 246 Metabolism
M Mechanical valve replacement, drug, 268
M phase, of cell cycle, 121 174–175, 268 after injury, 11
MACE. See Major adverse Mechanical ventilation Metastases
cardiac events nosocomial pneumonia Barrett esophagus, 208–209
Macrophages caused by, 44 colorectal cancer, 248–249
microbe response by, 37 peak airway pressure melanoma, 130
questions regarding, 10, 37 monitoring, 111 omental, 308
in wound healing, 79 Meckel diverticulum, 235, 335 questions regarding, 84–85
Mafenide acetate, 69 Medial thyroid anlage, 323 splenic, 298
Magnesium Median nerve, 355, 355f surgical resection of, 85
deficiency of, 19 Mediastinal lymphadenopathy, 151 Methicillin-resistant Staphylococcus
depletion of, 16 Mediastinal mass, 155 aureus, 129, 134
Magnesium citrate, 246 Medical errors, 97 Methionine, 120

Brunicardi_Index_p391-420.indd 404 30/06/22 11:32 AM


405
Methionyl-tRNA, 120 MRSA. See Methicillin-resistant Nephroureterectomy, for Wilms
Metoclopramide, 101 Staphylococcus aureus tumor, 331
MEWS system. See Modified Early MSI. See Microsatellite instability Nerve injuries, 77

INDEX
Warning Score system MSLT-1, 132 NETs. See Neutrophil extracellular
MH. See Malignant hyperthermia mtDNA. See Mitochondrial DNA traps
MHC. See Major histocompatibility mTOR inhibitors. See Mammalian Neurapraxia, 77
complex target of rapamycin inhibitors Neuroendocrine carcinoma, 148
Microbes, macrophage response to, 37 Mucosa-associated lymphoid tissue Neurogenic shock, 29–31, 35
Microbial peritonitis, primary, 41 lymphoma, 230 Neuropathic pain, 320
Microcirculation, 134 Mucosal biopsy, in Ménétrier disease, Neurosurgery, 345–350
Micro-organisms, 45 231, 231f Neurotmesis, 77
MicroRNA, 119 Mucous cells, 147f Neutropenia
Microsatellite instability, 247–249 Multidisciplinary teams, 5 in Felty syndrome, 298, 304–305
Microvascular thrombosis, in Multiple organ dysfunction silver sulfadiazine as cause of, 68–69
thrombotic thrombocytopenic syndrome, 32 Neutropenic enterocolitis, 255
purpura, 24–25 Murmur, holosystolic, 175 Neutrophil(s)
Middle cerebral artery, 112 MVT. See Mesenteric vein thrombosis in phagocytosis, 74
Midface fractures, 144 Mycophenolate acid, 88 polymorphonuclear, 74
Milliosmoles, 15 Mycophenolate mofetil, 88, 89t Neutrophil extracellular traps, 10
Minimally invasive surgery Mycoses, 153 New York Heart Association
colon resection, 243 Mycotic aneurysms, 182–183 Classification, 171, 171f, 179
esophagectomy, 221–222 Myocardial infarction, cardiogenic Newborn
questions regarding, 115–117 shock and, 33 congenital diaphragmatic
resection, 243 Myonecrosis, 358 hernia in, 329
Mismatch repair, 247 Myotomy, 116, 221 emesis in, 330
Mithramycin, 18 Myxomas, cardiac, 179 imperforate anus in, 336
Mitochondrial DNA, 8 intestinal obstruction in, 330
Mitral stenosis, 174–175 N packed red blood cell
Mitral valve replacement, 175 N-acetylcysteine, 336 transfusion in, 336
M&M conferences. See Morbidity Nasojejunal feeding tubes, 42 total body water in, 15
and mortality conferences National Nosocomial Infections NHBDs. See Non–heart-beating
MMF. See Mycophenolate mofetil Surveillance System, 47 donors
MMPs. See Matrix National Surgical Quality NHL. See Non-Hodgkin lymphoma
metalloproteinases Improvement Project, 99, 172 Nikolsky sign, 133
Model for End-Stage Liver Nausea and vomiting, Nissen fundoplication
Disease, 95, 274 postoperative, 378 gastric pressure after, 216
Modified Early Warning Score Near-total thyroidectomy, 324 laparoscopic, 116
system, 108–109 NEC. See Necrotizing enterocolitis; NNISS. See National Nosocomial
Modified Norwood procedure, 160 Neuroendocrine carcinoma Infections Surveillance System
Modified radical mastectomy, 141 Neck Nociceptive pain, 320
MODS. See Multiple organ injuries to, 52, 57 NOD2 gene, 237
dysfunction syndrome questions regarding, 143–144 Nodular basal cell carcinoma, 135
Mohs micrographic surgery, 129 Necrotizing enterocolitis, 332f, Nodular melanoma, 130, 131f
Molecular surgery, 119–125 332–333 Noncardiac surgery, 172
Mondor disease, 140 Necrotizing fasciitis, 80, 339 Non–heart-beating donors, 91
Monocytes, 10 Necrotizing infections, 46, 358 Non-Hodgkin lymphoma, 297, 299
Monopolar coagulation, 116 Necrotizing myositis, 358 Nonseminatous germ-cell
Morbidity and mortality Necrotizing soft-tissue infections, tumors, 155
conferences, 4 42, 134 Non-small-cell lung cancer, 151
Morpheaform basal cell Needle thoracostomy decompression, Nonsteroidal anti-inflammatory
carcinoma, 135 of tension pneumothorax, drugs, 226, 320
mOsm. See Milliosmoles 56, 58 Nontechnical skills, 387
Motilin, 208 Needlesticks, 43–44 Norepinephrine, 31
Motor vehicle collisions, 59 Neisseria gonorrhoeae, 343 Normal saline, 14, 18, 378
MRCP. See Magnetic resonance Nelson syndrome, 328 Normovolemia, 378
cholangiopancreatography Neobladder, orthotopic, 337 Northern blotting, 124
mRNA, 119–120 Neostigmine, 101, 251 Norwood operation, 158

Brunicardi_Index_p391-420.indd 405 30/06/22 11:32 AM


406
Nosocomial pneumonia, 44, 100 Orthotopic transplants, 87 description of, 14
NSAIDs. See Nonsteroidal OSCE. See Objective structured gallstone, 289
anti-inflammatory drugs clinical examination Ranson’s criteria for, 289, 289t
INDEX

NSCLC. See Non-small-cell Osmolality, 15 Panel-reactive antibody assay, 92


lung cancer Osmoles, 15 PAOP. See Pulmonary artery
NSQIP. See National Surgical Quality Osmotic pressure, 15, 16 occlusion pressure
Improvement Project Ostium secundum type defect, Papillary thyroid cancer, 281, 324
Null hypothesis, 382 157, 157f Paraesophageal hernia, 218,
Nutrition Outcome measures, 98, 98t 218f–219f
for burn patients, 69 Ovarian cancer, 342 Parathyroid adenoma, 155
enteral. See Enteral nutrition Overwhelming postsplenectomy Parathyroid glands
parenteral, 11 infection, 305–306 location of, 326
Roux-en-Y gastric bypass-related Oxygen delivery, 105 supernumerary, 325
deficiencies of, 234 intraoperative, 41 Parenteral nutrition
in wound healing, 79 during wound healing, 78 acute pancreatitis treated with, 290
Nutritional status, 366 Oxygen saturation, 110 enteral nutrition versus, 11
NYHA. See New York Heart Oxygen utilization, 105 Parietal cells, 224, 231
Association Classification Oxytocin, 139 Parkland formula, 68
Partial gastrectomy, 62
O P Partial pressure of carbon dioxide, 18
Obesity PAC. See Pulmonary artery catheters Partial-thickness burns, 67–68
surgical management of, 233–234 Pacesetting, 3–4 Pass-Muir valve, 144
total body water, 15 Packed red blood cell transfusion, 336 Patent ductus arteriosus, 162–163
Objective structured clinical Paco2. See Partial pressure of carbon Patent processus vaginalis, 314, 315f
examination, 5 dioxide Pathogen-associated molecular
Obstetrics, 341 PAI-1. See Plasminogen activator patterns, 7–8, 10, 29, 37
Obstructive pancreatography, 291f inhibitor-1 Patient safety
Obstructive shock, 29, 35 Pain causes of death, 97
Obturator bypass, 194 back, 352 National Surgical Quality
Occult gastrointestinal bleeding, 238 multimodal strategies for, 378 Improvement Project, 99
Octreotide, 271 neuropathic, 320 questions regarding, 97–103
Octreotide scan, 293f, 327 nociceptive, 320 Surgical Care Improvement
Ogilvie syndrome, 101, 251 Palliative care, surgical, 367, 370 Project, 98, 98t
Oliguria, 112 Pampiniform plexus, 320 wrong-site surgeries, 99
OLT. See Orthotopic liver PAMPs. See Pathogen-associated Pattern recognition receptors, 7, 29
transplantation molecular patterns PCC. See Prothrombin complex
Omentum Pancoast tumors, 149 concentrates
greater, 308 Pancreas PDGF. See Platelet-derived
tumors of, 308 blunt trauma to, 53 growth factor
Oncogenes contusions of, 53 Peak airway pressure, 111
description of, 124 cystic neoplasms of, 295f Pediatric patients
immunotherapy targeting of, 125 exocrine secretions, 93 biliary atresia in, 333
Online journal clubs, 389 exocrine tumors, 292, 293f choledochal cyst in, 335
Open splenectomy, 299, 305–306 fistula of, 294 duodenal obstruction in, 333, 334f
Operative field, hair removal from, 38 infections of, 42 emesis in, 330
Opioids, postoperative, 378 necrosis of, 42 esophageal atresia in, 330, 331f
OPSI. See Overwhelming neuroendocrine tumors of, 292 gastroschisis in, bowel
postsplenectomy infection transplantation of, 93–95, 94f reduction for, 335
Orchidopexy, 336 Pancreatectomy, distal, 54 hemangiomas in, 362
Orchiectomy, emergent, 320 Pancreatic cancer, 123 Hirschsprung disease in, 334
Organ donation Pancreatic duct stent, 295 imperforate anus in, 336
after cardiac death, 91 Pancreatic leak, 294–295 inguinal hernia repair in, 334
living donors, 91 Pancreaticoduodenectomy, intestinal obstruction in, 330
Organ transplant, 87 286, 294–295 intussusception in, 335–336
Orthopedic surgery, 351–353 Pancreatitis leading cause of death in, 333
Orthotopic liver transplantation, 277 acute, 42, 290 Meckel diverticulum in, 335
Orthotopic neobladder, 337 chronic, 290f–291f, 290–292, 292t N-acetylcysteine in, 336

Brunicardi_Index_p391-420.indd 406 30/06/22 11:32 AM


407
necrotizing enterocolitis in, PGBH. See Post-gastric bypass Pneumoperitoneum, 115–116
332f, 332–333 hypoglycemia Pneumothorax, 53
packed red blood cell Phagocytes, 77 in laparoscopic Nissen

INDEX
transfusion in, 336 Phagocytosis, 74 fundoplication, 116
pneumatosis intestinalis in, 332f, Pheochromocytoma, 327 simple, 56, 58
332–333 Pheromones, 133 tension. See Tension
short bowel syndrome in, 239 Phlegmasia alba dolens, 201 pneumothorax
thyroglossal duct cyst in, 330 Phlegmasia cerulea dolens, 201, 201f Poikilothermia, 194
tracheoesophageal fistula in, Phrenic nerve palsy, 149 Point of critical oxygen delivery, 105
330, 331f Phrenosplenic ligament, 300 Poisoning, cyanide, 66
undescended testes in, 336 Physician authorship, 390 Polycythemia vera, 303
Wilms tumor in, 331, 332t Physicians Polyethylene glycol, 246
Pediatric surgery, 329–336 diversity of, 5 Polymorphonuclear neutrophils, 74
Pedicle flaps, 361 laparoscopic skills training in, Polyps, gallbladder, 286
PEEP. See Positive end-expiratory 375–376 Polytetrafluoroethylene, 195
pressure Physiologic monitoring of surgical Popliteal artery, adventitial cystic
PEG. See Polyethylene glycol patient, 105–113 disease of, 196
Pelvic floor dysfunction, 342 PICO, 382 Portal hypertension, 270–271, 304
Pelvic fractures, 351–352 PID. See Pelvic inflammatory disease Portal hypertensive gastropathy, 231
Pelvic hemorrhage, 50 p15INK4B, 123 Positive end-expiratory pressure,
Pelvic inflammatory disease, 343 “Pipeline” program, 5 for acute respiratory distress
Pemetrexed, 149 Pituitary fossa tumor, 328 syndrome, 100–101
Penetrating trauma Plasma Positron emission tomography, for
to chest, 53 fresh frozen, for thrombotic mediastinal lymph node
to colon, 54 thrombocytopenic purpura, 25 staging, 151
diaphragmatic injury from, 59 interstitial fluid compartments Posterior cervical foraminotomy, 349
to neck, 57 and, 16 Posterior fossa lesions, 346, 346f
to thoracoabdominal region, 59 Plasminogen, 22 Post-gastric bypass hypoglycemia, 234
Pentaxins, 7 Plasminogen activator inhibitor-1, 22 Postoperative period
Peptic ulcer disease Plastic surgery, 361–362 delirium in, 365–366
antrectomy for, 227f–228f, 227–228 Plateau airway pressure, 111 empyema in, 102–103
description of, 223, 226 Platelet(s) hyperglycemia in, 40
primary hyperparathyroidism acquired hemostatic defect, 23 hyponatremia in, 13
associated with, 326 aggregation of, 21, 23, 25 ileus in, 101, 378
Percutaneous tracheostomy, 101 circulating, decreases in, 22 nausea and vomiting in, 378
Perforated appendicitis, 259 drugs that inhibit function of, 23 opioid requirements in, 378
Perianal abscess, 253 endothelial adherence of, 21 urinary tract infection in, 42–43
Pericardial adhesions, 178 in immune response, 10 Postsinusoidal hypertension, 270
Pericardial constriction, 178 life span of, 23 Potassium
“Pericardial knock,” 178 postsplenectomy increase in, 51 alkalosis-related changes in, 15
Pericardial tamponade sequestration of, 22, 25 elevated levels of.
in left subclavian artery trauma, 61 shortened survival of, 22 See Hyperkalemia
traumatic, 55 Platelet-derived growth factor, 74 serum, 15–16
Pericarditis, 178 Plicae circularis, 235 Pouchitis, 256
Pericholangitis, 246 Plummer-Vinson syndrome, 143 Ppeak. See Peak airway pressure
Perihilar cholangiocarcinoma, 287 PN. See Parenteral nutrition PPIs. See Proton pump inhibitors
Perihilar tumor, 281 PNETs. See Primitive Pplateau. See Plateau airway pressure
Perioperative care, 377–379 neuroectodermal tumors PPV. See Patent processus vaginalis;
Peripheral arterial injury, 54–55, 55t Pneumatic tourniquet, 70 Pulse pressure variability
Peritonitis Pneumatosis intestinalis, 332f, 332–333 PRA assay. See Panel-reactive
in elderly, 367 Pneumohemothorax, 57 antibody assay
in pediatric patients, 335 Pneumonia Pragmatic Randomized Optimal
primary microbial, 41 aspiration, 153 Platelet and Plasma Ratios
Permissive hypotension, 26 community-acquired, 44 trial, 26
PET. See Positron emission tomography empyema caused by, 102 Prasugrel, 23
Petco2, 111 nosocomial, 44, 100 pRB, 123
Peyer patches, 235–236 ventilator-associated, 100 Predictive marker, 85

Brunicardi_Index_p391-420.indd 407 30/06/22 11:32 AM


408
Pregnancy Proximal left subclavian artery, 60–61 dysphagia treated with, 210
breast cancer in, 139 PRRs. See Pattern recognition soft tissue sarcoma risks, 311
breast changes in, 137 receptors Radical lymphadenectomy, 238
INDEX

injuries during, 62 Pseudoaneurysms, 181, 183 Radical mastectomy, 141


laparoscopy in, 117 Pseudohyponatremia, 13 Randomized controlled trials, 381
physiologic changes in, 62 Pseudomyxoma peritonei, 260 Ranson’s criteria, 289, 289t
Preload, 106–107, 110 Pseudo-obstruction, colonic, 251 RCC. See Renal cell carcinoma
Preoperative evaluation, 377–378 Pseudostratified ciliated columnar RCTs. See Randomized
Preoperative prehabilitation cells, 147f controlled trials
programs, 367 PTC. See Papillary thyroid cancer Reading frame, 120
Pressure ulcers, 134 PTFE. See Polytetrafluoroethylene Reasonable person standard, 370
Priapism, 338 PUD. See Peptic ulcer disease Reconstructive surgery, 361–362
Primary aldosteronism, 327 Pudendal nerve, 242 Rectal biopsy, for Hirschsprung
Primary hemostasis, 21 Pulmonary artery banding, for disease, 334
Primary hyperparathyroidism, 326 truncus arteriosus, 165 Rectal cancer
Primary immune Pulmonary artery catheters, 109, 109f endorectal ultrasound for, 242–243
thrombocytopenia, 23 Pulmonary artery occlusion pressure, total mesorectal excision for, 249
Primary microbial peritonitis, 41 106–107, 109 Rectopexy, abdominal, 250
Primary sclerosing cholangitis, 246, 276 Pulmonary artery slings, 166, Rectovaginal fistula, 253–255, 254f
Primary survey, of trauma, 49, 59 167f–168f Rectum
Primitive neuroectodermal Pulmonary blood flow, 162 adenocarcinoma of, 256
tumors, 154 Pulmonary contusions, 58, 63 distention of, 241
Principalist approach, to medical Pulmonary edema, 14, 174 foreign body entrapment in, 255
ethics, 369 Pulmonary embolism, 101, 200 layers of, 256
Process measures, 98 Pulmonary function studies, in lung prolapse of, 245, 250
Proficiency-based simulation cancer, 152 questions regarding, 241–257
training, 388 Pulmonary hypertension, 154 wall of, 242, 242f
Progesterone, 139 Pulmonary vein isolation, 177 Rectus abdominis diastasis, 307
Prognostic marker, 85 Pulmonary venous obstruction, Recurrent laryngeal nerve,
Proinflammatory cytokines, 9 after total anomalous 212–213, 323
Prokaryotes, 119 pulmonary venous Refeeding syndrome, 19
Prokinetic agents, 224, 224t connection repair, 165, 166f Regional anesthesia, 245
Prolactin, 139 Pulmonary-to-systemic flow ratio, 162 Renal artery disease, 193
Prolapse, rectal, 245, 250 Pulse pressure, 17 Renal artery revascularization, 193
Proliferative phase, of wound Pulse pressure variability, 110, 110f Renal blood flow
healing, 73f, 74 Pulse volume recording, 190 intra-abdominal pressure
Promoter region, 119 Pulsus paradoxus, 55 effects on, 116
Prophylactic antibiotics Push enteroscopy, 238 pneumoperitoneum effects on, 115
for colectomy, 246 PVI. See Pulmonary vein isolation Renal cell carcinoma, 337, 339
for human bites, 133–134 Pylephlebitis, 41 Renal failure, 91
for inguinal hernias, 316 PVR. See Pulse volume recording Renal tubular acidosis, 17
for overwhelming postsplenectomy Pyogenic abscesses, 41, 272, 272f Replantation, of limb or digit, 356
infection, 306 Pyogenic pericarditis, 178 Resection
for surgical infections, 38–39, 45, 102 Pyruvate kinase deficiency, 301 of colon, 243–244, 378
PROPPR trial. See Pragmatic of colorectal cancer, 248, 367
Randomized Optimal Platelet Q of distant metastases, 85
and Plasma Ratios trial Quality measurements, Donebedian laparoscopic, of colon, 378
Prostaglandin E, 168 model of, 98 minimally invasive, 243
Prostate cancer, 338 Quid chewing, 144 Restrictive cardiomyopathy, 178
Prostatitis, 338 Resuscitation
Prosthetic grafts, 195 R for burns, 69
Protein C, 22 Radial nerve, 355, 355f endpoints in, 35
Protein synthesis, 120 Radiation enteritis, of small intestine, fluids for. See Fluid resuscitation
Prothrombin, 21 238–239 for shock, 35
Prothrombin complex concentrates, 24 Radiation therapy Resuscitative thoracotomy, 49
Proton pump inhibitors, 216, 226 for breast cancer, 140 RET tyrosine kinase receptor gene,
Proto-oncogenes, 154 description of, 138 324–325

Brunicardi_Index_p391-420.indd 408 30/06/22 11:32 AM


409
Retained surgical items, 99, 99t, 103 Saddle anesthesia, 350 SGLT1. See Sodium glucose
Retrograde embolization, 108 Saline co-transporter 1
Retrograde pyelogram, 338 hypertonic, 14 Shock

INDEX
Retroperitoneal fibrosis, 309 normal, 14, 18 blood loss percentages, 32–33
Retroperitoneal infections, 309 Salivary gland-type tumors, 147 cardiogenic, 29, 34, 55–56, 56f
Retroperitoneum, 309 Sarcoma categories of, 29
Retropubic urethropexy, 343 breast, 312 compensated, 35
Reverse-transcriptase PCR, 124 Ewing, 154–155 cytokine elevations after, 32
Reynolds pentad, 284 soft tissue, 311–312 hemorrhagic, 19, 30
Rhabdomyomas, 180 Scar hypotension in, 35
Rheumatic disease, 175–176 hypertrophic, 80–81 hypovolemic, 29, 31–32
Rheumatic fever, 174–175 remodeling of, 75 neurogenic, 29–31, 35
RI. See Rothman Index SCC. See Squamous cell carcinoma obstructive, 29, 35
Rib(s) Schatzki ring, 219 questions regarding, 29–36
fractures of, 63 Schwann cell basal lamina, 77 resuscitation for, 35, 62
“onion-peel” appearance of, 154 Scimitar sign, 196 septic, 32–33
Right colectomy, 243 SCIP. See Surgical Care Improvement signs and symptoms of, 32–33
Right hemicolectomy, 260 Project tension pneumothorax as
Right hepatic artery, 282, 289 Sclerosing adenosis, 137 cause of, 29
Right ventricular outflow tract Sclerosing cholangitis, 246, 276, 280 trauma-induced coagulopathy
obstruction, 161, 170 Sclerosing mesenteritis, 309 caused by, 23
Rituximab, 302 Scrotum, 316–317 traumatic, 29
Rivaroxaban, 24 Scurvy, 79 treatment of, 33–34
RNA, 120 Seborrheic keratoses, 136 vasodilatory, 33
RNA gel blot analysis, 124 Second-degree burns, 67 vasogenic, 29
RNA polymerases, 119 Second-degree hemorrhoids, 252 Short bowel syndrome, 239
Robotic prostatectomy, 117 Second-impact syndrome, 347 Shoulder dislocation, 352–353
Robotic surgery Secretin stimulation test, 225–226 Shouldice repair, 321
description of, 243 “Seed and soil” theory, SICU. See Surgical intensive care unit
inguinal hernia repair using, 319 of metastasis, 84 Side-to-side anastomoses, 244, 245f
laparoscopic surgery versus, SEER registry, 248 Sigmoid colectomy, 244, 250
116–117, 117f, 319 Seldinger technique, 109 Sigmoidectomy, 256
transabdominal preperitoneal, 319 Self-expanding stents, 189, 282 Silver sulfadiazine
Rothman Index, 108 Sengstaken-Blakemore tube, 211 for burn wounds, 68–69
Roux-en-Y gastric bypass Sentinel lymph node biopsy neutropenia caused by, 68–69
complications of, 233–234 for breast cancer, 139 Simple pneumothorax, 56, 58
gastric stasis after, 224 for melanoma, 132 Simulation, 387–388
illustration of, 228f Sepsis Simulation training, 388
nutritional deficiencies after, 234 incidence of, 43 Sinus bradycardia, 115
small bowel obstruction after, 234 postsplenectomy, 51, 299 Sirolimus, 89t, 90
Roux-en-Y hepaticojejunostomy, survival rates for, 43 SIRS. See Systemic inflammatory
285, 287 Sepsis syndrome, 46 response syndrome
RT. See Resuscitative thoracotomy Septic shock SJS. See Stevens-Johnson syndrome
Rubber band ligation, for characteristics of, 32 Skin
hemorrhoids, 252 hyperglycemia in, 33 epidermis, 133
“Rule of nines,” 65, 66f insulin resistance in, 33 Langerhans cells of, 133
R-value, 28 treatment of, 33 lesions of, 135–136
RVOT. See Right ventricular outflow Sequential Organ Failure Assessment partial thickness wounds of, 132
tract obstruction score, 38 stratum basale of, 132
RYGB. See Roux-en-Y gastric bypass Serine/threonine kinase receptors, 122 stratum spinosum of, 133
Serotonin, 21 wound healing of, 76t
S Serum calcium, 15 Skin cancer
S phase, of cell cycle, 83, 121 Serum osmolality, 15 basal cell carcinoma, 129, 135
SAA. See Splenic artery aneurysm Serum sodium, 18 melanoma. See Melanoma
Saccular aneurysms, 181 Sestamibi scans, 326 squamous cell carcinoma, 130, 136
Sacral fractures, 352 Sex hormones, 133 Skin-sparing mastectomy, 141
Sacroiliitis, 246 Sexually transmitted infections, 135 Skull base fractures, 346–347

Brunicardi_Index_p391-420.indd 409 30/06/22 11:32 AM


410
SLNB. See Sentinel lymph Sphincter of Oddi, 279, 290 Staphylococcus epidermidis, 46
node biopsy Spinal cord ischemia, 183 Staphylococcus sp.
SMA. See Superior mesenteric artery Spleen description of, 134
INDEX

SMAD4 protein, 123 accessory, 300 S. aureus, 140, 359


Small bowel obstruction aneurysms of, 304 Steatorrhea, in Crohn’s disease, 268
closed loop, 236 cysts of, 303–304 Stellate cells, 236
laparoscopic surgery for, 237 functions of, 301 Stents
after Roux-en-Y gastric bypass, 234 hereditary spherocytosis of, 297, 301 balloon-expandable, 189
treatment of, 237 ligaments of, 300, 301f pancreatic duct, 295
water-soluble contrast agent metastases to, 298 self-expanding, 189, 282
imaging of, 236 platelet sequestration in, 22 ureteral, 338
Small intestine questions regarding, 297–306 vascular, 189–190
Crohn disease of, 237 traumatic injury to, 51 Steroids. See Corticosteroids
enterocutaneous fistula of, 237 Splenectomy Stevens-Johnson syndrome, 133
fluid in, 235 cancer risks after, 299 Stewart-Hamilton equation, 108
ileum, 235 for chronic hemolytic anemia, 300 Stomach
ischemia of, 251–252 in chronic myelogenous carcinoid tumors of, 230
jejunum, 235 leukemia, 303 duodenal ulcers, 226–227
length of, 235 complications of, 305–306 gastric antral vascular ectasia of,
malignancies of, 237 description of, 54 230–231, 231f
mesentery of, 308 indications for, 297–299 gastrin secretion, 223
obstruction of. See Small bowel infections after, 302, 305–306 gastrointestinal stromal
obstruction laparoscopic, 299, 305 tumors of, 230
questions regarding, 235–239 for neutropenia in Felty syndrome, Ménétrier disease of, 231, 231f
radiation enteritis of, 238–239 304–305 parietal cells of, 224, 231
short bowel syndrome, 239 open, 299, 305–306 peptic ulcer disease of, 223,
transplantation of, 239 overwhelming postsplenectomy 226–228, 227f–228f
Small saphenous vein, 197 infection, 305–306 surface epithelial cells of, 231
Smoke inhalation injury, 66 in polycythemia vera, 303 Zollinger-Ellison syndrome of,
Smoking, 144 sepsis after, 51, 299 225, 231
“Sniff Test,” 149 Splenic artery, 299 Stratum basale, 132
SOAPs. See Surgical, anesthetic, and Splenic artery aneurysm, 304 Stratum spinosum, 133
obstetrical providers Splenic vein thrombosis, 304 Streptococcus sp., 134
Social media, 389–390 Splenocolic ligament, 300, 301f Stress, caloric adjustments after, 11
Sodium Splenomegaly, 299 Stress incontinence, 342–343
aldosterone-mediated Splenorenal ligament, 300 Stroke, 190, 346f
reabsorption of, 17 Splenorrhaphy, 51 Structural measures, 98
serum concentration of, 18 Split-thickness skin grafts, 361 Stryker device, 51
Sodium chloride, 14, 18 Spontaneous abortion, 22 Sturge-Weber syndrome, 362
Sodium glucose co-transporter 1, 236 Squamocolumnar junction, 209 Subclavian artery
Sodium thiosulfate, for cyanide Squamous cell carcinoma malperfusion of, 187
poisoning, 66 of esophagus, 209 proximal left, 60–61
SOFA score. See Sequential Organ of hand, 360 Subfalcine herniation, 345
Failure Assessment score of lung, 149 Sublay technique, for mesh repair
Soft tissue sarcomas, 311–312 precursor lesion to, 136 of abdominal incisional
Soft-tissue infections, of skin, 130 hernia, 308
necrotizing, 42, 134 “Square-root sign”, 178 Subphrenic abscess, 51
Solid organ injuries, 61 SSIs. See Surgical site infections Subtotal colectomy, 244
Somatostatin, 293 SSTRs. See Somatostatin receptors Subtotal parathyroidectomy, 326
Somatostatin receptor SSV. See Small saphenous vein Subvalvular aortic stenosis, 163
scintigraphy, 226 Stab wounds, 59, 60t Superficial burns, 66
Somatostatin receptors, 292 Staging Superficial spreading basal cell
Somatostatinoma, 293 of breast cancer, 137 carcinoma, 135
Sorafenib, for hepatocellular of colorectal cancer, 248 Superficial spreading melanoma,
carcinoma, 277 of lung cancer, 152 130, 130f
Source control, 38 of sarcoma, 312 Superior laryngeal nerve injury,
Southern blotting, 124 of Wilms tumor, 332t 100, 323

Brunicardi_Index_p391-420.indd 410 30/06/22 11:32 AM


411
Superior mesenteric artery source control, 38 TEN. See Toxic epidermal necrolysis
anatomy of, 265f treatment of, 46 Tension pneumothorax
injuries to, 61 Surgical innovation, 371 cardiac failure caused by, 55

INDEX
in mesenteric ischemia, 192 Surgical intensive care unit, 50, 61 diagnosis of, 35
stenosis of, 192 Surgical palliative care, 367, 370 diagnostic criteria for, 58
Superior thyroid artery, 323 Surgical research, 371 needle thoracostomy
Supernumerary parathyroid Surgical site infections, 39t, decompression of, 56, 58
glands, 325 39–41, 47, 246 obstructive shock versus, 35
Suppurative thrombophlebitis, 203 Surgical wounds, 39–40, 40t, 47 shock caused by, 29
Supracolic injuries, 61 Sustentaculum lienis, 301f signs of, 56, 58
Surface epithelial cells, gastric, 231 SvO2. See Fractional saturation of simple pneumothorax versus, 56, 58
Surgery hemoglobin tube thoracostomy for, 56, 57f
ambulatory, 385–386 SVT. See Suppurative TEP. See Totally preperitoneal repair
anorectal melanoma treated thrombophlebitis Testes, undescended, 336
with, 250 Sweat glands, 133 Testicular cancer, 337, 339
after bare coronary stent Syphilis Tetralogy of Fallot, 161, 170
placement, 364 ascending aortic aneurysms TF. See Tissue factor
bowel preparation for, 246 caused by, 183 TFPI. See Tissue factor pathway
cost-effectiveness of, 374f blood transfusion inhibitor
diversity in, 5 transmission of, 27 TGF-ß. See Transforming growth
in elderly, 365–368 Syphilitic aortitis, 183 factor-ß
genomic, 119–125 Systemic inflammatory response Th1 cells, 10
global, 373–376 syndrome, 37 Th1 cytokines, 9
hand, 355–360 Th2 cells, 10
major adverse cardiac events T Th17 cells, 10
screening, 172 T cells Thalassemia, 302
minimally invasive, 115–117 CD4+, 9 The Joint Commission, 99
molecular, 119–125 CD8+, 9 Thermal wounds, 129
noncardiac, 172 helper, 10 Thermodilution method, for cardiac
obesity treated with, 233–234 T regulatory cells, 9 output, 108
orthopedic, 351–353 TACO. See Transfusion-related Third-degree burns, 67
pediatric, 329–336 circulatory overload Third-degree hemorrhoids, 252
physiologic monitoring during, Tacrolimus, 88–89, 89t Thoracic aorta disease, 184
105–113 Tamoxifen, 138 Thoracic aortic disease, 183
plastic, 361–362 TAPP. See Transabdominal Thoracic compartment syndrome, 68
reconstructive, 361–362 preperitoneal repair Thoracic escharotomies, 68
retained surgical items, 99, 99t, 103 TAPVC. See Total anomalous Thoracoabdominal region,
robotic, 116–117, 117f pulmonary venous penetrating trauma to, 59
women in, 5 connection Thoracotomy
wrist, 355–360 Task sharing, 373 emergency department, 49
wrong-site, 99 TAVI/TAVR. See Transcatheter for empyema, 102
Surgical, anesthetic, and obstetrical aortic valve implantation/ indications for, 53
providers, 375 replacement trap door, 60
Surgical care, globalization of, 376 TBI. See Traumatic brain injury Thrombin, 21–22, 205
Surgical Care Improvement Project, TBSA. See Total body surface area Thrombocytopenia
98, 98t TBW. See Total body water; Total causes of, 22
Surgical excision body weight heparin-induced, 23–24, 173, 205
of basal cell carcinoma, 129 TCD. See Transcranial Doppler; primary immune, 23
of burn wounds, 70 Transcranial power Doppler Thromboelastography, 27–28
of melanoma, 132 T-cell–mediated rejection, 95 Thromboembolism, venous, 200,
Surgical infection TCL. See Transverse carpal ligament 202, 204, 204f, 366
definition of, 37 TEE. See Transesophageal Thrombolytic therapy
intravascular catheters as echocardiography catheter-directed, 201
cause of, 46–47 TEF. See Tracheoesophageal fistula contraindications for, 201
prophylactic antibiotics for, 38, 45 TEG. See Thromboelastography Thrombomodulin, 22
questions regarding, 37–47 Telemanipulation, 243 Thrombopoietin receptor
of retroperitoneum, 309 Temporal bone fracture, 346 antagonists, 302

Brunicardi_Index_p391-420.indd 411 30/06/22 11:32 AM


412
Thrombosis Toxic shock syndrome, 134 graft rejection in, 90
catheters and, 107 tPA. See Tissue plasminogen receptor graft thrombosis after, 93
deep vein, 200–202, 202t, 366 Trachea heart, 95
INDEX

graft, 93 anatomy of, 146, 146f hyperacute rejection in, 88


mesenteric vein, 206 blood supply to, 145, 145f immunosuppression in, 87, 89t, 95
microvascular, 24–25 cricoid cartilage of, 146, 146f infections in recipients, 90
splenic vein, 304 injuries to, 51 islet cell, 94–95
Thrombotic thrombocytopenic stenosis of, 146 kidney. See Kidney transplant
purpura, 23–25 upper, 146, 146f liver, 95, 276–277, 280, 287
Thymomas, 155 Tracheobronchial injury, 59 Mayo protocol for, 276
Thyroglobulin, 324 Tracheobronchial tree, 147, 147f mycophenolate mofetil in, 88, 89t
Thyroglossal duct cyst, 323, 330 Tracheoesophageal fistula, 330, 331f orthotopic liver, 277
Thyroid cancer Tracheostomy pancreas, 93–95, 94f
papillary, 281, 324 description of, 51 questions regarding, 87–95
treatment of, 325 emergent, 56 rejection in, 88, 90, 95
Thyroid gland percutaneous, 101 screening for, 92
arterial supply of, 323 questions regarding, 144 small intestine, 239
congenital anomaly of, 323 tracheal stenosis caused by, 146 tacrolimus in, 88–89, 89t
Thyroid hormones, 324 Tracheostomy tube, 144 whole-pancreas, 95
Thyroid nodules, 324 Traditional Chinese medicine, 379 Transposition of the great vessels, 161
Thyroid surgery, 325 TRALI. See Transfusion-related acute Transthoracic echocardiography,
Thyroidectomy, 323–324 lung injury 172–173
Thyroxine, 324 Tranexamic acid, 27 Transurethral resection of the
TIC. See Trauma-induced Transabdominal preperitoneal prostate, 339
coagulopathy repair, 321 Transverse aortic arch aneurysms, 185
Time management, 3 Transcatheter aortic valve Transverse carpal ligament, 357
TIPS. See Transjugular intrahepatic implantation/replacement, 368 Transverse colectomy, 244
portosystemic shunt Transcranial Doppler, 112 Trap door thoracotomy, 60
Tissue factor, 21 Transcranial power Doppler, 191 Trastuzumab, 125
Tissue factor pathway inhibitor, 22 Transcription Trauma. See also Blunt trauma;
Tissue injury, 23 in eukaryotes, 119 Penetrating trauma
Tissue plasminogen receptor, 22 in prokaryotes, 119 airway interventions in, 57–58
TKIs. See Tyrosine kinase inhibitors Transesophageal echocardiography, cardiogenic shock in, 55–56, 56f
TLRs. See Toll-like receptors 172–173 gastric injuries from, 62
TM. See Thrombomodulin Transforming growth factor-ß, 123 in geriatric patients, 63
TME. See Total mesorectal excision Transfusion head, 53
TNM staging complications of, 26 inflammatory response after, 7
of colorectal cancer, 248 description of, 25 liver, 50
of lung cancer, 152 diseases not transmitted by, 27 motor vehicle collisions, 59
To Err Is Human, 4 indications for, 25–26 neck, 52
TOF. See Tetralogy of Fallot Transfusion-related acute lung in pregnancy, 62
Toll-like receptors, 7, 9–10, 37 injury, 25–27 primary survey of, 49, 59
Tonsillar herniation, 345, 345f Transfusion-related circulatory questions regarding, 49–63
Total anomalous pulmonary venous overload, 27 solid organ injuries caused by, 61
connection, 159, 159f, Transhiatal esophagectomy, 209 spleen, 51
165–166, 166f “Transition wound,” 77 tracheal, 51
Total body surface area, 65–66, 66f Transjugular intrahepatic transcriptional events after, 10
Total body water, 14–15 portosystemic shunt, 271, 273 Trauma-induced coagulopathy, 23
Total body weight, 14–15 Translation, 120 Traumatic brain injury
Total colectomy, 244 Transmural infarction, 176 intracranial pressure
Total mesorectal excision, for rectal Transplantation monitoring in, 112
cancer, 249 azathioprine in, 88, 89t medical management of, 348–349
Total parathyroidectomy, 326 bleeding after, 92 Traumatic shock, 29
Total thyroidectomy, 324 cyclosporin in, 88, 89t Treponema pallidum, 183
Totally preperitoneal repair, 321 definition of, 87 Tricuspid atresia, 168
Tourniquet, 356 donation after cardiac death, 91 Tricuspid stenosis, 176
Toxic epidermal necrolysis, 133 graft loss, 93 tRNA, 119

Brunicardi_Index_p391-420.indd 412 30/06/22 11:32 AM


413
Trousseau sign, 16 Ultraviolet radiation exposure, 130 Ventricular septal defect
True aneurysms, 181 Umbilical stump bleeding, 22 aortic insufficiency
Truncus arteriosus, 165 Uncal herniation, 345, 345f secondary to, 175

INDEX
TTE. See Transthoracic Uncomplicated diverticulitis, 247 description of, 161–162
echocardiography Undescended testes, 336 in double-outlet right ventricle, 170
TTP. See Thrombotic Uremia, 23 holosystolic murmur associated
thrombocytopenic purpura Ureteral injuries, 338 with, 175
T-tube, 281 Ureteral stents, 338 tricuspid atresia without, 168
Tube thoracostomy, for tension Urinary incontinence, 343 Vibrio vulnificus, 134
pneumothorax, 56, 57f Urinary tract infection, 42–43, 338 VILI. See Ventilator-induced lung
Tuberculosis, 153 Urolithiasis, 338 injury
Tumor(s) Urology, 337–339 VIPomas, 294
appendiceal, 260 Uterine leiomyoma, 341 Virtual reality simulation, 387
carcinoid, 260 UTI. See Urinary tract infection Vision, 3, 3t
carotid body, 191 UV. See Ultraviolet radiation exposure Visual abstracts, 389
chest wall, 154 Vitamin(s), in wound healing, 79–80
conservative resections, 85 V Vitamin A, 78–80
desmoid, 154 Vaccinations, 135, 298, 342 Vitamin B12 deficiency, 224
esophageal, 209, 213 VADs. See Ventricular assist devices Vitamin C, 79
gastrointestinal stromal, 230, 238, 249 Vaginal cancer, 343 Vitamin K
nonseminatous germ-cell, 155 Vagus nerve, 208 deficiency of, 268
omentum, 308 Validity, external, 383 warfarin reversal using, 24
pancreatic exocrine, 292, 293f Valsalva maneuver, 241 VO2. See Oxygen utilization
pituitary fossa, 328 Valve replacement, 174–175, 268, 367 Vocal cords
salivary gland-type, 147 Valvulae conniventes, 235 anatomy of, 146, 146f
thymic, 155 VAP. See Ventilator-associated leukoplakia of, 143
Tumor markers, 85 pneumonia Volume receptors, 30
Tumor necrosis factor-alpha, 37 Variceal hemorrhage, 271 Volvulus
Tumor suppressor genes, 124 Vascular endothelial growth factor, 84 cecal, 250–251
Tumorigenic transformation of cells, 83 Vascular injuries, of extremities, endoscopy for, 250–251
TURP. See Transurethral resection of 54–55, 55t Vomiting
the prostate Vascular rings, 166, 167f metabolic derangements
TXA. See Tranexamic acid Vascular stents, 189–190 caused by, 17
Type A blood, 25 Vascular-type Ehlers-Danlos postoperative, 378
Type AB blood, 25 syndrome, 182 von Willebrand factor, 21, 24
Type II error, 382 Vasoactive medications, 271 VSD. See Ventricular septal defect
Type O-negative blood, 25 Vasoactive proteins, 31 VTE. See Venous thromboembolism
Typhlitis, 255 Vasodilatory shock, 33 Vulvar carcinoma, 342
Tyrosine kinase inhibitors, 249 Vasogenic shock, 29 vWF. See von Willebrand factor
Tyrosine kinase receptors, 122 Vasopressin, 31, 271
VEGF. See Vascular endothelial W
U growth factor Wallerian degeneration, 77
Ulcer(s) Veins Warfarin, 24, 205, 268
Cushing, 349 anatomy of, 197–198 Warm-antibody autoimmune
duodenal, 226–227 hepatic, 265, 266f hemolytic anemia, 299
malignant transformation of, 80 lower extremity, 197 Wartenberg sign, 358
pressure, 134 Vena cava filters, 202, 203f Water
Ulcerative colitis Venae comitantes, 197 free water deficit, 15
diagnosis of, 257 Venous anastomosis, 92f in total body weight, 14–15
primary sclerosing cholangitis Venous disease, 197–206 Water deficit, 15
and, 246 Venous thromboembolism, 200, 202, Watermelon stomach, 230, 230f
Ulnar nerve, 355, 355f, 358 204, 204f, 366 Water-soluble contrast agents, 236
Ultrasound Ventilator weaning, 101 Weaning, ventilator, 101
cardiac tamponade evaluations, Ventilator-associated pneumonia, 100 Web-based education, 389–390
55, 56f Ventilator-induced lung injury, 112 West Nile virus, 27
endoscopic. See Endoscopic Ventricular assist devices, 179 Whipple procedure, 286
ultrasound Ventricular compliance, 106–107 Whole-pancreas transplantation, 95

Brunicardi_Index_p391-420.indd 413 30/06/22 11:32 AM


414
Willingness, 3, 3t strength of, 74 polymorphonuclear
Willingness to lead, 3t surgical, 39–40, 40t, 47 neutrophils in, 74
Willingness to learn, 3t “transition wound,” 77 proliferative phase of, 73f, 74
INDEX

Wilms tumor, 331, 332t Wound healing scar remodeling, 75


Wilson disease, 269–270 amino acids in, 79 skin, 76t
Wolff-Parkinson-White syndrome, anastomoses, 75 steroid effects on, 78
169f, 169–170 anemia and, 78 vitamins in, 79–80
Women arginine effects on, 79 WPW syndrome. See Wolff-
fluid maintenance callus formation in, 77 Parkinson-White syndrome
calculations for, 16 collagen in, 74, 76t Wrist
in surgery, 5 diabetes mellitus effects on, 78 carpal tunnel syndrome of, 357
total body weight in, 15 fetal, 77 ganglion cyst of, 360
Wound(s) fibroblastic phase of, 73f, 74 surgery of, 355–360
burn. See Burn wounds full-thickness wounds, 75–76 Wrong-site surgeries, 99
classification of, 39–40, 40t gastrointestinal tract,
closure of, in Ehler-Danlos 75–76, 76t X
syndrome patients, 75 hypertrophic scars, 80–81 Xenotransplant, 87
collagen in, 74 impairment of, 78
in Ehler-Danlos syndrome inflammation phase of, 73f, Z
patients, 75 78–79 Zenker’s diverticulum, 220
gastric, 62 keloid formation, 80–81, 81f ZES. See Zollinger-Ellison syndrome
infection of, 102 low-oxygen tension effects on, 78 Zika virus, 27
malignant, 80 malnutrition effects on, 79 Zollinger-Ellison syndrome, 225, 231
matrix deposition at maturation phase of, 73f Zona fasciculata, 8
site of, 74 nutrition in, 79 Zone of coagulation, 67, 129
skin, 132 oxygen delivery during, 78 Zone of hyperemia, 67, 129
stab, 59, 60t phases of, 73f, 73–74, 361 Zygoma fractures, 144

Brunicardi_Index_p391-420.indd 414 30/06/22 11:32 AM

You might also like